Mathematical and Physical Journal for High Schools [2022, 2022 ed.]

At present the journal is published by the MATFUND Hungarian High School Mathematics and Physics Foundation, the János B

344 19 25MB

Hungarian Pages 286 Year 2022

Report DMCA / Copyright

DOWNLOAD FILE

Polecaj historie

Mathematical and Physical Journal for High Schools [2022, 2022 ed.]

Table of contents :
KöMaL 2022
Tartalomjegyzék-január
Freud Róbert: Számelmélet és valószínűségszámítás
Füredi Erik: Kedvcsináló a ,,Geometriafeladatok megoldása szinusztétellel'' című íráshoz
60. Rátz László Vándorgyűlés
Koncz Levente: Gyakorló feladatsor emelt szintű matematika érettségire
Deák Anna: Megoldásvázlatok a 2021/9. szám emelt szintű matematika gyakorló feladatsorához
Matematika feladat megoldása (5194.)
A K pontversenyben kitűzött gyakorlatok (714-718.)
A C pontversenyben kitűzött gyakorlatok (717-718., 1699-1703.)
A B pontversenyben kitűzött feladatok (5214-5221.)
Az A pontversenyben kitűzött nehezebb feladatok (815–817.)
Informatikából kitűzött feladatok (553-555., 59., 158.)
Matematikai képzések az ELTE TTK-n
Matematikatanár-képzés az ELTE TTK-n
Fizika alapszak és fizikatanár-képzés az ELTE TTK Fizikai Intézetében
Baranyai Klára: A kúpingáról
Mérési feladatok megoldása (403., 405.)
Fizika feladatok megoldása (5337., 5339., 5344., 5345.)
Fizikából kitűzött feladatok (410., 765–768., 5373–5381.)
Problems in Mathematics
Problems in Physics
Tartalomjegyzék-február
Jelentés a 2021. évi Kürschák József Matematikai Tanulóversenyről
Pach Péter Pál: A 2021. évi Kürschák József Matematikai Tanulóverseny feladatainak megoldása
Marczis György, Molnár István, Molnár Judit, Rókáné Rózsa Anikó: Gyakorló feladatsor emelt szintű matematika érettségire
Koncz Levente: Megoldásvázlatok a 2022/1. szám emelt szintű matematika gyakorló feladatsorához
Matematika feladatok megoldása (5150., 5201.)
Ifjú olvasóinkhoz régen és most
A K pontversenyben kitűzött gyakorlatok
A C pontversenyben kitűzött gyakorlatok
A B pontversenyben kitűzött feladatok
Az A pontversenyben kitűzött nehezebb feladatok
Tóth Tamás: A matematikai logika logikusabb, mint gondolnánk I
Informatikából kitűzött feladatok
Rátz Tanár Úr Életműdíjak átadása 2021 decemberében
Gnädig Péter, Széchenyi Gábor, Vankó Péter, Vigh Máté: Beszámoló a 2021. évi Eötvös-versenyről
Fizika gyakorlat megoldása
Fizika feladatok megoldása
Felhívás a Kunfalvi Rezső Olimpiai Válogatóversenyre
Fizikából kitűzött feladatok
Problems in Mathematics
Problems in Physics
Problems of the 2021 Kürschák competition
Tartalomjegyzék-március
Németh László: Gyakorló feladatsor emelt szintű matematika érettségire
Marczis György, Molnár István, Molnár Judit, Rókáné Rózsa Anikó: Megoldásvázlatok a 2022/2. szám emelt szintű matematika gyakorló feladatsorához
Matematika C gyakorlat megoldása
Matematika feladatok megoldása
A K pontversenyben kitűzött gyakorlatok
A C pontversenyben kitűzött gyakorlatok
A B pontversenyben kitűzött feladatok
Az A pontversenyben kitűzött nehezebb feladatok
Tóth Tamás: A matematikai logika logikusabb, mint gondolnánk II
Informatikából kitűzött feladatok
Fizika gyakorlat megoldása
Fizika feladatok megoldása
Fizikából kitűzött feladatok
Problems in Mathematics
Problems in Physics
Tartalomjegyzék-április
Bessenyei Mihály, Maksa Gyula: Mennyi a téglalap területe?
Jócsik Csilla: Gyakorló feladatsor emelt szintű matematika érettségire
Németh László: Megoldásvázlatok a 2022/3. szám emelt szintű matematika gyakorló feladatsorához
Matematika C gyakorlat megoldása
Matematika feladatok megoldása
Nehezebb feladat megoldása
A C pontversenyben kitűzött gyakorlatok
A B pontversenyben kitűzött feladatok
Az A pontversenyben kitűzött nehezebb feladatok
Informatikából kitűzött feladatok
Mérési feladat megoldása
Fizika gyakorlatok megoldása
Fizika feladat megoldása
Nyári fizikatábor
Fizikából kitűzött feladatok
Problems in Mathematics
Problems in Physics
Tartalomjegyzék-május
Kedves Olvasóink!
Ericsson-díj – tájékoztató
Naszódi Márton: Konvex testbe írható tetraéder d-dimenzióban
Kiss Melinda Flóra, Baran Zsuzsa, Janzer Lili: EGMO 2022
Az EGMO 2022 feladatai
EGMO beszámoló
Jócsik Csilla: Megoldásvázlatok a 2022/4. szám emelt szintű matematika gyakorló feladatsorához
Matematika feladatok megoldása
A C pontversenyben kitűzött gyakorlatok
A B pontversenyben kitűzött feladatok
Az A pontversenyben kitűzött nehezebb feladatok
Informatikából kitűzött feladatok
Könyvismertetés
Fizika gyakorlatok megoldása
Fizika feladatok megoldása
Fizikából kitűzött feladatok
Problems in Mathematics
Problems in Physics
Tartalomjegyzék-szeptember
Frenkel Péter: Beszámoló a 63. Nemzetközi Matematikai Diákolimpiáról
A 63. Nemzetközi Matematikai Diákolimpia feladatai
Olimpiai előkészítő szakkörök
Kiss Melinda Flóra, Baran Zsuzsa: EGMO 2022/2023 felhívás
Tóthmérész Lilla: Négyszín-sejtés I: A sejtés születése és egy bizonyítási kísérlet
Erdős Gábor: Gyakorló feladatsor emelt szintű matematika érettségire
Tájékoztató a folyóirat előfizetéséről
Versenykiírás a KöMaL pontversenyeire
Matematika feladatok megoldása
A K pontversenyben kitűzött gyakorlatok
A C pontversenyben kitűzött gyakorlatok
A B pontversenyben kitűzött feladatok
Kürschák-verseny
Az A pontversenyben kitűzött nehezebb feladatok
Informatikából kitűzött feladatok
Sarkadi Tamás, Tasnádi Tamás: Szép szereplés az 52. Nemzetközi Fizikai Diákolimpián
A Nemzetközi Csillagászati és Asztrofizikai Diákolimpiáról és válogatóversenyéről, az Athletica Galacticáról
Kunfalvi Rezső Olimpiai Válogatóverseny 1. elméleti forduló
Fizika gyakorlat megoldása
Fizika feladatok megoldása
Fizikából kitűzött feladatok
Eötvös-verseny
Problems in Mathematics
Problems in Physics
Tartalomjegyzék-október
A 63. Nemzetközi Matematikai Diákolimpia feladatainak megoldása I
Tóthmérész Lilla: Négyszín-sejtés II: Hol a hiba Kempe bizonyításában?
Miklós Ildikó: 61. Rátz László Vándorgyűlés
A középiskolai tanárok versenyének feladatai
A 2022. évi Beke Manó Emlékdíjasok
A 2022. évi Reményi-díjasok
Németh László: Gyakorló feladatsor emelt szintű matematika érettségire
Erdős Gábor: Megoldásvázlatok a 2022/6. szám emelt szintű matematika gyakorló feladatsorához
Matematika C gyakorlatok megoldása
Matematika feladatok megoldása
A K pontversenyben kitűzött gyakorlatok
A C pontversenyben kitűzött gyakorlatok
A B pontversenyben kitűzött feladatok
Az A pontversenyben kitűzött nehezebb feladatok
Informatikából kitűzött feladatok
Sztranyák Gabriella: Nyári matematika- és fizikatábor 2022
Szász Krisztián, Vankó Péter: Beszámoló a 6. Európai Fizikai Diákolimpiáról
Ifjú Fizikusok Nemzetközi Versenye
Fizika gyakorlatok megoldása
Fizika feladatok megoldása
Fizikából kitűzött feladatok
Problems in Mathematics
Problems in Physics
Tartalomjegyzék-november
A 63. Nemzetközi Matematikai Diákolimpia feladatainak megoldása II
Tóthmérész Lilla: Négyszín-sejtés III: A színezési polinom, avagy miért olyan nehéz a négyszín-tétel
Helyesbítés és közlemény
Az általános iskolai tanárok versenyének feladatai
Jócsik Csilla: Gyakorló feladatsor emelt szintű matematika érettségire
Németh László: Megoldásvázlatok a 2022/7. szám emelt szintű matematika gyakorló feladatsorához
Matematika feladatok megoldása
A K pontversenyben kitűzött gyakorlatok
A C pontversenyben kitűzött gyakorlatok
A B pontversenyben kitűzött feladatok
Az A pontversenyben kitűzött nehezebb feladatok
Informatikából kitűzött feladatok
Woynarovich Ferenc: A röntgenszórás, más néven Bragg-reflexió
Fizika gyakorlatok megoldása
Fizika feladatok megoldása
Fizikából kitűzött feladatok
Problems in Mathematics
Problems in Physics
Tartalomjegyzék-december
Freud Róbert, Horváth Eszter: Nyakláncok, lyukas négyzetek és oszthatóság
Kozma Katalin Abigél, Számadó László: Gyakorló feladatsor emelt szintű matematika érettségire
Jócsik Csilla: Megoldásvázlatok a 2022/8. szám emelt szintű matematika gyakorló feladatsorához
Matematika C gyakorlatok megoldása
Matematika feladat megoldása
Matematikai képzések az ELTE TTK-n
Matematikatanár-képzés az ELTE TTK-n
A K pontversenyben kitűzött gyakorlatok
A C pontversenyben kitűzött gyakorlatok
A B pontversenyben kitűzött feladatok
Az A pontversenyben kitűzött nehezebb feladatok
Informatikából kitűzött feladatok
Fizika alapszak és fizikatanár-képzés az ELTE TTK Fizikai Intézetében
Gnädig Péter: Térből síkba visszalépés fizikai problémák megoldásánál I)
Fizika gyakorlat megoldása
Fizika feladatok megoldása
Fizikából kitűzött feladatok
Problems in Mathematics
Problems in Physics

Citation preview

´ ´ FIZIKAI LAPOK ¨ EPISKOLAI KOZ MATEMATIKAI ES ˝ ´ITVE INFORMATIKA ROVATTAL BOV ´ ALAP´ITOTTA: ARANY DANIEL 1894-ben 72. ´evfolyam 1. sz´am

Sz´amelm´elet ´es val´ osz´ın˝ us´egsz´am´ıt´as∗ Budapest, 2022. janu´ar

´ Megjelenik ´evente 9 sz´amban, janu´art´ol m´ajusig ´es szeptembert˝ol decemberig havonta 64 oldalon. ARA: 1050 Ft

´ TARTALOMJEGYZEK Freud R´obert: Sz´amelm´elet ´es val´osz´ın˝us´egsz´am´ıt´as . . . . . . . . . . . . . . . . . . . . . . . . . . . . . . . . . . . . . . . . . . .

2

F¨uredi Erik: Kedvcsin´al´o a Geometriafeladatok ” megold´asa szinuszt´etellel” c´ım˝u ´ır´ashoz . . . . . . . .

11

60. R´atz L´aszl´o V´andorgy˝ul´es . . . . . . . . . . . . . . . . . . . .

13

Koncz Levente: Gyakorl´o feladatsor emelt szint˝u matematika ´eretts´egire . . . . . . . . . . . . . . . . . . . . . . . .

18

De´ak Anna: Megold´asv´azlatok a 2021/9. sz´am emelt szint˝u matematika gyakorl´o feladatsor´ahoz . . . . . . . . . . . . . . . . . . . . . . . . . . . . . . . . . . . . . . .

20

Matematika feladat megold´asa (5194.) . . . . . . . . . . .

27

A K pontversenyben kit˝uz¨ott gyakorlatok (714– 718.) . . . . . . . . . . . . . . . . . . . . . . . . . . . . . . . . . . . . . . . . .

30

A C pontversenyben kit˝uz¨ott gyakorlatok (717– 718., 1699–1703.) . . . . . . . . . . . . . . . . . . . . . . . . . . . . .

31

A B pontversenyben kit˝uz¨ott feladatok (5214– 5221.) . . . . . . . . . . . . . . . . . . . . . . . . . . . . . . . . . . . . . . . .

32

Az A pontversenyben kit˝uz¨ott nehezebb feladatok (815–817.) . . . . . . . . . . . . . . . . . . . . . . . . . . . . . . . . . . . .

34

Informatik´ab´ol kit˝uz¨ott feladatok (553–555., 59., 158.) . . . . . . . . . . . . . . . . . . . . . . . . . . . . . . . . . . . . . . . . .

34

Matematikai k´epz´esek az ELTE TTK-n . . . . . . . . . .

39

Matematikatan´ar-k´epz´es az ELTE TTK-n . . . . . . . .

40

Fizika alapszak ´es fizikatan´ar-k´epz´es az ELTE TTK Fizikai Int´ezet´eben . . . . . . . . . . . . . . . . . . . . . .

41

Baranyai Kl´ara: A k´uping´ar´ol . . . . . . . . . . . . . . . . . . .

43

M´er´esi feladatok megold´asa (403., 405.) . . . . . . . . . .

47

Fizika feladatok megold´asa (5337., 5339., 5344., 5345.) . . . . . . . . . . . . . . . . . . . . . . . . . . . . . . . . . . . . . . . .

51

Fizik´ab´ol kit˝uz¨ott feladatok (410., 765–768., 5373–5381.) . . . . . . . . . . . . . . . . . . . . . . . . . . . . . . . . . .

58

Problems in Mathematics . . . . . . . . . . . . . . . . . . . . . . . .

61

Problems in Physics . . . . . . . . . . . . . . . . . . . . . . . . . . . . .

63

K¨ oz´ episkolai Matematikai ´ es Fizikai Lapok, 2022/1

A matematika e k´et a´g´ anak o as´ aban u ´tt¨ or˝ o szerepet j´atszott ¨sszekapcsol´ a 20. sz´azadi matematika k´et ´ ori´asa, Erd˝ os P´ al (1913–1996) ´es Tur´ an P´ al (1910– 1976), akik egym´ asnak is k¨ ozeli bar´ atai ´es munkat´ arsai voltak. El˝ osz¨ or Erd˝os egyik kedvenc, r´eszben ma is megoldatlan probl´em´ aj´aval foglalkozunk, majd Tur´ an egyszer˝ u bizony´ıt´as´ at mutatjuk be Geoffrey Hardy (1877–1947) ´es Srinivasa Ramanujan (1887–1920) h´ıres t´etel´ere az eg´eszek pr´ımoszt´ oinak tipikus sz´am´ar´ ol. Mindkett˝ o elmondhat´o val´ osz´ın˝ us´egsz´ am´ıt´ as n´elk¨ ul is, de ´eppen a val´osz´ın˝ us´egi szeml´elet mutatja meg a l´enyeget. Mindk´et esetben a Csebisev-egyenl˝ otlens´eget fogjuk alkalmazni.

´ EVA ´ F˝oszerkeszt˝o: RATKO ´ ¨ Fizikus szerkeszt˝o: GNADIG PETER ´ ILDIKO ´ M˝uszaki szerkeszt˝o: MIKLOS Bor´ıt´o: BURGHARDT ZSUZSA ´ Kiadja: MATFUND ALAP´ITVANY ´ RITA Alap´ıtv´anyi k´epvisel˝o: KOS Felel˝os kiad´o: KATONA GYULA Nyomda: OOK-PRESS Kft. ´ Felel˝os vezet˝o: SZATHMARY ATTILA INDEX: 25 450 ISSN 1215-9247 A matematika bizotts´ag vezet˝oje: ´ HERMANN PETER ´ BALINT, ´ ´ Tagjai: B´IRO GYENES ZOLTAN, ´ ´ KISS HUJTER BALINT, IMOLAY ANDRAS, ´ ´ GEZA, ´ ´ RITA, KOZMA GEZA, KOS KOS ´ MATOLCSI DAVID, ´ KATALIN ABIGEL, ´ ´ PACH PETER ´ ´ V´IGH ¨ ORDI ¨ OK PETERN E, PAL, VIKTOR A fizika bizotts´ag tagjai: ´ ´ ´ BARANYAI KLARA, HOLICS LASZL O, ´ ´ HONYEK GYULA, OLOSZ BALAZS, SZASZ ´ SZECHENYI ´ ´ KRISZTIAN, GABOR, VIGH ´ E, ´ VLADAR ´ KAROLY, ´ MAT WOYNAROVICH FERENC

1. A Csebisev-egyenl˝otlens´eg altoz´ o egy olyan f¨ uggv´eny, amely v´eges sok val´ os Egy v´eges val´ osz´ın˝ us´egi v´ sz´ am´ert´eket vesz fel, ´es mindegyik ´ert´ekn´el megmondjuk, hogy az milyen val´ osz´ın˝ us´eggel ad´ odik. P´eld´ aul a kockadob´ asn´ al a lehets´eges ´ert´ekek 1, 2, 3, 4, 5 ´es 6, ´es mindegyiknek a val´ osz´ın˝ us´ege 1/6. A tov´ abbiakban csak ilyen legegyszer˝ ubb t´ıpus´ u val´ osz´ın˝ us´egi v´ altoz´ okat haszn´alunk, amelyek ´ert´ekk´eszlete N k¨ ul¨ onb¨ oz˝ o val´ os osz´ın˝ us´eggel veszi fel a v´ altoz´ o. sz´ am, v1 , v2 , . . . , vN , ´es mindegyik vi ´ert´eket 1/N val´ A val´ osz´ın˝ us´egi v´ altoz´ okat a´ltal´ aban ξ-vel, n´eha η-val fogjuk jel¨ olni. Egy val´osz´ın˝ us´egi v´ altoz´ o v´ arhat´ o ´ert´eke a lehets´eges ´ert´ekeknek a val´ osz´ın˝ us´egekkel s´ ulyozott a´tlaga. Ezt E(ξ)-vel jel¨ olj¨ uk. A mi speci´ alis eset¨ unkben a v´ arhat´ o ´ert´ek a felvett ´ert´ekek sz´ amtani k¨ ozepe: v1 + v2 + . . . + vN . (1.1) E = E(ξ) = N Pl. a kockadob´ as v´ arhat´ o ´ert´eke (1 + 2 + 3 + 4 + 5 + 6)/6 = 3,5. A val´osz´ın˝ us´egi v´ altoz´ o m´ asik fontos jellemz˝ oje a sz´ or´ as, amit D(ξ)-vel jel¨ ol¨ unk. Ezt u ´gy kapjuk, hogy a felvett ´ert´ekeknek a v´ arhat´ o ´ert´ekt˝ ol val´o n´egyzetes elt´er´eseit a val´ osz´ın˝ us´egekkel s´ ulyozva a´tlagoljuk ´es az eredm´enyb˝ ol n´egyzetgy¨ ok¨ ot vonunk. ´Igy a mi speci´ alis eset¨ unkben a n´egyzetes elt´er´esek sz´ amtani k¨ ozep´eb˝ ol kell n´egyzetgy¨ok¨ot vonni:  2 2 (v1 − E) + . . . + (vN − E) . (1.2) D = D(ξ) = N

Az informatika bizotts´ag vezet˝oje: ´ ´ SCHMIEDER LASZL O ´ E, ´ FARKAS CSABA, FODOR Tagjai: BUSA MAT ´ ´ NIKOLETT, LOCZI ´ ZSOLT, LASZL O LAJOS, ´ ´ ´ SIEGLER GABOR, SZENTE PETER, TOTH ´ TAMAS ´ ANDREA, TASNADI ´ ANIKO ´ Ford´ıt´ok: GROF ´ ´ ¨ Szerkeszt˝os´egi titk´ar: TRASY GYORGYN E A szerkeszt˝os´eg c´ıme: 1117 Budapest, P´azm´any P´eter s´et´any 1/C III. emelet 3.405. Telefon: 372-2850 A lap megrendelhet˝o az Interneten: www.komal.hu/megrendelolap/reszletek.h.shtml. El˝ofizet´esi d´ıj egy ´evre: 8800 Ft K´eziratokat nem ˝orz¨unk meg ´es nem k¨uld¨unk vissza. Minden jog a K¨oMaL tulajdonosai´e. E-mail: [email protected] Internet: http://www.komal.hu This journal can be ordered from the Editorial office: P´azm´any P´eter s´et´any 1/C III. emelet 3.405. 1117–Budapest, Hungary telephone: +36 (1) 372-2850 or on the Postal address H–1518 Budapest 112, P.O.B. 32, Hungary, or on the Internet: www.komal.hu/megrendelolap/reszletek.e.shtml. A Lapban megjelen˝o hirdet´esek tartalm´a´ert felel˝oss´eget nem v´allalunk.

A sz´ or´ as teh´at azt m´eri, milyen er˝ osen ingadozik a v´ altoz´ o a v´ arhat´ o ´ert´ek k¨ or¨ ul. A kockadob´ as sz´or´ asa  2 2 2 2 2 2 (1 − 3,5) + (2 − 3,5) + (3 − 3,5) + (4 − 3,5) + (5 − 3,5) + (6 − 3,5) ≈ 1,71. 6 ∗

1

2

Ehhez a t´em´ ahoz kapcsol´ odik a B. 5220. feladat is ebben a sz´ amban.

K¨ oz´ episkolai Matematikai ´ es Fizikai Lapok, 2022/1

Bel´ athat´ o, hogy val´osz´ın˝ us´egi v´ altoz´ ok ¨ osszeg´enek a v´arhat´ o ´ert´eke a v´ arhat´ o ´ert´ekek o ¨sszege:

Ilyen eg´eszek p´eld´ aul a 2 hatv´ anyai a kettes sz´amrendszerbeli fel´ır´ as egy´ertelm˝ us´ege miatt. A 2 kitev˝oje 0-t´ ol log2 n eg´eszr´esz´eig terjedhet, teh´at

(1.3)

(2.1)

E(ξ + η) = E(ξ) + E(η).

Ha a v´ altoz´ ok f¨ uggetlenek, akkor ugyanez ´erv´enyes a sz´ or´ asok n´egyzet´ere is: 2

(1.4)

2

A max k fels˝ o becsl´es´ehez megn´ezz¨ uk, h´ any r´esz¨ osszeg van, ´es ezek milyen intervallumba eshetnek. Mivel minden r´esz¨ osszeg k¨ ul¨ onb¨ oz˝ o eg´esz sz´ am, ez´ert legfeljebb annyi r´esz¨osszeg lehet, ah´ any eg´esz sz´ am tal´ alhat´ o ebben az intervallumban.

2

D (ξ + η) = D (ξ) + D (η).

R´ at´er¨ unk a k´et sz´ amelm´eleti alkalmaz´ as val´osz´ın˝ us´egsz´ am´ıt´asi alapj´ at k´epez˝ o nevezetes egyenl˝ otlens´egre, amely Pafnutyij Lvovics Csebisev (1821–1894) orosz matematikust´ ol sz´armazik. Ez inform´ alisan azt fejezi ki, hogy egy val´osz´ın˝ us´egi v´ altoz´ o nagy val´ osz´ın˝ us´eggel a v´ arhat´ o ´ert´ek´et˝ ol nem t´ ul t´ avoli ´ert´eket vesz fel. Itt a t´ avols´ ag m´ert´ekegys´ege a sz´ or´ as. Pontos megfogalmaz´asban: Jel¨ olje a ξ val´ osz´ın˝ us´egi v´ altoz´ o v´arhat´ o ´ert´ek´et E ´es sz´or´ as´at D, tov´ abb´ a legyen c > 0 tetsz˝ oleges. Ekkor 1/c2 -n´el kisebb annak a val´ osz´ın˝ us´ege, hogy a v´ altoz´ o a v´ arhat´ o ´ert´ekt˝ ol cD-n´el t´ avolabbi ´ert´eket vesz fel:

Ennek bizony´ıt´ asa: N D2 =

i=1

n + (n − 1) + . . . + (n − k + 1)  nk − 1. amnak el kell f´ernie 1 ´es Minden r´esz¨osszeg k¨ ul¨ onb¨ oz˝ o, teh´at 2k − 1 darab eg´esz sz´ nk − 1 k¨oz¨ott. Ez´ert (2.2)

2k − 1  nk − 1,

2



|vi −E|>cD

2

2

(vi − E) > s(cD) ,

k  2 log2 n.

(2.4)

odik, amit V´eve (2.4)-ben mindk´et oldal logaritmus´ at log2 k  1 + log2 log2 n ad´ (2.3)-ba be´ırva a k  log2 n + log2 log2 n + 1

fels˝ o becsl´est nyerj¨ uk. (2.1) ´es (2.5) alapj´ an

´ıgy N > c2 s.

Az els˝o egyenl˝ os´eg az (1.2) defin´ıci´ o egy m´asik alakja, ezut´an az o¨sszegb˝ ol csak a nagy tagokat tartottuk meg, majd ezek mindegyik´et lecs¨okkentett¨ uk a megadott cD als´ o korl´atra.

1
1 eset´en ad ´erdemi inform´ aci´ ot, hiszen a val´ osz´ın˝ us´eg eleve legfeljebb 1. A speci´alis eset¨ unkben ezt a k¨ ovetkez˝ o form´ aban fogjuk haszn´alni: Ha a v´altoz´ o altal egyforma val´ ´ osz´ın˝ us´eggel felvett N darab vi ´ert´ek k¨oz¨ott s olyan van, amelynek a v´ arhat´ o ´ert´ekt˝ ol val´o elt´er´ese nagyobb a sz´or´ as c-szeres´en´el, azaz s-szer fordul el˝ o, hogy |vi − E| > cD, akkor s/N < 1/c2 . N 

A r´esz¨osszegek sz´ ama 2k − 1. Mindegyik legal´abb 1 ´es legfeljebb

Mindk´et oldal 2-es alap´ u logaritmus´ at v´eve kapjuk, hogy

  1 P |ξ − E| > cD < 2 . c

(1.5)

max k  1 + log2 n > log2 n.

3

max k = 1, log2 n

azaz max k aszimptotikusan egyenl˝ o log2 n-nel. Erd˝os 500 doll´art aj´anlott fel annak eld¨ ont´es´e´ert, hogy | max k − log2 n| korl´atos-e. Ez tov´ abbra is rem´enytelen¨ ul neh´ez. R´at´er¨ unk a (2.1) als´ o ´es (2.5) fels˝ o becsl´esek jav´ıt´ as´ ara. B´ar sok´aig azt sejtett´ek, hogy a 2-hatv´ anyok adj´ ak a legjobb konstrukci´ ot, kider¨ ult, hogy enn´el eggyel(!) abbra is megoldatlan, hogy a kett¨obb sz´ amot is meg lehet adni, ha n  221 . Az tov´ t˝ohatv´ anyokn´ al kett˝ovel t¨ obb sz´ am megadhat´ o-e. A fels˝o becsl´essel kapcsolatban Erd˝osnek ´es Leo Moser (1921–1970) kanadai ult megfelezni¨ uk 70 ´evvel matematikusnak a (2.5)-beli log2 log2 n-es hibatagot siker¨ 4

K¨ oz´ episkolai Matematikai ´ es Fizikai Lapok, 2022/1

ezel˝ ott, ´es ma is ez a legjobb ismert eredm´eny. Ehhez a probl´em´ at ´atfogalmazt´ ak a val´ osz´ın˝ us´egsz´ am´ıt´ as nyelv´ere. ´Igy j¨ ott be a k´epbe a Csebisev-egyenl˝ otlens´eg, ami azt biztos´ıtotta, hogy a r´esz¨ osszegek az [1, nk − 1] intervallumban nem egyenletesen oszlanak el, hanem az ´atlag k¨ ozel´eben s˝ ur˝ us¨ odnek. N´ezz¨ uk mindezt r´eszletesen. Legyen ξ az a val´ osz´ın˝ us´egi v´ altoz´ o, amelyik mind osszeget (bele´ertve az u us´eggel veszi fel. a 2k darab r´esz¨ ¨reset is) 1/2k val´osz´ın˝ Ez azt jelenti, hogy az egyes aj eg´eszek egym´ ast´ ol f¨ uggetlen¨ ul 1/2–1/2 val´ osz´ıuggetlen val´ osz´ın˝ us´eggel szerepelnek a r´esz¨ osszegekben. Eszerint ξ az η1 , . . . , ηk f¨ n˝ us´egi v´ altoz´ ok o osz´ın˝ us´eggel: ¨sszege, ahol ηj a 0 ´es aj ´ert´ekeket veszi fel 1/2–1/2 val´ k ξ = j=1 ηj . Ennek megfelel˝ oen az ηj v´ altoz´ o v´ arhat´ o ´ert´eke ´es sz´ or´ asn´egyzete   2 aj aj ´es D2 (ηj ) = . (2.6) E(ηj ) = 2 2

3. Tipikusan h´any pr´ımoszt´oja van egy sz´amnak? ´ er¨ Att´ unk a m´asodik sz´ amelm´eleti k´erd´es¨ unkre. Jel¨ olje ω(n) az n pozit´ıv eg´esz k¨ ul¨onb¨oz˝ o pr´ımoszt´ oinak a sz´am´at. Pl. ω(20) = 2, ω(64) = 1. V´egtelen sok n-re, a pr´ımhatv´ anyokra ω(n) = 1. De b´ armilyen nagy is lehet: az els˝ o s pr´ım szorzat´ ara ω(n) = s, pl. ω(2 · 3 · 5 · 7 · 11 · 13 · 17 · 19) = ω(9 699 690) = 8. B´ar nem nagyon l´atszik semmi szab´alyoss´ ag, bel´ atjuk hogy egy j´ol ismert, ul, hogy a legt¨ obb sz´ amra ω(n) kb. f (n). sz´ep” f (n) f¨ uggv´ennyel m´egis teljes¨ ” A legt¨obb” ´es kb.” kifejez´esek persze pontos matematikai ´ertelmet nyernek majd. ” ” Legyen t egy tetsz˝ oleges (nagy) pozit´ıv eg´esz. El˝ osz¨ or kisz´am´ıtjuk, hogy ω(n) ´atlagosan mekkora 1 ´es t k¨ oz¨ ott: E=

A val´ osz´ın˝ us´egi v´ altoz´ ok ¨ osszeg´enek v´ arhat´ o ´ert´ek´ere ´es (f¨ uggetlens´eg eset´en) sz´or´ asn´egyzet´ere vonatkoz´ o (1.3) ´es (1.4) k´epletek alapj´an (2.6)-b´ol kapjuk, hogy

(2.7)

E(ξ) =

k 

E(ηj ) =

j=1

W (t) becsl´es´ehez tekints¨ uk azt a t × t-es t´abl´ azatot, amelynek az i-edik sor´aban az ω(i)-t sz´ amoljuk meg”, azaz minden j-edik helyre 1-et ´ırunk, ahol a j ” az i valamelyik pr´ımoszt´ oja, a t¨ obbi helyre pedig 0 ker¨ ul. Pl. t = 6-ra:

k  aj j=1

ω(1) + ω(2) + . . . + ω(t) W (t) = . t t

2

´es (2.8)

D2 (ξ) =

k 

D2 (ηj ) =

j=1

k   aj 2 j=1

2

kn2 . < 4

1. feladat. Bizony´ıtsuk be (2.7)-et ´es (2.8)-at k¨ozvetlen¨ ul, a v´arhat´ o ´ert´ek ´es sz´ or´ asn´egyzet additivit´ as´ ara t¨ ort´en˝ o hivatkoz´ as n´elk¨ ul. (A feladatok megold´as´ at a cikk v´eg´en v´ azoljuk.) √ A ξ v´ altoz´ o sz´or´ as´ ara (2.8)-b´ ol a D(ξ) < n k/2 fels˝o becsl´es ad´ odik. Alkalmazzuk most az (1.5) Csebisev-egyenl˝otlens´eget pl. c = 2-vel. Ez azt jelenti, hogy az ´ atlagt´ ol a sz´ or´ as k´etszeres´en´el t´ avolabb es˝ o r´esz¨osszegek sz´ ama kevesebb, mint az ¨ osszes r´esz¨ osszeg sz´ am´anak a negyedr´esze. √ osszeg esik egy 4D(ξ) < 2n k hossz´ us´ag´ u intervallumTeh´ at t¨ obb, mint 2k · 34 ¨ ba, amelynek a k¨ oz´eppontja E(ξ). Ezek az ¨ osszegek mind k¨ ul¨ onb¨ oz˝ ok, ´ıgy √ √ 8n k k 3 k . (2.9) 2 · < 2n k, azaz 2 < 4 3 ¨ Osszehasonl´ ıtva (2.9)-et az els˝ o gondolatmenettel ad´ od´o (2.2)-vel, azt nyer√ t¨ uk, hogy (2.9) jobb oldal´an k helyett csak k egy konstansszorosa ´all. Ha most a kor´ abbihoz hasonl´ oan k´etszer logaritm´ alunk, akkor (n > 8-ra)

1 0 0 0 0 0 0

2 0 1 0 1 0 1

3 0 0 1 0 0 1

4 0 0 0 0 0 0

5 0 0 0 0 1 0

6 0 0 0 0 0 0

Teh´at az i-edik sor j-edik eleme 1, ha j pr´ımsz´ am ´es oszt´ oja i-nek, egy´ebk´ent pedig 0. H´any 1-es van a t´abl´ azatban? Soronk´ent sz´ amolva (3.1)

t 

ω(i) = W (t).

i=1

Most sz´ amoljunk oszloponk´ent. Ha j = p pr´ımsz´ am, akkor a p-edik oszlopban a p t¨obbsz¨or¨oseinek megfelel˝o helyeken a´ll 1, teh´ at itt t/p darab 1-es van, minden m´as elem 0. ´Igy oszloponk´ent ¨ osszegezve az 1-esek sz´ ama

 t (3.2) , p pt

log2 log2 n +2 2 ad´ odik, teh´ at (2.5)-ben a hibatagot l´enyeg´eben megfelezt¨ uk. k < log2 n +

K¨ oz´ episkolai Matematikai ´ es Fizikai Lapok, 2022/1

ω(1) = 0 ω(2) = 1 ω(3) = 1 ω(4) = 1 ω(5) = 1 ω(6) = 2

ahol az ¨osszegz´es csak a pr´ımekre t¨ ort´enik. A tov´ abbiakban p, q mindig pr´ımsz´amot jel¨ol. 5

6

K¨ oz´ episkolai Matematikai ´ es Fizikai Lapok, 2022/1

A soronk´enti ´es oszloponk´enti ¨ osszegz´est ¨ osszevetve kapjuk, hogy  t

(3.3) W (t) = . p pt

Megjegyezz¨ uk, hogy (3.3)-at tulajdonk´eppen egy egyszer˝ u ¨osszeg´ atrendez´essel igazoltuk: t t     t

  ω(i) = 1= 1= . W (t) = p i=1 i=1 pt p|i, it

p|i

pt

(3.3)-at t-vel osztva ad´ odik, hogy ω(n) ´ atlaga 1  n  t-re (3.4)

 W (t) 1 t 1 E= = = − h(t), t t p p pt

ahol

0  h(t) < 1.

pt

Az utols´ o egyenl˝ os´eg az eg´eszr´eszek elhagy´ as´ ab´ ol ad´odott. Ekkor ugyanis mind a π(t) darab tag egy 1-n´el kisebb sz´ ammal n˝ott, ahol π(t) a pr´ımek sz´ama t-ig. ´Igy az ¨ osszeg π(t)-n´el kevesebbel n˝ ott, ezt t-vel osztva kapjuk, hogy 0  h(t) < π(t)/t < < 1. Mivel π(t)/t tart a 0-hoz, ha t → ∞, enn´el er˝ osebb a´ll´ıt´ as is igaz: lim h(t) = 0. t→∞

(3.4) teh´at azt fejezi ki, hogy az 1 ´es t k¨ oz¨ otti eg´eszek pr´ımoszt´ oinak ´atlagos sz´ ama l´enyeg´eben a t-n´el nem nagyobb pr´ımsz´ amok reciprok¨osszeg´evel egyenl˝ o. Felhaszn´ aljuk, hogy ez a reciprok¨ osszeg nagyon j´o k¨ozel´ıt´essel ln ln t: l´etezik olyan ut konstans, hogy b´ armely t-re c1 abszol´

 1

− ln ln t

< c1 ,

p pt

l´asd pl. [1], 5.6.2. t´etel. Ez azt jelenti, hogy ω(n) ´ atlagosan kb. ln ln t. Vagyis egy 1 ´es t k¨ oz¨ otti eg´esznek a´tlagosan ln ln t k¨ ul¨ onb¨ oz˝ o pr´ımoszt´ oja van. ln t mind¨ossze Az ln ln x f¨ uggv´eny nagyon ovekszik, pl. t = 10100 -ra ln√ √ lassan n¨ 5,44. Vegy¨ uk ´eszre, hogy ln ln t = ln(ln t/2) = ln ln t − ln 2 miatt a t ´es t k¨oz¨otti n¨ oveked´es csak ln 2. Ez´ert az 1 ´es t k¨ oz¨ otti majdnem minden” n eset´en kb. ln ln t” ” ” helyett nyugodtan mondhatunk kb. ln ln n”-et is. ” Abb´ ol azonban, hogy ln ln n az ´atlag, nem k¨ ovetkezik, hogy a legt¨obb esetben ehhez k¨ ozeli a f¨ uggv´eny´ert´ek, lehetn´enek nagy ingadoz´ asok ´es u ´gy j¨onne ki ez az ´ atlag. Hardy ´es Ramanujan bonyolult m´ odszerekkel bel´ att´ ak 1917-ben, hogy nem ´ıgy van, a legt¨ obb” n sz´ amnak t´enyleg kb. ln ln n pr´ımoszt´ oja van. ” 2. feladat. Ebben nem az a meglep˝ o, hogy a legt¨obb sz´ amnak ilyen kev´es pr´ımoszt´ oja van, hanem az, hogy ilyen sok. Magyar´azzuk meg ezt a kijelent´est.

A Hardy–Ramanujan-t´etelre Tur´ an adott egyszer˝ u bizony´ıt´ ast 1934-ben. Ebben tulajdonk´eppen a Csebisev-egyenl˝otlens´eget haszn´ alta (an´elk¨ ul, hogy ennek tudat´ aban lett volna), ´es ez lett a kiindul´ opontja a val´osz´ın˝ us´egsz´ am´ıt´ as sz´amelm´eleti alkalmaz´ asainak. A l´enyeget teh´ at itt is a val´osz´ın˝ us´egsz´ am´ıt´ asi megk¨ ozel´ıt´es adja. Legyen ξ az a val´ osz´ın˝ us´egi v´ altoz´ o, amelyik az ω(1), . . . , ω(t) ´ert´ekek mindegyik´et 1/t val´ o´ sz´ın˝ us´eggel veszi fel. Eppen ennek E ≈ ln ln t v´ arhat´ o ´ e rt´ e k´ e t sz´ a moltuk ki az el˝ o√ z˝oekben. Bel´ atjuk, hogy kicsi a sz´ or´ as, kisebb, mint 3E. Ekkor az (1.5) Csebisevegyenl˝ otlens´eget el´eg nagy c-vel alkalmazva ξ-nek az √ E ≈ ln ln t v´ arhat´ o ´ert´ekt˝ol val´o elt´er´ese 1-hez ak´ armilyen k¨ ozeli val´osz´ın˝ us´eggel E egy konstansszoros´an´al kisebb. Ez azt jelenti, hogy az 1 ´es t k¨ oz¨ otti majdnem minden eg´eszre ω(n) na´ ahogy kor´ gyon j´ol k¨ozel´ıthet˝ o ln ln t-vel. Es abban jelezt¨ uk, majdnem minden ilyen n eg´eszre ln ln t helyett ez a t˝ ole alig k¨ ul¨ onb¨ oz˝ o ln ln n-nel is teljes¨ ul. Azaz val´ oban majdnem minden n eg´esz sz´ amnak kb. ln ln n k¨ ul¨ onb¨ oz˝ o pr´ımoszt´ oja van. (A prec´ız, form´alis megfogalmaz´ ast l´ asd pl. [1], 6.7.7. ´es 6.7.7a. t´etelek.) N´ezz¨ uk teh´ at a sz´ or´ as fels˝o becsl´es´et. El˝ osz¨ or Gyenes Zolt´ an javaslata alapj´an egy heurisztikus gondolatmenetet mutatunk, ´es ut´ ana k¨ ovetkezik majd a prec´ız levezet´es. atorv´ altoz´ ok ¨ osszeg´ere. A ξ v´altoz´ ot fel tudjuk bontani pr´ımenk´enti ηp indik´ Legyen p pr´ım ´es ηp ´ert´eke 1/p val´ osz´ın˝ us´eggel 1 ( a sz´ am oszthat´o p-vel”) ´es ” (p − 1)/p val´osz´ın˝ us´eggel 0 ( a sz´ am nem oszthat´o p-vel”). Ekkor ” (3.5)

ηp .

Az ηp val´osz´ın˝ us´egi v´ altoz´ ok v´ arhat´ o ´ert´eke ´es sz´ or´ asn´egyzete

(3.6)

E(ηp ) =

1 p

´es D2 (ηp ) =

1 p



p−1 p

2

+

p−1 p

2 1 1 1 = − 2. p p p

Az ¨osszes pozit´ıv eg´eszre n´ezve az ηp v´ altoz´ ok f¨ uggetlenek. Ugyanis a k¨ ul¨ onb¨ oz˝ o pr´ımekkel val´o oszthat´ os´ agok a p´ aronk´ent relat´ıv pr´ıms´eg miatt egym´ ast´ ol f¨ uggetlenek. Form´ alisan: a p-vel val´ o oszthat´ os´ ag val´osz´ın˝ us´ege 1/p ´es a p1 , . . . , pr pr´ımek mindegyik´evel val´o oszthat´os´ ag ugyanaz, mint az M = p1 · . . . · pr szorzattal val´o osz´ın˝ us´egek oszthat´ os´ag, teh´at ennek a val´osz´ın˝ us´ege 1/M , ami val´ oban az 1/pi val´ szorzata. altoz´ ok el´eg nagy t-re az 1 ´es t Kicsit” csalva tekints¨ uk u ´gy, hogy az ηp v´ ” k¨oz¨otti eg´eszekre szor´ıtkozva is f¨ uggetlenek maradnak. Ekkor

D2 (ξ) =

 pt

i=1

7

 pt

oinak a sz´am´at, teh´at, hogy az n 3. feladat. Jel¨ olje Ω(n) az n ¨ osszes” pr´ımoszt´ ” h´ any pr´ımsz´ am szorzata. Pl. Ω(20) = 3, Ω(64) = 6. Mutassuk meg, hogy a Hardy– t    Ramanujan-t´etel Ω(n)-re is ´erv´enyes. Ehhez igazoljuk, hogy Ω(i) − ω(i) < t. K¨ oz´ episkolai Matematikai ´ es Fizikai Lapok, 2022/1

ξ=

8

D2 (ηp ) =

1 pt

p



 1 ≈ E. p2 pt

K¨ oz´ episkolai Matematikai ´ es Fizikai Lapok, 2022/1

 2 A heurisztika ut´ an n´ezz¨ uk a prec´ız levezet´est. Mivel D2 (ξ) = E (ξ − E) = = E(ξ 2 ) − E 2 , itt az els˝o tagot kell fel¨ ulr˝ ol becs¨ uln¨ unk. E(ξ 2 ) =

(3.7)

t t  1 2 1 1  ω (i) = ω(i) 1= ω(i) = t i=1 t i=1 t pt

p|i

=

p|i

 1  1   1 + ω(v) = ω(vp)  t t pt vt/p

a n´egyzetek ¨osszead´ as´an´ al az ai aj tagok kiesnek, mert ugyanannyiszor fordulnak el˝o pozit´ıv egy¨ utthat´ oval, mint negat´ıvval. Ugyanis pozit´ıv el˝ ojelet akkor kapunk, ¨ oz¨ ul vagy mindkett˝ o szerepel, vagy egyik sem. Osszesen ha a r´esz¨osszegben ai ´es aj k¨ osszeg van, hiszen a t¨ obbi k − 2 darab ar mindegyike egy2k−2 + 2k−2 ilyen r´esz¨ m´ ast´ ol f¨ uggetlen¨ ul lehet tagja vagy nem tagja a r´esz¨ osszegnek. Ugyanez a helyzet oz¨ ul pontosan az egyik szerepel. azokkal a r´esz¨ osszegekkel, amelyekben ai ´es aj k¨ ´Igy a k¨ ul¨onbs´egek n´egyzet¨ osszeg´enek ´ atlaga

pt vt/p

2k



1 t 1  = ω(v). + t p t pt

j=1

pt vt/p

Itt el˝ osz¨ or a v´arhat´ o ´ert´ek (1.1) defin´ıci´oj´at alkalmaztuk ξ helyett ξ 2 -re, majd az (egyik) ω(i)-re be´ırtuk annak defin´ıci´ oj´at. Ezut´ an ´atrendezt¨ uk az ¨osszeget ´es at´ırtuk a p | i oszthat´ ´ os´agot i = vp alakra. Az ω(vp)  1 + ω(v) egyenl˝ otlens´eg az´ert igaz, mert egy sz´amot egy pr´ımsz´ ammal szorozva legfeljebb eggyel n˝ o a pr´ımoszt´ ok sz´ ama. Az utols´ o l´ep´esben k´et r´eszre v´agtuk az ¨ osszeget. (3.7) utols´ o sor´ aban az els˝o ¨ osszeg ´eppen E(ξ) = E. A m´ asodik ¨osszeg bels˝ o szumm´aj´at a´t´ırjuk (3.3)-hoz hasonl´oan, csak t helyett t/p-vel, majd fel¨ ulr˝ ol becs¨ ulj¨ uk: ⎢ ⎥ ⎢ ⎥  t  ⎢  pt  ⎥  t1 ⎣ ⎦< < . ω(v) = q pq p q vt/p

qt/p

qt/p

qt

Ennek alapj´an a teljes m´asodik o ¨sszeg kisebb, mint  2  2 1 (3.8) = E + h(t) = E 2 + 2h(t)E + h2 (t) < E 2 + 2E. p

k  a2j j=1

4

.

3. A jelzett egyenl˝ otlens´eg alapj´ an a Ω(n)-re vonatkoz´ o k´erd´est visszavezethetj¨ uk a ω(n)-es Hardy–Ramanujan-t´etelre. Az egyenl˝otlens´eg igazol´ as´ ahoz pedig a (3.3) ´atalak´ıt´ as mint´aj´ara l´ assuk be, hogy a k´erd´eses k¨ ul¨ onbs´eg 

r2, pr t

(3.7) ´es (3.8) alapj´ an E(ξ 2 ) < E 2 + 3E, ´ıgy D2 = E(ξ 2 ) − E 2 (ξ) < E 2 + 3E − E 2 = = 3E, ahogy ´all´ıtottuk. Megjegyezz¨ uk, hogy a heurisztikusan kapott eredm´eny is el´erhet˝ o, azaz a 3-as szorz´ o l´enyeg´eben elhagyhat´ o a sz´ or´ asn´egyzet fels˝ o korl´ atj´ an´al. Ugyanut konstanssal (l´ asd pl. [1], 5.4.3. t´etel), ez´ert is π(t) < c2 lnt t alkalmas c2 abszol´



t . pr

´Igy el´eg megmutatni, hogy az ¨ osszes (egyn´el nagyobb kitev˝oj˝ u) pr´ımhatv´ any reciprok¨osszege kisebb, mint 1. Egy adott p ilyen hatv´ anyainak a reciprokai m´ertani 1 . Ezeket pr´ımek helyett minden t  2 sorozatot alkotnak, amelynek ¨ osszege p(p−1) sz´ amra ¨osszegezve ∞ 

π(t)

c2 , ´es ´ıgy (3.8)-ban h(t) < t < ln t

2c2 · 1,01 ln ln t → 0, ln t

4

/2k =

2. Ha majdnem minden n sz´ amnak l´enyegesen” t¨ obb, mint ln ln n pr´ımoszt´ o” ja lenne, akkor 1 ´es t k¨ oz¨ ott ezek a´tlaga t¨ obb lenne, mint ln ln t, ami ellentmond´as. Az nyugodtan el˝ ofordulhatna, hogy a legt¨ obb n sz´ amnak j´oval kevesebb, mint ln ln n pr´ımoszt´ oja van, ´es a nagyobb a´tlagot a kev´es kiugr´ o ω(n) ´ert´ek okozza. P´eld´aul (ellent´etben az ω(n)-nel ´es Ω(n)-nel) a d(n) oszt´ ok sz´ama f¨ uggv´enyn´el t´enylegesen ez a helyzet: d(n) ´ atlag´ert´eke ln n, ugyanakkor a legt¨ obb n eg´esznek csak ln 2 oja van. Ez ut´ obbi t´eny ´eppen a k¨ onnyen igazolhat´ o 2ω(n)  kb. (ln n) pozit´ıv oszt´ Ω(n) kett˝ os egyenl˝otlens´egb˝ ol ´es a Hardy–Ramanujan-t´etelb˝ ol k¨ ovetke d(n)  2 zik.

pt

2h(t)E
ε ´es 180◦ − 3β > 0, ´ıgy 60◦ > ε (> 0◦ ). Most, hogy kisz´amoltuk a bels˝ o sz¨ ogeket, j¨ ohet a szinuszt´etel alkalmaz´ asa. A P BC h´aromsz¨ ogre sin (2ε − β) PB = , PC sin ε a P AC h´aromsz¨ ogre pedig

Kedvcsin´al´ o a Geometriafeladatok megold´asa ” szinuszt´etellel” c´ım˝ u ´ır´ashoz Bevezet´es A geometriafeladatok megold´ as´ an´ al a klasszikus, megfelel˝o objektumok (pl. sz¨ ogek, hasonl´ os´agok) ´eszrev´etel´en ´es megfigyel´es´en alapul´ o megold´ asi m´odszerek mellett sz´ amol´asos m´odszerekkel is eredm´enyre juthatunk. Ezek k¨oz´e tartozik a sz¨ogek szinusz´ anak sz´amol´as´ an alapul´o megold´ asm´ od – t¨obbek k¨oz¨ott a koordin´ atageometriai, vektoros/komplex sz´amos ´es baricentrikus koordin´ at´ az´ o m´odszerek mellett – melynek f˝ o seg´edeszk¨ oze a szinuszt´etel (tov´ abbiak a Ceva-t´etel ´es az add´ıci´ os k´epletek). El˝ onye, hogy alkalmaz´ asa m´ asmilyen gondolkod´ ast, ¨otleteket ig´enyel, mint a klasszikus m´odszerek, ´ıgy az azokkal nehezen z¨old´ agra verg˝od˝o versenyz˝ oknek is alternat´ıv´ at ny´ ujt (kevesebb a vegy¨ uk ´eszre, hogy . . . ” t´ıpus´ u r´esz), mi” os´aga korl´ak¨ ozben nem ig´enyel sokkal t¨ obb el˝ oismeretet. Ugyanakkor alkalmazhat´ tozottabb. Ezen m´ odszert mutatja be a Geometriafeladatok megold´ asa szinusz” t´etellel” c´ım˝ u ´ır´ asom, mely megtal´ alhat´ o a K¨ oMaL honlapj´an a cikkek k¨oz¨ott: https://www.komal.hu/cikkek/cikklista.h.shtml. A bevezet´es ut´ an elm´eleti ¨ osszefoglal´ assal folytat´ od´o cikk versenyfeladatok megold´ asain kereszt¨ ul p´eld´ akat hoz a m´odszer alkalmazhat´ os´ag´ara ´es tov´abbi egy´eni kidolgoz´ast ig´enyl˝ o, vele megoldhat˝ o feladatokat tartalmaz. Az ismertet˝o tov´ abbi r´esze ezekb˝ ol mutat meg r´eszleteket. Egy geometriafeladat kidolgozott megold´assal EGMO/IMO/MEMO 1. v´alogat´ overseny 2019., 3. feladat. Legyen az ABC h´aromsz¨ ogben CAB = 2 · ABC. Tegy¨ uk fel, hogy l´etezik egy P pont a h´ aromsz¨ og belsej´eben, amelyre AP = BP ´es CP = AC. Bizony´ıtsd be, hogy ekkor P BC = 30◦ .

sin (180◦ − 2β − 2ε) sin (2ε − β) = . sin ε sin(β + ε) Mivel sin (180◦ − 2β − 2ε) = sin (2β + 2ε) = 2 cos (β + ε) sin (β + ε), ez´ert sin (180◦ − 2β − 2ε) = 2 cos (β + ε) sin (β + ε) (h´ aromsz¨og bels˝o sz¨ ogek´ent sin (β + ε) = 0 ´es sin ε = 0). Ezt felhaszn´ alva sin (2ε − β) = 2 cos (β + ε) sin ε ´es 2 cos (β + ε) sin ε = sin (2ε − β). Add´ıci´ os t´etelekb˝ ol 2 cos (β + ε) sin ε = 2(cos β cos ε − sin β sin ε) sin ε = 2 cos β cos ε sin ε − 2 sin β sin2 ε, m´ıg sin (2ε − β) = sin 2ε cos (−β) + sin(−β) cos 2ε = sin 2ε cos β − sin β cos 2ε = = 2 sin ε cos ε cos β − (sin β cos2 ε − sin β sin2 ε) =

Ezek egyenl˝os´eg´eb˝ ol 2 cos β cos ε sin ε − 2 sin β sin2 ε = 2 cos β sin ε cos ε − sin β cos2 ε + sin β sin2 ε, ´ıgy rendezve

P BA = ABC − P BC = β − ε.

sin β cos2 ε = 3 sin β sin2 ε,

Mivel az ABP h´ aromsz¨ og egyenl˝o sz´ ar´ u, a P AB is β − ε. Emellett CAB = 2β ´es CAP  = β + ε, az egyenl˝o sz´ ar´ u AP C h´ aromsz¨ ogben az AP C is β + ε. Ez´ert az AP C h´ aromsz¨ og harmadik bels˝ o sz¨ oge ACP  = 180◦ − 2β − 2ε. Mivel az ABC h´ aromsz¨ogben a B-n´el, illetve A-n´ al l´ev˝ o bels˝o sz¨ ogek β, illetve 2β, ACB = 180◦ − 3β ´es ebb˝ ol BCP  = ACB − ACP  = 180 − 3β − (180 − 2β − 2ε) = 2ε − β. K¨ oz´ episkolai Matematikai ´ es Fizikai Lapok, 2022/1

B PA =P , ´ıgy Mivel AP = BP , P PC C

= 2 cos β sin ε cos ε − sin β cos2 ε + sin β sin2 ε.

Megold´as. Jel¨ olj¨ uk a B-n´el l´ev˝ o bels˝ o sz¨ oget β-val ´es a P BC sz¨ oget, melyr˝ ol az ´ all´ıt´ as sz´ol, ε-nal. Sz´ amoljunk ki bel˝ ol¨ uk n´eh´ any m´ asik sz¨ oget. Mivel P bels˝ o pont,



PA sin (180◦ − 2β − 2ε) = . PC sin (β + ε)



11

ol cos2 ε = 34 ; sin β = 0-val osztva cos2 ε = 3 sin2 ε. Mivel cos2 ε + sin2 ε = 1, ez´ert ebb˝ mivel 60◦ > ε > 0◦ , ez´ert cos ε > 0. ´Igy  √ 3 3 cos ε = = 4 2 ´es ebb˝ ol ezen az intervallumon csak ε = 30◦ lehet. Teh´ at P BC = 30◦ , a feladat ´all´ıt´ as´at bel´attuk.  12

K¨ oz´ episkolai Matematikai ´ es Fizikai Lapok, 2022/1

A t´em´aba v´ag´ o tov´abbi feladatok

A k¨ oz´episkolai tan´arok verseny´enek feladatai

1. feladat (K¨ ursch´ ak verseny 1990/2). Az ABC h´aromsz¨og be´ırt k¨or´enek k¨oz´eppontja legyen K, a hozz´ a´ırt k¨ or¨ ok k¨ oz´eppontjai A0 , B0 , C0 . Jel¨olje A1 a BC oldal ´es a BKC sz¨ og felez˝oj´enek, B1 az AC oldal ´es az AKC sz¨og felez˝oj´enek, og felez˝oj´enek a metsz´espontj´at. Igazoljuk, C1 pedig az AB oldal ´es az AKB sz¨ hogy az A0 A1 , B0 B1 ´es C0 C1 egyenesek egy ponton mennek ´at.

1. A 2020–2021-es tan´ev kezdet´en Kiss tan´ ar u ´r oszt´ aly´ aban a Szereted-e a matekot?” ” k´erd´esre a tanul´ ok 50-50%-a v´ alaszolt igennel illetve nemmel. A tan´ev v´eg´en megism´etelt k´erd´esre m´ ar az igen v´ alaszok ar´ anya 70%-ra n˝ ott, m´ıg a nem v´ alaszok ar´ anya 30%-ra cs¨ okkent. A tan´ev sor´ an a di´ akok p%-a v´ altoztatta meg a v´elem´eny´et. Mennyi a k¨ ul¨ onbs´eg p maxim´ alis ´es minim´ alis ´ert´eke k¨ oz¨ ott? (A) 0; (B) 20; (C) 40; (D) 60; (E) 80. 2. Az ABCDE konvex ¨ otsz¨ og oldalainak hossza AB = 3, BC = 4, CD = 6, DE = 3, EA = 7. Az ¨ otsz¨ oget az ´ abr´ an l´ athat´ o m´ odon elhelyezz¨ uk a koordin´ atarendszerben u ´ gy, hogy az A cs´ ucs az orig´ oban, a B pedig az x tengely pozit´ıv fel´en helyezkedjen el. Ezut´ an az o oget az ´ oramutat´ o j´ ar´ a¨tsz¨ s´ anak megfelel˝ oen elkezdj¨ uk g¨ orgetni az x tengelyen. Melyik oldal fogja ´erinteni az x tengely (2021; 0) pontj´ at? (A) AB; (B) BC; (C) CD; (D) DE; (E) EA.

2. feladat (IMO 2019 Shortlist G2 – Sur´ anyi J´anos eml´ekverseny 2020/1). Legyen ABC hegyessz¨ og˝ u h´ aromsz¨ og, az A, B ´es C-b˝ ol indul´o magass´ agok talppontjai legyenek rendre D, E, F . Legyen kb ´es kc a BDF ´es CDE h´aromsz¨ogek be´ırt k¨ore, ezek ´erints´ek a DF ´es DE szakaszokat rendre az M ´es N pontokban. Az M N egyeor¨ okkel vett m´ asik metsz´espontja rendre P ´es Q. Igazoljuk, hogy nesnek a kb ´es kc k¨ M P = N Q. 3. feladat (IMO 2001/5). Az ABC h´ aromsz¨ ogben legyen AP a BAC felez˝oje, ahol P a BC oldalon van, BQ pedig az ABC felez˝oje, ahol Q a CA oldalon van. Tudjuk, hogy BAC = 60◦ ´es hogy AB + BP = AQ + QB. Mik az ABC h´ aromsz¨ og sz¨ ogeinek lehets´eges ´ert´ekei? Fu ¨redi Erik

3. A 10, 2, 5, 2, 4, 2, x sz´ amok ´ atlaga, medi´ anja ´es m´ odusza valamilyen sorrendben egy nem a ´lland´ o sz´ amtani sorozat h´ arom egym´ ast k¨ ovet˝ o tagja. Mennyi x lehets´eges ´ert´ekeinek osszege? (A) 3; (B) 6; (C) 9; (D) 17; (E) 20. ¨ 4. H´ any k¨ ul¨ onb¨ oz˝ o k´etjegy˝ u pozit´ıv eg´esz sz´ ammal oszthat´ o 224 − 1? (C) 10; (D) 12; (E) 14.

(A) 4;

(B) 8;

5. Az ABCD t´eglalapban AB = 6 ´es BC = 3. M a CD oldal azon pontja, melyre (B) 60◦ ; (C) 72◦ ; (D) 75◦ ; AM D = BM A. Mekkora az AM D? (A) 30◦ ; (E) 80◦ . 6. H´ any term´eszetes sz´ amokb´ ol ´ all´ o (a; b; c; d) sz´ amn´egyes megold´ asa az al´ abbi egyenletnek?

60. R´atz L´aszl´ o V´andorgy˝ ul´es 2021. j´ ulius 1–3.

a lg 2 + b lg 3 + c lg 5 + d lg 7 = 2021

A koronav´ırus-vil´agj´arv´ any miatt a Bolyai J´anos Matematikai T´arsulat a 2021. ´evi v´ andorgy˝ ul´est online form´aban rendezte meg. Az esem´enyr˝ol, illetve annak el˝ o´ k´esz¨ uleteir˝ ol, az online form´ar´ ol hossz´ u besz´ amol´o olvashat´ o az Erint˝ o Elektronikus Matematikai Lapok szeptemberi sz´ am´aban (https://ematlap.hu/hirekujdonsagok-2021-4/1120-a-60-online-ratz-laszlo-vandorgyules). Nagyon sajn´altuk, hogy nem tal´ alkozhattunk szem´elyesen, de az online-nak el˝onye is van: ut´ olag az ¨ osszes el˝oad´ as megtekinthet˝ o a v´andorgy˝ ul´es honlapj´an (https://www.bolyai.hu/60-ratz-laszlo-vandorgyules). A 2021. ´evi Beke Man´ o Eml´ekd´ıjas tan´ arok m´eltat´ as´at is meghallgathattuk, a d´ıjazottak: Czimmermann J´ozsef, Dr. M´ader Attila, Fony´on´e N´emeth Ildik´o, Hujter B´alint, Kozma L´aszl´ on´e, Moln´ar Judit ´es Dr. Moln´ar Istv´an (megosztott d´ıj) ´es Tomcs´anyi Szab´ o Katalin. Az ´altal´ anos iskol´as ´es k¨ oz´episkol´as tan´ arversenyt is online rendezt´ek meg, a feladatokat ´es az eredm´enyeket k¨ ul¨ on k¨ oz¨ olj¨ uk. A 2022-es v´ andorgy˝ ul´es tervezett helysz´ıne m´ar harmadik ´eve Eger, nagyon rem´elj¨ uk, hogy ez´ uttal m´ ar szem´elyesen vehet¨ unk r´eszt ezen a nagy m´ ult´ u ´es sz´ınvonalas esem´enyen. K¨ oz´ episkolai Matematikai ´ es Fizikai Lapok, 2022/1

13

(A) 0; (B)

(B) 1;

(C) 4;

(D) 2021;

(E) v´egtelen sok.

7. Ha tg α + tg β = 5 ´es ctg α + ctg β = 6, akkor mekkora tg(α + β) ´ert´eke? √ 3 ; (C) 3; (D) 11; (E) 30.

(A) 1;

8. K´et k˝ om˝ uves k¨ oz¨ ul az egyik 10, a m´ asik pedig 9 ´ ora alatt tud egy k´em´enyt felfalazni. Amikor egy¨ utt dolgoznak, akkor sokat besz´elgetnek, ez´ert romlik a teljes´ıtm´eny¨ uk, ´es ´ıgy egy¨ uttesen o ´r´ ank´ent 10 t´egl´ aval kevesebbet tudnak be´ep´ıteni. Egy¨ uttes munk´ aval 5´ ora alatt tudj´ ak fel´ep´ıteni a k´em´enyt. H´ any t´egl´ ab´ ol ´ all a k´em´eny? (A) 500; (B) 900; (C) 950; (D) 1000; (E) 1800. 9. Egy reggeli sor´ an Viki csal´ adj´ anak minden tagja azonos t´erfogat´ u tejesk´ av´et ivott. A k´ av´e ´es tej mennyis´ege cs´esz´er˝ ol cs´esz´ere v´ altozott, de minden poh´ ar tartalmazta mindk´et ¨ osszetev˝ ot. Viki a teljes tejmennyis´eg negyed´et, a k´ av´enak pedig a hatod´ at itta meg. H´ any szem´elyb˝ ol ´ all Viki csal´ adja? (A) 3; (B) 4; (C) 5; (D) 6; (E) 7. u halmaztri´ onak” nevezz¨ uk, ha 10. Az A, B, C halmazokat minim´ alis metszet˝ ” |A ∩ B| = |B ∩ C| = |C ∩ A| = 1, A, B, C = ∅ ´es az A ∩ B, B ∩ C, C ∩ A halmazok p´ aronk´ent k¨ ul¨ onb¨ oz˝ oek. Pl. az {1, 2}, {2, 3}, {1, 3, 4} halmazok ilyen tulajdons´ ag´ uak. Legyen n a H = {1, 2, 3, 4, 5, 6} halmaz r´eszhalmazaib´ ol alkothat´ o minim´ alis metszet˝ u halmaztri´ ok” ” on bel¨ ul a halmazok sorrendj´et nem vessz¨ uk figyelembe? sz´ ama. Mekkora n ´ert´eke, ha a tri´ (A) 27; (B) 64; (C) 384; (D) 540; (E) 1280.

14

K¨ oz´ episkolai Matematikai ´ es Fizikai Lapok, 2022/1

• Azok, akik legal´ abb 3 pontyot fogtak, azok ´ atlagosan 6 halat tudtak felmutatni a m´erlegel´esn´el.

11. Legyen B az A1 A2 . . . An szab´ alyos n-sz¨ og¨ on k´ıv¨ ul egy olyan pont, melyre aromsz¨ og szab´ alyos, B, A1 , An pedig egy m´ asik szab´ alyos soksz¨ og h´ arom az A1 BA2 h´ egym´ ast k¨ ovet˝ o cs´ ucsa. Mekkora lehet n legnagyobb ´ert´eke? (A) 6; (B) 9; (C) 12; (D) 18; (E) 42. 12. H´ any olyan 1  n  1000 pozit´ıv eg´esz sz´ am van, amely el˝ o´ all´ıthat´ o k´et n´egyzetsz´ am k¨ ul¨ onbs´egek´ent? (A) 250; (B) 500; (C) 750; (D) 999; (E) 1000. 13. Art´ ur kir´ aly 25 lovagja egy k¨ or alak´ u asztaln´ al u aly v´eletlenszer˝ uen ¨l. A kir´ kiv´ alaszt k¨ oz¨ ul¨ uk h´ armat ´es elk¨ uldi o ˝ket, hogy ¨ olj´ek meg a k¨ ozelben tart´ ozkod´ o gonosz s´ ark´ anyt. Jel¨ olje P annak val´ osz´ın˝ us´eg´et, hogy a kijel¨ olt lovagok k¨ oz¨ ott legal´ abb kett˝ o ortalakban fel´ırva mennyi a sz´ aml´ al´ o az asztaln´ al egym´ as szomsz´edja volt. P -t reduk´ alt t¨ ´es a nevez˝ o¨ osszege? (A) 5; (B) 57; (C) 67; (D) 113; (E) 287. 14. Az oszt´ aly matematika ´ or´ an T¨ unde n´enit˝ ol a faktori´ alis fogalm´ at tanulta. D´ avid lelkes volt, ´es kisz´ amolta 1-t˝ ol 20-ig a term´eszetes sz´ amok szorzat´ at, majd a kapott 19jegy˝ u sz´ amot fel´ırta a t´ abl´ ara. Sz¨ unetben azonban valaki let¨ or¨ olt n´eh´ any sz´ amjegyet, ´ıgy most a t´ abl´ an a k¨ ovetkez˝ o egyenl˝ os´eg l´ athat´ o:

• Azok, akik legfeljebb 12 pontyot fogtak, azok neve mell´e ´ atlagosan 5 hal ker¨ ult be az eredm´enylist´ aba. H´ any pontyot fogtak ¨ osszesen a bajnoks´ ag r´esztvev˝ oi? (D) 1024; (E) 2021.

(A) 936;

(B) 943;

(C) 960;

20. R, L, V olyan term´eszetes sz´ amok, melyekre R + L + V = 21. Mennyi a R · L · V + R · L + L · V + V · R kifejez´es maximum´ anak ´ert´eke? (B) 480; (C) 482; (D) 490; (E) 512.

(A) 221;

¨ egys´egn´egyzetet az a 21. Ot ´br´ an l´ athat´ o m´ odon elhelyez¨ unk egy koordin´ atarendszerben. A (c; 0) ´es a (3; 3) pontokat o ot˝ o szakasz a kijel¨ olt tartom´ anyt k´et egyenl˝ o ter¨ ule¨sszek¨ t˝ u r´eszre osztja fel. Mekkora a c ´ert´eke?

20! = 2432902081766,

2

(C) 3 ;

ahol a -ek hely´en ´ all´ o sz´ amjegyek m´ ar nem l´ athat´ ok. Mennyi a let¨ or¨ olt sz´ amjegyek osszege? (A) 4; (B) 5; (C) 6; (D) 8; (E) 10. ¨

3

(D) 4 ;

1

(A) 2 ;

3

(B) 5 ;

4

(E) 5 .

15. Az f (x), g(x), h(x) m´ asodfok´ u f¨ uggv´enyekre f (x) = x2 − 2x + 2,

g(x) = 2x2 − 4x + 3

´es b´ armely x ∈ R eset´en f (x)  h(x)  g(x). Ha h(11) = 181, akkor mekkora h(6) ´ert´eke? (A) 31; (B) 36; (C) 41; (D) 46; (E) t¨ obbf´ele is lehet. 16. Egy h´ aromsz¨ oget felosztunk h´ arom h´ aromsz¨ ogre ´es egy n´egysz¨ ogre u ´gy, hogy a h´ aromsz¨ og k´et cs´ ucs´ at ¨ osszek¨ otj¨ uk a vel¨ uk szemk¨ ozti oldal egy-egy pontj´ aval. A h´ arom h´ aromsz¨ og ter¨ ulete az ´ abra szerint 3, 7, 7 ter¨ uletegys´eg. Mekkora a sz¨ urke n´egysz¨ og ter¨ ulete?

(A) 15;

(B) 17;

35

(C) 2 ;

(D) 18;

23. Jel¨ olje n a 2021 fel´ır´ asainak sz´ am´ at 2021 = a3 · 103 + a2 · 102 + a1 · 10 + a0 foraul egy lehets´eges fel´ır´ asi lehet˝ os´eg: m´ aban, ahol 0  ai  99 (ai ∈ N, i = 0, 1, 2, 3). P´eld´ (A) 3; (B) 9; (C) 202; 2021 = 1 · 103 + 3 · 102 + 67 · 10 + 51. Mekkora n ´ert´eke? (D) 203; (E) 420.

55

(E) 3 .

17. A 48, 84, 108, . . . sorozat tagjai k´et sz´ amtani sorozat megfelel˝ o tagjainak ¨ osszeszorz´ as´ aval j¨ onnek l´etre. Mekkora a sorozat 9. tagja? (A) 0; (B) 32; (C) 126; (D) 180; (E) 420. 18. Az a, b, c, d pozit´ıv eg´esz sz´ amokra teljes¨ ulnek az al´ abbi felt´etelek: a > b > c > d,

a2 − b2 + c2 − d2 = 2022.

a + b + c + d = 2022,

Mekkora a lehets´eges ´ert´ekeinek sz´ ama?

(A) 0;

(B) 1;

(C) 502;

(D) 503;

(E) 2021.

19. Az al´ abbi t´ abl´ azat v´ azlatosan mutatja be az Orsz´ agos Pontyfog´ o Bajnoks´ ag eredm´enyeit: Fogott pontyok sz´ ama (n) Az n pontyot fogott versenyz˝ ok sz´ ama

0 9

1 5

2 7

3 23

... ...

13 5

14 2

15 1

24. K´et doboz mindegyik´eben feh´er ´es fekete goly´ ok vannak. A k´et dobozban egy¨ uttesen 25 goly´ o tal´ alhat´ o. Becsukott szemmel mindk´et dobozb´ ol v´eletlenszer˝ uen kivesz¨ unk 27 egy-egy goly´ ot. Ha annak a val´ osz´ın˝ us´ege, hogy mindkett˝ o fekete lesz 50 , akkor mi a val´ osz´ın˝ us´ege annak, hogy mindk´et kih´ uzott goly´ o feh´er lesz? 5

1

(A) 50 ;

2

(B) 50 ;

4

(C) 50 ;

(D) 50 ; (E) Nem hat´ arozhat´ o meg egy´ertelm˝ uen. 25. Z, a hangya az ABC szab´ alyos h´ aromsz¨ og A cs´ ucs´ ab´ ol indul, ´es minden l´ep´ese sor´ an v´eletlenszer˝ uen a ´tm´ aszik a h´ aromsz¨ og egyik oldal´ an haladva a m´ asik k´et cs´ ucs valamelyik´ebe. Mi a val´ osz´ın˝ us´ege annak, hogy a 6. l´ep´es ut´ an ism´et az A cs´ ucsban fog 8 10 11 12 16 tart´ ozkodni? (A) 32 ; (B) 32 ; (C) 32 ; (D) 32 ; (E) 32 . 26. Az ABCD konvex n´egysz¨ ogben DAB = ABC, BDA = BCD, CD = 5, DA = 8 ´es BD = 10. Mekkora a BC oldal hossza? (A) 12; (B) 12,5; (C) 13; (D) 13,5; (E) 145. 27. Egy pozit´ıv eg´esz sz´ amot k´ıgy´ oz´ onak” nevez¨ unk, ha t´ızes sz´ amrendszerbeli ” a1 a2 a3 . . . ak (k ∈ N+ ) alakj´ ara teljes¨ ul, hogy ai < ai+1 , ha i p´ aratlan ´es ai > ai+1 , ha any k¨ ul¨ onb¨ oz˝ o jegyekb˝ ol ´ all´ o n´egyjegy˝ u k´ıgy´ oz´ o” sz´ am i p´ aros (i ∈ N+ , 1  i < k − 1). H´ ” l´etezik? (A) 252; (B) 630; (C) 882; (D) 1050; (E) 1260.

A Sporthorg´ asz magazin ´ıgy sz´ amolt be a rendezv´enyr˝ ol: • A gy˝ oztes 15 pontyot fogott. K¨ oz´ episkolai Matematikai ´ es Fizikai Lapok, 2022/1

22. Az {1, 2, 3, . . . , 1000} halmazb´ ol v´eletlenszer˝ uen kiv´ alasztunk el˝ obb h´ arom sz´ amot ol m´eg h´ armat {b1 , b2 , b3 }. Legyen a1 , a2 , a3 egy {a1 , a2 , a3 }, majd a marad´ek 997 elemb˝ u t´egla h´ arom, egy cs´ ucsb´ ol t´eglatest alak´ u doboz, b1 , b2 , b3 pedig egy szint´en t´eglatest alak´ kiindul´ o ´el´enek hossza. Legyen P annak a val´ osz´ın˝ us´ege, hogy megfelel˝ o elforgat´ assal a t´egla kil´ og´ as n´elk¨ ul elhelyezhet˝ o a dobozban. P -t reduk´ alt t¨ ortalakban fel´ırva mennyi a sz´ aml´ al´ o ´es a nevez˝ oo ¨sszege? (A) 4; (B) 5; (C) 7; (D) 10; (E) 12.

15

16

K¨ oz´ episkolai Matematikai ´ es Fizikai Lapok, 2022/1

28. A R´ atz L´ aszl´ o V´ andorgy˝ ul´esen a Tan´ arverseny ut´ an az egyik versenyz˝ o a t´ ars´ aval besz´elgetve ´ıgy ´ert´ekelte saj´ at teljes´ıtm´eny´et:

Gyakorl´o feladatsor emelt szint˝ u matematika ´eretts´egire

80 pontn´ al t¨ obbet szereztem. Ha ¨ osszpontsz´ amomat megmondan´ am neked, akkor ” meg tudn´ ad ´ allap´ıtani, hogy h´ any j´ o ´es h´ any rossz v´ alaszt adtam. Viszont b´ armely enn´el gyeng´ebb, de 80 pontn´ al jobb eredm´eny eset´en m´ ar nem lenn´el erre k´epes.”

I. r´esz

(A versenyen a versenyz˝ ok pontsz´ am´ at a p = 30 + 4 · j − r k´eplettel hat´ arozz´ ak meg, ahol j ´es r jel¨ oli rendre a j´ o, illetve a rossz v´ alaszok sz´ am´ at, ´es a megv´ alaszolatlan k´erd´esek´ert nem j´ ar pontlevon´ as.) H´ any k´erd´esre nem adott v´ alaszt a versenyz˝ o? (E) 4.

(A) 0;

(B) 1;

(C) 2;

1. H´arom p´enzv´ alt´ o v´ allalkoz´ as aktu´alis forint-eur´ o´ arfolyamait ismerj¨ uk:

(D) 3;

29. Legyen H azon r (0 < r < 1) racion´ alis sz´ amok halmaza, amelyek v´egtelen periodikus tizedest¨ ort alakja 0,abcabcabc . . . = 0,ab ˙ c, ˙ ahol a, b, c nem felt´etlen¨ ul k¨ ul¨ onb¨ oz˝ o sz´ amjegyek. Fel´ırva H elemeinek reduk´ alt t¨ ort alakj´ at, h´ anyf´ele sz´ aml´ al´ ot kapunk? (A) 630; (B) 642; (C) 648; (D) 660; (E) 998. 30. Egy szab´ alyos p´enz´erm´et egym´ as ut´ an 15-sz¨ or feldobva r¨ ogz´ıtj¨ uk a fejek ´es az ´ır´ asok sorrendj´et. Ezut´ an megvizsg´ aljuk a k¨ ozvetlen¨ ul egym´ as ut´ an k¨ ovetkez˝ o dob´ asp´ arokat. Azt a ´llap´ıtottuk meg, hogy a sorozatban pontosan k´et FF, h´ arom FI, n´egy IF ´es ¨ ot II dob´ asp´ ar fordult el˝ o. H´ any k¨ ul¨ onb¨ oz˝ o sorrend szerint alakulhatott ki a felt´eteleknek megfelel˝ o dob´ assorozat? (A) 120; (B) 560; (C) 1568; (D) 5005; (E) 2 522 520.

A feladatsort Fony´ on´e N´emeth Ildik´ o ´es Fony´ o Lajos ´ all´ıtotta o ¨ssze

A ko ¨z´episkolai tan´arok verseny´enek eredm´enye 1. 2. 3. 4. 5. 6. 7.

Moln´ar Istv´an (B´ek´escsabai Andr´ assy Gyula Gimn.) Fridrik Rich´ard (H´ odmez˝ ov´ as´ arhely, E¨ otv¨ os J´ ozsef Technikum) Kall´ os B´ela (Ny´ıregyh´ aza, Szent Imre Katolikus Gimn.) ´ (Hajd´ Balla Eva uszoboszl´ o, H˝ ogyes Endre Gimn.) Kasztl Roz´alia (Fony´ od, M´ aty´ as Kir´ aly Gimn.) Pituk Andrea M´aria (M´ at´eszalkai Esze Tam´ as Gimn.) K´apl´ar Veronika (Marcali Berzsenyi D´ aniel Gimn.) ´es V´ertes Judit (Budapest, K¨ olcsey Ferenc Gimn.) 9. Bakos Enik˝ o (Budapest, ELTE Ap´ aczai Csere J´ anos Gyak. Gimn.) ´es Baloghn´e Cseh Judit (Szolnok, Varga Katalin Gimn.).

M´asodik

351,00

352,00

Harmadik

350,00

352,50

Illet´ek nincs a tranzakci´ o¨ osszeg´enek 0,3%-a, de maximum 1500 Ft 400 Ft

A v´eteli ´ arfolyam adja meg, hogy a valutav´ alt´ o h´ any Ft-´ert vesz meg az u ¨gyf´elt˝ ol 1 eur´ot. Az elad´ asi ´ arfolyam adja meg, hogy a valutav´ alt´ o h´ any Ft-´ert ad el az u ot. V´eg¨ ul az illet´ek adja meg, hogy minden egyes p´enzv´ alt´ asi ¨gyf´elnek 1 eur´ tranzakci´ o ut´an mekkora d´ıjat kell pluszban kifizetni. a) Ann´anak 250 eur´ora volt sz¨ uks´ege. Mennyit kellene ez´ert fizetnie az egyes p´enzv´ alt´ okn´ al? (3 pont) b) Bal´ azs 600 000 Ft-´ert vett eur´ ot az Els˝o P´enzv´ alt´ on´al. K´es˝ obb kider¨ ult, hogy nem lesz r´ a sz¨ uks´ege, ez´ert visszav´ altotta a p´enzt forintra a M´ asodik P´enzv´ alt´ on´ al. H´any forint vesztes´ege keletkezett? (4 pont) c) Hat´arozzuk meg, h´ any eur´o v´as´ arl´asa eset´en lesz a Harmadik P´enzv´ alt´ o´e alt´ asi aj´ anlat. (7 pont) a legkedvez˝ obb a´tv´

c) H´any olyan n´egyjegy˝ u pozit´ıv eg´esz sz´ am van, amely a 7 ´es a 11 k¨ oz¨ ul legal´ abb az egyikkel oszthat´ o? (4 pont) 3. a) Egy sz´ amtani sorozat els˝ o 10 tagj´ anak ¨ osszege megegyezik az ezt k¨ ovet˝o 5 tag ¨osszeg´evel. A sorozat 19-edik tagja a 777. Hat´ arozzuk meg a sorozat els˝ o tagj´ at ´es differenci´aj´at. (7 pont)

´ ´ Isk.) Mor´ on´e P´alos Zsuzsanna (Budapest, Ujbudai Teleki Blanka Alt. ´ Isk.) T´ oth Gabriella (Csantav´er, Hunyadi J´ anos Alt. Egyed L´aszl´o (Bajai III. B´ela Gimn.), ´ Tagisk.) ´es Mikl´os Ildik´o (V´ amosmikolai Alt. ´ Isk. ´es Gimn.) Par´ oczay Eszter (G¨ od¨ oll˝ oi Premontrei Szent Norbert Alt. 6. Borb´elyn´e Rostah´azi Krisztina (Sz´ekesfeh´erv´ ar, Ciszterci Szent Istv´ an Gimn).

1. 2. 3. 4.

b) Egy m´ertani sorozat els˝ o 2 tagj´ anak ¨ osszege hatszorosa a sorozat harmadik tagj´ anak. A sorozat 4-edik tagja az 1. Hat´ arozzuk meg a sorozat els˝ o tagj´ at ´es h´ anyados´ at. (6 pont) 4. a) Igaz-e a k¨ ovetkez˝ o´ all´ıt´ as? 2 Ha x = 3, akkor f (x) = 2x − 10x + 14 ´ert´eke pozit´ıv pr´ımsz´ ammal egyenl˝ o.

Fogalmazzuk meg az ´all´ıt´ as megford´ıt´ as´ at. Igaz-e az a´ll´ıt´ as megford´ıt´ asa? A v´alaszt indokoljuk. (5 pont)

Az ´ altal´ anos iskolai tan´ arok verseny´enek feladatait nem k¨ oz¨ olj¨ uk.

K¨ oz´ episkolai Matematikai ´ es Fizikai Lapok, 2022/1

Elad´as 352,90

2. a) Melyik az a legkisebb olyan 77-tel oszthat´ o n´egyjegy˝ u pozit´ıv eg´esz sz´ am, amelyik pontosan h´arom k¨ ul¨ onb¨ oz˝ o sz´ amjegyet tartalmaz? (4 pont) b) H´any olyan n´egyjegy˝ u pozit´ıv eg´esz sz´ am van, amelyik pontosan h´arom k¨ ul¨onb¨oz˝ o sz´ amjegyet tartalmaz? (4 pont)

Az ´altal´anos iskolai tan´arok verseny´enek∗ eredm´enye



Els˝ o

V´etel 348,50

17

18

K¨ oz´ episkolai Matematikai ´ es Fizikai Lapok, 2022/1

b) Oldjuk meg az al´abbi egyenletet a val´ os sz´amok halmaz´an: |2 sin2 x + 3 sin x − 1| = 1.

(7 pont)

II. r´esz 5. Nagyi a 31,5 cm × 30 cm (bels˝ o) m´eret˝ u tepsij´eben s¨ ut¨ott s¨ utem´enyt az unok´ainak. A s¨ utem´eny 4 cm magas lett. Nagyi a s¨ utem´eny n´egy oldal´ at ´es a tetej´et be szeretn´e vonni csokikr´emmel. a) H´any dkg csokikr´emre lesz ehhez sz¨ uks´ege, ha 1 dm2 fel¨ ulet bevon´ as´ ahoz 2 dkg csokikr´em elegend˝ o? A v´ alaszt eg´eszre kerek´ıtve adjuk meg. (3 pont) Az unok´ai k¨ oz¨ ul ugyanannyian szeretik a s¨ utem´eny sz´el´et”, mint a k¨ozep´et”. ” ” Ez´ert Nagyi szeretne a s¨ utem´eny sz´el´eb˝ ol mind a n´egy oldalon egy azonos sz´eless´eg˝ u cs´ıkot lev´agni u ´gy, hogy a lev´agott r´eszek alapter¨ ulete ´es a s¨ utem´eny k¨ozep´enek alapter¨ ulete egyenl˝ o legyen. b) Hat´ arozzuk meg a lev´ agand´ o cs´ık sz´eless´eg´et. (7 pont) Nagyi minden unok´ aj´anak ugyanannyi szeletet szeretne adni a s¨ utem´enyb˝ ol. Ha 10 · 5 szeletre v´ agn´ a a s¨ utem´enyt, akkor az oszt´as ut´an 2 szelet megmaradna. Ha 9 · 5 szeletre v´ agn´ a, akkor 3 szelet, ha pedig 10 · 4 szeletre v´ agn´a, akkor 4 szelet maradna meg az oszt´as ut´an. c) H´ any unok´ aja van Nagyinak? (6 pont) 6. Egy szab´ alyos 10-sz¨ og alak´ u asztal egy oldal´anak hossza 50 cm. Erre az asztalra egy olyan k¨ or alak´ u ter´ıt˝ ot k´esz´ıtenek, amely sehol nem l´ og le az asztalr´ ol. a) Hat´ arozzuk meg a legnagyobb ilyen ter´ıt˝o ter¨ ulet´et. (3 pont)

Egy szab´ alyos dob´ okock´ aval h´ aromszor egym´as ut´an dobunk. b) Hat´ arozzuk meg annak a val´ osz´ın˝ us´eg´et, hogy valamelyik dobott sz´am a m´asik k´et dobott sz´ amnak sz´ amtani vagy m´ertani k¨ ozepe lesz. (6 pont) c) Hat´arozzuk meg annak a val´ osz´ın˝ us´eg´et, hogy a dobott sz´ amok k¨ oz¨ ott van 6-os, felt´eve, hogy valamelyik dobott sz´ am a m´ asik k´et dobott sz´amnak a sz´ amtani vagy m´ertani k¨ ozepe. (3 pont) u g¨ orbe ´es az x-tengely ´altal hat´arolt z´ art 8. a) Az y = 83 x − 49 x2 egyenlet˝ 4 tartom´ anyt k´et r´eszre osztja az y = 3 x egyenlet˝ u egyenes. Hat´arozzuk meg a k´et r´esz ter¨ ulet´enek ar´ any´ at. (8 pont) b) Egy h´aromsz¨ og cs´ ucsai a koordin´ ata-rendszerben A(0; 0), B(3; 0) ´es C(3; 4). A h´ aromsz¨oget megforgatjuk a leghosszabb oldala k¨ or¨ ul. Hat´ arozzuk meg az ´ıgy kapott forg´ astest felsz´ın´et ´es t´erfogat´ at. (8 pont) 9. Egy ´ep´ıt˝ oipari v´ allalkoz´ onak a legut´obbi ´ep´ıtkez´es ut´ an megmaradt 200 kg cementje, ´es u ´gy d¨ ont¨ ott, hogy egyenl˝o t¨ omeg˝ u r´eszekre osztva ´ert´ekes´ıti. A kereskedelemben szok´ asos m´odon nagyobb kiszerel´es˝ u csomag eset´en alacsonyabb a cement kilogrammonk´enti ´ ara (egys´eg´ ara): ha egy csomag cement t¨ omege m peng˝ o s egys´ e g´ a ron k´ ın´ a lja elad´ a sra. A cement becsomagom kg, akkor (40 − 10 ) m l´as´anak is van k¨ olts´ege, m´egpedig m kg-os csomag eset´en (25 + 10 ) peng˝o csomagonk´ent.

a) Hat´ arozzuk meg, hogy mekkora lesz a v´ allalkoz´ onak az elad´as´ ab´ ol (a csomagol´as k¨olts´eg´enek levon´ asa ut´ an) sz´ armaz´ o bev´etele, ha a cementet 10 egyenl˝o t¨omeg˝ u r´eszre osztva ´ert´ekes´ıti. (5 pont) b) Hat´ arozzuk meg, h´ any egyenl˝o t¨ omeg˝ u r´eszre kell osztani a cementet ahhoz, hogy – azt a tervek szerint ´ert´ekes´ıtve – az elad´ asb´ ol sz´armaz´ o (a csomagol´ asi k¨olts´egek levon´ asa ut´ ani) bev´etel maxim´alis legyen. (11 pont)

b) Legfeljebb h´ any sz´ azal´ek´ at tudja lefedni ez a ter´ıt˝ o az asztal ter¨ ulet´enek? (3 pont)

Koncz Levente Budapest

Jel¨ olje F1 az A1 A2 ´es F2 az A3 A4 szakaszok felez˝ opontj´at. Az asztallapot ak. Jel¨olje M a k´et egyenes az A8 F1 ´es az A10 F2 egyenesekkel n´egy r´eszre osztj´ metsz´espontj´ at. og ´es az F2 A3 A2 F1 M ¨otsz¨og ter¨ ulete c) Igazoljuk, hogy az A10 A9 A8 M n´egysz¨ egyenl˝ o. (4 pont) Egy szab´ alyos 10-sz¨ og cs´ ucsai k¨ oz¨ ul v´eletlenszer˝ uen kiv´ alasztunk h´ armat, ´ıgy egy h´ aromsz¨ og cs´ ucsait kapjuk. d) Mennyi annak a val´ osz´ın˝ us´ege, hogy a h´ aromsz¨og tompasz¨og˝ u?

(6 pont)

Megold´asv´azlatok a 2021/12. sz´am emelt szint˝ u matematika gyakorl´o feladatsor´ahoz I. r´esz

7. Az egyetemen 220 di´ak ´ırt meg egy dolgozatot, az a´tlag sz´ azadokra kerek´ıtve 3,82 lett. (Csak az 1, 2, 3, 4, 5 eg´esz ´ert´ek˝ u oszt´ alyzatok lehettek az eredm´enyek.)

1. Fel´ırjuk az 1; 2; 3; 4; 5 sz´ amjegyek sorbarendez´es´evel k´epezhet˝ o o ot¨sszes ¨ jegy˝ u sz´ amot. a) Mennyi ezeknek az ¨ otjegy˝ u sz´ amoknak az ¨ osszege? (6 pont)

a) Legal´ abb ´es legfeljebb h´ any 5-¨ os dolgozat sz¨ uletett, ha nem volt 1-es? (7 pont)

b) H´ any olyan sz´ amtani sorozat l´etezik, melynek els˝ o tagja 12 345, szerepel benne az 54 321 is, ´es a differenci´ aja pozit´ıv eg´esz sz´ am? (6 pont)

K¨ oz´ episkolai Matematikai ´ es Fizikai Lapok, 2022/1

19

20

K¨ oz´ episkolai Matematikai ´ es Fizikai Lapok, 2022/1

Megold´as. a) Egy kiv´alasztott sz´ amjegy az egyesek hely´en 4! = 24-szer fordul el˝o, mert a t¨ obbi sz´ amjegyet ennyif´ele sorrendben ´ırhatjuk mell´e. Az egyesek hely´en all´ ´ o sz´ amok ¨ osszege ez´ert 24 · (1 + 2 + 3 + 4 + 5) = 360.

Ugyanez ´erv´enyes a t¨ obbi helyi´ert´ekre. Az ¨ osszeg teh´ at 11 111 · 360 = 3 999 960. b) 12 345 + (n − 1)d = 54 321 ⇒ (n − 1)d = 41 976 = 23 · 32 · 11 · 53. Oszt´oinak sz´ ama 4 · 3 · 2 · 2 = 48, a differencia s vele a sorozat teh´at 48-f´ele lehet. (Mivel minden d-hez van megfelel˝ o n ´es ´ıgy megfelel˝o sorozat is.) 2. Egy sorsjegy ´ ara 1000 Ft. A sorsjegy lehets´eges nyerem´enyei: 2 000 Ft, 5 000 Ft, 20 000 Ft, 100 000 Ft, 500 000 Ft. Ezek val´ osz´ın˝ us´ege rendre: 11%, 5%, 0,81%, 0,17%, 0,02%. a) Mennyi a nyerem´eny v´ arhat´ o ´ert´eke? (3 pont) b) Mekkora a val´ osz´ın˝ us´ege, hogy nem nyer¨ unk, ha egy sorsjegyet v´ as´ arolunk? (2 pont) T´ız alkalommal vesz¨ unk egy-egy sorsjegyet. Mekkora a val´ osz´ın˝ us´ege, hogy c) legal´ abb k´etszer nyer¨ unk; (5 pont) d) pontosan h´ aromszor nyer¨ unk?

(3 pont)

Megold´as. a) 0,11 · 2000 + 0,05 · 5000 + 0,0081 · 20 000 + 0,0017 · 100 000 + 0,0002 · 500 000 = 902. A nyerem´eny v´ arhat´ o ´ert´eke 902 Ft. b) 1 − (0,11 + 0,05 + 0,0081 + 0,0017 + 0,0002) = 0,83. c) Vonjuk ki 1-b˝ol annak a val´osz´ın˝ us´eg´et, hogy egyszer sem nyer¨ unk, illetve pontosan 1-szer nyer¨ unk: 1 − 0,8310 − 10 · 0,839 · 0,17 = 0,527. d)

10 3

· 0,837 · 0,173 = 0,16.

A sarkon kimarad´ o ´ıves h´ aromsz¨ og” ter¨ ulet´et megkapjuk, ha a 60◦ k¨ oz´epponti ” 2 sz¨og˝ u, 3 cm sugar´ u k¨ orcikk ter¨ ulet´et (3 π/6 = 4,71 cm2 ) kivonjuk az ABCD n´egyuttes ter¨ ulete sz¨og ter¨ ulet´eb˝ ol (5,196 cm2 ). A hat kis sarokr´esz egy¨ 6 · (5,196 − 4,71) = 2,92 cm2 . A befestett ter¨ ulet 259,8 − 24,51 − 2,92 = 232,37 cm2 = 23 237 mm2 . y2

4. a) Adjuk meg az x = 8 + 3 egyenlet˝ u parabol´ ahoz a P (0; −1) pontb´ ol h´ uzhat´ o ´erint˝ ok egyenlet´et. (8 pont) b) Hat´ arozzuk meg azokat a val´ os x ´ert´ekeket, melyekben az f (x) = sin2 x + + cos x f¨ uggv´eny grafikonj´ anak ´erint˝ oje p´ arhuzamos az x-tengellyel. (6 pont) Megold´as. a) Az ´erint˝o egyenlete y = mx − 1. Az ´erint˝o ´es a parabola egyenlet´eb˝ ol ´all´o egyenletrendszernek pontosan egy megold´ ast kell adnia. y-t behelyetteu s´ıtve a parabola egyenlet´ebe az m2 x2 − (2m + 8)x + 25 = 0 param´eteres m´asodfok´ egyenlethez jutunk. Ennek akkor van pontosan egy gy¨ oke, ha (1) m = 0. Ez nem megold´ as, mert a parabola tengely´evel p´ arhuzamos egyenest jelent, ami nem ´erint˝o. (2) az egyenlet diszkrimin´ansa 0, vagyis −96m2 + 32m + 64 = 0. Ezt megoldva m = 1, illetve m = −2/3 ad´ odik. Az ´erint˝ok egyenlete: y = x − 1, illetve y = −2/3x − 1. b) Azon x ´ert´ekeket keress¨ uk, melyekre a deriv´ alt ´ert´eke 0:

3. Egy 10 cm oldal´ u, szab´ alyos hatsz¨ og alak´ u feh´er t´ alca pereme mellett v´egigg¨ orget¨ unk egy 6 cm atm´er˝ ´ oj˝ u, alul fest´ekes korongot. Mekkora az ilyen m´ odon besz´ınezett ter¨ ulet? A v´ alaszt mm2 pontoss´ aggal adjuk meg. (12 pont)

f  (x) = 2 sin x cos x − sin x = sin x(2 cos x − 1) = 0. sin x = 0, ebb˝ ol x = k · π, vagy cos x = 1/2, ebb˝ ol x = ±π/2 + 2lπ (k, l ∈ Z). II. r´esz

Megold´as. A t´alca ter¨ ulet´eb˝ ol kivonjuk a feh´eren marad´o bels˝o kis hatsz¨og ´es a sarokr´eszek ter¨ ulet´et. A nagy hatsz¨ og√ k¨ oz´epponti h´ aromsz¨og´enek magass´ aga √ 10·5 3 2 5 3 ≈ 8,66 cm. A hatsz¨ og ter¨ ulete 6 · 2 = 259,8 cm . A kis hatsz¨ og k¨ oz´epponti h´ aromsz¨ og´enek magass´ aga 8,66 − 6 = 2,66 cm. A hatsz¨ og ter¨ ulete: 2 2, 66 · 259,8 = 24,51 cm2 . 8, 66 K¨ oz´ episkolai Matematikai ´ es Fizikai Lapok, 2022/1

21

5. A pitagoreusok azokat a term´eszetes sz´ amokat nevezt´ek h´ aromsz¨ ogsz´ amnak, amely sz´ am´ u kavicsot az ´ abr´ an l´ athat´ o m´ odon h´ aromsz¨ og alakba lehet rendezni.

22

K¨ oz´ episkolai Matematikai ´ es Fizikai Lapok, 2022/1

Az els˝ o hat h´ aromsz¨ ogsz´ am: 1, 3, 6, 10, 15, 21. a) Sz´ am´ıtsuk ki a kilencedik ´es a sz´ azadik h´ aromsz¨ ogsz´ amot.

(2 pont)

b) Bizony´ıtsuk be, hogy az els˝ o n h´ aromsz¨ ogsz´ am n(n + 1)(n + 2) . (6 pont) osszege ¨ 6 c) A goly´ os piramis nev˝ u t´erbeli logikai j´ at´ek elemeib˝ ol ezt a tetra´ederszer˝ u ´ep´ıtm´enyt kell ¨ ossze´ all´ıtani. Milyen magas az ´ep´ıtm´eny, ha a goly´ ok ´ atm´er˝ oje 2 cm? (A megold´ ast cm-ben egy tizedes jegy pontoss´ aggal adjuk meg.) (8 pont)

Sz´ am´ıtsuk ki a n´egy oz´eppont a´ltal meghat´ arozott tetra´eder magass´ at. √ ag´ √ k¨ 3 cm. Az F P D der´ e ksz¨ o g˝ u h´ a romsz¨ o g a ´ tfog´ o ja 3 cm, Az oldallap magass´ a ga √ √ asik befog´oja M . Ebb˝ ol M = 24/3 cm. Az eg´esz egyik befog´oja 3/3 cm, m´ ´ep´ıtm´eny magass´ aga 3M + 2 cm ≈ 6,9 cm. 6. A hanger˝ ot a hanghull´ amok intenzit´ asa hat´ arozza meg, amelynek m´ert´ekegyW s´ege m . Az egyenl˝ o nek ´ e rz´ e kelt hanger˝ o -k¨ u l¨ o nbs´ e gek egyenl˝ o intenzit´ as-ar´ anyokat 2 takarnak. A hanger˝ o m´ert´ekegys´ege a decibel. W Az emberi f¨ ul ingerk¨ usz¨ obe az I0 = 10−12 m uk 0 decibelnek. B´ ar2 . Ezt nevezz¨

mely m´ as I intenzit´ as´ u hang hangerej´et a H = 10 · lg II dB k´eplet adja meg. 0

−9 W

intenzit´ as´ u halk besz´ed? a) H´ any dB a 3 · 10 m2 b) Mekkora a mennyd¨ org´es intenzit´ asa, ha a hangereje 125 dB? Megold´as. a) Az n-edik h´ aromsz¨ ogsz´ am 1 + 2 + . . . + n = teh´at 9·10 = 45, a sz´ azadik pedig 100·101 = 5050. 2 2

n(n+1) . 2

n = 1-re az a´ll´ıt´ as igaz, mert 1·2 = 1·2·3 . 2 6 Tegy¨ uk fel, hogy n = k-ra igaz az ¨ osszef¨ ugg´es. Megmutatjuk, hogy ekkor n = = k + 1-re is igaz: k(k + 1) (k + 1)(k + 2) 1·2 2·3 3·4 + + + ... + + = 2 2 2 2 2

Az ´erz´ekelt hangmagass´ ag a hang rezg´essz´ am´ aval a ´ll ¨ osszef¨ ugg´esben. Az egyenl˝ onek hallott hangk¨ oz¨ ok egyenl˝ o rezg´essz´ am-ar´ anyokat takarnak. Pl. ha egy hangot egy m´ asikn´ al egy okt´ avval magasabbnak ´erz´ekel¨ unk, akkor a rezg´essz´ ama az el˝ obbi´enek 2-szerese. A rezg´essz´ am a hangmagass´ ag f¨ uggv´eny´eben teh´ at exponenci´ alisan n˝ o. A kromatikus sk´ ala az okt´ avot 12 egyenl˝ o hangk¨ ozre, u ´n. f´elhangokra osztja. Ha a sikn´ a l f´ e lhanggal magasabb (pl. C ´ e s Cisz), akkor a rezg´essz´ ama egy hang egy m´ √ 12 2-sz¨ or¨ ose az el˝ obbi´enek. d) H´ anyszorosa a nagyterc hangk¨ ozben (n´egy f´elhang) a magasabb hang rezg´essz´ ama a m´elyebb´enek? (Pontos ar´ anysz´ amot adjon meg.) (2 pont) e) Adjunk k´epletet, amellyel egy tiszta zenei hangk¨ ozr˝ ol a rezg´essz´ amok x ar´ anya ismeret´eben kisz´ am´ıthatjuk, hogy h´ any f´elhangnyi t´ avols´ agot jelent. (4 pont)

k(k + 1)(k + 2) 3(k + 1)(k + 2) k(k + 1)(k + 2) (k + 1)(k + 2) + = + = = 6 2 6 6 =

c) Az intenzit´ ast 5-sz¨ or¨ os´ere n¨ ovelve h´ any dB hanger˝ o-emelked´est ´er¨ unk el? (3 pont)

A kilencedik

b) Teljes indukci´ oval bizony´ıtunk.

(3 pont) (4 pont)

(k + 1)(k + 2)(k + 3) . 6

Megold´as.

c) Egy n´egy goly´ ob´ol ´ all´ o kis g´ ula” k¨ oz´eppontjai 2 cm ´el˝ u szab´ alyos tetra´edert ” alkotnak. Ha ennek magass´ aga M , akkor az eg´esz ´ep´ıtm´eny magass´ aga 3M + k´etszer a g¨ omb¨ ok sugara.

a) b)

H = 10 · lg

3 · 10−9 dB = 10 · lg 300 dB = 34,8 dB. 10−12

125 = 10 · lg

I 10−12 ,

12,5 = lg(1012 I), 1012,5 = 1012 I, √ W W I = 10 2 = 3,16 2 . m m c)

d) e) K¨ oz´ episkolai Matematikai ´ es Fizikai Lapok, 2022/1

23

24

10 lg

5I1 I0

− 10 lg

I1 I0

= 10 lg(5I1 ) − 10 lg I0 − (10 lg I1 − 10 lg I0 ) =

= 10 lg(5I1 ) − 10 lg I1 = 10 lg 5 + 10 lg I1 − 10 lg I1 = 10 lg 5 ≈ 7 dB. √ 4 √  12 3 2 = 2. √ n  12 2 = x, x12 = 2n , n = log2 x12 = 12 · log2 x. K¨ oz´ episkolai Matematikai ´ es Fizikai Lapok, 2022/1

7. a) A ferde h´ aztet˝ on egy k´em´eny ´ arny´eka ´epp a tet˝ o lejt´es´enek ir´ any´ aba esik. Mekkora a tet˝ o d˝ ol´essz¨ oge, ha a k´em´eny 1 m magas, a ´rny´eka 86 cm, ´es ugyanekkor a kertben n¨ ov˝ o 120 cm-es napraforg´ o´ arny´eka 75 cm? (8 pont) b) A telken a h´ az mellett szeretn´enk elu kisker´ıteni egy 450 m2 -es, t´eglalap alak´ kertet. Mekkor´ ak legyenek a kiskert oldalai, hogy a legr¨ ovidebb ker´ıt´est kelljen ´ep´ıteni? (Ahol fal van, nem kell ker´ıt´es. A kert m´elys´ege legal´ abb akkora legyen, mint a h´ az´e.) (8 pont)

2

2

Megold´as. a) A k´et egyenl˝ otlens´eget rendezve (x − 3) + (y − 3)  25, illetve 2 2 orgy˝ ur˝ u, melyet a (3; 3) (x − 3) + (y − 3)  16 ad´odik. A vizsg´alt alakzat egy k¨ k¨oz´eppont´ u, 4, illetve 5 egys´eg sugar´ u k¨ or¨ ok hat´ arolnak. A k´et k¨ or ter¨ ulet´enek k¨ ul¨onbs´ege 25π − 16π = 9π. 2 asa 1  x  5. A k´et f¨ uggv´enyt ezen az intervallumon b) (x − 3) + 1  5 megold´ integr´ aljuk, a ter¨ ulet a k´et integr´ al k¨ ul¨ onbs´ege. 5

5

5 d(x) = [5x]1 = 20,

1

Megold´as. a) A napraforg´o ´arny´ek´ anak hossz´ ab´ ol kisz´am´ıtva a napsugarak = 58◦ -os sz¨ ogben ´erik a talajt. arctan 120 75 A P AC der´eksz¨ og˝ u h´ aromsz¨ ogben kisz´am´ıtjuk az AC hossz´ us´agot:

5 1

=

75 · 1 m = 0,625 m. AC = 120 T = 20 −

Az ABC h´ aromsz¨ ogre a szinuszt´etelt alkalmazva: 0,86 sin 122◦ = , 0,625 sin β

A nevez˝ o pozit´ıv. A t¨ ort el˝ojele a sz´aml´ al´ot´ ol f¨ ugg. Az ´ertelmez´esi tartom´anyon bel¨ ul x = 15-re 0 a deriv´ alt, kisebb x-ekre negat´ıv, nagyobbakra pozit´ıv. A K(x) f¨ uggv´eny 15-ig szigor´ uan monoton cs¨ okken, ut´ ana szigor´ uan monoton n˝ o, ´ıgy a minimumhely x = 15. A kiskert oldalai 15 m ´es 30 m hossz´ uak.

2

K¨ oz´ episkolai Matematikai ´ es Fizikai Lapok, 2022/1

1

=

28 , 3

28 32 = . 3 3

at. b) Sz´ am´ıtsuk ki az A1 B1 , A2 B2 ´es A3 B3 szakasz hossz´ (5 pont) c) Az elj´ ar´ ast a v´egtelens´egig folytatva keletkezik a vonalk´ az´ assal jel¨ olt h´ aromsz¨ ogek v´egtelen sorozata. Sz´ am´ıtsuk ki a h´ aromsz¨ ogek ter¨ ulet´enek ¨ osszeg´et. (6 pont)

8. Sz´ am´ıtsuk ki a der´eksz¨ og˝ u koordin´ atarendszerben az egyenl˝ otlens´egekkel megadott k´et ponthalmaz pontos ter¨ ulet´et: b) (x − 3) + 1  y  5.

5

Az ABC der´eksz¨ og˝ u h´ aromsz¨ og AB befog´ oja 10 egys´eg, BC befog´ oja 20 egys´eg hossz´ us´ ag´ u. ucsb´ ol az AC oldalra a ´ll´ıtott mer˝ oleges, B1 A1 a B cs´ az A1 -b˝ ol BC-re ´ all´ıtott mer˝ oleges talppontja. Ugyan´ıgy A2 a B1 -b˝ ol AC-re ´ all´ıtott mer˝ oleges, B2 az A2 -b˝ ol BC-re ´ all´ıtott mer˝ oleges talppontja, ´es ´ıgy tov´ abb.

450 2x2 − 450 2(x − 15)(x + 15) = = . 2 2 x x x2

a) 6(x + y) − 2  x2 + y 2  6(x + y) + 7.

x3 − 3x2 + 10x 3

9. a) A h´ aromsz¨ og oldalain 5–5–5 pontot jel¨ olt¨ unk ki. H´ any h´ aromsz¨ oget hat´ aroz meg a tizen¨ ot pont?

amib˝ ol sin β = 0,6163 ´es ´ıgy β = 38,05◦ (mivel csak hegyessz¨ og lehet). Ebb˝ ol ϕ = 19,95◦ . A tet˝ o a v´ızszinteshez 19,95◦ -ban hajlik. b) A k˝ ofalra mer˝ oleges oldalt x-szel jel¨ olve a ker´ıt´es hossza K(x) = 2x − 10 + + 450/x, x  10. K  (x) = 2 −



(x2 − 6x + 10) d(x) =

(7 pont) (9 pont) 25

Megold´as. a) Egy oldalon lev˝o h´ arom pont nem alkot h´ aromsz¨ oget. A h´aromsz¨ og cs´ ucsai lehetnek h´ arom k¨ ul¨ onb¨ oz˝ o oldalon, az ilyen esetek sz´ ama 5 · 5 · 5 = 125. 26

K¨ oz´ episkolai Matematikai ´ es Fizikai Lapok, 2022/1

Ha k´et cs´ ucs egy adott oldalra esik, a harmadik cs´ ucs egy m´asikra, akkor · 5 = 50. A k´ e t oldalt 3 · 2 = 6-f´ e lek´eppen v´ alaszthatjuk a lehet˝ os´egek sz´ ama ( 5·4 ) 2 ki, az ilyen h´aromsz¨ ogek sz´ ama teh´ at 6 · 50 = 300.

I. megold´as. Legyen az ABC h´ aromsz¨ og be´ırt k¨ or´enek k¨ oz´eppontja O, sugara (OE =)r ´es CAB = 2α, ekkor OAB = α, mert az AO egyenes sz¨ ogfelez˝ o (1. ´ abra). A CBA = 4α a feladat felt´etele miatt, ´es ACB = 180◦ − 6α, F CB = 90◦ − 3α, mert a CO egyenes sz¨ ogfelez˝ o.

A tizen¨ ot pont ¨ osszesen 425 h´aromsz¨ oget hat´ aroz meg. √ b) A nagy h´ aromsz¨ og a´tfog´ oja 10 5 . Az a´br´ an a der´eksz¨og˝ u h´ aromsz¨ogek mind hasonl´ ok, mert sz¨ ogeik egyenl˝ok. Az oldalar´ anyok egyenl˝ os´eg´et t¨obb l´ep´esben egym´ as ut´an alkalmazva: A1 B 20 2 A1 B 1 = = √ =√ . A1 B 10 10 5 5

Mivel ABC = 2CAB, ez´ert AC > BC, ´ıgy C az OE egyenes B pont fel˝ oli oldal´an van, emiatt F az AE szakaszon van. Az F BC h´aromsz¨ ogben a sz¨ ogek ¨ osszege 180◦ , ez´ert OF E = ◦ aromsz¨ og der´eksz¨ og˝ u, ez´ert EOB = = 90 − α ´es ´ıgy F OE = α. Az OEB h´ = 90◦ − 2α, amelyhez α-t adva megkapjuk, hogy F OB = 90◦ − α, ´ıgy az F BO h´aromsz¨og egyenl˝o sz´ ar´ u, teh´ at BO = BF . 1. a ´bra

20 Ebb˝ ol el˝obb A1 B = √ , majd A1 B1 = 400 = 8. Hasonl´ oan: 50 5

A2 B2 2 A2 B1 A2 B1 =√ , = = A2 B1 A1 B1 8 5 2 16 , ´es ´ıgy A2 B2 = 16 = 6,4. V´eg¨ ul amib˝ ol A2 B1 = √ 5·8 5

Az AOE h´ aromsz¨ og hasonl´o az OF E h´ aromsz¨ ogh¨ oz, hiszen der´eksz¨ og˝ uek ´es van α nagys´ ag´ u bels˝ o sz¨ og¨ uk, ez´ert a megfelel˝ o oldalhosszak ar´anya egyenl˝ o, ´ıgy fel´ırhat´ o a k¨ovetkez˝ o egyenl˝ os´eg:

A3 B3 2 Ab B2 A3 B2 =√ , = = A3 B2 A2 B2 6,4 5 amib˝ ol

12,8 √ , 5

163,84 A3 B2 = ´es ´ıgy A3 B3 = 5 aromsz¨ og ter¨ ulete 8·4 Az A1 BB1 h´ 2

r EF = , r AE

: 6,4 = 5,12 egys´eg.

= 16 egys´eg. A ter¨ uletek m´ertani soroc) zatot alkotnak, melynek h´ anyadosa a h´ aromsz¨ ogek hasonl´ os´ag´anak n´egyzete: 2 2 2 A2 B2 6,4 4 16 . q= = = = A1 B1 8 5 25 A m´ertani sor o aromsz¨ogek egy¨ uttes ter¨ ulete ¨sszegk´eplet´ebe behelyettes´ıtve a h´ 16 ·

1 1−

16 25

= 16 ·

ami ekvivalens az EF · AE = r2 egyenlettel. Mivel AE = AF + F E, ´ıgy EF · (AF + F E) = r2 . Alkalmazzuk Pitagorasz t´etel´et az OEB h´ aromsz¨ ogre: r2 + BE 2 = BO2 . Ekkor 2 2 2 oz˝ oekben kapott kifejez´esben r2 BO = BF miatt r + BE = BF , majd az el˝ hely´ere helyettes´ıtve a k¨ ovetkez˝ ot kapjuk:

400 25 = = 44,4 ter¨ uletegys´eg. 9 9

EF · (AF + F E) + BE 2 = BF 2 . De´ak Anna Budapest

Felhaszn´ aljuk, hogy BF = BE + EF , ´ıgy az EF · (AF + F E) + BE 2 = (BE + EF )

2

egyenlethez jutunk. Felbontjuk a z´ ar´ ojeleket: EF 2 + EF · AF + BE 2 = BE 2 + 2 · EF · BE + EF 2 ,

Matematika feladat megold´asa

majd ekvivalens ´ atalak´ıt´ asok ut´an az B. 5194. Az ABC h´ aromsz¨ ogben ABC = 2CAB. Az AB oldal a be´ırt k¨ ort az E pontban ´erinti, a C-b˝ ol indul´ o sz¨ ogfelez˝ ot az F pontban metszi. Igazoljuk, hogy AF = 2BE.

AF = 2 · BE egyenletet kapjuk, ami ´eppen a bizony´ıtand´ o ´all´ıt´ as. Koltai Csaba Ferenc (Budapest XIV. ker¨ uleti Szent Istv´an Gimn., 9. ´evf.) dolgozata alapj´an

(4 pont) K¨ oz´ episkolai Matematikai ´ es Fizikai Lapok, 2022/1

27

28

K¨ oz´ episkolai Matematikai ´ es Fizikai Lapok, 2022/1

II. megold´as. Legyen a be´ırt k¨ or k¨ oz´eppontja O, sugara r, ´es OAB = α (l´ asd az 1. ´ abr´ at). Mivel O a sz¨ ogfelez˝ ok metsz´espontja, ez´ert OAC = = OAB = α. A feladat sz¨ ovege alapj´ an OBA = OBC = 2α,

´ıgy ACO = BCO = 90◦ − 3α.

A z´ ar´ ojel felbont´ asa ´es 2s = a + b + c felhaszn´ al´asa ut´ an kapjuk, hogy bc = 2s − 2b = a + c − b, a+b amib˝ol ekvivalens ´atalak´ıt´ asokkal a

Mivel ABC = 2CAB, ez´ert AC > BC, ´ıgy C az OE egyenes B fel˝oli oldal´an van, emiatt pedig F az AE szakaszon van.

innen F OE = 180◦ − OEF  − OF E = 180◦ − 90◦ − (90◦ − α) = α. Tov´ abb´ a BAC + ABC = 6α < 180◦ , ebb˝ ol k¨ovetkez˝ oen 0◦ < α < 30◦ , teh´at tg α ´es tg 2α is ´ertelmezve van, valamint tg α = 0, tg 2α = 0. Nyilv´ anval´oan tg α = 1, ez´ert 1 − tg2 α = 0.

A BEO h´ aromsz¨ ogben BE = tgr2α , az AEO h´ aromsz¨ogben AE = tgrα , az F EO h´aromsz¨ ogben pedig F E = r · tg α. Ezeket felhaszn´alva fel´ırjuk a sz´ oban forg´ o szakaszhosszak ar´ any´ at, ekvivalens a´talak´ıt´ asokat v´egz¨ unk, ´es alkalmazzuk a k´etszeres sz¨ og tangens´ere vonatkoz´ o add´ıci´ os t´etelt: r − r · tg α AE − F E tg 2α AF tg α = = − tg 2α · tg α = = r BE BE tg α tg 2α 2 tg α 1−tg2 α

tg α



2 tg α 2 2 tg2 α 2(1 − tg2 α) · tg α = − = = 2. 2 2 2 1 − tg α 1 − tg α 1 − tg α 1 − tg2 α

K¨ ovetkez´esk´eppen AF = 2BE, ezzel a´ll´ıt´ asunkat bebizony´ıtottuk. Fekete Rich´ ard (Debreceni Fazekas Mih´ aly Gimn., 12. ´evf.) dolgozata alapj´an III. megold´as. K´esz´ıts¨ unk a´br´ at, ´es haszn´ aljuk a 2. ´ abra jel¨ol´eseit.

Legyen a P pont a B cs´ ucson a´tmen˝ o bels˝ o sz¨ ogfelez˝ o ´es az AC oldal metsz´espontja. Mivel a BP C ´es az ACB h´ aromsz¨ ogek egyik bels˝ o sz¨ oge k¨ oz¨ os, egy m´asikr´ ol pedig tudjuk, hogy 2α nagys´ ag´ u, hiszen BP sz¨ ogfelez˝ o ´es a felt´etel szerint ABC = 2CAB, ez´ert ezek a h´ aromsz¨ ogek hasonl´ ok, ´ıgy a megfelel˝ o oldalhosszak ar´ any´ ara fel´ırhatjuk a k¨ ovetkez˝ o egyenletet: AC AB CB = = . CB PB PC Ebb˝ ol az is k¨ovetkezik, hogy: AC AB + CB = . CB PB + PC Az ABP h´aromsz¨ og egyenl˝o sz´ ar´ u, mert az A-n´ al ´es B-n´el l´ev˝ o sz¨ ogek egyenl˝ oek. ´Igy AP = P B, vagyis P B + P C = b, ez´ert az el˝obbi kifejez´est a´talak´ıthatjuk az al´abbiak szerint: b c+a = , a b amely a b2 = a2 + ac alakra hozhat´ o, err˝ol pedig az im´ent bel´ attuk, hogy ekvivalens a feladat ´all´ıt´as´ aval, ´ıgy a bizony´ıt´ as v´eg´ere ´ert¨ unk. ´ Szak´ acs Domonkos (Budapest, Jedlik Anyos Gimn., 9. ´evf.) ¨ Osszesen 80 dolgozat ´erkezett. 4 pontos 66, 3 pontos 7, 2 pontos 1 dolgozat. 1 pontot 4 versenyz˝ o kapott. Nem versenyszer˝ u: 1 dolgozat. Nem sz´ am´ıtjuk a versenybe a sz¨ ulet´esi d´ atum vagy a sz¨ ul˝ oi nyilatkozat hi´ anya miatt: 1 dolgozat.

= ab , A sz¨ ogfelez˝ ot´etel szerint c−AF AF c mindk´et oldalhoz 1-et adva: AF = a+b , b

A K pontversenyben kit˝ uzo¨tt gyakorlatok ABACUS-szal k¨oz¨os pontverseny 9. oszt´alyosoknak (714–718.)

bc amib˝ ol AF = a+b . Tudjuk azt is, hogy BE = s − b, ahol s a h´ aromsz¨ og f´elker¨ ulete. Ekkor a bizony´ıtand´ o´ all´ıt´ ast a k¨ ovetkez˝ ok´eppen ´ırhatjuk fel:

bc = 2(s − b). a+b

K¨ oz´ episkolai Matematikai ´ es Fizikai Lapok, 2022/1

majd a bc = a2 − b2 + ac + bc

egyenlethez jutunk. Az egyenlet mindk´et oldal´ ab´ol bc-t kivonunk, majd kifejezz¨ uk b2 -t: b2 = a2 + ac.

CF B = 180◦ − F CB − CBF  = 180◦ − (90◦ − 3α) − 4α = 90◦ − α,

=

bc = (a + b)(a + c − b),

K. 714. Egy sorozat els˝ o tagja 3, ´es a k¨ ovetkez˝ o tagot mindig u ´gy k´epezz¨ uk, hogy az el˝oz˝ o tag k´etszeres´eb˝ ol kivonunk 2-t. a) ´Irjuk fel a sorozat els˝ o 8 tagj´ at.

2. a ´bra

29

30

K¨ oz´ episkolai Matematikai ´ es Fizikai Lapok, 2022/1

b) Az al´ abbi sz´ amok k¨ oz¨ ul melyik sz´am tagja a sorozatnak ´es melyik nem? Ha a sz´ am tagja a sorozatnak, akkor mondjuk meg, h´ anyadik tagja, ha pedig nem, indokoljuk, mi´ert nem. 8194,

649 287 365,

29 453 759 372,

8 398 507 839 348.

K. 715. Van k´et darab k´etliteres kancs´ onk. Az els˝obe 2 liter 100%-os narancslevet o nt¨ u nk, a m´ a sodikba 1 liter vizet. ¨ 1. A narancsl´e fel´et a´t¨ ontj¨ uk a vizeskancs´ oba, annak tartalm´ at egy kan´allal osszekeverj¨ uk, majd visszat¨ olt¨ unk 1 liter folyad´ekot az els˝ o kancs´ oba. ¨ 2. Ezt az 1 literes ´att¨ olt´est kever´essel egy¨ utt megism´etelj¨ uk m´eg egyszer, teh´ at az els˝o kancs´ ob´ol 1 litert kever´es ut´an ´ att¨ olt¨ unk a m´ asodikba, ¨osszekeverj¨ uk a tartalm´ at, majd vissza¨ ont¨ unk 1 litert az els˝ obe. Ezek ut´ an melyik kancs´oban h´ any sz´ azal´ekos az u ¨d´ıt˝o a narancsl´ere n´ezve?

Feladatok mindenkinek C. 1699. Hat´ arozzuk meg, hogy az (x + 1) · (x2 + 1) · (x3 + 1) · . . . · (x12 + 1) any szorzatban szerepl˝o m˝ uveleteket elv´egezve, o as ut´an mennyi az x14 hatv´ ¨sszevon´ egy¨ utthat´ oja. C. 1700. Az O k¨ oz´eppont´ u k¨ ornek az O-t´ ol k¨ ul¨ onb¨ oz˝ o bels˝ o pontja A. A k¨ or ker¨ ulet´enek egy B pontj´ ara OAB = α. Legyen C a k¨ orvonal egy olyan pontja, amelyre BAC = β jel¨ ol´essel 2α + β = 180◦ teljes¨ ul ´es a BAO ´es BAC sz¨ ogtartom´ anyoknak az AB f´elegyenesen k´ıv¨ ul nincs k¨ oz¨ os pontja. Igazoljuk, hogy ekkor az O, A, B, C pontok egy k¨ or¨ on vannak. C. 1701. Mennyi azon x eg´esz sz´ amok ¨ osszege, amelyekre  2x2 − 6x − 20 < −x + 5

teljes¨ ul?

K. 716. Egy boltban h´arom f¨ uzet ´es k´et toll a´ra 1110 Ft, ¨ot f¨ uzet ´es n´egy toll ara pedig 2010 Ft. Mennyibe ker¨ ´ ul egy f¨ uzet ´es mennyibe ker¨ ul egy toll?

Feladatok 11. ´evfolyamt´ol C. 1702. Az ABCD n´egysz¨ og A cs´ ucsa illeszkedik az S s´ıkra, BD ´ atl´ oja p´arhuzamos a s´ıkkal, C cs´ ucsa 8 egys´eg t´ avols´ agra van az S s´ıkt´ ol. Azt tapasztaljuk, hogy a n´egysz¨ og S-re vonatkoz´ o mer˝ oleges vet¨ ulete egy n´egyzet, melynek a´tl´ oja 6 egys´eg. Bizony´ıtsuk be, hogy az ABCD n´egysz¨ og rombusz, valamint sz´am´ıtsuk ki az oldalainak hossz´at.

K/C. 717. Egy szab´ alyos ABCDEF GHIJKL tizenk´etsz¨ og AB ´es GH oldal´ ara az ABP Q ´es GHRS n´egyzeteket ´ırjuk befel´e az ´ abr´ an l´ athat´ o m´odon. Mutassuk meg, hogy P Q ´es RS egy szab´ alyos hatsz¨ og k´et szemk¨ ozti oldala.

Javasolta: Zagyva Tiborn´e (Baja) C. 1703. Az a ´es b 10-es sz´ amrendszerbeli term´eszetes sz´ amok, mindegyik sz´ amjegy¨ uk 1-es. Mutassuk meg, hogy ha a ´es b nem relat´ıv pr´ımek, akkor sz´amjegyeik S(a) ´es S(b) ¨ osszege sem az.

K/C. 718. H´ any olyan sz´am van 1-t˝ol 50-ig, amit fel lehet ´ırni legal´ abb k´et szomsz´edos nemnegat´ıv eg´esz sz´ am ¨ osszegek´ent?

Beku ¨ld´esi hat´arid˝o: 2022. febru´ar 10. Elektronikus munkafu ¨zet: https://www.komal.hu/munkafuzet

Beku ¨ld´esi hat´arid˝o: 2022. febru´ar 10. Elektronikus munkafu ¨zet: https://www.komal.hu/munkafuzet

A B pontversenyben kit˝ uzo¨tt feladatok (5214–5221.)

A C pontversenyben kit˝ u z¨ ott gyakorlatok (717–718., 1699–1703.)

B. 5214. A 110 egy olyan sz´amjegysorozat, amelyet b´ armilyen 1-n´el nagyobb pozit´ıv eg´esz alap´ u sz´ amrendszerben tekintve p´ aros sz´amot kapunk. Van-e olyan 1-esekb˝ol ´es 0-kb´ ol a´ll´ o sz´ amjegysorozat, amelyet b´ armilyen 1-n´el nagyobb pozit´ıv eg´esz alap´ u sz´ amrendszerben tekintve 3-mal oszthat´ o pozit´ıv eg´esz sz´ amot kapunk?

Feladatok 10. ´evfolyamig K/C. 717. A sz¨ oveg´et l´ asd a K feladatokn´ al. K/C. 718. A sz¨ oveg´et l´ asd a K feladatokn´ al. K¨ oz´ episkolai Matematikai ´ es Fizikai Lapok, 2022/1

(3 pont) 31

32

K¨ oz´ episkolai Matematikai ´ es Fizikai Lapok, 2022/1

B. 5215. Adjuk meg az ¨ osszes x pozit´ıv val´os sz´amot, amelyre x + x1 eg´esz sz´ am

´es x3 + x13 pr´ımsz´ am. (4 pont)

Az A pontversenyben kit˝ uz¨ott nehezebb feladatok (815–817.)

Szaszk´ o-Bog´ arn´e Eckert Bernadett ´es Szaszk´ o-Bog´ ar Viktor ¨otlete alapj´an

B. 5216. Az ABC der´eksz¨ og˝ u h´ aromsz¨ og k¨ or´e ´ırt k¨orh¨oz az A pontban ´es a der´eksz¨ og˝ u C cs´ ucsban ´erint˝ot rajzolunk, az ´erint˝ok metsz´espontja D. Bizony´ıtsuk be, hogy a BD egyenes felezi a C-b˝ ol indul´o magass´ agot. (3 pont) ab´ol mint oldalakb´ ol u ´jabb B. 5217. Egy h´ aromsz¨ og s´ ulyvonalainak √2 -szoros´ 3 h´ aromsz¨ oget szerkeszt¨ unk. Az elj´ar´ast megism´etelj¨ uk a kapott h´ aromsz¨ogre. Mutassuk meg, hogy a m´asodik l´ep´esben az eredetivel egybev´ ag´o h´aromsz¨oget kapunk. (4 pont)

Javasolta: B´ artfai P´ al (Budapest)

B. 5218. Legfeljebb h´ any v´ alaszthat´ o ki az els˝ o 2022 pozit´ıv eg´esz sz´ am k¨oz¨ ul u ´gy, hogy semelyik k´et kiv´ alasztott sz´ am k¨ ul¨ onbs´ege ne legyen pr´ımsz´ am? (5 pont)

A. 815. Legyen q egy 1 f˝ oegy¨ utthat´ os, eg´esz egy¨ utthat´ os polinom. Bizony´ıtand´o, hogy l´etezik olyan, csak a q polinomt´ol f¨ ugg˝o C konstans, melyre tetsz˝oleges p pr´ımsz´ am ´es tetsz˝oleges N  p pozit´ıv eg´esz eset´en az n! ≡ q(n) (mod p) kongruasa van b´armely N darab egym´ast k¨ ovet˝o eg´esz enci´ anak legfeljebb CN 2/3 megold´ k¨oz¨ott. Javasolta: Navid Safaei (Ir´ an) A. 816. Petinek 2022 darab l´atsz´ olag egyforma m´agneses vas´ uti kocsija van, melyek k´etf´ele t´ıpus´ uak: bizonyosoknak az eleje ´eszaki ´es a h´ atulja d´eli, m´asoknak pedig a h´ atulja ´eszaki ´es az eleje d´eli m´ agneses polarit´ as´ u (ezek olyan j´ at´ekkocsik, melyek eleje ´es h´ atulja megk¨ ul¨ onb¨ oztethet˝ o). Peti szeretn´e eld¨ onteni, hogy egyforma sz´am´ u van-e a k´etf´ele t´ıpus´ u kocsib´ ol. Egy pr´oba sor´ an ¨ ossze lehet illeszteni k´et vas´ uti kocsit. Legkevesebb h´any pr´ ob´ ara van ehhez sz¨ uks´eg?

B. 5219. Bizony´ıtsuk be, hogy tetsz˝ oleges a, b, c val´os sz´amokra

Javasolta: P´ alv¨ olgyi D¨ om¨ ot¨ or (Budapest)

|a| |b| |c| |a + b + c|  + + . 1 + |a + b + c| 1 + |a| 1 + |b| 1 + |c| Mikor a´ll fenn egyenl˝os´eg? (5 pont)

Javasolta: Schultz J´ anos (Szeged)

B. 5220. Legyen n pozit´ıv eg´esz sz´ am. Mutassuk meg, hogy megadhat´o 1-t˝ ol ´gy, hogy k¨ oz¨ ul¨ uk ak´ arh´ any k¨ ul¨onb¨oz˝ ot ¨osszeadva (bele´ert2n+2 -ig n n´egyzetsz´am u ve az egytag´ u ¨ osszegeket ´es az ¨ osszes sz´ am ¨ osszeg´et is) csupa k¨ ul¨onb¨oz˝ o sz´ amot kapjunk.∗ (6 pont)

ort, amely A. 817. Legyen ABC egy tetsz˝ oleges h´ aromsz¨ og. Tekints¨ uk azt a k¨ ´erinti az AB ´es AC oldalt, ´es bel¨ ulr˝ ol ´erinti a h´aromsz¨ og k¨ or¨ ul´ırt k¨ or´et a T pontban. A h´aromsz¨og be´ırt k¨ or´enek k¨ oz´eppontja legyen I, ´es a be´ırt k¨ or ´erintse a BC, CA, illetve AB oldalt a D, E, illetve F pontban. Legyen N a DF szakasz felez˝opontja. Bizony´ıtsuk be, hogy a BT N h´ aromsz¨ og k¨ or¨ ul´ırt k¨ ore, a T I egyenes ´es a D pontb´ ol az EF szakaszra ´all´ıtott mer˝ oleges egy ponton megy ´at. Javasolta: Diaconescu Tashi (Rom´ ania) Beku ¨ld´esi hat´arid˝o: 2022. febru´ar 10. Elektronikus munkafu ¨zet: https://www.komal.hu/munkafuzet

Javasolta: Freud R´ obert (Budapest)

B. 5221. Az ABC hegyessz¨ og˝ u h´ aromsz¨ ogben a be´ırt k¨or ´erint´esi pontja a BC, CA, AB oldalon rendre D, E, illetve F . A h´ aromsz¨og k¨or´e ´ırt k¨or az AEF k¨ort az A-t´ ol k¨ ul¨ onb¨ oz˝ o P , a BF D k¨ ort a B-t˝ ol k¨ ul¨ onb¨oz˝ o Q, a CDE k¨ort pedig a Ct˝ ol k¨ ul¨ onb¨ oz˝ o R pontban metszi. Mutassuk meg, hogy a DP , EQ ´es F R egyenesek egy ponton mennek ´at. (6 pont)

Javasolta: Lovas M´ arton (Budapest) Beku ¨ld´esi hat´arid˝o: 2022. febru´ar 10. Elektronikus munkafu ¨zet: https://www.komal.hu/munkafuzet



Informatik´ab´ ol kit˝ uz¨ott feladatok

I. 553. Faktori´ alis sz´amrendszerben a helyi´ert´ekek nem egy eg´esz sz´ am, az alapsz´ am hatv´anyai, hanem az n-edik helyi´ert´ek az n sz´ am faktori´ alisa. Teh´ at az els˝o helyi´ert´eken l´ev˝ o sz´ amjegyet 1-gyel, a m´asodik helyi´ert´eken a´ll´o sz´ amot 2-vel, a harmadik helyi´ert´eken a´ll´ o sz´ amot 6-tal kell szorozni, ´es ´ıgy tov´abb. Ennek megfelel˝ oen

L´ asd Freud R´ obert cikk´et a 2. oldalon.

K¨ oz´ episkolai Matematikai ´ es Fizikai Lapok, 2022/1

33

34

K¨ oz´ episkolai Matematikai ´ es Fizikai Lapok, 2022/1

a 3310! faktori´ alis sz´ am ´ert´eke t´ızes sz´ amrendszerben 3 · 4! + 3 · 3! + 1 · 2! = 92. Igazolhat´ o, hogy a fel´ır´as egy´ertelm˝ u, teh´ at minden pozit´ıv eg´esznek egy alakja van faktori´ alis sz´ amrendszerben. K´esz´ıts¨ unk programot i553 n´even, amely egy t´ızes sz´amrendszerben megadott pozit´ıv eg´esz sz´ amot fel´ır faktori´ alis sz´ amrendszerben. A program a standard bemenet els˝ o sor´ ab´ ol olvassa be a t´ızes sz´ amrendszerben fel´ırt pozit´ıv eg´eszet, ´es a standard kimenetre ´ırja ki a sz´amot faktori´ alis sz´ amrendszerben. A bemenet legf¨oljebb 18 sz´amjegyb˝ ol a´ll. Amennyiben a sz´ am faktori´ alis alakj´ aban egy helyi´ert´eken t¨obbjegy˝ u sz´ am a´ll, akkor azt tegy¨ uk z´ ar´ ojelbe. P´elda bemenetek 500 5698 89764351 1569787435467978

P´elda kimenetek 40310 1052120 22732241101 47068(11)0(10)725350300

bajnok (id, ev, vsz id, jatekos id, egyesulet id) id A bajnoki c´ım azonos´ıt´ oja (sz´ am), ez a kulcs. ev A bajnoks´ ag ´eve (sz´ am). A versenysz´ am azonos´ıt´ oja (sz´ am). vsz id A bajnoki c´ımet szerz˝ o j´at´ekos azonos´ıt´ oja (sz´ am). jatekos id at´ekos ebben az ´evben melyik egyes¨ uletet k´epviselte (sz´am). egyesulet id A j´ versenyszam (id, nev) id A versenysz´ am azonos´ıt´ oja (sz´ am), ez a kulcs. nev A versenysz´am neve (sz¨ oveg), ´ert´eke f´erfi egy´eni, n˝oi egy´eni, f´erfi p´ aros, n˝ oi p´ aros ´es vegyes p´ aros lehet. egyesulet (id, nev, orszag) id Az egyes¨ ulet azonos´ıt´ oja (sz´ am), ez a kulcs. nev Az egyes¨ ulet neve (sz¨ oveg). orszag Az egyes¨ ulet orsz´ aga (sz¨ oveg).

Bek¨ uldend˝o egy t¨ om¨ or´ıtett i553.zip ´ allom´anyban a program forr´ ask´odja ´es r¨ ovid dokument´ aci´ oja, amely megadja, hogy a forr´ as´allom´any melyik fejleszt˝oi k¨ ornyezetben ford´ıthat´ o. I. 554. A magyar asztalitenisz-bajnoks´ ag 1905-ben kezd˝od¨ott. Jelenleg ¨otf´ele versenysz´ am – f´erfi egy´eni, n˝ oi egy´eni, f´erfi p´ aros, n˝ oi p´ aros ´es vegyes p´ aros – bajnoki c´ım´et osztj´ak ki ´evente. A magyar nyelv˝ u Wikip´edia oldal´an megtal´alhat´ o adatok sok ´erdekes k´erd´es megv´ alaszol´ as´ ahoz ny´ ujtanak forr´ ast: https://hu.wikipedia.org/wiki/Magyar_asztalitenisz-bajnoks´ ag. A feladatunk a forr´ asadatok feldolgoz´ asa, ´atrendez´ese olyan form´aba, hogy azokat adatb´ azisba lehessen import´alni. Minden adatot a megadott webc´ımr˝ ol ments¨ unk le ´es szervezz¨ uk az al´ abb megadott adatb´ azis-szerkezetbe, m´as forr´ as nem a´ll rendelkez´esre. 1. Ments¨ uk le a megadott webc´ımr˝ ol a bajnokok adatait. Tetsz˝ oleges alkalmaz´ assal rendezz¨ uk a´t, t¨ or¨ olj¨ uk ki a felesleges r´eszeket, illetve eg´esz´ıts¨ uk ki a sz¨ uks´eges adatokkal a t´ abl´ akat. Haszn´alhatunk p´eld´ aul sz¨ovegszerkeszt˝ ot, t´ abl´ azatkezel˝ ot vagy k´esz´ıthet¨ unk saj´ at programot is. Az ´atalak´ıt´ as egyes l´ep´eseit m´as-m´ as programmal is v´egezhetj¨ uk. A rendezett adatokat utols´ o l´ep´esk´ent TXT t´ıpus´ u, tabul´ atorokkal tagolt UTF-8 k´ odol´as´ u egyszer˝ u sz¨oveges a´llom´anyokk´ent ments¨ uk, amelyek neve a t´ ablanevekkel egyezzen meg. Az ´allom´anyok els˝ o sora tartalmazza a mez˝ oneveket az adatb´ azisba import´ al´ashoz. 2. A t´ abl´ ak kialak´ıt´as´ ahoz vegy¨ uk figyelembe az al´ abbi t´ ablale´ır´asokat ´es kapcsolatokat: T´abla: jatekos (id, nev, neme) id A j´ at´ekos azonos´ıt´ oja (sz´ am), ez a kulcs. nev A j´ at´ekos neve (sz¨ oveg). neme A j´ at´ekos neme (logikai), ´ert´eke igaz (f´erfi) ´es hamis (n˝ o) eset´en. K¨ oz´ episkolai Matematikai ´ es Fizikai Lapok, 2022/1

35

A t´ abl´ ak els˝ odleges kulcs mez˝oj´et (id) tetsz˝ oleges ´ert´ekkel kit¨ olthetj¨ uk, m´ıg ok) megad´ asakor u ´gy j´ arjunk az idegen kulcsok (jatekos id, egyesulet id, vsz id mez˝ el, hogy azok helyes kapcsolatot mutassanak. Bek¨ uldend˝o egy t¨ om¨ or´ıtett i554.zip ´ allom´anyban a n´egy adatt´ abla sz¨ oveges ´allom´anya ´es r¨ ovid dokument´ aci´ oja, amely megadja a feldolgoz´as l´ep´eseit, eszk¨ ozeit. A k¨ovetkez˝ o feladat a most elk´esz´ıtett adatb´ azishoz kapcsol´ odik. Ha megoldottuk ezt a feladatot, akkor a k¨ ovetkez˝ o feladatban haszn´aljuk a saj´ at megold´ast az adatb´ azis forr´ asak´ent. Amennyiben ezt a feladatot nem vagy csak r´eszben oldottuk meg, akkor a k¨ ovetkez˝ o feladathoz mell´ekelt forr´ asokat haszn´aljuk annak megold´ as´ahoz. ´ A magyar asztalitenisz-bajnoks´ I. 555 (E). ag eddigi adatait kellett az I. 554. feladatban el˝ok´esz´ıteni, azaz sz¨ oveges t´ıpus´ u´ allom´anyokba, az adatb´ azisba t¨ ort´en˝ o import´ al´ashoz megfelel˝o szerkezetben ´es tartalommal menteni. Ebben a feladatban az adatb´azis l´etrehoz´ asa, majd a k´erd´esekre adand´ o v´ao probl´ema. Az adatok, ha az I. 554. laszokhoz lek´erdez´esek k´esz´ıt´ese a megoldand´ feladatot megoldottuk, akkor az ott l´etrehozott a´llom´anyokban, vagy cs¨ okkentett 36

K¨ oz´ episkolai Matematikai ´ es Fizikai Lapok, 2022/1

rekordsz´ ammal a jatekos.txt, a bajnok.txt, a versenyszam.txt ´es az egyesulet.txt ´ allom´anyokban a´llnak rendelkez´es¨ unkre. Az ´allom´anyok tabul´ atorral tagolt, UTF-8 k´ odol´as´ u sz¨ ovegf´ ajlok, az els˝ o sorok a mez˝oneveket tartalmazz´ ak. A t´abl´ ak kialak´ıt´as´ahoz vegy¨ uk figyelembe az I. 554. feladatn´ al megadott t´ablale´ır´ asokat ´es kapcsolatokat. A forr´ asadatok kieg´esz´ıt´ese nem r´esze ennek a feladatnak. 3. K´esz´ıts¨ unk u ´j adatb´ azist pingpong n´even. Import´ aljuk az adatt´ abl´ akat az adatb´ azisba.

felhaszn´ al´as´aval. Adjuk meg azt a legnagyobb H sz´eps´eget, amely eset´en mindegyik gyerek tud egy H sz´eps´eg˝ u h´ oembert ´ep´ıteni a rendelkez´esre a´ll´ o h´ og¨ omb¨ okb˝ ol. A bemenet els˝ o sor´ aban az N ´es D sz´ amok tal´ alhat´ oak. A k¨ ovetkez˝ o sor N darab sz´ amot tartalmaz, az i-edik sz´ am T [i]. A kimenet egyetlen sor´aban egy sz´ am szerepeljen: a legnagyobb el´erhet˝ o H sz´eps´eg. Ha nem lehets´eges a megadott felt´etelek mellett legal´ abb D azonos sz´eps´eg˝ u h´oembert ´ep´ıteni, akkor a kimenet egyetlen sor´ aban a −1 szerepeljen. P´elda:

4. A l´etrehoz´ as sor´ an ´ all´ıtsuk be a megfelel˝ o t´ıpusokat ´es els˝ odleges kulcsokat. K´esz´ıts¨ uk el a k¨ ovetkez˝ o feladatok megold´ as´ at. A z´ ar´ojelben l´ev˝ o n´even ment¨ ant mez˝ok szes¨ uk el azokat. Ugyelj¨ unk arra, hogy a megold´ asban pontosan a k´ıv´ repeljenek. 5. Adjuk meg lek´erdez´es seg´ıts´eg´evel, hogy melyik ´evben adt´ ak ki a vegyes p´ aros bajnoki c´ımet el˝ osz¨ or. (5vegyes) 6. List´azzuk ki lek´erdez´es seg´ıts´eg´evel az els˝ o 10 legt¨obb bajnoki c´ımet nyer˝ o j´at´ekos nev´et. (6top10) 7. Az egyik legeredm´enyesebb n˝ oi j´at´ekos B´ atorfi Csilla volt. Lek´erdez´essel adjuk meg, hogy n˝oi p´aros versenysz´amban kikkel nyert bajnoks´ agot. A list´ aban minden n´ev egyszer jelenjen meg. (7batorfiparjai) 8. Adjuk meg lek´erdez´es seg´ıts´eg´evel, hogy Klamp´ ar Tibor els˝o ´es utols´ o bajnoki c´ıme k¨ oz¨ ott f´erfi egyesben ki szerzett m´eg bajnoki c´ımet. A list´ aban minden j´at´ekos neve egyszer szerepeljen, de Klamp´ar Tibor nev´et m´ ar ne jelen´ıts¨ uk meg. (8klamparral) 9. Lek´erdez´essel hat´ arozzuk meg, hogy a magyar egyes¨ uleteken k´ıv¨ ul mely oragon. Minden sz´ agok klubjainak j´ at´ekosai nyertek bajnoki c´ımet Magyarorsz´ orsz´ ag neve egyszer szerepeljen a list´aban. (9nemzetek) 10. K´esz´ıts¨ unk lek´erdez´est, amely kilist´ azza azokat az ´eveket, amikor a n˝ oi p´aros bajnoki c´ımet azonos egyes¨ ulethez tartoz´ o j´at´ekosok nyert´ek. A list´ aban az ´evsz´ am, a j´ at´ekosok neve ´es az egyes¨ uletek neve jelenjen meg. (10kereszt) Bek¨ uldend˝o egy t¨om¨ or´ıtett i555.zip ´ allom´anyban a megold´ ast ad´ o pingpong adatb´ azis ´es egy r¨ ovid dokument´ aci´ o, amely le´ırja, hogy az adatb´ azis melyik program seg´ıts´eg´evel k´esz¨ ult. Let¨ olthet˝ o´ allom´any: jatekos.txt, bajnok.txt, versenyszam.txt, egyesulet.txt. I/S. 59. Egy t´ avoli orsz´ agban egy hosszas havaz´as ut´an D gyerek eld¨ont¨otte, hogy az eg´esz napot h´oember´ep´ıt´essel fogja t¨ olteni. A nap v´eg´ere el is k´esz´ıtettek N darab h´og¨ omb¨ ot. Az i-edik h´og¨ omb ´atm´er˝ oje T [i] millim´eter. Egy h´oember ´ep´ıt´esekor tetsz˝oleges sz´ am´ u h´ og¨ omb¨ ot kell egym´ as tetej´ere tenni (egy h´ oember legal´ abb k´et h´ og¨ ombb˝ol ´ all), de figyelni kell arra, hogy egy h´og¨ombre csak egy szigor´ uan kisebb h´ og¨ omb¨ ot tehet¨ unk. Egy h´ oember sz´eps´ege H, ha H darab h´ og¨ ombb˝ol ´all. A gyerekek megegyeztek, hogy mindegyik¨ uk pontosan egy darab H sz´eps´eg˝ u h´ oembert ´ep´ıt a h´og¨omb¨ok K¨ oz´ episkolai Matematikai ´ es Fizikai Lapok, 2022/1

37

Bemenet

Kimenet

6 2 1 4 2 1 5 1

2

Nem ´ep´ıthet˝ o k´et darab 3 sz´eps´eg˝ u h´ oember, de 2 sz´eps´eg˝ u igen. Korl´ atok: 1  N, D  100 000, 1  T [i]  109 . Id˝ olimit: 0,4 mp. ´ Ert´ekel´es: a pontok 50%-a kaphat´ o, ha a program az 1  N  10 tesztesetekre helyes megold´ ast ad. Bek¨ uldend˝o egy is59.zip t¨ om¨ or´ıtett a´llom´anyban a megfelel˝oen dokument´ alt ´es kommentezett forr´ asprogram, amely tartalmazza a megold´ as l´ep´eseit, valamint megadja, hogy a program melyik fejleszt˝ oi k¨ ornyezetben futtathat´ o. S. 158. Egy n´egyzeth´ al´on adott egy soksz¨ og, melynek oldalai a r´acsvonalakra illeszkednek. Van D sz´ am´ u dobozunk. Egy-egy dobozban 1, 2 ´es 3 egys´eg hossz´ u p´alcik´ ak vannak, melyek ¨ osszhossza megegyezik a soksz¨ og ker¨ ulet´evel. K´erd´es, hogy le lehet-e fedni a soksz¨ og oldalait az egyes dobozokban tal´alhat´ o p´ alcik´ akkal azok elt¨or´ese n´elk¨ ul? T¨ obb p´ alcika alkothat egy hosszabb oldalt. K´esz´ıts¨ unk programot, amely a soksz¨og ´es minden egyes doboz eset´en megv´ alaszolja a k´erd´est! og cs´ ucsainak N sz´ am´at. A k¨ ovetBemenet: az els˝ o sor tartalmazza a soksz¨ kez˝ o N sor mindegyike egy-egy cs´ ucs x ´es y koordin´ at´ aj´at tartalmazza a soksz¨ og valamilyen ir´ any´ u k¨ or¨ ulj´ar´ asa szerint. A soksz¨ og els˝ o ´es utols´ o cs´ ucsa is ´elet alkot. A k¨ovetkez˝ o sorban a dobozok D sz´ ama van. Ezut´ an D sorban az egyes dobozok tartalm´at ´ırjuk le. Ezek mindegyike h´ arom sz´am, melyek rendre az 1, 2, illetve 3 hossz´ u p´alcik´ ak darabsz´ ama a dobozban. Kimenet: D sort kell ki´ırni. Ezek mindegyike IGEN”, ha a soksz¨ og oldalai ” a doboz tartalm´aval lefedhet˝ok ´es NEM” k¨ ul¨ onben. ” P´elda: Bemenet (a / jel sort¨ or´est helyettes´ıt) 4 / 0 0 / 1 0 / 1 2 / 0 2 3 / 6 0 0 / 2 2 0 / 1 1 1

Kimenet (a / jel sort¨ or´est helyettes´ıt) IGEN / IGEN / NEM

Korl´ atok: a koordin´ at´ ak abszol´ ut ´ert´eke ´es a ker¨ ulet is legfeljebb 105 . Id˝olimit: 1 mp. ´ ekel´es: a pontok 30%-a kaphat´ Ert´ o, ha a program a legfeljebb 20 ker¨ ulet˝ u tesztesetekre helyes megold´ ast ad. 38

K¨ oz´ episkolai Matematikai ´ es Fizikai Lapok, 2022/1

Bek¨ uldend˝o egy s158.zip t¨ om¨ or´ıtett a´llom´anyban a megfelel˝oen dokument´ alt ´es kommentezett forr´ asprogram, amely tartalmazza a megold´ as l´ep´eseit, valamint megadja, hogy a program melyik fejleszt˝ oi k¨ ornyezetben futtathat´ o. A feladatok megold´asai regisztr´aci´o ut´an a k¨ovetkez˝o c´ımen t¨olthet˝ok fel: https://www.komal.hu/munkafuzet Beku ¨ld´esi hat´arid˝o: 2022. febru´ar 15.

Matematikai k´epz´esek az ELTE TTK-n

Kedves leend˝ o Egyetemista! A K¨ oMaL olvas´ ojak´ent bizony´ ara sz´ıvesen foglalkozol matematik´ aval, ´es felmer¨ ulhetett m´ar Benned az a gondolat, hogy ´eletp´ aly´ adul ennek a sz´ep tudom´ anynak a m˝ uvel´es´et v´ alasztod, illetve szeretn´el megismerkedni alkalmaz´ asaival a m˝ uszaki, gazdas´ agi ´es p´enz¨ ugyi ´elet k¨ ul¨onb¨oz˝ o ter¨ uletein. Az alkalmazott matematika ma m´ ar az ´elet szinte minden ter¨ ulet´en n´elk¨ ul¨ozhetetlen, ´es az ilyen k´epzetts´eg˝ u munkaer˝ o ir´ ant egyre n¨ovekszik az ig´eny. A Fortune magazin cikke szerint a legjelent˝osebb v´altoz´ as az u ¨zleti ´eletben az ipari forradalom ota a matematikai algoritmusok t´erh´ ´ od´ıt´ asa (http://fortune.com/2015/01/22/ the-algorithmic-ceo/). Egy amerikai felm´er´es ´evr˝ ol ´evre a legjobb foglalkoz´ asok k¨ oz¨ ott tartja sz´ amon a matematikust ´es a szint´en matematikai el˝ok´epzetts´eget o adattud´ ost, aktu´ ariust ´es statisztikust (https://www.careercast.com/ ig´enyl˝ jobs-rated/best-jobs-2021). Mindez Magyarorsz´agra is igaz, az ELTE-n v´egzett matematikusokat nemcsak a kutat´oint´ezetek, egyetemek v´ arj´ ak, hanem sz´ amos c´eg is, igen j´ o fizet´essel. Esetleg m´eg nem d¨ ont¨ ott´el, de legink´ abb matematik´ ab´ol folytatn´al fels˝ofok´ u tanulm´ anyokat? Minderre kit˝ un˝ o lehet˝os´eg ny´ılik az orsz´ag egyik legnagyobb m´ ult´ u egyetem´en, az E¨ otv¨ os Lor´ and Tudom´anyegyetem Term´eszettudom´anyi Kar´an, ahol vil´agh´ır˝ u professzorokt´ol ´es lelkes, k¨ ozvetlen fiatal oktat´ okt´ ol tanulhatsz. Pezsg˝ o di´ak´elet v´ ar r´ ad az ELTE korszer˝ u sz´ am´ıt´og´epparkkal felszerelt, a K¨oMaL szerkeszt˝ os´eg´enek is otthont ad´o modern l´ agym´anyosi ´ep¨ uletegy¨ uttes´eben. A bolognai k´epz´esi rendszerbe illeszkedik BSc k´epes´ıt´est ny´ ujt´ o h´ arom´eves maunk. Itt az els˝ o ´evben hallgat´oi ´es oktat´ oi mentorok biztos´ıtj´ ak, tematikai alapk´epz´es¨ hogy mindenki be tudjon illeszkedni ´es tal´ aljon el˝ oismereteinek, k´epess´egeinek ´es tanul´ asi sebess´eg´enek megfelel˝o neh´ezs´eg˝ u feladatokat. Az els˝o ´ev v´eg´en d¨onthetsz arr´ ol, hogy milyen t´em´ akkal szeretn´el a tov´ abbiakban behat´obban foglalkozni. A k´ın´ alat sz´eles: aki szeretne, az elm´elyedhet az elm´eleti matematika k´erd´eseiben, hiszen szinte minden fontos ter¨ uletr˝ ol hirdet¨ unk kurzusokat. Ezek ´ep´ıtenek a magyar matematikai kutat´asok m´elt´ an vil´agh´ır˝ u hagyom´ anyaira, ugyanakkor szil´ ard alapokat ny´ ujtanak a modern matematika m˝ uvel´es´ehez, j´ ol felk´esz´ıtve hallgat´oinkat a leend˝ o kutat´oi munk´ara. Akit viszont az alkalmaz´ asok ´erdekelnek, megteheti, hogy az alapok elsaj´at´ıt´ asa ut´ an olyan modern t´em´akkal is foglalkozzon, mint az adattudom´ any vagy a mesters´eges intelligencia matematikai k´erd´esei. Azoknak is aj´anljuk a matematiK¨ oz´ episkolai Matematikai ´ es Fizikai Lapok, 2022/1

39

ka alapk´epz´esi szakot, akik ismereteiket k´es˝ obb ink´ abb a matematik´an k´ıv¨ ul szeretn´ek majd gy¨ um¨ olcs¨ oztetni. Itt szerzett tud´ asukat hasznos´ıthatj´ ak p´eld´ aul gazdas´ agi ter¨ uleten, m´edi´aban, a matematika n´epszer˝ us´ıt´es´eben, a k¨ ozm˝ uvel˝ od´esben – ´es a megszerzett matematikai gondolkod´ asm´ od mindv´egig seg´ıteni fogja ˝oket a munk´ajukban. A k´epz´es egy´eb vonatkoz´ asair´ ol tov´ abbi r´eszletek a http://www.math.elte. hu/ honlapon a K´epz´esek men¨ upont alatt tal´ alhat´ ok. Aj´ anljuk a k¨ oz´episkol´ asoknak sz´ ol´o oldalainkat is, ahol v´egzett di´ akjainkkal k´esz¨ ult interj´ uk is l´athat´ ok. A legkiemelked˝obb hallgat´ ok az egyetemi oktat´omunk´aba is bekapcsol´ odhatnak, ´es j´o es´ellyel p´ aly´ azhatnak ¨ oszt¨ ond´ıjakra, k¨ ulf¨ oldi r´eszk´epz´esre (pl. az Eras´ oszt¨ ond´ıjprogramja m´ar a leend˝o els˝ o´evemus+ program keret´eben). Az UNKP ¨ seknek is el´erhet˝ o! R´eszletes t´ aj´ekoztat´ o: http://csikvarip.web.elte.hu/diak_ kutatas.html. Az alapk´epz´est tov´abbi k´et´eves szakasz k¨ ovet(het)i (mesterk´epz´es vagy r¨ oviden MSc), egyetem¨ unk¨ on a Matematikai Int´ezet gondoz´ as´ aban matematikus, alkalmazott matematikus, valamint biztos´ıt´ asi ´es p´enz¨ ugyi matematika mesterszakok indulnak. BSc-t v´egzett hallgat´ oink term´eszetesen m´as (bel- ´es k¨ ulf¨ oldi) oktat´ asi int´ezm´eny programjain is folytathatj´ ak tanulm´anyaikat. A mesterszakot v´egzettek k¨oz¨ ul a legkiv´ al´obbak sz´ am´ara biztos´ıtjuk a doktori fokozat megszerz´es´enek lehet˝os´eg´et (PhD-k´epz´es). Egyetem¨ unk¨ on gondosan a´polt hagyom´any, hogy a r´ atermett, tehets´eges di´ akok neves professzorok vezet´es´evel bekapcsol´ odnak a tudom´ anyos kutat´ asba. A legkiv´al´obb hallgat´ ok matematikai versenyeken is sikerrel szerepelnek, p´eld´ aul az Egyetemi Hallgat´ ok Nemzetk¨ ozi Matematikaverseny´en az elm´ ult t´ız ´evben k´etszer is az ELTE csapata v´egzett az ´elen t¨ obb, mint 70 egyetem csapat´ anak verseny´eben – olyan nagyh´ır˝ u egyetemeket is megel˝ozve, mint a Yale, a Princeton vagy a Moszkvai ´ Allami Egyetem.

Matematikatan´ar-k´epz´es az ELTE TTK-n Az ELTE Term´eszettudom´ anyi Kar´ an sok ´evtizedes m´ ultra tekint vissza a matematika szakos tan´ ark´epz´es. Az ´altal´ anos ´es k¨ oz´episkol´ak r´esz´er˝ ol mindig jelent˝os ig´eny mutatkozott a n´ alunk v´egzett matematikatan´ arok ir´ant, akik k¨ oz¨ ul sokan k¨ ulf¨old¨on is sikeres oktat´oi p´ aly´ at futottak be. A matematika szakos tan´ ari p´ aly´ at els˝ osorban azoknak a k¨ oz´episkol´ as di´akoknak aj´ anljuk, akik sz´ am´ara ¨ or¨ omet jelent ´erdekes matematikai feladatokon gondolkodni, ´es j´o ´erz´est okoz a megold´ asokra m´ asokat is r´ avezetni, m´asokkal is megosztani azt az ´elvezetet, amit a matematika megismer´ese jelent. A tan´ark´epz´es osztatlan form´aban zajlik. A tan´ark´epz´esre val´o jelentkez´es sor´ an a leend˝o hallgat´ oknak egy szakp´ art kell megjel¨ olni. Az ELTE-n a matematika szak mell´e term´eszettudom´ anyos szakokon ´es az informatik´an k´ıv¨ ul v´ alasztani lehet a b¨olcs´esz szakok (p´eld´ aul a magyar, a t¨ ort´enelem vagy a nyelvszakok) k¨ oz¨ ul is. A szakt´ argyi tan´ıt´ asi gyakorlatok teljes´ıt´es´ere az ELTE hallgat´ oinak a legjobb budapesti iskol´ akban, kiv´ al´o vezet˝ otan´ arok ir´any´ıt´ asa mellett ny´ılik lehet˝os´eg¨ uk. B´atran a´ll´ıthatjuk teh´ at, hogy a K¨ oMaL minden olvas´oj´anak testhez´all´ o k´epz´est tudunk ny´ ujtani az ELTE Matematikai Int´ezet´eben. Ha szem´elyesen is szeretn´el tal´ alkozni leend˝o oktat´ oiddal, besz´elgetni a mostani egyetemist´ akkal, akkor gyere el az ELTE TTK ny´ılt napj´ara janu´ ar 21-´en! 40

K¨ oz´ episkolai Matematikai ´ es Fizikai Lapok, 2022/1

Fizika alapszak ´es fizikatan´ar-k´epz´es az ELTE TTK Fizikai Int´ezet´eben A vil´agon az egyik legizgalmasabb ´es legszebb feladat a term´eszet kutat´ asa, m˝ uk¨ od´es´enek meg´ert´ese. A kutat´ as egy ´eletre sz´ ol´o ´elm´eny, egy ´eletre sz´ ol´o kih´ıv´ as ´es izgalom. Ugyanakkor a hallgat´oink olyan nyitotts´ agot, probl´emamegold´ o k´eszs´eget is elsaj´at´ıtanak, amely az ´elet b´ armely ter¨ ulet´en nagyon j´ol hasznos´ıthat´ o. Az itt v´egzettek k¨ oz¨ ott kiv´ al´o, a nemzetk¨ ozi ´elvonalban dolgoz´o fizikusokat tal´alunk, de olyan c´egvezet˝ ot is, aki egy patin´as Wall Street-i befektet´esi bank budapesti matematikai modellez˝ o csoportj´ at vezeti, vagy p´eld´ aul olyan, ma m´ ar az USA-ban ´el˝o v´allalkoz´ ot, aki az amerikai l´egier˝ onek sz´ all´ıt folyad´ekkrist´aly-kijelz˝os sisakokat. A fizika alapk´epz´es mellett int´ezet¨ unkben k´epezz¨ uk a fizikatan´ arok jelent˝os r´esz´et. Aki szereti a fizik´ at ´es m´ar most is szereti t´ arsait tan´ıtani, aj´ anljuk figyelm´ebe ar-k´epz´es¨ unket! a fizikatan´ Hogy mi´ert ´erdemes a fizika alapszakot v´alasztani? • Modern oktat´as A k´epz´es¨ unk t¨ obbszint˝ u ´es sokoldal´ u. A sokoldal´ us´ag abban mutatkozik meg, hogy a harmadik f´el´evvel kezd˝ od˝ oen ´erdekl˝ od´esi ter¨ ulet szerint specializ´aci´ ot (fizikus, informatikus fizikus, biofizikus, csillag´ asz, geofizikus, meteorol´ ogus) lehet v´ alasztani. A k´epz´es k¨ oz¨ os r´esz´eben a magas szint˝ u fizikai ismereteken t´ ul matematik´at, elektronik´ at ´es informatik´at is oktatunk. Mivel nincs fizikus k´ıs´erletek n´elk¨ ul, az alapvet˝ o fizikai m´er´esi k´eszs´egeket ´es mag´at a k´ıs´erletez˝ o szeml´eletet a fizikai laborat´oriumi gyakorlatokon lehet elsaj´at´ıtani. A laborokon a di´akok p´eld´ aul Raspberry Pi vez´erl´est haszn´ alva v´egzik alapm´er´eseiket, k´es˝ obb pedig olyan ´erdekes fizikai jelens´egekkel ´es berendez´esekkel tal´ alkoznak, mint a pozitronemisszi´ os az´ o elektronmikroszk´ op vagy ´eppen a kvantumtomogr´ afia, a hologr´ afia, a p´aszt´ rad´ır. A kurzusok jelent˝os r´esze k´et (norm´al ´es emelt) szinten v´egezhet˝ o, melyek k¨ onnyen a´tj´ arhat´ oak. A norm´al szint biztos´ıtja, hogy a nem elit iskol´ab´ol ´erkez˝ o, de motiv´ alt hallgat´ ok sz´am´ara is elsaj´ at´ıthat´ o ´es ´elvezhet˝ o legyen a tananyag. Az emelt szint˝ u´ or´ akon gyorsabb halad´ ast ´es kieg´esz´ıt˝o tartalmakat biztos´ıtunk. • Vil´agsz´ınvonal´ u kutat´asok A vil´agon k¨ or¨ ulbel¨ ul 1100 olyan fizikai int´ezet van, amelyet rangsorolnak (Sanghaj rangsor), ´es ez m´ar egy v´alogatott t´ arsas´ ag. Ebben int´ezet¨ unk a 100– 150. hely k¨ oz¨ ott van, k¨ ozel a fels˝ o 10%-hoz. ´Igy b´ atran a´ll´ıthatjuk, hogy az ELTE TTK Fizikai Int´ezet nemzetk¨ ozi viszonylatban is kiemelked˝o hely a fizika tanul´as´ara. Az ¨ osszes magyar int´ezet k¨ oz¨ ul itt a legsz´elesebb a v´ alaszt´eka azoknak a ter¨ uleteknek, amelyeket oktatunk ´es kutatunk. A fizika legmodernebb, legizgalmasabb ter¨ uleteivel foglalkozunk: a gravit´ aci´ os hull´amok kutat´ as´at´ ol a r´eszecske- ´es biofizik´ an kereszt¨ ul az asztrofizik´aig, a nanotechnol´ ogi´aig ´es a kvantumsz´ am´ıt´ og´epekig mindent lefed¨ unk, ami ma ´erdekes a fizik´ aban. K¨or¨ ulbel¨ ul sz´ az oktat´ onkkal ´es kutat´ onkkal, valamint di´ akjainkkal nagyon sok nemzetk¨ozi egy¨ uttm˝ uk¨od´esben

K¨ oz´ episkolai Matematikai ´ es Fizikai Lapok, 2022/1

41

vesz¨ unk r´eszt. A kutat´ as ir´ant is ´erdekl˝ od˝ o di´ akok sz´am´ara bej´ aratott u ´t vezet a tudom´anyos di´ akk¨ ori projektek fel´e. A di´ akk¨ ori kutat´ omunk´ ak kiv´al´o alapot adnak a k¨ ulf¨oldi egyetemeken t¨ ort´en˝ o mesterk´epz´esben vagy doktori iskol´aban t¨ ort´en˝ o tov´ abbtanul´asra. Az ELTE TTK-n foly´o fizikai t´em´ aj´ u kutat´ asok sok esetben vil´ agsz´ınvonal´ u kutat´ ohelyekkel t¨ ort´en˝ o egy¨ uttm˝ uk¨ od´esben val´ osulnak meg. Di´ akjaink eljuthatnak a sv´ ajci CERN r´eszecskefizikai kutat´ ocentrumba, vagy a LIGO amerikai gravit´ aci´ oshull´ am-detektor eredm´enyeit elemezhetik. • Kit˝ un˝o elhelyezked´esi lehet˝os´egek A fizika t´ argy tud´asa, a fels˝ o szint˝ u matematika ´es a programoz´ asi ismeretek, amit a fizika alapszakokon el lehet saj´at´ıtani, sz´ amos munkahelyen ad lehet˝ os´eget a karrier ´ep´ıt´es´ere. A fizika szakon v´egzetteket nemcsak a kutat´oint´ezetekben, egyetemeken v´ arj´ ak, hanem p´eld´ aul a p´enz¨ ugyekkel, informatik´ aval, t´ avk¨ ozl´essel, m´ern¨oki vagy orvostudom´anyokkal foglalkoz´ o c´egek is sz´ıvesen alkalmazz´ ak o˝ket. • Hallgat´oi ´elet Az ELTE TTK hallgat´ oi ´elete vid´ am ´es szerte´ agaz´o. A Magyar Fizikus Hallgat´ ok Egyes¨ ulete sz´ amos programot szervez a hallgat´oinknak. K¨ ulf¨ oldi di´ akkonferenci´akon vagy cseregyakorlatokon lehet r´eszt venni, szabadid˝ os programok ´es a fizika t´ argyakban felk´esz´ıt˝ o programok szerepelnek a palett´ an. A fizika szakokhoz j´ol szervezett mentorprogram t´ arsul. Minden ´evfolyamon t¨ obb kik´epzett mentor seg´ıt a t´argyak felv´etele k¨ or¨ uli k´erd´esekben, az optim´ alis egyetemi strat´egi´ ak megtal´ al´as´aban, ´es ´ atadj´ ak a fels˝ obb ´eves di´ akok a´ltal ¨ osszegy˝ ujt¨ ott tapasztalatokat. A fizika szak nem ´er v´eget a BSc-fokozat megszerz´es´evel. Az ELTE TTK Fizikai Int´ezet´eben 5 k´et´eves mesterk´epz´esi (MSc) szakra lehet jelentkezni: fizikus, geofizikus, csillag´asz, meteorol´ ogus, anyagtudom´ any. A fizikus mesterin bel¨ ul a kutat´ ofizikus, biofizikus, k¨ ornyezetfizikus, tudom´ anyos adatanalitika ´es modellez´es (ez ut´ obbi t¨obbek k¨ oz¨ ott napjaink forr´ o” t´em´ aj´aval, az ´ ori´asi adathalmazokon v´egzett ” kutat´ asokkal, a big data”-val foglalkozik). A k´epz´es harmadik szintje az int´ezetben ” a n´egy´eves doktori iskola (PhD-fokozat). Hogy mi´ert ´erdemes fizikatan´ar szakot v´alasztani? • 2022-t˝ ol u ´j rendszer˝ u, 5 ´eves osztatlan k´epz´es 2022 szeptember´eben indul az u ´j rendszer˝ u fizikatan´ ar-k´epz´es. Az u ´j rendszerben a k´epz´esi id˝o 5 ´ev, melynek teljes utols´o f´el´eve egy iskolai gyakorlat, de l´enyeg´eben minden f´el´evben lesz tan´ıt´asi gyakorlat. Az ELTE h´ arom gyakorl´oiskol´aja kiv´al´o terep a tan´ ari szakma komplex elsaj´ at´ıt´ as´ ara. A szakmai ´es m´ odszertani oz´episkolai tan´ arok tartt´argyakat az int´ezet kit˝ un˝ o kutat´oi-oktat´oi ´es a legjobb k¨ j´ak. A fizika szak mell´e az ELTE szinte b´ armilyen szakp´ art ind´ıt, ´ıgy k¨ onny˝ u m´asik szakot v´alasztani. • A tan´ari p´alya sz´eps´egei Napjainkban a fizikatan´ ar egy hi´ anyszakma, ez´ert biztos elhelyezked´est jelent azok sz´am´ara, akik szeretn´enek ezen a t´ arsadalmilag is fontos p´ aly´ an elhelyezkedni. Gyakorlatilag a 4. ´evt˝ ol hallgat´ oink jelent˝ os h´ anyada r´esz-, vagy f˝o´ all´asban tan´ıt. Nagy el˝onye a p´ aly´ anak, hogy a tan´ar nincs kit´eve a piac szesz´elyeinek, a versenyszf´era vissz´ass´ againak. A tan´ ari munka a csal´ adi ´elettel j´ol ¨ osszehangolhat´o munkabeoszt´ast (ny´ ari sz¨ unet, u altozatos ´all´ as, ¨nnepek stb.) jelent. Kreat´ıv ´es v´

42

K¨ oz´ episkolai Matematikai ´ es Fizikai Lapok, 2022/1

mely folyamatosan fiatalon tart. A pedag´ ogus el˝ omeneteli rendszer ´es b´erez´es megismer´es´ehez ´erdemes ´attekinteni a pedag´ogus ´eletp´ alyamodell”-t! ” • Hallgat´oi ´elet ¨ ond´ıj Program keret´eben egyetemistak´ent f´el´evente A Klebelsberg K´epz´esi Oszt¨ ak´ ar 375 000 Ft-ot lehet kapni, mely t¨ obb k¨ ul¨ onf´ele o¨szt¨ond´ıjjal is kieg´esz´ıthet˝ o. Fontos megeml´ıteni, hogy lehet˝os´eg van oktat´ assal kapcsolatos kutat´asokba val´ o becsatlakoz´ asra ´es doktori tanulm´anyok folytat´ as´ ara a Fizika Tan´ıt´ asa Doktori Program keret´eben. Tan´ arszakos hallgat´ok el˝omenetel´et mentorok seg´ıtik. A k´epz´esek r´eszleteir˝ ol az int´ezet honlapj´an (https://physics.elte.hu) lehet tov´ abbi inform´ aci´ okat szerezni, vagy ´erdemes ell´atogatni ny´ılt napunkra (https://ttk.elte.hu/nyiltnap2022).

hat´ o neh´ezs´egi er˝ ot.) Eset¨ unkben a p´ alca v´egein´el hat´ o er˝ onek az´ert kell a p´ alca ir´ any´ aba mutatnia, mert a p´ alca t¨ omege elhanyagolhat´ o, teh´ at a p´ alca fon´ alk´ent viselkedik”. ”

Az er˝ok f¨ ugg˝oleges komponensei kiegyenl´ıtik egym´ ast, a K er˝ o v´ızszintes ¨ osszetev˝oje biztos´ıtja a centripet´ alis gyorsul´ast. Ez egyenletekben kifejezve: (1)

K cos α = mg,

(2)

K sin α = mω 2 r.

A k¨orp´ alya sugar´at a p´ alca hossz´ aval ´es az α sz¨ oggel kifejezve r = sin α. Ezt ´ırjuk be a (2) egyenletbe, ´ıgy az egyenletrendszer a k¨ ovetkez˝ o alakot ¨ olti: K cos α = mg, K sin α = mω 2 sin α.

A k´ uping´ar´ ol∗

Egy hossz´ us´ag´ u, k¨ onny˝ u (elhanyagolhat´ o t¨ omeg˝ u), v´ekony (de merev) p´alca egyik v´eg´ere egy m t¨ omeg˝ u, kicsiny testet er˝ os´ıt¨ unk. A p´ alca m´ asik v´eg´et v´ızszintes tengellyel l´ atjuk el. Ily m´ odon egy matematikai ing´anak tekinthet˝o rendszert kapunk, amelynek a p´ alca f¨ ugg˝oleges a´ll´ as´ an´ al van stabil egyens´ ulyi helyzete, innen kicsit kimozd´ıtva 

T = 2π g

Az egyenletrendszernek az egyik nyilv´anval´ o (trivi´alis) megold´ asa: α = 0, a m´ asik megold´ as: g cos α = 2 .

ω Az els˝o megold´ as azt jelenti, hogy ha forgatni kezdj¨ uk az inga tengely´et, akkor az mindig lehets´eges, hogy az inga mindv´egig f¨ ugg˝oleges helyzetben l´ og. A m´ ag sik megold´ as csak akkor lehets´eges, ha cos α = ω2 < 1. Ez akkor nem teljes¨ ul, ha  g az ω sz¨ogsebess´eg viszonylag alacsony. Ha a forg´as sz¨ ogsebess´ege ω <  , az inga mindenk´eppen f¨ ugg˝olegesen fog l´ ogni. (A sz¨ ogsebess´eg k¨ usz¨ ob´ert´eke ´eppen a forgat´as n´elk¨ uli matematikai inga leng´esidej´ehez tartoz´ o 2π/T sz¨ ogsebess´eg”.) ” Ha az inga sz¨ ogsebess´ege a k¨ usz¨ ob´ert´ek f¨ ol´e n¨ ovekszik, az inga forg´asa az

leng´esidej˝ u harmonikus rezg˝ omozg´ast v´egezhet.

α = arccos

Vajon mi t¨ ort´enik, ha a p´ alca fels˝ o v´eg´en´el l´ev˝ o v´ızszintes tengelyt – egy f¨ ugg˝oleges r´ ud ´es egy kengyel seg´ıts´eg´evel – adott ω sz¨ ogsebess´eggel egyenletesen forgatjuk (1. a ´bra)? Az inga – ω nagys´ ag´ at´ ol f¨ ugg˝o m´ert´ekben – valamekkora sz¨ ogben kit´er¨ ul, ´es (´ alland´ osult ´allapotban) a p´alca egy k´ up pal´ astja ment´en mozog. Emiatt ezt az elrendez´est k´ uping´ anak is szokt´ ak nevezni.

g

ω 2

sz¨og mellett lesz stabil, az α = 0-nak megfelel˝o megold´ as pedig – mint l´atni fogjuk – instabill´a v´ alik. A stabilit´as vizsg´alata

1. ´ abra Vizsg´aljuk meg a k´ upinga mozg´as´ at, ´es hat´ arozzuk meg, hogy mik´ent f¨ ugg a p´ alca kit´er¨ ul´es´enek α sz¨oge az ω sz¨ogsebess´egt˝ ol! Az ingatestre k´et er˝ o hat: az mg neh´ezs´egi er˝ o, ´es a p´alca a´ltal kifejtett, a p´ alc´ aval megegyez˝o ir´ any´ u K er˝ o.

A p´alc´ aval egy¨ utt forg´o koordin´ ata-rendszerb˝ ol n´ezve az inga nehez´eke mozdulatlan, egyens´ ulyi ´allapotban van. Azt, hogy az egyens´ uly stabil vagy instabil, a potenci´alis energi´at megad´o f¨ uggv´eny vizsg´ alat´ aval d¨ onthetj¨ uk el. A stabil egyens´ uly a minim´ alis helyzeti energi´ aj´ u helyzetben val´ osul meg, az instabil helyzetre pedig az energia lok´alis maximuma jellemz˝ o. Az ingatest megemelked´ese miatt a neh´ezs´egi er˝ ob˝ol sz´armaz´ o helyzeti energi´ aja (ha ennek a 0-szintj´et az inga f¨ ugg˝oleges helyzet´ehez v´ alasztjuk – l´ asd a 2. ´ abr´ at):

Megjegyz´es. Egy p´ alca (r´ ud) a ´ltal´ aban nem csak p´ alca ir´ any´ u er˝ ot k´epes kifejteni. (P´eld´ aul egy v´ızszintes helyzet˝ u m´erleghinta ki tudja egyens´ ulyozni a v´eg´en u o gyerekre ¨l˝

E1 = mg (1 − cos α).



A cikk megtal´ alhat´ o honlapunkon, a fizika cikkek Ami a tank¨ onyvekb˝ ol kimaradt, ” de a versenyz˝ oknek hasznos lehet” r´esz´eben: https://www.komal.hu/cikkek/cikklista.h.shtml.

K¨ oz´ episkolai Matematikai ´ es Fizikai Lapok, 2022/1

43

Az inga forg´as´ ab´ ol is sz´armazik egyfajta helyzeti” energia. Ennek felt´erk´epe” z´es´ehez vizsg´aljuk meg, mennyi munk´ at kell v´egezn¨ unk, ha a m´ ar forg´asban l´ev˝ o ing´at a f¨ ugg˝oleges helyzetb˝ ol lassan az α sz¨ oggel jellemzett helyzetbe hozzuk. 44

K¨ oz´ episkolai Matematikai ´ es Fizikai Lapok, 2022/1

egy pozit´ıv ir´anyba eltolt parabola ´ıve. Ez´ert a m´asodfok´ u f¨ uggv´eny minimuma vagy az −mg

b g =− 1 x0 = − = 2 2 2 2a

ω 2 · mω

2

´ert´ekn´el, vagy pedig, ha ez az ´ert´ek kiesik az ´ertelmez´esi tartom´anyb´ol, akkor az ´ertelmez´esi tartom´any hat´ar´ an´ al (x = 1) van.

2. ´ abra

Ahhoz, hogy a tengelyt˝ol r t´ avols´ agban (a forg´ o rendszerb˝ol n´ezve) egyens´ ulyban tartsuk az ingatestet, a´lland´ o F (r) = mω 2 r er˝ ovel kell a tengely fel´e h´ uznunk. Ez az er˝o az r t´ avols´ aggal egyenesen ar´ anyos, ir´ anya pedig ellent´etes az elmozdul´ as ir´any´ aval. Az ´altalunk v´egzett munka az ´atlagos er˝ o ´es az elmozdul´as szorzata: W = −F´atlagos · r = −

1 mω 2 r + 0 1 · r = − mω 2 r2 = − mω 2 2 sin2 α. 2 2 2

(A negat´ıv el˝ ojel arra utal, hogy az F er˝ o ellent´etes ir´ any´ u az r elmozdul´ assal.) at Ahhoz, hogy visszah´ uzzuk az ingatestet a tengelyhez, + 12 mω 2 2 sin2 α munk´ kell v´egezn¨ unk. (Ha o´vatosan elengedj¨ uk az ingatestet, ugyanekkora munk´ at nyerhet¨ unk vissza, am´ıg r t´ avols´ agra jut az ingatest a tengelyt˝ ol.) Elmondhatjuk teh´ at, hogy az ingatestnek a tengelyt˝ol r t´avols´ agban a forg´as k¨ovetkezt´eben 1 E2 = − mω 2 2 sin2 α 2 helyzeti energi´ aja van. Az ingatest f¨ ugg˝oleges ´es v´ızszintes helyzet´eb˝ ol fakad´ o potenci´alis energi´aj´anak osszege: ¨

3. ´ abra

Ha x0  1, vagyis az ´ertelmez´esi tartom´anyon bel¨ ul van (3a. ´ abra), l´etezik g at stabil egyens´ ulyt (S) tal´ alunk. olyan α0 sz¨og, amelyre x0 = cos α0 = ω2 . Itt teh´ x = 1-n´el (α = 0-n´ al), ami pedig energiamaximum, instabil (I) egyens´ uly van. Ha asodfok´ u kifejez´est a´br´ azol´o parabola cs´ ucsa az ´ertelmez´esi tarx0 > 1, akkor a m´ tom´anyon k´ıv¨ ulre esik (3b. a ´bra), ez´ert E(x) minimuma x = 1-hez ker¨ ul, teh´ at ez v´alik stabil egyens´ ully´ a.

1 E(α) = E1 + E2 = W = mg (1 − cos α) − mω 2 2 sin2 α, 2 ag felhaszn´al´as´ aval ´ıgy is fel´ırhatunk: amit a sin2 α = 1 − cos2 α azonoss´ E(α) =

1 1 mω 2 2 cos2 α − mg cos α + mg − mω 2 2 . 2 2

Ez a kifejez´es x ≡ cos α-ra m´ asodfok´ u: E(x) = ax2 + bx + x alak´ u, ´es fontos szem el˝ott tartanunk, hogy a f¨ uggv´eny csak a (0; 1] intervallumon van ´ertelmezve, hiszen og koszinusza ebbe a tartom´anyba eshet. A f¨ uggv´eny k´epe egy −90◦ < α < 90◦ sz¨ K¨ oz´ episkolai Matematikai ´ es Fizikai Lapok, 2022/1

45

4. ´ abra

46

K¨ oz´ episkolai Matematikai ´ es Fizikai Lapok, 2022/1

A fent le´ırtak j´ ol l´atszanak, ha az energi´ at k¨ ozvetlen¨ ul az α sz¨og f¨ uggv´eny´eben abr´ ´ azoljuk (4. ´ abra). ´ azoljuk az inga egyens´ Abr´ ulyi helyzeteit jellemz˝o α sz¨oget az ω sz¨ogsebess´eg f¨ uggv´eny´eben! Egy koszinusz´ert´ekhez k´et sz¨ og tartozik, melyek egym´as ellentettjei (5. ´ abra).

Megold´as. K´etf´ele technik´ aval ´es h´ arom k¨ ul¨ onb¨ oz˝ o anyaggal ¨ osszesen n´egy m´er´est v´egezt¨ unk. El˝ osz¨ or a vizsg´aland´ o szemcs´es anyagot egy j´ ol meghat´arozott szintig m´er˝ opoh´arba t¨ olt¨ ott¨ uk, majd a tov´abbiakban k´etf´ele m´ odszert alkalmaztunk. I. Vizes m´ odszer. A szemcs´es anyaggal megt¨ olt¨ ott m´er˝ opoharat felt¨ oltj¨ uk v´ızzel, addig, hogy ´epphogy ellepje az anyagot. Ek¨ ozben konyhai m´erleggel m´erj¨ uk a bet¨olt¨ott v´ız t¨ omeg´et, amelyb˝ ol kisz´am´ıthatjuk a keresett t´erfogatot. II. Lisztes m´ odszer. A m´er˝ opoh´arb´ ol a´t¨ ontj¨ uk az anyagot egy m´ asik t´ alba, om¨ or massza legyen. (Felt´etelezz¨ uk, majd o¨sszekeverj¨ uk liszttel, u ´gy, hogy egy t¨ hogy a massza m´ar nem tartalmaz sz´amottev˝ o mennyis´eg˝ u leveg˝ ot, mert a szemcs´ek k¨oz¨otti u olti ki.) Ezut´ an vissza¨ ontj¨ uk a kever´eket a m´er˝ opoh´arba, ¨regeket a liszt t¨ ´es megn´ezz¨ uk, most most milyen t´erfogat´ u. Ezt k¨ ovet˝oen az anyagot ´es a lisztet sz´etv´alasztjuk szit´ aval, ´es a liszt t´erfogat´ at megm´erj¨ uk a m´er˝ opoh´ar seg´ıts´eg´evel. Ha a liszt t´erfogat´ anak ´es a szemcs´es anyag kezdeti t´erfogat´ anak o ol levonjuk ¨sszeg´eb˝ a lisztes kever´ek t´erfogat´ at, megkapjuk a szemcs´ek k¨ oz¨ otti leveg˝o t´erfogat´ at.

5. ´ abra

L´athat´ o, hogy egy darabig csak a f¨ ugg˝oleges helyzet stabil, majd egy ω0 ´ert´ekn´el ez a helyzet instabill´a v´ alik, ´es megjelenik k´et m´ asik stabil egyens´ ulyi helyzet. A stabil a´llapot hirtelen k´etszerez˝ od´es´et, k´et ´agra szakad´ as´at bifurk´ aci´ onak nevezik. ´ Megjegyz´esek. 1. Erdekess´ eg, hogy a bifurk´ aci´ o kezd˝ opontj´ aban, vagyis az ω0 ´ert´ekn´el a f¨ uggv´eny gr´ afj´ anak meredeks´ege v´egtelen, vagyis az ´erint˝ o f¨ ugg˝ oleges”. ” 2. A stabilit´ as vizsg´ alat´ at dinamikailag is el lehet v´egezni, ami a jelen esetben sokkal egyszer˝ ubb, mint az energetikai vizsg´ alat, de ennek a cikknek az egyik c´elja az volt, hogy bemutassa az energetikai megfontol´ as lehet˝ os´eg´et.

1. m´er´es. 100 ml s´ argabors´ ot a vizes technik´ aval fel¨ ont¨ unk v´ızzel. A v´ız t´erfogata 25 ml, ´ıgy a keresett leveg˝o t´erfogata is 25 ml. 2. m´er´es. 100 ml s´ argabors´ ot a lisztes technik´aval ¨ osszevegy´ıt¨ unk liszttel. Egy¨ uttes t´erfogatuk a m´er´es ut´ an 150 ml, a liszt t´erfogata ¨ onmag´aban 70 ml. A liszt ´es a s´ argabors´ o egy¨ uttes t´erfogat´ ab´ ol levonjuk az ¨ oszegy´ urt bors´ o-liszt kever´ek t´erfogat´ at, az eredm´eny: 70 + 100 − 100 = 20, teh´at a keresett leveg˝o t´erfogata 20 ml. 3. m´er´es. 200 ml mung´obabot m´erlegen fel¨ ont¨ unk v´ızzel. A v´ız t¨ omege 62 g, t´erfogata 62 ml, teh´ at a leveg˝ o t´erfogata is ennyi.

3. A cikkben le´ırtakhoz hasonl´ o probl´em´ aval foglalkozik a 2021–2022-es tan´evi fizika OKTV 1. fordul´ oj´ an az I. kateg´ oria 1. feladata is.

Baranyai Kl´ara Veresegyh´az

M´er´esi feladatok megold´asa

M. 403. A kereskedelemben kaphat´ o n´eh´ any szemcs´es anyag eset´eben (pl.: lencse, rizs, tarhonya stb.) m´er´essel hat´ arozzuk meg, hogy t´ arol´ asi t´erfogatuk h´ any sz´ azal´eka leveg˝ o! (6 pont) K¨ oz´ episkolai Matematikai ´ es Fizikai Lapok, 2022/1

K¨ozli: Zsigri Ferenc, Budapest 47

48

K¨ oz´ episkolai Matematikai ´ es Fizikai Lapok, 2022/1

4. m´er´es. Megism´etelj¨ uk a 3. m´er´est u ´j, eddig nem haszn´ alt mung´obabbal. A v´ız bet¨ olt´ese ut´ an a m´erleg 63 g-mal t¨ obbet mutat, mint kor´ abban, teh´at 63 ml volt a babszemek k¨ oz¨otti leveg˝o t´erfogata. M´er´esek anyagok

technika

s´ argabors´ o s´ argabors´ o mung´ obab mung´ obab

v´ızzel liszttel v´ızzel v´ızzel

a szemcs´es anyag kezdeti t´erfogata 100 ml 100 ml 200 ml 200 ml

a leveg˝ o t´erfogata 25 ml 20 ml 62 ml 63 ml

a leveg˝ o ar´ anya 25% 20% 31% 31,5%

2. A m´er´es elve ´es menete A m´er´es sor´ an a csapot teljesen megnyitva el˝ osz¨ or hideg, majd forr´o, ezut´an oed´enyt, mik¨ ozben m´erj¨ uk pedig langyos v´ızzel telet¨ oltj¨ uk a V = 2 dm3 -es m´er˝ az id˝ot. Miut´ an az ed´eny megtelt, megm´erj¨ uk a v´ız h˝ om´ers´eklet´et a magh˝ om´er˝ ovel. Ezt minden esetben h´ aromszor v´egezz¨ uk el, majd az eredm´enyeket t´ abl´ azatba foglaljuk ´es kisz´am´ıtjuk a k´ert adatokat.

A m´er´esi hib´ ak lehets´eges okai: – a m´er˝ opoh´ar jelz´es´enek pontatlan leolvas´ asa; – a szemcs´es anyagb´ ol nem tudunk s´ık fel¨ uletet kialak´ıtani a m´er˝ opoh´ar adott jelz´es´en´el; – az ´ att¨ olt¨ oget´esn´el fell´ep˝ o mell´e¨ ont´esek”; ” – a m´ert anyag v´ızfelv´etele; – a v´ız p´ arolg´asa; – a liszt szemcsem´eret´eb˝ ol ad´od´o nem teljes t´erkit¨olt´es; – a liszt egy r´esze r´ atapad a m´er˝ opoh´arra, a vizsg´alt anyag szemcs´eire, a szit´ ara ´es egy´eb t´ alakra; – a liszt a´t¨ ontet´es´en´el megfigyelhet˝ o porz´ as.

3. M´er´esi eredm´enyek ´es sz´ amol´ as Hideg v´ız m´er´esek 1. 2. 3. atlag ´

(A felsorolt hibaforr´ asokb´ ol neh´ez lenne sz´amszer˝ u hibabecsl´est adni.) ´ ´ ad Gimn., 11. ´evf.) ´es D´ ozsa Levente (Obudai Arp´ ´ ´ ad Gimn., 11. ´evf.) Arp´ Szalai Henrietta (Obudai m´er˝ op´ar dolgozata alapj´an

Qhideg =

M. 405. M´erj¨ uk meg egy kever˝ ocsaptelep v´ızhozam´ at el˝ osz¨ or u ´gy, hogy a csapb´ ol hideg v´ız folyj´ek, majd u ´gy is, ha forr´ o v´ız folyik a csapb´ ol! M´erj¨ uk meg a hideg ´es a forr´ o v´ız h˝ om´ers´eklet´et is. V´eg¨ ul m´erj¨ uk meg a csaptelep v´ızhozam´ at langyos v´ız eset´eben is, ´es sz´ am´ıtsuk ki, hogy a langyos v´ızhozam h´ anyad r´esz´et adja a hideg v´ız, illetve h´ anyad r´esz´et adja a forr´ o v´ız!

K¨ oz´ episkolai Matematikai ´ es Fizikai Lapok, 2022/1

2,0 dm3 dm3 V = = 0,16 . t 12,8 s s

Forr´ o v´ız m´er´esek 1. 2. 3. atlag ´

K¨ ozli: Honyek Gyula, Veresegyh´az

Megold´as. 1. A felhaszn´ alt eszk¨ oz¨ ok – k´etliteres m´er˝ oed´eny, – f¨ urd˝oszobai csap, – stopper´ ora, – magh˝om´er˝ o.

h˝ om´ers´eklet [◦ C] 24,0 23,4 22,9 23,4

A v´ızhozam:

6 dolgozat ´erkezett. Helyes 5 megold´ as. Kicsit hi´ anyos (4 pont) 1 dolgozat.

(6 pont)

id˝ o [s] 12,6 12,7 13,1 12,8

id˝ o [s] 13,4 13,3 13,6 13,4

h˝ om´ers´eklet [◦ C] 44,2 45,3 45,4 45,0

A v´ızhozam: Qforr´o =

49

50

2,0 dm3 dm3 V = = 0,15 . t 13,4 s s

K¨ oz´ episkolai Matematikai ´ es Fizikai Lapok, 2022/1

Langyos v´ız m´er´esek 1. 2. 3. atlag ´ A v´ızhozam: Qlangyos =

id˝ o [s] 10,5 9,8 10,2 10,2

a saj´at hossz´ us´ aguk. Ha a sebess´egeket m/s, az id˝ ot m´asodperc, a t´ avols´ agokat pedig m´eter egys´egekben m´erj¨ uk, akkor a k¨ ovetkez˝ ok teljes¨ ulnek:

h˝ om´ers´eklet [◦ C] 29,9 30,4 31,6 30,6

(1)

1 + 600 = 40 v1 ,

(2)

2 + 600 = 100 v2 .

Ha a k´et szerelv´eny egym´ assal szemben halad el, a mozdonyok elej´enek tal´ alkoz´ asakor a vonatok v´egei 1 + 2 t´ avols´ agra vannak egym´ ast´ ol. Ezek a vonatv´egek” ” ozelednek egym´ ashoz ´es 20 m´asodperc m´ ulva tal´ alkoznak, felv1 + v2 sebess´eggel k¨ ´ırhat´o teh´at a k¨ ovetkez˝ o¨ osszef¨ ugg´es:

2,0 dm3 dm3 V = = 0,20 . t 10,2 s s

(3) Jel¨ olj¨ uk a langyos v´ız eredetileg forr´ o¨ osszetev˝ oj´enek t¨omeg´et mforr´o -val, az eredetileg hideg ¨ osszetev˝ oj´enek t¨ omeg´et pedig mhideg -gel. A kalorimetrikus egyenlet szerint c mhideg (Tlangyos − Thideg ) = c mforr´o (Tforr´o − Tlangyos ), ahonnan a keresett t¨ omegar´ any:

1 + 2 = 20 (v1 + v2 ). Az (1)–(3) egyenletekb˝ol 1 -et, 2 -t ´es v1 -et kifejezhetj¨ uk v2 seg´ıts´eg´evel:

(4)

v1 = 60 − 4 v2 ,

1 = 1800 − 160 v2

´es

2 = 100 v2 − 600.

Amikor a vonatok azonos ir´anyban haladnak, a szerelv´enyek tal´ alkoz´ asakor az el¨ol halad´ o szerelv´eny mozdony´ anak eleje ´es a h´ atul halad´ o szerelv´eny v´ege

1 + 2 t´avols´ agra van egym´ast´ ol. Ezek a pontok most |v1 − v2 | sebess´eggel k¨ ozelednek egym´ashoz ´es 60 m´ asodperc m´ ulva tal´ alkoznak. Fenn´all teh´ at, hogy

mhideg Tforr´o − Tlangyos 45,0 − 30,6 = 2,0. = = mforr´o Tlangyos − Thideg 30,6 − 23,4 Eszerint (az adott csap´ all´ asn´ al) a langyos v´ızhozam 67%-´ at adta a hideg v´ız, 33%-´ at pedig a forr´ o v´ız. Jeszen˝ oi S´ ara (Kecskem´et, Katona J. Gimn., 10. ´evf.) 6 dolgozat ´erkezett. 6 pontot kapott 2 megold´ as. Hi´ anyos (4 pont) 3, nem ´ert´ekelhet˝ o 1 dolgozat.

(5)

1 + 2 = 60 |v1 − v2 |.

Ha (5)-ben minden ismeretlen hely´ere a v2 -t tartalmaz´ o kifejez´est helyettes´ıtj¨ uk be, a k¨ovetkez˝ ot kapjuk: (6)

20 − v2 = |60 − 5 v2 |.

A tov´ abbiakban k´et lehet˝ os´eget vizsg´ alunk. Amennyiben 60  5 v2 , vagyis v2  12, akkor (6) szerint

Fizika feladatok megold´asa

20 − v2 = 60 − 5 v2 ,

P. 5337. P´ arhuzamos p´ aly´ akon a ´lland´ o sebess´eggel k¨ ozlekedik k´et tehervonat. Ellent´etes ir´ anyban haladva 20 s alatt, azonos ir´ anyban haladva pedig 60 s alatt haladnak el egym´ as mellett. Egy 600 m hossz´ u h´ıdon az egyik szerelv´eny 40 s alatt, a m´ asik 100 s alatt halad ´ at. Hat´ arozzuk meg a szerelv´enyek hossz´ at ´es sebess´eg´et! (4 pont)

K¨ozli: Sz´ekely Gy¨ orgy, Budapest

aga v1 , Megold´as. Legyen az els˝o szerelv´eny hossza 1 , sebess´eg´enek nagys´ a m´ asik szerelv´eny hossza 2 , sebess´eg´enek nagys´ aga pedig v2 . A 600 m hossz´ us´ag´ u h´ıdon ´athaladva annyival t¨ obb utat kell megtenni¨ uk a h´ıd hossz´an´al, amekkora K¨ oz´ episkolai Matematikai ´ es Fizikai Lapok, 2022/1

51

vagyis

v2 = 10,

tov´ abb´ a (4)-b˝ ol a v1 = 20, 1 = 200 ´es 2 = 400 eredm´eny ad´ odik. Ha 60 − 5v2 negak¨ o vetkezik. Ehhez a sebess´egt´ıv, akkor (6)-b´ ol 20 − v2 = 5 v2 − 60, vagyis v2 = 40 3

hez 1 = − 1000 < 0 vonathossz tartozna, ami nyilv´ an lehetetlen, emiatt a 60 < 5 v2 3 lehet˝ os´eget ki kell z´ arnunk.

A feladat megold´ asa teh´ at az, hogy az egyik szerelv´eny hossza 1 = 200 m m km ´es v1 = 20 s = 72 h sebess´eggel halad, a m´asik tehervonat hossza 2 = 400 m, sebess´ege pedig v2 = 10 m = 36 km . s h

Egyh´ azi Hanna (Budapest, ELTE Ap´ aczai Csere J. Gyak. Gimn., 12. ´evf.) 104 dolgozat ´erkezett. Helyes 72 megold´ as. Kicsit hi´ anyos (3 pont) 12, hi´ anyos (1–2 pont) 18, hib´ as 2 dolgozat.

52

K¨ oz´ episkolai Matematikai ´ es Fizikai Lapok, 2022/1

P. 5339. V´ızszintes, s´ url´ od´ asmentes fel¨ uleten egy L = 0,6 m hossz´ us´ ag´ u, elhanyolhat´ o t¨ omeg˝ u, v´ekony r´ ud fekszik. A r´ ud v´egpontjaihoz elhanyagolhat´ o t¨ omeg˝ u, feszes fonalakkal m = 0,2 kg ´es M = 0,8 kg t¨ omeg˝ u testeket r¨ ogz´ıtett¨ unk. A fonalak mer˝ olegesek a r´ udra. Egy adott pillanatban a r´ ud k¨ oz´eppontj´ ara a v´ızszintes fel¨ ulettel p´ arhuzamos, a r´ udra mer˝ oleges, F = 8 N nagys´ ag´ u er˝ ot fejt¨ unk ki. a) Hat´ arozzuk meg a kezd˝ opillanatban a r´ ud k¨ oz´eppontj´ anak gyorsul´ as´ at! b) A r´ ud melyik pontj´ ara kellene kifejteni ezt az F er˝ ot, hogy a testek gyorsul´ asa azonos legyen? Mekkora er˝ ok ´ebrednek ekkor a fonalakban? (4 pont)

b) Egy alkalmasan v´alasztott T pontban hat´ o F er˝ o hat´ as´ ara a r´ ud nem fordul el, teh´ at az eg´esz rendszer a gyorsul´assal mozog (2. ´ abra). Ilyenkor F1 = M a ´es F2 = ma, vagyis M F1 = 4, = F2 m tov´ abb´ a F1 + F2 = F = 8 N, teh´at

K¨ozli: Kotek L´ aszl´ o, P´ecs

F1 = 6,4 N

Megold´as. a) Ha a r´ ud t¨ omege (´es ´ıgy a tehetetlens´egi nyomat´eka is) elhanyagolhat´ o, akkor a kezd˝opillanatban a r´ udra hat´ o er˝ ok ered˝oje, ´es a forgat´ onyomat´ekok ered˝ oje is nulla kell hogy legyen (1. ´ abra): F = F1 + F 2 ,

illetve

ahonnan F1 = F 2 =

F1

L L = F2 , 2 2

F2 = 1,6 N.

A T pont hely´et a forgat´ onyomat´ekok egyens´ uly´ ab´ ol kapjuk meg: F1 BT = = 4, AT F2 m´ asr´eszt AT + BT = AB = 60 cm, ahonnan AT = 12 cm ´es BT = 48 cm. ´ Isk. ´es Gimn., 9. ´evf.) Juh´ asz-Moln´ ar Erik (Budapesti Fazekas M. Gyak. Alt.

F =4N 2

43 dolgozat ´erkezett. Helyes 10 megold´ as. Kicsit hi´ anyos (3 pont) 1, hi´ anyos (1–2 pont) 29, hib´ as 3 dolgozat.

k¨ ovetkezik. A r´ ud v´egeihez r¨ogz´ıtett testek gyorsul´asa: 4N m F1 a1 = = = 5 2, M 0,8 kg s illetve

4N m F2 = = 20 2 . m 0,2 kg s A r´ ud K geometriai k¨ oz´eppontj´anak (ami nem egyezik meg a rendszer t¨omegk¨oz´eppontj´aval) a gyorsul´asa az a1 ´es a2 gyorsul´as sz´amtani k¨ozepe: a2 =

aK =

´es

P. 5344. Egyenl˝ o sz´ ar´ u h´ aromsz¨ og keresztmetanak t¨ or´esmutaszet˝ u prizma t¨ or˝ osz¨ oge 40◦ , anyag´ t´ oja 1,6. Mekkora α bees´esi sz¨ oggel ´erkezik a f´enysug´ ar az egyik oldallaphoz, ha ez a f´enysug´ ar a prizm´ aban az alappal p´ arhuzamosan halad tov´ abb? Mekkora n t¨ or´esmutat´ oja van annak az u ol k´e¨vegnek, amib˝ sz¨ ult has´ abot a prizma m´ asik oldal´ ahoz illesztve a t¨ or´esi sz¨ og az ´abra szerint α/2? (4 pont)

m a1 + a2 = 12,5 2 . 2 s

K¨ ozli: Kobzos Ferenc, Duna´ ujv´ aros Megold´as. Az ´ abr´ an l´ athat´ o jel¨ ol´eseket haszn´ alva (´es a leveg˝ o abszol´ ut t¨ or´esmutat´ oj´at j´o k¨ ozel´ıt´essel 1-nek tekintve) meg´allaabb´ a a t¨ op´ıthatjuk, hogy β = γ = 20◦ , tov´ r´esi t¨ orv´eny szerint sin α sin α = = 1,6, sin β sin 20◦ ahonnan

1. a ´bra

K¨ oz´ episkolai Matematikai ´ es Fizikai Lapok, 2022/1

sin α = 0,547,

2. a ´bra

53

54

teh´ at

α = 33,2◦ .

K¨ oz´ episkolai Matematikai ´ es Fizikai Lapok, 2022/1

A prizm´ ab´ ol az u abba l´ep˝ o f´enyre a t¨ or´esi t¨orv´eny ´ıgy ´ırhat´ o: ¨veghas´

teljes¨ ul. Az (1) ´es (2) egyenlet szorzat´ ab´ ol

sin γ sin 20◦ nhas´ab n = , = 1,20 = nrelat´ıv = = sin(α/2) sin 16,6◦ nprizma 1,6

gtr sin α = v 2 t sin α · cos α ad´odik. Mivel a t´enyleges ferde haj´ıt´ asn´ al t = 0 ´es α = 0, t sin α-val egyszer˝ us´ıthet¨ unk:

ahonnan a has´ ab anyag´ anak abszol´ ut t¨ or´esmutat´ oja: n = 1,6 · 1,2 = 1,92.

v2 =

(3)

Elekes Dorottya (Budapest, Fasori Evang´elikus Gimn., 9. ´evf.) 55 dolgozat ´erkezett. Helyes 37 megold´ as. Kicsit hi´ anyos (3 pont) 4, hi´ anyos (1–2 pont) 13, hib´ as 1 dolgozat.

P. 5345. V´ekony cs˝ ob˝ ol k´et R sugar´ u negyedk¨ ort k´esz´ıt¨ unk, majd egyegy r sugar´ u, α sz¨ oggel hi´ anyos” f´el” k¨ or´ıvet csatlakoztatunk hozz´ ajuk, v´eg¨ ul az eg´esz elrendez´est az ´ abr´ an l´ athat´ o m´ odon egy f¨ ugg˝ oleges s´ıklaphoz er˝ os´ıtj¨ uk. Az A pontb´ ol kezd˝ osebess´eg n´elk¨ ul beejt¨ unk egy kis goly´ ot a cs˝ obe. A goly´ o az AB ´es a BC k¨ or´ıven v´egigcs´ uszik, a C ´es a D pont k¨ oz¨ ott szabadon esik (ferde haj´ıt´ as szerint mozog), majd a DB ´es BE k¨ or´ıven cs´ uszik tov´ abb.(A s´ url´ od´ ast ´es a l´egellen´ all´ ast figyelmen k´ıv¨ ul hagyhatjuk.) = 52 ? a) Mekkora az α sz¨ og, ha R r b) Vizsg´ aljuk meg, hogy k¨ ul¨ onb¨ oz˝ o R ar´ anyokn´ al mekkora α sz¨ og (vagy sz¨ ogek) r eset´eben val´ osulhat meg a le´ırt mozg´ as! (6 pont)

Rom´ aniai versenyfeladat nyom´ an

Megold´as. Hat´ arozzuk meg, hogy egy adott α sz¨ogn´el mekkora v sebess´eggel kell rendelkezzen a goly´ o a C pontban, hogy ´eppen eljusson a D pontig. (Ha ez megt¨ ort´enik, akkor – az elrendez´es szimmetri´aja miatt – a D pontbeli sebess´eg´enek ir´ anya is megfelel˝ o lesz.) A v sebess´eg ismeret´eben – az energiamegmarad´ as t¨ orv´eny´et alkalmazva – meg tudjuk mondani, hogy milyen magasr´ ol indult a kis goly´ o, vagyis hogy mekkora az R sug´ar nagys´ aga. (Ha gondolatban megn¨ovelj¨ uk a v sebess´eget, akkor a goly´ o t´ ul fog rep¨ ulni a D ponton, ha pedig cs¨okkentj¨ uk, akkor nem fog elrep¨ ulni od´ aig, teh´at az α sz¨ ogb˝ ol egy´ertelm˝ uen meg tudjuk mondani a v sebess´eget, majd abb´ ol a mozg´as kezd˝opontj´anak magass´ ag´at.)

ami – az energiamegmarad´ as t¨ orv´enye szerint – annyit jelent, hogy az A ´es C pontok magass´ ag´anak k¨ ul¨ onbs´ege r v2 = . h= 2g 2 cos α M´asr´eszt igaz, hogy R = r + r cos α + h, teh´at

Szorozzuk meg (4)-et



2-vel, ´es vezess¨ uk be az √ R −1 , illetve x= 2 r

b=

√ 2 cos α

jel¨ol´eseket. Ezekkel 1 x=b+ , b vagyis b2 − xb + 1 = 0,

(5) aminek megold´ asa: (6)

b1,2 =





x2 − 4 . 2

L´atjuk, hogy a feladatnak csak x  2, vagyis R 1+ r re´ alis megold´ asa.



2 eset´en lehet fizikailag

= 52 , akkor x = √3 , teh´at a) Ha R r 2 b1,2 =

2r sin α = v cos α · t,

a f¨ ugg˝oleges ir´ any´ u mozg´asra pedig (2)

1 R − 1 = cos α + . r 2 cos α

(4)

Legyen a C ´es D pont k¨ oz¨ otti ferde haj´ıt´ as mozg´as´anak id˝otartama t. A v´ızszintes ir´any´ u elmozdul´ asra (1)

rg , cos α

3±1 √ , 2 2

vagyis gt = v sin α 2

K¨ oz´ episkolai Matematikai ´ es Fizikai Lapok, 2022/1

cos α1 = 1 55

56

´es

cos α2 =

1 , 2

K¨ oz´ episkolai Matematikai ´ es Fizikai Lapok, 2022/1

´es ennek megfelel˝ oen α1 = 0◦ ,

illetve

α2 = 60◦ .

Fizik´ab´ ol kit˝ uzo¨tt feladatok

Az els˝o megold´ as” nem felel meg t´enyleges haj´ıt´ asnak, azt elvethetj¨ uk. ” oleges, ´es vizsg´aljuk meg a le´ırt mozg´asnak megfelel˝o b) Legyen most Rr tetsz˝ α sz¨ og lehets´eges ´ert´ekeinek sz´ am´at. √ (i) A (6) ¨ osszef¨ ugg´es azt mutatja, hogy Rr < 1 + 2 eset´en α egyetlen ´ert´eke sem megfelel˝ o, vagyis a feladatnak nincs megold´ asa. √ R asa, esetleg t¨ obb is. (ii) Amennyiben r  1 + 2, a feladatnak lehet megold´ A legkisebb, val´os megold´asra vezet˝ o x = 2 ´ert´ekn´el, vagyis a sz´ oba j¨ohet˝ o legkisebb R/r ar´ anyn´al (6) szerint b1 = b2 = 1,

1 cos α = √ , 2

α = 45◦ .

(6 pont)

√ 2 eset´en egyetlen α sz¨ ogn´el val´ osulhat meg a le´ırt mozg´as. √ (iii) Ha R > 1 + 2 , akkor (6)-nak k´et k¨ ul¨ onb¨oz˝ o megold´ asa van. Ezek csak r √ akkor felelnek meg fizikailag ´ertelmezhet˝ o α sz¨ ogeknek, ha cos α  1, vagyis b  2 . Ez a m´asodfok´ u egyenlet kisebbik gy¨ ok´ere biztosan teljes¨ ul, hiszen (a gy¨ok¨ok ´es az egy¨ utthat´ ok o ugg´ese szerint) b1 b2 = 1, azaz a kisebbik gy¨ok nem lehet ¨sszef¨ nagyobb 1-n´el. A feladatnak ebben az R/r tartom´ anyban legal´ abb egy megold´ asa van. √ K´et k¨ ul¨ onb¨ oz˝ o α sz¨ oget akkor kapunk, ha (6) nagyobbik gy¨oke 2-n´el kisebb: Ezek szerint R =1+ r

x+

M. 410. Ha egy kis m´eret˝ u, er˝os m´agnes ´es egy v´ızszintes helyzet˝ u gemkapocs k¨oz´e egy k´ artyalapot helyez¨ unk, akkor a k´ arty´ an´ al fogva m´eg fel tudjuk emelni a gemkapcsot. M´erj¨ uk meg, h´ any darab egym´asra rakott k´ artyalap kell ahhoz, hogy m´ar ne tudjuk felemelni a gemkapcsot! Mekkora ezen egym´asra helyezett lapok vastags´ aga? Csatlakoztassunk egym´ ashoz k´et ugyanolyan kis m´ agnest, ´es vizsg´aljuk meg, h´ any k´ artyalap sz¨ uks´eges ahhoz, hogy a gemkapcsot m´ ar ne tudjuk felemelni! K¨ ozli: Sz´ asz Kriszti´ an, Budapest

G. 765. A k´epsorozat mind a 11 felv´etele ugyanarr´ol a helyr˝ol k´esz¨ ult, a f´enyk´epez˝ og´epet mindig a Nap fel´e ford´ıtott´ ak. A k´epek id˝ orendben balr´ ol jobbra k´esz¨ ultek. Mennyi id˝ot mutatott az o´ra, amikor a Nap k´epe legk¨ ozelebb volt a horizonthoz? Milyen ´egt´ aj fel´e fordult a kamera, amikor a Nap a legalacsonyabban j´ art az ´egen? Hol ´es milyen ´evszakban k´esz¨ ult a k´epsorozat?



x2 − 4 √ < 2, 2

vagyis

Ez

 √ x2 − 4 < 2 2 − x,

azaz 3 x< √ , 2

√  √ 2 x2 − 4 < 2 2 − x = x2 − 4 2 x + 8. vagyis

(4 pont)

5 R < r 2

eset´en teljes¨ ul.

√ Ha teh´at 1 + 2 < R < 52 , akkor k´et k¨ ul¨ onb¨ oz˝ o, R  52 esetben pedig csak egy r r megold´ asunk van. ´ Isk. ´es Gimn., 11. ´evf.) Fey D´ avid (Budapesti Fazekas M. Gyak. Alt. dolgozata alapj´an 53 dolgozat ´erkezett. Helyes 19 megold´ as. Kicsit hi´ anyos (5 pont) 3, hi´ anyos (1–2 pont) 29, hib´ as 2 dolgozat.

K¨ oz´ episkolai Matematikai ´ es Fizikai Lapok, 2022/1

57

G. 766. A fizika legh´ıresebb k´eplete a t¨ omeg ´es az energia egyen´ert´ek˝ us´eg´et osszef¨ ugg´es, ahol E az energia, m a t¨ omeg ´es c a v´akuumbekifejez˝o E = mc2 ¨ li f´enysebess´eg. Ennek felhaszn´ al´as´ aval becs¨ ulj¨ uk meg, hogy saj´ at mobiltelefonunk mennyivel nehezebb teljesen felt¨ olt¨ ott akkumul´ atorral ahhoz k´epest, mintha az akkumul´ator teljesen lemer¨ ult a´llapotban lenne! (3 pont) G. 767. Az Esthajnalcsillag (val´ oj´aban a V´enusz bolyg´ o) egy ideig est´enk´ent l´atszik, azut´an egy ideig csak hajnalonk´ent l´athat´ o. Mennyi ennek a v´ altoz´ asnak a peri´odusideje? (4 pont) 58

K¨ ozli: Rakovszky Andor´ as, Budapest K¨ oz´ episkolai Matematikai ´ es Fizikai Lapok, 2022/1

G. 768. Sz´ az ´eg˝ ob˝ ol ´all´ o, sorosan kapcsolt kar´acsonyfa-f¨ uz´er egyik volfr´amsz´ alas izz´ oj´anak ´aram-fesz¨ ults´eg ¨ osszef¨ ugg´es´et l´ athatjuk az ´ abr´ an.

P. 5374. A k´epen egy sorozatl¨ ov˝ o, rug´os j´at´ekpuska l´athat´ o, ami hat darab, v´ekony, henger alak´ u szivacsl¨ oved´eket k´epes kil˝oni. Minden egyes l¨ ov´es el˝ ott a fekete cs´ uszk´ at jobbra el kell h´ uzni u oz´esig, nagyj´ ab´ ol ¨tk¨ 10 cm-re. A puska felh´ uz´ as´ ahoz sz¨ uks´eges maxim´alis er˝ or˝ ol egy digit´alis tests´ ulym´erleg seg´ıts´eg´evel azt tal´ altuk, hogy ez az er˝o 6,6 kg-os t¨ omeg s´ uly´ anak felel meg. a) Hogyan t¨ ort´enhetett az er˝o meghat´ aroz´ asa, ha a m´erlegen k´ıv¨ ul semmilyen seg´edeszk¨ozt nem kellett ig´enybe venni? b) Becs¨ ulj¨ uk meg, hogy maxim´alisan mekkora sebess´eggel rep¨ ul ki a 3 g t¨ oodik a l¨ oved´ek meg˝ u szivacsl¨oved´ek, ha a rug´o o ¨sszes energi´aj´anak 10%-a ford´ıt´ gyors´ıt´as´ara! (4 pont)

K¨ ozli: Honyek Gyula, Veresegyh´az

P. 5375. S´ url´od´ asmentes, v´ızszintes s´ıkon fekv˝o v´ekony, homog´en p´ alca egyik v´eg´et hirtelen u ´gy u uk meg, hogy a v´egpont sebess´ege a p´ alc´ ara mer˝ oleges ´es v ¨tj¨ nagys´ ag´ u legyen. a) A p´ alc´ anak melyik r´esze lesz z´erus kezd˝ osebess´eg˝ u? a) A grafikon alapj´an ´ allap´ıtsuk meg, hogy ¨ osszesen mekkora elektromos teljes´ıtm´enyt vesz fel a teljes f¨ uz´er, ha 230 V fesz¨ ults´egre kapcsoljuk! b) Mekkora lesz a teljes felvett teljes´ıtm´eny, ha csak t´ız, sorosan kapcsolt ´eg˝ ob˝ol all´ ´ o f¨ uz´ert k¨ ot¨ unk 230 V-ra? Megjegyz´es: A m´asodik esetben az ´eg˝ ok viszonylag r¨ovid id˝ o m´ ulva ki´egnek. (4 pont)

P. 5373. A 7,3 km vonalhossz´ us´ag´ u M4-es metr´ ovonal Kelenf¨old vas´ ut´ allom´ast ´es a Keleti p´ alyaudvart k¨ oti ¨ ossze, mik¨ ozben tov´ abbi 8 a´llom´ast ´erint. A szerel2 ak el az a´llom´asokat, ´es f´ekez´esn´el is v´enyek a´lland´ o 1,0 m/s gyorsul´assal hagyj´ ekkora lassul´ assal ´allnak meg. Az ´allom´asok k¨ oz¨ ott a maxim´alis halad´ asi sebess´eg 80 km/h. A meg´all´ okban az utascsere a´tlagos ideje 0,5 perc. a) Mennyi ideig tart, am´ıg az ´allom´asr´ ol indulva a szerelv´eny el´eri az utaz´ asi sebess´eg´et? Mekkora utat tesz meg ezalatt? b) A szerelv´eny egy u ´tja sor´ an mennyi ideig halad a 80 km/h-s utaz´ osebess´eg´evel?

b) A p´ alca m´ asik v´ege mekkora ´es milyen ir´any´ u sebess´eggel indul el? (5 pont)

K¨ ozli: Gelencs´er Jen˝ o, Kaposv´ ar

P. 5376. Egy 2L hossz´ us´ ag´ u, v´ızszintes tart´ alyt egy h˝ oszigetel˝ o dugatty´ u oszt om´ers´eklet˝ u, n m´ ol k´etatomos k´et azonos t´erfogat´ u r´eszre. Mindk´et r´eszben T0 h˝ ide´ alis g´az van. A dugatty´ u mindk´et oldala egy-egy D direkci´os erej˝ u, v´ızszintes helyzet˝ u h´ uz´ o-nyom´ o rug´ oval van ¨ osszek¨ otve a tart´ aly f¨ ugg˝oleges falaival. A rug´ok kezdetben ny´ ujtatlanok. Ha a jobb oldali g´ azzal lassan h˝ ot k¨ ozl¨ unk, a dugatty´ u L/2 t´avols´ agot mozdul el balra. A folyamat sor´ an a bal oldali r´eszben l´ev˝ o g´ az egy u, nagy h˝ okapacit´as´ u h˝ otart´ alyhoz kapcsol´odik. T0 h˝om´ers´eklet˝ a) Mekkora a jobb oldali r´eszben a g´ az nyom´ asa akkor, amikor a dugatty´ u x t´avols´ aggal mozdult el az eredeti helyzet´et˝ ol? b) Adjuk meg a jobb oldali g´ azzal a teljes folyamat sor´ an k¨ oz¨ olt h˝ot! (5 pont)

K¨ ozli: Sz´ asz Kriszti´ an, Budapest

c) Mennyi az M4-es metr´ o menetideje a k´et v´eg´ allom´as k¨oz¨ott, azaz mennyi id˝ o telik el a szerelv´eny kelenf¨ oldi elindul´ asa ´es Keleti p´alyaudvari meg´erkez´ese k¨oz¨ott?

P. 5377. H´ arom, egyenk´ent q elektromos t¨ olt´es˝ u, pontszer˝ u testet egy egyenl˝o oldal´ u h´aromsz¨ og cs´ ucsaiban r¨ ogz´ıt¨ unk. Mekkora Q t¨ olt´es˝ u pontszer˝ u testet kell elhelyezn¨ unk a h´ aromsz¨ og k¨ oz´eppontj´aban, hogy a r¨ ogz´ıt´es felold´ asa ut´an mindegyik t¨olt´es nyugalomban maradjon?

(4 pont)

(4 pont)

Tarj´ an Imre Orsz´ agos Eml´ekverseny, Szolnok

K¨ oz´ episkolai Matematikai ´ es Fizikai Lapok, 2022/1

59

60

K¨ ozli: Holics L´ aszl´ o, Budapest K¨ oz´ episkolai Matematikai ´ es Fizikai Lapok, 2022/1

P. 5378. Az ´ abr´ an l´athat´ o´ aramk¨ or K kapcsol´oja hossz´ u ideje z´arva van. Egyszer csak a kapcsol´ ot kinyitjuk. Mekkora a tekercsben induk´ al´od´o fesz¨ ults´eg nagys´ aga k¨ ozvetlen¨ ul a kapcsol´o kinyit´ asa ut´ an? (5 pont)

P´eldat´ ari feladat nyom´ an

P. 5379. Ide´ alis pol´ arsz˝ ur˝ ok seg´ıts´eg´evel szeretn´enk a line´ arisan polariz´alt f´eny ´gy, hogy az intenzit´ asvesztes´eg legfeljebb polariz´ aci´ os s´ıkj´at 45◦ -kal elforgatni u 10% legyen. Legal´abb h´ any pol´arsz˝ ur˝ ore van sz¨ uks´eg¨ unk, ´es hogyan kell azokat optim´alisan elhelyezni? (5 pont)

P´eldat´ ari feladat nyom´ an

P. 5380. Egy speci´alis izot´ oplaborban a dozim´eterek hiteles´ıt´es´ere extr´em akasokat haszn´alnak. A k´et nagy tisztas´ ag´ u radioakt´ıv tivit´ as´ u 137 Cs, illetve 60 Co forr´ forr´ as ellen˝ orz´esekor azt tapasztalt´ak, hogy a 68 mg-nyi c´ezium ´es egy ismeretlen t¨ omeg˝ u kobaltforr´ as eset´eben is j´ o k¨ ozel´ıt´essel percenk´ent ugyanannyi boml´ as t¨ ort´ent. a) Mekkora a kobaltforr´ as t¨ omege? b) Mennyi id˝ o m´ ulva ´es melyik izot´ opminta aktivit´asa lesz a m´asik k´etszerese? (A 137 Cs felez´esi ideje: 30,17 ´ev, a 60 Co felez´esi ideje: 5,27 ´ev.) (4 pont)

K¨ozli: Kis Tam´ as, Heves

P. 5381. Egy u ol k´esz¨ ult (szige¨vegb˝ tel˝o) ed´eny higannyal van t¨ oltve. A higanyba egy f¨ ugg˝oleges, d = 0,5 mm a´tm´er˝ oj˝ u kapill´aris cs˝o mer¨ ul az ´ abr´ an l´athat´ o m´ odon. A higany felsz´ıne f¨ ol´e h = 6 mm magass´ agban egy nagy kiterjed´es˝ u, v´ızszintes f´emlemezt helyezt¨ unk. Mennyivel v´ altozik meg a kapill´ aris cs˝oben a higanyszint, ha a f´emlemez ´es a higany k¨ oz´e U = 20 kV egyenfesz¨ ults´eget kapcsolunk? (6 pont)

K¨ozli: Vigh M´ at´e, Biatorb´ agy Beku ¨ld´esi hat´arid˝o: 2022. febru´ar 15. Elektronikus munkafu ¨zet: https://www.komal.hu/munkafuzet

occur as terms of the sequence, and which of them do not occur? If a number occurs as a term, find the index of the term, otherwise explain why it is not a term of the sequence. 8194, 649 287 365, 29 453 759 372, 8 398 507 839 348. K. 715. We have two jugs. Each of them can hold 2 litres of liquid. Initially, one jug is filled with 2 litres of 100% orange juice, and the other contains 1 litre of water. 1. Half the orange juice is poured into the water jug, and stirred with a spoon. Then 1 litre of the mixture is poured back in the first jug. 2. The procedure is repeated once more: 1 litre is transferred from the first jug to the second, stirred, and 1 litre is transferred back to the first jug. Find the resulting percentage of orange juice in the content of each jug. K. 716. In a shop, three notebooks and two pens cost 1110 forints (HUF, Hungarian currency). Five notebooks and four pens cost 2010 forints. What is the price of one notebook, and what is the price of one pen? K/C. 717. In a regular dodecagon ABCDEF GHIJKL the squares ABP Q and GHRS are drawn on sides AB and GH, on the inside, as shown in the figure. Show that P Q and RS are two opposite sides of a regular hexagon. K/C. 718. How many numbers are there from 1 to 50 that can be represented as a sum of at least two consecutive non-negative integers? New exercises for practice – competition C (see page 31): Exercises up to grade 10: K/C. 717. See the text at Exercises K. K/C. 718. See the text at Exercises K. Exercises for everyone: C. 1699. In the expansion of the product (x + 1) · (x2 + 1) · (x3 + 1) · . . . · (x12 + 1), what is the coefficient of the term in x14 ? C. 1700. In a circle of centre O, A is an interior point different from O. For a point B on the circumference of the circle, ∠OAB = α. Let C be a point on the circumference such that 2α + β = 180◦ , where ∠BAC = β, and the angles ∠BAO and ∠BAC have no common points apart from the ray AB. Prove that the points O, A, B, C are concyclic. C. 1701. What is the sum of all integers x √ for which 2x2 − 6x − 20 < −x + 5? Exercises upwards of grade 11: C. 1702. Vertex A of a quadrilateral ABCD lies on the plane S, its diagonal BD is parallel to the plane S, and its vertex C is at a distance of 8 units from the plane S. Given that the orthogonal projection of the quadrilateral onto S is a square with a diagonal 6 units long, prove that quadrilateral ABCD is a rhombus, and calculate the length of its sides. (Proposed by N. Zagyva, Baja) C. 1703. The natural numbers a and b each have only digits of 1 in decimal notation. Prove that if a and b are not relatively prime then the sums of their digits, S(a) and S(b) are not relatively prime either. New exercises – competition B (see page 32): B. 5214. The sequence of digits 110 represents an even integer, whatever positive integer greater than 1 is the base of notation. Is there a sequence of digits 1 and 0 such that it represents a multiple of 3, whatever positive integer greater than 1 is the base of notation? (3 points) B. 5215. Find all 1

1

positive real numbers x for which x + x is an integer, and x3 + x3 is a prime number. (4 points) (Based on the idea of B. and V. Szaszk´ o-Bog´ ar ) B. 5216. The tangents drawn to the circumscribed circle of a right triangle ABC at the right-angled vertex C and at another vertex A intersect at D. Prove that the line BD bisects the altitude drawn from vertex C. (3 points) B. 5217. A new triangle is constructed out of the line segments 2 obtained by multiplying the medians of a triangle by √ . The procedure is repeated with 3

MATHEMATICAL AND PHYSICAL JOURNAL FOR SECONDARY SCHOOLS (Volume 72. No. 1. January 2022) Problems in Mathematics

|a+b+c|

New exercises for practice – competition K (see page 30): K. 714. The first term of a sequence is 3, and every further term is obtained by subtracting 2 from the double of the previous term. a) List the first 8 terms of the sequence. b) Which of the numbers below

K¨ oz´ episkolai Matematikai ´ es Fizikai Lapok, 2022/1

the triangle obtained. Show that the triangle obtained in the second step is congruent to the original triangle. (4 points) (Proposed by P. B´ artfai, Budapest) B. 5218. What is the largest number of elements that can be selected out of the first 2022 positive integers such that the difference of any two selected numbers is not a prime? (5 points)

61

|a|

|b|

|c|

B. 5219. Prove that 1+|a+b+c|  1+|a| + 1+|b| + 1+|c| for all real numbers a, b, c. On what condition will equality occur? (5 points) (Proposed by J. Schultz, Szeged) B. 5220. Let n be a positive integer. Prove that it is possible to select n perfect squares from the

62

K¨ oz´ episkolai Matematikai ´ es Fizikai Lapok, 2022/1

numbers 1 to 2n+2 such that the sums obtained by adding an arbitrary subset of the selected numbers (including sums of single terms and the sum of all the numbers) are all distinct. (6 points) (Proposed by R. Freud, Budapest) B. 5221. In an acute-angled triangle ABC, the points of tangency of the inscribed circle on sides BC, CA, AB are D, E, and F , respectively. The circumscribed circle of the triangle intersects circle AEF at a point P different from A, intersects circle BF D at a point Q different from B, and intersects circle CDE at a point R different from C. Show that the lines DP , EQ and F R are concurrent. (6 points) (Proposed by M. Lovas, Budapest) New problems – competition A (see page 34): A. 815. Let q be a monic polynomial with integer coefficients. Prove that there exists a constant C depending only on polynomial q such that for an arbitrary prime number p and an arbitrary positive integer N  p the congruence n! ≡ q(n) (mod p) has at most CN 2/3 solutions among any N consecutive integers. (Submitted by Navid Safaei, Iran) A. 816. Peter has 2022 pieces of magnetic railroad cars, which are of two types: some has the front with north and the rear with south magnetic polarity, and some has the rear with north and the rear with south magnetic polarity (on these railroad cars the front and the rear can be distinguished). Peter wants to decide whether there are the same number of both type of cars. He can try to fit together two cars in one try. What is the least number of tries needed? (Submitted by D¨ om¨ ot¨ or P´ alv¨ olgyi, Budapest) A. 817. Let ABC be a triangle. Let T be the point of tangency of the circumcircle of triangle ABC and the A-mixtilinear incircle (the circle which is tangent to sides AB, AC, and internally tangent to the circumcircle of triangle ABC). The incircle of triangle ABC has center I and touches sides BC, CA and AB at points D, E and F , respectively. Let N be the midpoint of line segment DF . Prove that the circumcircle of triangle BT N , line T I and the perpendicular from D to EF are concurrent. (Submitted by Diaconescu Tashi, Romania)

Problems in Physics (see page 58) M. 410. If a playing card is placed between a small, strong magnet and a horizontal paper clip, then we can lift the paper clip by the card. Measure how many cards you need to stack so that you can no longer lift the paper clip. What is the thickness of these stacked sheets? Connect two magnets of the same size and investigate how many cards are needed so that we can no longer lift the paper clip. G. 765. Each of the 11 shots in the series of photos was taken from the same location, with the camera always facing the Sun. The chronological order of the photos is from left to right. What was the time when the image of the Sun was the closest to the horizon? To which point of the compass did the camera face when the Sun was at its lowest position in the sky? Where and in which season was the series of photos taken? G. 766. The most famous formula in physics is the relation of E = mc2 expressing the equivalence of mass and energy where E is the energy, m is the mass, and c is the speed of light in vacuum. Using this, estimate how much heavier is our mobile phone, when its battery is fully charged, than when its battery is fully discharged. G. 767. The Morning Star (actually the planet Venus) is visible for a while only in the evenings, and then it appears for a while only at dawns. What is the period of this change? G. 768. The figure shows the currentvoltage characteristic of one of the tungsten filament bulbs of a string of a Christmas tree lights, which contains a hundred bulbs connected in series. a) Using the graph determine the total dissipated electrical energy by all the bulbs of the string if it is connected to a voltage supply of 230V. b) What is the total dissipated electrical energy by the string if it contains only ten bulbs and it is connected to 230 V? Note: In the second case the bulbs will quite soon burn out.

K¨ oz´ episkolai Matematikai ´ es Fizikai Lapok, 2022/1

63

P. 5373. The 7.3 km M4 metro line connects Kelenf¨ old railway station and Keleti railway station, and between them it has 8 other stops. Trains leave each station (or stop) with a constant acceleration of 1.0 m/s2 and and stop with the same deceleration when braking. The maximum speed between stations is 80 km/h. At the stations the average time to change passengers is 0.5 minutes. a) How long does it take for the train to reach its cruising speed when it leaves the station? How far does it travel during this time? b) During one journey, how long does the train travel at the speed of 80 km/h? c) What is the journey time of the M4 metro between the two terminals, i.e. how much time elapses between the departure of a train from Kelenf¨ old and its arrival at Keleti railway station? P. 5374. The picture shows a serial firing, spring-loaded toy rifle, which can fire six thin, cylindrical sponge bullets. Before each shot, the black slide must be pulled to the rightmost position by about 10 cm. We used a digital scale and found that the maximum force required to cock the rifle is the same as the weight of a 6.6 kg object. a) How could the force have been determined if no auxiliary means other than the balance was used? b) Estimate the maximum velocity at which a sponge bullet of mass 3 g will fly out if 10% of the energy stored in the spring is used to accelerate the bullet. P. 5375. One end of a thin, uniform-density stick lying in a frictionless horizontal plane is suddenly struck so that the the velocity of the end point is perpendicular to the stick and has a magnitude of v. a) Which part of the stick will have zero initial speed? b) What is the speed and direction of the motion of the other end of the stick, at the moment when it starts to move? P. 5376. A horizontal tank of length 2L is divided into into two parts, having equal volume, by a thermally insulating piston. Both parts contain a sample of ideal gas of n moles of diatomic molecules at a temperature of T0 . To each side of the piston a horizontal spring of spring constant D is attached. Both springs are designed for compression and tension and their other ends are fixed to the vertical walls of the container. The springs are initially unstretched. When the gas in the right part of the container is slowly heated, the piston moves a distance of L/2 towards the left. During the process the gas in the left part is connected to a high heat capacity heat reservoir of temperature T0 . a) What is the pressure of the gas in the right part when the piston has moved a distance of x from the original position? b) Determine the heat absorbed by the gas in the right part during the whole process. P. 5377. Three point-like objects, each having an electric charge of q, are fixed at the vertices of an equilateral triangle. Another point-like object of charge Q is placed to the centroid of the triangle. What should the charge Q of this fourth object be in order that even if the fixation of the objects is ceased all of the charged objects remain at rest? P. 5378. Switch K in the circuit shown in the figure has been closed for a long time. Once the switch is opened. What is the magnitude of the induced electromotive force in the coil immediately after the switch is opened? P. 5379. With the help of ideal polarizing filters, we would like to turn the polarization plane of linearly polarized light by 45◦ in order to reduce the intensity loss by at most 10%. At least how many polarizing filters are needed and how should they be optimally arranged? P. 5380. In a special isotope laboratory, extreme activity 137 Cs and 60 Co sources are used to validate dosimeters. When the two high-purity radioactive sources were checked, it was found that in a minute approximately the same number of decays occurred in the case of the 68 mg caesium source, and in the case of the cobalt source of unknown mass. a) What is the mass of the cobalt source? b) After how long and which source will have twice the activity of the other? (Half-life of 137 Cs: 30.17 years, half-life of 60 Co: 5.27 years.) P. 5381. An (insulating) glass container is filled with mercury. A vertical capillary tube of diameter d = 0.5 mm is immersed in the mercury as shown in the figure. Above the surface of the mercury at a height of h = 6 mm, a large, horizontal metal sheet is placed. How much does the mercury level in the capillary tube change if a DC voltage source of U = 20 kV is connected across the mercury and the metal sheet?

72. ´evfolyam 1. sz´am

K¨oMaL

Budapest, 2022. janu´ar

´ ´ FIZIKAI LAPOK ¨ EPISKOLAI KOZ MATEMATIKAI ES ˝ ´ITVE INFORMATIKA ROVATTAL BOV

Jelent´es a 2021. ´evi Ku ¨rsch´ak J´ozsef Matematikai Tanul´ oversenyr˝ol

´ ALAP´ITOTTA: ARANY DANIEL 1894-ben 72. ´evfolyam 2. sz´am

Budapest, 2022. febru´ar

´ Megjelenik ´evente 9 sz´amban, janu´art´ol m´ajusig ´es szeptembert˝ol decemberig havonta 64 oldalon. ARA: 1050 Ft

´ TARTALOMJEGYZEK Jelent´es a 2021. ´evi K¨ursch´ak J´ozsef Matematikai Tanul´oversenyr˝ol . . . . . . . . . . . . . . . . . . . . . . . . . . . . .

66

Pach P´eter P´al: A 2021. ´evi K¨ursch´ak J´ozsef Matematikai Tanul´overseny feladatainak megold´asa . . . . . . . . . . . . . . . . . . . . . . . . . . . . . . . . . . . . . . .

67

Marczis Gy¨orgy, Moln´ar Istv´an, Moln´ar Judit, R´ok´an´e R´ozsa Anik´o: Gyakorl´o feladatsor emelt szint˝u matematika ´eretts´egire . . . . . . . . . . .

72

Koncz Levente: Megold´asv´azlatok a 2022/1. sz´am emelt szint˝u matematika gyakorl´o feladatsor´ahoz . . . . . . . . . . . . . . . . . . . . . . . . . . . . . . . . . . . . . . .

75

Matematika feladatok megold´asa (5150., 5201.) . . .

84

Ifj´u olvas´oinkhoz r´egen ´es most. . . . . . . . . . . . . . . . . . .

86

A K pontversenyben kit˝uz¨ott gyakorlatok (719– 723.) . . . . . . . . . . . . . . . . . . . . . . . . . . . . . . . . . . . . . . . . .

89

A C pontversenyben kit˝uz¨ott gyakorlatok (722– 723., 1704–1708.) . . . . . . . . . . . . . . . . . . . . . . . . . . . . .

90

A B pontversenyben kit˝uz¨ott feladatok (5222– 5229.) . . . . . . . . . . . . . . . . . . . . . . . . . . . . . . . . . . . . . . . .

91

Az A pontversenyben kit˝uz¨ott nehezebb feladatok (818–820.) . . . . . . . . . . . . . . . . . . . . . . . . . . . . . . . . . . . .

92

T´oth Tam´as: A matematikai logika logikusabb, mint gondoln´ank I.. . . . . . . . . . . . . . . . . . . . . . . . . . . .

93

Informatik´ab´ol kit˝uz¨ott feladatok (556–558., 60., 159.) . . . . . . . . . . . . . . . . . . . . . . . . . . . . . . . . . . . . . . . . .

97

´ Eletm˝ ´ R´atz Tan´ar Ur ud´ıjak ´atad´asa 2021 december´eben . . . . . . . . . . . . . . . . . . . . . . . . . . . . . . . . . . . . . .

101

Gn¨adig P´eter, Sz´echenyi G´abor, Vank´o P´eter, Vigh M´at´e: Besz´amol´o a 2021. ´evi E¨otv¨osversenyr˝ol . . . . . . . . . . . . . . . . . . . . . . . . . . . . . . . . . . . .

105

Fizika gyakorlat megold´asa (759.) . . . . . . . . . . . . . . . .

113

Fizika feladatok megold´asa (5338., 5354., 5356.) . .

114

Felh´ıv´as a Kunfalvi Rezs˝o Olimpiai V´alogat´oversenyre . . . . . . . . . . . . . . . . . . . . . . . . . . . . . . . . . . . . . . . .

122

Fizik´ab´ol kit˝uz¨ott feladatok (411., 769–772., 5382–5390.) . . . . . . . . . . . . . . . . . . . . . . . . . . . . . . . . . .

122

Problems in Mathematics . . . . . . . . . . . . . . . . . . . . . . . .

125

Problems in Physics . . . . . . . . . . . . . . . . . . . . . . . . . . . . .

127

Problems of the 2021 K¨ursch´ak competition . . . . . .

128

K¨ oz´ episkolai Matematikai ´ es Fizikai Lapok, 2022/2

´ EVA ´ F˝oszerkeszt˝o: RATKO ´ ¨ Fizikus szerkeszt˝o: GNADIG PETER ´ ILDIKO ´ M˝uszaki szerkeszt˝o: MIKLOS Bor´ıt´o: BURGHARDT ZSUZSA ´ Kiadja: MATFUND ALAP´ITVANY ´ RITA Alap´ıtv´anyi k´epvisel˝o: KOS Felel˝os kiad´o: KATONA GYULA Nyomda: OOK-PRESS Kft. ´ Felel˝os vezet˝o: SZATHMARY ATTILA INDEX: 25 450 ISSN 1215-9247 A matematika bizotts´ag vezet˝oje: ´ HERMANN PETER ´ BALINT, ´ ´ Tagjai: B´IRO GYENES ZOLTAN, ´ ´ KISS HUJTER BALINT, IMOLAY ANDRAS, ´ ´ GEZA, ´ ´ RITA, KOZMA GEZA, KOS KOS ´ MATOLCSI DAVID, ´ KATALIN ABIGEL, ´ ´ PACH PETER ´ ´ V´IGH ¨ ORDI ¨ OK PETERN E, PAL, VIKTOR A fizika bizotts´ag tagjai: ´ ´ ´ BARANYAI KLARA, HOLICS LASZL O, ´ ´ HONYEK GYULA, OLOSZ BALAZS, SZASZ ´ SZECHENYI ´ ´ KRISZTIAN, GABOR, VIGH ´ E, ´ VLADAR ´ KAROLY, ´ MAT WOYNAROVICH FERENC Az informatika bizotts´ag vezet˝oje: ´ ´ SCHMIEDER LASZL O ´ E, ´ FARKAS CSABA, FODOR Tagjai: BUSA MAT ´ ´ NIKOLETT, LOCZI ´ ZSOLT, LASZL O LAJOS, ´ ´ ´ SIEGLER GABOR, SZENTE PETER, TOTH ´ TAMAS ´ ANDREA, TASNADI ´ ANIKO ´ Ford´ıt´ok: GROF ´ ´ ¨ Szerkeszt˝os´egi titk´ar: TRASY GYORGYN E A szerkeszt˝os´eg c´ıme: 1117 Budapest, P´azm´any P´eter s´et´any 1/C III. emelet 3.405. Telefon: 372-2850 A lap megrendelhet˝o az Interneten: www.komal.hu/megrendelolap/reszletek.h.shtml. El˝ofizet´esi d´ıj egy ´evre: 8800 Ft K´eziratokat nem ˝orz¨unk meg ´es nem k¨uld¨unk vissza. Minden jog a K¨oMaL tulajdonosai´e. E-mail: [email protected] Internet: http://www.komal.hu This journal can be ordered from the Editorial office: P´azm´any P´eter s´et´any 1/C III. emelet 3.405. 1117–Budapest, Hungary telephone: +36 (1) 372-2850 or on the Postal address H–1518 Budapest 112, P.O.B. 32, Hungary, or on the Internet: www.komal.hu/megrendelolap/reszletek.e.shtml. A Lapban megjelen˝o hirdet´esek tartalm´a´ert felel˝oss´eget nem v´allalunk.

65

A Bolyai J´ anos Matematikai T´arsulat a 2021. ´evi K¨ ursch´ ak J´ ozsef Matematikai Tanul´oversenyt okt´ober 8-´ an 14 o´rai kezdettel rendezte meg. A k¨ ovetkez˝ o nyolc helysz´ınen ´ırt´ak meg a versenydolgozatot a r´esztvev˝ ok: Budapest, Debrecen, Gy˝ or, Kecskem´et, Miskolc (k´et helysz´ınen), P´ecs ´es Szeged. A T´arsulat eln¨ oks´ege a verseny lebonyol´ıt´ as´ ara az al´abbi bizotts´agot k´erte fel: Bir´ o Andr´ as, Frenkel P´eter, Harangi Viktor, K´ os G´eza, Maga P´eter (titk´ ar), Pach P´eter P´ al (eln¨ ok), T´ oth G´eza. A bizotts´ ag szeptember 1-jei u ovetkez˝ o ¨l´es´en a k¨ feladatokat t˝ uzte ki: al1. A s´ıkbeli der´eksz¨ og˝ u koordin´ atarendszer Pi = (ai , bi ) (i = 0, 1, 2) pontjai ´ tal alkotott h´ aromsz¨ ognek az O = (0, 0) orig´ o bels˝ o pontja. Mutassuk meg, hogy aromsz¨ ogek ter¨ uletei (ebben a sorrendben) akkor ´es csak a P0 OP1 , P0 OP2 , P1 OP2 h´ akkor alkotnak m´ertani sorozatot, ha az a 0 x 2 + a 1 x + a 2 = b0 x 2 + b 1 x + b 2 = 0 egyenletrendszernek van val´ os x megold´ asa. 2. Csodaorsz´ ag n v´ arosa k¨ oz¨ ott n l´egit´ arsas´ ag u aratokat. Minden ¨zemeltet j´ egyes l´egit´ arsas´ aghoz p´ aratlan sok v´ aros tartozik, mondjuk v1 , v2 , . . . , vi , amelyek aratokra k¨ oz¨ ott k¨ orj´ aratot u anyban: a vj vj+1 , illetve a vj+1 vj j´ ¨zemeltet mindk´et ir´ alhat´ o p´ alehet jegyet v´ altani 1  j  i eset´en, ahol vi+1 = v1 . Igazoljuk, hogy tal´ ratlan sok v´ aros, mondjuk u1 , u2 , . . . , uk u ´gy, hogy az u1 u2 , u2 u3 , . . . , uk−1 uk , uk u1 j´ aratokra lehet jegyet v´ altani csupa k¨ ul¨ onb¨ oz˝ o l´egit´ arsas´ agn´ al. urhatsz¨ og, amelynek A1 B1 , A2 B2 ´es A3 B3 ´ atl´ oi 3. Adott az A1 B3 A2 B1 A3 B2 h´ atl´ ok egy ponton mennek ´ at. Minden i = 1, 2, 3 eset´en az Ai Bi ´es az Ai+1 Ai+2 ´ abb´ a Di olyan, a Bi -t˝ ol k¨ ul¨ onb¨ oz˝ o pont a hatsz¨ og k¨ or´e ´ırt metsz´espontja Ci , tov´ or ´erinti az Ai+1 Ai+2 egyenest. (A pontokat modulo 3 k¨ or¨ on, amelyre a Bi Ci Di k¨ sz´ amozzuk, teh´ at A4 = A1 ´es A5 = A2 .) Igazoljuk, hogy az A1 D1 , A2 D2 ´es A3 D3 szakaszok egy ponton mennek ´ at. A bizotts´ag a be´erkezett dolgozatok a´tn´ez´ese ut´ an, november 16-ai u ¨l´es´en a k¨ovetkez˝ o jelent´est fogadta el: A verseny minden helysz´ınen rendben zajlott le: a 78 regisztr´alt versenyz˝ o ” k¨oz¨ ul 73-an vettek r´eszt a versenyen, ´es t˝ ol¨ uk ¨ osszesen 64 dolgozat ´erkezett be. Az idei versenyen az els˝ o feladatot 21-en, a m´asodik feladatot 5-en, a harmadik feladatot pedig 3-an oldott´ ak meg helyesen vagy l´enyeg´eben helyesen. B´ ar mindegyik feladatra sz¨ ulettek helyes megold´ asok, k´et feladatn´ al t¨ obbet egyetlen versenyz˝ o sem oldott meg, ´ıgy a versenybizotts´ag id´en nem ad ki I. d´ıjat. Egy versenyz˝ o helyesen oldotta meg az els˝ o ´es a harmadik feladatot. Ez´ert 66

K¨ oz´ episkolai Matematikai ´ es Fizikai Lapok, 2022/2

II. d´ıjban ´es 50 000 Ft p´enzjutalomban r´eszes¨ ul ´ Seres-Szab´o M´arton, a Budapesti Fazekas Mih´ aly Gyakorl´ o Altal´ anos Iskola ´es ´ Gimn´ azium 11. oszt´alyos tanul´oja (tan´ arai Dobos S´ andor, Ad´ am R´eka ´es Fazakas T¨ unde). III. d´ıjban ´es 40 000 Ft p´enzjutalomban r´eszes¨ ul ´ Fleiner Zsigmond, a Budapesti Fazekas Mih´aly Gyakorl´o Altal´ anos Iskola ´es Gimn´ azium 12. oszt´alyos tanul´oja (tan´ arai Hujter B´ alint, Gyenes Zolt´ an, Dobos S´ andor ´es Juh´ asz P´eter ), aki jav´ıthat´ o hib´ at´ ol eltekintve helyesen oldotta meg az els˝o feladatot ´es kis hi´anyoss´ aggal oldotta meg a m´ asodik feladatot, ´ V´arkonyi Zsombor, a Budapesti Fazekas Mih´aly Gyakorl´ o Altal´ anos Iskola ´es Gimn´ azium ´eretts´egizett tanul´ oja (tan´ arai Fazakas T¨ unde, Kocsis Szilveszter, P´ osa andor ), aki l´enyeg´eben helyesen oldotta meg az els˝ o feladatot ´es Lajos ´es Dobos S´ helyes megold´ ast adott a m´asodik feladatra. Dics´eretben ´es 20 000 Ft p´enzjutalomban r´eszes¨ ul ´ Csapl´ar Viktor, a Budapesti Fazekas Mih´aly Gyakorl´ o Altal´ anos Iskola ´es Gimn´ azium ´eretts´egizett tanul´ oja (tan´ arai Fazakas T¨ unde ´es Kocsis Szilveszter ), aki apr´ o hi´anyoss´ agt´ ol eltekintve helyesen oldotta meg az els˝ o feladatot ´es r´eszeredm´enyeket ´ert el a harmadik feladatban. A versenybizotts´ ag ez´ uton k¨ osz¨ oni meg minden versenyz˝ o, felk´esz´ıt˝o tan´ ar ´es a lebonyol´ıt´ asban k¨ ozrem˝ uk¨ od˝ o koll´ega munk´ aj´at, a d´ıjazottaknak pedig tov´ abbi sikereket k´ıv´ anva gratul´al.”

old´ asa a feladatbeli egyenletrendszernek. Ford´ıtva, ha x megold´ asa a feladatbeli egyenletrendszernek, akkor 0 = 0 − t2 0 = (t0 v0 + t1 v1 + t2 v2 ) − t2 (x2 v0 + xv1 + v2 ) = = (t0 − t2 x2 )v0 + (t1 − t2 x)v1 ,

ahonnan, mivel v0 ´es v1 nem p´ arhuzamos, t0 = x2 t2 ´es t1 = xt2 , teh´ at t2 , t1 , t0 m´ertani sorozat. or¨ ul tetsz˝ o2. megold´as (v´azlat). Vegy¨ uk ´eszre, hogy a Pi pontokat az orig´o k¨ leges sz¨oggel elforgatva a k´erd´eses ter¨ uletek nem v´ altoznak, m´ asr´eszt a k´et r´egi egyenlet line´ aris kombin´ aci´ ojak´ent el˝ o´ all´ıthat´ o mindk´et u ´j egyenlet, ´es ford´ıtva. Ez azt jelenti, hogy elegend˝o az elforgatott pontokra bel´ atni a feladatbeli ekvivalenci´at. Hasonl´ oan, minden pont x koordin´ at´ aj´at megszorozhatjuk ugyanazzal a pozit´ıv val´os sz´ammal. K¨onnyen l´ athat´ o, hogy ilyen transzform´aci´ ok egym´asut´ anj´ aval minden esetben eljuthatunk egy olyan ponth´ armashoz, amelyn´el P0 = (1, 0), P1 = (0, 1), valamint os sz´amokra. Ekkor a ter¨ uletek: P2 = (−a, −b) valamely pozit´ıv a, b val´ t0 = a/2,

t1 = b/2,

t2 = 1/2;

a k´et egyenlet pedig a k¨ ovetkez˝ o: x2 − a = 0,

x − b = 0,

mely esetben a k´erd´eses ekvivalencia nyilv´ anval´ o.

A 2021. ´evi Ku ozsef Matematikai Tanul´ overseny ¨rsch´ak J´ feladatainak megold´asa

1. A s´ıkbeli der´eksz¨ og˝ u koordin´ atarendszer Pi = (ai , bi ) (i = 0, 1, 2) pontjai ´ altal alkotott h´ aromsz¨ ognek az O = (0, 0) orig´ o bels˝ o pontja. Mutassuk meg, hogy aromsz¨ ogek ter¨ uletei (ebben a sorrendben) akkor ´es csak a P0 OP1 , P0 OP2 , P1 OP2 h´ akkor alkotnak m´ertani sorozatot, ha az a 0 x 2 + a 1 x + a 2 = b0 x 2 + b 1 x + b 2 = 0 egyenletrendszernek van val´ os x megold´ asa. 1. megold´as. Legyenek t2 , t1 , t0 a feladatbeli ter¨ uletek (ezeket pozit´ıv sz´amoknak tekintj¨ uk a h´aromsz¨ ogek k¨ or¨ ulj´ar´ asi ir´ any´ at´ ol f¨ uggetlen¨ ul). Legyen vi = (ai , bi ). arhuzamos (pl. v0 -lal az´ert, mert A t0 v0 + t1 v1 + t2 v2 vektor mindegyik vi -vel p´ oleges komponens´enek ar´ anya a −t2 : t1 ar´ annyal a v1 ´es v2 vektorok v0 -ra mer˝ egyezik meg), ez´ert nullvektor. Vegy¨ uk ´eszre, hogy a feladatbeli egyenletrendszer ekvivalens az x2 v0 + xv1 + ociens meg+ v2 = 0 vektoregyenlettel. Ha t2 , t1 , t0 m´ertani sorozat, akkor a kv´ K¨ oz´ episkolai Matematikai ´ es Fizikai Lapok, 2022/2

67

2. Csodaorsz´ ag n v´ arosa k¨ oz¨ ott n l´egit´ arsas´ ag u aratokat. Minden ¨zemeltet j´ egyes l´egit´ arsas´ aghoz p´ aratlan sok v´ aros tartozik, mondjuk v1 , v2 , . . . , vi , amelyek aratokra k¨ oz¨ ott k¨ orj´ aratot u anyban: a vj vj+1 , illetve a vj+1 vj j´ ¨zemeltet mindk´et ir´ lehet jegyet v´ altani 1  j  i eset´en, ahol vi+1 = v1 . Igazoljuk, hogy tal´ alhat´ o p´ a´gy, hogy az u1 u2 , u2 u3 , . . . , uk−1 uk , uk u1 ratlan sok v´ aros, mondjuk u1 , u2 , . . . , uk u j´ aratokra lehet jegyet v´ altani csupa k¨ ul¨ onb¨ oz˝ o l´egit´ arsas´ agn´ al. Megold´as. Minden l´ep´esben v´ alasszunk egy u ´j l´egit´ arsas´ agot (amit kor´abbi l´ep´esekben m´eg nem v´ alasztottunk) ´es ennek a t´arsas´ agnak egy j´ arat´ at (azaz k´et v´arost, amik k¨ oz¨ ott k¨ ozlekedik) arra u alasztott j´ aratokkal ne ¨gyelve, hogy a kiv´ lehessen k¨orutaz´ ast csin´ alni. Ezt addig csin´ aljuk, ameddig tudjuk. Legfeljebb n − 1 l´ep´es lehets´eges, hiszen n cs´ ucs´ u k¨ ormentes gr´ afnak maximum n − 1 ´ele lehet. Teh´ at mindenk´epp lesz olyan l´egit´ arsas´ ag, amit egyik l´ep´esben sem v´ alasztottunk. Vegy¨ uk azt az a´llapotot, amikor elakadunk. A kiv´alasztott j´ aratok ´altal meghat´ arozott gr´ af k¨ ormentes, ´ıgy sz¨ uks´egk´eppen p´ aros gr´af, ami azt jelenti, hogy arosok megsz´ınezhet˝ ok piros ´es k´ek sz´ınnel u ´gy, hogy minden kiv´alasztott j´ aa v´ rat egy piros ´es egy k´ek v´ aros k¨ oz¨ ott megy. Most vegy¨ unk egy l´egit´ arsas´ agot, amit egyik l´ep´esben sem v´ alasztottunk. Az ehhez tartoz´ o p´ aratlan k¨ ornek biztosan van olyan ´ele, ami k´et egysz´ın˝ u v´ arost k¨ ot ¨ ossze. Mivel m´ ar nem tudtunk t¨ obb l´ep´est tenni, ez az ´el biztosan k¨ ort alkot n´eh´ any kiv´alasztott ´ellel. Ennek a k¨ ornek minden j´arata m´ as l´egit´ arsas´ aghoz tartozik, ´es biztosan p´ aratlan hossz´ u, mert k´et egysz´ın˝ u v´arost kiv´alasztott ´eleknek csak p´ aros hossz´ uu ´tja k¨ othet ¨ ossze, v´egezt¨ unk. 68

K¨ oz´ episkolai Matematikai ´ es Fizikai Lapok, 2022/2

3. Adott az A1 B3 A2 B1 A3 B2 h´ urhatsz¨ og, amelynek A1 B1 , A2 B2 ´es A3 B3 ´ atl´ oi atl´ ok egy ponton mennek ´ at. Minden i = 1, 2, 3 eset´en az Ai Bi ´es az Ai+1 Ai+2 ´ abb´ a Di olyan, a Bi -t˝ ol k¨ ul¨ onb¨ oz˝ o pont a hatsz¨ og k¨ or´e ´ırt metsz´espontja Ci , tov´ or ´erinti az Ai+1 Ai+2 egyenest. (A pontokat modulo 3 k¨ or¨ on, amelyre a Bi Ci Di k¨ sz´ amozzuk, teh´ at A4 = A1 ´es A5 = A2 .) Igazoljuk, hogy az A1 D1 , A2 D2 ´es A3 D3 szakaszok egy ponton mennek ´ at. ul´ırt Megold´as. Mivel az Ai+1 Ai+2 egyenes ´erinti a Bi Ci Di k¨ort, a Di pont a k¨or¨ k¨ ornek ugyanazon az Ai+1 Ai+2 ´ıv´en van, mint a Bi , teh´at az Ai -vel szemk¨ozt; az A1 , ovetik egym´ast a k¨or¨on. D3 , A2 , D1 , A3 , D2 ebben a sorrendben k¨

ami (1a) ´es (1b) szerint azzal ekvivalens, hogy P1 P2 P3 P4 P5 P6 · · = 1. P2 P3 P4 P5 P6 P1 K¨ozvetlen bizony´ıt´as a Lemma ´all´ıt´as´ara. Ha a h´ arom ´atl´ o egy k¨ oz¨ os Q ponton megy ´at, akkor az egyenl˝ o ker¨ uleti sz¨ ogek miatt QP1 P2  ∼ QP5 P4 , QP3 P4  ∼ ∼ QP1 P6  ´es QP5 P6  ∼ QP3 P2 , ez´ert P1 P2 P3 P4 P5 P6 QP1 QP3 QP5 · · = · · = 1. P4 P5 P6 P1 P2 P3 QP5 QP1 QP3 Megford´ıtva, ha P1 P2 · P3 P4 · P5 P6 = = P2 P3 · P4 P5 · P6 P1 , akkor legyen Q = or¨ ul= P1 P4 ∩ P2 P5 , ´es legyen P6 a k¨ ´ırt k¨ or ´es a P3 Q egyenes metsz´espontja. Az el˝obbiek szerint P1 P2 · P3 P4 · P5 P6 = ot ¨ osszevetve = P2 P3 · P4 P5 · P6 P1 ; a kett˝ P5 P6 : P6 P1 = P5 P6 : P6 P1 , m´arpedig ez az ar´any egy´ertelm˝ uen meghat´ arozza a P6 pontot, ez´ert P6 = P6 . (Nincs sz¨ uks´eg r´ a, de az ´ altal´ anoss´ ag csorb´ıt´ asa n´elk¨ ul felor´ıv f´elk¨ orn´el tehetj¨ uk, hogy a P5 P6 P1 k¨ r¨ ovidebb, ´es akkor m´eg vil´agosabban l´ atszik az egy´ertelm˝ us´eg.)

Lemma. B´ armely P1 P2 P3 P4 P5 P6 h´ urhatsz¨ ogben a P1 P4 , P2 P5 ´es P3 P6 ´ atl´ ok akkor ´es csak akkor mennek a ´t egy ponton, ha

A feladat megold´asa. A felt´etel ´es a lemma szerint A1 B3 · A2 B1 · A3 B2 = = B3 A2 · B1 A3 · B2 A1 , ´es azt kell igazolnunk, hogy

P1 P 2 · P 3 P 4 · P 5 P 6 = P2 P 3 · P 4 P 5 · P 6 P 1 .

A1 D3 · A2 D1 · A3 D2 = D3 A2 · D1 A3 · D2 A1 . El˝ osz¨ or vizsg´aljuk azt az esetet, amikor a D1 pont az A2 -t nem tartalor´ıven van. Legyen a k¨ omaz´ o B1 A3 k¨ r¨ ul´ırt k¨ or ´es a D1 C1 f´elegyenes metsz´eso pontja E1 , ez az A3 -at nem tartalmaz´ or´ıven fekszik. A1 A2 k¨ A ker¨ uleti sz¨ ogek t´etel´eb˝ ol

Bizony´ıt´as. A Lemma ´all´ıt´ asa val´ oj´aban a Ceva-t´etel trigonometrikus alakj´ anak ´atfogalmaz´ asa. Ha a k¨ or¨ ul´ırt k¨ or sugara r, k¨ oz´eppontja O, akkor a ker¨ uleti ´es k¨ oz´epponti sz¨ ogek t´etel´eb˝ ol (1a)

2 2r sin P1 OP sin P1 P5 P2  P1 P2 2 , = = P2 OP3  P2 P3 sin P2 P5 P3  2r sin 2

´es hasonl´ oan (1b)

sin P3 P1 P4  P3 P4 = P4 P5 sin P4 P1 P5 

´es

A2 C1 B1  = C1 D1 B1  = E1 D1 B1  =

P5 P6 sin P5 P3 P6  . = P6 P1 sin P6 P3 P1 

= E1 A1 B1 ,

A Ceva-t´etelt a P1 P3 P5 h´ aromsz¨ ogre alkalmazva, a P1 P4 , P3 P6 ´es P5 P2 szakaszok akkor ´es csak akkor mennek ´at egy ponton, ha

´ıgy A1 E1  A2 A3 ; az A1 E1 A2 A3 n´egysz¨ og szimmetrikus trap´ez, amelyben A2 E1 = A1 A3 ´es A3 E1 = A1 A2 .

sin P1 P5 P2  sin P3 P1 P4  sin P5 P3 P6  · · = 1, sin P2 P5 P3  sin P4 P1 P5  sin P6 P3 P1  K¨ oz´ episkolai Matematikai ´ es Fizikai Lapok, 2022/2

69

70

K¨ oz´ episkolai Matematikai ´ es Fizikai Lapok, 2022/2

A C1 A3 A1  ∼ C1 B1 A2  ´es C1 A1 A2  ∼ C1 A3 B1  hasonl´ os´agokb´ ol A2 B1 A3 A1 = C1 B 1 C1 A3

B1 A3 A1 A2 = , C1 B 1 C1 A2

´es

teh´ at

Gyakorl´o feladatsor emelt szint˝ u matematika ´eretts´egire

A2 B1 A3 A1 C1 A2 = · . B1 A3 A1 A2 C1 A3

Hasonl´oan, a C1 A3 E1  ∼ C1 D1 A2  ´es C1 E1 A2  ∼ C1 A3 D1  hasonl´ os´agokb´ ol A2 D1 A3 E1 A1 A2 = = C1 D1 C1 A3 C1 A3 teh´ at

´es

D1 A3 E1 A2 A3 A1 = = , C1 D1 C1 A2 C1 A2

I. r´esz 1. a) Hat´ arozzuk meg a k¨ ovetkez˝ o kifejez´es el˝ ojel´et, ha n tetsz˝ oleges term´eszetes sz´ am: 2n + 1 2n−1 + 1 − . (6 pont) 2n + 1 2n+1 + 1

A2 D1 A1 A2 C1 A2 = · . D1 A3 A3 A1 C1 A3

b) H´any val´os megold´asa van a   π π sin x + = cos 2x − 3 3

Ezekb˝ ol azt kapjuk, hogy A2 D1 A2 B1 A1 A22 = · . D1 A3 B1 A3 A3 A21

(2)

trigonometrikus egyenletnek a ]0; π[ intervallumon?

o A2 B1 ´ıven van, akkor az A2 ´es A3 Ha a D1 pont az A3 -at nem tartalmaz´ szerep´enek felcser´el´es´evel, ugyanezekkel a l´ep´esekkel juthatunk el a (2) k´eplethez. Az indexel´est ciklikusan elforgatva, az (2) megfelel˝oi A3 D2 A3 B2 A2 A23 = · D2 A1 B2 A1 A1 A22

´es

(7 pont)

2. A 12. ´evfolyam tanul´oi k¨ oz¨ ul 25-en matematik´ ab´ ol, 40-en pedig t¨ ort´enelemb˝ol tettek emelt szint˝ u ´eretts´egi vizsg´ at. Az ´erdemjegyek eloszl´as´ at a k¨ ovetkez˝ o k¨ordiagramokon l´atjuk:

A1 D3 A1 B3 A3 A21 = · . D3 A2 B3 A2 A2 A23

V´eg¨ ul A2 D1 A3 D2 A1 D3 · · = D1 A3 D2 A1 D3 A2 =



A2 B1 A1 A22 · B1 A3 A3 A21

     A3 B2 A2 A23 A1 B3 A3 A21 · · · · = B2 A1 A1 A22 B3 A2 A2 A23

A2 B1 A3 B2 A1 B3 · · = 1. B1 A3 B2 A1 B3 A2

Megjegyz´es. A (2) k´epletet hasonl´ os´ agok helyett projekt´ıv geometriai eszk¨ oz¨ okkel, kett˝ osviszonyokkal is bizony´ıthatjuk.

a) A k¨ordiagramok alapj´an t¨ olts¨ uk ki az al´ abbi gyakoris´agi t´ abl´ azatot. (4 pont) Tant´ argy \ jegyek Matematika T¨ ort´enelem

any´ u ide´ alis pont. A k¨ or¨ ul´ırt k¨ ort az A1 pontb´ ol az A2 A3 Legyen I1 az A2 A3 ir´ egyenesre, majd a D1 pontb´ ol visszavet´ıtve, (A2 , A3 ; B1 , E1 ) = (A2 , A3 ; C1 , I1 ) = (A2 , A3 ; D1 , A1 );

4

5

b) Hat´ arozzuk meg a t¨ ort´enelem eredm´enyek a´tlag´ at, medi´ anj´ at ´es sz´ or´ as´ at. (3 pont) c) A matematik´ ab´ ol 3-ast szerz˝ok k¨ oz¨ ul legal´ abb h´any tanul´onak kellett volna 4-est kapnia, hogy a t¨ obbiek v´ altozatlan teljes´ıtm´enye mellett a matematika a´tlag legal´ abb 3,8 legyen? (6 pont)

kibontva A 2 D1 · A 1 A 3 A2 B1 · E1 A3 = , B1 A3 · A2 E1 D1 A 3 · A 2 A 1 A 2 D1 A2 B1 E1 A3 · A1 A2 A2 B1 A1 A22 = · = · . D1 A 3 B1 A3 A2 E1 · A1 A3 B1 A3 A1 A23

Pach P´eter P´al

K¨ oz´ episkolai Matematikai ´ es Fizikai Lapok, 2022/2

3

71

3. Az Andr´assy gimn´ azium g´ olyat´ abor´anak egy sportverseny´ehez a k¨ ovetkez˝ o p´alya k´esz¨ ult el az udvar betonj´ an: egy k¨ orben az egy pontb´ ol kiindul´o 10 m ´es 15 m oget z´ arnak be. A j´ at´ekosoknak a h´ urok (nem hossz´ u h´ urok egym´assal 35◦ -os sz¨ 72

K¨ oz´ episkolai Matematikai ´ es Fizikai Lapok, 2022/2

k¨ oz¨ os) v´egpontj´ ab´ol indulva kell megszerezni a k¨ or k¨oz´eppontj´aban l´ev˝ o labd´at, majd visszafutni a kiindul´asi hely¨ ukre. a) K´esz´ıts¨ unk a sz¨ ovegnek megfelel˝o a´br´ at a l´enyeges adatok felt¨ untet´es´evel. (2 pont) b) Legal´ abb mekkora utat kell megtenni egy-egy j´ at´ekosnak? (4 pont) c) Kata azt a´ll´ıtja, hogy a j´at´ekosok kiindul´asi helye ´es a labda alkotta h´ aromanak? (6 pont) sz¨ og ter¨ ulete legfeljebb 25 m2 . Igaza van-e Kat´ u k¨or ´es a P (3; 9) pont. 4. Adott az x2 + y 2 − 14x − 12y + 65 = 0 egyenlet˝ a) A rajta”, k´ıv¨ ul”, bel¨ ul” szavak k¨ oz¨ ul ´ırjuk a pontozott vonalra azt, ame” ” ” lyikkel az al´abbi a´ll´ıt´ as igaz lesz. Sz´amol´assal is igazoljuk a v´ alaszt. (3 pont) A P pont . . . . . . . . . . . . van a fent megadott egyenlet˝ u k¨orvonalon. b) Hat´ arozzuk meg az orig´ on ´es a P ponton a´tmen˝ o g egyenesnek az x tengely pozit´ıv fel´evel bez´ art sz¨ og´et. A sz¨ og ´ert´ek´et egy tizedesjegy pontoss´ aggal adjuk meg. (3 pont) c) ´Irjuk fel a megadott k¨ or azon ´erint˝oj´enek egyenlet´et, amelynek nincs k¨oz¨os pontja a III. s´ıknegyeddel, nem megy ´at az orig´on, ´es ami az y tengelyt az orig´oavols´ agban metszi, mint az x tengelyt, tov´ abb´ a a tengelyekkel t´ ol k´etszer olyan t´ alkotott h´ aromsz¨ og ter¨ ulete a legkisebb. (7 pont) II. r´esz 5. a) Egy {an } sz´ amtani sorozat differenci´aja 4, az els˝ o n tag ¨osszege 1825, ´es az els˝o tag megegyezik ezen ¨ osszeadott tagok sz´ am´aval. Tagja-e ennek az {an } sorozatnak a 8115? (6 pont) Julcsi 2018. janu´ar 1-j´en betett a bankba egy bizonyos ¨osszeget, ´evi 5%-os kamatra u ´gy, hogy a banksz´ aml´ aj´an a minden ´ev v´eg´en esed´ekes kamatokat t˝ ok´es´ıtette (nem vette ki). H´ arom ´ev elteltekor megemelte a megtakar´ıtott p´enz´et az ´eppen bent l´ev˝ onek a 20%-´ aval. Ett˝ ol kezdve m´ar csak 4%-os ´evi kamatot kapott a bankint´ezett˝ ol. A k¨ ovetkez˝ o ´ev els˝ o napj´ an pedig kivette az addig megtakar´ıtott p´enz´enek a 10%-´ at. 2025. janu´ ar 1-j´en szeretn´e felvenni az ¨osszes p´enz´et. Testv´ere, Anna egyszerre kezdett vele takar´ekoskodni ugyanakkora ¨osszeggel. b) H´ any %-os, a´lland´ o ´evi kamatot kellene kapnia ahhoz Ann´ anak bet´et ´es kiv´et n´elk¨ uli takar´ekoss´ ag eset´en, hogy a k´et testv´ernek ugyanannyi megtakar´ıtott p´enze legyen 2025 els˝o napj´ an? A kamatl´ ab ´ert´ek´et egy tizedes pontoss´ aggal adjuk meg. (4 pont) ol ´all´o, Kar´ acsonyra Franci az ´ abr´ an l´athat´ o, n´egy cikkb˝ tengelyesen szimmetrikus feny˝ofa d´ıszt kezdte el rajzolni egy pap´ırra. Egy 5 cm sugar´ u f´elk¨ orb˝ ol indult ki, eggyel feljebb l´epve, a k¨ orcikk sugar´ at a fel´ere, k¨ oz´epponti sz¨ og´et 23 r´esz´ere v´ altoztatta, mik¨ ozben a k¨ orcikk k¨ oz´eppontj´at a felette l´ev˝ o k¨ or´ıv felez´esi pontj´aba tette. Elgondolkodott azon, ha ezt az elj´ar´ ast v´egtelen sok´ aig tudn´ a folytatni, lenne-e a mint´ anak v´eges ter¨ ulete. K¨ oz´ episkolai Matematikai ´ es Fizikai Lapok, 2022/2

73

c) Ha igen, pontosan mekkora lenne, ´es az h´ any %-a annak a minim´alis ter¨ ulet˝ u t´eglalap ter¨ ulet´enek, amelynek k´et oldala p´ arhuzamos a feny˝ ofa tengely´evel? (6 pont) ´ EK ´ Kft. u 6. a) A JAT ´j homokoz´ o v¨ odre csonkak´ up alak´ u. Ha a v¨ od¨ orbe beletesz¨ unk egy 14 cm a´tm´er˝ oj˝ u labd´ at, akkor az ´erinti a v¨ od¨ or alj´at, ´es egy k¨ orben a v¨od¨or oldal´ at is. A v¨ od¨ or alj´anak ´atm´er˝ oje 12 cm, fels˝ o ny´ıl´as´ anak ´atm´er˝ oje 18 cm. A v¨odr¨ot k´ıv¨ ul is, bel¨ ul is v´ız´ all´ o r´eteggel festik be. H´ any liter fest´ekre van sz¨ uks´eg ulet fest´es´ehez 0,5 dl 1000 darab v¨od¨ or elk´esz´ıt´esekor, ha 1 n´egyzetm´eternyi fel¨ fest´eket haszn´ alnak, ´es a v¨ od¨ or falvastags´ aga elhanyagolhat´ o? (8 pont) b) Panni ´es Peti kokt´elos poharakb´ol bodzasz¨ orp¨ ot iszik. A poh´ ar fels˝o r´esze forg´ ask´ up alak´ u, melynek magass´ aga 8 cm, alkot´ oja 10 cm, ´es 7 cm magasan ´all benne” az u d´ ıt˝ o . H´ a ny millim´ e terrel ¨ ” emelkedik meg a bodzasz¨ orp szintje”, ha a poh´ arba h´ arom darab, ” 2 cm ´el˝ u j´egkock´ at tesz¨ unk, ´es azok m´ar teljesen elolvadtak? A v´alaszt eg´eszre kerek´ıtve adjuk meg. (Tekints¨ unk el a j´eg olvad´ asakor k¨ ozismerten bek¨ ovetkez˝ o t´erfogatv´ altoz´ ast´ ol.) (8 pont) 7. Adott k´et teljes gr´ af. Az els˝ o gr´ afnak 4-gyel t¨ obb cs´ ucsa ´es 62-vel t¨ obb ´ele van, mint a m´asodiknak. a) H´any cs´ ucsa van annak a teljes gr´ afnak, amelynek annyi ´ele van, mint az adott k´et teljes gr´ af ´elei ´es cs´ ucsai sz´ am´anak ¨ osszege? (6 pont) √ uggv´eny ´ertelmez´esi tartom´aLegyen az A halmaz az f (x) = −2x2 + 5x + 3 f¨ otlens´eg megold´ ashalnya, a B halmaz pedig a log 3 (4x − 3) > log 3 (2x + 7) egyenl˝ π π maza. b) Hat´ arozzuk meg az A ∩ B, az A ∪ B ´es az A \ B halmazokat. (6 pont) T´ız bar´ at, Anna, Bea, Cili, D´ ori, Emese, Fruzsi, G´abor, Huba, Istv´ an ´es J´ anos moziba megy. A jegyek az els˝ o sorba egym´ as mell´e sz´ olnak. c) H´anyf´elek´eppen u u azt k´erte, hogy mindegyik¨ uk ¨lhetnek le, ha a n´egy fi´ k¨ozvetlen¨ ul k´et l´ any k¨ oz¨ ott u (4 pont) ¨lhessen? 8. Adott a NoSelejt c´eg ´ altal gy´ artott valamely term´ek K(x) = x3 − 12x2 + + 48x k¨olts´egf¨ uggv´enye ´es a B(x) = 300x bev´etelf¨ uggv´enye, ahol x az el˝o´ all´ıtott mennyis´eget jelenti (sz´ az darabban), m´ıg a K(x) ´es a B(x) ´ert´ekei milli´o forintban ´ertend˝ ok. A nyeres´eget a bev´etelek ´es a k¨ olts´egek k¨ ul¨ onbs´ege adja. a) Hat´arozzuk meg azt a term´ekmennyis´eget, amely eset´en a c´eg nyeres´ege maxim´alis. (5 pont) b) Hat´arozzuk meg a K(x) f¨ uggv´eny grafikonja ´es a grafikon x = 3 abszcissz´ aj´ u pontj´ ahoz tartoz´o ´erint˝o ´ altal hat´ arolt korl´atos, z´ art s´ıkidom ter¨ ulet´enek m´er˝ osz´ am´at. (9 pont) c) Adjunk p´eld´ at olyan egyv´ altoz´ os val´os f f¨ uggv´enyre (ha l´etezik), amely diffeo´ert´ekhelye renci´ alhat´ o a val´ os sz´amok halmaz´an, ´es f  (3) = 0, de az x = 3 nem sz´els˝ f -nek. (2 pont) 74

K¨ oz´ episkolai Matematikai ´ es Fizikai Lapok, 2022/2

9. Peches P´ al nagyon szereti a kapar´os sorsjegyeket. Kedvence a Lutri sorsjegy, melynek a´ra 500 Ft, ´es a sorsjegyek 25%-a nyer˝ o. P´alnak (most csak) n´egy darab 500 forintosa van. Bemegy egy lott´ oz´ oba, ´es elhat´ arozza, hogy addig v´ as´ arolja kedvenc sorsjegy´et, am´ıg nem nyer, vagy ameddig a p´enze el nem fogy. a) Hat´ arozzuk meg a P´al ´ altal a sorsjegy(ek)re elk¨olt¨ott 500 forintosok sz´am´anak v´ arhat´ o ´ert´ek´et ´es sz´ or´ as´ at. (7 pont) P´al h´ aromf´ele t¨ omegk¨ ozleked´esi eszk¨ ozzel tudja munkahely´et megk¨ozel´ıteni, ´espedig busszal, metr´oval, illetve villamossal, ez´ert (is) kombin´alt b´erlettel rendelkezik. Az esetek 25%-´ aban busszal megy, a metr´ot pedig n´egyszer olyan gyakran haszn´ alja, mint a villamost. A buszon a´tlagosan minden negyedik, a villamoson ´atlagosan minden tizedik alkalommal ellen˝ orzik a b´erlet´et, m´ıg annak a val´ osz´ın˝ us´ege, hogy a metr´on kap ellen˝orz´est, 0,85. b) Egyik alkalommal ellen˝orizt´ek a b´erlet´et. Mennyi annak a val´ osz´ın˝ us´ege, hogy villamossal utazott? (6 pont) Egyik nap (a munkanap v´eg´en) P´ al egy ¨ otf˝ os bar´ ati t´ arsas´ ag tagjak´ent busszal utazott haza. Az egyik meg´ all´ oban ellen˝or¨ ok sz´alltak fel, ´es a buszon (aktu´ alisan) tart´ ozkod´ o 48 utasb´ ol tal´ alomra kiv´ alasztott t´ız embernek a b´erlet´et (vagy jegy´et) ellen˝ orizt´ek. c) Mennyi annak a val´ osz´ın˝ us´ege, hogy az ¨ otf˝ os bar´ ati t´ arsas´ agb´ol legal´abb k´et f˝ot ellen˝oriztek? (3 pont) Marczis Gy¨orgy (Gyula) Moln´ar Istv´an (Gyula) Moln´ar Judit (Gyula) R´ok´an´e R´ ozsa Anik´ o (B´ek´escsaba)

M´asodik

351,00

352,00

Harmadik

350,00

352,50

Illet´ek nincs a tranzakci´ o ¨osszeg´enek 0,3%-a, de maximum 1500 Ft 400 Ft

Megold´as. a) A legkisebb 77-tel oszthat´ o n´egyjegy˝ u pozit´ıv eg´esz sz´ am az 1001. Ez nem j´o, mert csak k´et k¨ ul¨ onb¨ oz˝ o sz´ amjegyet tartalmaz. A k¨ovetkez˝ o az 1078, ez sem j´ o, mert ez pedig n´egyet. A k¨ovetkez˝ o az 1155, ez sem j´ o, mert ez megint csak kett˝ ot.

A v´eteli ´ arfolyam adja meg, hogy a valutav´ alt´ o h´ any Ft-´ert vesz meg az u ¨gyf´elt˝ ol 1 eur´ ot. Az elad´ asi ´ arfolyam adja meg, hogy a valutav´ alt´ o h´ any Ft-´ert ad el az u ot. V´eg¨ ul az illet´ek adja meg, hogy minden egyes p´enzv´ alt´ asi ¨gyf´elnek 1 eur´ tranzakci´ o ut´ an mekkora d´ıjat kell pluszban kifizetni. K¨ oz´ episkolai Matematikai ´ es Fizikai Lapok, 2022/2

M´asodikn´ al: 250 · 352 = 88 000 Ft, plusz ennek a 0,3%-a, ami 264 Ft, teh´at ¨osszesen 88 264 Ft. Harmadikn´ al: 250 · 352,50 + 400 = 88 525 Ft. 000 b) A 600 ezer Ft-´ert 600 ≈ 1700 eur´ ot kapott. A p´enz visszav´ alt´ asakor 352,90 1700 · 351 = 596 700 Ft-ot kapna vissza, de ki kell fizetnie az illet´eket. Az 596 700 Ft 0,3%-a 1790,10 Ft, de ez meghaladja az illet´ek maxim´ alis ¨ osszeg´et, teh´ at 1500 Ft illet´ek terheli a tranzakci´ot. ´Igy 596 700 − 1500 = 595 200 Ft-ot kap k´ezhez, teh´ at 4800 Ft vesztes´ege keletkezik a k´et tranzakci´on. c) Ha n eur´ ot v´ altunk, akkor a Harmadik P´enzv´ alt´ o aj´ anlata abban az esetben kedvez˝ obb az Els˝o P´enzv´ alt´ o aj´anlat´ an´ al, ha 352,5n + 400 < 352,9n. Ebb˝ ol 400 < 0,4n, azaz 1000 < n. A Harmadik P´enzv´ alt´ o aj´anlata abban az esetben kedvez˝ obb a M´ asodik P´enzv´alt´ o aj´ anlat´ an´ al, ha 352,5n + 400 < 352n · 1,003 ´es 352,5n + 400 < 352n + 1500. Innen egyr´eszt 352,5n + 400 < 353,056n, azaz 400 < 0,556n, azaz 719,42 < n, m´ asr´eszt 0,5n < 1100, azaz n < 2200.

b) H´ any olyan n´egyjegy˝ u pozit´ıv eg´esz sz´ am van, amelyik pontosan h´ arom k¨ ul¨ onb¨ oz˝ o sz´ amjegyet tartalmaz? (4 pont) c) H´ any olyan n´egyjegy˝ u pozit´ıv eg´esz sz´ am van, amely a 7 ´es a 11 k¨ oz¨ ul legal´ abb az egyikkel oszthat´ o? (4 pont)

1. H´ arom p´enzv´ alt´ o v´ allalkoz´ as aktu´ alis forint-eur´ o´ arfolyamait ismerj¨ uk:

Els˝ o

Megold´as. a) Els˝on´el: 250 · 352,90 = 88 225 Ft.

2. a) Melyik az a legkisebb olyan 77-tel oszthat´ o n´egyjegy˝ u pozit´ıv eg´esz sz´ am, amelyik pontosan h´ arom k¨ ul¨ onb¨ oz˝ o sz´ amjegyet tartalmaz? (4 pont)

I. r´esz

Elad´as 352,90

c) Hat´ arozzuk meg, h´ any eur´ o v´ as´ arl´ asa eset´en lesz a Harmadik P´enzv´ alt´ o´e a legkedvez˝ obb ´ atv´ alt´ asi aj´ anlat. (7 pont)

Ezeket o¨sszevetve teh´ at 1000 ´es 2200 k¨ oz¨ otti mennyis´eg˝ u eur´ o v´ alt´ asa eset´en lesz a Harmadik P´enzv´ alt´ o aj´ anlata a legkedvez˝ obb.

Megold´asv´azlatok a 2022/1. sz´am emelt szint˝ u matematika gyakorl´o feladatsor´ahoz

V´etel 348,50

a) Ann´ anak 250 eur´ ora volt sz¨ uks´ege. Mennyit kellene ez´ert fizetnie az egyes p´enzv´ alt´ okn´ al? (3 pont) b) Bal´ azs 600 000 Ft-´ert vett eur´ ot az Els˝ o P´enzv´ alt´ on´ al. K´es˝ obb kider¨ ult, hogy nem lesz r´ a sz¨ uks´ege, ez´ert visszav´ altotta a p´enzt forintra a M´ asodik P´enzv´ alt´ on´ al. H´ any forint vesztes´ege keletkezett? (4 pont)

75

A k¨ovetkez˝ o az 1232, ez pontosan h´ arom k¨ ul¨ onb¨ oz˝ o sz´ amjegyet tartalmaz, ez´ert ez a keresett sz´ am. b) A h´ arom k¨ ul¨ onb¨ oz˝ o sz´ amjegy k¨ oz¨ ul az egyik k´etszer, a m´asik kett˝ o egyszer fordul el˝ o a sz´ amban. 76

K¨ oz´ episkolai Matematikai ´ es Fizikai Lapok, 2022/2

Ha az ism´etl˝od˝ o sz´ amjegyek egyik´evel kezd˝ odik a sz´ am, akkor a p´ arja 3-f´ele helyen ´ allhat. Ez a sz´ amjegy (mivel 0-val nem kezd˝ odhet sz´ am) 9-f´elek´eppen, a m´ asik k´et sz´ amjegy 9, illetve 8-f´elek´eppen v´ alaszthat´ o ki. Ez 3 · 9 · 9 · 8 (= 1944) lehet˝ os´eg.

Ha nem az ism´etl˝od˝ o sz´ amjegyek egyik´evel kezd˝ odik a sz´ am, akkor a k´et ism´etl˝ od˝o sz´ amjegy 3-f´ele helyen a´llhat. Az els˝ o sz´ amjegy (mivel 0-val nem kezd˝odhet sz´ am) 9-f´elek´eppen, a m´ asik k´et sz´ amjegy 9, illetve 8-f´elek´eppen v´ alaszthat´ o ki. Ez ism´et 3 · 9 · 9 · 8 (= 1944) lehet˝ os´eg. Azaz o o n´egyjegy˝ u sz´am van. ¨sszesen 3888 a felt´eteleknek megfelel˝ + 1 = 1286 darab 7-tel oszthat´ o sz´ am van. c) 1001 ´es 9996 k¨oz¨ ott 9996−1001 7

innen az els˝o tag 8 ´es a h´ anyados 1/2, vagy pedig az els˝o tag −27 ´es a h´ anyados −1/3. Ellen˝ orz´es: 8 + 4 = 6 · 2, illetve −27 + 9 = 6 · (−3), ´es a 4. tag mindk´et esetben val´oban 1. 4. a) Igaz-e a k¨ ovetkez˝ o´ all´ıt´ as? ammal egyenl˝ o. Ha x = 3, akkor f (x) = 2x2 − 10x + 14 ´ert´eke pozit´ıv pr´ımsz´ Fogalmazzuk meg az ´ all´ıt´ as megford´ıt´ as´ at. Igaz-e az a ´ll´ıt´ as megford´ıt´ asa? A v´ alaszt indokoljuk. (5 pont) b) Oldjuk meg az al´ abbi egyenletet a val´ os sz´ amok halmaz´ an:

1001 ´es 9999 k¨ oz¨ ott 9999−1001 + 1 = 819 darab 11-gyel oszthat´ o sz´ am van. Ezek 11

k¨ oz¨ ul 7-tel ´es 11-gyel (teh´ at 77-tel) is oszthat´o 9933−1001 + 1 = 117 darab. 77 Ezeket a 7-tel ´es a 11-gyel oszthat´ oak k¨ oz¨ ott is megsz´ amoltuk, teh´ at 7 ´es 11 k¨ oz¨ ul legal´ abb az egyikkel oszthat´ o (1286 + 819 − 117 =)1988 darab n´egyjegy˝ u sz´ am. 3. a) Egy sz´ amtani sorozat els˝ o 10 tagj´ anak o ovet˝ o ¨sszege megegyezik az ezt k¨ 5 tag ¨ osszeg´evel. A sorozat 19-edik tagja a 777. Hat´ arozzuk meg a sorozat els˝ o tagj´ at ´es differenci´ aj´ at. (7 pont) b) Egy m´ertani sorozat els˝ o 2 tagj´ anak ¨ osszege hatszorosa a sorozat harmadik tagj´ anak. A sorozat 4-edik tagja az 1. Hat´ arozzuk meg a sorozat els˝ o tagj´ at ´es h´ anyados´ at. (6 pont) aj´at pedig d. Megold´as. a) Jel¨ olje a sz´ amtani sorozat n-edik tagj´at an , differenci´ A sz´ amtani sorozat ¨ osszegk´eplet´evel: (a11 + a15 ) · 5 (a1 + a10 ) · 10 = . 2 2

|2 sin2 x + 3 sin x − 1| = 1.

Megold´as. a) Igaz, hiszen f (3) = 2, ami val´oban pr´ım. Az ´all´ıt´as megford´ıt´ asa: Ha f (x) = 2x2 − 10x + 14 ´ert´eke pozit´ıv pr´ımsz´ am, akkor x = 3. A megford´ıtott ´ all´ıt´ as hamis. ara rendez´es ut´ an x1 = 3 vagy P´eld´ aul f (x) = 2x2 − 10x + 14 = 2 eset´en null´ odik, teh´ at nem biztos, hogy x = 3. x2 = 2 ad´ b) Az abszol´ ut´ert´ek jelet elhagyva: 2 sin2 x + 3 sin x − 1 = 1

2 sin2 x + 3 sin x − 2 = 0 vagy

2 sin2 x + 3 sin x = 0.

Az egyenletek sin x-ben m´asodfok´ uak, gy¨ okeik 1/2 ´es −2, illetve 0 ´es −1,5, melyek k¨oz¨ ul (sin x ´ert´ekk´eszlete miatt) csak sin x = 1/2 ´es sin x = 0 lehets´eges. Ekkor + 2lπ vagy x = mπ (k, l, m ∈ Z). x = π6 + 2kπ vagy x = 5π 6 Ekvivalens a´talak´ıt´ asokat v´egezt¨ unk.

II. r´esz

(2a1 + 9d) · 10 (2a1 + 24d) · 5 = , 2 2

5. Nagyi a 31,5 cm × 30 cm (bels˝ o) m´eret˝ u tepsij´eben s¨ ut¨ ott s¨ utem´enyt az unok´ ainak. A s¨ utem´eny 4 cm magas lett. Nagyi a s¨ utem´eny n´egy oldal´ at ´es a tetej´et be szeretn´e vonni csokikr´emmel.

2a1 + 9d = a1 + 12d, a1 = 3d. Ezt felhaszn´alva a19 = a1 + 18d = 21d = 777, ahonnan d = 37, teh´at a1 = 111. Ellen˝ orz´es: a10 = 444, a11 = 481, a15 = 629, az els˝ o 10, ´es az ezt k¨ovet˝o 5 tag osszege val´ oban egyenl˝ o (2775). ¨ b) Jel¨ olje a m´ertani sorozat n-edik tagj´at an , h´anyados´ at pedig q. a1 + a1 q = 6a1 q 2 . A nemnulla a1 -gyel osztva ´es rendezve: 0 = 6q 2 − q − 1. Az egyenlet gy¨okei (teh´at a h´ anyados lehets´eges ´ert´ekei) 1/2 ´es −1/3. a4 1 = 3, 3 q q

K¨ oz´ episkolai Matematikai ´ es Fizikai Lapok, 2022/2

vagy 2 sin2 x + 3 sin x − 1 = −1,

teh´at

A 10., 11. ´es 15. tagot az els˝ o tag ´es a differencia seg´ıts´eg´evel a´t´ırva:

a1 =

(7 pont)

77

a) H´ any dkg csokikr´emre lesz ehhez sz¨ uks´ege, ha 1 dm2 fel¨ ulet bevon´ as´ ahoz 2 dkg csokikr´em elegend˝ o? A v´ alaszt eg´eszre kerek´ıtve adjuk meg. (3 pont) Az unok´ ai k¨ oz¨ ul ugyanannyian szeretik a s¨ utem´eny sz´el´et”, mint a k¨ ozep´et”. ” ” Ez´ert Nagyi szeretne a s¨ utem´eny sz´el´eb˝ ol mind a n´egy oldalon egy azonos sz´eless´eg˝ u cs´ıkot lev´ agni u ´gy, hogy a lev´ agott r´eszek alapter¨ ulete ´es a s¨ utem´eny k¨ ozep´enek alapter¨ ulete egyenl˝ o legyen. b) Hat´ arozzuk meg a lev´ agand´ o cs´ık sz´eless´eg´et. (7 pont) Nagyi minden unok´ aj´ anak ugyanannyi szeletet szeretne adni a s¨ utem´enyb˝ ol. Ha 10 · 5 szeletre v´ agn´ a a s¨ utem´enyt, akkor az oszt´ as ut´ an 2 szelet megmaradna. agn´ a, akkor 3 szelet, ha pedig 10 · 4 szeletre v´ agn´ a, akkor 4 szelet Ha 9 · 5 szeletre v´ maradna meg az oszt´ as ut´ an. c) H´ any unok´ aja van Nagyinak? (6 pont) 78

K¨ oz´ episkolai Matematikai ´ es Fizikai Lapok, 2022/2

Megold´as. a) A bevonand´ o fel¨ ulet ¨ ot t´eglalapb´ ol ´all, ezek ter¨ ulet¨osszege:

A be´ırhat´ o k¨or r sugara egyben az A1 OA2 egyenl˝ o sz´ ar´ u h´ aromsz¨ og alaphoz tartoz´ o magass´ aga: r = 25 · tg 72◦ ≈ 76,94 cm.

A = 31,5 · 30 + 2 · (31,5 · 4 + 30 · 4) = 1437 cm2 = 14,37 dm2 . Ennek bevon´ as´ahoz 14,37 · 2 ≈ 29 dkg csokikr´emre lesz sz¨ uks´ege.

b) A lev´ agott cs´ık sz´eless´eg´et jel¨ olje x. Ekkor a k¨oz´epen megmarad´ o t´eglalap m´eretei: 31,5 − 2x, illetve 30 − 2x. Ennek a t´eglalapnak a ter¨ ulete fele a teljes s¨ utem´eny alapter¨ ulet´enek: (31,5 − 2x)(30 − 2x) =

A k¨or alak´ u ter´ıt˝ o ter¨ ulete Tter´ıt˝o = r2 π ≈ 18 600 cm2 = 1,86 m2 . b) Az asztal ter¨ ulet´et a 10 darab egyenl˝ o sz´ ar´ u h´ aromsz¨ og ter¨ ulet´enek ¨ osszegek´ent hat´ arozzuk meg:

31,5 · 30 . 2

Tasztal = 10 ·

Rendez´es ut´ an: 4x2 − 123x + 472,5 = 0. Ennek az egyenletnek a gy¨okei 26,25 (mely nyilv´ an nem megold´ asa a feladatnak) ´es 4,5. Teh´ at 4,5 cm sz´eless´eg˝ u cs´ıkot kell Nagyinak lev´agnia. Ellen˝ orz´es: a 22,5 × 21-es r´esz ter¨ ulete (472,5) val´oban fele a teljes s¨ uti alapter¨ ulet´enek (945).

600 Ha sehol nem l´ og le az asztalr´ol a ter´ıt˝ o, akkor az asztal ter¨ ulet´enek 18 · 100 ≈ 19 235 ≈ 96,7%-´ at fedi le.

otsz¨ oc) Az A8 A9 A10 A1 F1 ´es A10 A1 A2 A3 F2 ¨ gek egybev´ ag´ ok, mert a megfelel˝o oldalaik hossza ´es a k¨ ozbez´ art sz¨ ogeik is megegyeznek (vagy egy as a k´et alakza– az O pont k¨ or¨ uli – 72◦ -os forgat´ tot egym´asba viszi). Mindk´et ¨ otsz¨ ogb˝ ol elhagyva a k¨ oz¨ os A10 A1 F1 M n´egysz¨ oget, a marad´ek ter¨ uleteknek is meg kell egyezni¨ uk.

c) 50 szeletb˝ ol 2 megmarad, teh´ at a 48 oszthat´ o az unok´ ak sz´am´aval. 45 szeletb˝ol 3 megmarad, teh´ at a 42 oszthat´ o az unok´ ak sz´am´aval. 40 szeletb˝ol 4 megmarad, teh´ at a 36 oszthat´ o az unok´ ak sz´am´aval. Az unok´ak sz´ama ez´ert csak a 36, 42 ´es a 48 k¨oz¨os oszt´ oi k¨oz¨ ul ker¨ ulhet ki, ezek: 2, 3 ´es 6. 2 ´es 3 azonban nem lehet az unok´ak sz´ama, mert akkor vagy az 50 vagy a 45 szeletb˝ol nem maradt volna az oszt´ as ut´an, teh´ at Nagyinak 6 unok´ aja van. 6. Egy szab´ alyos 10-sz¨ og alak´ u asztal egy oldal´ aor alanak hossza 50 cm. Erre az asztalra egy olyan k¨ k´ u ter´ıt˝ ot k´esz´ıtenek, amely sehol nem l´ og le az asztalr´ ol. a) Hat´ arozzuk meg a legnagyobb ilyen ter´ıt˝ o ter¨ ulet´et. (3 pont) b) Legfeljebb h´ any sz´ azal´ek´ at tudja lefedni ez a ter´ıt˝ o az asztal ter¨ ulet´enek? (3 pont)

d) El˝osz¨or ¨ osszesz´ amoljuk, h´ any olyan tompasz¨ og˝ u h´aromsz¨ og van, melynek og˝ u h´ aromsz¨ og eset´en a k¨ or¨ ul´ırt k¨ or O k¨ oz´eppontja tompasz¨og˝ u cs´ ucsa A1 . Tompasz¨ a h´aromsz¨og¨on k´ıv¨ ul van. os´eg van (A8 -t´ ol A10 -ig). Ha a m´asodik cs´ ucs A2 , 3 lehet˝ Ha a m´asodik cs´ ucs A3 , 2 lehet˝ os´eg (A9 -t˝ ol A10 -ig). Ha a m´asodik cs´ ucs A4 , 1 lehet˝ os´eg (A10 ). Ha a tompasz¨ og˝ u cs´ ucs A1 , akkor teh´at 6 h´ aromsz¨ og van.

Jel¨ olje F1 az A1 A2 ´es F2 az A3 A4 szakaszok felez˝ opontj´ at. Az asztallapot ak. Jel¨ olje M a k´et egyenes az A8 F1 ´es az A10 F2 egyenesekkel n´egy r´eszre osztj´ metsz´espontj´ at. og ´es az F2 A3 A2 F1 M o og ter¨ ulete c) Igazoljuk, hogy az A10 A9 A8 M n´egysz¨ ¨tsz¨ egyenl˝ o. (4 pont) Egy szab´ alyos 10-sz¨ og cs´ ucsai k¨ oz¨ ul v´eletlenszer˝ uen kiv´ alasztunk h´ armat, ´ıgy egy h´ aromsz¨ og cs´ ucsait kapjuk. d) Mennyi annak a val´ osz´ın˝ us´ege, hogy a h´ aromsz¨ og tompasz¨ og˝ u? (6 pont) Megold´as. a) A lehets´eges legnagyobb k¨ or alak´ u ter´ıt˝o a szab´ alyos 10-sz¨ogbe ´ırhat´ o k¨ ornek felel meg, melynek k¨ oz´eppontj´at jel¨olj¨ uk O-val. A szab´ alyos 10arsz¨og˝ u egyenl˝ o sz´ ar´ u h´aromsz¨ogre. sz¨ og felbonthat´o 10 darab egybev´ag´ o, 36◦ -os sz´ K¨ oz´ episkolai Matematikai ´ es Fizikai Lapok, 2022/2

50 · 76, 94 = 19 235 cm2 . 2

79

B´armelyik cs´ ucsn´ al lehet a tompasz¨ 10og, ´ıgy a tompasz¨og˝u h´aromsz¨ogek sz´ama 10 · 6 = 60. A 10 cs´ ucs k¨ oz¨ ul h´ armat 3 = 120-f´elek´eppen tudunk kiv´ alasztani. 60 = 0,5. A keresett val´ osz´ın˝ us´eg 120

7. Az egyetemen 220 di´ ak ´ırt meg egy dolgozatot, az ´ atlag sz´ azadokra kerek´ıtve 3,82 lett. (Csak az 1, 2, 3, 4, 5 eg´esz ´ert´ek˝ u oszt´ alyzatok lehettek az eredm´enyek.) a) Legal´ abb ´es legfeljebb h´ any 5-¨ os dolgozat sz¨ uletett, ha nem volt 1-es? (7 pont) Egy szab´ alyos dob´ okock´ aval h´ aromszor egym´ as ut´ an dobunk. b) Hat´ arozzuk meg annak a val´ osz´ın˝ us´eg´et, hogy valamelyik dobott sz´ am a m´ asik k´et dobott sz´ amnak sz´ amtani vagy m´ertani k¨ ozepe lesz. (6 pont) c) Hat´ arozzuk meg annak a val´ osz´ın˝ us´eg´et, hogy a dobott sz´ amok k¨ oz¨ ott van 6-os, felt´eve, hogy valamelyik dobott sz´ am a m´ asik k´et dobott sz´ amnak a sz´ amtani (3 pont) vagy m´ertani k¨ ozepe. 80

K¨ oz´ episkolai Matematikai ´ es Fizikai Lapok, 2022/2

Megold´as. a) A dolgozatok pontos x ´ atlag´ ara 3,815  x < 3,825. A pontsz´amok S ¨ osszeg´ere 3,815 · 220  S < 3,825 · 220, azaz 839,3  S < 841,5, teh´at S = 840 vagy S = 841.

ahonnan x = 0 vagy x = 6. A g¨ orbe ´es az x-tengely ´altal hat´ arolt ter¨ uletet teh´ at (a Newton–Leibniz-szab´ aly felhaszn´al´as´ aval) az

Legt¨ obb 5-¨ os akkor lehets´eges, ha a 2-esek (a gyenge oszt´ alyzatok) sz´ ama min´el nagyobb. Ha z darab 5-¨ os volt, akkor legfeljebb 220 − z lehet a 2-esek sz´ ama. Ebb˝ ol , azaz (figyelembe v´ e ve, hogy S  (220 − z) · 2 + z · 5 = 3z + 440. Innen z  S−440 3 z eg´esz sz´ am) z  133 ad´odik.

8 4 x − x2 3 9

6  0

Ha az a ´ert´ek˝ u dob´ as a m´asik k´et (nem a ´ert´ek˝ u) dob´ as sz´amtani k¨ozepe, akkor a m´ asik k´et dob´as a − d ´es a + d, valamilyen alkalmas d-re. d = 1 eset´en a lehets´eges ´ert´ekei 2, 3, 4 vagy 5. d = 2 eset´en a lehets´eges ´ert´ekei 3 vagy 4.

d > 2 nem lehets´eges. Ha az a ´ert´ek˝ u dob´ as a m´asik k´et (a-val nem egyenl˝o) b ´es c ´ert´ek˝ u dob´ as o m´ertani k¨ ozepe, akkor a2 = bc, teh´at az egyik dob´as ´ert´ek´enek n´egyzete egyenl˝ a m´ asik kett˝ onek a szorzat´ aval. A dob´ asok lehets´eges ´ert´ekeit figyelembe v´eve csak at a h´ arom dob´ as 1, 2 ´es 4. 22 = 1 · 4 lehets´eges, teh´

Mind a h´et felsorolt esetben a h´ arom k¨ ul¨ onb¨ oz˝ o ´ert´eket 3! = 6-f´ele sorrendben ¨ dobhatjuk, ez teh´ at ¨ osszesen 42 lehet˝ os´eget jelent. Osszesen 6 + 42 = 48 a felt´eteleknek megfelel˝ o dob´ assorozat van. Az ¨ osszes lehets´eges dob´ assorozatok sz´ ama 48 osz´ın˝ us´eg teh´at 216 = 29 ≈ 0,222. 63 = 216, a keresett val´ c) A b) feladatban ¨ osszesz´ amolt 48 megfelel˝o dob´assorozat (e feladat tekintet´eben az ¨ osszes eset) k¨ oz¨ ul a kedvez˝ o esetek azok, amelyekben van 6-os. Ezek: 2–4–6 (6-f´ele lehets´eges sorrend), 4-5-6 (6-f´ele lehets´eges sorrend) ´es 6-6-6 (1-f´ele lehets´eges sorrend). Ez ¨ osszesen 13 dob´assorozat. A keresett val´osz´ın˝ us´eg teh´ at 13 ≈ 0,271. 48 8. a) Az y = 83 x − 49 x2 egyenlet˝ u g¨ orbe ´es az x-tengely a ´ltal hat´ arolt z´ art tar4 tom´ anyt k´et r´eszre osztja az y = 3 x egyenlet˝ u egyenes. Hat´ arozzuk meg a k´et r´esz ter¨ ulet´enek ar´ any´ at. (8 pont) b) Egy h´ aromsz¨ og cs´ ucsai a koordin´ ata-rendszerben A(0; 0), B(3; 0) ´es C(3; 4). A h´ aromsz¨ oget megforgatjuk a leghosszabb oldala k¨ or¨ ul. Hat´ arozzuk meg az ´ıgy kapott forg´ astest felsz´ın´et ´es t´erfogat´ at. (8 pont)

6  0



dx =



4 2 4 x − x3 3 27

6 0

= 48 − 32 = 16 ter¨ uletegys´eg.

A g¨orbe ´es az egyenes k¨ oz¨ os pontjait a 4 8 4 x − x2 = x 3 9 3 egyenlet megold´ as´ab´ ol kapjuk.   4 1 x 1 − x = 0, 3 3 ahonnan x = 0 vagy x = 3, a k¨ oz¨ os pontok teh´at (0; 0) ´es (3; 4). A g¨orbe alatti ter¨ uletet az x = 3 egyenes k´et szimmetrikus r´eszre v´agja, melyek ter¨ ulete 8-8 ter¨ uletegys´eg. A (0; 0), (3; 0) ´es a (3; 4) pontok a´ltal meghat´ arozott h´ aromsz¨ og ter¨ ulete 6 ter¨ uletegys´eg. Az egyenes teh´ at egy 2 ´es egy 14 egys´eg ter¨ ulet˝ u r´eszre v´agja a megadott tartom´ anyt, ezek ar´anya ´ıgy 1 : 7. b) Az ABC h´ aromsz¨ og der´eksz¨ og˝ u, leghosszabb oldala az 5 egys´eg hossz´ u AC ´atfog´ o. Ha a h´aromsz¨ oget megforgatjuk az AC oldal k¨ or¨ ul, a keletkezett forg´ astest egy kett˝ osk´ up (k´et, k¨ oz¨ os alaplappal rendelkez˝o k´ up) lesz. A k´ upok k¨ oz¨ os alaplapj´anak sugara a h´aromsz¨ og ´ atfog´ oj´ahoz tartoz´o m magass´ aga. A h´ aromsz¨ og ter¨ ulet´et k´etf´elek´eppen fel´ırva: AC · m AB · BC = . 2 2 Innen m = 2,4. A Pitagorasz-t´etellel kapjuk, hogy ez a magass´ag egy 1,8 ´es egy 3,2 egys´eg hossz´ us´ag´ u r´eszekre osztja a h´ aromsz¨ og ´ atfog´ oj´at. A kett˝ osk´ up t´erfogata: V =

  1 4 x 2 − x = 0, 3 3

K¨ oz´ episkolai Matematikai ´ es Fizikai Lapok, 2022/2

8 4 x − x2 3 9

2,42 · π · 5 2,42 · π · 3,2 2,42 · π · 1,8 + = = 9,6π ≈ 30,2 t´erfogategys´eg. 3 3 3

A forg´astest felsz´ıne a k´et k´ uppal´ast ter¨ ulet´enek ¨ osszege. A k´ upok alkot´ oi a der´eksz¨og˝ u h´aromsz¨ og befog´oi, teh´ at 3, illetve 4 egys´eg hossz´ uak.

Megold´as. a) Megkeress¨ uk a g¨ orbe ´es az x-tengely metsz´espontjait: 4 8 x − x2 = 0, 3 9

dx

integr´ al adja meg.

Ha z = 133, akkor az 5-¨ os¨ ok ¨ osszege S5 = 5 · 133 = 665, a marad´ek 175 vagy 176 ¨ osszeget 220 − 133 = 87 darab jegyb˝ ol kell el´erni. Ez lehets´eges, pl. 86 darab 2-es ´es egy darab 3-as vagy egy 4-es seg´ıts´eg´evel. Az 5-¨ os¨ ok sz´ama teh´ at legfeljebb 133. Az 5-¨ os¨ ok minim´alis sz´ama 0. Ez megval´osulhat p´eld´ aul 180 darab 4-es ´es 40 darab 3-as eset´en. b) Teljes¨ ul a felt´etel, ha a h´ arom dobott sz´am egyforma. Ez 6 lehet˝ os´eg.



A = 2,4 · π · 3 + 2,4 · π · 4 = 16,8π ≈ 52,8 ter¨ uletegys´eg. 81

82

K¨ oz´ episkolai Matematikai ´ es Fizikai Lapok, 2022/2

9. Egy ´ep´ıt˝ oipari v´ allalkoz´ onak a legut´ obbi ´ep´ıtkez´es ut´ an megmaradt 200 kg cementje, ´es u ´gy d¨ ont¨ ott, hogy egyenl˝ o t¨ omeg˝ u r´eszekre osztva ´ert´ekes´ıti. A kereskedelemben szok´ asos m´ odon nagyobb kiszerel´es˝ u csomag eset´en alacsonyabb a cement kilogrammonk´enti a ´ra (egys´eg´ ara): ha egy csomag cement t¨ omege m m kg, akkor (40 − 10 ) peng˝os egys´eg´aron k´ın´alja elad´asra. A cement becsomago-

Mivel n ´ert´eke csak eg´esz lehet, ´ıgy meg kell vizsg´alni f (12)-t ´es f (13)-at. f (13) ≈ 7347,3 > f (12) ≈ 7346,7, teh´ at 13 egyenl˝o r´eszre osztva lesz a legmagasabb az elad´ asb´ ol sz´armaz´ o bev´etel.

m l´ as´ anak is van k¨ olts´ege, m´egpedig m kg-os csomag eset´en (25 + 10 ) peng˝o csomagonk´ent.

Koncz Levente Budapest

a) Hat´ arozzuk meg, hogy mekkora lesz a v´ allalkoz´ onak az elad´ as´ ab´ ol (a csomagol´ as k¨ olts´eg´enek levon´ asa ut´ an) sz´ armaz´ o bev´etele, ha a cementet 10 egyenl˝ o t¨ omeg˝ u r´eszre osztva ´ert´ekes´ıti. (5 pont) b) Hat´ arozzuk meg, h´ any egyenl˝ o t¨ omeg˝ u r´eszre kell osztani a cementet ahhoz, hogy – azt a tervek szerint ´ert´ekes´ıtve – az elad´ asb´ ol sz´ armaz´ o (a csomagol´ asi k¨ olts´egek levon´ asa ut´ ani) bev´etel maxim´ alis legyen. (11 pont) Megold´as. a) 10 r´eszre osztva az eladand´ o cementet, egy csomag t¨omege 20 kg. Ekkor a cement egys´eg´ ara 38 peng˝o, az ¨ osszes cement elad´ as´ab´ol sz´armaz´ o bev´etel (a csomagol´ asi k¨ olts´egek n´elk¨ ul) 200 · 38 = 7600 peng˝o. olts´ege 27 peng˝o, az ¨osszes csomagol´ asi k¨olts´eg Egy csomag csomagol´asi k¨ 10 · 27 = 270 peng˝o. A bev´etel teh´ at 7600 − 270 = 7330 peng˝o.

b) Tegy¨ uk fel, hogy a v´allalkoz´ o n egyenl˝ o t¨omeg˝ u r´eszre osztva ´ert´ekes´ıti 200 a cementet (n ∈ Z). Ekkor egy csomag t¨ omege n kg. 20 A cement egys´eg´ ara (40 − n ) peng˝ o, az ¨ osszes cement elad´ as´ab´ol sz´armaz´ o bev´etel   20 4000 peng˝ o. 200 · 40 − = 8000 − n n

Egy csomag csomagol´asi k¨ olts´ege (25 + 20 peng˝ o, az ¨osszes csomag csomagol´asi n)

= 25n + 20 peng˝ o. Az elad´asb´ ol sz´armaz´ o haszon ´ıgy k¨ olts´ege n(25 + 20 n)   4000 4000 . − (25n + 20) = 7980 − 25n − 8000 − n n Tekints¨ uk a pozit´ıv val´os sz´amok halmaz´an ´ertelmezett f : x → 7980 − 25x −

4000 x

f¨ uggv´enyt, ´es keress¨ uk ennek maximumhely´et. Az f deriv´altf¨ uggv´enye: f  (x) = −25 +

4000 . x2

Az f -nek ott lehet maximumhelye, ahol f  (x) = 0, azaz −25 + 4000 = 0. Ebb˝ ol x2 √ (x > 0 miatt) x = 160 ≈ 12,65. Mivel itt uggv´eny el˝ ojelet v´ alt (pozi√ a deriv´altf¨ t´ıvb´ o√l negat´ıvba), az f (x) f¨ uggv´eny x < 160 eset´en szigor´ uan monoton n¨ovekv˝ o, uan monoton cs¨ okken˝ o. x > 160 eset´en pedig szigor´ K¨ oz´ episkolai Matematikai ´ es Fizikai Lapok, 2022/2

83

Matematika feladatok megold´asa

B. 5150. Igazoljuk, hogy csak v´eges sok olyan pozit´ıv eg´esz sz´ am van, amelyet nem lehet megkapni u ´gy, hogy egy kisebb sz´ amhoz hozz´ aadjuk annak valamelyik sz´ amjegy´et. Melyik a legnagyobb ezek k¨ oz¨ ul? (4 pont) Megold´as. Minden h´ arom-, vagy ann´ al t¨ obbjegy˝ u sz´ amot meg lehet kapni egyik sz´ amjegy´enek ´es egy kisebb sz´ amnak az ¨ osszegek´ent a k¨ ovetkez˝ o m´odon: a sz´amb´ol kivonjuk a legels˝o sz´ amjegy´et (ez biztosan nagyobb, mint 0), az ´ıgy kapott k¨ ul¨onbs´eg lesz a megfelel˝ o sz´ am, hiszen ha ehhez hozz´ aadjuk az els˝ o sz´ amjegy´et, akkor szinte minden esetben visszakapjuk az eredeti sz´amot. A fenti m´odszer akkor nem m˝ uk¨ odik, ha a kivon´ as sor´ an v´ altozik az els˝o sz´ amjegy. Mivel legal´ abb h´ aromjegy˝ u sz´ amokat vizsg´alunk, ez csak akkor fordulhat el˝o, ha a t´ızes helyi´ert´eken l´ev˝ o sz´ amjegy 0, ´es az els˝o sz´ amjegy nagyobb, mint az utols´o. Ebben az esetben m´ as m´odszerrel a´ll´ıtjuk el˝ o a sz´ amot. Ha kivonunk o 0-b´ ol 9-es lesz, mert az utols´o a sz´amb´ol 9-et, akkor a t´ızes helyi´ert´eken l´ev˝ sz´ amjegyn´el nagyobbat vontunk ki. Az ´ıgy kapott sz´ amhoz hozz´ aadva az utols´o el˝otti sz´ amjegy´et, amely 9, megkapjuk az eredetit sz´amot. ´ u, ahol x2 = 0, Altal´ anoss´ agban, ha egy pozit´ıv eg´esz sz´ am xn . . . x2 x1 alak´ aadva megkapjuk akkor a megfelel˝ o sz´ am: xn . . . x2 x1 − xn , amelyhez xn -t hozz´ az eredeti sz´amot. u, ahol 9  xn > x1 ,akkor a megfelel˝ o Ha egy pozit´ıv eg´esz sz´ am xn . . . 0x1 alak´ otti sz´ amjegye biztosan 9, ´ıgy hozz´ aadva sz´ am: xn . . . 0x1 − 9, amelynek utols´o el˝ 9-et, megkapjuk az eredeti pozit´ıv eg´esz sz´ amot. Ezzel bel´ attuk, hogy minden legal´ abb h´aromjegy˝ u sz´ am el˝o´ all´ıthat´ o a feladatban megadott m´odon. Most megvizsg´ aljuk a 100-n´ al kisebb sz´amokat. K¨ oz¨ ul¨ uk azokat a p´ aros sz´amokat, amelyek nem 0-ra v´egz˝ odnek, megkaphatjuk u ´gy, hogy az utols´o sz´ amjegy´enek a fel´et kivonjuk bel˝ ole, mert akkor a kapott sz´ amhoz az utols´ o sz´ amjegy´et hozz´ aadva megkapjuk az eredeti sz´amot. Ha pedig 0-ra v´egz˝ odik a sz´ am, akkor 5-¨ ot vonunk ki, ´ıgy a k¨ ul¨onbs´eg utols´ o sz´ amjegye 5 lesz. 84

K¨ oz´ episkolai Matematikai ´ es Fizikai Lapok, 2022/2

Most a k´etjegy˝ u p´ aratlan sz´ amok k¨ ovetkeznek:

A 3 pr´ım ´es nem oszthat´ o 2-vel, teh´at nem azonos dk = n-nel. A fentiek alapj´an vil´agos, hogy amennyiben n ¨ osszetett sz´ am, akkor oszthat´ o 2-vel ´es 3-mal is, teh´ at oszthat´ o 6-tal.

99 = 90 + 9, 97 = 89 + 8, 95 = 87 + 8,

Azt a´ll´ıtjuk, hogy az n = 6 megold´ asa a feladatnak, hiszen ekkor n = d4 = 6. oja d1 = 1, ami oszt´oja a 6-nak, d2 = 2, d1 + d2 = d3 = 3, ami ugyancsak oszt´ oja a 6-nak. A 6-nak a 6-nak; v´eg¨ ul d1 + d2 + d3 = d1 + d2 + dk−1 = 6, ami oszt´ ul kiz´ ar´ olag az 1, a 2 ´es a 3 a pozit´ıv oszt´ oja, teh´at m´as ¨ osszeg, illetve ¨onmag´an k´ıv¨ oszt´ o nem a´ll el˝ o, azaz a 6-ra a feladat minden felt´etele teljes¨ ul.

93 = 85 + 8, 91 = 83 + 8, 89 = 81 + 8. Azt a´ll´ıtom, hogy 87-et nem lehet ´ıgy el˝ o´all´ıtani, hiszen 8x alak´ u nem lehet j´ o, mert ha 8-at adunk hozz´ a, akkor az ¨ osszeg legal´ abb 88 lesz. Ha x-et adunk hozz´ a, akkor pedig p´ aros sz´amot kapunk. Pr´ ob´ aljuk meg a 79-b˝ ol el˝o´ all´ıtani a 87-et:

Ezek ut´ an megvizsg´aljuk, hogy van-e m´as megold´asa a feladatnak. Mivel a 6 oja n-nek. Figyelembe v´eve, hogy ennek oszt´oja n-nek, ez´ert n2 , n3 ´es n6 is oszt´ a h´arom oszt´onak az o sszege ¨ n n n + + = n, 2 3 6 tov´ abbi val´ odi oszt´oja m´ar nem is lehet n-nek, hiszen a feladat felt´etele szerint oja n-nek, ahol dk = n; ez´ert az egyetlen megold´ as d1 + d2 + . . . + dk−1 is oszt´ az n = 6. ´ Isk. ´es Gimn., 12. ´evf.) Baski Bence (Budapesti Fazekas M. Gyak. Alt.

79 + 7 = 87, 79 + 9 = 87.

¨ Osszesen 132 dolgozat ´erkezett. 4 pontos 73, 3 pontos 40, 2 pontos 12 dolgozat. 1 pontot 6, 0 pontot 1 versenyz˝ o kapott.

Sz´ oba j¨ ohet m´eg a 78 is: 78 + 8 = 87, 78 + 7 = 87. 78-n´ al kisebb sz´amb´ ol pedig biztosan nem lehet a 87-et egy sz´amjegy hozz´aad´ as´aval el˝o´ all´ıtani, hiszen k¨ ul¨ onbs´eg¨ uk nagyobb 9-n´el. Bel´ attuk, hogy a 87-et nem lehet ezzel a m´ odszerrel megkapni, de az ¨osszes n´ala nagyobbat igen, ´ıgy 87 a legnagyobb ilyen sz´ am. Ebb˝ ol az is k¨ovetkezik, hogy v´eges sok ilyen pozit´ıv eg´esz sz´ am van. Csizmadia Mikl´ os (Budapest XIV. Ker. Szent Istv´ an Gimn., 11. ´evf.) dolgozata alapj´an ¨ Osszesen 110 dolgozat ´erkezett. 4 pontos 89, 3 pontos 5, 2 pontos 7 dolgozat. 1 pontot 1, 0 pontot 7 versenyz˝ o kapott. Nem versenyszer˝ u: 1 dolgozat.

B. 5201. Legyenek az n pozit´ıv eg´esz sz´ am pozit´ıv oszt´ oi 1 = d1 < d2 < . . . < arozzuk meg azokat az ¨ osszetett n sz´ amokat, amelyekre d1 , d1 + d2 , < dk = n. Hat´ amok mind oszt´ oi n-nek. d1 + d2 + d3 , . . . , d1 + d2 + . . . + dk−1 sz´

Ifj´ u olvas´ oinkhoz r´egen ´es most

1894 (de ak´ar ma is ´ırhattuk volna) Az ezideig hozz´ank bek¨ uld¨ ott ´es megoldott feladatok szerkezete ´es k¨ ulalakja k¨or¨ ul szerzett tapasztalataink alapj´ an fordulunk jelen sorainkkal ifj´ u olvas´ oinkhoz ´es a k¨ovetkez˝ o k´erelmet int´ezz¨ uk hozz´ ajuk: Sz´ıveskedjenek bek¨ uld¨ ott dolgozataikat lehet˝ oleg gondosan szerkeszteni, nem haszn´ alv´ an semmif´ele r¨ ovid´ıt´est; t´ argyaljanak minden feladatot k¨ ul¨ on lapon ´es annak csak egyik oldal´ara ´ırva, l´ ass´ ak el minden megold´asukat n´eval´ a´ır´ asukkal.

oja. Megold´as. Az n o am, amelynek d2 a legkisebb pr´ımoszt´ ¨sszetett sz´ Tegy¨ uk fel indirekt m´ odon, hogy d2 = 2. Ekkor d1 + d2 = d2 + 1, ami d2 p´aratlan volta miatt p´ aros. Ezzel ellentmond´ asra jutottunk, mivel ekkor 2 oszt´ oja n-nek, ovetkezik, hiszen a legkisebb pr´ımoszt´ o ebben az esetben a 2 lenne. amib˝ ol d2 = 2 k¨

Sokkal ´erdekesebbnek ´es tanuls´ agosabbnak tartjuk, ha olvas´ oink dolgozatait v´altozatlanul ´es kijav´ıtatlanul k¨ oz¨ olhetj¨ uk, mintha t¨ ok´eletesen a´tidom´ıtott m´asolatot kell ny´ ujtanunk, melyekben szerz˝ o nem tal´ al meg semmit tulajdon´ ab´ol, hacsak nem a megold´ as al´a biggyesztett nev´et. De hogy ezt el´erhess¨ uk, okvetetlen¨ ul sz¨ uks´egesnek tartjuk, hogy olvas´ oink j´ oindulata seg´ıts´eg¨ unkre legyen ´es ´ ohajtva rem´elj¨ uk, hogy ezent´ ul nem magyar´az´ o sz¨oveg n´elk¨ uli k´epletsorozatokat, hanem gondosan elk´esz´ıtett ´es szerkesztett megold´ asokat fogunk kapni.

Ha teh´at van o as, akkor d2 = 2, amib˝ ol d1 + d2 = 1 + 2 = 3 ¨sszetett n megold´ k¨ ovetkezik. Mivel a 3 k¨ ozvetlen¨ ul a 2 ut´ an k¨ ovetkez˝ o eg´esz, ´ıgy d3 = 3 ad´odik.

Minden oldalr´ ol halljuk a tan´ arok panasz´ at az iskolai ´es h´ azi dolgozatok k¨ ulalakj´ anak ´es szerkezet´enek pongyola ´es elhanyagolt volta felett. Mid˝ on teh´at ifj´ u

(4 pont)

K¨ oz´ episkolai Matematikai ´ es Fizikai Lapok, 2022/2

Javasolta: S´ andor Csaba (Budapest)

85

86

K¨ oz´ episkolai Matematikai ´ es Fizikai Lapok, 2022/2

olvas´ oinkat k´erj¨ uk, hogy gondozz´ ak t˝ ol¨ uk telhet˝ oleg bek¨ uld¨ott megold´ asaikat, csak arra h´ıvjuk fel o˝ket, hogy halad´ asuknak es´elyeit az iskol´aban ´es az ´eletben nagyobb m´ert´ekben n¨ ovelj´ek, semmint azt ˝ok maguk hinni hajland´ ok voln´anak. Gy˝ or¨ ott, 1894. ´evi deczember hav´ aban. ´ ARANY DANIEL mint a K. M. L.” szerkeszt˝ oje. ” 2021–2022, a fentiekhez n´eh´any hasznos megjegyz´est hozz´at´eve (´erdemes a honlapon tal´alhat´ o megold´asokat is elolvasni) K. 696. Ellen˝ orz´es ´es v´ alasz hi´ anya vagy hi´anyos v´ alasz (csak az egyik zsebben l´ev˝ o ¨ osszeget adja meg). N´eh´ anyan sz¨ uks´egtelen¨ ul sok v´ altoz´ oval dolgoztak, bonyolultabb egyenletrendszerekkel nehez´ıtve a saj´at dolgukat. K/C. 698. Nagyon sok indokl´as n´elk¨ uli vagy minim´alis indokl´ assal ell´atott megold´ ast k¨ uldtek be (ez szerintem a sok 9-es miatt van, akik nem olyan rutinos megold´ ok). o, hogy valamelyik Kifejezett t´ıpushiba nem volt, p´ ar megold´asra volt jellemz˝ k´erd´est / krit´eriumot nem vett´ek figyelembe, ez´ert sz¨ uletett rossz eredm´eny¨ uk. K. 704. Jellemz˝o hib´ ak: – megkapja a v´egs˝ o pontsz´ amsorozatot, de nem ellen˝ orzi, hogy ilyen pontsz´ amok val´oban kialakulhatnak-e; – keveset, esetleg csak eredm´enyt´ abl´ azattal indokol; – szerte´ agaz´o, nehezen ´ertelmezhet˝ o, jav´ıt´ asokkal ´es sat´ıroz´ asokkal teli gr´ afokkal szeml´elteti a megold´ as´at (ha tudtam ´ertelmezni, ez´ert nem vontam le pontot). C. 1682. A feladat alapvet˝ oen nem volt neh´ez, de sok versenyz˝o extra nehez´ıt´esk´ent u ´gy pr´ ob´ alta az ABDE tetra´eder t´erfogat´ at sz´amolni, hogy a szab´alyos h´ aromsz¨ oget vett´ek alapter¨ uletnek. Ezt a m´odszert sokan elrontott´ak sz´amol´asi hiba miatt, vagy pedig a tetra´eder magass´ ag´ anak sz´amol´as´an´al elvi hib´at v´etettek. C. 1683. Viszonylag kev´es versenyz˝o – 86-b´ ol kb. 13 – ismerte fel, hogy az esem´enyek f¨ uggetlens´ege sz¨ uks´eges ahhoz, hogy a k´et r´eszeredm´enyt o¨sszeszorozhassuk, a legt¨ obb 4 pontos dolgozatn´ al csak ez hi´anyzott. Viszonylag sok olyan dolgozat volt – a 3 pontosok t´ ulnyom´ o r´esze –, ahol l´atszott, hogy a feladat megold´asa nem okozott probl´em´at, azonban m´egse v´egezt´ek el a v´egs˝ o szorz´ ast, azaz nem v´alaszolt´ ak meg a k´erd´est a versenyz˝ ok. C. 1690. Jellemz˝o hib´ ak: – nem bizony´ıtja, hogy a C pont val´ oban a DB szakaszra esik; – gyenge vagy kev´es indokl´ as; – hasonl´ os´agra hivatkozik, de nem indokol. B. 5185. Jellemz˝o hib´ ak: – ´ertelmez´esi tartom´any ´es / vagy ellen˝orz´es hi´ anya; – ismeretlennel val´o oszt´ as, ´ıgy k´et megold´ as elveszt´ese; – rossz ´ertelmez´esi tartom´any (a k¨ obgy¨ ok alatti kifejez´est nemnegat´ıvk´ent ´ertelmezt´ek); – j´o megold´ asok, de a rossz ellen˝ orz´es miatt elvesztettek egy megold´ asp´ art; K¨ oz´ episkolai Matematikai ´ es Fizikai Lapok, 2022/2

87

– grafikus a´br´ azol´ as (val´ osz´ın˝ uleg Geogebr´aval) ´es a megold´ asok leolvas´ asa, t¨obbsz¨or rosszul ´es indokl´ as n´elk¨ ul. B. 5188. A versenyz˝ ok jellemz˝oen az al´ abbi hib´ akat k¨ ovett´ek el: – az alapok sz´ amtani k¨ ozep´et vett´ek, nem a m´ertanit; – nem vett´ek figyelembe, hogy a cs´ ucsb´ ol szerkesztett magass´ag talppontja eshet az alapon k´ıv¨ ul is; – akik seg´edszerkeszt´essel vagy szab´alyosabb alakzatra val´ o visszavezet´essel pr´ ob´alkoztak, jellemz˝ oen kihagytak eseteket (pl. sz´arak metsz´espontja nem l´etezik rombusz eset´en). ´ B. 5189. Altal´ anos hib´ak: – a´bra hi´ anya; – speci´alis eset megold´ asa; – t´etelekre val´ o hivatkoz´ as hi´anya / hib´as kimond´ asa; – nem hivatkoztak a g´ ula szab´alyos alapj´ara, egyenes mivolt´ ara, de a bel˝ol¨ uk k¨ovetkez˝ o tulajdons´agokat, speci´ alis helyzeteket felhaszn´ alt´ ak. B. 5191. Jav´ıt´ asi tapasztalatok: A versenyz˝ ok nagy r´esze nem vette figyelembe a feladat azon r´esz´et, hogy nem k´et tetsz˝oleges, hanem k´et el´eg k¨ ozel lev˝ o pontot tudunk csak ¨ osszek¨ otni – teh´ at el´eg kis m´eret˝ u k¨ orre t¨ ok´eletes megold´ ast adtak, azonban nem kezelt´ek azt az esetet, ha a k¨ or t´ ul nagy. A m´ asik ´erdekess´eg az volt, amikor u ´gy ´ertelmezt´ek, hogy adott k¨ uls˝ o pontb´ ol is tudnak mer˝olegest bocs´ajtani adott szakaszra, mely a megengedett l´ep´esek k¨ ozt nem volt ott. Egy ´erdekess´eg az, hogy viszonylag kev´es versenyz˝o (max. 10 db) gondolta u ´gy, hogy a feladat nem megoldhat´o, ekkor ´altal´ aban a kreativit´as hi´anya miatt nem vett´ek ´eszre, hogy hogyan lehet a k¨ orz˝ o hi´ anya ´es a vonalz´ o v´egess´ege adta limit´ aci´ ok keret´eben mozogva felez˝ omer˝ olegest szerkeszteni. V´egs˝ o – ´es kiss´e szomor´ u – tapasztalatom a g´eppel szerkesztett a´br´ ak nehezen ´atl´ athat´ os´aga, vagy nem konzekvens jel¨ ol´esm´od, fogalmaz´ as, ami miatt a gondoak hi´anya, mely latmenet nehezebben ´ertelmezhet˝ o. Enn´el szomor´ ubb volt az ´abr´ sokszor megnehez´ıtette a szerkeszt´esm´od ´ertelmez´es´et. B. 5193. Jellemz˝o hib´ ak: – el´ır´asok (pl. E ´es F o an hasonl´ıtanak, p´ arhuzamos ¨sszecser´el´ese, mert az a´br´ ´es mer˝ oleges felcser´el´ese stb.); – l´ep´esek kifelejt´ese pl. hasonl´os´ ag meg´allap´ıt´ as´ an´ al, hosszok defini´ al´as´ an´al; – AF ´es F B felcser´el´ese (´es ´ıgy az ´all´ıt´ as hamiss´ag´ anak bel´at´ asa); – a´bra hi´ anya. B. 5196. A H´ any elem˝ u lehet az A halmaz?” k´erd´esre adott v´ alaszban t¨ obben ” ” csak a maximumot hat´arozt´ ak meg”. B. 5203. Jellemz˝o hib´ ak: aran – csak az egyik ir´ any bizony´ıt´ asa, nem az akkor ´es csak akkor” a´ll´ıt´ as´e (p´ ” megeml´ıtett´ek a m´ asik ir´anyt is a megold´ as elej´en, azt´ an v´eg¨ ul kihagyt´ ak); – nagy ugr´ asok a bizony´ıt´ asban, a´ll´ıt´ asokat t¨ obbsz¨ or nem bizony´ıtottak, vagy ob´ alt´ ak bizony´ıtani, ¨osszekevertek dolgokat (pl.: 3 pont egy egyenesen fekv´es´et pr´ de a sz¨ogeket u ´gy sz´ amolt´ ak, mintha a 3 pont egy egyenesen lenne).

88

K¨ oz´ episkolai Matematikai ´ es Fizikai Lapok, 2022/2

B. 5213. T¨ obbsz¨ or is el˝ofordult, hogy a versenyz˝o gy¨ony¨or˝ uen bel´ atta az egyenl˝ otlens´eget – ak´ar saj´at m´odon, ak´ ar a Ptolemaiosz t´etel haszn´ alat´ aval, ak´ ar izomb´ol kialgebr´azva –, de elfelejtett az egyenl˝os´egre felt´etelt adni.

A C pontversenyben kit˝ uz¨ott gyakorlatok (722–723., 1704–1708.)

A. 809. Jellemz˝o hib´ ak: – nem egy´ertelm˝ u jel¨ ol´esek; – a sz´ amol´as le nem ´ır´ asa vagy egy −1-es szorz´ o elhagy´asa; – hib´ as becsl´esek (egyenl˝ otlens´eg ir´any´ at eln´ezve); – a´bra hi´ anya.

Feladatok 10. ´evfolyamig

Jav´ıt´ok 2021. o˝sz – 2022. t´el

K/C. 722. A sz¨ oveg´et l´ asd a K feladatokn´ al. K/C. 723. A sz¨ oveg´et l´ asd a K feladatokn´ al.

A K pontversenyben kit˝ u z¨ ott gyakorlatok ABACUS-szal k¨ oz¨ os pontverseny 9. oszt´alyosoknak (719–723.)

Feladatok mindenkinek C. 1704. Mely a val´ os sz´amok eset´en lesz a [0; 2] intervallumon ´ertelmezett f (x) = 4x2 − 4ax + a2 − 2a + 2 f¨ uggv´eny minimumhely´en a f¨ uggv´eny ´ert´eke 3?

K. 719. Kisz´ınezz¨ uk a sz´ amegyenesen az eg´esz sz´ amokat jelz˝o pontok mindegyik´et k´ek vagy piros sz´ınnel. Igaz-e b´ armilyen, a felt´etelnek megfelel˝o sz´ınez´es eset´en, hogy a) biztosan lesz k´et azonos sz´ın˝ u pont, melyek t´ avols´ aga 3; b) biztosan lesz k´et azonos sz´ın˝ u pont, melyek t´ avols´ aga 3 vagy 4?

(MC&IC ) C. 1705. Egy deltoidr´ ol tudjuk, hogy h´ urn´egysz¨ og, oldalainak hossza 42 ´es 56 hossz´ us´agegys´eg. Milyen messze van egym´ast´ ol a be´ırt ´es a k¨ or´e´ırt k¨ or´enek k¨oz´eppontja? Javasolta: Siposs Andr´ as (Budapest)

K. 720. V´ agjunk fel h´arom egyenl˝o ter¨ ulet˝ u r´eszre egy szab´alyos hatsz¨oget az egyik cs´ ucs´ an ´ atmen˝ o k´et egyenessel. K. 721. Sanyi eg´esz cm hossz´ us´ag´ u p´ alcik´ akat k´esz´ıtett, m´eghozz´ a olyanokat, hogy k¨ oz¨ ul¨ uk semelyik h´ aromb´ ol nem lehet h´aromsz¨oget ¨ossze´ all´ıtani. Tudjuk, hogy Sanyi 1 ´es 10 hossz´ us´ag´ u p´ alcik´ at is k´esz´ıtett, a leghosszabb p´ alcika pedig 100 cm hossz´ u. Maxim´alisan h´any p´ alcik´ at k´esz´ıthetett Sanyi? K/C. 722. K´et h´aromjegy˝ u sz´am a´tlaga pont annyi, mintha a k´et sz´ am k¨oz´e tizedesvessz˝ ot t´eve egym´ as mell´e ´ırjuk azokat. Mi lehet a k´et sz´ am?

C. 1706. Bizony´ıtsuk be, hogy 2022 darab pozit´ıv eg´esz sz´ am k¨ oz¨ ott biztosan van 2 olyan, amelyek k¨ ul¨ onbs´ege vagy ¨ osszege oszthat´ o 4040-nel. Javasolta: S´ af´ ar Lajos (R´ackeve) Feladatok 11. ´evfolyamt´ol C. 1707. Az ABC h´ aromsz¨ ogben a szok´ asos jel¨ ol´esekkel b = 6, a = 2 ´es arozzuk meg a γ sz¨ og CD bels˝ o sz¨ ogfelez˝ oj´enek pontos hossz´at. γ = 120◦ . Hat´ (MC&IC )

K/C. 723. A toki´oi olimpi´ara a Magyar K´ezilabda Sz¨ovets´eg 17 n˝oi k´ezilabd´az´ ot nevezett: 3 kapust, 1 jobbsz´els˝ ot, 4 jobb´ atl¨ ov˝ ot, 2 ir´ any´ıt´ ot, 3 be´ all´ot, 2 balatl¨ ´ ov˝ot ´es 2 balsz´els˝ ot. H´ anyf´elek´eppen a´llhatnak fel a himnuszhoz, ha az ugyanolyan posztokon szerepl˝o j´at´ekosok mindenk´eppen egym´as mellett ´allnak? (A himnusz alatt a j´ at´ekosok egym´as mellett, egy sorban ´allnak.)

C. 1708. Oldjuk meg a k¨ ovetkez˝ o egyenletet a val´os sz´amp´ arok halmaz´an: log22 (x + y) + log22 (xy) + 1 = 2 log2 (x + y). (MC&IC )

Javasolta: R´ oka B´ alint (Budapest)





Beku ¨ld´esi hat´arid˝o: 2022. m´arcius 10. Elektronikus munkafu ¨zet: https://www.komal.hu/munkafuzet

Beku ¨ld´esi hat´arid˝o: 2022. m´arcius 10. Elektronikus munkafu ¨zet: https://www.komal.hu/munkafuzet





K¨ oz´ episkolai Matematikai ´ es Fizikai Lapok, 2022/2

89

90

K¨ oz´ episkolai Matematikai ´ es Fizikai Lapok, 2022/2

B. 5228. Egy parabola az ABC h´ aromsz¨ og AB oldal´ at a C1 ´es C2 , BC oldal´ at at a B1 ´es B2 bels˝ o pontokban metszi. Igazoljuk, hogy az A1 ´es A2 , m´ıg CA oldal´ ha AC1 = C2 B ´es BA1 = A2 C, akkor CB1 = B2 A.

A B pontversenyben kit˝ u z¨ ott feladatok (5222–5229.)

(5 pont)

B. 5222. Legyenek az A halmaz elemei azok a p´ aros pozit´ıv eg´eszek, amelyeket 2-vel osztva a sz´ amjegyek ¨ osszege 2-vel cs¨ okken, a B halmaz elemei pedig azok a pozit´ıv eg´eszek, melyeket 5-tel szorozva a sz´ amjegyek ¨osszege 5-tel n˝ o. Adjuk meg az A ∩ B ´es a B \ A halmazok elemsz´ am´at. (3 pont)

Javasolta: K´ asp´ ari Tam´ as (Paks)

B. 5229. Az a = 0 val´ os sz´amra ´es az f : R → R f¨ uggv´enyre   f x + f (y) = f (x) + f (y) + ay

teljes¨ ul minden x, y ∈ R eset´en. Bizony´ıtsuk be, hogy f addit´ıv, vagyis f (x + y) = = f (x) + f (y) minden x, y ∈ R eset´en. (6 pont)

Javasolta: George Stoica (Saint John, New Brunswick, Kanada)



ovetkez˝ ok´eppen: B. 5223. Defini´ aljuk az {an } sorozatot a k¨ a1 = −3,

Javasolta: Holl´ o G´ abor (Budapest)

Beku ¨ld´esi hat´arid˝o: 2022. m´arcius 10. Elektronikus munkafu ¨zet: https://www.komal.hu/munkafuzet

√ an+1 = 4 + an + 4 an + 4 .



Hat´ arozzuk meg a2022 ´ert´ek´et. (3 pont)

Javasolta: K´ asp´ ari Tam´ as (Paks)

B. 5224. Az ABCD egys´egn´egyzet BC oldal´ an u ´gy vessz¨ uk fel a P pontot, tov´ abb´ a a CD oldal´ an a Q pontot, hogy P AQ = 45◦ . A P ´es Q pontok melyik helyzet´eben lesz BP + P Q + QD minim´alis? (4 pont)

Az A pontversenyben kit˝ uz¨ott nehezebb feladatok (818–820.)

Javasolta: Szoldatics J´ ozsef (Budapest)

B. 5225. Az ABC h´ aromsz¨ og A-val szemk¨ ozti oldala a, be´ırt k¨or´enek k¨oz´eppontja I, sugara , a k¨or¨ ul´ırt k¨ or sugara R. Bizony´ıtsuk be, hogy ha AI = R, akkor az ABC h´ aromsz¨ og ter¨ ulete a·R +  · a. 4

A. 818. Hat´ arozzuk meg mindazokat az m, n pozit´ıv eg´esz sz´ amokb´ ol ´ all´ o o m-mel ´es n-nel is. p´arokat, amelyekre 9|m−n| + 3|m−n| + 1 oszthat´

Javasolta: Kocsis Szilveszter (Budapest)

Javasolta: K´ os G´eza (Budapest)

B. 5226. Egy h´ aromsz¨ og mindh´arom oldal´ anak hossza legfeljebb 2 egys´eg. Minden cs´ ucsp´ art ¨ osszek¨ ot¨ unk egy-egy olyan k¨ or´ıvvel, amely egy-egy egys´egsugar´ u k¨ ornek a f´elk¨ orn´el nem hosszabb ´ıve. Igazoljuk, hogy

A. 819. Legyen G egy tetsz˝ olegesen v´ alasztott v´eges egyszer˝ u gr´ af. A gr´ af cs´ ucsaira olyan m´odon ´ırunk nemnegat´ıv eg´esz sz´ amokat, hogy minden cs´ ucson az a sz´ am szerepeljen, ah´ any olyan szomsz´edja van az adott cs´ ucsnak, melyre p´aros sz´ amot ´ırtunk. Bizony´ıtsuk be, hogy az ilyen kit¨ olt´esek sz´ ama kett˝ ohatv´ any.

(4 pont)

a + b > 2c /3, 





or´ıvek hossz´at jel¨ oli. ahol a , b , c a k¨ (5 pont) B. 5227. Adjunk p´eld´ at olyan k pozit´ıv eg´eszre ´es legal´ abb k cs´ ucs´ u F v´eges fagr´ afra, amelyben minden cs´ ucs legfeljebb harmadfok´ u, ´es F -nek tetsz˝oleges k cs´ ucs´ u¨ osszef¨ ugg˝o r´eszgr´ afj´ at elhagyva a megmarad´o gr´af legal´abb 2022 komponensre esik sz´et. (6 pont) K¨ oz´ episkolai Matematikai ´ es Fizikai Lapok, 2022/2

(Monthly feladat nyom´an) 91

A. 820. Legyen ABC egy tetsz˝ oleges h´ aromsz¨ og. A h´ aromsz¨ og a oldal´ ahoz hozz´ a´ırt k¨or az AB, BC ´es CA egyeneseket rendre a Ca , Aa ´es Ba pontokban ´erinti. Hasonl´oan, a h´ aromsz¨ og b oldal´ ahoz hozz´a´ırt k¨ or az AB, BC ´es CA egyeneseket ul a h´ aromsz¨ og c oldal´ ahoz hozz´a´ırt k¨ or rendre a Cb , Ab ´es Bb pontokban ´erinti. V´eg¨ az AB, BC ´es CA egyeneseket rendre a Cc , Ac ´es Bc pontokban ´erinti. Legyen A az Ab Cb ´es Ac Bc egyenesek metsz´espontja. Hasonl´oan, legyen B  a Ba Ca ´es Ac Bc egyenesek, C  pedig az Ab Cb ´es Ba Ca egyenesek metsz´espontja. V´eg¨ ul legyen Ta , or ´erint´esi pontja rendre az a, b ´es c oldalon. Tb ´es Tc a be´ırt k¨ a) Bizony´ıtsuk be, hogy az A Aa , B  Bb ´es C  Cc egyenesek egy ponton mennek a´t. 92

K¨ oz´ episkolai Matematikai ´ es Fizikai Lapok, 2022/2

´ O ´ VAGY m˝ • A KIZAR uvelet pontosan akkor IGAZ, ha a k´et r´esz´ all´ıt´ asunk ´ ´ EK ´ U ˝ (olvasata: A ⊗ B = A kiz´ ELLENTETES LOGIKAI ERT ar´ o vagy B). ´ m˝ • Az AZONOSSAG uvelet pontosan akkor IGAZ, ha a k´et r´esz´ all´ıt´ as AZONOS ´ ´ U ˝ (olvasata: A ⇔ B = A azonos B-vel). LOGIKAI ERTEK ´ m˝ ¨ • A KOVETKEZTET ES uvelet pontosan akkor HAMIS, ha az az a´ll´ıt´ as, amir˝ol k¨ovetkeztet¨ unk IGAZ, de az, amire k¨ ovetkeztet¨ unk HAMIS (olvasata: A → B = A-b´ ol k¨ ovetkezik B).

b) Bizony´ıtsuk be, hogy az A Ta , B  Tb ´es C  Tc egyenesek is egy ponton mennek a´t, ´es ez a pont rajta van az ABC h´ aromsz¨ og magass´ agpontja ´es be´ırt k¨or´enek k¨ oz´eppontja ´altal alkotott egyenesen. Javasolta: Csapl´ ar Viktor (B´atorkeszi) ´es Heged˝ us D´ aniel (Gy¨ongy¨os) Beku ¨ld´esi hat´arid˝o: 2022. m´arcius 10. Elektronikus munkafu ¨zet: https://www.komal.hu/munkafuzet



at. Hagyom´ aEzek ut´ an n´ezz¨ uk is meg a k´et´ all´ıt´ asos m˝ uveletek ´ert´ekt´abl´ azat´ nyosan az al´ abbi m´ odon adj´ak meg ezeket: Ez a le´ır´ as prec´ız, de a fent mutatottnak t¨ obb az inform´aci´ otartalma: A∨B A igaz, B igaz igaz A igaz, B hamis igaz A hamis, B igaz igaz A hamis, B hamis hamis

A matematikai logika logikusabb, mint gondoln´ank I. M´ ar j´o egy ´evtizedes informatikatan´ ari gyakorlat alatt foglalkoztatott, hogy a vagy, az ´es, a kiz´ ar´ o vagy, a k¨ ovetkeztet´es ´es az azonoss´ ag m˝ uveletek nem fedik le az o sszes kimeneti lehet˝ o s´ e get a k´ e t a ´ ll´ ıt´ a s logikai kapcsolata ter´en. ¨ A k´et r´esz´ all´ıt´ as (nevezz¨ uk A-nak ´es B-nek) egy ´all´ıt´ ass´ a¨ osszevon´ asa n´egy alapesetet jelent, hisz egym´ ast´ ol f¨ uggetlen¨ ul mindkett˝ o lehet igaz ´es hamis. Mivel a n´egy eredm´enymez˝ oben a k´et-k´et kimenet b´armelyike el˝ oforol az el˝obb dulhat, teh´at 24 = 16 kimenet lehets´eges, ebb˝ eml´ıtettek csak ¨ ot esetet fednek le. Mi van a m´asik tizenegy lehet˝ os´eggel? Ebben a cikkben ennek j´ arunk a v´eg´ere. El˝osz¨ or – a k´es˝ obbi f´elre´ert´esek elker¨ ul´ese miatt – tiszt´ azzuk a jel¨ol´eseket. Az ´ all´ıt´ asokat latin nagybet˝ ukkel jel¨ olj¨ uk (A, B, . . . , Z). Van k´et kit¨ untetett bet˝ u: I az azonosan igaz ´all´ıt´ ast jel¨ oli, ami a k¨ or¨ ulm´enyekt˝ ol f¨ uggetlen¨ ul mindig igaz logikai ´ert´ek˝ u, tov´ abb´ a H az azonosan hamis ´all´ıt´ast, ami az I ´all´ıt´ as tagad´ asa, ellent´ete. A k´etf´ele logikai ´ert´ek jel¨ ol´es´ere az i (igaz) ´es a h (hamis) bet˝ uket haszn´ aljuk. A logikai m˝ uveletek jel¨ ol´es´ere a matematik´ aban haszn´alatos jel¨ol´eseket fogjuk uvelet´e ∧, a vagy m˝ uvelet´e ∨, a kiz´ ar´ o vagy haszn´ alni, a tagad´ as jele ¬, az ´es m˝ m˝ uvelet´e ⊗, az azonoss´ ag´e ⇔, v´eg¨ ul a k¨ ovetkeztet´es´e →. A teljess´eg kedv´e´ert vegy¨ uk ´ at a fent eml´ıtett m˝ uveletek szab´ aly´ at. ´ • A TAGADAS m˝ uvelet ellent´et´ere v´ altoztatja az eredeti ´all´ıt´ as logikai ´ert´ek´et (olvasata: ¬A = nem A). ´ m˝ • Az ES uvelet pontosan akkor IGAZ, ha mindk´et r´esz´ all´ıt´asunk IGAZ (olvasata: A ∧ B = A ´es B). • A VAGY m˝ uvelet pontosan akkor HAMIS, ha mindk´et r´esz´ all´ıt´asunk HAMIS (olvasata: A ∨ B = A vagy B).

K¨ oz´ episkolai Matematikai ´ es Fizikai Lapok, 2022/2

93

Nem j´ arunk messze az igazs´ agt´ ol, ha ennek l´att´ an esz¨ unkbe jut a szorz´ ot´ abla, de arr´ ol majd k´es˝ obb ejt¨ unk sz´ ot. Ahhoz, hogy ezt az u ´j elrendez´est megszokjuk, n´ezz¨ uk meg az alapm˝ uveletek ´at´ır´as´at ebbe a form´aba.

´Igy egym´ as mell´e rakva o˝ket, sokkal jobban l´atszik a szab´ alyszer˝ us´eg, ´es megfigyelhet˝ o, hogy az igazi l´enyeg a vastagon keretezett r´eszekben, azok k¨ ul¨ onb¨ oz˝ os´eg´eben rejlik, ez´ert rejts¨ uk el a kev´esb´e fontos elemeket.

Teh´at az ilyen 2 × 2 ´ert´ekt´ abl´ azatb´ ol van tizenhat, amelyekb˝ ol eddig ¨ ot¨ ot neves´ıtett¨ unk. 94

K¨ oz´ episkolai Matematikai ´ es Fizikai Lapok, 2022/2

Ideje r´ at´ern¨ unk arra, hogy mit is jelent az az inform´aci´ ohi´ any, amelyet eml´ıtettem. N´emi t¨ opreng´es ut´ an ´eszrevehet˝ o, hogy az els˝o n´egy t´ abla szimmetrikus a f˝ o´ atl´ ora, ahogy a szorz´o- ´es ¨ osszead´ ot´ abla is. Ez a logikai m˝ uveletekn´el is azt jelenti, mint a sz´ amokn´ al: az els˝ o n´egy m˝ uvelet kommutat´ıv (A ∧ B = B ∧ A, A ∨ B = B ∨ A, A ⊗ B = B ⊗ A, A ⇔ B = B ⇔ A), a k¨ovetkeztet´es azonban nem az (A → B = B → A). Ezt persze eddig is tudtuk, de pont a t¨ ukr¨oz´es adja a keazata z¨ unkbe a hatodik m˝ uveletet, ami – bizony´ ara sejtj¨ uk – a B → A, ´ert´ekt´abl´ a m´asik ir´any´ u k¨ ovetkeztet´es f˝o´ atl´ ora t¨ ukr¨ oz´es´evel ad´ odik:

Ez ´ıgy m´eg mindig csak 6 lehet˝ os´eg, m´eg 10 h´atravan. A lehets´eges 16 t´ ablabels˝ ot elhelyezni egy 4 × 4-es, n´egyzet alak´ u t´ abl´ azatba ´erdemes, ami 8 × 8 bet˝ ut fog tartalmazni. Ezt m´ ar csak u olten¨ unk h, i bet˝ ukkel, hogy el˝o´all´ıtsuk ¨gyesen fel kell t¨ mind a 16 lehets´eges eredm´enyt. ´Igy n´ez ki u abl´ ara eml´ekeztet, el is neveztem ¨resen, mivel 8 × 8-as, a sakkt´ logikai sakkt´ abl´ anak. L´assuk a felt¨ olt´esi m´ odszert.

Ezek szerint, ha a t´abla valamelyik fel´et fel tudjuk t¨ olteni, akkor k´esz is lesz¨ unk, a t¨ ukr¨ oz´es tagad´ o volt´ at kihaszn´ alva ad´ odik a t¨ obbi. Erre a t´ abla als´ o fele az es´elyesebb, m´ar csak h´arom eredm´enyt´abla nincs neves´ıtve, vegy¨ uk ezeket szem¨ ugyre.

Az els˝o t´ abla igaz eredm´enyt ad, ha A igaz, ´es hamisat, ha A hamis. Ez nem lett m´ as, mint A. Hasonl´o megfontol´ asok alapj´an a m´asodikban B-t ´es a harmadikban az azonosan igazat, I-t azonos´ıthatjuk. Ezeket ´es tagad´ asukat (¬A, ¬B ´es H) is feljegyezve, a logikai sakkt´ abl´ ank kin´ezete a h´ ats´ o bels˝ o bor´ıt´ o 3. ´ abr´ aj´ an l´ athat´ o. A marad´ek n´egy helyre az ´es, ´es a vagy m˝ uveletek, tov´ abb´ a a k´et ir´any´ u  ul. k¨ ovetkeztet´es tagad´ asa ¬(A ∧ B), ¬(A ∨ B), ¬(A → B) ´es ¬(B → A) ker¨ Szemrev´etelezve a munk´ ankat megnyugodhatunk, mind a 16 lehets´eges kimenetet neves´ıtett¨ uk. Nem hanyags´ agb´ ol vagy tudatlans´agb´ ol haszn´aljuk csak az ´es, a vagy, a kiz´ ar´ o vagy, az azonoss´ ag ´es a k¨ ovetkeztet´es m˝ uveleteket, tov´ abb´ a a tagad´ ast, hanem az´ert, mert egyszer˝ uen nincs t¨ obb. (L´ asd a h´ats´ o bels˝ o bor´ıt´ o 4. ´ abr´ aj´ at.)

• Az els˝o sorba ´ırjunk csupa h bet˝ ut. • A harmadikba hi p´ arokat.

• Az o odikbe ih p´arokat. ¨t¨ • A hetedik sorba pedig ker¨ ulj¨ on csupa i bet˝ u.

Na de itt ´alljunk csak meg egy pillanatra. Lehet, hogy buzgalmunkban t´ ull˝ott¨ unk a c´elon? Mire is gondolok?

• Az els˝o oszlopp´art t¨ olts¨ uk fel h bet˝ ukkel. • A m´ asodikat hi p´arokkal.

• A harmadikat ih p´ arokkal. • V´eg¨ ul az u uket. ¨res cell´akba ´ırjunk i bet˝ ´Igy a t´ abl´ azat minden 2 × 2-es cell´ aja elt´er ´ a t¨ obbit˝ ol. Erdekes j´ at´ek a t´ abl´ azatban megkeresni a m´ar megismert hat m˝ uvelet hely´et (l´ asd a h´ ats´ o bels˝ o bor´ıt´ o 1. ´ abr´ aj´ at).

A tagad´asra. Egy adott m˝ uvelet, mondjuk az ´es eset´en t¨ obbf´ele m´ odon alkalmazhatjuk a tagad´ ast. Tagadhatjuk az els˝ o, vagy ak´ar a m´ asodik r´esz´ all´ıt´ ast, net´ an mindkett˝ ot k¨ ul¨ on-k¨ ul¨ on, vagy az eg´esz m˝ uvelet eredm´eny´et. Ezek persze mind m´ ast jelentenek, hogy jobban ´erthet˝ ov´e v´ aljon, legyen az A ´all´ıt´ as az, hogy Kedd van, a B ´ all´ıt´ as pedig az, hogy Esik az es˝ o. Ekkor: A∧B ¬A ∧ B A ∧ ¬B ¬A ∧ ¬B ¬(A ∧ B)

Helyezz¨ unk el egy kis p¨ otty¨ ot a sakkt´abla k¨ oz´eppontj´aba. A konstrukci´ ob´ol k¨ ovetkez˝ oen az erre a pontra szimmetrikusan elhelyezked˝o eredm´enyt´ abl´ ak egym´as tagad´ asai (amit a h´ats´ o bels˝ o bor´ıt´ o 2. ´ abr´ aj´ an ki-ki ellen˝ orizhet is: ha a t´ abla egyik mez˝ oj´en i vagy h bet˝ u van, akkor a t¨ uk¨ ork´ep´enek ugyanazon mez˝ oj´en h vagy i tal´ alhat´ o), ez´ert ezt a pontot a t´abla tagad´ oanak nevezz¨ uk. pontj´ K¨ oz´ episkolai Matematikai ´ es Fizikai Lapok, 2022/2

Az el˝oz˝ o r´esz utols´ o´ abr´ aj´an is megfigyelhet˝ o, hogy ¬(A ⇔ B) = A ⊗ B. Az´ert haszn´ alunk k´et sz´ınt, hogy minden m˝ uvelet ´es a tagad´ asa elt´er˝ o sz´ınnel l´atsz´odjon.

Kedd van ´es esik az es˝o. Nem kedd van ´es esik az es˝o. Kedd van ´es nem esik az es˝ o. Nem kedd van ´es nem esik az es˝ o. Nem igaz az, hogy kedd van ´es esik az es˝o.

Az ´ert´ekt´ abl´ azatok elk´esz´ıt´es´et az olvas´ ora b´ızom, azt azonban sz¨ ogezz¨ uk le, hogy a kapott ´ert´ekek mind a m´ ar t´ abl´ an l´ev˝ ok valamelyik´evel azonosak.

95

96

K¨ oz´ episkolai Matematikai ´ es Fizikai Lapok, 2022/2

Magyar´ azat: az eredetileg 100 ¨ osszpontsz´ ammal rendelkez˝ o lak´ onegyedben az 5 szintes h´azat nem ´ep´ıtj¨ uk meg, ´ıgy a cs¨ okkent´es ut´ an a pontsz´amok ¨ osszege 28 + 21 + 36 = 85. Bek¨ uldend˝o egy t¨ om¨ or´ıtett i556.zip ´ allom´anyban a program forr´ ask´ odja ´es dokument´ aci´ oja, amely tartalmazza a megold´ as r¨ ovid le´ır´ as´ at, ´es megadja, hogy a forr´ as´allom´any melyik fejleszt˝o k¨ ornyezetben ford´ıthat´ o.

K¨ onnyen bel´ athatjuk, hogy b´armely m˝ uveletb˝ ol indulunk is ki, az eredm´enyt´ abl´ aban az els˝o r´esz´ all´ıt´ as tagad´ asa oszlopcser´et (OCS), a m´ asodik tagad´ asa sor´ cser´et (SCS), mindkett˝ o tagad´asa ´ atl´ os cser´et (ACS), az eg´esz m˝ uvelet tagad´ asa pedig az i ´es h ´ert´ekek cser´ej´et, vagyis bet˝ ucser´et (BCS) jelent. A logikai kifejez´esek egyszer˝ us´ıt´ese, formaliz´al´asa ´es annak a megfejt´ese, hogy ennek az eg´esznek milyen szoros a kapcsolata a sz´am´ıt´og´epekkel, a cikk k¨ovetkez˝ o r´esz´eb˝ ol der¨ ul majd ki. T´oth Tam´as Budapest

Informatik´ab´ ol kit˝ u z¨ ott feladatok

I. 556. Egy v´ arosnegyedbe N lak´ oh´azat terveztek, melyek mindegyik´eben egyegy szinten azonos sz´am´ u lak´ as van. A h´ azakban a lak´ asok ´ert´eke a szintek sz´am´aval cs¨ okken. A lak´ asokat jellemezhetj¨ uk egy olyan P pontsz´ ammal, ami azt mutatja meg, hogy h´anyadik szinten van a h´ azban (p´eld´ aul egy harmadik szinten l´ev˝ o lak´ as pontsz´ama 3). A tervben szerepl˝ o h´ azak magass´ ag´ at v´ arosrendez´esi okokb´ ol cs¨ okkenteni kell ¨ osszesen K darab szinttel. A cs¨ okkent´es sor´ an el˝ofordulhat, hogy a tervezettn´el kevesebb sz´ am´ u lak´ oh´ azat ´ep´ıtenek a v´ arosnegyedbe. K´esz´ıts¨ unk programot i556 n´even, amely a tervben szerepl˝o h´azak K darab szinttel val´ o cs¨ okkent´es´et elv´egzi u ´gy, hogy k¨ ozben a tervben szerepl˝o h´azak ¨ossz´ert´eke a legkisebb m´ert´ekben cs¨ okken. o sor´ aban a lak´oh´azak N (2  N  10 000) A program standard bemenet´enek els˝ sz´ ama ´es K (2  K  10 000) ´ert´eke tal´ alhat´ o, a k¨ ovetkez˝ o N sorban pedig az, hogy o egy-egy h´ az h´ any szintes (2  Mi  25). A program standard kimenet´ere az el´erhet˝ maxim´alis pontsz´ amot ´ırjuk ki. Bemenet (a / jel sort¨ or´est helyettes´ıt) 4 5 / 7 / 5 / 6 / 8 K¨ oz´ episkolai Matematikai ´ es Fizikai Lapok, 2022/2

Kimenet 85 97

´ Egy kiskereskedelmi bolth´al´ozat moderniz´al´asba kezdett, ¨ I. 557 (E). onm˝ uk¨ od˝o kassz´akat a´ll´ıtottak be a nagyobb boltjaikba. Nem t´ ul s˝ ur˝ un, de vak ´es gyeng´en l´at´ o v´as´arl´ok is megfordulnak a boltban. Nekik egy sz¨ ovegfelolvas´ o programot k´esz´ıttettek. A programnak egy ismert hi´anyoss´ aga van: a sz´ amjegyekkel szerepl˝o oveget Egy ¨ ot nulla sz´ amokat karakterenk´ent olvassa fel, teh´at az 1500 forint” sz¨ ” ” nulla forint” form´aban. Mennyivel ´erthet˝ obb lenne az Ezer¨ otsz´ az forint” hangala” kot hallani, de ehhez a sz´amot sz¨ ovegg´e kell alak´ıtani. T´abl´ azatkezel˝ o program seg´ıts´eg´evel oldjuk meg az al´ abbi feladatokat. 1. Hozzuk l´etre a t´ abl´ azatkezel˝ oben a p´ enzt´ ar nev˝ u´ allom´anyt a program alap´ertelmezett form´ atum´ aban, ebben legyen h´ arom munkalap alap, p´ enzt´ ar ´es nagy sz´ amok n´even. 2. Tegy¨ uk lehet˝ ov´e, hogy az alap munkalap A1 cell´ aj´aba egy legfeljebb tizen¨ ot sz´ amjegyb˝ ol ´ all´ o pozit´ıv eg´esz sz´ amot lehessen bevinni. 3. Amennyiben az A1 ´ert´eke a felt´eteleknek megfelel˝ o, akkor az A3 cell´aban ott jelenjen meg a sz´ am karakterenk´enti sz¨ oveges v´ altozata. A sz´ amjegyek k¨ oz¨ pontosan egy sz´ ok¨ oz legyen, de a sz¨ oveg v´eg´en ne legyen f¨ ol¨ osleges sz´ ok¨ oz. P´eld´ aul ha A1 tartalma 23 710 346, akkor A3-ban jelenjen meg a kett˝ o h´arom ” h´et egy nulla h´arom n´egy hat” sz¨ oveg. 4. Tegy¨ uk lehet˝ ov´e, hogy a p´ enzt´ ar munkalap A1 cell´ aj´aba az o ot, ¨sszeg” sz´ ” a B1 cell´aba pedig egy legfeljebb kilenc sz´ amjegyb˝ ol a´ll´ o pozit´ıv eg´esz sz´ amot lehessen bevinni. 5. A munkalap D1, D3 ´es D5 cell´ aj´aba rendre ker¨ ulj¨ on a Fizet´esi m´ od”, az Euro ” ” ´arfolyam” ´es a Fizetend˝ o:” sz¨ oveg. ” o lista, az al´abbi n´egy listaelemmel: ” (¨ ures 6. A D2 cella legyen leg¨ord´ıthet˝ ” sz¨oveg), Forint k´arty´aval”, Forint k´eszp´enzzel”, Euro”. ” ” ” 7. Amennyiben B1 ´ert´eke a felt´eteleknek megfelel˝ o, u ´gy a D6 cell´aban jelenjen meg a helyes´ır´ asi szab´ alyok szerint B1 ´ert´eke az al´ abbiak szerint: a. Ha a D2 cella tartalma Forint k´arty´aval”, akkor a pontos ¨ osszeg a p´enznem” mel megtoldva, p´eld´ aul: 503 118 eset´en az ¨ otsz´azh´aromezer-sz´aztizennyolc ” forint”. b. Ha a D2 cella tartalma Forint k´eszp´enzzel”, akkor az ¨ osszeg ¨ ot¨ osre kere” k´ıtve, hiszen a lekisebb ´ert´ek˝ u p´enz´erme az ¨ otforintos, ´es a p´enznemmel otsz´azh´aromezer-sz´azh´ usz forint”. megtoldva, p´eld´ aul: 503 118 eset´en az ¨ ” c. Ha a D2 cella tartalma Euro” ´es a D4 cell´ aban szerepel az a´rfolyam, akkor ” az ¨osszeg a´tv´ altva eurora ´es centekre, p´eld´ aul: 503 118 ´es az euro ´arfolyam 360, akkor az ezerh´aromsz´azkilencvenh´et euro ¨ otven¨ ot cent”. ” 98

K¨ oz´ episkolai Matematikai ´ es Fizikai Lapok, 2022/2

d. A sz´ amok sz¨ oveg´e alak´ıt´ asa feleljen meg az al´abbi nyelvtani szab´ alyoknak: 2000-ig egybe kell ´ırni a sz´ amot, f¨ ol¨ otte balr´ol h´ armas´ aval csoportos´ıtva k¨ ot˝ ojellel. sz´ am 110 315 503 824

le´ır´as sz´ azt´ız h´ aromsz´ aztizen¨ ot otsz´ azh´ arom ¨ nyolcsz´ azhuszonn´egy

sz´ am 1 051 1 895 3 420 45 000 403

Bek¨ uldend˝o egy i557.zip t¨ om¨ or´ıtett a´llom´anyban a munkaf¨ uzet ´es egy r¨ ovid dokument´ aci´ o, amely megadja, hogy a megold´ as milyen t´ abl´ azatkezel˝ o program melyik verzi´oj´aban k´esz¨ ult. I. 558. A 2022. janu´ ari sz´ amban megjelent B. 5214. feladatban egy olyan 1-esekb˝ol ´es 0-kb´ ol ´ all´ o sz´ amsorozatot kerestek, amely minden pozit´ıv alap´ u sz´ amrendszerben ´ertelmezve h´ arommal oszthat´ o sz´ amot jelent.

le´ır´as ezer¨otvenegy ezernyolcsz´ azkilencven¨ot h´ aromezer-n´egysz´azh´ usz negyven¨otmilli´ o-n´egysz´ azh´ arom

A feladat ´altal´ anos´ıt´ asak´ent k´esz´ıts¨ unk programot, amely megvizsg´ alja a legf¨oljebb 8 jegy˝ u, csupa 0 ´es 1 jegyekb˝ ol a´ll´ o sz´ amokat egy adott R sz´ amrendszerben ´ertelmezve, ´es megadja, hogy k¨ oz¨ ul¨ uk h´ any oszthat´o egy adott K sz´ ammal.

8. Tegy¨ uk lehet˝ ov´e, hogy a nagy sz´ amok munkalap A1 cell´aj´aba egy legfeljebb tizenhat sz´ amjegyb˝ ol ´ all´ o pozit´ıv eg´esz sz´ amot lehessen bevinni. 9. Amennyiben az A1 ´ert´eke a felt´eteleknek megfelel˝ o, az A3 cell´aban jelenjen meg az A1-be ´ırt sz´am sz¨ oveges v´ altozata, pontosan u ´gy, mint a p´enzt´ar munkalapn´ al, csak nem kell a c´ımlet megnevez´ese. P´eld´ aul, ha A1 tartalma 5 742 568 741 225, akkor A3-ban jelenjen meg a ¨otbilli´ o-h´etsz´aznegyvenkett˝ o” milli´ard-¨ otsz´azhatvannyolcmilli´o-h´etsz´aznegyvenegyezer-k´etsz´azhuszon¨ot” sz¨oveg.

A program a standard bemenet els˝o sor´ ab´ ol olvassa be a sz´ amrendszer R alapot, majd adja meg, hogy h´ any sz´ am´at (2  R  1000), ´es a K (2  K  1000) oszt´ olyan vizsg´ alt sz´ am van, amely a megadott sz´ amrendszerben ´ertelmezve K-val oszthat´ o. Bemenet 29 7

Kimenet 8

Bek¨ uldend˝o egy t¨ om¨ or´ıtett i556.zip ´ allom´anyban a program forr´ ask´ odja ´es dokument´ aci´ oja, amely tartalmazza a megold´ as r¨ ovid le´ır´ as´ at, ´es megadja, hogy a forr´ as´allom´any melyik fejleszt˝oi k¨ ornyezetben ford´ıthat´ o. I/S. 60. Egy ´erdekes t´eny a vir´ agokr´ ol, hogy szirmaik sz´ama gyakran Fibonaccisz´ am. Egy nap tal´altunk N vir´ agot a r´eten, az i-ediknek T [i] vir´ agszirma van. Adjuk meg azt a minim´alis sziromsz´amot, amennyit el kell t´ avol´ıtani a vir´ agokr´ ol ¨osszesen, hogy mindegyiken Fibonacci-sz´ am legyen a szirmok sz´ama. A bemenet els˝ o sor´ aban az N sz´ am tal´ alhat´ o. A k¨ ovetkez˝ o sor N sz´ amot tartalmaz, az i-edik sz´ am T [i], az i-edik vir´ag sziromsz´ama. A kimenet egyetlen sor´ aban egy sz´ am szerepeljen, hogy minim´alisan mennyi szirmot kell elt´ avol´ıtani agr´ ol o sszesen, hogy a fentieknek megfeleljen. az N vir´ ¨

Minta az alap munkalaphoz

Bemenet 4 4 10 5 11

Kimenet 6

Minta a p´ enzt´ ar munkalaphoz

Magyar´ azat: t´ avol´ıtsunk el az 1. vir´ agr´ ol egy szirmot, a 2. vir´agr´ ol k´et szirmot ´es a 4. vir´agr´ ol h´ arom szirmot, teh´at ¨ osszesen hatot.

Minta a nagy sz´ amok munkalaphoz

olimit: 0,4 mp. Korl´ atok: 1  N  10 000, 1  T [i]  109 . Id˝ ´ Ert´ekel´es: a pontok 50%-a kaphat´ o, ha 1  N  100 ´es 1  T [i]  100 eset´en helyesen m˝ uk¨odik a program. Bek¨ uldend˝o egy is60.zip t¨ om¨ or´ıtett a´llom´anyban a megfelel˝oen dokument´ alt ´es kommentezett forr´ asprogram, amely tartalmazza a megold´ as l´ep´eseit, valamint megadja, hogy a program melyik fejleszt˝ oi k¨ ornyezetben futtathat´ o.

Seg´edsz´ am´ıt´ asokat mindh´arom munkalapon a K oszlopt´ ol jobbra v´egezhet¨ unk. A megold´ ashoz makr´ o vagy m´ as program nem haszn´alhat´ o, csak a t´abl´ azatkezel˝ o be´ep´ıtett f¨ uggv´enyei.

S. 159. Egy kalandj´at´ek N × M t´eglalap alak´ u ter¨ uletb˝ ol ´ all. Minden ter¨ uleten egy pr´ oba tal´alhat´ o, melynek neh´ezs´ege pozit´ıv eg´esz sz´ am. Van egy h˝ os¨ unk, aki kezdetben a bal fels˝ o sarokban a´ll ´es egyes szint˝ u. A h˝ os akkor l´ephet r´ a egy,

K¨ oz´ episkolai Matematikai ´ es Fizikai Lapok, 2022/2

99

100

K¨ oz´ episkolai Matematikai ´ es Fizikai Lapok, 2022/2

a jelenlegi hely´evel oldalszomsz´edos ter¨ uletre, ha a szintje legal´ abb akkora, mint az ott tal´ alhat´ o pr´ oba neh´ezs´ege. A ter¨ uleten tal´ alhat´ o pr´ oba teljes´ıt´ese ut´ an a h˝ os szintje eggyel nagyobb lesz. A h˝ os tetsz˝olegesen l´epkedhet a t´abla m´ar megl´atogatott (teljes´ıtett) mez˝ oin, de csak egyszer – az els˝ o r´ al´ep´eskor – emelkedik a szintje. Teljes´ıteni tudja-e a h˝ os a j´at´ekot, azaz el tud-e jutni a jobb als´ o ter¨ uletre? Bemenet: az els˝o sor tartalmazza a sorok N ´es az oszlopok M sz´ am´at. A k¨ovetkez˝ o N sor mindegyike M sz´ amot tartalmaz, az adott ter¨ uleten l´ev˝ o pr´ oba neh´ezs´eg´et. A bal fels˝o sarokban nincs pr´oba, ´ıgy az az ´ert´ek mindig 0. Kimenet: az els˝ o sorban az IGEN” sz´ o szerepeljen, ha a j´at´ek teljes´ıthet˝ o, ´es ” o, adjuk meg a legkisebb szintet, amivel a NEM” sz´ o k¨ ul¨ onben. Ha a j´ at´ek teljes´ıthet˝ ” teljes´ıteni lehet. Ha az els˝o sor NEM” volt, adjuk meg a szab´ alyos l´ep´esekkel el´erhet˝ o ” legnagyobb szintet. Bemenet (a / jel sort¨ or´est helyettes´ıt) 3 3 / 0 1 2 / 3 5 6 / 4 4 2

Kimenet (a / jel sort¨or´est helyettes´ıt) IGEN / 7

Magyar´ azat: az 5-¨ os ´es 6-os pr´ob´ ak kiv´etel´evel mindet teljes´ıteni kell. Ezek ut´ an a h˝ os 7-es szint˝ u lesz. Korl´ atok: 2  N, M  100, egy pr´ oba neh´ezs´ege legfeljebb 20 000. Id˝olimit: 0,5 mp. ´ ekel´es: a pontok 50%-a kaphat´ Ert´ o, ha a kimenet els˝ o sora helyes. Bek¨ uldend˝o egy s159.zip t¨ om¨ or´ıtett a´llom´anyban a megfelel˝oen dokument´ alt ´es kommentezett forr´ asprogram, amely tartalmazza a megold´ as l´ep´eseit, valamint megadja, hogy a program melyik fejleszt˝ oi k¨ ornyezetben futtathat´ o.

 A feladatok megold´asai regisztr´aci´o ut´an a k¨ovetkez˝o c´ımen t¨olthet˝ok fel: https://www.komal.hu/munkafuzet Beku ¨ld´esi hat´arid˝o: 2022. m´arcius 15.



´ Eletm˝ ´ R´atz Tan´ar Ur ud´ıjak ´atad´asa 2021 december´eben ´ Eletm˝ ´ Huszonegyedik alkalommal adt´ ak a´t a R´ atz Tan´ ar Ur ud´ıjat azon a´ltal´ anos ´es k¨ oz´episkol´ aban oktat´ o pedag´ogusoknak, akik matematika-, fizika-, k´emia-, biol´ogiaoktat´ as ter¨ ulet´en kimagasl´ o teljes´ıtm´enyt ny´ ujtanak a tant´ argyak n´epszer˝ us´ıt´es´eben ´es a tehets´eggondoz´ asban. K¨ oz´ episkolai Matematikai ´ es Fizikai Lapok, 2022/2

101

A Magyar Tudom´anyos Akad´emia d´ıszterm´eben megrendezett d´ıj´atad´ on kilenc tan´ ar vehette ´ at az Alap´ıtv´ any a Magyar Term´eszettudom´ anyos Oktat´ as´ert R´ atz L´aszl´or´ol elnevezett d´ıj´at. A kit¨ untet´est az Ericsson Magyarorsz´ag, a Graphisoft ´es a Richter Gedeon a´ltal l´etrehozott alap´ıtv´ any kurat´oriuma ´ıt´eli oda minden ´evben, ¨osszesen 12 milli´o forint ´ert´ekben. A vil´agban, amelyben ´el¨ unk a technol´ ogia gyors u u fejl˝ od´es´enek k¨ osz¨ onhe¨tem˝ t˝oen az inform´aci´ o elk´epeszt˝o sebess´eggel terjed ´es b´ arki sz´ am´ara pillanatok alatt el´erhet˝o. Nagy k´erd´es azonban, hogy a gyerekek, akik b´ar ebben az u ´j vil´ agban digit´ alis bennsz¨ ul¨ ottk´ent ´elnek, mit kezdenek ezzel a rengeteg inform´ aci´ oval, hogyan v´alasztj´ak ki ezek k¨ oz¨ ul, mi a val´odi ´es mi a megb´ızhat´ o – hogy mik a t´enyek, hogy mit mond a tudom´ any. Ezt a tud´ ast, amire alapozva k´es˝ obb d¨ ont´eseket hoznak a fiatalok, az iskol´ aban ´ Eletm˝ ´ a pedag´ogusokt´ ol kapj´ak meg. A R´atz Tan´ ar Ur ud´ıj imm´ar 21. ´eve azon kiemelked˝o term´eszettudom´ anyos t´ argyakat tan´ıt´ o pedag´ogusok munk´ aj´ara h´ıvja fel a figyelmet, akik a tud´ asukkal ´es pedag´ ogiai ´erz´ek¨ ukkel sz´eles l´ at´ ok¨ or˝ u feln˝ottekk´e nevelik di´akjaikat. A 1,5 milli´o forinttal j´ ar´ o kit¨ untet´est az Ericsson Magyarorsz´ag, a Graphisoft ´es a Richter Gedeon ´ altal l´etrehozott alap´ıtv´ any kurat´oriuma ´ıt´eli oda minden ´evben. A mi feladatunk az, hogy megmutassuk ´es elismerj¨ uk azokat a term´eszet” tudom´anyos t´ argyakat oktat´ o pedag´ogusokat, akik hivat´ astudatukkal gener´aci´ ok j¨ov˝oj´ere vannak pozit´ıv hat´ assal. Szeretn´enk egy´ uttal p´eld´ at mutatni a gazdas´agi amogass´ ak az oktat´ ast, hiszen az igaszerepl˝oknek is, hogy lehet˝os´egeikhez m´erten t´ zi befektet´es a magyar gazdas´ ag sz´am´ara a tud´ asban rejlik, a tud´ as ´ atad´ as´ anak kulcsa pedig a pedag´ogusainkban.” – mondta el Kro´o Norbert, professzor, akad´emikus, az Alap´ıtv´ any a Magyar Term´eszettudom´ anyos Oktat´ as´ert kurat´ orium´anak eln¨oke. ´ Eletm˝ ´ A R´atz Tan´ ar Ur ud´ıjat az orsz´ag b´ armely a´ltal´ anos vagy k¨ oz´episkol´aj´aban tan´ıt´o vagy kor´ abban ott tev´ekenyked˝ o pedag´ogus megkaphatja. A d´ıjazott pedag´ogusok valamennyien a re´ al tant´argyak oktat´asi sz´ınvonal´anak emel´es´e´ert dolgoznak, di´ akjaik sikeresen szerepelnek orsz´ agos tudom´anyos versenyeken, az oktat´as mellett rendszeresen tov´ abbk´epzik magukat, t´ aj´ekozottak az adott tudom´ any ter¨ ulet´en el´ert eredm´enyekr˝ ol, gyakran tank¨ onyvek ´es szakmai foly´ oiratok szerz˝oi. Az idei ´evben a k¨ ovetkez˝o pedag´ ogusoknak ´ıt´elte oda a d´ıjat a kurat´orium: ´ ol Fodor Erika (Budapest, ELTE Trefort Agoston Gyakorl´ o Gimn´azium) K´emi´ ab´ ´ ´es Sarka Lajos (Ny´ıregyh´ azi Egyetem E¨ otv¨ os J´ ozsef Gyakorl´ o Altal´ anos Iskola ´es Gimn´ azium); ´ Biol´ ogi´ab´ol Dr. Moln´ar Katalin (Budapest, ELTE Radn´ oti Mikl´ os Gyakorl´ o Altal´ anos Iskola ´es Gyakorl´o Gimn´azium) ´es Horv´ath Zsolt (G¨ od¨ oll˝ oi Reform´atus L´ıceum Gimn´ azium); Matematik´ ab´ ol Csah´ oczi Erzs´ebet (Budapest, ELTE Gyerty´ anffy Istv´ an Gyakorl´ o ´ Altal´anos Iskola) ´es Kov´acs Istv´an (Szegedi Tudom´ anyegyetem Gyakorl´ o Gimn´a´ zium ´es Altal´ anos Iskola);

102

K¨ oz´ episkolai Matematikai ´ es Fizikai Lapok, 2022/2

Fizik´ ab´ ol Dr. Vank´ o P´eter (Budapest, BME Fizikai Tansz´ek) ´es P´apai Gyul´an´e ´es ´ P´apai Gyula (Fert˝od, Babos J´ ozsef Altal´ anos Iskola /Soproni Szakk´epz´esi Centrum, Vas- ´es Villamosipari Technikum). Id´ez¨ unk a matematika, illetve a fizika ter¨ ulet´en d´ıjazott tan´ arokr´ ol k´esz´ıtett r¨ ovidfilmekb˝ ol. Matematika Csah´ oczi Erzs´ebet (Budapest): A d´elut´ anjaim azzal teltek, hogy korrepet´ altam ´es tehets´eget gondoztam, versenyre k´esz´ıtettem a gyerekeket. Egy´ebk´ent nagyon szerettem, mert j´ o volt l´ atni, amikor egy gyerek elkezdett kiny´ılni ´es elkezdte elhinni mag´ ar´ ol, hogy ˝ oneki ez menni fog. Ez egy nagyon fontos dolog. A gyerek ´erezze azt, hogy most megint el˝ orel´eptem egy picit, megint, megint; ez is kivil´ agosodott, ez is kivil´ agosodott. Van egy pont, ahol azt´ an ´ ori´ asi gyors fejl˝ od´es k¨ ovetkezik. Az egy boldog´ıt´ o pillanat, l´ atni egy gyereket, amikor m´ ar ´erzi azt, hogy: Megvan! Megvan! ” ´ is tudom.” En A tan´ arn˝ or˝ ol sz´ol´o r¨ ovidfilm itt tekinthet˝o meg: https://www.youtube.com/watch?v=sWhhxNbY4Sw. Kov´acs Istv´an (Szeged): Ilyen tan´ıtv´ anyom is volt, aki j¨ ott ´es mondta, hogy: Tan´ ar u ´r, ne haragudjon, de m´ ar any´ anak se ment a matematika.” Ez nem ¨ or¨ ok” ´ min´el kisebb a matematika ir´ l˝ odik! Es anti motiv´ aci´ oja a gyereknek, ann´ al t¨ obbet ´ t¨ ´ h´ kell gyakorolni. Es urelemmel. Es at sok-sok dics´erettel. . . . Sokkal jobban ´ atl´ atj´ ak a bonyolult dolgokat is. El´eg sok bonyolult dologgal tal´ alkoznak itt az o ´r´ akon. . . . Az emelt szint˝ u ´erets´egi tananyag f¨ ol¨ ott is vannak bizonyos anyagr´eszek, amiket tan´ıtunk, teh´ at nem csak azt pr´ ob´ aljuk el´erni, hogy k¨ onnyed´en bejussanak k¨ ul¨ onb¨ oz˝ o egyetemekre, hanem azt is, hogy ott j´ o eredm´ennyel bent is maradjanak. J´ o hangulat legyen o ´r´ an. Nem nagyon ´erzem j´ ol magam egy ilyen fesz´elyezett, k¨ ot¨ ott t´ arsas´ agban. Mikor kij¨ ov¨ ok az ´ or´ ar´ ol, akkor van egy olyan j´ o ´erz´ese az embernek, hogy ezt most klasszul megcsin´ altam. Merthogy tudom, hogy mit akartam, ´es eljutottak od´ aig ´ szerintem a di´ akok, amit ´en szerettem volna, meg is ´ertett´ek – ez egy j´ o ´erz´es. Es sz´ amukra is j´ o ´erz´es. A tan´ ar u ´rr´ ol sz´ol´o r¨ ovidfilm itt tekinthet˝o meg: https://www.youtube.com/watch?v=mth-sSzouGY. Ki volt R´ atz tan´ ar u ´r ´es mit jelent ma nek¨ unk pedag´ ogiai munk´ass´ aga?: https://www.youtube.com/watch?v=y2gHM5d1n5M. Fizika Dr. Vank´ o P´eter (Budapest): Az igaz´ an m´ely, l´enyeges tapasztalatokat tulajdonk´eppen mi is csak sejtj¨ uk, ´es neh´ez megosztani. De v´eg¨ ul is az eg´esz ´elet¨ unk arr´ ol sz´ ol, hogy az ember ezt szereti ´ atadni. . . . Az a ´ltal´ anos iskol´ aban meg a gimn´ aziumban is nagyon j´ o matematikatan´ arom volt, ´es akkor ´en matekkal kezdtem foglalkozni. Fizikatan´ arral nem volt szerencs´em, . . . de ott volt a K¨ oMaL, ´es elkezdtem n´ezeget´ azt´ ni a feladatokat, ´es k¨ onyvekb˝ ol hozz´ a magam tanultam meg. Es an egy id˝ o ut´ an ez ´ aziumba, akkor 18 ´ ora jobban ment, mint a matek. 91-ben ker¨ ultem az Arp´ ad Gimn´ volt a k¨ otelez˝ o´ orasz´ am. Ebben az iskol´ aban speci´ alis matematika tagozatos oszt´ aly K¨ oz´ episkolai Matematikai ´ es Fizikai Lapok, 2022/2

103

volt, ´es a fizik´ at minden oszt´ alyban bontva tan´ıtottuk. 18 vagy 36 gyerek, az nem egy 2-es szorz´ o ´es egy ilyen sz´ ambeli k¨ ul¨ onbs´eg, hanem egy min˝ os´egileg m´ as dolog. Teh´ at 18 gyerek k¨ oz¨ ott szavak n´elk¨ ul uralni lehet a helyzetet. 92-ben oszt´ alyf˝ on¨ ok lettem. Borzaszt´ o szerencs´em volt, mert kaptam egy olyan oszt´ alyt, ahol az oszt´ alynak a nagyobbik fele nyitott volt a dolgokra, . . . ´es ott t´enyleg egy ilyen n´egy ´ev flow volt, ´es j¨ ottek az eredm´enyek, . . . Belevetettem magam, vittem a tan´ıtv´ anyaimat kir´ andulni, biciklizni, t´ ajfut´ o szakoszt´ alyt csin´ altam az iskol´ aban. K¨ oz´episkol´ asok, az olimpi´ ara k´esz¨ ul˝ ok, h´ at ˝ ok pedig egy hihetetlen ´erdekl˝ od˝ o ´es nyitott t´ arsas´ ag, ´es t´enyleg az orsz´ agunknak a legtehets´egesebb gyerekei. Egy ilyen olimpiai szakk¨ or¨ on van, hogy a di´ akok mondj´ ak el a megold´ ast ´es olyan megold´ ast mondanak el, ami nekem nem jutott volna az eszembe. Ezek szint´en klassz ´elm´enyek. A tan´ ar u ´rr´ ol sz´ol´o r¨ ovidfilm itt tekinthet˝o meg: https://www.youtube.com/watch?v=FfmKoGCTA9Q. P´apai Gyul´an´e ´es P´apai Gyula (Fert˝ od ´es Sopron): ´ – En azt gondolom, hogy a pedag´ ogus szem´elyis´ege ´es a szakmai tud´ asa k¨ oz¨ ott nem tenn´ek k¨ ul¨ onbs´eget. S˝ ot, ha helyezhetn´em, akkor t´ an a pedag´ ogus szem´elyis´eg´et helyezn´em el˝ obbre. (P. Gy.-n´e) – Mikor k´erdezik t˝ olem, hogy ki a j´ o pedag´ ogus, azt szoktam mondani, hogy azt nem tudom, de ut´ olag mindig tudni lehet: az a j´ o pedag´ ogus, akir˝ ol h´ usz ´ev m´ ulva is u ´gy eml´ekeznek a tan´ıtv´ anyai. (P. Gy.) – Az ismerets´eg¨ unk majd hatvan ´evre ny´ ulik vissza. P´ecsre ker¨ ult¨ unk mind ´ ott el˝ a ketten a Pedag´ ogiai F˝ oiskol´ ara matematika–fizika–m˝ uszaki szakra. Es osz¨ or munkakapcsolatunk volt. (P. Gy.-n´e) – Az´ert, mert engem v´ alasztottak meg csoportvezet˝ onek, ˝ ot meg csoportvezet˝ ohelyettesnek. Nagyon ragaszkodtam hozz´ a, mert pont egy ilyen agilis csoportvezet˝ ohelyettesre volt sz¨ uks´egem. (P. Gy.) ´ h´ – Es at ez tartott k¨ or¨ ulbel¨ ul h´ arom ´es f´el ´evig. Azt´ an j¨ ott a p´ alyav´ alaszt´ as. (P. Gy.-n´e) – Egyszer csak ´eszrevett¨ uk, hogy mi egy¨ utt akarunk elhelyezkedni. A l´enyeg az, hogy ideker¨ ult¨ unk Fert˝ odre, az els˝ o k¨ usz¨ ob¨ ok ´ atl´ep´ese ut´ an. De amikor ´ıgy bet´evedt¨ unk a fizika szert´ arba, akkor arra j¨ ott¨ unk r´ a, hogy ez egy romhalmaz. Na, ´ hajr´ a! Ugyis energi´ ank van, azzal j¨ ott¨ unk ki, neki´ allunk eszk¨ oz¨ oket gy´ artani. (P. Gy.) – Ha a gyerek eszk¨ ozh¨ oz ny´ ulhat ´es tev´ekenykedhet, akkor m´ ask´epp a ´ll a tant´ argyhoz. Teh´ at megszereti azt a t´ argyat. Az volt a v´elem´eny¨ unk, hogy az alapvet˝ o fizikai jelens´eget tapasztal´ as u ´tj´ an lehet eredm´enyesen tan´ıtani. (P. Gy.-n´e) – A fizika k¨ onnyebben megmagyar´ azhat´ o. Benne ´el¨ unk egy term´eszetben. Ha azt akarom, hogy a term´eszettel j´ o legyen a kapcsolatom, akkor legal´ abb valamit ´ ez lelkes´ıtette is a gyerekeket. (P. Gy.) ismerjek meg bel˝ ole, . . . Es ´ azt hiszem, hogy a mi kett˝ – En onk sikere meg eredm´enye . . . abban rejlett, hogy tudtunk megfelel˝ o kontaktust teremteni a gyerekekkel. (P. Gy.-n´e) A tan´ ar h´ azasp´ arr´ ol sz´ol´o r¨ ovidfilm itt tekinthet˝o meg: https://www.youtube.com/watch?v=6GTHs9wvJIo. 104

K¨ oz´ episkolai Matematikai ´ es Fizikai Lapok, 2022/2

Feladatunk a T1 h˝ om´ers´eklet meghat´ aroz´ asa. Ezt t¨ obbf´ele m´odszerrel is megtehetj¨ uk.

Besz´amol´ o a 2021. ´evi Eo ol ¨tvo ¨s-versenyr˝

Az E¨ otv¨ os Lor´ and Fizikai T´arsulat 2021. ´evi E¨otv¨os-versenye okt´ober 15-´en ult megrendez´esre. Ez´ert d´elut´ an 3 ´orai kezdettel t´ız magyarorsz´ agi helysz´ınen1 ker¨ k¨ ul¨ on k¨ osz¨ onettel tartozunk mindazoknak, akik ebben szervez´essel, fel¨ ugyelettel a seg´ıts´eg¨ unkre voltak. A versenyen a h´ arom feladat megold´as´ara 300 perc a´ll rendelkez´esre, b´ armely ´ırott vagy nyomtatott seg´edeszk¨oz haszn´alhat´ o, de (nem programozhat´ o) zsebsz´amol´og´epen k´ıv¨ ul minden elektronikus eszk¨oz haszn´alata tilos. Az E¨ otv¨ os-versenyen azok vehetnek r´eszt, akik vagy k¨oz´episkolai tanul´ok, vagy ¨ a verseny ´ev´eben fejezt´ek be k¨ oz´episkolai tanulm´anyaikat. Osszesen 69 versenyz˝o adott be dolgozatot, 14 egyetemista ´es 55 k¨ oz´episkol´as. Ismertetj¨ uk a feladatokat ´es azok megold´ as´ at.



I. megold´as. Legyen a h´eliumg´ az lassan v´altoz´ o pillanatnyi h˝ om´ers´eklete T , t´erfogata V . Az elv´ alaszt´ o fal j´o h˝ ovezet´ese miatt az oxig´eng´az h˝ om´ers´eklete is T . Ha a dugatty´ u elmozdul´ asa miatt a h˝ om´ers´eklet ΔT ´ert´ekkel n˝ o, a h´eliumg´ az t´erfogata pedig ΔV ´ert´ekkel v´ altozik meg (ΔV < 0), akkor az eg´esz rendszer bels˝ o energi´aj´anak v´ altoz´ asa (1)

ΔE =

5 3 nR ΔT + nR ΔT = 4nR ΔT. 2 2

A h´eliumg´az nyom´ asa:

T . V Az eg´esz rendszerre alkalmazott els˝ o f˝ot´etel szerint p = nR

−pΔV = ΔE, vagyis

1. feladat. Egy h˝ oszigetelt, hengeres tart´ alyt egy j´ o h˝ ovezet˝ o, r¨ ogz´ıtett fal oszt k´et egyforma henger alak´ u t´err´eszre. Az egyik t´erf´elben h´eliumg´ az, a m´ asikban azzal megegyez˝ o anyagmennyis´eg˝ u oxig´eng´ az tal´ alhat´ o, mindk´et g´ az kezdeti h˝ om´ers´eklealy egyik v´eg´et k¨ onnyen mozg´ o, h˝ oszigetel˝ o te T0 , kezdeti t´erfogata pedig V0 . A tart´ dugatty´ u z´ arja le, amellyel a h´eliummal t¨ olt¨ ott t´err´esz t´erfogata v´ altoztathat´ o. Hat´ arozzuk meg a hengerben l´ev˝ o g´ azok v´egs˝ o h˝ om´ers´eklet´et, miut´ an a dugatty´ u lass´ u okkentett¨ uk! mozgat´ as´ aval a h´eliumg´ az t´erfogat´ at V0 /2-re cs¨ (Vigh M´ at´e ) Megold´as. Az 1. ´ abra a kezdeti a´llapotot ´es a v´eg´ allapotot mutatja.

ΔV ΔT +4 = 0. V T

(2)

Szorozzuk meg (2)-t T 4 V -vel, ´es haszn´ aljuk ki, hogy a megv´ altoz´ asok kicsik (ez´ert a n´egyzet¨ uket ´es a magasabb hatv´ anyaikat elhanyagolhatjuk): T 4 ΔV + 4T 3 V ΔT = Δ(T 4 V ) = 0, teh´at T 4 V a folyamat sor´ an ´ alland´ o marad. A h´eliumg´ az kezdeti ´es v´eg´ allapot´ at ¨osszehasonl´ıtva kapjuk, hogy √ V0 4 vagyis T1 = 2 T0 ≈ 1,2 T0 . T04 V0 = T14 , 2 Ugyanezt az eredm´enyt az (1)-ben szerepl˝o kicsiny v´ altoz´ asok ¨ osszegz´es´evel (integr´al´assal) is megkaphatjuk: V 0 /2

V0

1 dV + 4 V

T1

T0

T1 1 dT = − ln 2 + 4 ln = 0, T T0

vagyis T1 =

√ 4

2 T0 .

II. megold´as. Az (1) egyenlet szerint a folyamat tekinthet˝o egy f = 8 szabads´agi fok´ u g´az adiabatikus ¨ osszenyom´ as´ anak. Erre a folyamatra a fajh˝ oh´ anyados f +2 κ = f = 54 , teh´ at az adiabatikus ´allapotv´ altoz´ as egyenlete: alland´ o, T V κ−1 = T V 1/4 = ´

1. ´ abra 1

R´eszletek a verseny honlapj´ an: http://eik.bme.hu/~vanko/fizika/eotvos.htm.

K¨ oz´ episkolai Matematikai ´ es Fizikai Lapok, 2022/2

105

ahonnan T1 = 106

√ 4 2 T0 . K¨ oz´ episkolai Matematikai ´ es Fizikai Lapok, 2022/2

III. megold´as. K´ezik¨ onyvekben2 ´es k´epletgy˝ ujtem´enyekben megtal´ alhat´ o, hogy n mol anyagmennyis´eg˝ u, f szabads´ agi fok´ u molekul´akb´ ol ´all´o, T h˝om´ers´eklet˝ u ´es V t´erfogat´ u ide´ alis g´az entr´opi´ aja S(T, V ) =

T V f nR ln + nR ln . 2 T0 V0

Az entr´opia nullpontja ¨ onk´enyesen v´ alaszthat´ o, a fenti k´epletben p´eld´ aul S(T0 , V0 ) = 0 o kezdeti h˝om´ers´eklet ´es t´erfogat). (ahol T0 ´es V0 lehet a feladatban szerepl˝ A vizsg´ alt folyamatban nincs h˝ocsere a rendszer ´es a k¨ornyezete k¨oz¨ott, tov´ abb´ a (a dugatty´ u lass´ u mozgat´ asa eset´en) a folyamat reverzibilis, ´ıgy a rendszer entr´ opi´ aja v´altozatlan marad:     fO2 fHe T1 T1 V0 /2 V0 nR ln nR ln + nR ln + nR ln + = 0, 2 T0 V0 2 T0 V0 vagyis (tudva, hogy fHe = 3 ´es fO2 = 5) 4 ln azaz a T1 =

1 T1 + ln = 0, T0 2

√ 4 2 T0 eredm´eny ad´ odik.

Megjegyz´es. Ha a h´eliumg´ az t´erfogat´ at olyan gyorsan cs¨ okkentj¨ uk a fel´ere, hogy az oxig´eng´ az nem tud azonnal felmelegedni, akkor a folyamat irreverzibiliss´e v´ alik, vagyis az entr´ opia n˝ oni fog. Mivel adott t´erfogat eset´en a magasabb h˝ om´ers´eklethez tartozik nagyobb entr´ opia, a dugatty´ u hirtelen elmozd´ıt´ asa ut´ an a k´et g´ az v´eg¨ ul (a h˝ om´ers´eklet kiegyenl´ıt˝ od´ese ut´ an) jobban felmelegszik, mint a feladatban szerepl˝ o lass´ u¨ osszenyom´ asn´ al.

2. feladat. Egy henger alak´ u,  hossz´ us´ ag´ u ´es R   sugar´ u, l´egmagos szolenoid meneteinek sz´ ama N . A tekercs belsej´ebe egy r  R sugar´ u, a szolenoid szimmetriatengely´ere mer˝ oleges s´ık´ u, L induktivit´ as´ u szupravezet˝ o gy˝ ur˝ ut helyez¨ unk (a gy˝ ur˝ u ´es a szolenoid k¨ oz´eppontja egybeesik). a) N¨ ovekszik vagy cs¨ okken a szolenoid induktivit´ asa a gy˝ ur˝ u behelyez´ese k¨ ovetkezt´eben? b) Hat´ arozzuk meg az induktivit´ as megv´ altoz´ as´ anak nagys´ ag´ at! (Sz´echenyi G´ abor ) Megold´as. a) A szupravezet˝ o f´azisban l´ev˝ o anyagoknak az az egyik k¨ ul¨onleges tulajdons´ aguk, hogy az elektromos ellen´ all´ asuk nulla. Ha egy szupravezet˝ o gy˝ ur˝ uorv´eny alapj´ an v´egtelen nagy a´ramnak ben fesz¨ ults´eg induk´al´odna, akkor az Ohm-t¨ kellene benne folynia. Ennek a fizikai k´eptelens´egnek a felold´ asa az, hogy a szupravezet˝ o gy˝ ur˝ uben nem induk´ al´odhat fesz¨ ults´eg, azaz a gy˝ ur˝ un a´thalad´ o m´agneses fluxus ´ert´eke nem v´ altozhat meg. 2

L´ asd pl. a 333+ Furfangos Feladat Fizik´ ab´ ol 194. feladat´ anak megold´ as´ at.

K¨ oz´ episkolai Matematikai ´ es Fizikai Lapok, 2022/2

107

Az egyszer˝ us´eg kedv´e´ert t´etelezz¨ uk fel, hogy kezdetben, amikor a szolenoidban nulla az ´aramer˝ oss´eg, akkor a szupravezet˝ o gy˝ ur˝ uben sem folyik ´aram, ´ıgy a rajta ´athalad´ o m´agneses fluxus ´ert´eke nulla. Ez az ´ert´ek akkor sem v´ altozhat meg, ha a tekercsben ´aram folyik. Hogyan lehets´eges ez, hiszen a szolenoid m´ agneses tere ´ miatt meg kellene jelennie egy v´eges fluxusnak a gy˝ ur˝ uben. Ugy, hogy a gy˝ ur˝ uben olyan ´aram induk´al´odik, mely azonos nagys´ ag´ u, de ellent´etes el˝ ojel˝ u fluxust hoz l´etre a gy˝ ur˝ un. Ennek az ´ aramnak a hat´ as´ ara a tekercsen a´thalad´ o m´agneses fluxus ´ert´eke ´es ´ıgy a tekercs induktivit´ asa is kisebb lesz, mint a szupravezet˝ o gy˝ ur˝ u n´elk¨ uli esetben. b) Vizsg´aljuk az el˝obb le´ırt jelens´eget kvantitat´ıvan. Legyen a szolenoid a´rama I. A szolenoid k¨ ozep´en elhelyezett szupravezet˝o gy˝ ur˝ un a´thalad´ o m´ agneses fluxus ´ert´eke Φgy˝ur˝u = L · i + M · I,

ahol L a gy˝ ur˝ u¨ oninduktivit´asa, i a gy˝ ur˝ u´ arama, M a szolenoid ´es a gy˝ ur˝ u k¨ olcs¨ on¨ os indukci´ os egy¨ utthat´ oja, ami megadja, hogy az egyikben foly´o egys´egnyi er˝ oss´eg˝ u ´aram hat´as´ara mekkora m´ agneses fluxus j¨ on l´etre a m´asikban. (Bel´ athat´ o, hogy M nagys´ aga a szerepl˝ok felcser´el´esekor nem v´altozik, teh´ at mindegy, hogy a gy˝ ur˝ u ´arama a´ltal a szolenoidban keltett m´agneses fluxust sz´ am´ıtjuk ki, vagy a szolenoid ´arama a´ltal a gy˝ ur˝ uben keltett fluxust vizsg´ aljuk. Ez ut´ obbi nyilv´ an k¨ onnyebb feladat.) M ´ert´ek´et a feladatban megadott geometri´ ara k¨ onnyen kisz´ amolhatjuk. ovekAz I er˝ oss´eg˝ u´ arammal ´atj´ art szolenoidban a homog´en m´ agneses t´er indukci´ μ NI tor´ anak nagys´ aga 0 . Mivel a gy˝ ur˝ u s´ıkja mer˝ oleges a m´ agneses t´er ir´any´ ara, a gy˝ ur˝ un ´athalad´ o m´agneses fluxus dukci´ os egy¨ utthat´ o ´ert´ek´et:

μ0 N I 2 r π. 

M=

Innen kiolvashatjuk a k¨ olcs¨ on¨ os in-

μ0 N 2 r π. 

A gy˝ ur˝ u fluxusa nem v´ altozik meg, ha a szolenoid a´ram´ at null´ar´ ol I-re n¨ ovelj¨ uk, ´ıgy Φgy˝ur˝u = 0, ahonnan a gy˝ ur˝ uben foly´ o´ aram ´ert´eke i=−

MI . L

A szolenoidon ´athalad´ o m´ agneses fluxus ´ert´eke: Φszolenoid = L0 · I + M · i, ahol L0 a szolenoid ¨ oninduktivit´asa. Behelyettes´ıtve a gy˝ ur˝ u´ aram´ at, a k¨ ovetkez˝ ot kapjuk:   M2 Φszolenoid = L0 − I. L

L´athatjuk, hogy a szolenoidon a´thalad´ o m´ agneses fluxus ar´ anyos a szolenoid a´ram´ aval. Az ar´ anyoss´ agi t´enyez˝ o a szupravezet˝ o gy˝ ur˝ ut tartalmaz´ o szolenoid induktivit´ asa, mely μ2 N 2 r 4 π 2 M2 = 0 2 ΔL0 ≡ L  L ´ert´ekkel kisebb, mint a gy˝ ur˝ u n´elk¨ uli szolenoid ¨ oninduktivit´asa. 108

K¨ oz´ episkolai Matematikai ´ es Fizikai Lapok, 2022/2

Ugyanezt az eredm´enyt kaptuk volna, ha a sz´amol´as sor´ an nem t´etelezz¨ uk fel, hogy kezdetben a szupravezet˝o gy˝ ur˝ uben nulla ´aram folyik. A le´ırt levezet´es kis m´ odos´ıt´ assal haszn´ alhat´ o a szupravezet˝ o tetsz˝ oleges el˝ o´elete eset´en is. Ekkor i, altozott meg a gy˝ ur˝ u ´arama, Φgy˝ur˝u ´es Φszolenoid azt adja meg, hogy mennyivel v´ valamint a gy˝ ur˝ un ´es a szolenoidon ´ athalad´ o m´ agneses fluxus ´ert´eke, mik¨ozben a tekercs a´ram´ at null´ar´ ol I-re n¨ ovelt¨ uk. 3. feladat. Egy felf´ ujhat´ o strandlabda k¨ onny˝ u, v´ekony, igen hajl´ekony, de nem ny´ ujthat´ o m˝ uanyagb´ ol k´esz¨ ult. Felf´ ujt ´ allapot´ aban a labda majdnem pontosan g¨ omb alak´ u, sugara 20 cm. Egy k´ıs´erletben a labd´ at u ˝rtartalm´ anak fel´eig felf´ ujjuk leveg˝ ovel, majd egy v´ızszintesen tartott, nagy kiterjed´es˝ u s´ıklap seg´ıts´eg´evel fokozatosan v´ız al´ a nyomjuk, m´ıg az teljesen el nem mer¨ ul a v´ızben. V´ azoljuk fel, milyen alakot vesz fel a v´ız al´ a nyomott labda! Ha tudjuk, hat´ arozzuk meg az alak relev´ ans m´ereteinek sz´ amszer˝ u ´ert´ekeit is!

3. ´ abra

A feladat m´asodik r´esz´eben meg kell hat´ aroznunk a g¨ omb¨ ov m´ereteit. A jel¨ ol´esek a 4. ´ abr´ an l´ athat´ ok. Vizsg´aljuk el˝osz¨ or a geometriai felt´etelt: a g¨ omb¨ ov t´erfogata a g¨ omb t´erfogat´ anak fele. (A g¨ omb¨ ov t´erfogata k´epletgy˝ ujtem´enyekb˝ ol kikereshet˝o, vagy integr´ al´assal k¨ onnyen kisz´ am´ıthat´ o.)

(Vigh M´ at´e ) Megold´as. A feladat sz¨ ovege szerint a labda anyaga igen hajl´ekony, de nem ” ny´ ujthat´ o”. Ez´ert az egyetlen lehets´eges m´ odszer a labda t´erfogat´ anak cs¨okkent´es´ere, ha a labd´at behorpasztjuk” (els˝ o rajz a 2. ´ abr´ an), ekkor a fel¨ ulet k´et (ugyan” olyan r sugar´ u) g¨ ombfel¨ uletdarabb´ ol ´ all. A behorpadt g¨ombfel¨ uleten azonban u ´jabb horpad´as is lehets´eges – ez´ uttal kifele –, ahogy az ´abra m´ asodik rajz´ an l´atszik. Ezt tetsz˝ oleges sz´ amban megism´etelhetj¨ uk, ´ıgy ak´ ar k¨ ozel s´ıklapot is kialak´ıthatunk, amely azonban a val´os´agban egy kicsit r´ ancos”, v´ekony, ki-behajl´o g¨ombfelsz´ında” rabokb´ ol ´ all (k¨ oz´eps˝ o rajz ). L´atni fogjuk, hogy fizikai felt´etelek miatt a labda als´ o ´es fels˝ o r´esze is ´ıgy fog deform´ al´odni (negyedik rajz ). A r´ ancokat” (amelyek elvileg tetsz˝ olegesen fino” atszanak) m´ ar nem ´abr´ azolva egy mak lehetnek, de egy val´odi k´ıs´erletben az´ert l´ g¨ omb¨ ovet kapunk (utols´ o rajz a 2. ´ abr´ an).

4. a ´bra

πr2 (h1 + h2 ) −

π 3 2π 3 (h1 + h32 ) = r . 3 3

otlan A numerikus megold´ ashoz ´erdemes bevezetni az x1 = hr1 ´es x2 = hr2 dimenzi´ v´altoz´ okat, ´ıgy ´attekinthet˝ obb´e v´ alik az egyenlet. (3)

x31 + x32 − 3(x1 + x2 ) + 2 = 0.

Ez egy k´etismeretlenes (harmadfok´ u) egyenlet. A m´ asik egyenletet a fizikai felt´etel matematikai megfogalmaz´ as´ aval kapjuk meg. Erre k´et lehets´eges utat mutatunk meg. I. megold´as. Az er˝oegyens´ uly alapj´an: a lapra kifejtett nyom´oer˝ o megegyezik a labd´ara hat´ o felhajt´ oer˝ ovel.

2. ´ abra

Eddig csak a geometria a´ltal lehets´eges deform´ aci´ okr´ ol besz´elt¨ unk. Ezut´an meg kell vizsg´ alnunk, hogy az adott k´ıs´erletben a fizikai felt´etelek k¨ovetkezt´eben milyen alak j¨ on l´etre. A labda tetej´et a s´ıklap nyomja le a v´ız al´ a, ´ıgy ott a labda ´ r´ asimul a fel¨ uletre. Erdekesebb k´erd´es a labda alj´ anak alakja: mivel a labda igen ” hajl´ekony”, a gy˝ urt fel¨ uleten olyan alakot vesz fel, hogy a bels˝ o ´es a k¨ uls˝ o nyom´as mindenhol azonos legyen. A labd´an bel¨ ul mindenhol azonos a l´egnyom´ as (a leveg˝o csek´ely aerosztatikus nyom´ as´ at elhanyagoljuk), a v´ız nyom´asa viszont a m´elys´eggel anak is v´ızszintes s´ıklapnak kell lennie v´ altozik (p = p0 + gh), ´ıgy a labda alj´ (3. ´ abra). A labda alakja teh´at egy v´ızszintes s´ıklapokkal hat´ arolt g¨ omb¨ ov. K¨ oz´ episkolai Matematikai ´ es Fizikai Lapok, 2022/2

109

2r3 π g, 3 ahol p a labd´ aban l´ev˝ o nyom´ as, p0 a k¨ uls˝ o l´egnyom´ as, r1 a g¨ omb¨ ov fels˝o lapj´anak sugara,  pedig a v´ız s˝ ur˝ us´ege. Ahogy a 3. ´ abr´ an is l´athat´ o, a labda belsej´eben a leveg˝ o nyom´ asa a k¨ uls˝ o l´egnyom´as ´es a h magass´ ag´ u v´ızoszlop hidrosztatikai nyom´ as´ anak ¨ osszeg´evel egyenl˝ o: (p − p0 )r12 π =

p = p0 + gh = p0 + g(h1 + h2 ). Ezt be´ırva az el˝oz˝ o egyenletbe, ´es kihaszn´ alva, hogy r12 = r2 − h21 , megkapjuk a fizikai felt´etelt:   2r3 π g(h1 + h2 ) r2 − h21 π = g, 3 110

K¨ oz´ episkolai Matematikai ´ es Fizikai Lapok, 2022/2

amelyet a kor´ abban bevezetett dimenzi´ otlan v´ altoz´ okkal ism´et ´attekinthet˝ obb alakra hozhatunk:

A t¨ omegk¨ oz´eppont minim´ alis t´ avols´ aga a lapt´ ol dmin = rδmin = 6,9 cm.

 2  (x1 + x2 ) 1 − x21 = . 3

Megjegyz´es. T¨ obb versenyz˝ o is ´eszrevette, hogy a feladat ekvivalens azzal, hogy a labd´ at f´elig megt¨ oltj¨ uk v´ızzel, ´es egy sima, v´ızszintes fel¨ uletre helyezz¨ uk. Ilyenkor ´ertelemszer˝ uen a v´ız t¨ omegk¨ oz´eppontj´ anak a lehet˝ o legalacsonyabban kell lennie.

(4)

Ezut´ an a k´etismeretlenes (3)–(4) egyenletrendszert kell megoldanunk. Az egyenletrendszert legegyszer˝ ubb numerikusan, pr´ ob´algat´ assal” megoldani. ” x1 ´es x2 ´ert´eke 0 ´es 1 k¨ oz¨ ott lehet, ´ert´ek¨ uket durv´an megbecs¨ ulve behelyettes´ıthetj¨ uk az egyenletekbe, majd az ´ert´ekeket u ´gy finom´ıtjuk, hogy az egyenletek min´el ink´ abb teljes¨ uljenek. Az egyenletrendszer megold´ asa (itt 3 ´ert´ekes jegyre, de term´eszetesen a versenyen kev´esb´e pontos megold´ as is el´eg lett volna) ´es az ¨osszenyomott labda 4. ´abr´ an l´athat´ o geometriai param´eterei: x1 = 0,235, h1 = 4,7 cm,

h2 = 9,4 cm,

Az esem´eny v´eg´en ker¨ ult sor az eredm´enyhirdet´esre. A d´ıjakat Ormos P´ al, az E¨otv¨os Lor´ and Fizikai T´arsulat eln¨ oke adta a´t.

r2 = 17,6 cm.

II. megold´as. Energetikai megfontol´as alapj´ an: a kiszor´ıtott v´ız t¨omegk¨oz´eppontja a lehet˝ o legmagasabban legyen. A g¨ omb¨ ov t¨ omegk¨ oz´eppontj´anak t´ avols´ aga a lapt´ ol (a g¨omb¨ov t¨omegk¨oz´eppontj´ anak helye k´epletgy˝ ujtem´enyekb˝ ol kikereshet˝o, vagy integr´ al´assal k¨onnyen meghat´arozhat´ o):     3 h22 − h21 3 h42 − h41 d= − , 4r 8r3 a kor´ abbi m´ odon dimenzi´otlan´ıtva     3 x22 − x21 3 x42 − x41 d − . δ= = r 4 8 Ezut´ an δ minimum´at keress¨ uk, figyelembe v´eve a kor´ abban fel´ırt x31 + x32 − 3(x1 + x2 ) + 2 = 0 geometriai felt´etelt is. Legegyszer˝ ubben ism´et pr´ ob´ algat´ assal” oldhatjuk meg a feladatot. Eszerint ” δmin = 0,343,

ha x1 = 0,235 ´es x2 = 0,470,

az el˝oz˝ o megold´ assal ¨ osszhangban. K¨ oz´ episkolai Matematikai ´ es Fizikai Lapok, 2022/2

Az u asra 2021. november 26-´ an d´el¨nnep´elyes eredm´enyhirdet´esre ´es d´ıjkioszt´ ut´ an ker¨ ult sor az ELTE TTK E¨ otv¨ os-term´eben. Megh´ıv´ ast kaptak az 50 ´es 25 ´evvel ezel˝ otti E¨otv¨os-verseny nyertesei is. A 25 ´evvel ezel˝ otti d´ıjazottak k¨ oz¨ ul T´ oth G´ abor Zsolt j¨ott el – o˝ p´ ar mondatban besz´elt a p´ alyafut´ as´ ar´ ol. Ezut´ an k¨ ovetkezett a 2021. ´evi verseny feladatainak ´es megold´ asainak bemutat´ asa. Az 1. feladat megold´as´ at Gn¨ adig P´eter, a 2. feladat´et Sz´echenyi G´ abor, a 3. feladat´et Vank´ o P´eter ismertette.

x2 = 0,470,

h = h1 + h2 = 14,1 cm, r1 = 19,4 cm,

 5. ´ abra

111

Egyetlen versenyz˝ o sem oldotta meg mindh´ arom feladatot, ´ıgy a versenybizotts´ ag nem adott ki els˝ o d´ıjat. Az els˝o feladat helyes megold´ as´ a´ert, valamint a m´ asodik ´es harmadik feladat´ ban el´ert l´enyeges eredm´enyek´ert m´ asodik d´ıjat nyert T´ oth Abel, az ELTE fizika ´ BSc szakos hallgat´ oja, aki a Budapesti Fazekas Mih´aly Gyakorl´ o Altal´ anos Iskola ´es Gimn´ aziumban ´eretts´egizett Schramek Anik´ o tan´ıtv´ anyak´ent. Az els˝o feladat helyes, vagy l´enyeg´eben helyes megold´ as´ a´ert, valamint a m´asodik vagy a harmadik feladatban el´ert l´enyeges eredm´enyek´ert harmadik d´ıjat nyert Kert´esz Bal´azs Zolt´an, a Debreceni Reform´atus Koll´egium D´ oczy Gimn´azium´anak 12. oszt´ alyos tanul´oja, T´ ofalusi P´eter tan´ıtv´ anya; Sz´epv¨ olgyi Gergely, a B´ek´ asmegyeri Veres P´eter Gimn´ azium 12. oszt´alyos tanul´oja, Sz´ekely Gy¨ orgy ´es Rakovszky anya, valamint Tak´acs Bendeg´ uz, a Budapesti Fazekas Mih´aly GyaAndor´ as tan´ıtv´ ´ korl´ o Altal´ anos Iskola ´es Gimn´ azium 12. oszt´alyos tanul´oja, Nagy Piroska M´ aria ´es Csefk´ o Zolt´ an tan´ıtv´ anya. Az els˝o feladat helyes, vagy l´enyeg´eben helyes megold´ as´ a´ert, valamint a m´asodik feladatban el´ert r´eszeredm´enyek´ert dics´eretet kapott Bonifert Bal´azs, az ELTE ´ fizika BSc szakos hallgat´ oja, aki a Ba´ar-Madas Reform´atus Gimn´ azium, Altal´ anos Iskola ´es Di´akotthonban ´eretts´egizett Horv´ ath Norbert tan´ıtv´ anyak´ent; Csord´as Kevin, a Bajai III. B´ela Gimn´ azium 12. oszt´alyos tanul´oja, Lakner Attila ´es P´ alfalvi L´ aszl´ o tan´ıtv´ anya; D´ek´any Csaba, a gy˝ ori R´evai Mikl´ os Gimn´azium ´es Koll´egium anya; Fonyi M´at´e S´andor, a BME fizika 12. oszt´ alyos tanul´oja, Juh´ asz Zolt´ an tan´ıtv´ BSc szakos hallgat´ oja, aki a szolnoki Verseghy Ferenc Gimn´aziumban ´eretts´egizett Veres D´enes tan´ıtv´ anyak´ent; Gurz´ o J´ozsef, a Budapesti Fazekas Mih´ aly Gyakorl´ o ´ Altal´ anos Iskola ´es Gimn´ azium 12. oszt´alyos tanul´oja, Nagy Piroska M´ aria tan´ıt´ v´anya, valamint Toronyi Andr´as, a Ba´ ar-Madas Reform´atus Gimn´ azium, Altal´ anos Iskola ´es Di´akotthon 12. oszt´alyos tanul´oja, Horv´ ath Norbert tan´ıtv´ anya. 112

K¨ oz´ episkolai Matematikai ´ es Fizikai Lapok, 2022/2

A m´ asodik d´ıjjal Zim´ anyi Gergely adom´any´ ab´ol 60 ezer, a harmadik d´ıjjal 40 ezer, a dics´erettel 25 ezer forint p´enzjutalom j´art. A d´ıjazottak tan´ arai a Hazal´ atogatott Wigner Jen˝ o, illetve az Az E¨ otv¨ os k´ıs´erlet t¨ ort´enelmi keretben c´ım˝ u k¨onyveket kapt´ ak az E¨ otv¨ os Lor´ and Fizikai T´arsulat aj´and´ekak´ent. K¨osz¨onj¨ uk az adom´ anyoz´ ok ¨ onzetlen t´ amogat´ as´ at! Gn¨adig P´eter, Sz´echenyi G´abor, Vank´o P´eter, Vigh M´at´e

Fizika gyakorlat megold´asa

G. 759. Egy v´ızszintes, s´ url´ od´ asmentes, r¨ ogz´ıtett p´ alc´ ara felf˝ uzve n´egy darab m t¨ omeg˝ u, n´egy darab M t¨ omeg˝ u (m < M ), majd ism´et egy m t¨ omeg˝ u, t¨ ok´eletesen rugalmas goly´ o a ´ll k¨ ozel egym´ ashoz az ´ abr´ an l´ athat´ o elrendez´esben. Balr´ ol egy m t¨ omeg˝ u, szint´en t¨ ok´eletesen rugalmas goly´ o ´erkezik v sebess´eggel, ´es u ozik ¨tk¨ a goly´ osor els˝ o tagj´ aval.

1. Amikor az ´erkez˝o goly´ o vele azonos t¨ omeg˝ u goly´ oval u ozik, vagyis ¨tk¨ at ha k´et azonos t¨ omeg˝ u goly´ ou ozik, azok m1 = m2 , akkor v1 = 0 ´es v2 = v. Teh´ ¨tk¨ sebess´eget cser´elnek. A folyamat els˝ o4u oz´ese ilyen lesz. ¨tk¨ 2. Ha a kisebb (m) t¨ omeg˝ u goly´ ou ozik egy nagyobb (M ) t¨ omeg˝ uvel, akkor ¨tk¨ at a kisebb t¨ omeg˝ u goly´ o balra fog visszapat(v > 0 eset´en) v1 negat´ıv lesz, teh´ tanni, a nagyobb t¨ omeg˝ u goly´ o pedig az u oz´es ut´ an jobbra halad, hiszen annak ¨tk¨ pozit´ıv lesz a sebess´ege. A balra halad´ o kis goly´ o (az 1. esetnek megfelel˝oen) sorra sebess´eget cser´el a t˝ ole balra l´ev˝ o goly´ okkal, ez´ert a bal oldali legsz´els˝ o goly´ o v´eg¨ ul balra fog mozogni, a t¨ obbi kis goly´ o pedig meg´ all. Ha k´et, egyenk´ent M t¨ omeg˝ u goly´ ou ¨tk¨ozik, akkor azok is csak sebess´eget cser´elnek (1. eset). 3. Amikor a jobb oldali utols´ o nagy goly´ ou ozik a sor v´eg´en ´ all´ o kis goly´ oval, ¨tk¨ at m1 > m2 . Ebben az esetben v1 is ´es v2 is pozit´ıv akkor m1 = M ´es m2 = m, teh´ lesz, azaz mindk´et goly´ o jobbra fog mozogni az u oz´es ut´an, ´es term´eszetesen ¨tk¨ ul. v1 < v2 is teljes¨ Az u an a bal oldali legsz´els˝ o (m t¨ omeg˝ u) goly´ o balra, a sor ¨tk¨oz´essorozat ut´ jobb oldali sz´el´en l´ev˝ o (M ´es m t¨ omeg˝ u) k´et goly´ o jobbra fog mozogni, a t¨ obbi goly´ o pedig egyhelyben marad. Beke Botond (Budapest, B´ek´ asmegyeri Veres P´eter Gimn., 9. ´evf.) 34 dolgozat ´erkezett. Helyes 16 megold´ as. Kicsit hi´ anyos (3 pont) 3, hi´ anyos (1 pont) 7, hib´ as 7, nem versenyszer˝ u 1 dolgozat.

A tov´ abbi u oz´esek lezajl´ asa ut´ an mely goly´ ok maradnak nyugalomban, ´es ¨tk¨ a t¨ obbiek milyen ir´ anyban fognak mozogni? (4 pont)

Fizika feladatok megold´asa

Megold´as. A megold´ asban a p´ alca menti sebess´egeket el˝ojelesen ´ertj¨ uk, ´es a jobbra halad´ o testek sebess´eg´et tekintj¨ uk pozit´ıvnak. Minden rugalmas u ¨tk¨oz´esn´el teljes¨ ul a lend¨ uletmegmarad´ as ´es az energiamegmarad´ as t¨orv´enye. Ezek szerint omeg˝ u test v sebess´eggel u ozik egy m2 t¨omeg˝ u ´all´o testtel, akkor ha egy m1 t¨ ¨tk¨ all, hogy az u oz´es ut´ ani v1 ´es v2 sebess´egekre fenn´ ¨tk¨ m1 v = m1 v1 + m2 v2 ,

valamint

P. 5338. A bal oldali ´ abr´ an l´ athat´ o m´ odon egy domin´ op´ art helyez¨ unk el egy harmadikon. a) Hat´ arozzuk meg x lehets´eges ´ert´ekeit, hogy a domin´ ok egyens´ ulyban legyenek. b) Ezt k¨ ovet˝ oen tov´ abbi domin´ op´ arokat helyez¨ unk el a jobb oldali a´br´ anak megfelel˝ oen. Legfeljebb h´ any domin´ ot helyezhet¨ unk el a legals´ ora, hogy az egyens´ ulyi ´ allapot fennmaradjon?

m1 v 2 = m1 v12 + m2 v22 .

Ennek az egyenletrendszernek 2 megold´asa van: v1 = v, valamint v1 =

m1 − m 2 v, m1 + m 2

v2 = 0,

v2 =

(5 pont)

2m1 v. m1 + m 2

Az els˝o megold´ as fizikailag elfogadhatatlan (ekkor az els˝o goly´ o u ul ¨tk¨oz´es n´elk¨ menne ´ at a m´ asodikon), ez´ert a m´asodik adja meg helyesen az u ani ¨tk¨oz´es ut´ sebess´egeket. Eset¨ unkben h´ aromf´ele u oz´es k´epzelhet˝ o el, ´es mindegyik meg is t¨ort´enik. ¨tk¨ K¨ oz´ episkolai Matematikai ´ es Fizikai Lapok, 2022/2

113

K¨ ozli: Simon P´eter, P´ecs

Megold´as. a) Az al´ at´ amaszt´ as bal oldal´an kil´og´ o r´esz x hossz´ at u ´gy kell megv´alasztanunk, hogy a domin´ op´ar t¨ omegk¨ oz´eppontja a f¨ ugg˝oleges tart´ ohas´ ab” f¨ ol´e ” omegk¨ oz´eppontj´anak elegend˝o a v´ızszintes” xS koessen. A k´et fels˝ o domin´ o S t¨ ” ordin´ at´ aj´at kisz´am´ıtanunk, hiszen a stabilit´ as felt´etel´eben csak ez szerepel. A domin´ okat egyforma, homog´en t¨ omegeloszl´ as´ u has´ aboknak tekintj¨ uk. Az 1. ´ abr´ ar´ ol 114

K¨ oz´ episkolai Matematikai ´ es Fizikai Lapok, 2022/2

leolvashat´ o, hogy xA = 52 = 26, xB = 10 = 5 ´es 2 2 xS =

xA + xB = 15,5 mm. 2

A t´ avols´ agokat millim´eter egys´egekben m´erj¨ uk, ´es ezt a dimenzi´ ot a tov´ abbiakban nem jelezz¨ uk.

a 3, 4, 5, 6, . . . , n − 1, n jelz´es˝ u (halv´ anysz¨ urke) domin´ okb´ ol ´ all´ o ´ep´ıtm´eny lebillen, elfordul az 1 jel˝ u domin´ o jobb fels˝ o ´ele k¨ or¨ ul. Ez mindaddig nem k¨ ovetkezik be, at´ aja nem haladja meg ameddig az ´ep´ıtm´eny t¨ omegk¨ oz´eppontj´anak xT koordin´ az al´at´ amaszt´ o fel¨ ulet 42 millim´eteres hossz´at. (Az x koordin´ at´ akat most a 4-es jel˝ u domin´ o bal oldali lapj´ at´ ol m´erj¨ uk.) A 3–4 domin´op´ ar t¨ omegk¨ oz´eppontja – mint l´attuk – az xP = 15,5 adattal adhat´o meg. A tov´ abbi domin´ op´ arok t¨ omegk¨ oz´eppontja v´ızszintesen 10-10 millim´eterrel tol´ odik el jobbra, teh´at az (n − 1)-edik ´es az n-edik domin´ ora xQ = xP + (k − 1) · 10 = 5,5 + 10 k, ahol k = n−2 a halv´ anysz¨ urke domin´ op´ arok sz´ama. A stabilit´as szempontj´ab´ ol 2 fontos T t¨omegk¨ oz´eppontra fenn´ all, hogy xT =

1. a ´bra

2. a ´bra

A 2. ´ abr´ an l´ atszik, hogy xmin = xS − 10 = 5,5 ´es xmax = xS = 15,5. A domin´ op´ar egyens´ uly´ anak felt´etele ezek szerint: 5,5  x  15,5.

xP + xQ = 10,5 + 5k = 5,5 + 2,5 n. 2

Az xT  42 felt´etel akkor teljes¨ ul, ha n  14,6, vagyis a term´eszetes sz´ amok k¨ or´eben n  14. A v´ızszintes asztalon teh´ at legfeljebb 14 domin´ ot (7 p´ arat) helyezhet¨ unk el u ´gy, hogy stabil egyens´ ulyban legyenek. (Amennyiben m´eg egy tov´ abbi v´ızszintes domin´ ot rakunk fel, a t¨ omegk¨ oz´eppont 2 mm-rel t´ ulny´ ulik a megengedett hat´aron. K¨onnyen bel´ athat´ o, hogy ha az ´ep´ıtm´eny nem billen le a legals´ o domin´or´ ol, akkor m´ashol sem billenhet le.)

(A k´et sz´els˝ o helyzet labilis (instabil) egyens´ ulynak felel meg.)

Vajon r´ a´ all´ıthat´ o-e (r´ aemelhet˝ o-e) ez a 14 domin´ o egyetlen, a legkisebb lapj´an ´all´ o (0-val jel¨ olt) domin´ ora? Ez akkor tehet˝ o meg, ha a 14 domin´ob´ ol ´ all´ o rendszer R t¨ omegk¨ oz´eppontja nincs messzebb az 1-es jel˝ u domin´ o bal oldali lapj´ at´ ol, mint a domin´ o 52 millim´eteres hossza. A fenti k´epletekb˝ ol kisz´am´ıthatjuk, hogy abr´ an l´athat´ o´ allapot xR = 45,5 < 52, teh´at a 4. ´ megval´os´ıthat´ o. Megjegyz´es. Amennyiben a 14 domin´ ot nem az asztalon rakjuk egym´ asra, hanem a 0-val jel¨ olt elemre egyes´evel pakoljuk r´ a a t¨ obbi domin´ ot, akkor az ´ep´ıtkez´es k¨ ozben – ´ atmenetileg – stabiliz´ alnunk kell az ingatag szerkezetet. Ezt p´eld´ aul a 4. ´ abr´ an l´ athat´ o, szaggatott vonallal jel¨ olt domin´ oval oldhatjuk meg. Az ´ allv´ anyt” az ´ep´ıtm´eny elk´esz¨ ulte ut´ an ” ovatosan elt´ ´ avol´ıthatjuk. 4. ´ abra

3. ´ abra

b) Helyezz¨ uk el el˝ osz¨ or a domin´op´ arokat a v´ızszintes asztallapra a 3. a ´br´ an l´athat´ o m´ odon. Ha a domin´ ok n sz´ ama meghalad egy nmax ´ert´eket, akkor K¨ oz´ episkolai Matematikai ´ es Fizikai Lapok, 2022/2

115

Szab´ o M´ arton (Szeghalom, P´eter Andr´ as Gimn. ´es Koll´egium, 11. ´evf.) dolgozata alapj´an

55 dolgozat ´erkezett. Helyes Szab´ o M´ arton megold´ asa. Kicsit hi´ anyos (4 pont) 14, hi´ anyos (1–3 pont) 34, hib´ as 6 dolgozat.

116

K¨ oz´ episkolai Matematikai ´ es Fizikai Lapok, 2022/2

P. 5354. Motoros j´ at´ekvonat halad R sugar´ u, k¨ or alak´ u p´ aly´ an, ´ alland´ o nagys´ aaj´ u g´ u v sebess´eggel. A k¨ or k¨ oz´eppontj´ at´ ol d < R t´ avols´ agra egy ´ alland´ o, f0 frekvenci´ hangot kibocs´ at´ o, pontszer˝ u hangforr´ as helyezkedik el. A vonatra egy mikrofont r¨ ogz´ıt¨ unk. Milyen hat´ arok k¨ oz¨ ott v´ altozik a mikrofon a ´ltal ´eszlelt hang frekvenci´ aja? (A hang sebess´ege c.) (6 pont)

K¨ozli: Vigh M´ at´e, Biatorb´ agy

I. megold´as. Legyen a vonat k¨ or alak´ u p´ aly´ aj´anak k¨oz´eppontja O, a r¨ogz´ıtett hangforr´ as helye F , a vonat pillanatnyi helye pedig a k¨orp´ alya P pontja (l´ asd ´ o, f0 frekvenci´aj´ az 1. ´ abr´ at). All´ u hangot kibocs´at´ o hangforr´ as hangj´ at egy mozg´o megfigyel˝o (eset¨ unkben a mikrofon) a Doppler-effektus szerint  u f = f0 1 + c

frekvenci´aj´ unak ´eszleli, ahol u az ´eszlel˝ o sebess´eg´enek a hangforr´ as ir´ any´ aba mutat´ o komponense. A hangforr´ as hely´et pl. az a´br´ an l´athat´ o β sz¨oggel adhatjuk meg. Bontsuk fel a mikrofon v nagys´ ag´ u sebess´egvektor´ at egy P F -fel p´ arhuzamos ´es egy arra mer˝ oleges komponensre. A p´ arhuzamos ¨ osszetev˝ ot jel¨olj¨ uk u-val. A hangforr´ as ´es a mikrofon t´ avols´ aga u = v sin α sebess´eggel cs¨ okken, ahol α az OP F sz¨og. A legnagyobb ´eszlelt frekvencia u legnagyobb ´ert´ek´enek, vagyis α legnagyobb ´ert´ek´enek felel meg. Mivel az OP F h´ aromsz¨ ogre fel´ırhat´o szinuszt´etel szerint sin α =

Hasonl´o m´ odon kapjuk a hangforr´ ast´ ol t´ avolod´ o mikrofon a´ltal ´eszlelt frekvenciacs¨okken´est is. Ha t¨ ukr¨ ozz¨ uk a P pontot az OF egyenesre, az F ´es P  pontok t´ avolod´ as´ anak sebess´ege ugyanakkora lesz, mint F ´es P k¨ ozeled´es´enek sebess´ege d v, az ´eszlelt frekvencia legkisebb ´ert´eke: volt (2. a ´bra). Mivel u legnagyobb ´ert´eke R   v d fmin = f0 1 − . c R K¨ openczei Csan´ ad (Bonyh´ ad, Pet˝ ofi S. Ev. Gimn. ´es Koll., 11. ´evf.) ´es Yokota Adan (G¨ od¨ oll˝ oi T¨ or¨ ok Ign´ ac Gimn., 12. ´evf.) dolgozata alapj´an II. megold´as. A hangmagass´ ag v´ altoz´ as´ at a Doppler-effektussal magyar´azzuk. u, a leveg˝ oh¨ oz k´epest a´ll´ o hangforr´ as frekvenci´ aj´at egy, Eszerint az f0 frekvenci´aj´ a hangforr´ ashoz u sebess´eggel k¨ ozeled˝o (vagy t´ avolod´ o) mikrofon  u f = f0 1 ± c nagys´ ag´ unak r¨ ogz´ıti. A feladatunk teh´ at u legnagyobb ´ert´ek´enek meghat´ aroz´ asa.

K´et test t´ avols´ aga nem f¨ ugg att´ ol, hogy milyen koordin´ ata-rendszerben sz´ am´ıtjuk ki azt. V´alasszuk azt a koordin´ ata-rendszert, amelynek orig´ oja a k¨ orv sz¨ ogsebesp´ alya O k¨ oz´eppontj´aban van, ´es ω = R s´eggel forog az O pont k¨ or¨ ul a vonat k¨ ormozg´ as´ aval megegyez˝o ir´ anyban. A j´ at´ekvonat – ebben a forg´ o vonatkoztat´ asi rendszerben – mindig ugyanazon a helyen a´ll, a hangforr´ as pedig egy d sugar´ u k¨ orp´ a-

d sin β, R

d ez a maximum´ at sin β = 1, vagyis β = 90◦ -n´ al veszi fel. Ezek szerint (sin α)max = R , d umax = R v, teh´ at a mikrofon ´altal ´eszlelt legnagyobb frekvencia   v d fmax = f0 1 + . c R

sebess´eggel egyenletesen mozog ly´ an u = dω = vd R (3. ´ abra). A vonat ´es a hangforr´ as t´ avols´ aga akkor v´ altozik a leggyorsabban, amikor o vonat ir´any´ aba a hangforr´ as ´eppen az F1 pontban van, ahol a sebess´ege az a´ll´ any´ u. Ilyenkor mutat, vagy az F2 pontban, ahol a sebess´ege a vonattal ellent´etes ir´ F ´es P k¨ozeled´es´enek, illetve t´ avolod´ as´ anak sebess´ege u, a megv´ altozott frekvencia legnagyobb ´es legkisebb ´ert´eke teh´ at     vd vd fmax = f0 1 + , illetve fmin = f0 1 − . Rc Rc 3. ´ abra

Megjegyz´es. A forg´ o koordin´ ata-rendszerben a hangforr´ as mozog, az ´eszlel˝ o (mikroc fon) pedig a ´ll. Ennek ellen´ere a Doppler-effektusnak nem az f = f0 c−u k´eplet´et alkalc+u

osszef¨ ugg´essel sz´ amoltunk. Ezt maztuk, hanem az ´ all´ o hangforr´ asra vonatkoz´ o f = f0 c ¨ az´ert tehett¨ uk meg, mert a forg´ o vonatkoztat´ asi rendszerben a leveg˝ o is mozog (forog), ´es a Doppler-effektusn´ al mindig a k¨ ozeghez viszony´ıtott sebess´egek sz´ am´ıtanak. (G. P.)

1. a ´bra K¨ oz´ episkolai Matematikai ´ es Fizikai Lapok, 2022/2

28 dolgozat ´erkezett. Helyes 14 megold´ as. Kicsit hi´ anyos (4–5 pont) 9, hi´ anyos (1–3 pont) 3, hib´ as 1, nem versenyszer˝ u 1 dolgozat.

2. a ´bra

117

118

K¨ oz´ episkolai Matematikai ´ es Fizikai Lapok, 2022/2

P. 5356. V´ızszintes talajon fekszik egy t´eglalap keresztmetszet˝ u gerenda. A t´eglalap v´ızszintes oldala L, f¨ ugg˝ oleges oldala H hossz´ us´ ag´ u. Elhanyagolva a k¨ ozegellen´ all´ ast, honnan ´es hogyan kell elugrania egy sz¨ ocsk´enek, hogy a lehet˝ o legkisebb energiar´ aford´ıt´ assal siker¨ ulj¨ on ´ atugrania ezt a gerend´ at? Hol lesz az ugr´ asi parabola f´ okuszpontja ebben az esetben? (5 pont)

Radnai Gyula (1939–2021) feladata

Megold´as. A sz¨ ocske az elugr´ asa ut´ an parabolap´aly´ an fog mozogni. Ez a parabola nyilv´ an a gerenda leghosszabb oldal´el´ere mer˝ oleges s´ıkban fekszik, elegend˝ o teh´ at ezt a s´ıkot vizsg´alni. (Ha a sz¨ ocsk´enek lenne hosszanti” ir´any´ u sebess´ege, ” akkor ennek null´ara cs¨okkent´es´evel cs¨ okkenthetn´e az energiar´ aford´ıt´ ast.) Ha a sz¨ ocske p´ alyag¨ orb´eje a gerenda fels˝ o ´eleit˝ol v´eges t´ avols´ agra haladna, anak v´ altozatlanul tart´ asa mellett kisebb kezd˝ oakkor az elugr´ as hely´enek ´es ir´any´ sebess´eg (kisebb energiar´ aford´ıt´ as) is elegend˝ o lenne (1. ´ abra). A szaggatott vonallal jel¨ olt parabolap´aly´ an´ al kedvez˝ obb a folytonos vonallal jel¨olt p´aly´ ahoz tartoz´o mozg´ as. A p´ alyag¨ orb´enek teh´ at legal´abb az egyik fels˝ o ´elet ´erintenie” kell, an” nak k¨ ozvetlen k¨ ozel´eben kell elhaladnia. Ha ugyanekkor nem ´erinten´e a gerenda m´ asik fels˝ o ´el´et, akkor (a kezd˝ osebess´eget ´es az elugr´as sz¨og´et v´ altozatlannak tartva) az elugr´ as hely´enek megv´altoztat´ as´ aval olyan parabol´ahoz juthatn´ank, amelyik a gerenda felett, att´ ol v´eges t´ avols´ agban halad, teh´at ez sem lehet a legkedvez˝ obb elrendez´es (2. ´ abra).

3. ´ abra

vagyis v02 = 2gH + v12 = 2gH +

gL . sin(2α)

Innen leolvashatjuk, hogy v0 akkor minim´alis (akkor legkisebb a sz¨ ocske energiar´ aford´ıt´asa az elugr´ askor), amikor sin (2α) = 1, vagyis α = 45◦ . Hat´ arozzuk meg a parabola f (x) = ax2 + bx + c alakban keresett egyenlet´et a 3. a´br´ an l´athat´ o koordin´ ata-rendszerben. Mivel a C = L L = ( 2 , H ) ´es a D = ( − 2 , H ) pontok rajta vannak a parabol´ an, teljes¨ ul, hogy H=a

 2 L L + b + c, 2 2

 2 L L illetve H = a − − b + c. 2 2 2

1. a ´bra

Ezekb˝ ol k¨ovetkezik, hogy b = 0 ´es c = H − a L4 , vagyis a parabola egyenlete:

2. a ´bra

Optim´alis esetben a parabola szimmetriatengelye a gerenda t´eglalap alak´ u keresztmetszet´enek f¨ ugg˝oleges k¨ oz´epvonal´ an´ al tal´alhat´ o, ´es a parabola illeszkedik a t´eglalap mindk´et fels˝ o cs´ ucs´ ara. A 3. ´ abra jel¨ ol´eseit haszn´ alva fel´ırhatjuk, hogy v1 t cos α =

L , 2

valamint

v1 sin α = gt,

ahol t azt az id˝ ot jel¨ oli, amennyi alatt a sz¨ ocske a D pontb´ ol a p´ alyag¨orbe legmagasabb (M -mel jel¨ olt) pontj´ aba ker¨ ul. Ebb˝ol a k´et egyenletb˝ ol t kik¨ usz¨ob¨ol´ese ut´ an kapjuk, hogy gL . v12 = sin(2α) ´Irjuk fel most az energiamegmarad´ as t¨ orv´eny´et a K ´es D pontok k¨oz¨otti mozg´ asra: 1 1 mv 2 = mv12 + mgH, 2 0 2 K¨ oz´ episkolai Matematikai ´ es Fizikai Lapok, 2022/2

119

  L2 f (x) = a x2 − + H. 4 L´attuk, hogy a parabola meredeks´ege a D pontban tg α = tg 45◦ = 1. A parabola ismert tulajdons´ aga szerint ez a meredeks´eg ´eppen k´etszerese a DM szel˝o meredeks´eg´enek: 1=2

−aL2 /4 , L/2

ahonnan

1 a=− , L

teh´at a p´ alyag¨ orbe egyenlete: f (x) = H +

L x2 − . 4 L

Megjegyz´es. Ugyanezt az ¨ osszef¨ ugg´est differenci´ alsz´ am´ıt´ assal is megkaphatjuk. f  (x) = 2ax, ami az xD = −L/2 helyen −2aL/2 = 1, vagyis a = −1/L.

120

K¨ oz´ episkolai Matematikai ´ es Fizikai Lapok, 2022/2

A sz¨ ocske elugr´ as´anak xK < 0 koordin´ at´ aj´at az f (xK ) = 0 felt´etelb˝ ol kapjuk meg: L2 , xK = − HL + 4

Felh´ıv´as a Kunfalvi Rezs˝o Olimpiai V´alogat´ oversenyre

vagyis az elugr´ asi hely ´es a gerenda sz´el´enek KA t´ avols´ aga d=



L L2 − . HL + 4 2

Az elugr´ as sz¨ og´et a v´ızszintes ir´any´ u sebess´eg ´ alland´ os´ag´at kifejez˝o v0 cos β = osszef¨ ugg´esb˝ ol kaphatjuk meg: = v1 cos 45◦ ¨ v1 cos β = √ = 2 v0 amit ´ıgy is kifejezhet¨ unk: tg β =





L , 2(L + 2H)

4H . 1+ L

Megjegyz´es. Ezt az ¨ osszef¨ ugg´est differenci´ alsz´ am´ıt´ assal is megkaphatjuk. Az f (x)  f¨ uggv´eny deriv´ altja az xK = − HL + (L2 /4) helyen: 

tg β = f (xK ) = −2axK =



1+

A Nemzetk¨ ozi Fizikai Di´akolimpi´an (IPhO) ´es az Eur´opai Fizikai Di´akolimpi´an (EuPhO) szerepl˝o magyar csapat tagjait minden ´evben a Kunfalvi Rezs˝ o Olimpiai V´alogat´ overseny keret´eben v´ alasztjuk ki a budapesti ´es vid´eki di´akolimpiai szakk¨ or¨okre j´ ar´ o di´akok k¨ oz¨ ul. A j´ arv´ anyhelyzet miatt a Kunfalvi-verseny els˝ o (elm´eleti) fordul´oj´at id´en is online szervezz¨ uk meg. A verseny teljesen nyitott: r´eszt vehet b´ arki, aki jelenleg k¨ oz´episkolai tanul´o. A feladatsor 2022. m´arcius 21-´en (h´etf˝on), 15:00 o´r´at´ ol lesz el´erhet˝o a http://ipho.elte.hu honlap f˝ooldal´an. A versenyre el˝ozetesen jelentkezni nem kell, el´eg a feladatsoron tal´ alhat´ o szab´ alyzatnak megfelel˝oen elk´esz´ıteni a dolgozatot, ´es annak szkennelt v´ altozat´ at legk´es˝ obb m´ arcius 21. 18:30-ig elk¨ uldeni az [email protected] c´ımre. A feladatok tematik´aja azonos az IPhO hivatalos tematik´aj´aval1 . A versenyen zsebsz´ amol´og´epen k´ıv¨ ul semmilyen seg´edeszk¨ oz sem haszn´alhat´ o (teh´ at f¨ uggv´enyt´abl´ azat, k¨onyvek, f¨ uzetek ´es internet se). Felk´esz¨ ul´eshez javasoljuk a kor´ abbi ´evek feladatsorait ´es a budapesti szakk¨ or YouTube-csatorn´ aj´an (IPhO Hungary) tal´ alhat´ o vide´okat. A versenybizotts´ag

4H . L

Fizik´ab´ ol kit˝ uz¨ott feladatok H´ atra van m´eg a legkisebb energiar´aford´ıt´ ashoz tartoz´ o parabolap´alya f´ okuszpontj´ anak meghat´aroz´ asa. A szimmetria miatt ez a pont a gerenda M P szimmetriatengely´en, vagyis az y tengelyen tal´ alhat´ o. Egy optikai anal´ogia seg´ıts´eg´evel k¨ onnyen bel´ athatjuk, hogy a f´ okuszpont ´eppen a DC szakasz felez˝opontja, vagyis a gerenda fels˝o lapj´anak P pontja. K´epzelj¨ uk el, hogy a sz¨ocske p´ alyag¨orb´eje egy parabolat¨ uk¨ ornek (forg´ asparaboloidnak) a szimmetriatengely´ere illeszked˝ o s´ıkkal val´o metszete. Ha erre a t¨ uk¨ orre az AD egyenes ment´en halad´ o f´enysug´ar esik, abb. M´ asr´eszt a t¨ uk¨or szimmetriatengely´eaz (α = 45◦ miatt) v´ızszintesen halad tov´ vel p´ arhuzamos f´enysugarak a f´ okuszpont ir´any´ aba ver˝ odnek vissza, a f´okuszpont teh´ at csakis a P pont lehet. (Enn´el a megfontol´ asn´ al hallgat´olagosan felhaszn´altuk azt a t´enyt is, hogy a parabola geometriai ´ertelemben vett f´okuszpontja ´es a parabolat¨ uk¨ or fizikai ´ertelemben vett f´okuszpontja egybeesik.) ´ Isk. ´es Gimn., 11. ´evf.) ´es G´ abriel Tam´ as (Budapesti Fazekas M. Gyak. Alt. ´ ´ ad Gimn., 10. ´evf.) Sepr˝ odi Barnab´ as Bendeg´ uz (Budapest, Obudai Arp´ dolgozata alapj´an 43 dolgozat ´erkezett. Helyes 13 megold´ as. Kicsit hi´ anyos (4 pont) 6, hi´ anyos (1–3 pont) 18, hib´ as 6 dolgozat.

K¨ oz´ episkolai Matematikai ´ es Fizikai Lapok, 2022/2

121

M. 411. M´erj¨ uk meg egy u or¨ os¨ uvegnek a szimmetriatengely´ere vonatkozta¨res s¨ tott tehetetlens´egi nyomat´ek´ at! A m´er´es pontos´ıt´ asa ´erdek´eben v´egezz¨ uk el a m´er´est legal´ abb k´etf´ele m´ odon! Hasonl´ıtsuk ¨ ossze az alkalmazott m´ odszerek pontoss´ ag´ at! (6 pont)

K¨ ozli: Gn¨ adig P´eter, V´ acduka

G. 769. V´ızszintes u ´ton egyenletesen halad´ o aut´ o fogyaszt´ asm´er˝ oje 5 liter/100 km ´ert´eket mutat. Ha ugyanez az aut´ o ugyanezen az u ´ton gyorsulva mozog, akkor a fogyaszt´asm´er˝ o 10 liter/100 km-t mutat abban a pillanatban, amikor a kocsi el´eri az egyenletes halad´as sebess´eg´et. Ha az aut´o ugyanekkora sebess´eggel egy emelked˝ on halad, akkor a fogyaszt´ asa 12 liter/100 km. Mit mutat a fogyaszt´asm´er˝ o, ha az aut´ o ugyanezen az emelked˝ on az el˝oz˝ oekben le´ırt gyorsul´ assal mozog felfel´e, ´es a sebess´ege is ´eppen megegyezik az el˝ oz˝ oekben le´ırtakkal? (3 pont) 1

122

L´ asd a https://www.ipho-new.org/statutes-syllabus weboldalt.

K¨ oz´ episkolai Matematikai ´ es Fizikai Lapok, 2022/2

G. 770. A legelterjedtebb f˝ ut´esi energiahordoz´ o a f¨oldg´az. A g´azszolg´altat´ o a k¨ ovetkez˝ o m´odszerrel hat´ arozza meg a havonta fizetend˝o d´ıjat, amit a g´ azsz´ aml´ an t¨ untet fel: az elfogyasztott g´ az mennyis´eg´et megszorozza egy u ´gynevezett korrekci´os t´enyez˝ ovel, majd kisz´am´ıtja az ´ıgy kapott g´ azmennyis´eg f˝ ut˝ o´ert´ek´et MJ m´ert´ekegys´egben, ´es v´eg¨ ul a f˝ ut˝ o´ert´ek szerint sz´ am´ıtj´ ak ki a fizetend˝ o ¨osszeget. Ehhez m´eg hozz´ aadj´ ak a havi h´ aztart´ asi alapd´ıjat is. Keress¨ unk egy nem t´ uls´ agosan r´egi g´ azsz´ aml´ at ´es villanysz´ aml´ at, ´es v´ alaszoljunk az al´abbi k´erd´esekre! (A v´ alaszban mindig a brutt´ o ´ert´ekeket t¨ untess¨ uk fel!) ore? a) Mi´ert van sz¨ uks´eg korrekci´ os t´enyez˝ b) Mennyibe ker¨ ul 1 m3 norm´ al a´llapot´ u g´az? c) Mennyibe ker¨ ul 1 MJ energia, ha g´ az-, illetve ha elektromos energia form´ aj´aban ´erkezik a lak´asunkba? (4 pont)

Tarj´ an Imre eml´ekverseny nyom´ an, Szolnok

G. 771. Az ´ abr´ an l´athat´ o kapcsol´asban a fogyaszt´ok azonos R ellen´ all´as´ uak, ´es U fesz¨ ults´eg eset´en a teljes´ıtm´eny¨ uk P . Mekkora az egyes fogyaszt´ok teljes´ıtm´enyfelv´etele a kapcsol´ ok nyitott (ny), illetve z´ art (z) a´ll´ as´ an´al? T¨ olts¨ uk ki a t´ abl´ azatot! 1. ny ny z z

2. ny z ny z

A

B

P. 5383. Mekkor´ anak kellene lennie – v´altozatlan tengelyforg´ as ´es a´tm´er˝ o mellett – a F¨old t¨ omeg´enek ahhoz, hogy Budapesten ne lehessen parabolaantenn´ aval m˝ uholdas t´ev´ead´ asokat fogni? (5 pont) K¨ ozli: Kis Tam´ as, Heves P. 5384. Egy v´ekony, homog´en, f¨ ugg˝oleges p´ alca tetej´en kicsiny goly´ o helyezkedik el. A p´ alca t¨ omeg´ehez k´epest a goly´ o t¨ omege elhanyagolhat´ o, ´es a p´ alca s´ url´od´asmentesnek tekinthet˝o asztalon a´ll. A p´alca egyszer csak eld˝ol. Mikor csap´odik a goly´ o nagyobb sebess´eggel az asztallapra, ha a p´alca tetej´ere van ragasztva, vagy ha egyszer˝ uen csak r´atett¨ uk a p´ alc´ ara, ahonnan nagyon k¨ onnyen lebillenhet? (A merev testek forg´ omozg´as´ ar´ ol r¨ ovid cikk olvashat´ o a K¨ oMaL honlapj´an.1 ) (4 pont) K¨ ozli: Honyek Gyula, Veresegyh´ az P. 5385. H´ anyadr´esz´ere cs¨ okken az ablakon kisz¨ ok˝ o h˝o´ aram, ha az egyr´eteg˝ u, du¨veg = 3 mm vastag u ol k´esz¨ ult ablakot ugyanilyen u abl´ ab´ ol k´esz¨ ult, ¨vegb˝ ¨vegt´ k´etr´eteg˝ u ablakra cser´elj¨ uk, melynek u oz¨ ott dleveg˝o = 7 mm-es leveg˝or´es ¨vegei k¨ van? A leveg˝ o ´es az u ovezet´esi t´enyez˝ oje κleveg˝o = 0,025 W/(m K) ´es κu¨veg = ¨veg h˝ = 1,2 W/(m K). (4 pont) P´eldat´ ari feladat nyom´an P. 5386. Egy α = 30◦ -os lejt´es˝ u, d = 2 m´eter hossz´ u, szigetel˝ o anyagb´ ol k´esz¨ ult v´aly´ u alj´ahoz Q = 5,55 μC t¨ olt´es˝ u kis goly´ ot r¨ ogz´ıt¨ unk. A v´ aly´ u tetej´er˝ ol m = 100 g t¨omeg˝ u, q = 10 μC t¨ olt´es˝ u kis goly´ ot enged¨ unk el. Milyen messzire jut el ez a goly´o, ha tiszt´an g¨ord¨ ul? (A mozg´asa sor´ an a goly´ o t¨ olt´ese nem v´ altozik meg.) (3 pont) K¨ ozli: Kobzos Ferenc, Duna´ ujv´ aros

C

(4 pont)

asi fesz¨ ults´eg˝ u, Rb bels˝ o ellen´ all´ as´ u telepre k¨ ul¨ onb¨ oz˝ o P. 5387. Egy U0 u ¨resj´ar´ R nagys´ ag´ u k¨ uls˝ o ellen´ all´ asokat kapcsolunk. a) Mekkora maxim´ alis hasznos” (a k¨ uls˝ o ellen´ all´ asra jut´ o) teljes´ıtm´enyt ny´ ujt” hat ez a telep? Milyen R eset´en ´erhetj¨ uk el a legnagyobb Pmax teljes´ıtm´enyt? al kisebb P hasznos teljes´ıtm´eny k´et b) Mutassuk meg, hogy b´armely, Pmax -n´ ag´ u k¨ uls˝ o ellen´ all´ as eset´en is megval´ osulhat. Mekkora k¨ ul¨onb¨oz˝ o, R1 = R2 nagys´ amtani, illetve m´ertani k¨ oz´ep´ert´eke? az R1 ´es R2 sz´ c) Mekkora a fenti k´et esetben m´erhet˝ o kapocsfesz¨ ults´egek ¨ osszege? all´ ason foly´o ´ aramok ¨ osszege? d) Mekkora az R1 , illetve R2 ellen´ e) Az energialead´ as hat´ asfok´ at a hasznos teljes´ıtm´eny ´es a telep a´ltal leadott anyadosak´ent ´ertelmezz¨ uk. Mekkora a fenti k´et eset hat´ as¨osszes teljes´ıtm´eny h´ fok´ anak ¨osszege? (4 pont) K¨ ozli: Siposs Andr´ as, Budapest

P. 5382. Egy r´egi szalagos magnetofon gyors a´ttekercsel´eskor a szed˝ oors´ ot ´alo bels˝ o a´tm´er˝ oje 5 cm, a k¨ uls˝ o ´atm´er˝ oj¨ uk land´ o fordulatsz´ammal forgatja. A k´et ors´ pedig 15 cm. A teljesen teli ors´ or´ ol a magn´oszalag a´tcs´ev´el´esi ideje 3 perc. A szalagot az u at. Ind´ıt´ast´ ol sz´am´ıtva mennyi id˝ o m´ ulva lesz a k´et ¨res ors´ora tekercselik ´ ors´ on ´eppen egyenl˝o hossz´ us´ag´ u magn´oszalag?

P. 5388. Egy 15 mW-os l´ezer λ = 632,8 nm hull´ amhossz´ u, line´ arisan polariz´alt f´enye egy 2 mm a´tm´er˝ oj˝ u k¨ ork¨ or¨ os apert´ ur´ an l´ep ki a l´ezer doboz´ ab´ ol. a) Mekkora az elektromos t´erer˝ oss´eg maxim´ alis ´ert´eke a l´ezernyal´abban? b) Mekkora az impulzusa a l´ezernyal´ab 1 m´eter hossz´ u darabj´anak? (4 pont) P´eldat´ ari feladat nyom´an

(4 pont)

K¨ozli: Zsigri Ferenc, Budapest

G. 772. A gyerekek k¨ orj´at´ekot j´atszanak a mez˝on. Szerencs´etlen m´odon a k¨or k¨ ozep´en a´ll´ o t´ arsuk dar´ azsf´eszekbe l´ep, ´es a m´erges darazsak sz´etrep¨ ulnek. A mez˝ on keleti ir´anyb´ol 4,5 m/s sebess´eg˝ u sz´el f´ uj, a gyerekek 6 m/s nagys´ ag´ u sebess´eggel sug´ arir´anyban menek¨ ulnek. A tud´ osok vizsg´alata szerint ezek a darazsak sz´elcsendben 8 m/s sebess´eggel tudnak rep¨ ulni. Becs¨ ulj¨ uk meg, hogy a gyerekek h´any sz´ azal´eka menek¨ ul meg biztosan a dar´ azscs´ıp´esekt˝ ol! A v´ alasz megad´ as´ahoz haszn´alhatunk ak´ ar vonalz´ot, k¨ orz˝ ot ´es sz¨ ogm´er˝ ot is.

(4 pont) K¨ oz´ episkolai Matematikai ´ es Fizikai Lapok, 2022/2

K¨ozli: Holics L´ aszl´ o, Budapest

1

123

124

https://www.komal.hu/cikkek/cikklista.h.shtml.

K¨ oz´ episkolai Matematikai ´ es Fizikai Lapok, 2022/2

P. 5389. Egy (pontszer˝ unek tekinthet˝ o) l´egy rep¨ ul ´alland´ o v sebess´eggel az f f´okuszt´avols´ ag´ u lencse optikai tengely´evel p´ arhuzamosan, att´ ol d t´avols´ agra. Legal´abb mekkora nagys´ ag´ u a l´egy ´es a l´egy k´ep´enek relat´ıv sebess´ege? ´ (5 pont) Esztorsz´ agi versenyfeladat nyom´ an P. 5390. Az ´ abr´ an l´ athat´ o R sugar´ u, v´ekony fal´ u, t¨ oltetlen f´emg¨ ombh´ej k¨ oz´eppontj´aban egy kicsiny, p dip´ olmomentum´ u elektromos dip´ olus helyezkedik el. Hat´arozzuk meg a g¨ ombh´ej bels˝o fel¨ ulet´en l´ev˝ o A ´es B pontokban a fel¨ uleti uls˝ o t¨ olt´ess˝ ur˝ us´eget! Adjuk meg a f´emg¨ombh´ej k¨ fel¨ ulet´en l´ev˝ o t¨ olt´ess˝ ur˝ us´eget is! ´ (Utmutat´ as: Alkalmazhatjuk a g¨ ombi t¨ uk¨ ort¨ olt´es m´ odszer´et. Hasznos lehet m´eg a dip´ olus elektromos ter´enek ismerete az u ´ n. Gaussf´ele f˝ohelyzetekben.) (6 pont) K¨ozli: Sz´ asz Kriszti´ an, Budapest

 Beku ¨ld´esi hat´arid˝o: 2022. m´arcius 15. Elektronikus munkafu ¨zet: https://www.komal.hu/munkafuzet

 MATHEMATICAL AND PHYSICAL JOURNAL FOR SECONDARY SCHOOLS (Volume 72. No. 2. February 2022) Problems in Mathematics New exercises for practice – competition K (see page 89): K. 719. Each integer on the number line is coloured either red or blue. Is it certain for all possible colourings that a) there will be two numbers of the same colour separated by a distance of 3; b) there will be two numbers of the same colour separated by a distance of 3 or 4? K. 720. Divide the area of a regular hexagon into three equal parts with two lines passing through the same vertex. K. 721. Alex made some wooden sticks of integer lengths such that no three of them could be used to form a triangle. Given that there were sticks of lengths 1 and 10 and that the longest stick was 100 units long, what is the maximum possible number of sticks that Alex may have made? K/C. 722. The arithmetic mean of two three-digit numbers equals the number obtained by writing them next to each other, separated by a decimal point. What may be the two numbers? K/C. 723. The Hungarian Handball Federation nominated 17 players for women’s handball in the Tokyo Olympic Games: 3 goalkeepers, 1 right winger, 4 right backs, 2 playmakers, 3 pivots, 2 left backs and 2 left wingers. In how many different ways may the players line up for the anthem if players of the same position must stand together? (During the anthem, players line up next to each other in a single line.) (Proposed by B. R´ oka Budapest) New exercises for practice – competition C (see page 90): Exercises up to grade 10: K/C. 722. See the text at Exercises K. K/C. 723. See the text at Exercises K. Exercises K¨ oz´ episkolai Matematikai ´ es Fizikai Lapok, 2022/2

125

for everyone: C. 1704. For which real numbers a will the minimum of the function f (x) = 4x2 − 4ax + a2 − 2a + 2 defined on the segment [0; 2] be equal to 3? (MC&IC ) C. 1705. Given that a certain quadrilateral is a kite, it is cyclic and its sides are 42 and 56 units long, what is the distance between the centres of the inscribed and circumscribed circles? (Proposed by A. Siposs, Budapest) C. 1706. Prove that in every set of 2022 positive integers there exist two numbers such that their difference or sum is divisible by 4040. (Proposed by L. S´ af´ ar, R´ ackeve) Exercises upwards of grade 11: C. 1707. In a triangle ABC (with the usual notations) b = 6, a = 2 and they enclose an angle of γ = 120◦ . Find the exact length of the interior angle bisector of angle γ. (MC&IC ) C. 1708. Solve the following equation over the set of real number pairs: log22 (x + y) + log22 (xy) + 1 = 2 log2 (x + y). (MC&IC ) New exercises – competition B (see page 91): B. 5222. Let A denote the set of even positive integers for which the sum of the digits decreases by 2 if the number is halved. Let B denote the set of positive integers for which the sum of the digits increases by 5 if the number is multiplied by 5. What is the number of elements in the set A ∩ B and in the set B \ A? (3 points) (Proposed by T. √ K´ asp´ ari, Paks) B. 5223. Define the sequence {an } as follows: a1 = −3, an+1 = 4 + an + 4 an + 4 . Determine the value of a2022 . (3 points) (Proposed by T. K´ asp´ ari, Paks) B. 5224. P is a point on side BC of a unit square ABCD, and Q is a point on side CD, such that ∠P AQ = 45◦ . For which positions of points P and Q will the sum BP + P Q + QD be minimal? (4 points) (Proposed by J. Szoldatics, Budapest) B. 5225. The inscribed circle of triangle ABC is centred at I and has a radius , the radius of the circumscribed circle is R. Prove that if AI = R, then the area of the a·R triangle ABC is 4 +  · a, where a denotes the length of the side opposite to vertex A. (4 points) (Proposed by Sz. Kocsis, Budapest) B. 5226. The length of each side of a triangle is at most 2 units. Each pair of vertices is joined with an arc of a unit circle, not longer than a semicircle. Prove that a + b > 2c /3, where a , b , c denote the lengths of the arcs. (5 points) B. 5227. Give an example of a positive integer k, along with a finite tree graph F of at least k vertices in which the degree of each vertex is at most 3, and F will fall apart into at least 2022 components if an arbitrary connected subgraph of k vertices is deleted from F . (6 points) (Based on a Monthly problem) B. 5228. A parabola intersects side AB of a triangle ABC at interior points C1 and C2 , side BC at interior points A1 and A2 , and it intersects side CA at interior points B1 and B2 . Prove that if AC1 = C2 B and BA1 = A2 C o, Budapest) then CB1 = B2 A. (5 points) (Proposed by G. Holl´  B. 5229. Let a = 0 be a real number, and let f : R → R be a function, such that f x + f (y) = f (x) + f (y) + ay for all x, y ∈ R. Prove that f is additive, that is, f (x + y) = f (x) + f (y) for all x, y ∈ R. (6 points) (Proposed by G. Stoica, Saint John, New Brunswick, Canada) New problems – competition A (see page 92): A. 818. Find all pairs of positive integers m, n such that 9|m−n| + 3|m−n| + 1 is divisible by m and n simultaneously. (Submitted by G´eza K´ os, Budapest) A. 819. Let G be an arbitrarily chosen finite simple graph. We write non-negative integers on the vertices of the graph such that for each vertex v in G the number written on v is equal to the number of vertices adjacent to v where an even number is written. Prove that the number of ways to achieve this is a power of 2. A. 820. Let ABC be an arbitrary triangle. Let the excircle tangent to side a be tangent to lines AB, BC and CA at points Ca , Aa and Ba , respectively. Similarly, let the excircle tangent to side b be tangent to lines AB, BC and CA at points Bc , Ba and Bb , respectively. Finally, let the excircle tangent to side c be tangent to lines AB, BC and CA at points Cc , Ac and Bc , respectively. Let A be the intersection of lines Ab Cb and Ac Bc . Similarly, let B  be the intersection of lines Ba Ca and Ac Bc , and let C  be the intersection of lines Ba Ca and Ab Cb . Finally, let the incircle be tangent to sides a, b and c at points Ta , Tb

126

K¨ oz´ episkolai Matematikai ´ es Fizikai Lapok, 2022/2

and Tc , respectively. a) Prove that lines A Aa , B  Bb and C  Cc are concurrent. b) Prove that lines A Ta and B  Tb and C  Tc are also concurrent, and their point of intersection is on the line defined by the orthocentre and the incentre of triangle ABC. (Submitted by Viktor Csapl´ ar, B´ atorkeszi and D´ aniel Heged˝ us, Gy¨ ongy¨ os)

Problems in Physics (see page 122) M. 411. Measure the rotational inertia of an empty beer bottle about its symmetry axis. In order to make the measurement more accurate, carry out the measurement in two different ways. Compare the accuracy of the two measurements. G. 769. The fuel consumption meter of a car, moving uniformly along a level road, reads 5 litres/100 km. If the same car is moving along the same road, and accelerates uniformly, then the reading on the fuel consumption meter at the moment when the car reaches the speed of the uniform motion is 10 litres/100 km. When the car goes at the same speed on a hill the fuel consumption is 12 litres/100 km. What does the meter read when the car is driving up the same hill at the acceleration as described before at the moment when its speed also reaches the value described above? G. 770. The most common heating fuel is natural gas. The gas utility company uses the following method to determine the monthly price to be paid, which is shown on the gas bill (in Hungarian): the amount of consumed gas is multiplied by a so-called correction factor, then the heating value of the gas in MJ is calculated, and finally the heating value is used to calculate the amount of money to be paid. To this, a basic monthly household charge is added. Find a recent gas bill and a recent electricity bill and answer the following questions (in your answers always give the gross values!) a) Why is it necessary to use the correction factor? b) What is the price of 1 m3 natural gas at standard conditions? c) How much does 1 MJ of energy cost if it comes into your home as gas or as electricity? G. 771. In the connection shown in the figure, the resistors have the same resistance R, 1. 2. A B C and at a voltage U the their power is P . What is the power dissipated in each resistor with the switches open (O) or O O closed (C)? Fill in the table! G. 772. Children play a circle O C game in the field. Unfortunately, the child in the middle C O of the circle steps in a wasps’ nest and the angry wasps C C fly away. The wind is blowing from the east at a speed of 4.5 m/s in the field, and the children are running radially outward at a speed of 6 m/s. According to the measurements of scientists these wasps in calm conditions can fly at a speed of 8 m/s. Estimate the percentage of children who are surely safe from wasp stings! You can also use a ruler, a pair of compasses and a protractor to find out the answer. P. 5382. An old tape magnetophone spins the takeup reel at a constant speed during a fast rewind. The inner diameter of both reels is 5 cm and their outer diameter is 15 cm. From the fully loaded feed reel, the rewinding time of the tape is 3 minutes. The tape is wound onto the initially empty takeup reel. How much time elapses from the start until the two reels have equal lengths of tape wound on them? P. 5383. What would the mass of the Earth have to be — with unchanged rotation and diameter — so that we would not be able to receive the satellite TV broadcast with a parabolic dish in Budapest. P. 5384. There is a small ball placed on top of a thin, uniform, vertical stick. Compared to the mass of the stick, the mass of the ball is negligible and the stick stands on a table which can be considered frictionless. Suddenly the stick falls over. In which case will the ball strike the tabletop at a higher speed, if it is glued to the top of the stick, or if it is simply

K¨ oz´ episkolai Matematikai ´ es Fizikai Lapok, 2022/2

127

put on the stick, from where it can fall off very easily? P. 5385. To what fraction does the heat flux released through a window, which has a single glass layer in it, decreases if the window is replaced by a double pane one? The thickness of each glass layer in both cases is dglass = 3 mm, and in the case of the double pane window there is a dair = 7 mm air gap between the two glass layers. The thermal conductivity of air is κair = 0.025 W/(m K) and the thermal conductivity of glass is κglass = 1.2 W/(m K). P. 5386. There is a small ball of charge Q = 5.55 μC fixed at the bottom of a 2 m long trough made of some insulating material. The trough makes an angle of elevation of α = 30◦ with the horizontal. From the top of the trough another small ball of mass m = 100 g with charge q = 10 μC is released from rest. How far does this ball can move if it rolls without slipping? (The charge of the ball does not change during its motion.) P. 5387. Different resistors of resistances R are connected to a battery of electromotive force U0 and of internal resistance Rb . a) What is the maximum “useful” power (dissipated in the external resistor) that this battery can deliver? At what external resistance value R can we achieve this maximum power Pmax ? b) Show that for any other power P which is smaller than Pmax , there are two external resistors of resistances R1 = R2 at which the dissipated power is P . What is the arithmetic and the geometric mean of R1 and R2 ? c) What is the sum of the terminal voltages across the battery in the above two cases? d) What is the sum of the currents through R1 and R2 ? e) The efficiency of the delivered energy is defined as the ratio of the useful power to the total power delivered by the battery. What is the sum of the efficiencies in the above two cases? P. 5388. Linearly polarized light from a 15 mW laser having a wavelength of λ = 632.8 nm is emitted from the 2 mm diameter circular aperture of the laser box. a) What is the maximum value of the electric field in the laser beam? b) What is the total linear momentum of a one metre long piece of the laser beam? P. 5389. A (point-like) fly flies at a constant speed of v parallel to the principal axis of a lens, having a focal length of f , at a distance of d from it. What is the least speed of the fly with respect to its image? P. 5390. There is a small electric dipole of dipole moment p at the centre of the thin-walled uncharged metal spherical shell of radius R, shown in the figure. Determine the surface charge density at points A and B, which are interior points of the shell. Determine the surface density of the charge on the outer surface of the shell as well. (Hint: Use the method of image charges applied for a sphere. It might be also useful to know the electric field due to a dipole at a point on the axial and equatorial lines.)

Problems of the 2021 Ku ¨rsch´ak competition 1. In the Cartesian coordinate system of the plane, the triangle determined by the points Pi = (ai , bi ) (i = 0, 1, 2) contains the origin O = (0, 0) in its interior. Show that the areas of the triangles P0 OP1 , P0 OP2 , P1 OP2 (in this order) form a geometric sequence if and only if the system of equations a0 x2 + a1 x + a2 = b0 x2 + b1 x + b2 = 0 has a real solution x. 2. In Wonderland, n airlines operate flights between n cities. For each airline, there are an odd number of cities, say, v1 , v2 , . . . , vi such that the airline operates the following flights: vj vj+1 and vj+1 vj for 1  j  i, where vi+1 = vi . Prove that we may choose an odd number of cities, say, u1 , u2 , . . . , uk in such a way that it is possible to buy tickets for the flights u1 u2 , u2 u3 , . . . , uk−1 uk , uk u1 from pairwise different airlines. 3. In the cyclic hexagon A1 B3 A2 B1 A3 B2 , the diagonals A1 B1 , A2 B2 and A3 B3 are concurrent. For i = 1, 2, 3, let Ci be the intersection of the diagonals Ai Bi and Ai+1 Ai+2 , and let Di be a point on the circumscribed circle, different from Bi , such that the circle Bi Ci Di is tangent to the line Ai+1 Ai+2 . (The points are indexed modulo 3, that is, A4 = A1 and A5 = A2 .) Prove that the segments A1 D1 , A2 D2 and A3 D3 are concurrent.

72. ´evfolyam 2. sz´am

K¨oMaL

Budapest, 2022. febru´ar

´ ´ FIZIKAI LAPOK ¨ EPISKOLAI KOZ MATEMATIKAI ES ˝ ´ITVE INFORMATIKA ROVATTAL BOV

Gyakorl´o feladatsor emelt szint˝ u matematika ´eretts´egire

´ ALAP´ITOTTA: ARANY DANIEL 1894-ben 72. ´evfolyam 3. sz´am

Budapest, 2022. m´arcius

I. r´esz

´ Megjelenik ´evente 9 sz´amban, janu´art´ol m´ajusig ´es szeptembert˝ol decemberig havonta 64 oldalon. ARA: 1050 Ft

´ TARTALOMJEGYZEK N´emeth L´aszl´o: Gyakorl´o feladatsor emelt szint˝u matematika ´eretts´egire . . . . . . . . . . . . . . . . . . . . . . . .

130

Marczis Gy¨orgy, Moln´ar Istv´an, Moln´ar Judit, R´ok´an´e R´ozsa Anik´o: Megold´asv´azlatok a 2022/2. sz´am emelt szint˝u matematika gyakorl´o feladatsor´ahoz . . . . . . . . . . . . . . . . . . . . . . .

133

Matematika C gyakorlat megold´asa (1685.) . . . . . . .

149

Matematika feladatok megold´asa (5114., 5207., 5212.) . . . . . . . . . . . . . . . . . . . . . . . . . . . . . . . . . . . . . . . .

152

A K pontversenyben kit˝uz¨ott gyakorlatok (724– 728.) . . . . . . . . . . . . . . . . . . . . . . . . . . . . . . . . . . . . . . . . .

155

A C pontversenyben kit˝uz¨ott gyakorlatok (727– 728., 1709–1713.) . . . . . . . . . . . . . . . . . . . . . . . . . . . . .

156

A B pontversenyben kit˝uz¨ott feladatok (5230– 5237.) . . . . . . . . . . . . . . . . . . . . . . . . . . . . . . . . . . . . . . . .

157

Az A pontversenyben kit˝uz¨ott nehezebb feladatok (821–823.) . . . . . . . . . . . . . . . . . . . . . . . . . . . . . . . . . . . .

159

T´oth Tam´as: A matematikai logika logikusabb, mint gondoln´ank II. . . . . . . . . . . . . . . . . . . . . . . . . . . .

159

Informatik´ab´ol kit˝uz¨ott feladatok (559–561., 61., 160.) . . . . . . . . . . . . . . . . . . . . . . . . . . . . . . . . . . . . . . . . .

164

Fizika gyakorlat megold´asa (757.) . . . . . . . . . . . . . . . .

169

Fizika feladatok megold´asa (5349., 5350., 5351., 5352., 5353., 5354., 5362., 5363., 5379., 5381.) . .

169

Fizik´ab´ol kit˝uz¨ott feladatok (412., 773–776., 5391–5399.) . . . . . . . . . . . . . . . . . . . . . . . . . . . . . . . . . .

186

Problems in Mathematics . . . . . . . . . . . . . . . . . . . . . . . .

189

Problems in Physics . . . . . . . . . . . . . . . . . . . . . . . . . . . . .

191

K¨ oz´ episkolai Matematikai ´ es Fizikai Lapok, 2022/3

´ EVA ´ F˝oszerkeszt˝o: RATKO ´ ¨ Fizikus szerkeszt˝o: GNADIG PETER ´ ILDIKO ´ M˝uszaki szerkeszt˝o: MIKLOS Bor´ıt´o: BURGHARDT ZSUZSA ´ Kiadja: MATFUND ALAP´ITVANY ´ RITA Alap´ıtv´anyi k´epvisel˝o: KOS Felel˝os kiad´o: KATONA GYULA Nyomda: OOK-PRESS Kft. ´ Felel˝os vezet˝o: SZATHMARY ATTILA INDEX: 25 450 ISSN 1215-9247 A matematika bizotts´ag vezet˝oje: ´ HERMANN PETER ´ BALINT, ´ ´ Tagjai: B´IRO GYENES ZOLTAN, ´ ´ KISS HUJTER BALINT, IMOLAY ANDRAS, ´ ´ GEZA, ´ ´ RITA, KOZMA GEZA, KOS KOS ´ MATOLCSI DAVID, ´ KATALIN ABIGEL, ´ ´ PACH PETER ´ ´ V´IGH ¨ ORDI ¨ OK PETERN E, PAL, VIKTOR A fizika bizotts´ag tagjai: ´ ´ ´ BARANYAI KLARA, HOLICS LASZL O, ´ ´ HONYEK GYULA, OLOSZ BALAZS, SZASZ ´ SZECHENYI ´ ´ KRISZTIAN, GABOR, VIGH ´ E, ´ VLADAR ´ KAROLY, ´ MAT WOYNAROVICH FERENC Az informatika bizotts´ag vezet˝oje: ´ ´ SCHMIEDER LASZL O ´ E, ´ FARKAS CSABA, FODOR Tagjai: BUSA MAT ´ ´ NIKOLETT, LOCZI ´ ZSOLT, LASZL O LAJOS, ´ ´ ´ SIEGLER GABOR, SZENTE PETER, TOTH ´ TAMAS ´ ANDREA, TASNADI ´ ANIKO ´ Ford´ıt´ok: GROF ´ ´ ¨ Szerkeszt˝os´egi titk´ar: TRASY GYORGYN E A szerkeszt˝os´eg c´ıme: 1117 Budapest, P´azm´any P´eter s´et´any 1/C III. emelet 3.405. Telefon: 372-2850 A lap megrendelhet˝o az Interneten: www.komal.hu/megrendelolap/reszletek.h.shtml. El˝ofizet´esi d´ıj egy ´evre: 8800 Ft K´eziratokat nem ˝orz¨unk meg ´es nem k¨uld¨unk vissza. Minden jog a K¨oMaL tulajdonosai´e. E-mail: [email protected] Internet: http://www.komal.hu This journal can be ordered from the Editorial office: P´azm´any P´eter s´et´any 1/C III. emelet 3.405. 1117–Budapest, Hungary telephone: +36 (1) 372-2850 or on the Postal address H–1518 Budapest 112, P.O.B. 32, Hungary, or on the Internet: www.komal.hu/megrendelolap/reszletek.e.shtml. A Lapban megjelen˝o hirdet´esek tartalm´a´ert felel˝oss´eget nem v´allalunk.

129

1. a) Oldjuk meg a 2x+1 + 3 = 21−x egyenletet a val´ os sz´amok halmaz´an. A H = {0; 1; 2; 3; 4; 5; 6; 7; 8; 9} alaphalmaz A, B, C r´eszhalmazair´ ol az al´abbiakat ismerj¨ uk: B ⊂ A; A ∪ C = {0; 8}; A ∩ C = {3; 4; 7}; C = {0; 1; 2; 8; 9}; A \ B = {2; 7; 9}. b) Elemeinek felsorol´as´aval adjuk meg az A, B, C halmazokat.

(11 pont)

2. Egy 10 cm oldal´ u n´egyzet minden oldal´ara kifel´e egyenl˝ o sz´ ar´ u h´aromsz¨ ogeket rajzoltunk, melyeknek sz´ arai 13 cm-esek, ´ıgy egy csillagszer˝ u alakzatot kaptunk. a) Mekkora a csillag ter¨ ulete? Felhajtogatva az egyenl˝ o sz´ ar´ u h´ aromsz¨ ogeket, egy n´egyzet alap´ u egyenes g´ ula keletkezett. b) Mekkora a g´ ul´ aba ´ırhat´ o g¨ omb sugara? (13 pont) 3. Andr´as ´es Bal´ azs zs´ıroznak”. A zs´ıroz´ as” a 32 lapos magyar k´ artya egyik ” ” egyszer˝ u j´ at´eka, amelynek az a l´enyege, hogy a v´eg´en az u an megszerzett ¨t´esek” sor´ ” lapok zs´ır” tartalma alapj´an d˝ ol el, ki nyerte a j´ at´ekot. ” arty´ aban n´egy sz´ın”: piros, z¨ old, makk, t¨ ok; mindegyik sz´ınen bel¨ ul A magyar k´ ” ´asz, kir´ aly, fels˝o, als´ o, t´ızes, kilences, nyolcas, hetes tal´ alhat´ o. Zs´ırnak” sz´ am´ıt ” az a´sz ´es a t´ızes, az nyer, akinek t¨ obb a zs´ırja”. (Ha mindketten 4 – 4 zs´ırt” ” ” szereztek, akkor az nyert, aki utolj´ara u ott”; d¨ ontetlen nincs.) ¨t¨ ” Az els˝ o leoszt´ asn´ al egyszerre n´egy-n´egy lapot kapnak a j´at´ekosok. a) Mennyi a val´ osz´ın˝ us´ege annak, hogy Andr´as els˝ o leoszt´askor kapott n´egy lapja k¨oz¨ott legal´ abb egy zs´ır”, ´es legal´ abb egy hetes tal´alhat´ o? ” Az´ert, hogy eld¨ onts´ek, ki kezdi a j´ at´ekot, sorsolnak u ´gy, hogy a megkevert csomagb´ ol felv´altva visszatev´es n´elk¨ ul leemelnek egy-egy lapot. Aki az els˝ o hetest at´ekot. (Ha pl. Andr´ as h´ uzza, az kapja el˝osz¨ or a n´egy lapot, ´es kezdi meg a j´ h´ uz el˝ osz¨or hetest, akkor Bal´azs kever, ´es oszt el˝ obb Andr´ asnak n´egy lapot, majd saj´ at mag´anak n´egyet; Andr´ as kezdi a j´at´ekot. K´es˝ obb ez a kever´es, oszt´ as- kezd´es felv´altva t¨ort´enik.) Andr´as kezdte a sorsol´ ast, ´es Bal´ azsnak a m´asodik h´ uz´ as´ ara siker¨ ult hetest h´ uznia. b) Mennyi ennek a val´ osz´ın˝ us´ege?

(13 pont)

4. Vegy¨ uk az al´ abbi kijelent´eseket: A) Ha egy m´ertani sorozatnak van v´eges hat´ ar´ert´eke, akkor h´ anyadosa egyn´el kisebb. 130

K¨ oz´ episkolai Matematikai ´ es Fizikai Lapok, 2022/3

2 2 B) Ha f (x) = x + 1,  ´es g ◦ f = x + 2x + 1, akkor g(x) = x . (g ◦ f = g f (x) , a g f¨ uggv´eny az f f¨ uggv´enynek k¨ozvetett f¨ uggv´enye.) C) Ha k´et sorozat ¨ osszege ´es szorzata konvergens, akkor a sorozatok k¨ ul¨onk¨ ul¨ on is konvergensek.

´ ´ ıt´asainkat a) Allap´ ıtsuk meg a kijelent´esek logikai ´ert´ek´et (igaz, hamis). All´ igazoljuk. D) Ha egy n cs´ ucs´ u egyszer˝ u gr´ af minden cs´ ucsa legal´ abb [ n2 ] fok´ u, akkor

a gr´ af ¨ osszef¨ ugg˝o. ([ n2 ] az n2 eg´esz r´esz´et jelenti.) b) Fogalmazzuk meg a D) ´all´ıt´ as megford´ıt´ as´ at, majd d¨onts¨ uk el, hogy ez igaz, vagy hamis. Meg´allap´ıt´ asunkat indokoljuk. (14 pont) II. r´esz 5. a) F¨ uggv´eny-transzform´ aci´ ok felhaszn´al´as´ aval a´br´ azoljuk az f (x) = 4|x| − x2 f¨ uggv´enyt a [−5; 5] intervallumon. A H halmaz elemeit az f (x) f¨ uggv´eny z´erushelyei ´es lok´ alis maximumhelyei alkotj´ ak. Ism´etl´es n´elk¨ ul, v´eletlenszer˝ uen kiv´ alasztunk h´ arom elemet H-b´ ol. b) Mennyi a val´ osz´ın˝ us´ege annak, hogy a h´arom elem ¨osszege oszthat´ o 9-cel? 2 Egy korl´ atos s´ıkidomot a g(x) = 4x − x , x ∈ R f¨ uggv´eny grafikonja ´es az x tengely z´ ar k¨ ozre. c) Sz´am´ıtsuk ki a s´ıkidom ter¨ ulet´et. (16 pont) 6. Egy vitorl´az´ orep¨ ul˝ o pil´ota teljes´ıtm´enyrep¨ ul´est tervez a k¨ovetkez˝ ok´eppen: Szombathelyr˝ol indul, d´elkelet fel´e rep¨ ul, majd Kaposv´ar k¨orny´ek´en ir´anyt v´ alt ´ EK ´ fel´e. Mikor Sz´ekesfeh´erv´ Ear l´egter´et el´erte, nyugatra tart, ´ıgy ´erkezik vissza ´ EK ´ az ´eszaki ´es ´eszakkeleti ir´ a kiindul´ o rep¨ ul˝ ot´erre. (Eany sz¨ogfelez˝ oj´ebe mutat´ o ir´ any.) Az 1 : 450 000-es m´eretar´ any´ u t´erk´epen a Kaposv´ ar-Sz´ekesfeh´erv´ar t´ avols´ ag 22 cm. (Az 1 : 450 000-es m´eretar´ any azt jelenti, hogy a t´erk´epen m´ert t´ avols´ ag 450 000-szerese van a val´os´ agban a k´et objektum k¨oz¨ott.) a) Mekkora a tervezett t´ avrep¨ ul´es hossza l´egvonalban? A v´egeredm´enyt 10 kmes pontoss´ ag´ ura kerek´ıtve km-ben adjuk meg. A rep¨ ul˝ og´ep hagyom´ anyos magass´ agm´er˝ oje a p l´egk¨ori nyom´ asb´ ol hat´ arozza h

meg a tengerszint feletti magass´agot a p = p0 · 2− 5500 k´eplet alapj´an, ahol p0 a tengerszinten m´ert nyom´ ast, h pedig a tengerszint feletti magass´ agot jelenti m´eter m´ert´ekegys´egben megadva. b) Milyen magasan van a rep¨ ul˝ og´ep, ha p0 = 103 kPa, p = 88 kPa? A vitorl´ az´ o rep¨ ul˝ og´epek leveg˝ obe emel´es´enek leggyakrabban alkalmazott m´ odja a cs¨ orl˝ovel vontat´ as. Ez u ´gy t¨ ort´enik, hogy a cs¨orl˝oaggreg´ator egy k¨ot´eldobra r´etegenk´ent szorosan feltekercseli a dr´ otk¨ otelet, amelynek v´eg´en a vitorl´ az´ o rep¨ ul˝og´ep van. A mell´ekelt ´ abr´ an a k¨ ot´eldob legfontosabb m´ereteit millim´eter m´ert´ekegys´egben t¨ untett¨ uk fel. A dr´ otk¨ ot´el a´tm´er˝ oje 6 mm. c) Legfeljebb milyen hossz´ u dr´ otk¨ otelet lehet feltekerni erre a dobra? (16 pont) K¨ oz´ episkolai Matematikai ´ es Fizikai Lapok, 2022/3

131

7. Egy trap´ez r¨ ovidebbik alapja 1, egy m´asik oldala 7 egys´eg hossz´ u. A trap´ez oldalainak hossz´ at megfelel˝o sorrendbe rakva egy sz´ amtani sorozat szomsz´edos elemeit kapjuk. a) Mekkora a trap´ez nagyobbik alapja, mekkor´ak a sz´ arak? Az al´abbi adatsokas´ agban n´eh´ any trap´ez oldalhossz´ anak m´er˝ osz´ am´at soroltuk fel v´eletlenszer˝ uen: 1, 3, 5, 7, 1, 4, 7, 10, 1, 7, 13, 19, 1, 6, 12, 15. b) Sz´am´ıtsuk ki az adatok a´tlag´ at, sz´ or´ as´ at, hat´ arozzuk meg a m´oduszt ´es a medi´ant. Az egy s´ıkban lev˝ o 1, 3, 7, 5 egys´eg hossz´ u szakaszokat ebben a sorrendben csukl´ osan r¨ogz´ıtett¨ uk egym´ ashoz, majd addig mozgattuk, m´ıg egy h´ urn´egysz¨ oget siker¨ ult kialak´ıtani. c) Mekkora sz¨ oget z´ ar be egym´assal ekkor az 5 ´es 7 egys´eg hossz´ u szakasz? (16 pont) 8. Egy vegyi anyagokat gy´ art´ o v´ allalat egy bizonyos term´eket, melynek ¨ osszet´etele csak hat´ oanyag´ anak koncentr´aci´ oj´aban k¨ ul¨ onb¨ ozik, k´etf´ele kiszerel´esben forgalmaz az al´ abbiak szerint. A v´ altozat: 60%-os t¨ om´enys´eg˝ u, 2 kg-os, 3 dm3 -es dobozban; B v´altozat: 20%-os t¨ om´enys´eg˝ u, 5 kg-os, 8 dm3 -es dobozban. A v´allalat mintaboltj´ aban a fenti a´rukb´ ol ´ arkedvezm´enyt adnak azoknak a vev˝ oknek, akik ¨ osszesen legal´ abb 40 kg-ot v´ as´ arolnak ezekb˝ ol. Egy vev˝ o, akinek 50%-os kever´ekre van sz¨ uks´ege, v´ as´ arolni szeretne bel˝ ol¨ uk u ´gy, hogy minden megv´as´arolt doboz tartalm´at teljes m´ert´ekben felhaszn´ alja. a) H´any dobozzal vegyen az egyes v´ altozatokb´ ol, ha r´eszes¨ ulni k´ıv´ an az a´rkedvezm´enyben, sz´all´ıt´ oeszk¨ oz´ere legfeljebb ¨ osszesen 120 kilogrammnyi terhet rakhat, a megv´ as´arolt a´ru teljes t´erfogata nem haladhatja meg a 130 dm3 -t, ´es a kedvezm´eny m´ert´eke egyenesen ar´ anyos a megv´ as´ arolt a´ru ¨ osszt¨ omeg´evel? Ez a v´allalat egyike annak az o t v´ a llalatb´ o l a ´ ll´ o csoportnak, melyben mind¨ egyik v´ allalat b´armelyik m´asikkal u zleti kapcsolatban a ´ ll, ´ e s az egym´ assal szembe¨ ni k¨ovetel´eseiket forintban, vagy eur´ oban egyenl´ıtik ki. K´et szerepl˝ o egym´ as k¨ oz¨ ott ugyanabban a p´enznemben fizeti ki a sz´ aml´ at. A szerz˝ od´esek megk¨ ot´ese ut´ an ´eszrevett´ek, hogy nincs h´arom olyan v´allalat, melyek egym´ as k¨ oz¨ ott azonos valut´aban rendezik tartoz´ asaikat. b) Igazoljuk, hogy mindegyik v´ allalat kett˝ onek forinttal, a m´ asik kett˝onek pedig eur´ oval fizet. (16 pont) 132

K¨ oz´ episkolai Matematikai ´ es Fizikai Lapok, 2022/3

9. a) A p val´ os param´eter mely ´ert´eke eset´en lesz az f (x) = x3 − 3x2 + 94 x + p (x ∈ R) f¨ uggv´enynek h´arom k¨ ul¨ onb¨ oz˝ o z´erushelye a val´os sz´amok halmaz´an? 3 2 uggv´eny b ´es c egy¨ utthat´ oit szab´ alyos A g(x) = x + bx + cx + 2022 (x ∈ R) f¨ dob´ okock´ aval sorsoljuk ki; az els˝o dob´ as b-t, a m´asodik c-t eredm´enyezi. b) Mennyi annak a val´ osz´ın˝ us´ege, hogy az ´ıgy kapott f¨ uggv´enynek nem lesz helyi sz´els˝ o´ert´eke? (16 pont)

hiszen a sz´aml´ al´o pozit´ıv, a nevez˝ o mindk´et t´enyez˝ oje, ´ıgy maga nevez˝o is pozit´ıv, vagyis a t¨ort pozit´ıv el˝ ojel˝ u, ami azt jelenti, hogy az eredeti kifejez´es el˝ ojele is pozit´ıv. osszef¨ ugg´es miatt: b) A cos α = sin (α + π2 ) ¨   π sin x + = sin 2x − 3   π sin x + = sin 2x + 3

N´emeth L´aszl´o Fony´od

A sin α = sin β t´ıpus´ u egyenletek megold´asi s´em´ aja alapj´an: 1. lehet˝ os´eg:

Megold´asv´azlatok a 2022/2. sz´am emelt szint˝ u matematika gyakorl´o feladatsor´ahoz

x+

I. r´esz 1. a) Hat´ arozzuk meg a k¨ ovetkez˝ o kifejez´es el˝ ojel´et, ha n tetsz˝ oleges term´eszetes sz´ am: 2n−1 + 1 2n + 1 − n+1 . (6 pont) n 2 +1 2 +1 b) H´ any val´ os megold´ asa van a   π π sin x + = cos 2x − 3 3

trigonometrikus egyenletnek a ]0; π[ intervallumon?

π π + , 3 2 π . 6

π π = 2x + + k2π (k ∈ Z), 3 6 π x1 = − k2π (k ∈ Z). 6

Ezen megold´ asok k¨ oz¨ ul a π6 esik a ]0; π[ intervallumba. 2. lehet˝ os´eg:  π π + l2π (l ∈ Z), x + = π − 2x + 3 6 x2 =

(7 pont)

Megold´as. a) Egyik lehet˝ os´eg. Osszuk el az els˝o (pozit´ıv) t¨ortkifejez´est a m´ asodik (pozit´ıv) t¨ ortkifejez´essel, felhaszn´ alva a hatv´ anyoz´ as ismert azonoss´ agait: 2n−1 + 1 2n + 1 2n−1 + 1 2n+1 + 1 22n + 2n+1 + 1 + 2n−1 : = · = = 2n + 1 2n+1 + 1 2n + 1 2n + 1 22n + 2n+1 + 1

2π π +l 6 3

(l ∈ Z).

Ezen megold´ asok k¨ oz¨ ul csak az l = 0 ´es az l = 1 eset´en kapunk olyan megold´ ast, ami a ]0; π[ intervallumba esik. Ezek a megold´ asok: π6 ´es 5π , amelyek ellen˝ o rz´ e ssel 6 igazolhat´ ok. Az egyenletnek teh´at k´et val´ os megold´asa esik a ]0; π[ inervallumba. 2. A 12. ´evfolyam tanul´ oi k¨ oz¨ ul 25-en matematik´ ab´ ol, 40-en pedig t¨ ort´enelemb˝ ol tettek emelt szint˝ u ´eretts´egi vizsg´ at. Az ´erdemjegyek eloszl´ as´ at a k¨ ovetkez˝ o k¨ ordiagramokon l´ atjuk:

2n−1 = 1 + 2n > 1, 2 + 2n+1 + 1 mert az ¨ osszegben szerepl˝o t¨ ort sz´ aml´ al´oja ´es nevez˝ oje is, ´ıgy maga a t¨ortkifejez´es is pozit´ıv el˝ ojel˝ u (az exponenci´alis f¨ uggv´eny tulajdons´aga miatt), amib˝ ol az k¨ovetkezik, hogy az eredeti k¨ ul¨ onbs´egben az els˝ o (pozit´ıv) tag nagyobb, mint a m´asodik (pozit´ıv) tag, ´ıgy a k¨ ul¨ onbs´eg¨ uk el˝ ojele pozit´ıv. M´ asik lehet˝ os´eg. Hozzunk k¨ oz¨ os nevez˝ore, ism´et felhaszn´alva a hatv´ anyoz´ as ismert azonoss´ agait: 2

22n + 2n−1 + 2n+1 + 1 − 22n − 2 · 2n − 1 (2n−1 + 1)(2n+1 + 1) − (2n + 1) = = n n+1 (2 + 1)(2 + 1) (2n + 1)(2n+1 + 1) =

2n−1 2n−1 + 2n+1 − 2 · 2n = n > 0, n n+1 (2 + 1)(2 + 1) (2 + 1)(2n+1 + 1)

K¨ oz´ episkolai Matematikai ´ es Fizikai Lapok, 2022/3

a) A k¨ ordiagramok alapj´ an t¨ olts¨ uk ki az al´ abbi gyakoris´ agi t´ abl´ azatot. (4 pont)

133

134

K¨ oz´ episkolai Matematikai ´ es Fizikai Lapok, 2022/3

Tant´ argy \ jegyek Matematika T¨ ort´enelem

3

4

5

b) Hat´ arozzuk meg a t¨ ort´enelem eredm´enyek ´ atlag´ at, medi´ anj´ at ´es sz´ or´ as´ at. (3 pont) c) A matematik´ ab´ ol 3-ast szerz˝ ok k¨ oz¨ ul legal´ abb h´ any tanul´ onak kellett volna 4-est kapnia, hogy a t¨ obbiek v´ altozatlan teljes´ıtm´enye mellett a matematika a ´tlag legal´ abb 3,8 legyen? (6 pont) Megold´as. a) Matematika: T¨ ort´enelem:

0,56 · 25 = 14, 0,5 · 40 = 20,

Tant´ argy \ jegyek Matematika T¨ ort´enelem b)

x=

sz´ or´ as: σ =

3 14 20

25 − (14 + 6) = 5. 40 − (15 + 20) = 5.

4 6 15

c) Kata azt ´ all´ıtja, hogy a j´ at´ekosok kiindul´ asi helye ´es a labda alkotta h´ aromanak? (6 pont) sz¨ og ter¨ ulete legfeljebb 25 m2 . Igaza van-e Kat´ Megold´as. a) (L´ asd az ´ abr´ at.) b) Az AOB = 2α = 70◦ , mert az AB ´ıven nyugv´ o k¨ oz´epponti sz¨ og. Az ACB h´ aromsz¨ogben alkalmazzuk a koszinusz-t´etelt:

5 5 5

c2 = 102 + 152 − 2 · 10 · 15 · cos 35◦ , c2 = 79,25,

20 · 3 + 15 · 4 + 5 · 5 = 3,625, 40 medi´an:



0,24 · 25 = 6, 0,375 · 40 = 15,

3. Az Andr´ assy gimn´ azium g´ olyat´ abor´ anak egy sportverseny´ehez a k¨ ovetkez˝ o p´ alya k´esz¨ ult el az udvar betonj´ an: egy k¨ orben az egy pontb´ ol kiindul´ o 10 m ´es oget z´ arnak be. A j´ at´ekosoknak a h´ urok (nem 15 m hossz´ u h´ urok egym´ assal 35◦ -os sz¨ k¨ oz¨ os) v´egpontj´ ab´ ol indulva kell megszerezni a k¨ or k¨ oz´eppontj´ aban l´ev˝ o labd´ at, majd visszafutni a kiindul´ asi hely¨ ukre. a) K´esz´ıts¨ unk a sz¨ ovegnek megfelel˝ o´ abr´ at a l´enyeges adatok felt¨ untet´es´evel. (2 pont) b) Legal´ abb mekkora utat kell megtenni egy-egy j´ at´ekosnak? (4 pont)

c = 8,9 m.

3+4 = 3,5, 2

2

2

2

20 · (3 − 3,625) + 15 · (4 − 3,625) + 5 · (5 − 3,625) = 0,695 ≈ 0,7. 40

c) A k´erd´eses tanul´ ok sz´ama legyen x, ahol 0 < x  14, x ∈ Z. Vel¨ uk egy¨ utt a k¨oz¨ os ´atlagt´ ol elv´art ´ert´ek:

tAOB =

7,762 · sin 70◦ = 28,29 m2 . 2

Mivel 28,29 m2 > 25 m2 , ´ıgy a kerek´ıt´esek m´ert´ek´ere is figyelve, nincs igaza Kat´anak.

42 − 3x + 24 + 4x + 25  3,8, 25

u k¨ or ´es a P (3; 9) pont. 4. Adott az x2 + y 2 − 14x − 12y + 65 = 0 egyenlet˝ a) A rajta”, k´ıv¨ ul”, bel¨ ul” szavak k¨ oz¨ ul ´ırjuk a pontozott vonalra azt, ame” ” ” lyikkel az al´ abbi ´ all´ıt´ as igaz lesz. Sz´ amol´ assal is igazoljuk a v´ alaszt. (3 pont)

91 + x  3,8, 25 91 + x  95, x  4.

Legal´abb 4 tanul´ onak kellett volna m´eg 4-est kapnia matematik´ ab´ol. 10·3+10·4+5·5 Ellen˝ orz´es: x = 4-re sz´ am´ıtott a´tlag = 3,8. 25 A (∗) egyenl˝ otlens´eg bal oldal´ an ´all´ o 91+x f¨ uggv´eny, az a´tlag f¨ uggv´enye, szi25 ´ gor´ uan monoton n¨ ov˝ o, ´ıgy az x n¨ ovel´esekor az ´atlag is n¨ovekszik. Igy a v´ alaszunk helyes. K¨ oz´ episkolai Matematikai ´ es Fizikai Lapok, 2022/3

A h´ ur, a hozz´ a tartoz´ o ker¨ uleti sz¨ og ´es a sug´ar k¨oz¨otti ¨ osszef¨ ugg´est felhaszn´ alva: c 8,9 = 2R =⇒ R = 7,76 m. ◦ = 2R =⇒ sin 35 0,5736 ´Igy egy-egy j´at´ekosnak 2 · R = 2 · 7,76 = 15,52 m-t kell megtennie. c) A h´ aromsz¨ og trigonometrikus ter¨ uletk´eplete alapj´an:

(14 − x) · 3 + (6 + x) · 4 + 5 · 5  3,8, 25

(∗)

c > 0,

135

A P pont . . . . . . . . . . . . van a fent megadott egyenlet˝ u k¨ orvonalon. b) Hat´ arozzuk meg az orig´ on ´es a P ponton ´ atmen˝ o g egyenesnek az x tengely pozit´ıv fel´evel bez´ art sz¨ og´et. A sz¨ og ´ert´ek´et egy tizedesjegy pontoss´ aggal adjuk meg. (3 pont) ´ c) Irjuk fel a megadott k¨ or azon ´erint˝ oj´enek egyenlet´et, amelynek nincs k¨ oz¨ os ot´ ol pontja a III. s´ıknegyeddel, nem megy a ´t az orig´ on, ´es ami az y tengelyt az orig´ k´etszer olyan t´ avols´ agban metszi, mint az x tengelyt, tov´ abb´ a a tengelyekkel alkotott h´ aromsz¨ og ter¨ ulete a legkisebb. (7 pont) 136

K¨ oz´ episkolai Matematikai ´ es Fizikai Lapok, 2022/3

Megold´as. a) A k¨ or k¨ oz´eppontj´anak koordin´at´ ai ´es a sugara az egyenlet´enek teljes n´egyzetes alakra hoz´ asa ut´ an k¨ ozvetlen¨ ul leolvashat´ ok: √ 2 2 (x − 7) + (y − 6) = 20 =⇒ C(7; 6) ´es r = 20 .

Mivel egy ´erint˝onek a k¨ orrel pontosan egy k¨ oz¨ os pontja van, ez´ert ennek a m´asodfok´ u, param´eteres egyenletnek a diszkrimin´ ansa 0 kell, hogy legyen: 2

D = (10 − 8a) − 4 · 5(4a2 − 24a + 65) = 0,

avols´ ag´at: Sz´ am´ıtsuk ki a C(7; 6) ´es a P (3; 9) pont dCP t´  √ √ 2 2 dCP = (3 − 7) + (9 − 6) = 25 = 5 > 20 .

A P pont . . . . . k´ıv¨ ul . . . . . van a fent megadott egyenlet˝ u k¨orvonalon.

Ennek az a-ban m´asodfok´ u egyenletnek a megold´ asai: a1 = 5 ´es a2 = 15, melyek helyess´eg´et az azonos a´talak´ıt´ asok biztos´ıtj´ ak, vagy az ellen˝ orz´esek igazolj´ ak.

b) Mivel az orig´o koordin´ at´ ai O(0; 0), ez´ert a keresett sz¨og tangense (g egye9 = = 3, amib˝ ol a megfelel˝ o kerek´ıt´essel α ≈ 71,6◦ nes ir´ anytangense) tg α = 9−0 3−0 3 ad´ odik.

assal ad´ odik (t1 = A minim´alis ter¨ ulet˝ u der´eksz¨ og˝ u h´ aromsz¨ og az a1 = 5 megold´ = 25 < t2 = 225), ennek az ´erint˝onek az egyenlet´et kell csak fel´ırni. A felt´eteleknek megfelel˝o e ´erint˝o egyenlete:

Ugyanezt a megold´ ast kapjuk, ha fel´ırjuk a g egyenes egyenlet´et (egyenes ar´ anyoss´ ag f¨ uggv´eny): y = 3x, ahonnan m = tg α = 3 egyenl˝os´egekb˝ ol ism´et a v´art odik. α ≈ 71,6◦ ad´

y = −2x + 10.

a2 − 20a + 75 = 0.

II. r´esz 5. a) Egy {an } sz´ amtani sorozat differenci´ aja 4, az els˝ o n tag ¨ osszege 1825, ´es az els˝ o tag megegyezik ezen ¨ osszeadott tagok sz´ am´ aval. Tagja-e ennek az {an } sorozatnak a 8115? (6 pont) Julcsi 2018. janu´ ar 1-j´en betett a bankba egy bizonyos ¨ osszeget, ´evi 5%-os kamatra u ´gy, hogy a banksz´ aml´ aj´ an a minden ´ev v´eg´en esed´ekes kamatokat t˝ ok´es´ıtette (nem vette ki). H´ arom ´ev elteltekor megemelte a megtakar´ıtott p´enz´et az ´eppen bent l´ev˝ onek a 20%-´ aval. Ett˝ ol kezdve m´ ar csak 4%-os ´evi kamatot kapott a bankint´ezett˝ ol. A k¨ ovetkez˝ o ´ev els˝ o napj´ an pedig kivette az addig megtakar´ıtott p´enz´enek a 10%-´ at. 2025. janu´ ar 1-j´en szeretn´e felvenni az ¨ osszes p´enz´et. Testv´ere, Anna egyszerre kezdett vele takar´ekoskodni ugyanakkora ¨ osszeggel.

4.b)

4.c)

c) K´esz´ıts¨ unk v´ azlatot egy ´erint˝o ´es a tengelymetszetek (t´avols´ agok) berajzol´ as´aval. A felt´etelek miatt az ´erint˝o mindk´et tengelyt a pozit´ıv fel´en (nem az orig´oban) metszi. Legyen Mx (a; 0) ´es My (0; 2a). ´Igy az ´erint˝o egyenlete (a > 0 param´eterrel): y = −2x + 2a. Keress¨ uk a k¨ or ´es az ´erint˝o k¨ oz¨ os pontj´ at, az ´erint´esi pontot (pontokat). Ezt az egyenleteikb˝ol alkotott egyenletrendszer megold´ asa adja: 2

2

(x − 7) + (y − 6) = 20, y = −2x + 2a. A behelyettes´ıt˝ o m´odszert alkalmazva, a z´ ar´ ojelek felbont´asa, ¨osszevon´ as ´es rendez´es ut´ an a k¨ ovetkez˝ o m´ asodfok´ u, param´eteres (a) egyenletet kapjuk: 5x2 + (10 − 8a)x + 4a2 − 24a + 65 = 0. K¨ oz´ episkolai Matematikai ´ es Fizikai Lapok, 2022/3

137

b) H´ any %-os, a ´lland´ o ´evi kamatot kellene kapnia ahhoz Ann´ anak bet´et ´es kiv´et n´elk¨ uli takar´ekoss´ ag eset´en, hogy a k´et testv´ernek ugyanannyi megtakar´ıtott p´enze legyen 2025 els˝ o napj´ an? A kamatl´ ab ´ert´ek´et egy tizedes pontoss´ aggal adjuk meg. (4 pont) Kar´ acsonyra Franci az ´ abr´ an l´ athat´ o, n´egy cikkb˝ ol ´ all´ o, tengelyesen szimmetrikus feny˝ ofa d´ıszt kezdte el rajzolni egy pap´ırra. Egy 5 cm sugar´ u f´elk¨ orb˝ ol indult ki, eggyel feljebb l´epalve, a k¨ orcikk sugar´ at a fel´ere, k¨ oz´epponti sz¨ og´et 23 r´esz´ere v´ toztatta, mik¨ ozben a k¨ orcikk k¨ oz´eppontj´ at a felette l´ev˝ o k¨ or´ıv felez´esi pontj´ aba tette. Elgondolkodott azon, ha ezt az elj´ ar´ ast v´egtelen sok´ aig tudn´ a folytatni, lenne-e a mint´ anak v´eges ter¨ ulete. c) Ha igen, pontosan mekkora lenne, ´es az h´ any %-a annak a minim´ alis ter¨ ulet˝ u t´eglalap ter¨ ulet´enek, amelynek k´et oldala p´ arhuzamos a feny˝ ofa tengely´evel? (6 pont) Megold´as. a) A sz´ amtani sorozat jel¨ ol´eseit felhaszn´ alva: a1 = n; d = 4; amtani sorozatok ¨ osszeg´ere vonatkoz´ o szab´ aly a felSn = 1825; a1 = n =?. A sz´ 138

K¨ oz´ episkolai Matematikai ´ es Fizikai Lapok, 2022/3

t´eteleket figyelembe v´eve: n n 2a1 + (n − 1)d = 2n + (n − 1)d = 3n2 − 2n = 1825, Sn = 2 2

A cikkek sugarai, a r´eszmagass´ agok rendre: 5 cm, 2,5 cm, 1,25 cm, 0,625 cm, . . . . ociens˝ u m´ertani soroA k´epz´esi szab´ aly alapj´an ezek a sz´ amok egy qm = 12 kv´ ol az elemekb˝ol k´epzett v´egtelen m´ertani zatot alkotnak. Mivel |qm | < 1, ´ıgy ezekb˝ sornak szint´en van v´eges ¨ osszege, amely a k´epzeletbeli d´ısz¨ unk magass´ aga, egyben a minim´alis ter¨ ulet˝ u (´erintkez˝o) t´eglalap egyik oldala lenne:

3n2 − 2n − 1825 = 0,

n1 = 25,

1 n2 = −24 . 3

m=

A negat´ıv sz´am tartalmi ellentmond´as (sorsz´am) miatt nem megold´ as, ´ıgy a sorozat els˝ o tagja: a1 = 25. A sz´ amtani sorozat m-edik elem´ere fel´ırhat´ oo ugg´est alkalmazva: ¨sszef¨

A k´erd´eses k´et ter¨ ulet ar´ anya %-ban kifejezve, ahol tt = 2r1 m: t 15π 3π 15π = = ⇒ 15π%. = tt 2r1 m 2 · 5 · 10 20

8115 = 25 + (m − 1)4, m = 2023,5 ∈ / Z+ .

V´ alasz: Ha Franci el tudn´ a k´esz´ıteni v´egtelen sok l´ep´esb˝ ol ezt a mint´ at, akkor a d´ısz ter¨ ulete 15π%-a (≈ 47%) lenne a t´eglalap ter¨ ulet´enek.

Ebb˝ ol az k¨ ovetkezik, hogy a 8115 nem eleme a sorozatnak. Ellen˝ orz´es: 25  n Sn = 2a1 + (n − 1)d = 2 · 25 + (25 − 1) · 4 = 1825 = S25 . 2 2

A megold´ asunk ´ıgy helyes. ugyi b) Julcsi megtakar´ıt´asa (aj ; a0 ; p1 = 5%; p2 = 4%) a kiv´etelkor a p´enz¨ folyamatot is szeml´eltetve a m˝ uveletek sorrendj´evel:

´ EK ´ Kft. u 6. a) A JAT ´j homokoz´ o v¨ odre csonkak´ up alak´ u. Ha a v¨ od¨ orbe beletesz¨ unk egy 14 cm ´ atm´er˝ oj˝ u labd´ at, akkor az ´erinti a v¨ od¨ or alj´ at, ´es egy k¨ orben a v¨ od¨ or oldal´ at is. A v¨ od¨ or alj´ anak ´ atm´er˝ oje 12 cm, fels˝ o ny´ıl´ as´ anak a ´tm´er˝ oje 18 cm. A v¨ odr¨ ot k´ıv¨ ul is, bel¨ ul is v´ız´ all´ o r´eteggel festik be. H´ any liter fest´ekre van sz¨ uks´eg 1000 darab v¨ od¨ or elk´esz´ıt´esekor, ha 1 n´egyzetm´eternyi fel¨ ulet fest´es´ehez 0,5 dl fest´eket haszn´ alnak, ´es a v¨ od¨ or falvastags´ aga elhanyagolhat´ o? (8 pont) b) Panni ´es Peti kokt´elos poharakb´ ol bodzasz¨ orp¨ ot iszik. A poh´ ar fels˝ o r´esze forg´ ask´ up alak´ u, melynek magass´ aga 8 cm, alkot´ oja 10 cm, ´es 7 cm magasan ´ all benne” az u o. H´ any millim´eterrel ¨d´ıt˝ ” emelkedik meg a bodzasz¨ orp szintje”, ha a poh´ arba h´ arom darab, ” 2 cm ´el˝ u j´egkock´ at tesz¨ unk, ´es azok m´ ar teljesen elolvadtak? A v´ alaszt eg´eszre kerek´ıtve adjuk meg. (Tekints¨ unk el a j´eg olvad´ asakor k¨ ozismerten bek¨ ovetkez˝ o t´erfogatv´ altoz´ ast´ ol.) (8 pont)

aj = a0 · 1,053 · 1,2 · 1,04 · 0,9 · 1,043 ≈ 1,463a0 . Anna megtakar´ıt´ asa (aa ; a0 ; p) a kiv´etelkor:  p 7 aa = a0 1 + . 100 Mivel

 p 7 aj ≈ 1,463a0 = a0 1 + = aa , 100 p ≈ 5,6%. Teh´ at Ann´anak megk¨ ozel´ıt˝oleg 5,6%-os, a´lland´ o ´eves kamatot kell kapnia ahhoz, hogy a k´et testv´ernek (k¨ ozel) azonos megtakar´ıtott p´enze legyen 2025 els˝o napj´an. c) A k¨ orcikkek ter¨ ulete ar´ anyos a sug´ ar n´egyzet´evel ´es a k¨oz´epponti sz¨og´evel, ez´ert a r´eszter¨ uletek a k¨ ovetkez˝ ok lesznek (cs¨ okken˝ o sorrendben): 2

Megold´as. a) K´esz´ıts¨ uk el a v¨ od¨ ornek (szimmetrikus csonka k´ up) egy tengelymetszet´et, ami egy szimmetrikus trap´ez lesz, ´es azon jel¨ olj¨ uk be a l´enyeges adatokat. uzott sug´ arra, Felhaszn´ alva, hogy az ´erint˝o mer˝ oleges az ´erint´esi pontba h´ valamint azt, hogy adott k¨ orh¨ oz k¨ uls˝ o pontb´ ol h´ uzott ´erint˝o szakaszok hossza egyenl˝ o: OA = OE = r = 7 cm ´es BA = BE = 6 cm. Mivel az adott k¨ or k¨ oz´eppontja rajta van az ABE = β sz¨ ogfelez˝ oj´en, ´es az alapokat felez˝ o szimmetriatengely DA ⊥ AB, az OAB der´eksz¨ og˝ u h´ aromsz¨ ogben:

3

2 2 2 · 180◦ · 180◦ · 180◦ 180◦ 2 3 2 (3) 2 (3) 5 π ; 2,5 π ; 1,25 π ; 0,625 π ; ... . 360◦ 360◦ 360◦ 360◦ 2

tg

u m´ertani sorozatot A k´epz´esi szab´ aly alapj´an ezek a sz´ amok egy qt = 16 kv´ociens˝ ol az elemekb˝ol k´epzett v´egtelen m´ertani sornak alkotnak. Mivel |qt | < 1, ´ıgy ezekb˝ van v´eges ¨ osszege: t1 12,5π t= = = 15π cm2 . 1 − qt 1− 1 139

7 β = ⇒ β = 98,8◦ . 2 6

Toljuk el p´ arhuzamosan AD-t B-be, ´ıgy AD BM , ´es M BC = γ = 98,8◦ − 90◦ = 8,8◦ ,

6

K¨ oz´ episkolai Matematikai ´ es Fizikai Lapok, 2022/3

r1 5 = 10 cm. = 1 − qm 1 − 0,5

140

illetve M C = 9 − 6 = 3 cm.

K¨ oz´ episkolai Matematikai ´ es Fizikai Lapok, 2022/3

3 A BM C der´eksz¨og˝ u h´aromsz¨ ogben: sin 8,8◦ = BC =⇒ BC = 19,61 cm (ez az alkot´ o). A fel¨ ul nyitott csonkak´ up felsz´ın´et k´etszer kell venn¨ unk (k´ıv¨ ul-bel¨ ul):  2A = 2 62 π + (6 + 9)π · 19,61 = 2074,4 cm2 .

Befestend˝ o fel¨ ulet: 1000 · 2A = 1000 · 2074,4 = 2 074 400 cm2 = 207,44 m2 . A felhaszn´ aland´ o fest´ek t´erfogata: 207,44 · 0,5 = 103,72 dl = 10,372 liter. Teh´ at megk¨ ozel´ıt˝oen 10,4 liter fest´ekre van sz¨ uks´eg.

(hasonl´ os´agi ar´any). Fel´ırva a t´erfogatok ar´any´ at, ´es kihaszn´ alva a k¨ ozismert t´etelt: V 226,04 ≈ 1,12 = k 3 ⇒ k ≈ 1,038. = Vb 202,04 Ezt behelyettes´ıtve az oldalak ar´ any´ aba: 7+h = 1,038 =⇒ h = 0,266 ≈ 0,3 (cm) = 3 mm. 7 Teh´at a poh´arban megk¨ ozel´ıt˝oen 3 mm-t fog emelkedni a bodzasz¨ orp szintje a j´egkock´ak elolvad´asa ut´ an. Megjegyz´esek: 1. Ez az ´ert´ek csek´ely m´ert´ekben v´ altozhat, ha a szok´ asos ´es megengedett kerek´ıt´eseket m´ ar kor´ abban elv´egezz¨ uk, de az biztos, hogy a poh´ arba m´eg belef´er a kokt´el. 2. Az olvad´ ast az´ert kell megeml´ıteni, ez fontos felt´etel, mert a j´eg s˝ ur˝ us´ege kisebb a v´ız s˝ ur˝ us´eg´en´el, ´es ez´ert a feladat megold´ asa m´ as lesz, ha a j´eg nem olvad el (j´eghegy), ebben az esetben a t´erfogatok nem ad´ odnak o ¨ssze.

7. Adott k´et teljes gr´ af. Az els˝ o gr´ afnak 4-gyel t¨ obb cs´ ucsa ´es 62-vel t¨ obb ´ele van, mint a m´ asodiknak.

6.a)

6.b)

b) Rajzoljuk meg a poh´ ar fels˝o r´esz´enek (forg´ask´ up) a tengelymetszet´et, amely egy tengelyesen szimmetrikus, egyenl˝ o sz´ ar´ u h´aromsz¨og lesz. Mivel AC = 8 ´es AB = 10, ´ıgy alkalmazva az ABC der´eksz¨ og˝ u h´ aromsz¨ogre a Pitagorasz-t´etelt: CB = 6 (cm). Mivel AD = 7 ´es ADE ∼ ACB (sz¨ogek p´ aronk´ent egyenl˝oek), ´ıgy: DE 7 DE 42 AD = =⇒ = =⇒ DE = = 5,25 (cm). AC CB 8 6 8 A bodzasz¨ orp t´erfogata: Vb =

1029 5,252 · π · 7 = π ≈ 202,04 (cm3 ). 3 16

A j´egkock´ ak t´erfogat´ anak ¨ osszege: 3V k = 3a3 = 3·23 = 3 · 8 = 24 (cm3 ). A kokt´el (bodza ´es az elolvadt j´eg kever´eke) t´erfogata:

a) H´ any cs´ ucsa van annak a teljes gr´ afnak, amelynek annyi ´ele van, mint az adott k´et teljes gr´ af ´elei ´es cs´ ucsai sz´ am´ anak ¨ osszege? (6 pont) √ uggv´eny ´ertelmez´esi tartom´ aLegyen az A halmaz az f (x) = −2x2 + 5x + 3 f¨ otlens´eg megold´ ashalnya, a B halmaz pedig a log 3 (4x − 3) > log 3 (2x + 7) egyenl˝ π π maza. b) Hat´ arozzuk meg az A ∩ B, az A ∪ B ´es az A \ B halmazokat. (6 pont) T´ız bar´ at, Anna, Bea, Cili, D´ ori, Emese, Fruzsi, G´ abor, Huba, Istv´ an ´es J´ anos moziba megy. A jegyek az els˝ o sorba egym´ as mell´e sz´ olnak.

c) H´ anyf´elek´eppen u u azt k´erte, hogy mindegyik¨ uk k¨ oz¨lhetnek le, ha a n´egy fi´ vetlen¨ ul k´et l´ any k¨ oz¨ ott u (4 pont) ¨lhessen? Megold´as. a) Legyen rendre x, y ´es z a h´ arom teljes gr´ af cs´ ucsainak sz´ama, ahol x, y, z ∈ Z+ .

A megold´ as sor´ an felhaszn´aljuk azt a t´enyt, hogy ha egy teljes gr´afnak n darab n(n−1) . A feladat felt´etelei alapj´an a k¨ ovetkez˝ o cs´ ucsa van, akkor az ´eleinek sz´ ama 2 ugg´esek ´ırhat´ ok fel: ¨osszef¨ x = y + 4,

3

V = Vb + 3V k = 202,04 + 24 = 226,04 (cm ).

x(x − 1) y(y − 1) = + 62, 2 2

Mivel ADE ∼ AF G (sz¨ ogek p´ aronk´ent egyenl˝oek), ez´ert:

z(z − 1) x(x − 1) y(y − 1) =x+y+ + . 2 2 2

AF AD + DF AD + h 7+h = = = =k AD AD AD 7 K¨ oz´ episkolai Matematikai ´ es Fizikai Lapok, 2022/3

141

142

K¨ oz´ episkolai Matematikai ´ es Fizikai Lapok, 2022/3

Felhaszn´ alva, hogy x = y + 4, a m´asodik ¨ osszef¨ ugg´es a´t´ırhat´o az al´abbi alakra:

A B halmaz meghat´aroz´ as´ ahoz figyelembe kell venni a logaritmusf¨ uggv´eny u monoton cs¨ okken˝ o tulaj´ertelmez´esi tartom´any´ at ´es a 0 < π3 < 1 miatti szigor´ dons´ ag´at, ´ıgy a k¨ ovetkez˝ o line´aris egyenl˝ otlens´egrendszert kell megoldani:

y(y − 1) (y + 4)(y + 3) = + 62, 2 2 y 2 + 7y + 12 = y 2 − y + 124, 8y = 112,

3 , 4

4x − 3 > 0,

⇒ x>

2x + 7 > 0,

⇒ x > −3,5,

4x − 3 < 2x + 7; ⇒ x < 5.

y = 14 ⇒ x = 18. Ezeket az ´ert´ekeket be´ırva a harmadik egyenletbe:

 Az egyenl˝otlens´egrendszer megold´ asa: 34 < x < 5, m´ask´eppen x ∈ 34 ; 5 , azaz: B = {x ∈ R 3 < x < 5}. 4

z(z − 1) 18 · 17 14 · 13 z(z − 1) = 18 + 14 + + ⇒ = 276 ⇒ z(z − 1) = 552. 2 2 2 2

A keresett halmazm˝ uveletek eredm´enyei:  

3 3 ;3 , A∩B = x∈R 0).

A kapott N (x) f¨ uggv´eny maximumhely´et kell megkeresn¨ unk. A sz´els˝ o´ert´eket (az els˝orend˝ u) deriv´ alt seg´ıts´eg´evel hat´ arozzuk meg: N  (x) = −3x2 + 24x + 252.

Megoldjuk az N  (x) = 0 egyenletet: −3x2 + 24x + 252 = 0, ahonnan az egyenlet o az x > 0 felt´etel miatt), az x2 = 14 teljes´ıti a felt´etelt. megold´ asai x1 = −6 (nem j´ x N (x) N (x)

Az eredm´enyeket” t´ abl´ azatba foglalva: ”



]0; 14[ + 

14 0 max.

]14; ∞[ − 

Az x = 14 (glob´ alis) maximumhelye az N (x) f¨ uggv´enynek. Teh´ at 1400 darab term´ek ´ert´ekes´ıt´ese ut´ an lesz a c´eg nyeres´ege maxim´ alis. (Ekkor a maxim´ alis nyeres´eg: Nmax = N (14) = −143 + 12 · 142 + 252 · 14 = 3136 milli´o Ft.) uzzuk, b) K(3) = 33 − 12 · 32 + 48 · 3 = 63, teh´at a pont, ahov´a az ´erint˝ot h´ a P (3; 63). Az ´erint˝o meredeks´eg´et a deriv´ altf¨ uggv´eny adott pontbeli helyettes´ıt´esi ´ert´eke adja meg. Mivel K  (x) = 3x2 − 24x + 48, ez´ert az ´erint˝o meredeks´ege:

2

Teh´at: (x − 3) (x − 6) = 0 ⇒ x = 6 (lesz a m´asik metsz´espont abszcissz´ aja). A szorzatt´ a alak´ıt´ as megt¨ ort´enhet a polinomok oszt´ as´ anak, a m´asodfok´ u egyenletek megold´ as´anak, majd a polinomok gy¨ okt´enyez˝ os alakj´ anak a felhaszn´al´as´aval is. 2 onnyen beFigyelembe v´eve, hogy a [3; 6] intervallumon (x − 3) (x − 6)  0, k¨ l´athat´ o, hogy ezen az intervallumon a K(x) f¨ uggv´eny grafikonja mindv´egig az ´erint˝o alatt halad, hiszen: 2

x3 − 12x2 + 45x − 54 = (x − 3) (x − 6)  0 ⇒ x3 − 12x2 + 48x  3x + 54. ´Igy a k´erd´eses korl´atos, z´ art s´ıkidom ter¨ ulet´enek m´er˝ osz´ ama: 6 3

=

3

(−x3 + 12x2 − 45x + 54) dx =





  4  64 3 62 32 + 4 · 63 − 45 · + 54 · 6 − − + 4 · 33 − 45 · + 54 · 3 = 4 2 4 2

f ∗ : R → R, f ∗ (x) = x3 . Ebb˝ ol egyszer˝ u f¨ uggv´enytranszform´acioval megkaphat´ ´ o a feladat felt´eteleinek megfelel˝o f¨ uggv´eny: 3

f (x) = (x − 3) . Megjegyz´es. B´ ar a feladat nem k´er r´eszletes indokl´ ast, de ellen˝ orizhetj¨ uk, megfelel-e a felt´eteleknek az f (x):

2

x − 13x + 45x − 54 = x − 3x −9x + 27x +18x − 54 =         

f (x) = (x − 3)3 ⇒ f  (x) = 3(x − 3)2 ⇒ f  (3) = 0.

= x2 · (x − 3) − 9x · (x − 3) + 18 · (x − 3) = (x − 3)(x − 3)(x − 6) =

uan monoton n¨ ovekv˝ o minden Tov´ abb´ a f  (x) > 0 minden x = 3 eset´en, ´ıgy f (x) szigor´ x ∈ R-re, teh´ at nem lesz sz´els˝ o´ert´eke az x = 3 helyen, annak ellen´ere, hogy f  (3) = 0.

2

= (x − 3) (x − 6). K¨ oz´ episkolai Matematikai ´ es Fizikai Lapok, 2022/3

− (x − 12x + 45x − 54) dx =

6

Van ilyen f¨ uggv´eny, pl. az

Megfelel˝ o csoportos´ıt´ assal szorzatt´ a alak´ıthatunk: 2



c) Olyan f ∗ f¨ uggv´enyt kell keres” n¨ unk”, amely (p´eld´ aul) szigor´ uan n¨ ovek v˝ o, ´es (f ∗ ) null´aval egyenl˝o egyes pontokban.

x3 − 12x2 + 48x = 3x + 54 ⇒ x3 − 12x2 + 45x − 54 = 0. 3

2

  81 405 189 27 = (−324 + 864 − 810 + 324) − − + 108 − + 162 = 54 − = . 4 2 4 4

´Igy az ´erint˝o egyenlete: y − 63 = 3(x − 3), ahonnan y = 3x + 54. A tov´ abbiakban el˝ osz¨ or hat´ arozzuk meg, hogy hol metszi (m´eg) egym´ast az ´erint˝o ´es a f¨ uggv´eny grafikonja. Ehhez meg kell oldanunk az x3 − 12x2 + 48x = = 3x + 54 (harmadfok´ u) egyenletet. A megold´ asban seg´ıts´eg¨ unkre lesz”, hogy tud” juk, az x = 3 megold´ asa az egyenletnek (hiszen az x = 3 abszcissz´ aj´ u pontj´ aban ´erinti az ´erint˝o a f¨ uggv´eny grafikonj´ at).

2

3

6  4 x3 x2 x − 45 · + 54x = = − + 12 · 4 3 2 3

m = K  (3) = 3 · 32 − 24 · 3 + 48 = 3.

3



145

146

K¨ oz´ episkolai Matematikai ´ es Fizikai Lapok, 2022/3

9. Peches P´ al nagyon szereti a kapar´ os sorsjegyeket. Kedvence a Lutri sorsjegy, melynek ´ ara 500 Ft, ´es a sorsjegyek 25%-a nyer˝ o. P´ alnak (most csak) n´egy darab 500 forintosa van. Bemegy egy lott´ oz´ oba, ´es elhat´ arozza, hogy addig v´ as´ arolja kedvenc sorsjegy´et, am´ıg nem nyer, vagy ameddig a p´enze el nem fogy. a) Hat´ arozzuk meg a P´ al ´ altal a sorsjegy(ek)re elk¨ olt¨ ott 500 forintosok sz´ am´ anak v´ arhat´ o ´ert´ek´et ´es sz´ or´ as´ at. (7 pont) P´ al h´ aromf´ele t¨ omegk¨ ozleked´esi eszk¨ ozzel tudja munkahely´et megk¨ ozel´ıteni, ´espedig busszal, metr´ oval, illetve villamossal, ez´ert (is) kombin´ alt b´erlettel rendelkezik. Az esetek 25%-´ aban busszal megy, a metr´ ot pedig n´egyszer olyan gyakran haszn´ alja, mint a villamost. A buszon ´ atlagosan minden negyedik, a villamoson a ´tlagosan minden tizedik alkalommal ellen˝ orzik a b´erlet´et, m´ıg annak a val´ osz´ın˝ us´ege, hogy a metr´ on kap ellen˝ orz´est, 0,85. b) Egyik alkalommal ellen˝ orizt´ek a b´erlet´et. Mennyi annak a val´ osz´ın˝ us´ege, hogy villamossal utazott? (6 pont) Egyik nap (a munkanap v´eg´en) P´ al egy ¨ otf˝ os bar´ ati t´ arsas´ ag tagjak´ent busszal utazott haza. Az egyik meg´ all´ oban ellen˝ or¨ ok sz´ alltak fel, ´es a buszon (aktu´ alisan) tart´ ozkod´ o 48 utasb´ ol tal´ alomra kiv´ alasztott t´ız embernek a b´erlet´et (vagy jegy´et) ellen˝ orizt´ek. c) Mennyi annak a val´ osz´ın˝ us´ege, hogy az ¨ otf˝ os bar´ ati t´ arsas´ agb´ ol legal´ abb k´et f˝ ot ellen˝ oriztek? (3 pont)

Megold´as. a) A ξ val´osz´ın˝ us´egi v´ altoz´ o jelentse a sorsjegy´ert elk¨olt¨ott 500 forintosok sz´am´at. Annak a val´ osz´ın˝ us´ege, hogy egy sorsjegy nyer˝ o, 14 , m´ıg annak,

hogy nem nyer˝o, 34 . A ξ val´osz´ın˝ us´egi v´ altoz´ o lehets´eges ´ert´ekei: 1; 2; 3; 4 (a ξ = 4 eset mag´aban foglalja” azt az esetet is, amikor az utols´ o v´ as´arolt sorsjegy nyer˝ o, ” illetve azt is, amikor nem nyer˝ o).   1 16 P (ξ = 1) = = (m´ ar az els˝o megv´ as´ arolt sorsjegy nyer˝ o). 4 64   3 12 3 1 = (az els˝ o megv´ as´arolt sorsjegy nem nyer˝ o, de P (ξ = 2) = · = 4 4 16 64 a m´ asodik igen). 9 3 3 1 (az els˝ o k´et sorsjegy nem nyer˝ o, de a harmadik P (ξ = 3) = · · = 4 4 4 64 nyer˝ o). 108 27 3 3 3 1 3 3 3 3 = (az els˝ o h´arom sorsjegy nem P (ξ = 4) = · · · + · · · = 4 4 4 4 4 4 4 4 256 64 nyer˝ o, de a negyedik nyer˝ o, illetve egyik v´ as´ arolt sorsjegy sem nyer˝ o). A ξ val´osz´ın˝ us´egi v´ altoz´ o eloszl´ asa:

A ξ val´osz´ın˝ us´egi v´ altoz´ o v´ arhat´ o ´ert´eke: M (ξ) =

i=1

1

2

3

4

P (ξ = xi )

16 64

12 64

9 64

27 64

K¨ oz´ episkolai Matematikai ´ es Fizikai Lapok, 2022/3



xi · P (ξ = xi ) = 1 ·

12 9 27 175 16 +2· +3· +4· = ≈ 2,73. 64 64 64 64 64

A sz´ or´ ashoz sz¨ uks´eges (a ξ val´ osz´ın˝ us´egi v´ altoz´ o m´ asodik momentuma): M (ξ 2 ) =

4  i=1

x2i · P (ξ = xi ) = 12 ·

577 16 12 9 27 + 22 · + 32 · + 42 · = ≈ 9,02. 64 64 64 64 64

A ξ val´osz´ın˝ us´egi v´ altoz´ o sz´ or´ asa:   D(ξ) = D2 (ξ) = M (ξ 2 ) − M 2 (ξ) = =



6303 = 4096





577 − 64



175 64

2

=

6303 ≈ 1,24. 64

b) Jel¨olje B azt az esem´enyt, hogy busszal, M azt, hogy metr´ oval, illetve V azt, hogy villamossal k¨ ozel´ıti meg P´al a munkahely´et. A feladat felt´etelei alapj´an: P (B) = 0,25, P (V) = x, P (M) = 4x (x ∈ R+ ). Mivel a B, M ´es V esem´enyek teljes esem´enyrendszert alkotnak, ´ıgy: P (B) + P (M) + P (V) = 1 ⇒ 0,25 + 4x + x + 1 = 1 ⇒ 5x = 0,75 ⇒ x = 0,15. Ebb˝ ol k¨ovetkezik, hogy: P (B) = 0,25, P (M) = 0,60, P (V) = 0,15. Jel¨olje E azt az esem´enyt, hogy P´al ellen˝ orz´est kap valamelyik (´altala haszn´ alt) ´ k¨ozleked´esi eszk¨ oz¨ on. Igy (felt´eteles val´osz´ın˝ us´egekkel van dolgunk): P (E | B) =

1 = 0,25, 4

P (E | M) = 0,85,

P (E | V) =

1 = 0,10. 10 P (V·E)

A feladat k´erd´ese”: P (V | E) =?. Figyelembe v´eve, hogy P (V | E) = P (E) , ” a megadott ´es kisz´am´ıtott adatok alapj´ an (alkalmazva a teljes val´osz´ın˝ us´eg t´etel´et): P (E) = P (E | B) · P (B) + P (E | M) · P (M) + P (E | V) · P (V) = = 0,25 · 0,25 + 0,85 · 0,60 + 0,10 · 0,15 = 0,5875. Mindezek alapj´an: P (V | E) =

xi

4 

P (V | E) P (E | V) · P (V) 0,10 · 0,15 0,0150 6 = = = = ≈ 0,0255. P (E) P (E) 0,5875 0,5875 235

Teh´at annak a val´osz´ın˝ us´ege, hogy Peches P´ al villamossal utazott:

1

P (V | E) ≈ 0,0255. 147

148

K¨ oz´ episkolai Matematikai ´ es Fizikai Lapok, 2022/3

c) Jel¨ olje C azt az esem´enyt, hogy legal´ abb k´et f˝ ot ellen˝ oriztek. Ekkor a C: az ¨ otf˝ os bar´ ati t´ arsas´ agb´ ol kett˝on´el kevesebb f˝ ot (azaz 0-t vagy 1-et) ellen˝ oriztek. A nem ellen˝ oriztek senkit a bar´ ati t´ arsas´ agb´ ol (esem´eny) val´ osz´ın˝ us´ege:

5 43 · 0  10 ≈ 0,2931. 48 10

azokat az eseteket ebb˝ol ki kell vonnunk, amikor 3-n´ al t¨ obb, korban egym´ast k¨ ovet˝o testv´er ker¨ ul tr´ onra, hiszen a csal´adon ´atok u ¨l, ´ıgy ezek nem lehets´egesek. Ezeket a kivonand´ o lehet˝ os´egeket esetekre bontottam aszerint, hogy mennyi a legnagyobb m sz´ ama azon testv´ereknek, akik korban egym´ast k¨ ovetik, ´es egym´ as ut´an uralkodnak. Ezek a testv´erek megkapj´ak az M c´ımk´et is. A t´ abl´ azatokban az x-szel jel¨ olt testv´erek uralkodnak, m´ıg a t¨ obbiek nem. 1 x

5 43 · 1  9 ≈ 0,4311. 48 10

Az ¨ otf˝ os bar´ ati t´ arsas´ agb´ ol kett˝on´el kevesebb f˝ ot (azaz 0-t vagy 1-et) ellen˝ oriztek val´osz´ın˝ us´ege: 5 43 5 43 · · 0  10 + 1  9 ≈ 0,7242. 48 48 10

2 x

3 x

4 x

5 x

6 x

7 x

8 x

Az els˝o esetben m = 8, teh´ at mindenki tr´ onra ker¨ ul, ami 1-f´elek´epp t¨ ort´enhet meg. A m´ asodik esetben m = 7, ami csak u ´gy m=8 lehets´eges, hogy az utols´ot kiv´eve mindenki uralkodik (hiszen az els˝o biztosan uralkodik). Ez szint´en 1 lehet˝ os´eg.

Az egy f˝ot ellen˝oriztek esem´eny val´ osz´ın˝ us´ege:

1 x

2 x

3 x

4 x

5 x

6 x

7 x

8

m=7

10

1 x x x

at annak a val´osz´ın˝ us´ege, hogy Mindezek alapj´an: P (C) = 1 − P (C) ≈ 0,2758. Teh´ az o tf˝ o s bar´ a ti t´ a rsas´ a gb´ o l legal´ a bb k´ e t f˝ o t ellen˝ o riztek: ¨ P (C) ≈ 0,2758.

Marczis Gy¨orgy (Gyula) Moln´ar Istv´an (Gyula) Moln´ar Judit (Gyula) R´ok´an´e R´ ozsa Anik´ o (B´ek´escsaba)

C gyakorlat megold´asa

C. 1685. Egy kir´ alyi csal´ ad nyolc gyermeke k¨ oz¨ ul a legid˝ osebb uralkodik. A testv´erek mindegyike pontosan akkor uralkodik, amikor ˝ o a legid˝ osebb m´eg ´el˝ o szem´ely k¨ oz¨ ul¨ uk. Viszont ezen a kir´ alyi csal´ adon a ´tok u arom testv´er, kik korban egy¨l: ha h´ m´ ast k¨ ovetik, mind tr´ onra ker¨ ulnek, akkor a r´ ak¨ ovetkez˝ o testv´er¨ uk meghal rem´enytelens´eg´eben. H´ anyf´elek´eppen uralkodhatnak, ha csak arra vagyunk tekintettel, hogy kik ker¨ ulnek tr´ onra a testv´erek k¨ oz¨ ul? I. megold´as. Ha nem u atok a kir´alyi csal´ adon, azaz uralkodhatn´anak a kor¨lne ´ ban egym´ast k¨ ovet˝o testv´erek k¨ oz¨ ul 3-n´ al t¨ obben is egym´as ut´an, akkor az els˝ o kiv´etel´evel minden gyermek vagy tr´ onra ker¨ ul, vagy nem (az els˝o uralkodik ´eppen, ez´ert ulhetn´enek tr´ onra. Viszont o biztosan tr´ ˝ onra ker¨ ul). ´Igy ekkor 27 = 128-f´elek´epp ker¨ K¨ oz´ episkolai Matematikai ´ es Fizikai Lapok, 2022/3

149

2 x x

3 x x x

4 x x x

5 x x x

6 x x x

7

x

8 x x

1 x x x x x x x

2 x x x x

x

3 x x x x x

4 x x x x x x x

5 x x x x x x x

6

7

x x x

x x x x x

8 x x x x

m=5

m=6

A harmadik esetben m = 6. Ekkor vagy az els˝ o 6 M c´ımk´ej˝ u, vagy az utols´o 6, hiszen ha a k¨oz´eps˝ o 6 lenne M , akkor k¨ oz¨ ul¨ uk az els˝ o a legid˝ osebb gyermek melletti, ez´ert m = 7 lenne. Ha az els˝ o 6 M c´ımk´ej˝ u, akkor az utols´ o gyermek vagy uralkodik, vagy nem, ami 2 lehet˝os´eg, m´ıg ha az utols´ o 6 az M c´ımk´ej˝ u, csak egy lehet˝os´eg van, mivel az els˝ o gyermek biztosan uralkodik. Ez ¨ osszesen 3 lehet˝ os´eg. A k¨ovetkez˝ o esetben m = 5. Ekkor M c´ımk´ej˝ u lehet az els˝ o 5, ekkor az utols´ o os´eg; a 3.-t´ ol a 7. gyermek, ekkor kett˝ o vagy uralkodik, vagy nem, ami 22 = 4 lehet˝ (az els˝ on k´ıv¨ ul) m´ as nem uralkodhat; vagy az utols´ o 5, amikor a 2. vagy uralkodik, vagy nem (2 lehet˝ os´eg). Ez ¨ osszesen 4 + 1 + 2 = 7 lehet˝ os´eg. 1 x x x x x x x x x

2 x

3 x x x

x x x

x x

4 x x x x x

5 x x x x x x x x

6 x x x x x x x x x

7 x

8 x x

x x x x x x

x x x x

Az utols´ o esetben m = 4. Ekkor ha az els˝ o 4 M c´ımk´ej˝ u, az utols´ o 3 vagy uralkodik, os´eg (ezekb˝ol vagy nem, ami 23 = 8-f´ele lehet˝ csak egyet t¨ untettem fel a t´ abl´ azatban). Ha a 3.-t´ ol a 6.-ig M c´ımk´ej˝ uek, az utols´o vagy uralkodik, vagy nem, ami 2-f´elek´epp t¨ ort´enhet. Ha M c´ımk´es a 4.-t˝ ol a 7., akkor a 2. vagy uralkodik, vagy nem, ez szint´en 2 lehet˝ os´eg. Ha pedig az utols´ o 4 uralkodik, akkor a 2. ´es a 3. vagy uralkodik, vagy nem, ami 22 = 4f´elek´epp val´ osulhat meg. Azaz m = 4 ¨ osszesen 8 + 2 + 2 + 4 = 16-f´elek´epp lehets´eges.

m=4

150

K¨ oz´ episkolai Matematikai ´ es Fizikai Lapok, 2022/3

Teh´ at ¨ osszesen 1 + 1 + 3 + 7 + 16 = 28-f´elek´epp lehet m > 3. ´Igy val´ oj´aban a testv´erek 128 − 28 = 100-f´elek´epp ker¨ ulhetnek tr´ onra.

Egyh´ azi Hanna (Budapest, ELTE Ap´ aczai Csere J. Gyak. Gimn., 12. ´evf.)

Ez alapj´an, ha kiv´ alasztjuk az A – B – C – D n´egy f˝ ob˝ ol ´ all´ o csoportot, tudjuk, hogy k¨oz¨ ul¨ uk legfeljebb 3 fog uralkodni. Ugyanez elmondhat´ o az E – F – G – H csoportra is, ez´ert legfeljebb (3 + 3 =) 6 f˝ o uralkodhat a 8 gyerek k¨ oz¨ ul. Adjuk ¨ossze a kapott lehets´eges eseteket:

II. megold´as. Jel¨olj¨ uk a 8 gyereket bet˝ ukkel ´eletkor szerint cs¨okken˝ o sorrendben: A, B, C, D, E, F , G, H. Tudjuk, hogy a legid˝ osebb, azaz A uralkodik el˝osz¨or. Bontsuk csoportokra azokat az eseteket, ahol k¨ ul¨onb¨oz˝ o sz´ am´ u gyerek uralkodik. Helyes uralkod´o kiv´alaszt´ as alatt olyan kiv´ alaszt´ ast ´ertek, ahol maximum 3, ´eletkorban egym´ast k¨ovet˝o szem´ely van kiv´alasztva. Ha helyesen kiv´alasztjuk az uralkod´ okat, akkor egy´ertelm˝ uen meghat´ aroztuk az esetet, mert a kiv´alasztott emberek ´eletkor szerint cs¨okken˝ o sorrendben fognak ul¨ onb¨ oz˝ o kiv´alaszt´ as k¨ ul¨onb¨oz˝ o esetet fog jelenteni. uralkodni. Ez´ert minden k¨ – Ha pontosan 1 f˝ o uralkodott a 8 f˝ o k¨ oz¨ ul: Mivel A biztosan el˝ osz¨or uralkodott, ez´ert itt csak 1 lehets´eges helyes kiv´ alaszt´ as van. – Ha pontosan 2 f˝ o uralkodott a 8 f˝ o k¨ oz¨ ul: Az els˝o uralkod´ o biztosan A, a m´asodik pedig b´arki lehet a t¨ obbi 7 gyerek k¨ oz¨ ul. Ez´ert ez 7 lehets´eges helyes kiv´ alaszt´ as. – Ha pontosan 3 f˝ o uralkodott a 8 f˝ ok¨ oz¨ul:7·6Az els˝o uralkod´o biztosan A, 7 alaszthata m´ asodik ´es a harmadik uralkod´ ot pedig 2 = 2! = 21-f´elek´eppen v´ juk ki. (Az A-n k´ıv¨ uli 7 f˝ob˝ ol 2-t kell kiv´alasztani u ´gy, hogy a kiv´alaszt´ as sorrendje nem sz´ am´ıt). Ezek k¨ oz¨ ul az esetek k¨ oz¨ ul mindegyik kiv´ alaszt´ as helyes, mert egyik esetben sem volt 4, ´eletkorban egym´ast k¨ ovet˝o uralkod´ o. 21 lehets´eges helyes kiv´ alaszt´ as van. – Ha pontosan 4 f˝ o uralkodott a 8 f˝ o k¨ oz¨ ul: Az els˝o uralkod´ o biztosan A. 7 = 35-f´ e lek´ e ppen v´ a laszthatjuk ki. Mivel A biztosan A m´asik 3 uralkod´ ot 3 = 7·6·5 3! a kiv´ alasztottak k¨ oz¨ ott van, ez´ert pontosan 1 eset lesz helytelen kiv´alaszt´ as, amikor a 4 kiv´alasztott f˝ o A, B, C ´es D. A t¨ obbi 34 esetben nem lesz 3-n´al t¨obb, ´eletkorban egym´ ast k¨ ovet˝o szem´ely a kiv´ alasztottak k¨ oz¨ ott. Azaz 34 helyes kiv´alaszt´ as van. – Ha pontosan 5 f˝ o uralkodott a 8 f˝ o k¨ oz¨ ul: Az els˝o uralkod´ o biztosan A. 7 = 35-f´ e lek´ e ppen v´ a laszthatjuk ki. A helytelen A m´asik 4 uralkod´ ot 4 = 7·6·5·4 4! kiv´ alaszt´ asok, amikor az A-n k´ıv¨ uli 4 uralkod´ o ´eletkorban egym´ast k¨oveti, ebb˝ol 4 eset van (B – C – D – E, C – D – E – F , D – E – F – G, E – F – G – H); illetve, amikor A miatt l´etezik egy 4 f˝ob˝ ol ´eletkorban egym´ast k¨ovet˝o szem´elyekb˝ ol a´ll´o uralkod´ o n´egyes, ebb˝ ol 3 eset van (A – B – C – D – F , A – B – C – D – G, A – B – C – D – H). alaszt´ as van. Vonjuk ki az ¨ osszes kiv´ alaszt´ asb´ol a helytelen (4 + 3 =)7 helytelen kiv´ kiv´ alaszt´ asokat, ´es megkapjuk a helyeseket: 35 − 7 = 28 helyes kiv´alaszt´ as van. – Ha pontosan 6 f˝ o uralkodott a 8 f˝ o k¨ oz¨ ul: Ekkor pontosan 2 f˝o nem fog uralkodni. Ha ˝oket meghat´ arozzuk, akkor egyben az uralkod´ o 6 f˝ot is meghat´ arozuk. A k´et f˝o lehet: B – E, B – F , C – E, C – F , C – G, D – E, D – F , D – G, D – H. Ez 9 eset, 9 helyes kiv´ alaszt´ as van. Egym´ ast k¨ ovet˝o ´eletkor´ u 3 gyerek eset´en meghal az ´eletkorban cs¨okken˝ o sorrendben r´ak¨ ovetkez˝ o szem´ely, ´ıgy elmondhat´ o, hogy b´ armely 4 egym´ ast k¨ovet˝o gyerek k¨ oz¨ ul legfeljebb 3 uralkodhat. K¨ oz´ episkolai Matematikai ´ es Fizikai Lapok, 2022/3

151

1 + 7 + 21 + 34 + 28 + 9 = 100. A kir´ alyi csal´ ad 8 gyermeke 100-f´elek´eppen uralkodhatott. Nagy Korina (Kecskem´eti B´ anyai J´ ulia Gimn., 9. ´evf.) Megjegyz´es. A honlapon ezekt˝ ol k¨ ul¨ onb¨ oz˝ o megold´ asok olvashat´ ok, azonban a versenyz˝ ok z¨ ome a k´et fenti megold´ asmenet egyik´et v´ alasztotta. 159 dolgozat ´erkezett. 5 pontos 77 versenyz˝ o. 4 pontos 22, 3 pontos 15, 2 pontos 8, 1 pontos 8, 0 pontos 9 dolgozat. Nem versenyszer˝ u 1 dolgozat. Nem sz´ am´ıtjuk a versenybe a sz¨ ulet´esi d´ atum vagy a sz¨ ul˝ oi nyilatkozat hi´ anya miatt: 1 dolgozat.

Matematika feladatok megold´asa

B. 5114. Az ABCDEF GH egys´egkock´ at elmetszett¨ uk egy s´ıkkal u ´gy, hogy az AB ´es AD ´eleket az A-t´ ol azonos, x t´ avols´ agra lev˝ o P ´es Q bels˝ o pontjaikban, a BF ´elt pedig az R pontban metszi. Mekkora a BR t´ avols´ ag, ha QP R = = 120◦ ? (4 pont) I. megold´as. A feladatban egys´egkocka szerepel, ez´ert a BP szakasz 1 − x hossz´ us´ag´ u. Hosszabb´ıtsuk meg a kocka CB ´el´et a B cs´ ucson t´ ul az 1 − x hossz´ us´ ag´ u szakasszal, legyen ez a pont S (1. ´ abra). A szakaszok egyenl˝ os´ege alapj´ an QAP ´es P BS egyenl˝ o sz´ ar´ u der´eksz¨ og˝ u h´ aat a Q, P ´es az S pontok egy egyenesbe romsz¨ogek, AP Q = BP S = 45◦ . Teh´ esnek. Ehhez hozz´ av´eve az R pontot, azt is l´ atjuk, hogy a Q, P , S ´es R pontok egy s´ıkban vannak. A P S szakasz felez˝omer˝ oleges s´ıkj´ara illeszkedik a BF ´el, ´ıgy P R = SR. A P SR egyenl˝ o sz´ ar´ u h´aromsz¨ og alapon fekv˝o RP S sz¨ og´enek k¨ uls˝ o sz¨ oogek 60◦ -osak, a P SR ge a feladat felt´etele alapj´ an 120◦ ; vagyis az alapon fekv˝o sz¨ h´aromsz¨og szab´alyos. V´eg¨ ul tekints¨ uk a BP S ´es BP R der´eksz¨ og˝ u h´ aromsz¨ ogeket. A BP befog´ ojuk k¨oz¨os, ´atfog´oik egyenl˝o hossz´ uak, teh´ at a k´et h´ aromsz¨ og egybev´ag´ o. Ezzel bel´attuk, hogy a BR szakasz BR = BS = 1 − x hossz´ us´ ag´ u. ´ Isk., 8. ´evf.) Varga Boldizs´ ar (Ver˝ oce, G´eza Fejedelem Ref. Alt. dolgozata alapj´an 152

K¨ oz´ episkolai Matematikai ´ es Fizikai Lapok, 2022/3

B. 5207. Bizony´ıtsuk be, hogy minden n  2 term´eszetes sz´ amra l´eteznek olyan amok, amelyekre 2  x1 < x2 < x3 < . . . < xn pozit´ıv eg´esz sz´ x1 ! · x2 ! · x3 ! · . . . · xn ! n´egyzetsz´ am. (4 pont)

Javasolta: Szoldatics J´ ozsef (Budapest)

Megold´as. Lemma. Minden 1-n´el nagyobb pozit´ıv eg´esz a-ra (a2 − 1)! · a2 ! n´egyzetsz´ am. 1. a ´bra

A lemma bizony´ıt´asa. Mivel a2 ! = a2 · (a2 − 1)!, ez´ert

2. a ´bra

II. megold´as. Ismert, hogy az AB, AD, DH, HG, GF ´es F B oldalak felez˝ opontjai egy szab´alyos hatsz¨ oget hat´ aroznak meg (hiszen egyenl˝ o hossz´ uak, a hatsz¨og szemk¨ oztes oldalai p´ arhuzamosak ´es 120◦ -ra forg´asszimmetrikus). Ez a hatsz¨ og megkaphat´o u ´gy is, ha vessz¨ uk a kocka felsz´ın´enek ´es a k¨oz´eppontj´ an ´ athalad´ o, CE f˝ o´ atl´ oj´ara mer˝ oleges s´ıkj´anak a metszet´et (2. ´ abra). K¨ onnyen l´ athat´ o, hogy a k¨ oz´eppont helyett m´as pontot v´eve a CE ´atl´ on szint´en olyan hatsz¨ og lesz a metszet (persze csak olyan esetben, ha az ´ıgy kapott s´ık metszi az oldalakat), melynek minden bels˝o sz¨ oge 120◦ -os. Ez amiatt van ´ıgy, mert az oldalai p´ aronk´ent p´ arhuzamosak lesznek azzal az esettel ¨osszevetve, amikor a k¨ oz´eppontot v´ alasztjuk ki, ´es a hatsz¨ og szab´alyos. Vegy¨ uk fel teh´ at azt a hatsz¨ oget ilyen m´ odon, melynek AB ´es AD oldalon l´ev˝ o P ´es Q cs´ ucsai x t´ avols´ agra vannak az A pontt´ ol. Ekkor a p´ arhuzamos eltol´ as miatt o sz´ ar´ u der´eksz¨ og˝ u h´ aromsz¨og, vagyis RP B = 45◦ , P BR egyenl˝ BR = 1 − x. M´ asik R pontra tudjuk, hogy nem lesz igaz, hogy QP R = 120◦ , hiszen ahogy az R pont t´ avolodik B-t˝ ol, u ´gy a QP R sz¨ og szigor´ uan monoton cs¨okken, ez´ert abbiak miatt ez csak egy olyan helyzet lehet, melyre ´eppen 120◦ a sz¨og ´es a kor´ pontosan az, amikor BR = BP = 1 − x. ´ Isk. ´es Gimn., 12. ´evf.) Hervay Bence (Budapesti Fazekas M. Gyak. Alt. Megjegyz´es: A feladat r¨ ovid sz´ amol´ assal is megoldhat´ o. BR hossz´ at, mint v´ altoz´ ot bevezetve, Pitagorasz-t´etellel BR-b˝ ol ´es x-b˝ ol kifejezhet˝ o a P Q, P R ´es RQ szakaszok hossza, majd a P QR h´ aromsz¨ ogre fel´ırt koszinuszt´etelben a QP R sz¨ og koszinusza −0,5. Kapunk egy param´eteres m´ asodfok´ u egyenletet BR-re, melynek egyetlen 0 ´es 1 k¨ oz¨ otti megold´ asa 1 − x. ´ Isk. ´es Gimn., 12. ´evf.) Hervay Bence (Budapesti Fazekas M. Gyak. Alt. ¨ Osszesen 106 dolgozat ´erkezett. 4 pontot kapott 84 versenyz˝ o, 3 pontos 10, 2 pontos 3, 1 pontos 3 tanul´ o dolgozata. 0 pontot kapott 1 tanul´ o. Nem versenyszer˝ u 5 dolgozat. K¨ oz´ episkolai Matematikai ´ es Fizikai Lapok, 2022/3

153

 2  2 (a2 − 1)! · a2 ! = (a2 − 1)! · a2 · (a2 − 1)! = a2 · (a2 − 1)! = a · (a2 − 1)! . Most r´ at´er¨ unk a feladat ´all´ıt´ as´ anak bizony´ıt´ as´ ara.

1. eset: Ha n p´ aros, akkor az x1 < x2 < x3 < . . . < xn sz´ amokat p´ arokba rendezem u ´gy, hogy minden x2k az x2k−1 -gyel ´all p´arban, ahol x2k a (k + 1)-edik pozit´ıv al 1-gyel kisebb sz´ am. Min´egyzetsz´am ´es x2k−1 a (k + 1)-edik pozit´ıv n´egyzetsz´amn´ vel v´egtelen sok n´egyzetsz´am van, ez´ert mindig ki tudunk k darab n´egyzetsz´amot v´alasztani. aEkkor a lemma szerint mindegyik x2k−1 ! · x2k ! szorzat n´egyzetsz´am, ez´ert a p´ rok szorzata is n´egyzetsz´am. 2. eset: Ha n p´ aratlan, akkor legyen x1 = 4, x2 = 5, x3 = 6. Ezek faktori´alisainak szorzata: 4! · 5! · 6! = 210 · 34 · 52 , amelynek pr´ımt´enyez˝ os felbont´as´ aban az ¨ osszes kitev˝ o p´ aros, ´ıgy n´egyzetsz´am, ezzel n = 3-ra k´eszen vagyunk. Ha n  5, akkor legyen tov´abbra is x1 = 4, x2 = 5, ol xn -ig szint´en p´ arba rendezz¨ uk a sz´ amokat az el˝oz˝ o esetben mux3 = 6, ´es x4 -t˝ arral kezdve, ut´ ana pedig minden x2k+1 tatott m´ odszerrel, az x4 = 8, x5 = 9 p´ a (k + 1)-edik pozit´ıv n´egyzetsz´am, minden x2k pedig 1-gyel kisebb x2k+1 -n´el, ´ıgy az x1 ! · x2 ! · x3 ! · . . . · xn ! szorzat ebben az esetben is n´egyzetsz´am. Ezzel a feladat ´all´ıt´ as´ at mindk´et esetben bel´ attuk.

Andai M´ ark (Tata, E¨ otv¨ os J. Gimn., 10. ´evf.) ¨ Osszesen 131 dolgozat ´erkezett. 4 pontos 99, 3 pontos 17, 2 pontos 8 dolgozat. 1 pontot 3 versenyz˝ o kapott. Nem versenyszer˝ u: 4 dolgozat.

B. 5212. Igazoljuk, hogy l´etezik olyan pozit´ıv eg´esz sz´ am, amely legal´ abb 2021f´elek´eppen ´ all´ıthat´ o el˝ o u ´gy, hogy egy (t´ızes sz´ amrendszerben fel´ırt) pozit´ıv eg´esz sz´ amhoz hozz´ aadjuk a sz´ amjegyeinek o ¨sszeg´et. (6 pont)

Javasolta: S´ andor Csaba (Budapest)

Megold´as. Teljes indukci´ oval fogunk bizony´ıtani. A legnagyobb sz´am, ami nu lesz, a legnagyobb sz´ amb´ol val´o el˝ o´ all´ıt´ asa f´elek´eppen el˝o´ all, mindig 10k + 1 alak´ 154

K¨ oz´ episkolai Matematikai ´ es Fizikai Lapok, 2022/3

pedig az, ha 10k -hoz adjuk hozz´a sz´amjegyeinek ¨ osszeg´et, ami 1 (enn´el nagyobb sz´ amhoz a sz´ amjegyei ¨ osszeg´et adva biztos nagyobbat kapunk, ez´ert mondhatjuk, hogy ez a legnagyobb sz´amb´ ol val´ o el˝ o´ all´ıt´ as). o´all´ıt´ as. n = 2-re 102 + 1 = 100 + 1 = 91 + 9 + 1 a k´et el˝ Tegy¨ uk fel, hogy van egy 10k + 1 sz´ am, ami n k¨ ul¨onb¨oz˝ o sz´amb´ol el˝o´all´ıthat´ o, atni, hogy van egy amelyek k¨ oz¨ ul a legnagyobb 10k . Azt akarjuk teh´at most bel´ am, ami (n + 1)-f´elek´eppen is el˝o´all. olyan 10K + 1 sz´ Minden eddigi sz´amhoz, ami a 10k + 1 k¨ ul¨ onb¨oz˝ o el˝oa´ll´ıt´asaiban szerepelt, adjuk hozz´ aa k

9 · 10k+1 + 9 · 10k+2 + 9 · 10k+3 + . . . + 9 · 10k+10 = 1010

k

+k+1

− 10k+1

sz´amot. Mivel a 10k + 1 el˝ o´ all´ıt´ asaiban szerepl˝ o legnagyobb sz´ am a 10k volt, ´ıgy k+1 -es helyi´ert´ekt˝ol ezekkel a sz´ amokkal annyi t¨ ort´ent, hogy a sz´ amok el´e a 10 k kezdve a 10k+10 -es helyi´ert´ekig 9-eseket ´ırtunk (´es ut´anuk esetleg valah´any 0-t), ez 10k darab 9-es. ´Igy az ¨ osszes eddigi n sz´ am ´es sz´ amjegyeinek ¨osszege a hozz´ aadott ´j sz´ ammal ´es annak sz´ amjegyeinek ¨ osszeg´evel, 9 · 10k -nal n˝ott, ´ıgy minden kapott u sz´am ´es sz´ amjegyeinek ¨ osszege   k k (10k + 1) + 1010 +k+1 − 10k+1 + 9 · 10k = 1010 +k+1 − 10k+1 + 10 · 10k + 1 = = 1010

k

+k+1

K. 725. Egy 3 × 3-as t´ abl´ azat kilenc mez˝ oj´ere valamilyen sorrendben egy-egy sz´ amot ´ırunk a k¨ ovetkez˝ o szab´ aly szerint: minden mez˝ ore azt a sz´amot ´ırjuk, amely megmutatja, hogy annak a mez˝ onek h´ any olyan oldalszomsz´edja van, amire m´ ar ´ırtunk sz´amot. Milyen sorrendben t¨ olt¨ ott¨ uk ki a t´ abl´ azat mez˝ oit? H´ any lehet˝os´eg van? (A mez˝ oket a1, a2, . . . , c2, c3 k´ odokkal jel¨ olj¨ uk.)

+ 1.

10k +k+1

Ez a sz´ am pedig u ´gy is el˝o´ all, ha a 10 sz´ amhoz hozz´ aadjuk sz´ amjegyeam, ami (n + 1)-f´elek´eppen el˝o´all: inek ¨ osszeg´et, 1-et. Vagyis a keresett 10K + 1 sz´ k 1010 +k+1 + 1. ´Igy minden pozit´ıv eg´esz n-re l´etezik olyan sz´am, ami n-f´elek´eppen el˝o´all egy sz´ am ´es a sz´ amjegy¨ osszeg´enek ¨ osszead´ as´ aval, vagyis n = 2021-re is.

K. 726. Rendezz¨ uk el az 1, 2, 3, 4, . . . , 31, 32 sz´amokat egy k¨ or ment´en u ´gy, hogy b´ armely k´et szomsz´edos sz´am ¨ osszege n´egyzetsz´am legyen. ´Irjuk le azt is, hogy hogyan gondolkoztunk. K/C. 727. Egy n × n-es t´ abl´ azat mez˝ oire egy-egy p´enz´erm´et helyez¨ unk el u ´gy, hogy mindegyik ´erm´en a fej” van fel¨ ul. Egy l´ep´esben b´ armelyik sorban vagy ” oszlopban pontosan h´arom ´erm´et ford´ıthatunk meg, ´ıgy azokon a fejb˝ ol ´ır´ as lesz, az ´ır´asb´ ol pedig fej. El´erhetj¨ uk-e ´ıgy valah´any l´ep´esben, hogy minden ´erm´en ´ır´ as legyen fel¨ ul, ha n > 2? V´ alaszunkat indokoljuk. K/C. 728. Van 10 darab sz´amk´ arty´ ank, rajtuk az 1, 2, 3, 4, 5, 6, 7, 8, 9, 10 amusz´ amok. A sz´ amk´ arty´ akat letessz¨ uk egy sorba az asztalra ´es r´ ajuk ´ırjuk a sorsz´ kat, azaz 1-t˝ ol 10-ig besz´ amozzuk a lapokat. ´Igy minden lapon k´et sz´ am szerepel. Minden lapon ¨osszeszorozzuk a k´et sz´ amot, majd a szorzatokat ¨ osszeadjuk. Mennyi lesz a kapott ´ert´ek, a) ha ez a lehet˝ o legkisebb, b) ha ez a lehet˝ o legnagyobb?

J´ anosik M´ at´e (Gy˝ or, R´evai Mikl´ os Gimn., 12. ´evf.)

Beku ¨ld´esi hat´arid˝o: 2022. ´aprilis 10. Elektronikus munkafu ¨zet: https://www.komal.hu/munkafuzet

42 dolgozat ´erkezett. 6 pontos 34, 5 pontos 1, 4 pontos 3, 3 pontos 1, 0 pontos 1 dolgozat. Nem sz´ am´ıtjuk a versenybe a sz¨ ulet´esi d´ atum vagy a sz¨ ul˝ oi nyilatkozat hi´ anya miatt: 2 dolgozat.

A K pontversenyben kit˝ u z¨ ott gyakorlatok ABACUS-szal k¨ oz¨ os pontverseny 9. oszt´alyosoknak (724–728.)

A C pontversenyben kit˝ uz¨ott gyakorlatok (727–728., 1709–1713.)

K. 724. Juli felv´agott egy pizz´ at egyforma szeletekre. Ezut´ an n´eh´ any szeletet megevett, 3 szelet viszont megmaradt. Kicsit sz´amolgatva azt vette ´eszre, hogy az eg´esz pizza 3/4 r´esz´et plusz egy szelet 3/4 r´esz´et ette meg. H´ any szeletre v´agta a pizz´ at? K¨ oz´ episkolai Matematikai ´ es Fizikai Lapok, 2022/3

155

Feladatok 10. ´evfolyamig K/C. 727. A sz¨ oveg´et l´ asd a K feladatokn´ al. K/C. 728. A sz¨ oveg´et l´ asd a K feladatokn´ al. 156

K¨ oz´ episkolai Matematikai ´ es Fizikai Lapok, 2022/3

Feladatok mindenkinek

B. 5231. Bizony´ıtsuk be, hogy minden n pozit´ıv eg´eszre teljes¨ ul, hogy

C. 1709. Az a ´es b eg´esz sz´ amok oszt´ oi a 720-nak, ab pedig nem oszt´ oja 720nak. H´ any ilyen rendezett (a; b) sz´ amp´ ar van?

n 

Javasolta: R´ oka S´ andor Ny´ıregyh´ aza) C. 1710. Egy egys´egnyi oldal´ u n´egyzetbe n´egy k¨ ort rajzolunk az ´ abr´ an l´ athat´ o m´odon. A k´et nagyobbik k¨ or egyforma m´eret˝ u ´es ´erintik egym´ast ´es a n´egyzet oldalait is. A k´et kisebbik egybev´ ag´ o, ezek szint´en ´erintik a n´egyzet oldalait ´es a nagy k¨ or¨ oket is. Mekkora a k¨ or¨ ok k¨ oz´eppontjai a´ltal meghat´ arozott rombusz ter¨ ulete?

C. 1711. Oldjuk meg az  √ x − 1801 + y − 1860 = 2 − √

egyenletet, ha x ´es y val´os sz´amok.

k=1

k=1

2n−k · (2k − 1).

B. 5232. Az ABC hegyessz¨ og˝ u h´aromsz¨ og belsej´eben, a C-b˝ ol indul´o s´ ulyvonalon vegy¨ uk fel a P pontot u ´gy, hogy AP B = 180◦ − ACB teljes¨ ulj¨ on. Igazoljuk, hogy az AB egyenes ´erinti az AP C k¨ ort. (4 pont)

Javasolta: Kocsis Szilveszter (Budapest)

B. 5233. Egy szab´ alyos hatsz¨ og cs´ ucsaira v´eletlenszer˝ u sorrendben fel´ırjuk az 1, 2, . . . , 6 sz´ amokat. Ezut´ an a hatsz¨ og minden oldal´ara r´ a´ırjuk a k´et v´egpontj´ an szerepl˝o sz´ amok k¨ ul¨ onbs´eg´enek abszol´ ut´ert´ek´et. Hat´ arozzuk meg az oldalakra ´ırt hat sz´ am ¨osszeg´enek v´ arhat´ o ´ert´ek´et. 1 x − 1801

(4 pont)

Feladatok 11. ´evfolyamt´ol

Javasolta: K´ arolyi Gergely (Budajen˝ o) C. 1713. Az x ´es a olyan val´os sz´amok, amelyekre teljes¨ ul, hogy x + x1 = a.

Hat´ arozzuk meg a f¨ uggv´eny´eben az x13 + x113 ´ert´ek´et.

Javasolta: Szoldatics J´ ozsef (Budapest)

B. 5234. Az n pozit´ıv eg´esz sz´ amot nevezz¨ uk mitikusnak, ha minden oszt´ oja 2-vel kisebb egy pr´ımsz´amn´ al. P´eld´ aul a 15 mitikus sz´ am. Legfeljebb h´ any oszt´oja amot, amelynek lehet egy mitikus sz´amnak? Adjuk meg az ¨ osszes olyan mitikus sz´ maxim´alis sz´ am´ u oszt´ oja van. (5 pont)

Javasolta: R´ oka S´ andor (Ny´ıregyh´ aza)

B. 5235. Mutassuk meg, hogy a Fibonacci-sorozatban minden 3-n´ al nagyobb pr´ımsz´ am 4k + 1 alak´ u. (5 pont) B. 5236. Legyen a, b, c h´ arom pozit´ıv val´ os sz´am u ´gy, hogy abc = 1. Mutassuk meg, hogy

Beku ¨ld´esi hat´arid˝o: 2022. ´aprilis 10. Elektronikus munkafu ¨zet: https://www.komal.hu/munkafuzet

2

a + a2 + a3 + b + b2 + b3 + c + c2 + c3  (a2 + b2 + c2 ) .

 (6 pont)

Javasolta: Lovas M´ arton (Budapest) ´es Michael Rozenberg (Izrael)

B. 5237. Egy h´ aromsz¨ ogben r a be´ırt k¨ or sugar´at, R a k¨ or´e ´ırt k¨ or sugar´at, s pedig a h´ aromsz¨ og f´elker¨ ulet´et jel¨ oli. Mutassuk meg, hogy ha r + 2R = s, akkor a h´aromsz¨og der´eksz¨ og˝ u.

A B pontversenyben kit˝ u zo ¨tt feladatok (5230–5237.)

(6 pont)

Beku ¨ld´esi hat´arid˝o: 2022. ´aprilis 10. Elektronikus munkafu ¨zet: https://www.komal.hu/munkafuzet



Javasolta: Sur´ anyi L´ aszl´ o (Budapest) 157

Javasolta: Fridrik Rich´ ard Szeged)



B. 5230. Az AB ´atm´er˝ oj˝ u f´elk¨ or´ıven kijel¨ olt¨ uk a C ´es D pontokat. Az A ´es B pontb´ ol a CD egyenesre a´ll´ıtott mer˝ olegesek talppontj´ at jel¨olje A , illetve B  .   Bizony´ıtsuk be, hogy az A C ´es B D szakaszok hossza egyenl˝o.

K¨ oz´ episkolai Matematikai ´ es Fizikai Lapok, 2022/3

n 

(4 pont)

C. 1712. Mekkor´ ak lehetnek annak az o ognek az ismeretlen sz¨ogei, melynek ¨tsz¨ minden oldala egyforma hossz´ us´ ag´ u ´es van k´et der´eksz¨oge?

(3 pont)

k · 2k−1 =

158

K¨ oz´ episkolai Matematikai ´ es Fizikai Lapok, 2022/3

azonos, v´eg¨ ul a k¨ ovetkeztet´es m˝ uvelet hamis, ha az, amir˝ ol k¨ ovetkeztet¨ unk igaz, de ¨ az, amire k¨ovetkeztet¨ unk hamis. Osszefoglalva:

Az A pontversenyben kit˝ u z¨ ott nehezebb feladatok (821–823.) A. 821. a) L´etezik-e olyan f : N2 → N f¨ uggv´eny, melyre minden g : N → N   f¨ uggv´eny ´es m pozit´ıv eg´esz eset´en l´etezik n ∈ N, melyre a k ∈ N : f (n, k) = g(k) halmaz elemsz´ama legal´abb m? uggv´eny, melyre minden g : N → N f¨ uggv´eny b) L´etezik-e olyan f : N2→ N f¨ eset´en l´etezik n ∈ N, melyre a k ∈ N : f (n, k) = g(k) halmaz elemsz´ama v´egtelen?

V´egezet¨ ul a tagad´ asok okozta cser´eket is rendszerezz¨ uk:

A. 822. L´eteznek-e p, q, r racion´ alis sz´amok, melyekre p + q + r = 0 ´es pqr = 1?

Javasolta: Weisz M´ at´e (Cambridge)  A. 823. Legyen n pozit´ıv eg´ esz, ´es tekints¨ uk az Sn = (x, y, z) : 1  x  n,  1  y  n, 1  z  n, x, y, z ∈ N kockar´ acsot. L´etezik-e√olyan n pozit´ıv eg´esz, oz¨ ul t¨ obb, mint n n-t u ´gy, hogy b´ armely melyre ki lehet v´ alasztani Sn elemei k¨ k´et kiv´ alasztott r´ acspont k¨ oz¨ ul az egyiknek legal´abb k´et koordin´ at´ aja szigor´ uan nagyobb legyen, mint a m´ asik megfelel˝o k´et koordin´ at´ aja? Javasolta: Cs´ oka Endre (Budapest) Beku ¨ld´esi hat´arid˝o: 2022. ´aprilis 10. Elektronikus munkafu ¨zet: https://www.komal.hu/munkafuzet

amit tagadunk A B A ´es B a m˝ uvelet



A matematikai logika logikusabb, mint gondoln´ank II.

bet˝ ucsere (BCS)

Az o ol a sz´am´ıt´ og´epig ¨sszetett ´all´ıt´asokt´ A formaliz´ al´as ban´ alisan egyszer˝ u, a szomsz´edos ´ abra a modell. A bal fels˝o r´eszbe a bal oldalon l´ev˝ o kifejez´es (A ∧ B), a jobb fels˝o r´eszbe pedig a jobb oldali (B → A) logikai sakkt´ abl´ aban tal´alhat´ o ´ert´ekei ker¨ uljenek, ezeket a k¨ oz´eps˝ o mez˝o n´egy szegmens´enek bal ´es jobb fels˝o r´esz´ebe m´ asoljuk ´ at, k¨ oz´epre pedig a k´et oldal k¨ oz¨ otti m˝ uvelet jel´et (⊗) illessz¨ uk be. Ezek ut´ an szegmensenk´ent k¨ ul¨ onk¨ ul¨ on ´ert´ekelj¨ uk ki a m˝ uveleti szab´ alyokban foglaltak alapj´ an: i ⊗ i (= h), h ⊗ h (= h), h ⊗ i (= i) ´es h ⊗ i (= i).

Ha alaposan megfigyelj¨ uk a cikk els˝o r´esz´eben kapott a´br´ at, egy tov´ abbi nyeres´eget is elk¨ onyvelhet¨ unk: ahogy az algebr´aban, u ´gy a logik´ aban is vannak azonoss´ agok. Ezek egyike-m´ asika k¨ ozismert, ahogy az algebr´aban is vannak neveze2 aban ilyen a ¬(A ∧ B) = tes azonoss´ agok, p´eld´ aul (a + b) = a2 + 2ab + b2 . A logik´ = ¬A ∨ ¬B, vagy (A → B) ∧ (B → A) = A ⇔ B.

Ez ut´obbi kapcs´ an felmer¨ ul, hogy mik´ent lehet ¨osszetettebb logikai kifejez´eseket egyszer˝ ubb alakra hozni. Nos, ez egy´ altal´ an nem bonyolult. A k¨onnyebb meg´ert´es kedv´e´ert foglaljuk t´abl´ azatba az ¨ ot alapm˝ uvelet eredm´eny´enek szab´ aly´ at. El´eg csak az egyik ´allapotot megjegyezni, a t¨ obbi esetben mindig a tagad´ asa lesz az eredm´eny: az ´es m˝ uvelet igaz, ha mind a k´et r´esz´ all´ıt´ as igaz, a vagy m˝ uvelet hamis, ha mind a k´et r´esz´ all´ıt´ as hamis, a kiz´ ar´ o vagy m˝ uvelet igaz, ha a k´et r´esz´ all´ıt´ as logio, az azonoss´ ag m˝ uvelet igaz, ha a k´et r´esz´ all´ıt´ as logikai ´ert´eke kai ´ert´eke k¨ ul¨ onb¨oz˝ K¨ oz´ episkolai Matematikai ´ es Fizikai Lapok, 2022/3

mozgat´as oszlopcsere (OCS) sorcsere (SCS) ´ atl´ ´ os csere (ACS)

159

160

K¨ oz´ episkolai Matematikai ´ es Fizikai Lapok, 2022/3

Az eredm´enyt r¨ ogz´ıts¨ uk az eddig u ¨res cell´ akba, majd a logikai sakkt´abl´ aban keress¨ uk ki, hogy ez melyik m˝ uvelet ´ert´ekt´ abl´ azata (¬B).

M´ asik magyar´az´ o p´eldak´ent ´alljon itt az (A → B) ∧ (B → A) = A ⇔ B nevezetes logikai azonoss´ ag igazol´asa.

A v´ allalkoz´ o kedv˝ ueknek aj´ anlok n´eh´ any feladatot: 1. (A ∧ B) ∨ (A ⊗ B). 2. (A ∨ B) ∧ (B → A). 3. ¬(B → A) ⊗ A. 4. (A ⇔ B) ⇔ ¬A. 5. (A ∨ ¬B) → ¬(¬A ∨ B).  6. ¬ A → (A ⊗ B) .

  7. ¬B ⊗ (A ∧ B) ∧ B → ¬(A ∧ B) .

Ideje r´ at´ern¨ unk arra, hogy ez az eg´esz mik´ent kapcsol´ odik a sz´ am´ıt´ og´epekhez. ´ A sz´am´ıt´ og´epek elektromos ´arammal m˝ uk¨ odnek. Es nem csak energiaforr´ ask´ent haszn´ alj´ak az elektromos energi´at, az inform´aci´ ot is elektromosan tov´abb´ıtj´ ak. Feldolgoz´ askor a leggyorsabb eld¨ ont´esi technika a nyer˝ o, m´arpedig az elektromos technol´ogi´aban a folyik vagy nem folyik ´ aram eld¨ ont´ese l´enyegesen gyorsabb ´es m´eg egyszer˝ ubb is, mint az ´aram nagys´ ag´ at megm´erni. Ez k´et ´ allapot megk¨ ul¨ onb¨ oztet´es´et jelenti, teh´at a k´et´ allapot´ u rendszerek haszn´ alata gyorsabb ´es egyszer˝ ubb is.

N´ezz¨ uk meg egy olyan kifejez´es ki´ert´ekel´es´et, ahol az el˝obbieket kicsit m´ odos´ıtva jutunk el a megold´ ashoz. Egyszer˝ us´ıts¨ uk le a

Ilyen rendszert kett˝ ot is ismer¨ unk, a matematikai logika igaz/hamis rendszere ´es a kettes sz´ amrendszer, ahol k´et k¨ ul¨ onb¨ oz˝ o sz´ amjegy, a 0 ´es az 1 l´etezik. M´ ar az elektronika hajnal´ an k´epesek voltak olyan a´ramk¨ or¨ oket k´esz´ıteni, amelyek mouveleteket, ezeket logikai kapuknak nevezik. A teljess´eg ig´enye dellezik a logikai m˝ n´elk¨ ul eml´ıts¨ unk meg n´eh´ anyat. A tagad´ as megfelel˝oje az egy bemenettel ´es egy kimenettel rendelkez˝ o NOT kapu, a kimenet´en folyik a´ram, ha a bemenet´en nem, ´es a kimeneten nem folyik a´ram, ha a bemeneten igen. A k´et bemenettel ´es egy kimenettel rendelkez˝ o AND, OR, XOR kapu pedig az ´es, a vagy ´es a kiz´ ar´ o vagy m˝ uvelet szab´ alya szerint m˝ uk¨ odik. A sz´am´ıt´ og´ep neve a sz´ am´ıt´ asra ´es nem a matematikai logik´ ara utal, hiszen akkor tal´ an it´el˝ og´epnek h´ıvn´ ank. De az a´ll´ıt´ asok ki´ert´ekel´ese a mindennapokban sokkal kisebb szerephez jut, mint a sz´ am´ıt´ asok elv´egz´ese. Abba azonban ritk´ an gondolunk bele, hogy a kettes sz´ amrendszerben mennyire egyszer˝ u a m˝ uveletek elv´egz´ese, ez´ert ehhez mondan´ek n´eh´ any adal´ekot. A kettes sz´amrendszerben az ¨ osszead´ as ´es a szorz´ as szab´alya is el´eg egyszer˝ u ar egy sorban el is mondhat´o: 0 + 0 = 0, 0 + 1 = 1, 1 + 0 = 1 egy sz´ amjegy eset´en, ak´ ´es 1 + 1 = 0, valamint 0 · 0 = 0, 0 · 1 = 0, 1 · 0 = 0 ´es 1 · 1 = 1. Az algebra szab´alyai f¨ uggetlenek a sz´ amrendszert˝ ol: a sz´ amrendszer alapsz´am´anak k-adik hatv´ any´ aval szorz´as k darab 0 sz´ amjegynek a sz´ am v´eg´ere ´ır´ as´ at jelenti, ´ıgy p´eld´ aul kettes sz´amrendszerben az 101 2 (= 5 10 ) k´etszerese, n´egyszerese, nyolcszorosa: 1010 2 , 10100 2 ´es 101000 2 .

¬(B → ¬A) ∧ (¬B → ¬A) kifejez´est. Els˝ ore tal´ an ijeszt˝onek t˝ unhet, de a tagad´asokat k¨ onnyen megoldhatjuk cser´ekkel. Egyszer ´erdemes belegondolni, hogy ez mit is jelent. A cikk el˝ oz˝ o r´esz´eben az A ´ all´ıt´ as az volt, hogy Kedd van, a B ´ all´ıt´ as pedig az, hogy Esik az es˝ o. Ezeket felhaszn´alva ez a kifejez´es ´ıgy sz´ ol: Nem igaz az, hogy, ha esik az es˝ o, akkor nem kedd van, ´es ha nem esik az es˝ o, akkor nem kedd van. Leegyszer˝ us´ıtve: Kedd van ´es esik az es˝ o.

K¨ oz´ episkolai Matematikai ´ es Fizikai Lapok, 2022/3

161

162

K¨ oz´ episkolai Matematikai ´ es Fizikai Lapok, 2022/3

A kettes sz´ amrendszerben a szorz´ as k¨ onnyen kiv´ althat´ o ¨osszead´ assal. Az egyik t´enyez˝ o kettes sz´amrendszerbeli alakja pont azt mutatja meg a k¨ ul¨onb¨oz˝ o helyi´ert´ekeken a´ll´ o 1 sz´ amjegyeivel, hogy a m´asik t´enyez˝ onek melyek azok kett˝ ohatv´ anyast. szorosai, amelyeket ¨ ossze kell adnunk. N´ezz¨ uk meg a 9 · 11 10 szorz´

´ a mikroprocesszor icipici logikai kapuk milEs li´oit hordozza, ´es t¨ obbsz´ az milli´ard m˝ uveletre k´epes m´ asodpercenk´ent, ez´ert tudnak ezek a g´epek t¨ obbf´ele, az ember sz´ am´ara bonyolult vagy unalmas munk´at kiv´altani.

am egyszeres´et ´es nyolcszoros´ at kell o¨ssze1001 · 1011 2 . Ebben az 1011 2 sz´ adni, hiszen 9 = 1 + 8 10 . Csak h´arom null´at kell a v´eg´ere biggyeszteni, hogy

Ez´ert is ´erdemes legal´ abb egy f˝ohajt´ assal tisztelegni George Boole (1815. november 2. – 1864. december 8.) angol matematikus el˝ ott, aki lerakta a matematikai logika alapjait. Az arck´ep forr´ asa: https://www.sciencephoto.com/media/ 223560/view/english-mathematiciangeorge-boole. T´ oth Tam´as Budapest

meglegyen a nyolcszoros, ´es azt az eredetivel ¨ osszeadni. 101000 2 + 1011 2 .

Ez val´oban nagy k¨ onnyebbs´eg, csak ki kell dolgozni azokat az ´aramk¨or¨oket, amelyek kettes sz´ amrendszerben elv´egzik az ¨ osszead´ ast. Vagy m´egsem? Nem bizony. El´eg csak szemrev´etelezni az al´abbi ´ abr´ at, ´es felt˝ unik, hogy az im´ent eml´ıtett AND ´es XOR logikai kapuk lehet˝ ov´e teszik az ¨ osszead´ ast. Teh´ at nem kell ezeket kidolgozni, hiszen ezek a logikai kapuk m´ ar megvannak.

Ha el´eg sok, el´eg apr´ o ´es el´eg gyors logikai kaput tudunk ¨osszedolgozni, akkor asn´ al l´enyegesen ¨ osszetettebb m˝ uveleteket is el tudunk v´egezni. a 9 · 11 10 szorz´

K¨ oz´ episkolai Matematikai ´ es Fizikai Lapok, 2022/3

163

Informatik´ab´ ol kit˝ uz¨ott feladatok

I. 559. Egy sz´ am´ıt´ og´epes j´ at´ekban k´et h´ aromf˝ os csapat j´atszik egym´ assal. A j´at´ekosok a kor´ abbi j´ atszm´ak eredm´enyei alapj´ an pontsz´amokkal rendelkeznek, melyeket a gy˝ozelmeik ´es veres´egeik alapj´an sz´am´ıt ki a j´ at´ekprogram. A kapott pontok minden esetben pozit´ıv eg´esz sz´ amok. A sz´ am´ıt´ og´epes j´at´ekban egy j´atszm´aba hat j´at´ekos jelentkezik be. Kezdetben az els˝ o h´ arom bejelentkez˝ o j´ at´ekos az els˝ o, a m´ asik h´ arom pedig a m´asodik csapatba ker¨ ul. A program igyekszik a pontsz´amok alapj´an egyenl˝o er˝ oss´eg˝ u csapatokat l´etrehozni. A csapatok er˝oss´eg´et a j´ at´ekosok pontsz´am´anak ¨ osszeg´evel adjuk meg. A csapatok eloszt´ as´ at u ´gy v´egzi a program, hogy legf¨ oljebb egy j´ at´ekost az egyik csapatb´ ol kicser´el egy m´ asik j´ at´ekosra a m´ asik csapatb´ ol. A j´ atszm´aban a 6 j´at´ekos oljebb egy ilyen cser´evel a k´et csapat er˝oss´ege u ´gy alkot k´et csapatot, hogy legf¨ a lehet˝o legkev´esb´e t´erjen el egym´ast´ ol. K´esz´ıts¨ unk programot, amely a 6 j´ at´ekos pontsz´ ama alapj´an megad egy eloszt´ast. Ha t¨obb ilyen eloszt´as lehets´eges, akkor b´ armelyik megadhat´ o. A bemenet egyetlen sor´ aban a j´at´ekosok pontsz´ama szerepel egy-egy sz´ ok¨ ozzel elv´ alasztva. Az i-edik sz´ am az i-edik j´ at´ekos pontsz´ ama. A kimenet egyetlen sor´ aban a csapatok eloszt´asa szerepel: az els˝ o h´ arom sz´am az egyik csapat, m´ıg a m´asodik h´arom sz´am a m´ asodik csapat tagjait jelenti. Amennyiben nem sz¨ uks´eges cser´elni, akkor az 1 2 3 4 5 6 sz´ amsorozatot kell ki´ırni sz´ ok¨ozzel elv´ alasztva. Ha t¨ ort´ent csere, akkor a megfelel˝ o helyen l´ev˝ o sorsz´ amokat kell cser´elni. 164

K¨ oz´ episkolai Matematikai ´ es Fizikai Lapok, 2022/3

Bemenet 125 68 93 77 83 119

Kimenet 6 2 3 4 5 1

Magyar´ azat: az 1. j´at´ekost cser´elt¨ uk a 6. j´at´ekosra. ´Igy a 6., a 2. ´es a 3. j´at´ekos alkotja az egyik csapatot, melynek er˝ oss´ege 119 + 68 + 93 = 280, valamint a 4., 5. ´es 1. j´at´ekos alkotja a m´asik csapatot, melynek er˝ oss´ege 77 + 83 + 125 = 285. Bek¨ uldend˝o egy i559.zip t¨ om¨ or´ıtett a´llom´anyban a forr´ asprogram ´es egy r¨ ovid dokument´ aci´ o, amely megadja, hogy a program melyik fejleszt˝oi k¨ornyezetben futtathat´ o. I. 560. A hagyom´ anyos keresztrejtv´enyben a szavak elv´ alaszt´ as´ ara fekete mez˝ ok szolg´alnak. A megfejt´esekkel az u ok v´ızszintesen balr´ ol jobbra, illetve ¨res mez˝ f¨ ugg˝olegesen fel¨ ulr˝ ol lefel´e t¨ olthet˝ ok ki. A mez˝ok sz´amoz´ asa a bal fels˝ o sarokb´ ol indul. Minden olyan mez˝o sz´ amot kap, ahol v´ızszintesen, vagy f¨ ugg˝olegesen megfejt´es kezd˝ odik, teh´ at egy mez˝ot akkor kell sz´ amozni, ha u ul r´ a, hogy felette ¨res, ´es teljes¨ fekete, alatta pedig u o van, vagy hogy t˝ole balra fekete, jobbra pedig u ¨res mez˝ ¨res mez˝ o tal´ alhat´ o. A keresztrejtv´eny egybet˝ us szavakat ´altal´ aban nem tartalmaz. Ha a mez˝ o a keresztrejtv´eny sz´el´en´el van, akkor a keresztrejtv´eny sz´el´et fekete mez˝ onek tekintj¨ uk. A honlapunkr´ ol let¨ olthet˝ o keresztrejtveny.txt tabul´atorokkal tagolt adatf´ajlban egy 10 × 10-es keresztrejtv´eny h´ al´oja van le´ırva. A fekete mez˝ oket f” az u ¨re” seket pedig .” karakter a´br´ azolja. ” Feladatunk, hogy t´abl´ azatkezel˝ o program seg´ıts´eg´evel a szab´ alyoknak megfelel˝ oen sz´ amozzuk be a keresztrejtv´eny mez˝ oit ´es jelen´ıts¨ uk meg a mint´ anak megfelel˝ oen. Minta:

Forr´ as r´eszlete

Megold´ as r´eszlete

A megold´ as sor´ an tetsz˝oleges oszlopokban v´egezhet¨ unk seg´edsz´ am´ıt´asokat, de ezek ´ertelmez´es´et seg´ıts¨ uk el˝ o feliratokkal. A form´ az´ ast a mint´ anak megfelel˝oen v´egezz¨ uk el, u ´gy, hogy a megjelen´es k¨ ovesse a forr´ as esetleges v´altoz´ as´ at. A megold´ asban saj´ at f¨ uggv´eny vagy makr´o nem haszn´ alhat´ o. Bek¨ uldend˝o egy t¨ om¨ or´ıtett i560.zip ´ allom´anyban a megold´ ast tartalmaz´ o munkaf¨ uzet ´es a megold´ as r¨ ovid le´ır´ as´at bemutat´ o dokument´ aci´ o. ´ Egy nagy, nemzetk¨ I. 561 (E). ozi ´ekszerforgalmaz´ o c´eg el´erkezettnek l´atta az id˝ot, hogy Magyarorsz´agon is megjelenjen. Kezdetben a budapesti pl´az´ akban K¨ oz´ episkolai Matematikai ´ es Fizikai Lapok, 2022/3

165

nyitottak n´egy boltot. Ezek n´egyhavi forgalmi adatai alapj´ an k´esz´ıts¨ unk adatb´ azist ekszerboltok n´even. 1. Import´aljuk az adatb´ azisba a sz¨ ovegf´ ajlok nev´evel megegyez˝o t´ abl´ akba a boltok.txt, a forgalom.txt ´es a honap.txt sz¨ ovegf´ ajlok tartalm´ at. A f´ ajlok UTF-8 k´ odol´as´ uak, tabul´ atorral tagoltak, els˝ o sorukban a mez˝ onevek szerepelnek. A forgalom t´ abl´ ahoz adjuk hozz´a a vid mez˝ ot. Bolt (id, irsz, cim, boltvezeto) id A bolt azonos´ıt´ oja (sz´ am), ez a kulcs. irsz A bolt ir´ any´ıt´ osz´ ama (sz´ am). cim A bolt Budapesten bel¨ uli c´ıme (sz¨ oveg). boltvezeto A boltvezet˝ o neve. Forgalom (vid, boltid, datum, idopont, osszeg, kartya) aml´ al´o), ez a kulcs. vid A v´ as´arl´as azonos´ıt´oja (sz´ boltid A bolt azonos´ıt´ oja, ahol a v´as´ arl´as t¨ ort´ent (sz´ am). datum A v´ as´arl´as napja (d´ atum). idopont A v´ as´ arl´as id˝opontja (id˝o). osszeg A v´ as´ arl´askor fizetett ¨ osszeg (sz´ am). kartya A fizet´es k´ arty´ aval t¨ ort´ent (igen/nem). Honap (hid, hnev) hid A h´ onap sorsz´ ama (sz´ am), ez a kulcs. hnev A h´ onap neve (sz¨ oveg). A datum megjelen´ese legyen r¨ ovid d´ atum, az idopont mez˝ o legyen r¨ ovid id˝ o. A p´enz¨osszegeket mindenhol forint p´enznem form´ aban, 0 tizedesjeggyel jelen´ıts¨ uk meg. A t´ abl´ ak kapcsolata:

A k¨ovetkez˝ o feladatok megold´ as´ an´al a lek´erdez´eseket ´es a jelent´est a z´ ar´ ojelben ¨ olvashat´o n´even ments¨ uk. Ugyelj¨ unk arra, hogy a megold´ asban pontosan a k´ıv´ ant mez˝ ok szerepeljenek. 2. Hat´ arozzuk meg az egyes boltok n´egyhavi ¨ osszbev´etel´et boltonk´ent. Csak a bolt c´ım´et ´es az ¨ osszbev´etelt ´ırassuk ki. (2bevetel) 3. K´esz´ıts¨ uk el a havi forgalmi jelent´est a minta szerint. (3havijel) 4. Mikor, melyik boltban (el´eg a c´ım mez˝ o) v´ as´ aroltak 500 000 Ft felett? Id˝ orendben list´ azzuk ki az adatokat a v´ as´ arl´as ¨ osszeg´evel egy¨ utt. (4draga) 5. A boltvezet˝ok jutal´ekot kapnak a k´eszp´enzes v´as´arl´asok ut´an. A jutal´ek ¨ osszege a v´ as´arl´as ¨ osszeg´enek fixen r¨ ogz´ıtett sz´ azal´eka. Adjuk meg, hogy ki az a boltvetet˝ o, aki a legt¨ obb jutal´ekot kapta. (5legjobb) 166

K¨ oz´ episkolai Matematikai ´ es Fizikai Lapok, 2022/3

Kimenet: egyetlen sor´ aban egy sz´ am szerepeljen: h´anyf´elek´eppen v´ alaszthatjuk meg az u ´j pap´ırp´enz ´ert´ek´et. Bemenet (a / jel sort¨ or´est helyettes´ıt) 2 11 / 2 5

´ 6. Aprilisban h´ usv´et el˝ ott (elsej´et˝ ol tizenhatodik´aig) a boltokban akci´os a´ron adtak minden term´eket. A c´eg vezet´ese k´ıv´ ancsi arra, hogy ezeken a napokon naponta mekkora volt az egyes boltok bev´etele ´es ez h´any elad´ asb´ ol sz´armazik? Adjuk meg a v´alaszt, rendezz¨ uk d´ atum ´es azon bel¨ ul a bolt ir´ any´ıt´osz´ ama szerint. Ezeken t´ ul a bolt c´ıme, a napi v´ as´ arl´asok ¨osszege ´es sz´ ama jelenjen meg. (6akcio) 7. Nem tan´acsos ´ejszak´ ara sok p´enzt tartani a boltokban, Ez´ert egy p´enzsz´ all´ıt´o c´eg est´enk´ent a napi bev´etel 150 000 Ft feletti r´esz´et begy˝ ujti ´es elsz´all´ıtja a bankba. A c´eg tarif´ aja a bankba sz´ all´ıtott p´enz 2%-a. Mennyi a p´enzsz´ all´ıt´ o c´eg bev´etele az ´ekszerbolth´ al´ozatt´ ol a n´egy h´ onap alatt? (7szallitas) 8. Az alkalmazottakban felmer¨ ult, hogy sz´ oba j¨ ohetne a h´etv´egi z´ arvatart´ as, ez´ert a vezet˝ os´eg szeretn´e megtudni a h´etv´egi, illetve h´etk¨oznapi bev´etelek a´tlag´ at. Ha a h´etv´egi l´enyegesen elmarad a h´atk¨ oznapit´ ol, a vezet´es megfontolja az o¨tletet. Azt, hogy egy adott nap a h´et melyik napja, a WeekDay f¨ uggv´eny seg´ıts´eg´evel der´ıthet˝ o ki. Haszn´ alat´ anak n´ezz¨ unk ut´ ana a s´ ug´oban. (8ahetvege, 8bhetkoznap) Bek¨ uldend˝o egy t¨om¨ or´ıtett a´llom´anyban az ekszerboltok adatb´ azis ´es egy r¨ovid dokument´ aci´ o, amely tartalmazza az alkalmazott adatb´ azis-kezel˝ o program nev´et ´es verzi´osz´ am´at. I/S. 61. Egy t´ avoli orsz´ agban N -f´ele pap´ırp´enz van forgalomban, az i-edik onb¨ oznek, nincs k´et egyenl˝ o. Egy p´enz´ert´eke P [i]. A P [i] ´ert´ekek p´ aronk´ent k¨ ul¨ rendszer ´eszszer˝ u, ha nem l´etezik olyan x ´es y ´ert´ek˝ u pap´ırp´enz, amelyekre teljes¨ ul az x < y < 2x egyenl˝ otlens´eg. Tudjuk, hogy kezdetben a p´enzrendszer ´eszszer˝ u. Szeretn´enk bevezetni egy u ´j, az eddigiekt˝ ol k¨ ul¨ onb¨ oz˝ o ´ert´ek˝ u pap´ırp´enzt u ´gy, hogy a p´enzrendszer tov´ abbra is ´eszszer˝ u maradjon. Tov´ abb´ a az u ´j pap´ırp´enz ´ert´eke legyen pozit´ıv eg´esz, ´es nem nagyobb, mint K. Adjuk meg, hogy h´ anyf´elek´eppen v´alaszthatjuk meg az u ´j pap´ırp´enz ´ert´ek´et. Bemenet: az els˝ o sor´ aban az N ´es K sz´ amok tal´ alhat´ ok sz´ok¨ozzel elv´ alasztva. am P [i], azaz az i-edik pap´ırp´enz A k¨ ovetkez˝ o sor N sz´ amot tartalmaz: az i-edik sz´ ´ert´eke. K¨ oz´ episkolai Matematikai ´ es Fizikai Lapok, 2022/3

167

Kimenet 3

Magyar´azat: az 1, a 10 ´es a 11 ´ert´ekeket v´ alaszthatjuk. 9 Korl´ atok: 1  N  100, 1  K  10 , 1  P [i]  109 p´ aronk´ent k¨ ul¨ onb¨ oznek. Id˝ olimit: 0,4 mp. ´ ekel´es: a pontok 50%-a kaphat´ Ert´ o, ha 1  K  105 eset´en a program helyes kimenetet ad. Bek¨ uldend˝o egy is61.zip t¨ om¨ or´ıtett a´llom´anyban a megfelel˝oen dokument´ alt ´es kommentezett forr´ asprogram, amely tartalmazza a megold´ as l´ep´eseit, valamint megadja, hogy a program melyik fejleszt˝ oi k¨ ornyezetben futtathat´ o. ´ am meg szeretn´e l´atogatni Ev´ ´ at. A kettej¨ S. 160. Ad´ uk k¨ ozt l´ev˝ o utak h´ al´ozata le´ırhat´o egy ir´ any´ıtatlan gr´ affal, melyben a cs´ ucsok a keresztez˝ od´esek ´es az ´elek ´ am az egyes sorsz´am´ a k¨ozt¨ uk l´ev˝ o utak. Ad´ u keresztez˝ od´esb˝ ol indul ´es a legnagyobb, N -es sorsz´ am´ uba szeretne eljutni. ´ am eddig mindig a k´et pont k¨ Ad´ oz¨ otti legr¨ ovidebb utak valamelyik´en ment. oz¨ ott. Adjuk meg, h´anyf´ele k¨ ul¨ onb¨ oz˝ o legr¨ ovidebb u ´t van a k´et pont k¨ Mivel gyakran megy l´ atogat´ oba, unja m´ar ezeket a lehet˝ os´egeket, ´ıgy most egy olyan u ´tvonalat szeretne v´ alasztani, amelynek hossza szigor´ uan nagyobb, mint a legr¨ovidebb u ´t hossza. Adjunk meg, hogy legal´ abb mekkora lesz az u ´j u ´tvonal hossza. Bemenet: az els˝ o sor tartalmazza a keresztez˝od´esek N ´es az utak M sz´ am´at. A k¨ovetkez˝ o M sor mindegyike egy-egy utat ´ır le a k´et v´egpontj´ aval. Kimenet: az els˝ o sor´ aba a legr¨ ovidebb utak sz´ ama ker¨ ulj¨ on. A kimenet m´asodik sor´ aba ´ırjuk az u ´j u ´tvonal minim´alis hossz´at. Ha nincs a legr¨ ovidebbn´el hosszabb u ´t, akkor -1-et kell ki´ırni. Minta: Bemenet (a / jel sort¨ or´est helyettes´ıt) 3 3 / 1 2 / 2 3 / 3 1

Kimenet (a / jel sort¨ or´est helyettes´ıt) 1 / 2

Korl´ atok: 2  N  300. Id˝olimit: 1 mp. ´ Ert´ekel´es: a pontok 50%-a kaphat´ o arra a megold´ asra, amelyn´el a kimenet els˝ o sora helyes. Bek¨ uldend˝o egy s160.zip t¨ om¨ or´ıtett a´llom´anyban a megfelel˝oen dokument´ alt ´es kommentezett forr´ asprogram, amely tartalmazza a megold´ as l´ep´eseit, valamint megadja, hogy a program melyik fejleszt˝ oi k¨ ornyezetben futtathat´ o. A feladatok megold´asai regisztr´aci´o ut´an a k¨ovetkez˝o c´ımen t¨olthet˝ok fel: https://www.komal.hu/munkafuzet Beku ¨ld´esi hat´arid˝o: 2022. ´aprilis 15.

168

K¨ oz´ episkolai Matematikai ´ es Fizikai Lapok, 2022/3

vagyis a f˝ o´ag ´ aramer˝ oss´ege

Fizika gyakorlat megold´asa

(1)

G. 757. Van egy p´ ar kiford´ıthat´ o keszty˝ um, mindk´et darabja k´ıv¨ ul fekete, bel¨ ul feh´er. Tudom-e ezeket felem´ as keszty˝ uk´ent hordani? (3 pont)

I = I1 + I2 =

U . R1 R2 Rb + R1 + R2

A telep a´ltal leadott ¨ osszteljes´ıtm´eny:

K¨ ozli: Vlad´ ar K´ aroly, Kiskunhalas

´ Megold´as. Altal´ aban nem hordhatjuk a keszty˝ uket u ´gy, hogy az egyiket kiford´ıtjuk, mert a kiford´ıt´ as sor´ an a jobbkezes keszty˝ u balkezess´e, a balkezes pedig jobbkezess´e v´alik. A felem´as sz´ın˝ u keszty˝ up´ ar teh´at vagy k´et jobbkezes, vagy k´et balkezes keszty˝ ub˝ ol a´llna. Abban az esetben, ha olyan rugalmas anyagb´ ol k´esz¨ ult keszty˝ ukr˝ ol van sz´ o, amelyekn´el nincs k¨ ul¨onbs´eg a jobb ´es a bal keszty˝ u k¨oz¨ott, akkor term´eszetesen hordhatunk felem´ as sz´ın˝ u keszty˝ uket is.

(2)

U2 . R1 R2 Rb + R1 + R2

Az R2 ellen´ all´ as´ u fogyaszt´ ora jut´ o fesz¨ ults´eg: U2 = U − IRb ,

T¨ or¨ ok Hanga (Budapest, Fasori Evang´elikus Gimn., 10. ´evf.) 47 dolgozat ´erkezett. Helyes 35 megold´ as. Hi´ anyos (1 pont) 3, hib´ as 9 dolgozat.

P = UI =

teh´at az erre a fogyaszt´ora jut´ o teljes´ıtm´eny: 2

(3)

Fizika feladatok megold´asa

P2 =

(U − IRb ) . R2

A megadott felt´etel szerint P2 = 0,6 P . Ebb˝ ol (1), (2) ´es (3) felhaszn´ al´as´ aval, o egyszer˝ us´ıt´es ut´an ad´odik, hogy majd U 2 -tel val´ P. 5349. 1,5 Ω bels˝ o ellen´ all´ as´ u zsebtelep p´ arhuzamosan kapcsolt R1 = 40 Ω all´ as´ u fogyaszt´ okat m˝ uk¨ odtet. Hat´ arozzuk meg az ismeretlen ´es ismeretlen R2 ellen´ ellen´ all´ as ´ert´ek´et, ha a zsebtelep ¨ osszteljes´ıtm´eny´enek 60%-a jut erre a fogyaszt´ ora. (4 pont)

K¨ozli: Kis Tam´ as, Heves

Megold´as. Legyen a telep u ults´ege U , a bels˝ o ¨resj´arati fesz¨ ellen´ all´ as´ at pedig jel¨ olj¨ uk Rb -vel. Tudjuk, hogy R1 = 40 Ω, ults´eg nagys´ ag´ at nem ismerj¨ uk (de mint Rb = 1,5 Ω. Az U fesz¨ l´ atni fogjuk, erre az adatra nincs is sz¨ uks´eg¨ unk), ´es keress¨ uk ag´ at. az ismeretlen ellen´all´ as R2 nagys´ 

1 1 + R1 R2

−1

=

Ennek az egyenletnek k´et gy¨ oke van: R2 = 2,9 ´es R2 = 19,9. A m´ asodik fogyaszt´ o teljes´ıtm´enye teh´ at akkor lesz az ¨ osszteljes´ıtm´eny 60%-a, ha R2 ≈ 3 Ω, vagy ha R2 ≈ 20 Ω. ´ Isk., 11. ´evf.) Kiss-Beck Regina (Szeged, SZTE Gyak. Gimn. ´es Alt. dolgozata alapj´an

R1 R2 , R1 + R2

´ıgy a teljes a´ramk¨ orre fel´ırhat´ o Ohm-t¨ orv´eny:

69 dolgozat ´erkezett. Helyes 32 megold´ as. Kicsit hi´ anyos (3 pont) 5, hi´ anyos (1–2 pont) 21, hib´ as 9, nem versenyszer˝ u 2 dolgozat.

U = I (Rb + Rered˝o ) , K¨ oz´ episkolai Matematikai ´ es Fizikai Lapok, 2022/3

Behelyettes´ıtva R1 ´es Rb ismert ´ert´ek´et, az ismeretlen R2 ellen´ all´ asra (ohm egys´egekben sz´ amolva) az al´ abbi m´ asodfok´ u egyenletet kapjuk: 24,9 R22 − 568 R2 + 1440 = 0.

A p´ arhuzamosan kapcsolt k´et ellen´ all´ as ered˝oje Rered˝o =

R12 R2 = 0,6(R1 + R2 )(R1 Rb + R2 Rb + R1 R2 ).

169

170

K¨ oz´ episkolai Matematikai ´ es Fizikai Lapok, 2022/3

P. 5350. Egy ´ atl´ atsz´ o g¨ omb k¨ ozep´et keskeny, p´ arhuzamos f´enynyal´ abbal megvil´ ag´ıtva a sugarak ´eppen a g¨ omb fel¨ ulet´enek ´ atellenes pontj´ an f´ okusz´ al´ odnak. Mekkora a g¨ omb anyag´ anak t¨ or´esmutat´ oja? (4 pont)

K¨ ozli: Sz´echenyi G´ abor, Budapest

I. megold´as. Tekints¨ uk az ´ abr´ an l´athat´ o sug´armenetet. Az α bees´esi sz¨ og ´es a β t¨ or´esi sz¨ og k¨ oz¨ ott (a Snellius–Descartes-t¨ orv´eny szerint) fenn´ all, hogy

Sz´ am´ıtsuk ki, hogy a legkisebb, teh´ at 0,1 mW teljes´ıtm´eny˝ u l´ezer f´enye 80%-os f´enyelnyel´es mellett mennyi id˝ o alatt meleg´ıt fel egy sejtet a k´ arosod´ ast okoz´ o o alatt a biztos roncsol´ ast okoz´ o 100 ◦ C-ra. Az egyszer˝ u50 ◦ C-ra, ´es mennyi id˝ s´eg kedv´e´ert tekints¨ unk egy idegsejtet 5 μm ´ atm´er˝ oj˝ u ´es 7 μm m´elys´eg˝ u hengernek, amelynek s˝ ur˝ us´eg´et ´es fajh˝ oj´et a v´ız´evel vehetj¨ uk egyenl˝ onek. A szem h˝ om´ers´ekleasokkal (elmozdul´ asok, h˝ ovezet´es stb.) most ne t´et vegy¨ uk 36 ◦ C-nak, ´es egy´eb hat´ t¨ or˝ odj¨ unk. A kapott id˝ ot vess¨ uk ¨ ossze az emberi szem kb. 0,2 m´ asodperces reakci´ oidej´evel! (4 pont)

sin α = n, sin β

Megold´as. Tudjuk, hogy egy m t¨ omeg˝ u test ΔT h˝ om´ers´eklettel t¨ ort´en˝ o felmeleg´ıt´es´ehez sz¨ uks´eges h˝ o Q = cmΔT , valamint egy P teljes´ıtm´eny˝ u l´ezer a´ltal t id˝ o alatt leadott h˝ o η hat´ asfok´ u f´enyelnyel´es mellett: Q = ηP t. Eszerint

ahol n a g¨ omb anyag´ anak t¨ or´esmutat´ oja. M´ asr´eszt az ´abr´ an l´athat´ o ABO h´ aromsz¨ og k¨ uls˝ o sz¨oge (α) a m´ asik k´et bels˝o sz¨ og (β) ¨ osszeg´evel egyenl˝ o, vagyis α = 2β. Ezek szerint

cmΔT = ηP t, ahonnan az id˝ otartam: t=

2 sin β cos β sin(2β) = = 2 cos β. n= sin β sin β Ha a sug´ arnyal´ ab keskeny, a β sz¨ og nagyon kicsiny, teh´at cos β ≈ 1 ´es n ≈ 2. II. megold´as. Ha a sug´ arnyal´ ab keskeny, elfogadhatjuk a Gauss-f´ele paraxi´ alis k¨ ozel´ıt´est. Ha a f´eny egy R g¨ orb¨ uleti sugar´ u fel¨ uleten n t¨or´esmutat´ oj´ u k¨ozegbe jut, akkor a k´epalkot´ as egyenlete a szok´asos jel¨ ol´esekkel:

n n−1 = , 2R R teh´at

Az ismert adatok ´es kisz´am´ıtott mennyis´egek: – A v´ız fajh˝oje: c = 4,2 kJ/(kg ◦ C). – Az idegsejtek k´ arosod´ as´ anak megfelel˝o h˝ om´ers´eklet-k¨ ul¨ onbs´eg: ΔT1 = 50 ◦ C − 36 ◦ C = 14 ◦ C. ΔT2 = 100 ◦ C − 36 ◦ C = 64 ◦ C.

Eset¨ unkben, amikor a bees˝ o f´enysugarak p´arhuzamosak, t = ∞, vagyis 1/t null´anak vehet˝ o. A k´ept´ avols´ ag a g¨ omb ´ atm´er˝ oje: k = 2R. A k´epalkot´ asi t¨orv´eny szerint:

n = n − 1, 2

n = 2.

– – – – – – –

A A A A A A A

Nagy Imre (Marosv´ as´ arhely, Bolyai Farkas L´ıceum, 10. ´evf.) 37 dolgozat ´erkezett. Helyes 20 megold´ as. Kicsit hi´ anyos (3 pont) 8, hi´ anyos (1–2 pont) 5, hib´ as 3, nem ´ert´ekelhet˝ o 1 dolgozat.

P. 5351. Vajon mi´ert nem szabad a l´ezerf´enybe belen´ezni? Az ember szemlencs´eje a f´enyt igen kicsi fel¨ uletre, jellemz˝ oen n´eh´ any μmes tartom´ anyra k´epes f´ okusz´ alni. A leg´erz´ekenyebb sejtek a retin´ aban vannak, itt a csap” ´es p´ alcika” nev˝ u idegsejtek m´erete a μm-es tartom´ anyba esik. A minden” ” napi ´eletben haszn´ alt l´ezerek teljes´ıtm´enye 0,1 mW ´es 100 mW k¨ oz¨ ott van. K¨ oz´ episkolai Matematikai ´ es Fizikai Lapok, 2022/3

cmΔT . ηP

– Az idegsejtek biztos roncsol´od´ as´ anak megfelel˝o h˝ om´ers´eklet-k¨ ul¨ onbs´eg:

1 n n−1 + = . t k R

vagyis

K¨ ozli: Vass L´ aszl´ o, Budapest

171

80 f´enyelnyel´es hat´ asfoka: η = 100 = 0,8. sz´ am´ıt´asba vett legkisebb l´ezerteljes´ıtm´eny: P = 0,1 mW = 10−4 W. hengernek tekintett sejt alapk¨ or´enek sugara: r = 2,5 μm. hengernek tekintett sejt magass´ aga: h = 7 μm. sejt t´erfogata: V = r2 πh = 1,4 · 10−16 m3 . kg sejt s˝ ur˝ us´ege: ρ = 1000 m3 . sejt t¨omege: m = ρV = 1,4 · 10−13 kg. Ezeknek megfelel˝ oen a keresett id˝ otartamok:     4,2 · 103 · 1,4 · 10−13 · 14 J cmΔT1 t1 = = ≈ 0,1 ms, ηP (0,8 · 10−4 ) W

illetve

    4,2 · 103 · 1,4 · 10−13 · 64 J cmΔT1 = ≈ 0,5 ms. t2 = ηP (0,8 · 10−4 ) W

Ezek az id˝otartamok mintegy 2000-szer, illetve 400-szor kisebbek, mint a szem reakci´oideje. 172

K¨ oz´ episkolai Matematikai ´ es Fizikai Lapok, 2022/3

Teh´ at val´oban nem tan´acsos a l´ezerf´enybe belen´ezni! Csillingek csapat ´ Isk. ´es Gimn., 9. ´evf.) Csilling D´ aniel (Budapesti Fazekas M. Gyak. Alt. Csilling Katalin (Budapest, Szil´ agyi E. Gimn., 12. ´evf.) 77 dolgozat ´erkezett. Helyes 14 megold´ as. Kicsit hi´ anyos (3 pont) 25, hi´ anyos (1–2 pont) 27, hib´ as 2, nem versenyszer˝ u 9(!) dolgozat.

P. 5352. Egy R ellen´ all´ as´ u, A keresztmetszet˝ u, z´ art k¨ orvezet˝ ot B indukci´ ovektor´ u m´ agneses t´erben szeretn´enk forgatni a s´ıkj´ aban l´ev˝ o szimmetriatengelye k¨ or¨ ul alland´ ´ o ω sz¨ ogsebess´eggel. Mekkora a ´tlagteljes´ıtm´ennyel tudjuk ezt megtenni? (4 pont)

K¨ozli: Sz´ asz Kriszti´ an, Budapest

I. megold´as. Felt´etelezz¨ uk, hogy a forg´ astengely mer˝ oleges a m´ agneses indukci´ovektorra. A k¨ orvezet˝ on ´ athalad´ o m´ agneses fluxus

II. megold´as. Tegy¨ uk fel, hogy a m´ agneses t´er mer˝ oleges a forg´ astengelyre. A m´ agneses mez˝ oben forg´ o k¨ orvezet˝ oben fesz¨ ults´eg induk´al´odik, ami ´ aramot hoz l´etre. Az ´aramj´ arta k¨ orvezet˝ o m´ agneses dip´ olusk´ent viselkedik, amire a k¨ uls˝ o m´ agneses mez˝ o forgat´ onyomat´ekot fejt ki. Ezzel a forgat´ onyomat´ekkel megegyez˝ o nagys´ag´ u, de ellent´etes ir´ any´ u forgat´ onyomat´ekot kell kifejten¨ unk, hogy a k¨ orvezet˝ ot ´alland´ o ω sz¨ogsebess´eg˝ u mozg´asban tartsuk. Z´arjon be a k¨ orvezet˝ o s´ıkj´anak norm´alvektora α = ωt sz¨ oget a B indukci´ ovektorral. Ekkor az induk´ alt fesz¨ ults´eg U = BAω sin α, a k¨orben foly´ o´ aram er˝oss´ege I=

BAω U = sin α. R R

A k¨or´ aram dip´ olnyomat´eka

Φ(t) = BA cos(ωt)

m = IA =

m´ odon v´ altozik id˝ oben. (Az id˝om´er´es kezd˝ opontj´anak azt a pillanatot v´alasztottuk, amikor a m´ agneses fluxus maxim´ alis.) Az induk´alt fesz¨ ults´eg:

BA2 ω sin α, R

amelyre a m´agneses mez˝ o M = Bm sin α =

U (t) = −

ΔΦ = BAω sin(ωt). Δt

(Ezt az ¨ osszef¨ ugg´est a harmonikus rezg˝ omozg´asn´ al a kit´er´es ´es a sebess´eg k¨oz¨otti osszef¨ ugg´es alapj´ an, vagy pedig differenci´alsz´ am´ıt´ assal l´ athatjuk be.) Ez Umax = ¨ = BAω nagys´ ag´ u, ω k¨ orfrekvenci´ aj´ u v´ alt´ ofesz¨ ults´eg, amelynek effekt´ıv ´ert´eke: BAω Umax Ueff = √ = √ . 2 2

forgat´onyomat´ekot fejt ki. Ismert, hogy egy tengely k¨ or¨ ul ´ alland´ o ω sz¨ ogsebess´eggel forg´o k¨ orvezet˝ o eset´en a forgat´ ashoz sz¨ uks´eges pillanatnyi teljes´ıtm´eny: P = Mω =

B 2 A2 ω 2 2 B 2 A2 ω 2 . sin α = R 2R Soml´ an Gell´ert (P´ecs, Le˝ owey Kl´ara Gimn., 12. ´evf.)

P =

2 Ueff B 2 A2 ω 2 = . R 2R

11 dolgozat ´erkezett. Helyes 7 megold´ as. Hi´ anyos (1–2 pont) 4.

Az energiamegmarad´ as t¨ orv´enye szerint nek¨ unk ugyanekkora a´tlagos teljes´ıtm´enyt kell kifejten¨ unk a k¨ orvezet˝ o forgat´ asa sor´ an. Megjegyz´es. Ugyanezt az eredm´enyt a pillanatnyi teljes´ıtm´eny, vagyis 2

P (t) =

2

2

2

U (t) B A ω = sin2 (ωt) R R

u id˝ ore id˝ obeli a ´tlag´ert´ekek´ent is megkaphatjuk, ha kihaszn´ aljuk, hogy sin2 (ωt) hossz´ vonatkoz´ o´ atlag´ert´eke 1/2.

Budai Csan´ ad (Budapest, De´ak T´eri Evang´elikus Gimn., 11. ´evf.) dolgozata alapj´an K¨ oz´ episkolai Matematikai ´ es Fizikai Lapok, 2022/3

B 2 A2 ω 2 sin2 α, R

aminek a´tlagos ´ert´eke

Az R ellen´ all´ ason fejl˝od˝ o´ atlagos h˝ o id˝ oegys´egenk´ent P =

B 2 A2 ω sin2 α R

173

P. 5353. Mi az oka annak, hogy a kib´ any´ aszott ur´ an´erc aktivit´ asa jelent˝ osen nagyobb, mint a bel˝ ole k´esz¨ ul˝ o ur´ ans´ o´e? (4 pont)

K¨ ozli: Simon P´eter, P´ecs

Megold´as. Az ur´an´erc aktivit´ as´ at nem csup´an az ur´an adja. A kib´any´ aszott ´ercben – ha az el´eg ¨ oreg” – radioakt´ıv egyens´ uly a´ll fenn az ur´ an ´es a le´ anyelemei” ” ” k¨oz¨ott. Mindegyik le´ anyelem aktivit´ asa megegyezik az ur´ an aktivit´as´ aval. A 238 U boml´asi sor´ aban 10-n´el is t¨ obb le´ anyelem van, emiatt az ur´ an´erc aktivit´ asa egy nagys´ agrenddel nagyobb, mint a vele azonos t¨ omeg˝ u ur´ ans´ o aktivit´ asa. Magyar G´ abor Bal´ azs (Eger, Dob´ o Istv´ an Gimn., 11. ´evf.) 29 dolgozat ´erkezett. Helyes 12 megold´ as. Kicsit hi´ anyos (3 pont) 3, hi´ anyos (1–2 pont) 6, hib´ as 5, nem versenyszer˝ u 3 dolgozat.

174

K¨ oz´ episkolai Matematikai ´ es Fizikai Lapok, 2022/3

P. 5354. Motoros j´ at´ekvonat halad R sugar´ u, k¨ or alak´ u p´ aly´ an, ´ alland´ o nagys´ aaj´ u g´ u v sebess´eggel. A k¨ or k¨ oz´eppontj´ at´ ol d < R t´ avols´ agra egy ´ alland´ o, f0 frekvenci´ hangot kibocs´ at´ o, pontszer˝ u hangforr´ as helyezkedik el. A vonatra egy mikrofont r¨ ogz´ıt¨ unk. Milyen hat´ arok k¨ oz¨ ott v´ altozik a mikrofon a ´ltal ´eszlelt hang frekvenci´ aja? (A hang sebess´ege c.) (6 pont)

d ez a maximum´ at sin β = 1, vagyis β = 90◦ -n´ al veszi fel. Ezek szerint (sin α)max = R , d at a mikrofon ´altal ´eszlelt legnagyobb frekvencia umax = R v, teh´

fmax = f0

K¨ozli: Vigh M´ at´e, Biatorb´ agy

I. megold´as. Legyen a vonat k¨ or alak´ u p´ aly´ aj´anak k¨oz´eppontja O, a r¨ogz´ıtett hangforr´ as helye F , a vonat pillanatnyi helye pedig a k¨orp´ alya P pontja (l´ asd ´ o, f0 frekvenci´aj´ u hangot kibocs´at´ o hangforr´ as hangj´ at egy mozg´o az 1. ´ abr´ at). All´ megfigyel˝o (eset¨ unkben a mikrofon) a Doppler-effektus szerint 

f = f0 1 +

d sin β, R

1. a ´bra

K¨ oz´ episkolai Matematikai ´ es Fizikai Lapok, 2022/3

.

K¨ openczei Csan´ ad (Bonyh´ ad, Pet˝ ofi S. Ev. Gimn. ´es Koll., 11. ´evf.) ´es Yokota Adan (G¨ od¨ oll˝ oi T¨ or¨ ok Ign´ ac Gimn., 12. ´evf.) dolgozata alapj´an

frekvenci´aj´ unak ´eszleli, ahol u az ´eszlel˝ o sebess´eg´enek a hangforr´ as ir´ any´ aba mutat´ o komponense.

sin α =



  v d fmin = f0 1 − . c R

c

sebess´eggel cs¨ okken, ahol α az OP F sz¨ og. A legnagyobb ´eszlelt frekvencia u legnagyobb ´ert´ek´enek, vagyis α legnagyobb ´ert´ek´enek felel meg. Mivel az OP F h´aromsz¨ ogre fel´ırhat´o szinuszt´etel szerint

v d 1+ c R

Hasonl´o m´ odon kapjuk a hangforr´ ast´ ol t´ avolod´ o mikrofon a´ltal ´eszlelt frekvenciacs¨okken´est is. Ha t¨ ukr¨ ozz¨ uk a P pontot az OF egyenesre, az F ´es P  pontok t´ avolod´ as´ anak sebess´ege ugyanakkora lesz, mint F ´es P k¨ ozeled´es´enek sebess´ege d volt (2. ´ abra). Mivel u legnagyobb ´ert´eke R v, az ´eszlelt frekvencia legkisebb ´ert´eke:

u

A hangforr´ as hely´et pl. az a´br´ an l´athat´ o β sz¨oggel adhatjuk meg. Bontsuk fel a mikrofon v nagys´ ag´ u sebess´egvektor´ at egy P F -fel p´ arhuzanos ´es egy arra mer˝ oleges komponensre. A p´ arhuzamos ¨ osszetev˝ ot jel¨olj¨ uk u-val. A hangforr´ as ´es a mikrofon t´ avols´ aga u = v sin α



II. megold´as. A hangmagass´ ag v´ altoz´ as´ at a Doppler-effektussal magyar´azzuk. u, a leveg˝ oh¨ oz k´epest a´ll´ o hangforr´ as frekvenci´ aj´at egy, Eszerint az f0 frekvenci´aj´ a hangforr´ ashoz u sebess´eggel k¨ ozeled˝o (vagy t´ avolod´ o) mikrofon  u f = f0 1 ± c nagys´ ag´ unak r¨ ogz´ıti. A feladatunk teh´ at u legnagyobb ´ert´ek´enek meghat´ aroz´ asa.

K´et test t´ avols´ aga nem f¨ ugg att´ ol, hogy milyen koordin´ ata-rendszerben sz´ am´ıtjuk ki azt. V´alasszuk azt a koordin´ ata-rendszert, amelynek orig´ oja a k¨ orv sz¨ ogsebesp´ alya O k¨ oz´eppontj´aban van, ´es ω = R s´eggel forog az O pont k¨ or¨ ul a vonat k¨ ormozg´ as´ aval megegyez˝o ir´ anyban. A j´at´ekvonat – ebben a forg´ o vonatkoztat´ asi rendszerben – mindig ugyanazon a helyen a´ll, a hangforr´ as pedig egy d sugar´ u k¨ orp´ avd ly´ an u = dω = R sebess´eggel egyenletesen mozog 3. ´ abra (3. ´ abra). A vonat ´es a hangforr´ as t´ avols´ aga akkor v´ altozik a leggyorsabban, amikor o vonat ir´any´ aba a hangforr´ as ´eppen az F1 pontban van, ahol a sebess´ege az a´ll´ any´ u. Ilyenkor mutat, vagy az F2 pontban, ahol a sebess´ege a vonattal ellent´etes ir´ F ´es P k¨ozeled´es´enek, illetve t´ avolod´ as´ anak sebess´ege u, a megv´ altozott frekvencia legnagyobb ´es legkisebb ´ert´eke teh´ at     vd vd fmax = f0 1 + , illetve fmin = f0 1 − . Rc Rc Megjegyz´es. A forg´ o koordin´ ata-rendszerben a hangforr´ as mozog, az ´eszlel˝ o (mikroc fon) pedig a ´ll. Ennek ellen´ere a Doppler-effektusnak nem az f = f0 c−u k´eplet´et alkal-

2. a ´bra

175

176

K¨ oz´ episkolai Matematikai ´ es Fizikai Lapok, 2022/3

c+u

A kil´ep˝ o protonok relativisztikus mozg´ asi energi´aja:     25 f0 mp c 2 2 2 Em =  −1 ≈ − m p c = mp c − 1 = 938 MeV · f1 18,9 1 − (v 2 /c2 )

osszef¨ ugg´essel sz´ amoltunk. Ezt maztuk, hanem az ´ all´ o hangforr´ asra vonatkoz´ o f = f0 c ¨ az´ert tehett¨ uk meg, mert a forg´ o vonatkoztat´ asi rendszerben a leveg˝ o is mozog (forog), ´es a Doppler-effektusn´ al mindig a k¨ ozeghez viszony´ıtott sebess´egek sz´ am´ıtanak. (G. P.)

≈ 303 MeV ≈ 48 pJ.

28 dolgozat ´erkezett. Helyes 14 megold´ as. Kicsit hi´ anyos (4–5 pont) 9, hi´ anyos (1–3 pont) 3, hib´ as 1, nem versenyszer˝ u 1 dolgozat.

P. 5362. Szinkrociklotronban az elemi r´eszecsk´ek t¨ omeg´enek a sebess´egt˝ ol val´ o f¨ ugg´es´et a gyors´ıt´ o elektromos t´er frekvenci´ aj´ anak cs¨ okkent´es´evel kompenz´ alj´ ak. P´eld´ aul ha protonokat gyors´ıtanak, a du´ ansokra (D alak´ u, f´emb˝ ol k´esz¨ ult, u ¨reges f´elkorongokra) ker¨ ul˝ o fesz¨ ults´eg frekvenci´ aj´ at 25 MHz-r˝ ol 18,9 MHz-ig v´ altoztatj´ ak ciklusonk´ent. Hat´ arozzuk meg ebben az esetben a) a m´ agneses indukci´ ovektor nagys´ ag´ at; b) a kil´ep˝ o protonok kinetikus energi´ aj´ at! (5 pont)

P´eldat´ ari feladat nyom´an

Megold´as. a) A ciklotronokban a m´agneses t´er biztos´ıtja a r´eszecsk´ek k¨ormozaja megegyezik a du´ ang´ as´ ahoz sz¨ uks´eges er˝ ot, ´ıgy a r´eszecsk´ek kering´esi frekvenci´ u protonokra, sokra kapcsolt fesz¨ ults´eg f frekvenci´aj´aval. Az e t¨olt´es˝ u, mp t¨omeg˝ ha azok v sebess´eggel r sugar´ u k¨ orp´ aly´ an mozognak, fenn´ all, hogy evB = mp

v2 , r

tov´ abb´ a

v = 2πrf0 .

asakor (m´eg biztosan nemrelativisztikus Ebben a k´epletben f0 a protonok indul´ mozg´ asakor) alkalmazott frekvencia. A m´ agneses indukci´ ovektor nagys´ aga ezek szerint 2π(25 · 106 )(1,67 · 10−27 ) 2πf0 mp = T ≈ 1,64 T. B= e 1,6 · 10−19

Bencz Benedek (Budapest, Ba´ar-Madas Ref. Gimn., 9. ´evf.) dolgozata alapj´an Megjegyz´esek. 1. A szinkrociklotronban a m´ agneses indukci´ o nagys´ aga a du´ ansokon bel¨ ul mindenhol ugyanakkora, ´es a gyors´ıt´ as sor´ an a r´eszecsk´ek sebess´ege, valamint a p´ aly´ ajuk sugara fokozatosan n¨ ovekszik. Nagyon nagy energi´ aj´ u (´es emiatt nagyon nagy m´eret˝ u) gyors´ıt´ okban gyakorlatilag lehetetlen a m´ agneses mez˝ o homogenit´ as´ at nagy t´err´eszben biztos´ıtani, ´es az eg´esz berendez´esben ultranagy v´ akuum fenntart´ asa is technikailag megoldhatatlan. Ehelyett a gyors´ıt´ as k¨ ozben a m´ agneses t´er er˝ oss´eg´et v´ altoztatj´ ak oly m´ odon, hogy a r´eszecsk´ek p´ aly´ aj´ anak sugara mindv´egig ugyanakkora maradjon. Ekkor a v´ akuumot ´es a m´ agneses teret elegend˝ o a r´eszecskenyal´ abot k¨ or¨ ulvev˝ o v´ekony cs˝ oben l´etrehozni ´es fenntartani. 2. A relativisztikus fizika ´es a klasszikus fizika t¨ orv´enyei k¨ oz¨ otti k¨ ul¨ onbs´eg ´ altal´ aban omeg hely´ere a megn¨ ovekedett t¨ omeget” ´ırjuk. nem csup´ an abb´ ol ´ all, hogy a nyugalmi t¨ ” A r´eszecsk´ek mozg´ asi energi´ aja p´eld´ aul a relativit´ aselm´elet (´es a tapasztalat) szerint nem v2

osszef¨ ugg´esb˝ ol, hanem az Em = m(v)c2 − m0 c2 k´eplet lapj´ an sz´ am´ıthat´ o ki. az m(v) 2 ¨ Sz´ amos m´ as esetben (p´eld´ aul a homog´en er˝ ot´erben t¨ ort´en˝ o egyenesvonal´ u mozg´ asn´ al) hib´ as eredm´enyt kapunk, ha csak az egyszer˝ u t¨ omegcser´et” hajtjuk v´egre. Egyenletes ” (vagy csak lassan v´ altoz´ o sebess´eg˝ u) k¨ ormozg´ asn´ al azonban a naiv m´ odszer ´eppen a helyes eredm´enyt adja. (G. P.) 15 dolgozat ´erkezett. Helyes 8 megold´ as. Kicsit hi´ anyos (4 pont) 4, hi´ anyos (1–3 pont) 3 dolgozat.

P. 5363. Egy v´ekony, magas u ob˝ ol homok´ or´ at k´esz´ıtett¨ unk. ¨vegcs˝ omege megegyezik az u o ´es a tart´ otalpak A benne l´ev˝ o homok m0 t¨ ¨vegcs˝ egy¨ uttes t¨ omeg´evel. Kezdetben a homok az als´ o t´erf´el h = 5 cm hossz´ u r´esz´et t¨ olti ki, ´es az eszk¨ oz megford´ıt´ asa ut´ an egyenletes u ¨temben t0 = = 1 perc alatt pereg le. (A fels˝ o ´es az als´ o t´erf´elben l´ev˝ o homok alakj´ at k¨ ozel´ıts¨ uk hengerekkel.)

b) Az egyre nagyobb sebess´eggel mozg´o protonokn´ al a mozg´asegyenlet (k¨ormozg´ as eset´en) form´alisan annyiban t´er el a klasszikus (newtoni) mozg´asegyenletomege hely´ere a megn¨ovekedett” t˝ ol, hogy a proton mp nyugalmi” t¨ ” ” mp m(v) =  1 − (v 2 /c2 )

a) Hat´ arozzuk meg, hogy hol van a homok´ ora t¨ omegk¨ oz´eppontja t id˝ ovel az o ´ra elind´ıt´ asa ut´ an! (Ne foglalkozzunk a homok´ ora ind´ıt´ as´ at k¨ ovet˝ o, illetve a meg´ all´ as´ at k¨ ozvetlen¨ ul megel˝ oz˝ o nagyon r¨ ovid id˝ otartamokkal, amikor a homokzuhatag m´eg vagy m´ ar nem t¨ olti ki a kifoly´ ony´ıl´ as ´es az als´ o becsap´ od´ asi hely k¨ oz¨ otti teljes t´ avols´ agot.)

t¨ omeget ´ırjuk. A m´ agneses indukci´ ovektor nagys´ ag´ anak ´alland´ os´ aga miatt B=´ alland´ o=

vagyis

2πf1 mp 1 2πf0 mp  = , e e 1 − (v 2 /c2 ) 

1 1 − (v 2 /c2 )

=

b) Sz´ am´ıtsuk ki, hogy mekkora a homok´ ora impulzusa (lend¨ ulete) t id˝ ovel a homok´ ora elind´ıt´ asa ut´ an! ozben a hoc) Nagyon ´erz´ekeny m´erleggel megm´erj¨ uk a homok´ ora s´ uly´ at, mik¨ mok a fels˝ o tart´ alyb´ ol az als´ oba pereg. Azt tal´ aljuk, hogy a m´ert s´ uly egy kicsivel nagyobb, mint a m´ ar lepergett homok´ ora s´ ulya. Az el˝ oz˝ o k´et r´eszfeladatra adott

f0 , f1

ahol f1 a gyors´ıt´ ofesz¨ ults´eg lecs¨ okkentett frekvenci´ aja. K¨ oz´ episkolai Matematikai ´ es Fizikai Lapok, 2022/3

177

178

K¨ oz´ episkolai Matematikai ´ es Fizikai Lapok, 2022/3

v´ alaszt felhaszn´ alva adjuk meg, hogy h´ any ezrel´ekkel nagyobb a m˝ uk¨ od˝ o homok´ ora s´ ulya a m´ ar lej´ art” homok´ or´ a´en´ al! ” (6 pont) K¨ ozli: Sz´echenyi G´ abor, Budapest

A m˝ uk¨od˝o homok´ora s´ ulya teh´ at 0,0014 ezrel´ekkel nagyobb, mint a m´ar lej´ art” ” homok´ or´ a´e.

Megold´as. a) Jel¨ olj¨ uk a homok´ ora teljes magass´ ag´at 2H-val. A homok´ ora megford´ıt´ asa ut´ an t < t0 id˝ o eltelt´evel a fels˝ o t´erf´elb˝ ol m0 tt t¨omeg˝ u homok pergett a´t

11 dolgozat ´erkezett. Helyes Toronyi Andr´ as, G´ abriel Tam´ as, Budai Tam´ as, Kert´esz Bal´ azs ´es T´egl´ as Panna megold´ asa. Kicsit hi´ anyos (4–5 pont) 2, hi´ anyos (2–3 pont) 4 dolgozat.

0

az als´ oba, ´ıgy az als´ o t´erf´elben h tt magass´ ag´ u homokhenger alakult ki. Hasonl´ o0 t omeg˝ u homokoszlop teteje a homok´ora k´epp, a fels˝ o t´arol´ oban maradt m0 (1 − t ) t¨ 0

agban lesz, az alja pedig mindv´egig H magasalj´ at´ ol m´erve H + h(1 − tt ) magass´ 0 s´ agban marad. Mivel a homok alakj´ at k¨ ozel´ıthetj¨ uk hengerekkel, az egyes t´erfelekben l´ev˝ o homokmennyis´eg t¨ omegk¨ oz´eppontja (a homok´ ora alj´ahoz viszony´ıtva) rendre   h t h t ´es H+ 1− 2 t0 2 t0 magass´ agban lesz. Ezek szerint a homok´ ora t¨ omegk¨oz´eppontj´anak helyzete t id˝ opillanatban   m0 H + m0 tt · h2 tt + m0 (1 − tt ) H + h2 [1 − tt ] 0 0 0 0 , s(t) = 2m0 amit

a s(t) = s0 + v0 t + t2 2

alakban is fel´ırhatunk, ahol s0 = H +

h , 4

v0 = −

H +h 2t0

´es

a=

(5 pont)

h H +h t− . 2 t0 2t0

b) Az eg´esz rendszer lend¨ ulete a t¨ omegk¨ oz´eppont sebess´eg´enek ´es az ¨osszt¨omegnek a szorzata:   h H +h I(t) = 2m0 2 t − . t0 2t0 c) Az ´erz´ekeny m´erlegre ´ all´ıtott, m´eg m˝ uk¨ od˝ o” homok´ or´ ara f¨ ugg˝olegesen lefel´e ” o, f¨ ugg˝olegesen felfel´e pedig a m´erleg a´ltal kifejtett G s´ ulyer˝ o hat. 2m0 g neh´ezs´egi er˝ ulyn¨oveked´es A rendszer mozg´ asegyenlete: G − 2m0 g = 2m0 a, ahonnan a relat´ıv s´ a h G − 2m0 g = = 2 ≈ 1,4 · 10−6 . 2m0 g g gt0

179

P´eldat´ ari feladat nyom´ an

Megold´as. El˝ osz¨ or l´assuk be, hogy abban az esetben lesz maxim´ alis az intenzit´as, amikor a szomsz´edos pol´arsz˝ ur˝ ok egym´assal bez´ art sz¨ oge ugyanakkora. (A pol´arsz˝ ur˝ ok polariz´aci´ os ir´any´ anak egym´assal bez´ art sz¨ og´et a r¨ ovidebb sz´ ohaszn´ alat kedv´e´ert egyszer˝ uen a pol´arsz˝ ur˝ ok sz¨ og´enek fogjuk nevezni.) oggel Tekints¨ unk el˝ osz¨ or k´et szomsz´edos pol´arsz˝ ur˝ ot, amelyek ϕ1 , illetve ϕ2 sz¨ u f´eny polariz´ aci´ os s´ıkj´at, ´es legyen ezen k´et forgatj´ ak el a r´ ajuk es˝ o, I0 intenzit´as´ orv´enye szerint a k´et sz˝ ur˝ on ´ athalad´ o f´eny sz¨og ¨osszege egy adott ϕ0 ´ert´ek. Malus t¨ uk (a ϕ1 + ϕ2 = v´egs˝ o intenzit´ asa I = I0 cos2 ϕ1 cos2 ϕ2 . Ennek a kifejez´esnek keress¨ at. Ez a sz´els˝ o´ert´ek ugyanott van, ahol cos ϕ1 · = ϕ0 felt´etel mellett) a maximum´ cos ϕ2 a legnagyobb ´ert´eket veszi fel. Differenci´alsz´ am´ıt´ as (deriv´al´as) helyett elemi u ´ton, trigonometrikus a´talak´ıt´assal is megtal´ alhatjuk a sz´els˝ o´ert´ek hely´et: cos ϕ1 · cos ϕ2 =

A t¨ omegk¨ oz´eppont mindv´egig lefele mozog, hiszen v(t) < 0, de mivel a > 0, a t¨omegk¨ oz´eppont f¨ ugg˝olegesen felfel´e gyorsul.

K¨ oz´ episkolai Matematikai ´ es Fizikai Lapok, 2022/3

P. 5379. Ide´ alis pol´ arsz˝ ur˝ ok seg´ıts´eg´evel szeretn´enk a line´ arisan polariz´ alt f´eny ´gy, hogy az intenzit´ asvesztes´eg legfeljebb 10% polariz´ aci´ os s´ıkj´ at 45◦ -kal elforgatni u legyen. Legal´ abb h´ any pol´ arsz˝ ur˝ ore van sz¨ uks´eg¨ unk, ´es hogyan kell azokat optim´ alisan elhelyezni?

h . t20

Felismerhetj¨ uk, hogy a t¨ omegk¨ oz´eppont mozg´ asa egyenletesen gyorsul´o mozg´as, amelynek pillanatnyi sebess´ege v(t) = v0 + a t =

T´egl´ as Panna (Selye J´ anos Gimn., 12. ´evf.) R´evkom´arom, Szlov´ akia

cos ϕ0 cos Δϕ 1 cos(ϕ1 + ϕ2 ) + cos(ϕ1 − ϕ2 ) = + . 2 2 2

Mivel a k´et sz¨ og ¨ osszege a´lland´ o, ´ıgy csak a k´et sz¨ og Δϕ = ϕ1 − ϕ2 k¨ ul¨ onbs´eg´enek f¨ uggv´eny´eben v´ altozik a szorzat ´ert´eke. Ennek akkor van maximuma, ha arsz˝ ur˝ onk van, akcos Δϕ = 1, vagyis Δϕ = 0, azaz ϕ1 = ϕ2 . Ha teh´at csak k´et pol´ kor adott nagys´ ag´ u sz¨ ogelford´ıt´ashoz a k´et sz˝ ur˝ ot azonos sz¨ ogben kell egym´ ashoz, illetve a r´ ajuk es˝ o f´eny polariz´ aci´ os s´ıkj´ahoz k´epest elforgatni, hogy az intenzit´ascs¨okken´es a lehet˝ o legkisebb legyen. Mi a helyzet n darab pol´arsz˝ ur˝ o beiktat´ asa eset´en? Ekkor is kiv´ alaszthatunk 2 sz˝ ur˝ ot. Ahhoz, hogy a rajtuk a´thalad´ o f´eny intenzit´ asa (a sz¨ ogek ¨ osszeg´enek adott ´ert´eke mellett) maxim´ alis legyen, a pol´arsz˝ ur˝ oket azonos sz¨ og˝ u elford´ıt´ assal kell egym´ as ut´an elhelyezni. Mivel ez minden p´arra igaz, ´ıgy a legkedvez˝obb esetben az ¨osszes pol´arsz˝ ur˝ o egym´ ashoz viszony´ıtott elfordul´asa azonos nagys´ ag´ u lesz: ϕ1 = ϕ2 = . . . = ϕn =

45◦ . n

A k´erd´es teh´ at: legal´abb h´ any darab ide´alis pol´ arsz˝ ur˝ o sz¨ uks´eges, hogy egym´ashoz k´epest (az els˝ ot pedig a bees˝ o f´eny polariz´ aci´ os ir´any´ ahoz k´epest) 45◦ /n 180

K¨ oz´ episkolai Matematikai ´ es Fizikai Lapok, 2022/3

sz¨ ogben elforgatva az intenzit´ asvesztes´eg kevesebb legyen, mint 10%? ´Irjuk fel u ´jra Malus t¨ orv´eny´et:  ◦ 45 I0 · (cos2 ϕ1 ) · (cos2 ϕ2 ) · . . . · (cos2 ϕn ) = I0 cos2n  0,9I0 , n azaz fn ≡ cos

2n



45◦ n



 0,9.

Numerikusan kapjuk, hogy:

P. 5381. Egy u ol k´esz¨ ult (szige¨vegb˝ tel˝ o) ed´eny higannyal van t¨ oltve. A higanyba egy f¨ ugg˝ oleges, d = 0,5 mm ´ atm´er˝ oj˝ u kapill´ aris cs˝ o mer¨ ul az ´ abr´ an l´ athat´ o m´ odon. A higany felsz´ıne f¨ ol´e h = 6 mm magass´ agban egy nagy kiterjed´es˝ u, v´ızszintes f´emlemezt helyezt¨ unk. Mennyivel v´ altozik meg a kapill´ aris cs˝ oben a higanyszint, ha a f´emlemez ´es a higany k¨ oz´e U = 20 kV egyenfesz¨ ults´eget kapcsolunk? (6 pont)

f1 = 0,5; f2 = 0,729; f3 = 0,812; f4 = 0,856; f5 = 0,884; f6 = 0,902 > 0,9. 45◦ 6



= 7,5 -kal kell minden Teh´ at legal´abb 6 db pol´ arsz˝ ur˝ ore van sz¨ uks´eg¨ unk, ´es sz˝ ur˝ ot az el˝oz˝ oh¨ oz k´epest elforgatni, illetve az els˝ ot a bees˝o, line´arisan polariz´alt f´eny polariz´ aci´ os s´ıkj´ahoz k´epest be´ all´ıtani. Nemesk´eri D´ aniel (Budapest, ELTE Ap´ aczai Csere J. Gyak. Gimn., 11. ´evf.) Megjegyz´es. Azt, hogy optim´ alis esetben a pol´ arsz˝ ur˝ oket egym´ ashoz k´epest ugyanakkora sz¨ oggel elforgatva kell be´ all´ıtani, m´ as m´ odszerrel, a Jensen-egyenl˝ otlens´eg ∗ alkalmaz´ as´ aval is bel´ athatjuk. ´ ıt´ All´ as. Ha egy (v´eges vagy v´egtelen) I intervallumon az f f¨ uggv´eny konk´ av, a1 , a2 , . . . , an ∈ I,

p1 , p2 , . . . , pn

pozit´ıv sz´ amok, amelyekre p1 + p2 + . . . + pn = 1 teljes¨ ul, akkor

K¨ ozli: Vigh M´ at´e, Biatorb´ agy

I. megold´as. A fesz¨ ults´eg bekapcsol´ asa el˝ ott a kapill´ aris cs˝oben a higany valaul¨ onbs´eggel alacsonyabban a´ll, mint a cs¨ ov¨ on k´ıv¨ ul, mert a higany mekkora x1 szintk¨ nem nedves´ıti az u uleti fesz¨ ult¨veget. Ekkor a fel¨ s´egb˝ ol sz´armaz´ o F er˝ o tart egyens´ ulyt a k¨ uls˝ o ´es a bels˝ o higanyoszlop nyom´ ask¨ ul¨ onbs´eg´eb˝ ol sz´ armaz´ o er˝ ovel. A fel¨ uleti fesz¨ ults´eg (α) a folyad´ek felsz´ın´enek egys´egnyi hossz´ us´ ag´ u darabk´ aj´ara hat´ o er˝ o, egy Δ hossz´ us´ag´ u vonaldarab ment´en teh´ at αΔ er˝ o hat. A kapill´ aris cs˝o 2rπ ker¨ ulet´en hat´ o er˝ ok f¨ ugg˝oleges ir´ any´ u ered˝ oje 2rπα cos ϑ, ahol ϑ a higany ´es az u ¨veg illeszkeo f¨ ugg˝olegesen d´esi sz¨ oge. Mivel ϑ > 90◦ , az F er˝ 1. a ´bra lefel´e hat (1. ´ abra).

f (p1 a1 + p2 a2 + . . . + pn an )  p1 f (a1 ) + p2 f (a2 ) + . . . + pn f (an ).

Az er˝oegyens´ uly felt´etele:

Ha f szigor´ uan konk´ av, akkor egyenl˝ os´eg csak az a1 = a2 = . . . = an esetben teljes¨ ul. Ha f konvex, akkor az a ´ll´ıt´ as ford´ıtott ir´ any´ u egyenl˝ otlens´eggel teljes¨ ul.

F = A Δp hidroszt. ,

azaz

2rπ α| cos ϑ| = r2 πgx1 ,

π

anyon alulr´ ol konk´ av, fenn´ all, hogy Mivel a koszinuszf¨ uggv´eny a [0, 4 ] tartom´  ϕ + ϕ + . . . + ϕ  cos ϕ + cos ϕ + . . . + cos ϕ 1 2 n 1 2 n  cos , n n

ahonnan x1 =

tov´ abb´ a a sz´ amtani-m´ertani k¨ ozepek egyenl˝ otlens´ege szerint

(A fenti k´epletben  a higany s˝ ur˝ us´ege, r = d/2 a cs˝ o bels˝ o sugara ´es A = r2 π a kapill´aris cs˝o bels˝ o keresztmetszete. Ezek mindegyike ismert, valamint α ´es ϑ ´ert´ekei t´ abl´ azatokban megtal´ alhat´ o adatok, de ezekre a tov´ abbiakban nem lesz sz¨ uks´eg¨ unk.)

√ cos ϕ1 + cos ϕ2 + . . . + cos ϕn  n cos ϕ1 cos ϕ2 · . . . · cos ϕn . n ´Igy teh´ at cos2n



45◦ n



 cos2 ϕ1 cos2 ϕ2 · . . . · cos2 ϕn ,

ami ´eppen a bizony´ıtand´ o´ all´ıt´ as. Papp Marcell Imre (Budapest, ELTE Ap´ aczai Csere J. Gyak. Gimn., 11. ´evf.) 9 dolgozat ´erkezett. Helyes 6 megold´ as. Kicsit hi´ anyos (4 pont) 3 dolgozat. ∗

2α| cos ϑ| . gr

A fesz¨ ults´eg bekapcsol´ asa ut´ an a f´emlemez ´es a higany felsz´ıne k¨ oz¨ ott – mint egy s´ıkkondenz´ator belsej´eben – elektromos er˝ ot´er alakul ki, amely az ed´eny sz´eleit ´es a kapill´ aris cs˝o k¨ ozvetlen k¨ ornyezet´et lesz´am´ıtva homog´ennek tekinthet˝ o, ´es a t´erer˝ oss´eg nagys´ aga: E = U/h. Ez az er˝ ot´er – a s´ıkkondenz´ator lemezeire hat´ o er˝ oh¨oz hasonl´oan – f¨ ugg˝olegesen felfel´e h´ uzza a higany felsz´ın´et, ami miatt a higany nyom´ asa k¨ozvetlen¨ ul a felsz´ıne alatt egy kicsit kisebb lesz, mint a l´egk¨ ori nyom´ as.

L´ asd pl. https://abesenyei.web.elte.hu/theses/molnar.pdf

K¨ oz´ episkolai Matematikai ´ es Fizikai Lapok, 2022/3

181

182

K¨ oz´ episkolai Matematikai ´ es Fizikai Lapok, 2022/3

A higany fel¨ ulet´en (Gauss t¨ orv´enye szerint) fel¨ uletegys´egenk´ent σ = ε0 E t¨olt´es 1 ag´ u elektromos t´er jelenik meg, amelyre az ´atlagosan E = 2 E nagys´ Δp elekt. = σE = ε0

E2 U2 = ε0 2 2 2h

nyom´ asv´ altoz´ ast (cs¨ okken´est) id´ez el˝ o. A kapill´aris cs˝o belsej´eben (a k¨ uls˝ o higanyszint alatti t´err´eszben) az elektromos t´erer˝ oss´eg gyakorlatilag nulla, mivel az a f´emesen vezet˝ o higany majdnem teljesen z´ art r´esz´eben van (Faraday-kalitka). Azt, hogy egy v´ekony cs˝ o belsej´ebe nem hatolhat be az elektromos t´er a k¨ ovetkez˝ o m´odon is bel´ athatjuk: a higany minden pontj´aban ugyanakkoal, ´ıgy a kapill´aris cs˝o ra (Φ0 ) az elektromos potenci´ al (2. ´ abra). belsej´eben is mindenhol Φ0 a potenci´ (Ha nem ´ıgy lenne, hanem az u veg k´ e t oldala k¨ o¨ z¨ ott ugr´ asa” lenne a potenci´ alnak, akkor az nagyon ” nagy (v´ızszintes ir´any´ u) elektromos teret jelentene, ami biztosan nincs jelen. Ha viszont a cs˝ o belsej´eben (annak a higanyszint alatti r´esz´eben) nincs potenci´ alk¨ ul¨ onbs´eg, akkor ott az elektromos t´erer˝ oss´eg nulla.

II. megold´as. Az egyens´ ulyi ´allapotot (ak´ ar az eredeti, fesz¨ ults´egmentes esetben, ak´ ar pedig a bekapcsolt fesz¨ ults´eg hat´ as´ ara megv´ altozott helyzetben) a rendszer energi´ aj´anak minimum´at keresve is meg lehet hat´ arozni. Tekints¨ uk el˝ osz¨ or a fesz¨ ults´egmentes esetet! Legyen az ed´eny alapter¨ ulete S, a kapill´aris cs˝o keresztmetszete pedig A. (Nyilv´an feltehetj¨ uk, hogy S  A). Sz´am´ıtsuk ki a higanyb´ol ´es az u ob˝ ol ´ all´ o rendszer E o ¨vegcs˝ ¨sszenergi´aj´at a higanyszint x nagys´ ag´ u s¨ ullyed´es´enek f¨ uggv´eny´eben. Kiindul´ asi ´ allapotnak (az energia nullpontj´ anak) v´ alasszuk a 3.a ´ abr´ an l´athat´ o helyzetet, amikor x = 0. (Ez nyilv´ an nem egyens´ ulyi helyzet, teh´ at az ¨ osszenergia ilyenkor nem minim´ alis.)

2. ´ abra

´Igy a higanynak a hajsz´alcs˝ o belsej´eben l´ev˝ o r´esz´ere nem hat elektromos vonz´ oer˝ o (felfel´e), a´m a t¨ obbi r´esz´ere igen, emiatt a kapill´ arisban a higanyszint sz¨ uks´egszer˝ uen cs¨ okken. Ha az er˝oegyens´ uly valamekkora x2 > x1 szintk¨ ul¨onbs´egn´el a´ll be, akkor fenn´ all: F = A(Δphidroszt. − Δp elekt. ), azaz



U2 2rπα| cos ϑ| = r π gx2 − ε0 2 2h 2



Ha a higanyszint a cs˝oben x ´ert´ekkel cs¨ okken, akkor az ed´eny t¨ obbi r´esz´eben A abra). Δh  x miatt a szintemelked´est Δh = S x m´ert´ekben megemelkedik (3.b ´ a fesz¨ ults´eg n´elk¨ uli esetben elhanyagolhatjuk. A rendszer ¨ osszenergi´aja k´et tagb´ ol, a higany gravit´ aci´ os helyzeti energi´ aj´ab´ol ´es a higany u ¨veggel ´erintkez˝o r´esz´enek fel¨ uleti energi´ aj´ab´ ol tev˝odik ¨ ossze.

,

ahonnan x2 =

U2 U2 2α| cos ϑ| + ε0 2 = x 1 + ε0 2 . gr 2h g 2h g

A helyzeti energia akkora, amennyi munk´ aval az xA t´erfogat´ u, xA t¨ omeg˝ u hi´ anyz´o higanydarabot” a felsz´ın magass´ ag´ aba tudjuk emelni. Mivel a higanydarab ” t¨omegk¨oz´eppontja x/2 m´elys´egben volt, a k´erd´eses energia:

Leolvashatjuk, hogy a higanyszint s¨ ullyed´ese: x 2 − x 1 = ε0

U2 (8,85 · 10−12 ) · 20 0002 m ≈ 0,36 mm. = 2 2 2h g 2 · (6 · 10−3 ) · 13 546 · 9,81

E1 (x) = gr2 π

Hauber Henrik (Gy˝ or, R´evai Mikl´ os Gimn., 11. ´evf.) ´es T´egl´ as Panna (R´evkom´arom, Szlov´ akia, Selye J´ anos Gimn., 12. ´evf.) dolgozata alapj´an K¨ oz´ episkolai Matematikai ´ es Fizikai Lapok, 2022/3

3. ´ abra

183

x2 . 2

A fel¨ uleti fesz¨ ults´eg nemcsak az egys´egnyi hosszon hat´ o er˝ot, hanem az egym´ assal ´erintkez˝o anyagok egys´egnyi fel¨ ulet´ehez tartoz´ o energi´ at is megadja. Az u o, ¨vegcs˝ a leveg˝ o ´es a higany ´erintkez´es´en´el h´ aromf´ele fel¨ uleti fesz¨ ults´egr˝ ol is besz´elhat¨ unk: ovel ´erintkez˝o higany egys´egnyi fel¨ ulet´ere jut´ o energia (ezt szoαHg–leveg˝o a leveg˝ k´as szerint egyszer˝ uen α-val jel¨ olik), tov´ abb´ a αHg-¨uveg ´es αu¨veg–leveg˝o a higany ´es 184

K¨ oz´ episkolai Matematikai ´ es Fizikai Lapok, 2022/3

az u o ´es az u aj´at adja meg fel¨ uletegys´egenk´ent. (Meg¨veg, illetve a leveg˝ ¨veg energi´ ugg´es.) mutathat´ o, hogy fenn´all az αu¨veg–leveg˝o − αHg-¨uveg = αHg–leveg˝o · cos ϑ ¨osszef¨ A fel¨ uleti energia ezek szerint E2 (x) = 2rπx(αu¨veg–leveg˝o − αHg–¨uveg ) = −2rπα| cos ϑ| · x,

Ez is egy m´ asodfok´ u polinom, amelynek minimumhelye: x2 = −

Mivel Q = ε0 S Uh , v´eg¨ ul azt kapjuk, hogy a higanyszint s¨ ullyed´ese

a rendszer ¨ osszenergi´aja: x2 − 2rπα| cos ϑ| · x. E(x) = E1 (x) + E2 (x) = gr2 π 2

x 2 − x 1 = ε0

Ennek a m´asodfok´ u kifejez´esnek a minimumhelye (mint az teljes n´egyzett´e alak´ıt´ assal, vagy a m´asodfok´ u egyenlet megold´ ok´eplet´enek alkalmaz´ as´aval, esetleg deriv´al´assal bel´ athat´ o) 2α| cos ϑ| x1 = gr

10 dolgozat ´erkezett. 6 pontot kapott G´ abriel Tam´ as, Hauber Henrik, K¨ urti Gergely, Nemesk´eri D´ aniel ´es T´egl´ as Panna, 5 pontos Schmercz Blanka megold´ asa. Hi´ anyos (1–2 pont) 4 dolgozat.

Fizik´ab´ ol kit˝ uz¨ott feladatok

US h

t¨ olt´es jut a lemezeire”. Ennek a kondenz´ atornak az energi´ aja: ” Q2 Q2 = · h. Eelekt. = 2C 2ε0 S Hogyan v´ altozik ez az energia, ha a kapill´ arisban x-et s¨ ullyed a higanyszint, ´es emiatt az ed´eny t¨ obbi r´esz´eben Δh = A x m´ e rt´ e kben megemelkedik? Az eddigiS ekben elhanyagolhat´ oan kicsinek tekintett Δh-t most nem hanyagolhatjuk el, mert akkor – t´evesen – az eg´esz elektrosztatikus energiav´ altoz´ ast figyelmen k´ıv¨ ul hagyn´ ank. A szintemelked´es hat´ as´ ara a kondenz´ ator energi´aja ´ıgy v´altozik:   A Q2 · h− x . E3 (x) = 2ε0 S S Megjegyz´es. Fontos, hogy a kondenz´ ator energi´ aj´ anak v´ altoz´ as´ at olyan k¨ or¨ ulm´enyek k¨ oz¨ ott vizsg´ aljuk, amikor nem az U fesz¨ ults´eget, hanem a Q t¨ olt´es nagys´ ag´ at tartjuk alland´ ´ onak. Ezt p´eld´ aul u ´gy val´ os´ıthatjuk meg, hogy a m´ ar felt¨ olt¨ ott kondenz´ atort lekapcsoljuk a telepr˝ ol. Ha nem ´ıgy j´ arn´ ank el, akkor a rendszer nem lenne energetikailag z´ art, mert a lemezek t´ avols´ ag´ anak v´ altoztat´ asa k¨ ozben a telep energi´ at adna le, vagy venne fel.

A rendszer teljes energi´ aja:   Q2 A x2 Q2 h ≡ E(x) = E1 (x) + E2 (x) + E3 (x) = gA − 2rπα| cos ϑ| + x + 2 2ε0 S 2 2ε0 S ≡ ax2 + bx + c. K¨ oz´ episkolai Matematikai ´ es Fizikai Lapok, 2022/3

U2 ≈ 0,36 mm. 2h2 g

Schmercz Blanka (Budapest, ELTE Ap´ aczai Csere J. Gyak. Gimn., 11. ´evf.) dolgozat´anak felhaszn´al´as´ aval

szints¨ ullyed´esn´el van, ennek megfelel˝ o helyzetben lesz a higany egyens´ ulyban. Foglalkozzunk most a f´emlemezre kapcsolt fesz¨ ults´eg eset´evel (3.c ´ abra). A lemez ´es az S nagys´ ag´ u fel¨ ulet˝ u higany egy olyan s´ıkkondenz´atornak tekinthet˝ o, ults´eg hat´ as´ara amelynek kapacit´asa C = ε0 S/h, ´es ´ıgy U fesz¨ Q = ±CU = ±ε0

2α| cos ϑ| Q2 b = + . 2a gr 2ε0 gS 2

185

M. 412. Helyezz¨ unk egym´ asba n´eh´ any, pap´ırb´ ol k´esz¨ ult muffin kos´ ark´ at, majd v´egezz¨ unk ejt´esi k´ıs´erleteket! M´erj¨ uk meg, hogyan f¨ ugg az ´ alland´ osult es´esi sebess´eg a kos´ark´ ak sz´am´at´ ol! Hat´ arozzuk meg a pap´ırkos´ ark´ ak k¨ ozegellen´ all´ asi alakt´enyez˝oj´et! ´ (6 pont) K¨ ozli: Eero Uustalu, Esztorsz´ ag G. 773. A F¨ old–Hold rendszer a k´et ´egitest k¨ oz¨ os t¨ omegk¨ oz´eppontja k¨ or¨ ul kering 27,32 napos kering´esi id˝ovel a t´ avoli ´all´ ocsillagokhoz k´epest. Ehhez k´epest t¨obb, mint k´et nappal hosszabb id˝ o, ´ atlagosan 29,53 nap telik el k´et egym´ ast k¨ oott. Magyar´ azzuk meg a k´etf´ele peri´odusid˝ o k¨ oz¨ otti k¨ ul¨ onbs´eget, vet˝ o holdt¨olte k¨ oz¨ ´es egyszer˝ us´ıtett sz´ am´ıt´ assal mutassuk meg, hogy val´ oban nagyj´ab´ ol k´et nap az elt´er´es! (4 pont) s [m] 0 17 34 51 68 85 102 186

v [m/s] 0,00 0,41 1,00 1,05 1,15 1,19 1,26

s [m] 119 136 153 170 187 204 221

v [m/s] 1,21 1,14 1,17 1,17 1,10 1,07 1,02

G. 774. Az al´ abbi diagramon a Duna felsz´ıni sebess´egprofilja l´athat´ o az Erzs´ebet-h´ıdn´ al 2018. m´ arcius 10-´en. A v´ızszintes tengelyen a bal partt´ ol m´ert t´avols´ ag (s) l´ athat´ o m´eterben, a f¨ ugg˝oleges tengelyen a Duna sebess´ege (v) m/sban. A mell´ekelt t´ abl´ azatban tal´ alhat´ oak a m´ert adatok.

K¨ oz´ episkolai Matematikai ´ es Fizikai Lapok, 2022/3

Becs¨ ulj¨ uk meg, hogy h´ any m´eterrel sodorna le a Duna, ha a bal partt´ ol a partra mindig mer˝ olegesen 1 m/s sebess´eggel evezn´enk a´t egy, a bal partt´ol 221 m´eterre l´ev˝ o haj´ oig! (4 pont)

K¨ ozli: Csernovszky Zolt´ an, Budapest

G. 775. Elhanyagolhat´ o h˝ okapacit´as´ u, h˝ oszigetel˝ o tart´ alyban 1 kg nagyon hideg j´egk´ asa van, amire 1 kg 100 ◦ C-os forr´ ovizet ¨ ont¨ unk. Milyen h˝om´ers´eklet˝ u volt alyban? a j´egk´ asa, ha az egyens´ ulyi ´allapotot el´erve 2 liter 0 ◦ C-os v´ız lesz a tart´ (3 pont)

P. 5393. Egy m t¨ omeg˝ u ´es egy M = = 3m t¨ omeg˝ u, kicsiny goly´ ohoz fonalakat er˝ os´ıt¨ unk, melyek m´ asik v´eg´et a bal oldali a ´bra szerint azonos magass´ agban r¨ ogz´ıtj¨ uk. A goly´ ok k¨ oz´eppontja ekkor a felf¨ uggeszt´es alatt L m´elys´egben van. A kisebb t¨ omeg˝ u goly´ ot felemelj¨ uk u ´gy, hogy a hozz´ a kapcsol´od´ o fon´ al v´ızszintes legyen (jobb oldali ´ abra), majd a goly´ ot elengedj¨ uk. A k´et goly´ o t¨ ok´eletesen rugalmasan ´es egyenesen u ozik. ¨tk¨ a) Az u oz´es el˝ otti pillanatban mekkora egy¨ uttes er˝ ovel terheli a k´et fon´ al ¨tk¨ a felf¨ uggeszt´est? oz´es ut´ ani pillanatban? b) Mekkora a terhel´es az u ¨tk¨ c) Az els˝o ´es a m´ asodik u oz´es k¨ oz¨ ott mekkora a k´et fon´ al ´ altal bez´ art ¨tk¨ legnagyobb sz¨ og? d) A c) esetben mekkora nagys´ ag´ u, ´es milyen ir´any´ u az egy¨ uttes terhel´es? e) Mekkora sz¨ oget z´ arnak be a fonalak a f¨ ugg˝olegessel, amikor bek¨ ovetkezik a m´asodik u oz´es? ¨tk¨ (5 pont)

K¨ ozli: Zsigri Ferenc, Budapest

G. 776. Egy kutat´olaborat´oriumban ´ora¨ uveget szeretn´enek steriliz´alni UV f´eny seg´ıts´eg´evel. A steriz´ al´ashoz az o´ra¨ uveg 1 cm2 nagys´ ag´ u ter¨ ulet´ere 150 mJ ¨osszenergi´aj´ u UV f´enynek kell be´erkeznie. Becs¨ ulj¨ uk meg, hogy mennyi ideig kell ehhez u ¨zemeltetni az o´ra¨ uveg felett 75 cm-re felszerelt (pontszer˝ unek tekinthet˝ o) UV-l´ amp´ at, ha a gy´ ari adatok szerint a l´amp´ at´ ol 1 m´eter t´ avols´ agra az UV f´eny intenzit´ asa 125 μW/cm2 .

P. 5394. Egy m t¨ omeg˝ u, homog´en t¨ omegeloszl´ as´ u, ellipszis alak´ u lemez f´eltengelyeinek hossza a ´es b. Mekkora a test tehetetlens´egi nyomat´eka a 2a hossz´ us´ ag´ u nagytengely v´egpontj´ an ´ atmen˝ o, a lemez s´ıkj´ara mer˝ oleges tengelyre vonatkoztatva? (A feladat elemi u ´ton is megoldhat´ o.)

(4 pont)

P. 5395. Egy ´eve, 2021 m´ arcius´ aban meg´erkezett az els˝o hang¨ uzenet a Perseverance marsj´ ar´ ot´ ol (go.nasa.gov/3ly2OE4). Mekkora lehet a hangsebess´eg a Mars l´egk¨or´eben? ´ agh´ır alapj´an (4 pont) Ujs´

P. 5391. Egy m´ely k´ utba k¨ ovet ejt¨ unk. A csobban´ as hangj´ at 4,25 s-mal az elejt´es ut´ an halljuk meg. Milyen m´elynek tal´aljuk a kutat, ha g = 10 m/s2 -tel ´es vhang = = 320 m/s-mal sz´ amolunk? Mekkor´ anak ad´odik a k´ ut m´elys´ege, ha g = 9,81 m/s2 amolunk? (A k¨ ozegellen´ all´as hat´ as´at hanyagoljuk el.) tel ´es vhang = 340 m/s-mal sz´ (4 pont)

(5 pont)

P. 5396. Egy f¨ ugg˝oleges, h˝ oszigetel˝ o tart´ alyban l´eom´ers´eklet˝ u k´etatomos ide´alis g´azt szabadon v˝ o T0 h˝ mozg´ o h˝ oszigetel˝ o dugatty´ u z´ ar el k¨ ornyezet´et˝ ol. A g´ azt lassan meleg´ıtj¨ uk, melynek k¨ ovetkezt´eben t´erfogata n¨ ovekedni kezd. Meleg´ıt´es k¨ ozben, amikor a g´ az t´erfogata ´eppen megdupl´ az´ odott, a dugatty´ u a hengerben tal´ alhat´ o sz˝ uk´ıt˝ o perem miatt megakadt. Hat´ arozzuk meg a g´ az v´egs˝ o T h˝ om´ers´ekletet´et, ha ismert, hogy a g´azzal k¨ oz¨ olt h˝o 80%-a ford´ıt´ odott a bels˝o energia n¨ ovel´es´ere.

K¨ ozli: Honyek Gyula, Veresegyh´az

P. 5392. Egy sz¨ ok˝ok´ ut k¨ oz´eps˝ o ny´ıl´as´ an f¨ ugg˝olegesen ki´ araml´o v´ekony v´ızsug´ar H magass´ agig jut el. A v´ızsug´ar v´ızhozama”, azaz az id˝oegys´egenk´ent ki´araml´o ” v´ız t´erfogata: Φ = ΔV . Milyen h magass´ agban lebeg egy m t¨omeg˝ u labda, ha Δt a v´ızsug´arba helyezz¨ uk? (Felt´etelezhetj¨ uk, hogy a v´ızsug´ar teljes keresztmetszete el´eri a labd´ at, ´es arr´ ol v´ızszintes ir´anyban spriccel sz´et.) (5 pont) K¨ oz´ episkolai Matematikai ´ es Fizikai Lapok, 2022/3

(4 pont)

A Kvant nyom´ an 187

K¨ ozli: Gelencs´er Jen˝ o, Kaposv´ ar

188

P´eldat´ ari feladat nyom´an

K¨ oz´ episkolai Matematikai ´ es Fizikai Lapok, 2022/3

P. 5397. Egy Q = 10−9 C t¨ olt´es˝ u kicsiny testet egy nagy m´eret˝ u, f¨oldelt f´emlemezt˝ ol d = 10 cm t´ avols´ agban szigetel˝ o ´allv´ anyon r¨ogz´ıtett¨ unk. a) Mekkora a f´emlemez fel¨ uleti t¨ olt´ess˝ ur˝ us´ege a kicsiny testhez legk¨ozelebb es˝ o P pontj´ aban? b) Milyen messze van P -t˝ ol az a pont, ahol a f´emlemez fel¨ uleti t¨olt´ess˝ ur˝ us´ege a maxim´alis ´ert´eknek egyharmada? (4 pont)

K¨ozli: Holics L´ aszl´ o, Budapest

P. 5398. Digit´ alis f´enyk´epez˝ og´epen 35 mm gy´ ujt´ ot´ avols´ ag´ u objekt´ıv tal´ alhat´ o, melynek k¨ ozelpontja 25 cm. A k¨ ozelpont az a szenzort´ ol m´ert legkisebb t´ avols´ ag, ahonnan az objekt´ıv m´eg k´epes f´okusz´ alni. ozelpont t´ avols´ aga, ha az objekt´ıv ´es a f´enyk´ea) Hogyan v´ altozik meg a k¨ pez˝ og´ep k¨ oz´e egy k¨ ozgy˝ ur˝ ut helyez¨ unk, melynek hat´ as´ara az objekt´ıv 12 mm-rel messzebbre ker¨ ul a szenzort´ ol? b) K´esz´ıts¨ unk egy k¨ ozelpontba helyezett t´ argyr´ ol felv´etelt k¨ozgy˝ ur˝ uvel ´es an´elk¨ ul. Hogyan ar´anylik egym´ashoz ezen k´et k´ep nagys´ aga? (5 pont)

K¨ ozli: Sz´echenyi G´ abor, Budapest

P. 5399. Egy v´ekony, δ vastags´ ag´ u f´emlemezb˝ ol nagy, k´ up alak´ u fel¨ uletet hegesztett¨ unk ¨ ossze. A k´ up A-val jel¨ olt cs´ ucs´ aba I er˝ oss´eg˝ u´ aramot vezet¨ unk, majd az egyik alkot´on l´ev˝ o B pontb´ ol elvezetj¨ uk azt. Hat´ arozzuk meg a B-vel a´tellenes C pontban az a´rams˝ ur˝ us´eg-vektor ir´ any´ at ´es nagys´ ag´ at! Ismert, hogy az AB t´ avols´ ag ´ert´eke 3R, m´ıg a B ´es C pontok t´ avols´ aga 2R. (6 pont)

K¨ ozli: Vigh M´ at´e, Biatorb´ any

New exercises for practice – competition C (see page 156): Exercises up to grade 10: K/C. 727. See the text at Exercises K. K/C. 728. See the text at Exercises K. Exercises for everyone: C. 1709. The integers a and b are factors of 720, but ab is not a factor of 720. How many such ordered pairs (a; b) are there? (Proposed by S. R´ oka, Ny´ıregyh´ aza) C. 1710. Four circles are drawn in a unit square as shown in the figure. The two larger circles have the same size, and they are tangent to each other as well as to the sides of the square. The two smaller circles are also congruent, and they are also tangent to the sides of the square and to the larger circles. What is the area of the rhombus formed by the centres √ √ 1 , of the four circles? C. 1711. Solve the equation x − 1801 + y − 1860 = 2 − √ x−1801 where x and y denote real numbers. Exercises upwards of grade 11: C. 1712. The sides of a pentagon are all equal in length, and two of its angles are right angles. What may be the measures of the other three angles? (Proposed by G. K´ arolyi, Budajen˝ o) C. 1713. Let 1 1 x and a denote real numbers such that x + x = a. Determine the value of x13 + x13 as a function of a. New exercises – competition B (see page 157): B. 5230. Points C and D lie on a semicircular arc of diameter AB. Let A and B  denote the feet of the perpendiculars dropped to the line CD from the points A and B, respectively. Prove that the line segments anyi, Budapest) B. 5231. A C and B  D are equal in length. (3 points) (Proposed by L. Sur´ n n   k−1 n−k k k·2 = 2 · (2 − 1) for all positive integers n. (4 points) B. 5232. Prove that

 Beku ¨ld´esi hat´arid˝o: 2022. ´aprilis 15. Elektronikus munkafu ¨zet: https://www.komal.hu/munkafuzet



k=1

MATHEMATICAL AND PHYSICAL JOURNAL FOR SECONDARY SCHOOLS (Volume 72. No. 3. March 2022) Problems in Mathematics New exercises for practice – competition K (see page 155): K. 724. Julie cut a pizza into identical slices. Then she ate a few slices, but 3 slices remained. With a little calculation, she observed that she had eaten 3/4 of the whole pizza, plus 3/4 of a slice. How many slices were there? K. 725. We filled in the nine fields of a 3 × 3 table one by one, according K¨ oz´ episkolai Matematikai ´ es Fizikai Lapok, 2022/3

to the following rule: in each field, we entered the number of adjacent fields (i.e. fields sharing a common side with it) that had been filled in before. What was the order of the fields filled in? How many orders are possible? (Use the notation a1, a2, . . . , c2, c3 to refer to individual fields.) K. 726. Arrange the numbers 1, 2, 3, 4, . . . , 31, 32 along the circumference of a circle such that the sum of any pair of adjacent numbers should be a perfect square. Explain your reasoning. K/C. 727. On each field of an n × n table there is a coin, on all of them “heads” showing on top. In each move, we can turn over exactly three coins in any row or column, changing heads to tails and tails to heads. If n > 2, is it possible to achieve with a sufficient number of moves that tails should show on top of each coin? Explain your answer. K/C. 728. We have 10 cards, with the numbers 1, 2, 3, 4, 5, 6, 7, 8, 9, 10 written on them. The cards are laid on the table in a random order in a row, and then we write the number of its position on each card (that is, the cards are numbered from 1 to 10). Thus there will be two numbers on each card. The two numbers on each card are multiplied together, and the products are all added up. a) What is the smallest possible value of the final sum? b) What is the largest possible value of the final sum?

189

k=1

P is an interior point lying on the median drawn from vertex C of an acute-angled triangle ABC such that ∠AP B = 180◦ − ∠ACB. Show that the line AB is tangent to the circle AP C. (4 points) (Proposed by Sz. Kocsis, Budapest) B. 5233. The vertices of a regular hexagon are labelled 1, 2, . . . , 6 in a random order. Then the absolute value of the difference of the labels of the adjacent endpoints is written on each side of the hexagon. Find the expected value of the sum of the six numbers written on the sides. (4 points) (Proposed by J. Szoldatics, Budapest) B. 5234. A positive integer n is defined to be a mythical number if each of its divisors is 2 smaller than a prime number. For example, 15 is a mythical number. What is the largest possible number of divisors that a mythical number may have? Find all mythical numbers that have the maximum number of divisors. (5 points) (Proposed by S. R´ oka, Ny´ıregyh´ aza) B. 5235. Show that all prime

190

K¨ oz´ episkolai Matematikai ´ es Fizikai Lapok, 2022/3

numbers greater than 3 that occur in the Fibonacci sequence are of the form 4k + 1. (5 points) B. 5236. Let a, b, c denote positive real numbers such that abc = 1. Show that 2 a + a2 + a3 + b + b2 + b3 + c + c2 + c3  (a2 + b2 + c2 ) . (6 points) (Proposed by M. Lovas, Budapest and M. Rozenberg, Israel) B. 5237. In a triangle, r denotes the inradius, R is the circumradius, and s denotes the semiperimeter. Prove that if r + 2R = s then the triangle is right angled. (6 points) (Proposed by R. Fridrik, Szeged) New problems – competition A (see page 159): A. 821. a) Is it possible to find a function f : N2 → N such that  for every function g: N → N and positive integer m there exists n ∈ N such that set k ∈ N : f (n, k) = g(k) has at least m elements? b) Is it that for every function g : N → N there exists possible to find a function f : N2 → N such   n ∈ N such that set k ∈ N : f (n, k) = g(k) has an infinite number of elements? A. 822. Is it possible to find rational numbers p, q and r such that p + q + r = 0 and pqr = 1? (Proposed by M´ at´e Weisz, Cambridge) A. 823. For positive integersn consider the lattice points Sn = (x, y, z) : 1  x  n, 1  y  n, 1  z  n, x, y, z ∈ N . Is it possible to find √ a positive integer n for which it is possible to choose more than n n lattice points from Sn such that for any two chosen lattice points at least two of the coordinates of one is strictly greater than the corresponding coordinates of the other? (Proposed by Endre Cs´ oka, Budapest)

Problems in Physics (see page 186) M. 412. Place several muffin cupcake paper cases into each other and carry out drop experiments. Measure how the terminal speed of the cases depends on the number of cases. Determine the drag coefficient of the cases. G. 773. The Earth–Moon system revolves about the centre of mass of the two celestial bodies with a period of 27.32 days, relative to the distant fixed stars. However, in comparison, more than two days more time elapses between two full Moons, namely an average of 29.53 days. Explain the difference between the two period values, and with a simplified calculation show that the difference between them is indeed about two days. G. 774. The diagram on page 187 shows the surs [m] v [m/s] s [m] v [m/s] face flow velocity profile for the river Danube at bridge Erzs´ebet on 10 Marc 2018. On the 0 0.00 119 1.21 horizontal axis the distance s in metres mea17 0.41 136 1.14 sured from the left riverbank, and on the ver34 1.00 153 1.17 tical axis the speed of the water v in m/s can 51 1.05 170 1.17 be seen. The attached table shows the recorded data. Estimate the distance at which the river 68 1.15 187 1.10 would drift our boat down, if we were to row at 85 1.19 204 1.07 a constant velocity of 1 m/s, perpendicularly to 102 1.26 221 1.02 the riverbank, to a ship which is at a distance of 221 metres from the left riverbank. G. 775. In a thermally insulated flask of negligible heat capacity, there is 1 kg very cold crushed ice, to which 1 kg hot water of temperature 100 ◦ C is poured. What was the temperature of the ice originally if finally there are 2 litres of water at a temperature of 0 ◦ C in the flask? G. 776. Watch glasses are to be sterilized in a research laboratory by using UV light. In the sterilization process UV light of total energy of 150 mJ should fall onto a 1 cm2 surface area of the watch glass. Estimate how long should the UV lamp be operated for this, if

K¨ oz´ episkolai Matematikai ´ es Fizikai Lapok, 2022/3

191

the lamp is point-like and it is 75 cm above the watch glass. According to the factory data, the intensity of the UV light at a distance of 1 m from the lamp is 125 μW/cm2 . P. 5391. A stone is dropped into a deep well. The sound of the splash is heard 4.25 s after the drop. How deep is the well if we carry out the calculation with g = 10 m/s2 and vsound = 320 m/s? What is the depth of the well if instead we use the values of g = 9.81 m/s2 and vsound = 340 m/s? (We neglect air resistance.) P. 5392. A thin water jet ejected vertically out of the central nozzle of a fountain reaches a height of H. The ΔV “flow rate” of the water jet, i.e. the volume of water flowing out per unit time is Φ = Δt . At what height h does a ball of mass m float when it is placed into the water jet? (We can assume that the total cross-section of the water jet reaches the ball, and that the water splashes off from the ball in horizontal direction.) P. 5393. Threads are attached to two small balls of masses m and M = 3m such that the other ends of the threads are fixed at the same height, as it is shown in the figure on the left. The centres of the balls are then at a depth of L below the suspension. Then the ball with the smaller mass is raised such that the thread attached to it becomes horizontal (right figure), and then this ball is released. The two balls collide head-on and totally elastically. a) Right before the collision what is the total force exerted on the suspension by the two threads? b) What is this total force right after the collision? c) Between the first and the second collisions of the balls what is the greatest angle enclosed by the two threads? d) In the case of c) what is the direction and the magnitude of the total force exerted by the threads on the suspension? e) What is the angle enclosed by the threads and the vertical when the second collision occurs? P. 5394. The semi-major and semi-minor axes of an ellipse-shaped uniform-density plate of mass m are a and b, respectively. What is the rotational inertia of the plate with respect to an axis which is perpendicular to the plane of the plate and goes through one end of the major axis of length 2a? (The problem can also be solved by elementary considerations not requiring higher mathematics.) P. 5395. One year ago, in March 2021, the first voice message arrived from the Mars Perseverance Rover (go.nasa.gov/3ly2OE4). What might the speed of sound in the atmosphere of the Mars be? P. 5396. A sample of diatomic ideal gas at a temperature T0 is enclosed in a vertical, thermally insulated container by an easily moveable and thermally insulated piston. The gas is slowly heated, thus its volume begins to increase. During heating, when the volume of the gas has just doubled, the piston in the cylinder got stuck in the rim which narrowed the cylinder. Determine the final temperature of the gas T , if it is known that 80% of the heat added to the gas was used to increase the internal energy. P. 5397. A small object of charge Q = 10−9 C is fixed on an insulated stand, which is at a distance of d = 10 cm from a big grounded metal plate. a) What is the surface charge density of the metal plate at its point P , which is the closest to the small object? b) What is the distance between P and that point of the plate at which the surface charge density of the plate is one-third of the maximum surface charge density value? P. 5398. A digital camera has an objective lens of focal length of 35 mm, and its near point is at a distance of 25 cm. The near point is that smallest distance from the sensor from which the objective can still focus. a) How does the distance of the near point change if a ring is placed between the lens and the camera, such that the lens gets 12 mm further away from the sensor? b) Take a picture of an object at the near point, with and without using the ring. What is the ratio of the sizes of the two images? P. 5399. From a thin metal plate of width δ a large conical surface was welded. A current of I flows from the vertex of the cone A to a point B which lies on a slant height of the cone. Determine both the magnitude and the direction of the current density vector at point C on the surface of the cone, if C opposite to B. It is known that the distance of AB is equal to 3R and the distance between points B and C is 2R.

72. ´evfolyam 3. sz´am

K¨oMaL

Budapest, 2022. m´arcius

´ ´ FIZIKAI LAPOK ¨ EPISKOLAI KOZ MATEMATIKAI ES ˝ ´ITVE INFORMATIKA ROVATTAL BOV ´ ALAP´ITOTTA: ARANY DANIEL 1894-ben 72. ´evfolyam 4. sz´am

Mennyi a t´eglalap teru ¨lete? Budapest, 2022. ´aprilis

´ Megjelenik ´evente 9 sz´amban, janu´art´ol m´ajusig ´es szeptembert˝ol decemberig havonta 64 oldalon. ARA: 1050 Ft

´ TARTALOMJEGYZEK Bessenyei Mih´aly, Maksa Gyula: Mennyi a t´eglalap ter¨ulete? . . . . . . . . . . . . . . . . . . . . . . . . . . . . . . . . . .

194

J´ocsik Csilla: Gyakorl´o feladatsor emelt szint˝u matematika ´eretts´egire . . . . . . . . . . . . . . . . . . . . . . . .

198

N´emeth L´aszl´o: Megold´asv´azlatok a 2022/3. sz´am emelt szint˝u matematika gyakorl´o feladatsor´ahoz . . . . . . . . . . . . . . . . . . . . . . . . . . . . . . . . . . . . . . .

202

Matematika C gyakorlat megold´asa (1703.) . . . . . . .

214

Matematika feladatok megold´asa (5139., 5149., 5173., 5188.) . . . . . . . . . . . . . . . . . . . . . . . . . . . . . . . . . .

215

Nehezebb feladat megold´asa (812.) . . . . . . . . . . . . . . .

224

A C pontversenyben kit˝uz¨ott gyakorlatok (1714– 1720.) . . . . . . . . . . . . . . . . . . . . . . . . . . . . . . . . . . . . . . . .

226

A B pontversenyben kit˝uz¨ott feladatok (5238– 5245.) . . . . . . . . . . . . . . . . . . . . . . . . . . . . . . . . . . . . . . . .

227

Az A pontversenyben kit˝uz¨ott nehezebb feladatok (824–826.) . . . . . . . . . . . . . . . . . . . . . . . . . . . . . . . . . . . .

229

Informatik´ab´ol kit˝uz¨ott feladatok (562–564., 62., 161.) . . . . . . . . . . . . . . . . . . . . . . . . . . . . . . . . . . . . . . . . .

229

M´er´esi feladat megold´asa (410.) . . . . . . . . . . . . . . . . . .

233

Fizika gyakorlatok megold´asa (755., 760., 761., 764.) . . . . . . . . . . . . . . . . . . . . . . . . . . . . . . . . . . . . . . . . .

235

Fizika feladatok megold´asa (5347., 5357., 5364., 5366., 5368., 5370., 5371., 5378.) . . . . . . . . . . . . . . .

239

Ny´ari fizikat´abor . . . . . . . . . . . . . . . . . . . . . . . . . . . . . . . .

250

Fizik´ab´ol kit˝uz¨ott feladatok (413., 777–780., 5400–5408.) . . . . . . . . . . . . . . . . . . . . . . . . . . . . . . . . . .

251

Problems in Mathematics . . . . . . . . . . . . . . . . . . . . . . . .

254

Problems in Physics . . . . . . . . . . . . . . . . . . . . . . . . . . . . .

255

K¨ oz´ episkolai Matematikai ´ es Fizikai Lapok, 2022/4

´ EVA ´ F˝oszerkeszt˝o: RATKO ´ ¨ Fizikus szerkeszt˝o: GNADIG PETER ´ ILDIKO ´ M˝uszaki szerkeszt˝o: MIKLOS Bor´ıt´o: BURGHARDT ZSUZSA ´ Kiadja: MATFUND ALAP´ITVANY ´ RITA Alap´ıtv´anyi k´epvisel˝o: KOS Felel˝os kiad´o: KATONA GYULA Nyomda: OOK-PRESS Kft. ´ Felel˝os vezet˝o: SZATHMARY ATTILA INDEX: 25 450 ISSN 1215-9247 A matematika bizotts´ag vezet˝oje: ´ HERMANN PETER ´ BALINT, ´ ´ Tagjai: B´IRO GYENES ZOLTAN, ´ ´ KISS HUJTER BALINT, IMOLAY ANDRAS, ´ ´ GEZA, ´ ´ RITA, KOZMA GEZA, KOS KOS ´ MATOLCSI DAVID, ´ KATALIN ABIGEL, ´ ´ PACH PETER ´ ´ V´IGH ¨ ORDI ¨ OK PETERN E, PAL, VIKTOR A fizika bizotts´ag tagjai: ´ ´ ´ BARANYAI KLARA, HOLICS LASZL O, ´ ´ HONYEK GYULA, OLOSZ BALAZS, SZASZ ´ SZECHENYI ´ ´ KRISZTIAN, GABOR, VIGH ´ E, ´ VLADAR ´ KAROLY, ´ MAT WOYNAROVICH FERENC Az informatika bizotts´ag vezet˝oje: ´ ´ SCHMIEDER LASZL O ´ E, ´ FARKAS CSABA, FODOR Tagjai: BUSA MAT ´ ´ NIKOLETT, LOCZI ´ ZSOLT, LASZL O LAJOS, ´ ´ ´ SIEGLER GABOR, SZENTE PETER, TOTH ´ TAMAS ´ ANDREA, TASNADI ´ ANIKO ´ Ford´ıt´ok: GROF ´ ´ ¨ Szerkeszt˝os´egi titk´ar: TRASY GYORGYN E A szerkeszt˝os´eg c´ıme: 1117 Budapest, P´azm´any P´eter s´et´any 1/C III. emelet 3.405. Telefon: 372-2850 A lap megrendelhet˝o az Interneten: www.komal.hu/megrendelolap/reszletek.h.shtml. El˝ofizet´esi d´ıj egy ´evre: 8800 Ft K´eziratokat nem ˝orz¨unk meg ´es nem k¨uld¨unk vissza. Minden jog a K¨oMaL tulajdonosai´e. E-mail: [email protected] Internet: http://www.komal.hu This journal can be ordered from the Editorial office: P´azm´any P´eter s´et´any 1/C III. emelet 3.405. 1117–Budapest, Hungary telephone: +36 (1) 372-2850 or on the Postal address H–1518 Budapest 112, P.O.B. 32, Hungary, or on the Internet: www.komal.hu/megrendelolap/reszletek.e.shtml. A Lapban megjelen˝o hirdet´esek tartalm´a´ert felel˝oss´eget nem v´allalunk.

193

Kivonat Cikk¨ unkben megmutatjuk, hogy a t´eglalap ter¨ uletk´eplete levezethet˝ o n´eh´ any ´eszszer˝ u feltev´esb˝ ol. Megk¨ ozel´ıt´es¨ unkben a Cauchy-f´ele f¨ uggv´enyegyenlet j´ atszik kulcsszerepet, melynek matematikai ´es t¨ ort´eneti vonatkoz´ asaira szint´en kit´er¨ unk r¨ oviden.

1. Bevezet´es A ter¨ uletfogalom kialak´ıt´ asa m´ ar az a´ltal´ anos iskol´aban elkezd˝odik. Bevezet´esk´ent a t´eglalapot szok´ as vizsg´alni, r´eszben egyszer˝ us´ege” miatt, r´eszben, mert ” sz´amos alakzat ter¨ uletk´eplete ebb˝ ol sz´armaztathat´ o. ´Igy mindenki sz´ am´ara mag´at´ol ´ertet˝ od˝o t´eny, hogy a t´eglalap ter¨ ulete a k´et mer˝ oleges oldal hossz´ anak szorza´ a k´erd´es val´ ta. Am oj´aban nem az, hogy mennyi a t´eglalap ter¨ ulete, hanem hogy mi´ert pont ennyi. Az intu´ıci´ o ugyanis m´eg a ter¨ uletk´eplet megismer´ese el˝ ott sugalmaz bizonyos evidenci´ akat. Elv´arjuk, hogy a ter¨ ulet csak az oldalak hossz´ at´ ol f¨ ugg˝o nemnegat´ıv ´ert´ek legyen; elv´arjuk, hogy ha egy t´eglalapot b´ armelyik p´ arhuzamos oldalp´ arja ment´en azonos m´ert´ekben ´es ir´anyban meghosszabb´ıtva u ´jabb t´eglalapp´ a eg´esz´ıt¨ unk ki, akkor a k´et r´esz ter¨ ulet´enek ¨ osszege egyezzen meg az ¨ osszter¨ ulettel; v´egezet¨ ul elv´ arjuk, hogy az egys´egn´egyzet ter¨ ulete egys´egnyi legyen. Mindezeket szabatosan a k¨ ovetkez˝ o meg´ allapod´ asokban r¨ ogz´ıthetj¨ uk. (A tov´ abbiakban R a vaos, Q a racion´ alis, Q+ a nemnegat´ıv racion´ alis sz´ amok l´os, R+ a nemnegat´ıv val´ halmaz´ at jel¨oli.) uggv´eny. 1. meg´allapod´as. A ter¨ ulet egy T : R2+ → R+ f¨ 2. meg´allapod´as. Ha a, a1 , a2 , valamint b, b1 , b2 adott nemnegat´ıv sz´amok, akkor T (a1 + a2 , b) = T (a1 , b) + T (a2 , b) ´es T (a, b1 + b2 ) = T (a, b1 ) + T (a, b2 ). 3. meg´allapod´as. T (1, 1) = 1. Ezekre a tov´ abbiakban rendre nemnegativit´ asi, additivit´ asi ´es norm´ alts´ agi felt´etelk´ent fogunk hivatkozni. Els˝ ok´ent azt igazoljuk, hogy e h´ arom geometriai tartalm´ u meg´allapod´ as egy ´es csakis egy algebrai formul´ at eredm´enyezhet: az ´altal´ anos iskol´aban megismert ter¨ uletk´epletet. ¨ ond´ıj ´es az Innov´ A cikk a Bolyai J´ anos Kutat´ asi Oszt¨ aci´ os ´es Technol´ ogiai Miniszt´eri´ ´ Nemzeti Kiv´ um UNKP-20-5 k´ odsz´ am´ u Uj al´ os´ ag Programj´ anak t´ amogat´ as´ anak a Nemzeti Kutat´ asi, Fejleszt´esi ´es Innov´ aci´ os Alapb´ ol finansz´ırozott szakmai t´ amogat´ as´ aval k´esz¨ ult.

194

K¨ oz´ episkolai Matematikai ´ es Fizikai Lapok, 2022/4

2. Ennyi a t´eglalap teru ¨lete

Teh´at

Tegy¨ uk fel, hogy az A ⊆ R halmaz z´ art az ¨osszead´ asra, azaz b´ armely k´et elem´evel egy¨ utt azok ¨ osszeg´et is tartalmazza. Azt mondjuk, hogy az f : A → R f¨ uggv´eny addit´ıv, ha minden x, y ∈ A eset´en teljes¨ ul r´ a az u ´gynevezett Cauchy-f´ele f¨ uggv´enyegyenlet: (1)

f (x + y) = f (x) + f (y).

Az egyenlet ´altal´ anos megold´as´ anak le´ır´ asa nem k¨ onny˝ u feladat. Nyilv´ an az identit´ as sz´amszorosa, vagyis egy f (x) = cx alak´ u f¨ uggv´eny mindig addit´ıv. S˝ ot, az alis v´ alaszt´ as ´es a nemnegativit´ asi felt´etel mellett valamennyi addit´ıv A = R+ speci´ f¨ uggv´eny csakis ilyen alak´ u lehet: uggv´eny, akkor minden x ∈ R+ eset´en T´etel. Ha f : R+ → R+ addit´ıv f¨ f (x) = f (1)x. Bizony´ıt´as. Els˝ ok´ent megmutatjuk, hogy f racion´ alisan homog´en, azaz minul. Az (1) egyenletben az den r ∈ Q+ ´es minden x ∈ R+ eset´en f (rx) = rf (x) teljes¨ x = y helyettes´ıt´est alkalmazva kapjuk, hogy f (2x) = 2f (x). Innen teljes indukci´ oval igazolhat´ o, hogy f (nx) = nf (x) teljes¨ ul minden n pozit´ıv eg´esz eset´en. Legyen most r = m/n alakban adott, ahol m ´es n pozit´ıv eg´eszek. Az el˝obbi tulajdons´ agot k´etszer felhaszn´ alva,     m  1 1 m m x = mf x = · nf x = f (x) = rf (x). f (rx) = f n n n n n

M´ asodszor azt igazoljuk, hogy f monoton n¨ ovekv˝o. Legyenek x  y nemnegat´ıv sz´amok. Ekkor y − x is nemnegat´ıv, ´ıgy az f additivit´ asa ´es a nemnegativit´ asa miatt   f (y) = f (y − x) + x = f (y − x) + f (x)  f (x).

A harmadik ´es egyben utols´ o l´ep´esben megmutatjuk, hogy minden x ∈ R+ eset´en az f (x) ´es az f (1)x ´ert´ekek elt´er´ese ak´ armilyen pozit´ıv sz´amn´ al kisebb. Ez ugyanis pontosan azt jelenti, hogy az elt´er´es nulla, azaz f (x) = f (1)x. Jel¨olje [x] az x val´os sz´am (als´o) eg´eszr´esz´et. Mivel minden n ∈ N ´es minden x ∈ R+ eset´en [nx]  nx < [nx] + 1, ez´ert

[nx] 1 [nx] 1 rn := x< + = rn + . n n n n Azonban f racion´ alisan homog´en ´es monoton n¨ ov˝ o, ez´ert ebb˝ ol az egyenl˝otlens´egl´ ancb´ol k¨ ovetkezik, hogy     1 1 rn f (1) = f (rn )  f (x)  f rn + = rn + f (1). n n otlens´eget, akkor itt f (1)  0 miatt Ha m´eg f¨ olhaszn´ aljuk az x − n1  rn  x egyenl˝ a bal oldal als´o, m´ıg a jobb oldal fels˝o becsl´essel folytathat´ o:     1 1 f (1)  f (x)  x + f (1). x− n n K¨ oz´ episkolai Matematikai ´ es Fizikai Lapok, 2022/4

195

  f (x) − f (1)x  f (1) n teljes¨ ul minden n ∈ N ´es minden x ∈ R+ eset´en, ´es ´eppen ezt akartuk bel´atni.



R¨ogz´ıtett nemnegat´ıv b sz´ am mellett az a → T (a, b) f¨ uggv´eny nemnegat´ıv ´es addit´ıv az els˝ o k´et meg´ allapod´ as ´ertelm´eben, ´ıgy a fenti t´etel miatt T (1, b)a alak´ u. Hasonl´ oan, a b → T (1, b) f¨ uggv´eny sz¨ uks´egk´eppen T (1, 1)b alak´ u. Vagyis meg´allapod´ asaink egy´ertelm˝ uen meghat´ arozz´ ak a t´eglalap ter¨ uletk´eplet´et: T (a, b) = T (1, b)a = T (1, 1)ab = ab. A ter¨ uletfogalom megalapoz´as´ anak most bemutatott u ´tja Legendre munk´ ass´ ag´ara eredeztethet˝ o [5]. Meg´allapod´ asai l´enyeg´eben a mieinkhez hasonl´ oak, egyetlen apr´ o elt´er´essel: nincs felt´etelezve a ter¨ ulet nemnegativit´asa. R¨ ovidesen l´ atni fogjuk, hogy e felt´etel hi´ any´ aban az el˝oz˝ o t´etel ´erv´eny´et veszti. 3. Kitekint´es: a Cauchy-f´ele fu ¨ggv´enyegyenlet Mint eml´ıtett¨ uk, az identit´ as konstansszorosa, vagyis egy cx alak´ u f¨ uggv´eny addit´ıv a val´ os egyenesen. Az ilyen t´ıpus´ u megold´ asokat regul´ aris megold´ asnak nevezz¨ uk. Mit ´all´ıthatunk a nem regul´ aris addit´ıv val´ os f¨ uggv´enyekr˝ ol? A tov´ abbiakban erre a k´erd´esre keres¨ unk v´ alaszt a koordin´ atageometria seg´ıts´eg´evel. A Descartes-f´ele s´ık, azaz R2 rendezett sz´ amp´ arjait hol pontoknak, hol vektoroknak fogjuk tekinteni. Ez ugyanis sohasem okoz majd f´elre´ert´est, viszont adott esetben biztos´ıtja a v´ alaszt´ as k´enyelm´et. A szok´ asos m´odon k´et ilyen vektor ¨ osszege a komponensek ¨ osszeg´eb˝ ol k´epzett vektor, egy vektor sz´amszorosa pedig az eredeti komponensek sz´amszoros´ab´ ol ´ all´ o vektor. Egy vektor norm´ aja alatt a Pitagoraszt´etel szerint sz´am´ıtott euklideszi hossz´at ´ertj¨ uk: ahol v = (x, y) ∈ R2 . v := x2 + y 2 , Az ´ıgy bevezetett norma pozit´ıv definit, abszol´ ut homog´en ´es szubaddit´ıv. Azaz, minden v, w ∈ R2 ´es minden λ ∈ R eset´en • v  0, ´es egyenl˝ os´eg pontosan akkor a´ll, ha v a nullvektor; • λv = |λ| · v;

• v + w  v + w.

K¨onnyen ellen˝ orizhet˝ o, hogy a norma seg´ıts´eg´evel a p k¨ oz´eppont´ u, ε > 0 sugar´ u ny´ılt k¨orlap a k¨ ovetkez˝ o m´ odon adhat´ o meg:

U (p, ε) := v ∈ R2 : v − p < ε .

Az U (p, ε) k¨orlemezt szok´ as a p pont ε sugar´ u k¨ ornyezetek´ent is eml´ıteni. Az f :   2 R → R f¨ uggv´eny gr´ afja alatt az { x, f (x) ∈ R | x ∈ R} halmazt ´ertj¨ uk. Vil´ agos, hogy regul´aris addit´ıv val´os f¨ uggv´eny gr´ afja egy orig´on a´thalad´ o egyenes. Teljesen m´ask´ent viselkednek a nem regul´aris addit´ıv f¨ uggv´enyek:

196

K¨ oz´ episkolai Matematikai ´ es Fizikai Lapok, 2022/4

T´etel. Ha f : R → R nem regul´ aris addit´ıv f¨ uggv´eny, akkor gr´ afja minden¨ utt s˝ ur˝ u a Descartes-f´ele s´ıkon. Azaz, a s´ık minden pontj´ anak minden k¨ ornyezete tartalmaz gr´ afpontot. Bizony´ıt´as. Tegy¨ uk fel, hogy f : R → R nem regul´ u.  aris, azaz  nem cx alak´ os sz´amok, hogy az x1 , f (x1 ) ´es az x2 , f (x2 ) Ekkor l´eteznek olyan x1 = x2 val´ pontok k´et k¨ ul¨ onb¨ oz˝ o orig´ on ´atmen˝ o egyenest hat´aroznak meg. M´ask´eppen fogalmazva, a     v1 = x1 , f (x1 ) ´es v2 = x2 , f (x2 )

vektorok egy nem elfajul´ o paralelogramm´ at fesz´ıtenek ki a Descartes-f´ele s´ıkon. ar tetsz˝olegesen adott. Azt kell megmuLegyen most a p ∈ R2 pont ´es az ε > 0 sug´ tatnunk, hogy az U (p, ε) ny´ılt k¨ orlemez tartalmaz f gr´ afj´ar´ ol valamilyen q pontot. o Mivel a v1 ´es v2 vektorok nem p´arhuzamosak, ez´ert a s´ık minden pontja el˝o´all´ıthat´ os sz´amok, hogy a seg´ıts´eg¨ ukkel. Azaz, l´eteznek olyan α1 ´es α2 val´ p = α1 v 1 + α2 v 2 . V´ alasszunk olyan r1 ´es r2 racion´ alis sz´ amokat, amelyek el´eg k¨ozel vannak az α1 ´es utthat´ okhoz az al´ abbi ´ertelemben: α2 egy¨ |α1 − r1 |
0 ´es p < 0 ⇒ − < p < 0. 2 2   A f¨ uggv´enynek akkor van h´arom k¨ ul¨ onb¨ oz˝ o val´ os z´erushelye, ha p ∈ − 12 ; 0 .

Megjegyz´es. Ha ismerj¨ uk a harmadfok´ u egyenlet Cardano-k´epletes megold´ as´ at, akkor a casus irreducibilis (F¨ uggv´enyt´ abl´ azat: 23. oldal) eset v´egigsz´ amol´ as´ aval a fenti eredm´enyre jutunk.

b) A lok´ alis sz´els˝ o´ert´ek l´etez´es´enek sz¨ uks´eges felt´etele, hogy a f¨ uggv´eny els˝o deriv´ altja 0 legyen. 2

g  (x) = 3x2 + 2bx + c; 0 = 3x2 + 2bx + c; D = (2b) − 4 · 3c = 4b2 − 12c = 4(b2 − 3c). Ha D > 0, akkor az egyenletnek k´et k¨ ul¨ onb¨ oz˝ o val´os gy¨oke van, ebben az esetben a f¨ uggv´enynek van lok´ alis maximuma is ´es lok´ alis minimuma is. (Ez egy harmadfok´ u f¨ uggv´enyn´el el´egs´eges felt´etel is.) Ha D < 0, akkor az egyenletnek nincs val´os gy¨oke, a f¨ uggv´enynek nem lehet helyi sz´els˝ o´ert´eke. Ha D = 0, akkor az egyenletnek egy val´ os gy¨ oke van. Ebben az esetben sem lehet ez az (esetleges) sz´els˝ o´ert´ek egyszerre maximum is ´es minimum is (ekkor az egyenlet gy¨ oke az inflexi´os pont els˝ o koordin´ at´ aj´aval egyezik meg). Az esem´eny szempontj´ ab´ol teh´at ez is kedvez˝ o esetnek sz´ am´ıt. Ism´et t´ abl´ azatba foglaltuk az ¨ osszes esetet.

C. 1703. Az a ´es b 10-es sz´ amrendszerbeli term´eszetes sz´ amok, mindegyik sz´ amjegy¨ uk 1-es. Mutassuk meg, hogy ha a ´es b nem relat´ıv pr´ımek, akkor sz´ amjegyeik S(a) ´es S(b) ¨ osszege sem az. Megold´as. Az a ´es a b sz´ am egyike sem lehet 1, mert az 1 b´armelyik eg´esz sz´ ammal relat´ıv pr´ım, ez´ert a > 1 ´es b > 1. 1. eset Ha a = b, akkor S(a) = S(b) > 1, ´ıgy a feladat ´ all´ıt´ asa igaz. 2. eset Ha a = b, akkor az ´altal´ anoss´ ag megs´ert´ese n´elk¨ ul feltehetj¨ uk, hogy am utols´o sz´ amjegye 1, ez´ert sem 2-vel, sem a > b, ekkor S(a) > S(b). Mindk´et sz´ 5-tel nem oszthat´ oak ´es mivel nem relat´ıv pr´ımek, ´ıgy van k¨ oz¨ os pr´ımoszt´ ojuk. Legyen ez a k¨oz¨os pr´ımt´enyez˝ o p, ekkor p = 2 ´es p = 5. Innent˝ ol elfogyasztjuk”a sz´ am” jegyeket a k¨ovetkez˝ o m´ odszerrel. Kivonjuk a-b´ ol b-t, ´ıgy p | a ´es p | b miatt nyilv´ an p | (a − b) is teljes¨ ul. Ekkor (a − b) egy olyan pozit´ıv sz´am, amely (S(a) − S(b)) darab 1-essel kezd˝ odik, k¨ ozvetlen¨ ul ut´ anuk pedig S(b) darab nulla van. A null´ akt´ ol onnyed´en megszabadulhatunk”, ha elosztjuk 10S(b) -nel, ´es mivel term´eszetesen k¨ ” a t´ızhatv´ anyok csak 2-vel ´es 5-tel oszthat´ oak, ez´ert h´ anyadosk´ent (nevezz¨ uk c-nek) egy olyan csupa 1-es sz´amjegyet tartalmaz´o sz´ amot kapunk, amelyre p | c, ez´ert c > 1, valamint S(c) = S(a) − S(b) ´es c < a is igaz. Ekkor el˝ ofordulhat, hogy b = c, ekkor nyilv´an S(c) | S(b), ez´ert S(c) | S(c) + S(b) = S(a). Tal´altunk egy k¨ oz¨ os oszt´ ot, S(c)-t, ami (c > 1 miatt) 1-n´el nagyobb, ´ıgy bel´ attuk, hogy S(a) ´es S(b) nem relat´ıv pr´ımek. Ha b = c, akkor a nagyobbikb´ ol kivonjuk a kisebbiket ´es megism´etelj¨ uk az el˝obb ismertetett elj´ ar´ ast. V´eges sok l´ep´esben biztosan eljutunk od´ aig, hogy k´et egyenl˝ o sz´ amot kapunk, amelyek nagyobbak 1-n´el, mindegyik sz´amjegy¨ uk 1, ´ıgy sz´ amjegyeik ¨osszege is nagyobb 1-n´el ´es oszt´ oja S(a)-nak ´es S(b)-nek, azaz ut´obbiak nem relat´ıv pr´ımek. Ezzel a gondolatmenet v´eg´ere ´ert¨ unk, a feladat a´ll´ıt´as´ at bel´attuk. Egyh´ azi Hanna (Budapest, ELTE Ap´ aczai Cs. J. Gyak. Gimn. ´es Koll., 12. ´evf.) dolgozata alapj´an ¨ Osszesen 25 dolgozat ´erkezett. 5 pontos 10, 4 pontos 3, 3 pontos 5 dolgozat. 1 pontot 3, 0 pontot 1 versenyz˝ o kapott. Nem versenyszer˝ u: 3 dolgozat.

A keresett val´ osz´ın˝ us´eg: P (A) = 16 = 49 = 0,4444. 36 K¨ oz´ episkolai Matematikai ´ es Fizikai Lapok, 2022/4

213

214

K¨ oz´ episkolai Matematikai ´ es Fizikai Lapok, 2022/4

A jav´ıt´ o megjegyz´esei. 1. A versenyz˝ ok egy r´esze ¨ osszekeverte az oszthat´ os´ ag fogalm´ at azzal, hogy k´et sz´ am relat´ıv pr´ım. Abb´ ol, hogy (a; b) > 1 nem k¨ ovetkezik, hogy a | b vagy b | a.

2. A m´ asik komoly probl´ema a t´ ulzott a ´ltal´ anos´ıt´ as volt, p´eld´ aul a k¨ oz¨ os oszt´ o l´etez´es´enek megmutat´ asa helyett sokan azonnal k¨ oz¨ os oszt´ ot mutattak, amelyre legt¨ obbsz¨ or l´etezik trivi´ alis ellenp´elda. 3. V´eg¨ ul, s´ ulyos elvi hibak´ent el˝ ofordult, hogy csak n´eh´ any konkr´et p pr´ımre bizony´ıtotta a versenyz˝ o az ´ all´ıt´ ast, de ´ altal´ anosan nem.

Matematika feladatok megold´asa

Legyen a n´egysz¨ og ter¨ ulete T , ´ıgy a t¨ ukr¨ oz´essel kapott konk´ av hatsz¨ og ter¨ ulete 2T . Enn´ e l biztosan nagyobb a paralelogramma k´ e t szomsz´ e dos oldal´ a nak szorzata  √ ag legfeljebb x 8 − x , mert a paralelogramma AD = x alapj´ahoz tartoz´o magass´ √  AA = 8 − x, √  2T < x 8 − x .

A befejez´eshez haszn´ aljuk fel a sz´ amtani-m´ertani k¨ ozepek k¨ oz¨ otti egyenl˝ otlens´eget:

innen azonnal a

√  √  x+ 8−x √ = 2, x 8−x  2 2T < x

vagyis az igazoland´ o T < 1 ad´ odik. B. 5139. Az ABCD konvex n´egysz¨ og ´ atl´ oinak metsz´espontja M . Az ADM h´ aromsz¨ og ter¨ ulete nagyobb a BCM h´ aromsz¨ og ter¨ ulet´en´el. A n´egysz¨ o√g BC oldal´ anak felez˝ opontja P , AD oldal´ anak felez˝ opontja pedig Q, AP + AQ = 2 . Bizony´ıtsuk be, hogy ekkor az ABCD n´egysz¨ og ter¨ ulete kisebb, mint 1. (5 pont) Megold´as. El˝ osz¨ or megmutatjuk, hogy az ABCD n´egysz¨ogben β + γ > 180◦ (1. ´ abra). A felt´etel szerint T (ADM ) > T (BCM ), ez´ert az AM szakasz belsej´eben l´etezik olyan E pont, amelyre T (EM D) = T (BCM ). Ismert tov´ abb´ a, hogy ez akkor ´es csak akkor teljes¨ ul, ha az EBCD n´egysz¨ og trap´ez, u ´gy hogy EB ´es CD a trap´ez alapjai. Ekkor EBC + γ = 180◦ , azonban az E pont az AM szakasz bels˝o pontja, azaz EBC < β, ´ıgy β + γ > 180◦ .

1. a ´bra

2. a ´bra

T¨ ukr¨ ozz¨ uk a n´egysz¨ oget a BC oldal P felez˝opontj´ara ´es haszn´ aljuk a 2. ´ abra jel¨ ol´eseit. av hatsz¨oget alkot, mert A t¨ ukr¨ oz´esn´el ABD A CD eg´eszen biztosan konk´ al. L´atjuk teh´ at, B-n´el ´es C-n´el l´etrej¨ ott sz¨ ogei a fentiek alapj´an nagyobbak 180◦ -n´ hogy az AD A D paralelogramma ter¨ ulete nagyobb a hatsz¨og ter¨ ulet´en´el. √ √ √ AD + AA = 2 2 = 8, ez´ert ha AD = x, akkor AA = 8 − x. K¨ oz´ episkolai Matematikai ´ es Fizikai Lapok, 2022/4

215

√

 8 − x  2,

Csonka Ill´es (Ciszterci Rend Nagy Lajos Gimn., P´ecs, 9. ´evf.) dolgozata alapj´an ¨ Osszesen 44 dolgozat ´erkezett. 5 pontot kapott 36, 4 pontot 5 tanul´ o. 3 pontos 2, 1 pontos 1 versenyz˝ o dolgozata.

B. 5149. H´ anyf´elek´eppen lehet kit¨ olteni egy 6 × 6-os t´ abl´ azat mez˝ oit az 1, 2, . . . , 36 sz´ amokkal u ´gy, hogy b´ arhogy v´ alasztunk 6 mez˝ ot, melyek k¨ oz¨ ul semelyik kett˝ o nincs egy sorban vagy oszlopban, a kiv´ alasztott mez˝ okbe ´ırt sz´ amok ¨ osszege mindig ugyanannyi legyen? (6 pont) Megold´as. El˝ osz¨ or oldjuk meg a k¨ ovetkez˝ o feladatot: h´ any, az 1-est tartalmaz´ o S r´eszhalmaza van a H := {1, 2, . . . , 36} halmaznak, ha S-nek l´etezik 6 diszjunkt eltoltja, melyek uni´ oja H? Tegy¨ uk fel, hogy S egy ilyen r´eszhalmaz; bontsuk az S-ben tal´ alhat´ o elemeket nem ´erintkez˝o intervallumok uni´ oj´ara; megmutatjuk, hogy ezek az intervallumok egyenl˝ o hossz´ uak. Legyen az els˝o intervallum hossza x; ennek az intervallumnak o elaz elemei: 1, 2, . . . , x. Az itt nem szerepl˝o x + 1-et az S halmaz x-szel val´ toltja tartalmazza. Ha S-ben lenne egy m´ asik, y > x hossz´ us´ ag´ u intervallum, akkor az nem lenne diszjunkt az x-szel val´ o eltoltj´ aval. Hasonl´oan bel´ athat´ o, hogy az utols´o intervallum is legal´abb olyan hossz´ u, mint a leghosszabb intervallum. Ha nem mindegyik intervallum ugyanolyan hossz´ csak (2, 1, 1, 2)

u, akkor a hosszak sorozata lehet, ez azonban nem lehets´eges: S = [1, 2], [a], [b], [c, c + 1] -ben a, b, c, c + 1 > 3

miatt 3-at csak S-nek a 2-vel val´o eltoltja, S + 2 = [3, 4], [a + 2], [b + 2], [c + 2, c + 3]

tartalmazhatja. Hasonl´ oan, az ebben nem szerepl˝ o 5-¨ ot csak S + 4 = [5, 6], [a + 4], [b + 4], [c + 4, c + 5] tartalmazhatja, stb. Az a + 1 sem S-nek, sem pedig S + 2-nek nem eleme, ez´ert a + 1  5, ´ıgy a + 1 az S + 4, S + 6, S + 8, S + 10 eltoltak valamelyik´eben, sz¨ uks´egszer˝ uen az [5, 6], [7, 8], [9, 10], [11, 12] intervallumok egyik´eben van. Ez azt jelenti, hogy a viszont az S + 2, . . . , S + 10 eltoltak egyik´ehez tartozik – akkor azonban ez nem lenne diszjunkt S-sel. 216

K¨ oz´ episkolai Matematikai ´ es Fizikai Lapok, 2022/4

Ha az intervallumok egyenl˝o hossz´ uak, hosszuk a 6 oszt´ oja, 1,2,3 vagy 6 lehet. Ha 6, akkor – mivel S tartalmazza az 1-et – S csak {1, 2, 3, 4, 5, 6} lehet. Ha a k¨ oz¨ os hossz 3: A k´et intervallum k¨ ozti t´ avols´ ag 3-mal oszthat´o (kit¨ olthet˝ o 3 hossz´ u intervallumokkal), ami lehet 6, 9, vagy

18, ´ıgy a halmazok: S ∈ {1, 2, 3, 7, 8, 9}, {1, 2, 3, 10, 11, 12}, {1, 2, 3, 19, 20, 21} (az els˝ ot 3-mal eltolva egy 12, a m´asodikat 3-mal k´etszer eltolva egy 18, a harmadikat 3-mal ¨otsz¨or eltolva egy 36 hossz´ u, lefedett intervallumot kapunk, ezeket rendre 2-szer, 1-szer, illetve 0-szor ism´etelve megkapjuk az {1, . . . , 36} halmazt).

Ha a k¨ oz¨ os hossz 2: Az els˝o ´es a m´asodik, illetve a m´asodik ´es a harmadik intervallum k¨ ozti t´ avols´ ag ugyanaz (k¨ ul¨ onben a kor´ abbiakhoz hasonl´ as

o ellentmond´ ad´ odik), ez a t´ avols´ ag lehet 4, 6, vagy 12, teh´at a halmazok: S ∈ {1, 2, 5, 6, 9, 10}, {1, 2, 7, 8, 13, 14}, {1, 2, 13, 14, 25, 26} (az els˝ ot 2-vel tolva egy 12, a m´asodikat 2-vel k´etszer eltolva egy 18, a harmadikat 2-vel ¨ otsz¨or eltolva egy 36 hossz´ u, lefedett intervallumot kapunk, ezt rendre 2-szer, 1-szer ´es 0-szor ism´etelve megkapjuk az {1, . . . , 36} halmazt).

ast, amely a kor´abbi 7 megfeHa a k¨ oz¨ os hossz 1: Alkalmazzuk azt az o¨ndualit´ lel˝o r´eszhalmazt a megfelel˝ o, 1 hossz´ u intervallumokkal b´ır´o r´eszhalmazokhoz p´ aros´ıtja, u ´gy, hogy egy r´eszhalmaz du´ alisa a hozz´ a tartoz´ o eltol´ ashalmaz legyen! Mivel minden kor´ abbi r´eszhalmazt legal´ abb 2-vel kellett eltolnunk, teh´ at az intervallumok t´ avols´ aga a du´ alisban legal´abb 2, tov´ abb´ a az eltol´ as kommutat´ıv m˝ uvelet, a kor´ abbi eltol´ asokban felcser´elve a tagok sorrendj´et az uni´o az {1, . . . , 36} marad, tov´ abb´ a du´alis du´ alisa az eredeti r´eszhalmaz. ´Igy 7 ilyen r´eszhalmaz van, teh´ at ¨osszesen 1 + 3 + 3 + 7 = 14 a megfelel˝o S r´eszhalmazok sz´ ama.

Az ilyen t´ abl´ azatokat (6!) · (6!) f´elek´eppen lehet visszarendezni, vagyis pontosan 2 14 · (6!) j´o kit¨ olt´es l´etezik. N´emeth M´ arton (Nagykanizsa, Batthy´ any L. Gimn., 10. ´evf.)

´ 49 dolgozat ´erkezett. 6 pontot kapott 16 versenyz˝ o: Argay Zsolt, B´ an-Szab´ o Aron, Baski Bence, Csizmadia Mikl´ os, Duchon M´ arton, Heged˝ us D´ aniel, Kalocsai Zolt´ an, Kercs´ o-Moln´ ar Anita, Kov´ acs Tam´ as, K¨ ok´enyesi M´ ark P´eter, M´ oricz Benj´ amin, N´ ador ´ Benedek, N´emeth M´ arton, Seres-Szab´ o M´ arton, Szanyi Attila, T¨ or¨ ok Agoston. 5 pontos 6, 4 pontos 4, 3 pontos 4, 2 pontos 4, 1 pontos 5, 0 pontos 10 dolgozat.

B. 5173. Az ABC hegyessz¨ og˝ u h´ aromsz¨ og magass´ agpontja H, k¨ or¨ ul´ırt k¨ or´enek k¨ oz´eppontja O. Legyen D ´es E az AB, illetve AC szakasz bels˝ o pontja. Az ADE h´ aromsz¨ og magass´ agpontja ´es k¨ or¨ ul´ırt k¨ or´enek k¨ oz´eppontja H  , illetve O . Mutas  arhuzamosak, ha suk meg, hogy a HH ´es OO egyenesek akkor ´es csak akkor p´ BD = CE. (6 pont)

´ Javasolta: B´ an-Szab´ o Aron (Budapest)

Megold´as. Jel¨ olj¨ uk az ABC-ben az AB, illetve AC oldalakhoz tartoz´ o magass´ agvonalak talppontj´ at T1 -gyel ´es T2 -vel, az ADE-ben az AD ´es AE oldalakhoz tartoz´ o magass´ agvonalak talppontj´at pedig T3 -mal ´es T4 -gyel. Legyen opontja T1 H ∩ T4 H  = M1 ´es T2 H ∩ T3 H  = M2 , az AB, illetve AC szakaszok felez˝ opontja F3 ´es F4 , tov´ abb´ a F1 O ∩ F4 O  = F1 ´es F2 , az AD, illetve AE szakaszok felez˝ = K1 ´es F2 O ∩ F3 O = K2 .

P´eld´ aul az S = {1, 2, 5, 6, 9, 10} du´ alis´ at a k¨ ovetkez˝ ok´eppen kaphatjuk meg. Az S megfelel˝o eltoltjai: ( 1, 2, 5, 6, 9, 10), ( 3, 4, 7, 8, 11, 12), (13, 14, 17, 18, 21, 22), (15, 16, 19, 20, 23, 24), (25, 26, 29, 30, 33, 34), (27, 28, 31, 32, 35, 36). Ekkor az S du´ alisa: (1, 3, 13, 15, 25, 27). T´erj¨ unk vissza az eredeti feladathoz. Egy megfelel˝o kit¨olt´esben jel¨olje a t´ abl´azat x-edik sor´ anak y-adik elem´et (x, y). Vegy¨ uk ´eszre, hogy ha tetsz´es szerint permut´ aljuk a t´ abl´ azat sorait ´es oszlopait, akkor tov´ abbra is egy megfelel˝o kit¨ olt´est kapunk. Ezzel el´erhetj¨ uk, hogy (alkalmas sor- ´es oszlopcser´ekkel) a t´abl´azat els˝ o sor´anak els˝o eleme az 1-es legyen. Tetsz˝oleges x1 , x2 , y1 , y2 eset´en (x1 , y1 ) + (x2 , y2 ) = (x1 , y2 ) + (x2 , y1 ), mivel egy, a feladat felt´eteleinek megfelel˝ o, o r´esz¨ osszegben felcser´elve az y1 -edik ´es az y2 -edik (x1 , y1 )-et ´es (x2 , y2 )-t tartalmaz´ oszlopot nem v´altozik az ¨ osszeg. Ekkor viszont (i, j) − (1, j) = (i, 1) − (1, 1), teh´at az i-edik sor az els˝o sor eltoltja, u ´gy, hogy az els˝o sor els˝ o cell´aj´aban az 1 van, teh´at az ´ıgy kaphat´ o t´abl´ azatok sz´ama 14. K¨ oz´ episkolai Matematikai ´ es Fizikai Lapok, 2022/4

217

Vegy¨ uk ´eszre, hogy az O ´es O pontok nem eshetnek egybe, mert p´eld´ aul D bels˝o pontja az ABC h´aromsz¨ og k¨ or´e´ırt k¨ or´enek, ´ıgy OD < OA, m´ıg O D = O A. 218

K¨ oz´ episkolai Matematikai ´ es Fizikai Lapok, 2022/4

Ez´ert az OO szakasz nem fajulhat egy pontt´a. Mivel EAD = CAB hegyessz¨og, az ABC ´es az ADE h´aromsz¨ og magass´ agpontja sem lehet az A pont, teh´at H = A ´es H  = A. 

Az O K1 OK2 paralelogramma eset´eben az el˝ oz˝ oekhez hasonl´ oan azt kapjuk, hogy OK1 F2 F4 = .  O K1 F1 F3



Tekints¨ uk most a H M1 HM2 ´es az O K1 OK2 n´egysz¨ogeket, amelyeknek szemk¨ ozti oldalaik mind mer˝ olegesek AB-re, illetve AC-re, ez´ert mindk´et n´egysz¨og paralelogramma. Az egyik paralelogramma megfelel˝o oldalai p´ aronk´ent p´ arhuzamosak a m´ asik paralelogramma megfelel˝ o oldalaival, hiszen mind az oldalfelez˝ o mer˝ olegesek, mind a magass´ agvonalak mer˝ olegesek a megadott oldalra, tov´ abb´ a a megfelel˝o o sz¨ogek vannak, hiszen mindkett˝ ot cs´ ucsokn´ al (H  ↔ O , H ↔ O) ugyanakkora bels˝ arolja, amelyek k´et A-hoz k¨ ozelebbi egyenes (T3 H  , T4 H  , illetve F3 O ´es F4 O ) hat´ aromsz¨ogek azonos k¨op´ arhuzamosak. Ebb˝ ol k¨ ovetkez˝ oen a H  M1 H ´es az O K1 O h´ r¨ ulj´ar´ asi ir´ any´ uak, valamint k´et-k´et oldaluk p´ aronk´ent p´ arhuzamos (H  M1  O K1 ´es HM1  OK1 ), ´ıgy ha HH   OO , akkor a sz´ oban forg´o h´aromsz¨ogek hasonl´ ok, teh´ at megfelel˝o oldalaik hossz´anak ar´anya megegyezik: H  M1 O  K1 = . HM1 OK1

Sz´ amoljuk ki az F2 F4 ´es F1 F3 szakaszok hossz´at. Mivel F2 az AC szakasz , az F4 pedig az AE szakasz felez˝opontja, ´ıgy felez˝opontja, ez´ert AF2 = AC 2 AF4 =

AC − CE AE CE = = AF2 − , 2 2 2

amib˝ol F2 F4 =

CE 2

, ´ıgy k¨ovetkezik. Hasonl´ok´eppen F1 F3 = BD 2

Ez teh´at sz¨ uks´eges felt´etele annak, hogy HH   OO . Ez a felt´etel azonban el´egok, azaz s´eges is, mert ha teljes¨ ul, akkor a H  M1 H ´es O K1 O h´aromsz¨ogek hasonl´ a harmadik oldalp´ ar is p´ arhuzamos. Kimondhatjuk teh´ at, hogy HH   OO akkor ´es csakis akkor teljes¨ ul, ha

OK1 F2 F4 CE 2 . = = BD =  O K1 F1 F3 BD

O  K1 H  M1 = . HM1 OK1

HM1 OK1 BD CE Bel´ attuk teh´ at, hogy H es O osszevetve azt  M = CE ´  K = BD , amit (1)-gyel ¨ 1 1 kapjuk, hogy

Most vizsg´ aljuk meg a H  M1 HM2 paralelogramma oldalainak ar´any´ at. Ismert, hogy egy paralelogramma ter¨ ulet´et kisz´am´ıthatjuk egy oldal hossz´ anak ´es az oldalhoz tartoz´ o magass´ ag hossz´anak szorzatak´ent:

CE BD = , CE BD

(1)

TH  M1 HM2 = HM1 · T1 T3 = H  M1 · T2 T4 ,

CE 2

s mivel szakaszok hosszai pozit´ıv sz´amok, ebb˝ ol BD = CE

amib˝ ol azt kapjuk, hogy HM1 T2 T4 = ,  H M1 T1 T3 ahol a nevez˝ ok ´ert´eke nem 0, hiszen a feladat felt´etelei szerint E az AC szakasz bels˝ o pontja, ez´ert EH  nem eshet egybe CM1 -gyel. Mivel ABT2  ´es ACT1 , mer˝ oleges sz´ ar´ u hegyessz¨ ogek, ez´ert egyenl˝o nagys´ ag´ uak, ´ıgy egy megfelel˝o egybev´ ag´os´agi transzform´aci´ oval fed´esbe hozhat´ ok egym´assal. Ekkor alkalmazhatjuk a p´arhuzamos szel˝ oszakaszok t´etel´et a k¨ ovetkez˝ ok´eppen:

ami a fentiek ´ertelm´eben azt jelenti, hogy

K¨ oz´ episkolai Matematikai ´ es Fizikai Lapok, 2022/4

teh´ at

Kolesz´ ar Domonkos (Miskolc, Herman Ott´ o Gimn., 11. ´evf.), K¨ ok´enyesi M´ ark P´eter (Budapest, Szent Istv´ an Gimn., 11. ´evf.), ´ Isk., 8. ´evf.) Varga Boldizs´ ar (Ver˝ oce, G´eza Fejedelem Reform´atus Alt. ´ ¨ Osszesen 33 dolgozat ´erkezett. 6 pontot kapott: B´ an-Szab´ o Aron, Baski Bence, ´ am, Bencsik D´ Bencsik Ad´ avid, Bogn´ ar Andr´ as K´ aroly, Diaconescu Tashi, Duchon M´ arton, Fekete Rich´ ard, Heged˝ us D´ aniel, Kalocsai Zolt´ an, Kercs´ o-Moln´ ar Anita, Kolesz´ ar Domonkos, Kov´ acs Tam´ as, K¨ ok´enyesi M´ ark P´eter, Mohay Lili Veronika, Moln´ ar-Szab´ o Vilmos, M´ oricz Benj´ amin, N´ ador Benedek, P´ ah´ an Anita Dalma, Romaniuc Albert-Iulian, Seres-Szab´ o M´ arton, Simon L´ aszl´ o Bence, Somogyi Dalma, Terj´ek Andr´ as J´ ozsef, Varga Boldizs´ ar, Wiener Anna, Z¨ ombik Barnab´ as. 5 pontos 3, 4 pontos 3 dolgozat.

BD T2 T 4 , = T1 T3 CE

HM1 T2 T4 BD , = =  H M1 T1 T3 CE

k¨ovetkezik. Ez teh´at sz¨ uks´eges felt´etel, m´ ar csak azt kell megmutatnunk, hogy el´egs´eges is. Mivel az el˝ obbiekben csupa egyenl˝os´eggel dolgoztunk, ezek megford´ıtva is igazak, ´ıgy a bizony´ıt´ as v´eg´ere ´ert¨ unk.

BD HM1 =  . CE H M1 219

220

K¨ oz´ episkolai Matematikai ´ es Fizikai Lapok, 2022/4

B. 5188. Igazoljuk, hogy az ´erint˝ otrap´ez magass´ aga nem lehet nagyobb alapjai m´ertani k¨ ozep´en´el. (5 pont)

II. megold´as. Legyenek a trap´ez cs´ ucsai A, B, C ´es D ebben a sorrendben u ´gy, hogy AB ´es CD a trap´ez alapjai. Legyen a be´ırt k¨ or ´erint´esi pontja az AB, BC, CD ´es DA oldalakon rendre E, F , G ´es H, tov´ abb´ a a be´ırt k¨ or k¨ oz´eppontja O, sugara r (2. ´ abra). Mivel a k¨ orh¨ oz egy pontb´ol h´ uzott ´erint˝oszakaszok egyenl˝ o hossz´ uak, ez´ert

Javasolta: N´emeth L´ aszl´ o (Fony´od)

I. megold´as. Legyen az ABCD ´erint˝otrap´ez k´et alapj´ anak hossza AB = a ´es CD = b, amelyekre teljes¨ ul, hogy a  b. (Mivel az (A, B) ↔ (C, D) pontp´ arok cser´ej´evel az a´ll´ıt´ as o nmag´ a ba megy a ´ t, ezt mindig megtehetj¨ u k.) H´ u zzunk p´ a rhuzamost ¨ a D ponton a´t a BC sz´ arral, messe ez az AB alapot a P pontban, ekkor a  b miatt P az AB szakasz pontja (bele´ertve azt az esetet is, amikor P = A). Mivel a BCDP n´egysz¨ og k´et-k´et szemozti oldala p´ arhuzamos, ez´ert paralelogramma. k¨ A paralelogramma szemk¨ ozti oldalainak hossza egyenl˝ o, ez´ert BP = CD = b, amib˝ol AP = a − b, tov´ abb´ a DP = BC, amib˝ ol az ´erint˝on´egysz¨ og tulajdons´ agait felhaszn´ alva a + b = AB + CD = = BC + AD = AD + DP . T¨ ukr¨ ozz¨ uk a P pontot a CD alap egyeneabra). s´ere, legyen a t¨ uk¨ ork´epe a P  pont (1. ´ A t¨ ukr¨ oz´es miatt egyr´eszt nyilv´anval´o, hogy aga: P P  = 2m, ´ıgy az ´erint˝otrap´ez m magass´ PP  m = 2 , m´asr´eszt DP = DP . Ekkor az ADP  h´ aromsz¨ ogre (amely lehet elfajul´ o is) alkalmazzuk a h´ aromsz¨ og-egyenl˝ otlens´eget, ´ıgy

AH = AE =: x, BE = BF =: y, CF = CG =: z ´es DG = DH =: w, 2. a ´bra

Mivel a be´ırt k¨or k¨ oz´eppontja illeszkedik a szomsz´edos oldalak sz¨ ogfelez˝ oj´ere, AO ´es DO felezik a trap´ez A-n´ al, illetve D-n´el l´ev˝ o bels˝ o sz¨ og´et, teh´ at OAD + ADO =

(x + y)(z + w) = xz + xw + yz + yw = xz + yw + 2r2 .

Szint´en a t¨ ukr¨ oz´es k¨ ovetkezm´enye, hogy P P  ⊥CD  AB, teh´at P P  ⊥AB, ez´ert abban az esetben, amikor a P pont nem esik egybe A-val, az AP P  h´aromsz¨og der´eksz¨ og˝ u. A Pitagorasz-t´etel alapj´ an fel´ırhatjuk, hogy 2

2

2

2

BAD + ADC 180◦ BAD ADC + = = = 90◦ , 2 2 2 2

hiszen a trap´ez egy sz´ ar´ an fekv˝ o bels˝ o sz¨ ogei egym´ ast 180◦ -ra eg´esz´ıtik ki. Ismert, ◦ oge hogy a h´aromsz¨ og bels˝o sz¨ ogeinek ¨ osszege 180 , ekkor az AOD harmadik sz¨ at az AOD der´eksz¨ og˝ u. Mivel a k¨ orh¨ oz h´ uzott AOD = 180◦ − 90◦ = 90◦ -os, teh´ ´erint˝o mer˝oleges az ´erint´esi pontba h´ uzott sug´ arra, OH ⊥ AD, vagyis az OH(= r) szakasz az AOD a ´ tfog´ o hoz tartoz´ o magass´ a ga. Ekkor a magass´ agt´etel alapj´ an √ xw = r ⇒ xw = r2 . Teljesen hasonl´oan a m´ asik sz´ aron az yz = r2 teljes¨ ul, ´ıgy a trap´ez alapjainak szorzata:

1. a ´bra

AP   AD + DP  = AD + DP = a + b.

2

x, y, z, w > 0.

Az el˝oz˝ oek alapj´ an xw = r2 , ´ıgy

2

(P P  ) = (AP  ) − AP 2 = (AP  ) − (a − b)  (a + b) − (a − b) = 4ab,

xz =

amelyb˝ ol rendez´essel kapjuk a feladat a´ll´ıt´ as´ at: √ 2m = P P   2 ab .

yz

Ha P = A, akkor a = b = m miatt szint´en igaz az ´all´ıt´as. Mivel a bizony´ıt´as sor´ an csak egyetlenegy egyenl˝otlens´eg volt, ez´ert egyenl˝ os´eg pontosan akkor a´ll fenn, ha a h´ aromsz¨ og-egyenl˝ otlens´egben egyenl˝os´eg ´all, azaz A, D ´es P  kolline´ arisak. R¨ ovid sz¨ ogsz´ am´ıt´ assal k¨onnyen bel´ athat´ o, hogy ez pontosan AD = P D esetben teljes¨ ul. Ebb˝ol az k¨ ovetkezik, hogy AD = P D = BC, ez´ert az ABCD ´erint˝otrap´ez egy´ uttal h´ urtrap´ez is, ami az egyenl˝os´eg sz¨ uks´eges ´es el´egs´eges felt´etele. Varga Boldizs´ ar (B´ek´ asmegyeri Veres P´eter Gimn., 9. ´evf.) dolgozata alapj´an K¨ oz´ episkolai Matematikai ´ es Fizikai Lapok, 2022/4

r2 z xw ·z = · z = r2 , w w w

221

´es yz = r2 miatt yw = z · w = r2 w , amib˝ ol z z w + . xz + yw = r2 w z K¨ozismert, hogy b´armely pozit´ıv sz´amhoz a reciprok´ at hozz´aadva legal´abb 2-t z kapunk, ez´ert a t´avols´ agok pozitivit´ asa miatt: w +w  2. Mivel r2 pozit´ıv, ekkor z xz + yw = r2

z

w

+

w  2r2 , z

vagyis az alapok szorzata 2

(x + y)(z + w) = xz + yw + 2r2  2r2 + 2r2 = 4r2 = (2r) . 222

K¨ oz´ episkolai Matematikai ´ es Fizikai Lapok, 2022/4

Tudjuk, hogy az ´erint´esi pontba h´ uzott sug´ ar mer˝ oleges az ´erint˝ore, ez´ert OE ´es OG mer˝ oleges a trap´ez alapjaira, ´ıgy az alapok p´arhuzamoss´ aga miatt az O, E, G pontok egy egyenesre esnek, ´es ez az egyenes mer˝ oleges az alapokra. Ebb˝ ol k¨o2 vetkez˝ oen a trap´ez magass´ aga EG = OE + OG = 2r =: m, vagyis m2 = (2r) , ´ıgy az alapok szorzata nem kisebb a magass´ ag n´egyzet´en´el:

A k´et egyenletet ¨ osszeszorozva: m4 = 16xyzv, majd n´egyzetgy¨ ok¨ ot vonva (megtehetj¨ uk, mert mindk´et oldal pozit´ıv): √ m2 = 4 xyzv .

2

(x + y)(z + w)  (2r) = m2 . Pozit´ıv mennyis´egekr˝ ol l´ev´en sz´ o, az egyenl˝otlens´eg mindk´et oldal´ ab´ol n´egyzetgy¨ok¨ ot vonhatunk: (x + y)(z + w)  m,

Alkalmazzuk a sz´ amtani-m´ertani k¨ oz´ep k¨ oz¨ otti k¨ ozismert ¨ osszef¨ ugg´est (x ´es v, illetve y ´es z k¨ ozepeire): √ √ x+v y+z m2 = 4 xv yz  4 · · = (x + v)(y + z), 2 2

ami azt jelenti, hogy az alapok m´ertani k¨ ozepe nem kisebb a trap´ez magass´ ag´ an´al. Ezzel a feladat ´all´ıt´ as´at bel´attuk. Tudjuk, hogy b´ armely pozit´ıv sz´amnak ´es reciprok´anak ¨osszege legal´ abb 2, egyenl˝ os´eg pontosan akkor a´ll fenn, ha ez a sz´am az 1, vagyis jelen esetben akkor, z = 1 ⇒ z = w. Ez pedig pontosan a h´ urtrap´ezokban teljes¨ ul (ha az ´erint´esi ha w pont felezi az alapot, akkor a trap´ez szimmetrikus, ha pedig a trap´ez szimmetrikus, akkor az ´erint´esi pont felezi az alapot). ´ Isk. ´es Gimn., 10. ´evf.) Duchon M´ arton (Budapesti Fazekas M. Gyak. Alt. dolgozata alapj´an

majd helyettes´ıts¨ uk be a trap´ez (a = x + v ´es c = y + z) alapjait ´es vonjunk ism´et n´egyzetgy¨ok¨ot. ´Igy ´eppen az √ m  (x + v)(y + z) = ac

III. megold´as. K´esz´ıts¨ uk el a 3. ´ abr´ at. Az ´ abr´ an az egy pontb´ol h´ uzott ugyanolyan hossz´ u ´erint˝oket x, y, z, v-vel jel¨ oltem, illetve C ´es D mer˝ oleges vet¨ uletei AB-re T2 ´es T1 , ´ıgy DG = T1 E = y ´es GC = ET2 = z. Alkalmazzuk a Pitagoraszaromsz¨ ogre (az ´erint˝otrap´ez magass´ ag´at m-mel jel¨ol¨om): t´etelt a DAT1 h´

Ilyenkor a Pitagorasz-t´etelt alkalmazva az (x + y) = m2 + (y − x) egyenletet 2 2 kapjuk. Mivel (x − y) = (y − x) , ´ıgy ez sem befoly´ asolja a megold´ ast, ´es ugyan´ıgy nincs baj, ha y ´es z ar´ anyait v´ altoztatjuk.

2

2

2

(x + y) = m + (x − y) ,

egyenl˝ otlens´eghez jutunk, azaz a feladat ´all´ıt´ as´ at bel´attuk.

Diszkusszi´ o: Az x = y esetben l´etrej¨ ov˝ o elfajult der´eksz¨ og˝ u h´ aromsz¨ ogre is igaz az ´all´ıt´ as, az x < y esetet a 4. ´ abr´ an l´ athatjuk. 2

Egyenl˝ os´eg a k¨ ozepek miatt x = v ´es y = z eset´en ´ all fenn, vagyis ha a trap´ez nemcsak ´erint˝o-, hanem h´ urtrap´ez is. J´ anosik M´ at´e (Gy˝ or, R´evai Mikl´ os Gimn., 12. ´evf.) dolgozata alapj´an

m2 = 4xy.

¨ Osszesen 79 dolgozat ´erkezett. 4 pontos 61, 3 pontos 9, 2 pontos 4 dolgozat. 1 pontot 1, 0 pontot 3 versenyz˝ o kapott. Nem versenyszer˝ u: 1 dolgozat.

A BCT2 h´ aromsz¨og is der´eksz¨ og˝ u, ´ıgy az el˝ oz˝ oekhez hasonl´ oan: 2

2

2

(v + z) = m2 + (v − z) , m2 = 4vz.

Nehezebb feladat megold´asa∗

A. 812. K´et j´ at´ekos a k¨ ovetkez˝ o j´ at´ekot j´ atssza: van k´et kupac, melyekb˝ ol felv´ altva kell kavicsokat elvenni¨ uk, ´es az nyer, aki az utols´ o kavicsot elveszi. Ha a kupacok m´erete egy adott pillanatban A ´es B, akkor a soron k¨ ovetkez˝ o j´ at´ekos valamelyik kupacb´ ol elveheti A egy t¨ obbsz¨ or¨ os´et vagy B egy t¨ obbsz¨ or¨ os´et. ∗

3. a ´bra K¨ oz´ episkolai Matematikai ´ es Fizikai Lapok, 2022/4

Szeptembert˝ ol ism´et minden A-jel˝ u feladat megold´ asa megtal´ alhat´ o honlapunkon, ez az egyik k¨ oz¨ ul¨ uk.

4. a ´bra

223

224

K¨ oz´ episkolai Matematikai ´ es Fizikai Lapok, 2022/4

Hat´ arozzuk meg azokat az (k, n) sz´ amp´ arokat, melyekre a m´ asodik j´ at´ekosnak van nyer˝ o strat´egi´ aja, ha kezdetben az egyik kupacban k, a m´ asikban pedig n darab kavics van.

A C pontversenyben kit˝ uz¨ott gyakorlatok (1714–1720.)

Javasolta: P´ alv¨ olgyi D¨ om¨ ot¨ or (Budapest) Megold´as. Azt a´ll´ıtjuk, hogy pontosan akkor van a m´asodik j´ at´ekosnak nyer˝ o √

az u ´gynevezett aranymetsz´es. strat´egi´ aja, ha n  ϕk ´es k  ϕn, ahol ϕ = 5+1 2 2 Ismert, ´es k¨ onnyen ellen˝ orizhet˝ o, hogy ϕ − ϕ − 1 = 0, ezt az azonoss´agot t¨obbsz¨or fogjuk alkalmazni. Nevezz¨ unk egy helyzetet nyer˝ onek, ha onnan a kezd˝ o j´at´ekos nyer, veszt˝ onek, ha a m´asodik. Egy helyzet pontosan akkor nyer˝ o, ha onnan lehet veszt˝ ore l´epni, ´es akkor veszt˝ o, ha onnan csak nyer˝ ore lehet l´epni. Vil´agos, hogy a (0, k) ´es (k, 0) helyzetek nyer˝ ok, ´es a (k, k) helyzet veszt˝ o, mivel onnan csak (0, k)-ra vagy (k, 0)-ra lehet l´epni (k > 0), ´ıgy ezekben az esetekben t´enyleg igaz az ´all´ıt´ asunk. Mostant´ ol a (k, n) ´ allapotot vizsg´aljuk, ´es feltessz¨ uk, hogy n, k > 0 ´es k < n. El˝ osz¨ or tegy¨ uk fel, hogy (k, n) veszt˝ o helyzet, igazoljuk, hogy ekkor csak nyer˝ o helyzetre lehet l´epni. Azt tudjuk, hogy k < n  ϕk < 2k, ´ıgy ebb˝ ol az a´ll´asb´ ol o kett˝ o nyer˝ o, ´es csak a (0, n), (k, 0) ´es (k, n − k) helyzetekbe lehet l´epni. Az els˝ a harmadik is, mivel ϕ(n − k)  ϕ2 k − ϕk = k, ´es ϕ irracion´ alis, ´ıgy ϕ(n − k) < k. Most tegy¨ uk fel, hogy (k, n) nyer˝ o helyzet, azaz ϕk < n. Indirekten tegy¨ uk fel, hogy nem tudunk innen veszt˝o helyzetre l´epni. Osszuk el n-t marad´ekosan kval, legyen n = dk + r ahol r < k. Ekkor a (k, r) p´arra lehet l´epni, ´ıgy ϕr < k. Tov´abb´ a a ϕk < r + k egyenl˝ otlens´eg is teljes¨ ul, mivel vagy tudunk ide l´epni, ´es akkor az indirekt feltev´es miatt igaz, vagy r + k = n, ekkor az´ert igaz, mert (k, n) nyer˝ o helyzet. Ezt a k´et egyenl˝ otlens´eget ¨ osszevetve kapjuk, hogy ϕk < r + k
90◦ , a k¨ or¨ ul´ırt k¨ or k¨ oz´eppontja O. A k¨or¨ ul´ırt k¨orh¨ oz C-ben h´ uzott ´erint˝o az AB egyenest a P pontban, a P -b˝ ol BC-re ´all´ıtott mer˝ oleges pedig az OC egyenest Q-ban metszi. Igazoljuk, hogy AB mer˝ oleges AQ-ra.

C. 1718. Egy s´ıkon elhelyezt¨ unk 8 darab egys´egnyi ´el˝ u kock´ at, majd ezekre m´eg 5 darab egys´egkock´ at tett¨ unk az ´ abra szerint. Hat´ arozzuk meg az ABC h´ aromsz¨ og oldalainak hossz´ at.

(4 pont)

Feladatok 11. ´evfolyamt´ol C. 1719. Tekints¨ uk az ABC szab´ alyos h´aromsz¨og azon P bels˝o pontjait, ameogben l´ atszik. Bizony´ıtsuk be, hogy a P A, P B, lyekb˝ ol az AB oldal 135◦ -os sz¨ P C szakaszokb´ ol mindig szerkeszthet˝o h´ aromsz¨ og, ´es a P pont b´armely, a felt´etelnek megfelel˝o elhelyezked´ese eset´en ennek a h´ aromsz¨ognek az egyik sz¨oge mindig ugyanakkora. C. 1720. Adott egy 10 elem˝ u halmaz, amelynek elemei legfeljebb k´etjegy˝ u, pozit´ıv eg´esz sz´ amok. Igaz-e, hogy ennek a halmaznak mindig van k´et olyan diszjunkt r´eszhalmaza, amelyekben az elemek ¨ osszege egyenl˝o?

Javasolta: Nagy Zolt´ an L´ or´ ant (Budapest)

B. 5242. Legyenek m ´es n tetsz˝ oleges pozit´ıv eg´esz sz´ amok. Tekints¨ uk azon (x; y) r´acspontokat a der´eksz¨ og˝ u koordin´ atarendszerben, amelyekre 1  x  m ´es 1  y  n teljes¨ ul. Legfeljebb h´ anyat v´ alaszthatunk ki ezen mn darab r´ acspont ul u ´gy, hogy semelyik n´egy kiv´ alasztott pont se alkosson nem elfajul´ o paralek¨oz¨ logramm´ at? (6 pont)

Javasolta: F¨ uredi Erik (Budapest)

aromsz¨ og B. 5243. Az ABC h´ aromsz¨ ogben CAB = 48◦ ´es ABC = 54◦ . A h´ egy bels˝ o D pontj´ ara teljes¨ ul, hogy CDB = 132◦ ´es BCD = 30◦ . Igazoljuk, hogy az ACDB t¨or¨ ottvonalat alkot´ o szakaszokb´ ol nem szerkeszthet˝ o h´ aromsz¨ og. (5 pont)

 B. 5244. Hat´ arozzuk meg azokat az n > 4 eg´esz sz´ amokat, melyekre minden n-n´el kisebb k ¨ osszetett sz´ amra (k, n) > 1.

Beku ¨ld´esi hat´arid˝o: 2022. m´ajus 10. Elektronikus munkafu ¨zet: https://www.komal.hu/munkafuzet

(5 pont)



B. 5245. a) Bizony´ıtsuk be, hogy v´egtelen sok, p´aronk´ent nem hasonl´ o h´ aromsz¨og l´etezik, melynek mindh´ arom oldala eg´esz sz´ am, ´es az egyik sz¨ oge 3-szor akkora, mint egy m´ asik. b) A fenti tulajdons´ ag´ u h´ aromsz¨ ogek k¨ oz¨ ott van-e olyan, amelynek mindh´ arom oldala legfeljebb 10?

A B pontversenyben kit˝ u z¨ ott feladatok (5238–5245.)

(6 pont) B. 5238. Oldjuk meg a k¨ ovetkez˝ o egyenletet a pozit´ıv eg´esz sz´ amok k¨or´eben: (k + n)! = k 3 + n3 + (k + n)(3kn − 1). (3 pont)

Javasolta: R´ oka S´ andor (Ny´ıregyh´ aza)

Hujter Mih´ aly (Budapest) o ¨tlete alapj´an

´ unk el hat feh´er b´ abut egy sakkt´ abl´ ara k´et szok´ asos Aprilisi fejto o∗ . Helyezz¨ ¨r˝ k´eszletb˝ol u ´gy, hogy egy feket´et let´eve b´ armely szabad mez˝ore, az biztosan u ¨thet˝o legyen. Kov´ acs Bence (Szombathely) ´es Monos Attila (Budapest)

Javasolta: Szalai M´ at´e (Szeged)

B. 5239. Egy h´aromsz¨ og oldalai a, b ´es c, ebben a sorrendben sz´ amtani sorozatot alkotnak. Mutassuk meg, hogy a be´ırt k¨ or k¨ oz´eppontja harmadolja a b oldalhoz tartoz´ o sz¨ ogfelez˝ ot.

 Beku ¨ld´esi hat´arid˝o: 2022. m´ajus 10. Elektronikus munkafu ¨zet: https://www.komal.hu/munkafuzet

(3 pont)



B. 5240. Mutassuk meg, hogy minden n pozit´ıv eg´esz sz´ amnak van olyan t¨ obbsz¨ or¨ ose, amelyben a sz´ amjegyek ¨ osszege n. (4 pont) K¨ oz´ episkolai Matematikai ´ es Fizikai Lapok, 2022/4

Javasolta: S´ andor Csaba (Budapest)



227

228

Pontversenyen k´ıv¨ ul. Egy lehets´eges megold´ ast k¨ ozl¨ unk a m´ ajusi bor´ıt´ on.

K¨ oz´ episkolai Matematikai ´ es Fizikai Lapok, 2022/4

Az A pontversenyben kit˝ u z¨ ott nehezebb feladatok (824–826.) A. 824. at t¨om´enynek nevezz¨ uk, ha  Pozit´ıv sz´amok egy v´egtelen H halmaz´ minden 1/(n + 1), 1/n alak´ u intervallumban (ahol n tetsz˝ oleges pozit´ıv eg´esz sz´ am) van egy olyan sz´am, amely el˝ o´all k´et H-beli elem k¨ ul¨onbs´egek´ent. L´etezik-e olyan t¨ om´eny halmaz, amelyben a sz´ amok ¨ osszege v´eges? Javasolta: Sz˝ ucs G´ abor (Sziksz´o) uggv´enyt, amelyre tetsz˝ oleges A. 825. Keress¨ uk meg az ¨ osszes f : Z+ → R+ f¨ f (n+1)

n ´es k pozit´ıv eg´eszekre f (nk 2 ) = f (n)f 2 (k), tov´abb´ a f (n) tart 1-hez.

Rendelkez´es¨ unkre ´all a terkep.txt adatf´ ajl, amelynek els˝ o sor´ aban k´et eg´esz sz´ am (1  N, M  100) tal´ alhat´ o, a t´erk´ep sorainak ´es oszlopainak sz´ ama. Az ´allom´any k¨ovetkez˝ o N sor´ aban, soronk´ent M darab 0 ´es 100 k¨ oz¨ otti eg´esz sz´ am tal´ alhat´ o, a f´enyer˝ oss´eg ´ert´ekek. A t´erk´ep sz´el´en a f´enyer˝ oss´eg ´ert´eke mindenhol 0. Az ´allom´any soraiban az adatokat egy-egy sz´ok¨ oz v´ alasztja el egym´ast´ ol. K´esz´ıts¨ unk programot i562 n´even, amely az ´allom´any adatait felhaszn´alva a k¨ovetkez˝ o k´erd´esekre ad v´ alaszt. 1. Olvassuk be ´es t´ aroljuk el a terkep.txt ´ allom´any adatait, ´es annak felhaszn´al´as´aval oldjuk meg a k¨ ovetkez˝ o feladatokat. 2. Hat´ arozzuk meg, hogy a ter¨ ulet h´ any sz´ azal´eka nem s¨ ot´et teljesen. Az eredm´enyt k´et tizedesjegy pontoss´ aggal ´ırjuk ki. F´enyesnek nevezz¨ uk azokat a m´er´esi pontokat, amelyek nagyobb f´enyer˝ oss´eg˝ uek a n´egy k¨ozvetlen szomsz´edjukn´al. Minta bemenet (a f´enyes m´er´esi pontok f´elk¨ ov´er st´ılussal):

A. 826. Az antilop egy sakkb´abu, amely a husz´ arhoz hasonl´ oan l´ep: az (x1 ; y1 ) assal, ha mez˝ or˝ ol pontosan akkor ´erhet˝ o el az (x2 ; y2 ) mez˝o antilopugr´

|x1 − x2 |, |y1 − y2 | = {3, 4}.

u t´ abl´ azat mez˝ oit kit¨ oltj¨ uk az eg´esz sz´ amokkal 1-t˝ ol 1012 -ig. Egy 106 × 106 m´eret˝ Legyen D azon sz´amok halmaza, amelyek |a − b| alakban ´ırhat´oak, ahol az a-hoz tartoz´ o mez˝ or˝ ol el´erhet˝ o a b-hez tartoz´ o mez˝ o antilopugr´ assal. H´anyf´ele m´odon lehet elrendezni a sz´ amokat u ´gy, hogy D pontosan n´egy elemb˝ol a´lljon? Javasolta: Nikolai Beluhov (Bulgaria) Beku ¨ld´esi hat´arid˝o: 2022. m´ajus 10. Elektronikus munkafu ¨zet: https://www.komal.hu/munkafuzet



Bek¨ uldend˝o egy t¨ om¨ or´ıtett i562.zip ´ allom´anyban a program forr´ ask´ odja ´es r¨ovid dokument´ aci´ oja, amely megadja, hogy a forr´ as´ allom´any melyik fejleszt˝oi k¨ornyezetben ford´ıthat´ o.

Informatik´ab´ ol kit˝ u z¨ ott feladatok

´ Egy m˝ I. 562 (E). uhold seg´ıts´eg´evel t´eglalap alak´ u ter¨ uletr˝ ol f´enyer˝ oss´eg ´ert´ekeket m´ertek ´ejszakai id˝oszakban. A f´enyer˝ oss´eg 0 azon a helyen, ahol teljes a s¨ot´ets´eg, ´es 100, ahol a m˝ uszer ´erz´ekel˝ oje maximumot ´erz´ekel. A t´eglalap alak´ u ter¨ ulet N × M n´egyzet ter¨ uletegys´egb˝ ol ´ all, amelyek mindegyik´et egy-egy f´enyer˝ oss´eg ´ert´ek jellemez. A t´erk´ep sz´elein l´ev˝ ok kiv´etel´evel minden ter¨ uletegys´egnek n´egy k¨ozvetlen szomsz´edja van. K¨ oz´ episkolai Matematikai ´ es Fizikai Lapok, 2022/4

3. ´Irjuk a k´eperny˝ore a f´enyes m´er´esi pontok sz´am´at. 4. Hat´ arozzuk meg a legkisebb ter¨ ulet˝ u azon t´eglalap bal fels˝o ´es jobb als´ o sark´anak a koordin´at´ ait, amelyben az ¨ osszes f´enyes m´er´esi pont benne van. 5. Hat´ arozzuk meg annak a K × K-s n´egyzetnek a bal fels˝o koordin´ at´ ait, amelyikben a legt¨ obb f´enyes m´er´esi pont van. K ´ert´ek´et (1  K  min (N, M )) olvassuk be a billenty˝ uzetr˝ ol. Ha t¨ obb megold´ as van, akkor b´ armelyiket megadhatjuk.

229

I. 563. Anna ´es P´eter j´at´ek rulettel j´atszanak, illetve a j´ at´ek nyer´esi lehet˝ os´egeit tanulm´ anyozz´ ak. A rulettben 0-t´ ol 36-ig terjednek a sz´ amok. A rulettker´eken a sz´ amok egy k¨ orgy˝ ur˝ u 37 fi´ okj´ aban tal´alhat´ ok, ezek k¨ oz¨ ul z¨ old sz´ın˝ u a null´ a´e, a t¨obbi harminchat sz´am fele-fele fekete, illetve piros sz´ın˝ u dobozban foglal helyet. Most csak sz´ınekre fogadnak, amely nyer´es eset´en a t´et k´etszeres´et fizeti, veszt´esn´el a t´et a bank´e. Anna csak pirosra ´es P´eter csak feket´ere tesz. C´eljuk a kezd˝ ot˝ ok´ej¨ uk megk´etszerez´ese. Egyforma zsetonsz´ ammal kezdenek ´es els˝ ore mindketten 1-1 zsetont tesznek. Ha Anna vesz´ıt, akkor megk´etszerezi a t´etj´et, ha nyer, akkor u ´jra egyet tesz. P´eter veszt´es eset´en eggyel n¨ oveli a t´etj´et, m´ıg ha nyer, akkor ˝ o is 230

K¨ oz´ episkolai Matematikai ´ es Fizikai Lapok, 2022/4

visszat´er a kezdeti t´etre. A piros ´es a fekete val´ osz´ın˝ us´ege term´eszetesen azonos, de nem pontosan 50%, mivel a goly´ o a 0-ra ´erkezhet 1/37-ed es´ellyel, amely z¨old sz´ın˝ u. Ekkor mindketten vesztenek. A bank t˝ ok´eje korl´ atlan. T´ abl´ azatkezel˝ o program seg´ıts´eg´evel oldjuk meg a j´ at´ek szimul´aci´ oj´at. A t´ abl´ azat elrendez´ese tetsz˝ oleges lehet, de u unk az ´attekinthet˝ os´egre ´es a meg´ert´est ¨gyelj¨ feliratokkal seg´ıts¨ uk el˝ o. A t´ abl´ azat legyen felk´esz´ıtve arra, hogy a j´at´ek hossz´ u is lehet, de a m´eg fel nem haszn´ alt cell´ ak maradjanak u ot˝ oke 10 ´es 100 ¨resen. A kezd˝ zseton k¨ oz¨ ott v´ altozhat. Indul´ asnak adjuk meg, hogy mennyi Anna ´es P´eter kezd˝ ot˝ ok´eje. A j´ at´ek addig tartson, am´ıg az egyik˝oj¨ uk vagy el´eri a kezd˝ ot˝ oke k´etszeres´et, vagy elvesz´ıti az o¨sszes zsetonj´ at. ´ ekel´es: a feladat megold´ Ert´ asa eddig 7 pontot ´er. Tov´ abbi 3 pont kaphat´ o, ha egym´ as ut´ani 10 j´at´ek alapj´ an meghat´arozzuk, hogy h´ anyszor tesz t´etet Anna ´es P´eter a j´ at´ek sor´ an. Bek¨ uldend˝o egy i563.zip t¨ om¨ or´ıtett a´llom´anyban a t´abl´ azatkezel˝ o munkaf¨ uzet, illetve egy r¨ ovid dokument´ aci´ o, amelyben szerepel a megold´ askor alkalmazott t´ abl´ azatkezel˝ o neve, verzi´osz´ ama. I. 564. Az ingamozg´as sor´ an a test egy k¨ or´ıven, k´et sz´els˝ o helyzet k¨oz¨ott periodikus mozg´ ast v´egez. K´esz´ıts¨ unk anim´ aci´ ot egy SVG-t´ıpus´ u vektorgrafikus k´ep´ allom´anyba, amely egy inga mozg´as´at mutatja be. Az anim´ aci´ oban szerepl˝o alakzatok tetsz˝ oleges ot a k´ep´ allom´any sz¨oveg´enek m´eret˝ uek, sz´ın˝ uek ´es kit¨ olt´es˝ uek lehetnek. Az anim´aci´ szerkeszt´es´evel ´erdemes megoldani. Az anim´ aci´ oban az alakzatok olyan sebess´eggel mozogjanak, hogy a mozg´ as megfigyelhet˝o legyen. A lej´ atsz´ asi id˝ o, az ism´etl˝od´esek sz´ama, valamint m´ as param´eterek szabadon v´ alaszhat´ ok. SVG-´abra anim´ aci´ oj´anak k´esz´ıt´ese szerepelt az I. 357. feladatban, illetve az a´bra szerkezet´er˝ ol olvashatunk a http://svg.elte.hu/ c´ımen. ´ ekel´es: a matematikai inga mozg´as´ Ert´ anak bemutat´ as´aval 7 pont ´es az ingamozg´ as tov´ abbi jelens´eg´enek ´ abr´ azol´as´ aval m´eg 3 pont ´erhet˝ o el. Bek¨ uldend˝o t¨ om¨ or´ıtett i564.zip ´ allom´anyban az anim´aci´ o SVG-´allom´anya ´es egy r¨ ovid dokument´ aci´ o, amely tartalmazza a megold´ as v´ azlatos le´ır´as´at. alhat´ o, ´es k¨oz¨ott¨ uk M darab k´etir´ any´ uu ´t I/S. 62. Egy orsz´ agban N v´ aros tal´ vezet. Szeretn´enk ki´ep´ıteni egy h´al´ozatot u ´gy, hogy bizonyos v´ arosokba ad´ otornyokat telep´ıt¨ unk (mindegyikbe legfeljebb egyet). A v´ arosokat 1-t˝ol N -ig indexelj¨ uk. Egy u ´t t¨ ok´eletesen lefedett, ha pontosan az egyik v´egpont v´ aros´ aba telep´ıt¨ unk ad´ otornyot. Adjuk meg, hogy maximum h´any ad´ otornyot tudunk telep´ıteni u ´gy, hogy mind az M u ´t t¨ok´eletesen legyen lefedve.

Bemenet 5 3 1 2 / 3 2 / 4 5

Bek¨ uldend˝o egy is62.zip t¨ om¨ or´ıtett a´llom´anyban a megfelel˝oen dokument´ alt ´es kommentezett forr´ asprogram, amely tartalmazza a megold´ as l´ep´eseit, valamint megadja, hogy a program melyik fejleszt˝ oi k¨ ornyezetben futtathat´ o. S. 161. Egy ´ep¨ ulet k¨ ul¨ onb¨ oz˝ o pontjaira vizet kell vezetni. Az ´ep¨ ulet tervrajz´an N darab pont mutatja ezeket a helyeket. A tervrajzra gondolatban egy koordin´atarendszert illeszt¨ unk, amelyen az N pont mindegyike eg´esz koordin´ at´ akra esik. A vezet´ekeket u ´gy ´ep´ıtik, hogy a pontokat o sszek¨ o tik egyenes cs¨ o vekkel u ´gy, hogy b´ ar¨ melyikb˝ol b´armelyik m´asikba el lehessen jutni a cs¨ oveken haladva. A merev cs¨ oveket csak a tervrajzon jel¨ olt pontokban tudjuk el´agaztatni ´es nem is keresztezhetik ul ´es k´et megegym´ ast. Minden cs˝ o a koordin´ ata-rendszer egy-egy r´ acsvonal´ ara ker¨ adott pontot k¨ ot o ssze. Adjuk meg, hogy legal´ a bb milyen hossz´ u lesz a cs˝ ovezet´ekek ¨ hossza, ha k¨ozvetetten b´ armely k´et pontot ¨ osszek¨ otj¨ uk. Bemenet: az els˝ o sor tartalmazza a pontok N sz´ am´at. A k¨ ovetkez˝ o N sor mindegyike egy-egy pont x ´es y koordin´ at´ aj´at tartalmazza. Kimenet: a kimenet els˝ o ´es egyetlen sor´ aba a cs¨ ovek lehet˝o legkisebb ¨ osszhossz´ at kell ki´ırni. Ha nem lehet ˝oket mind ¨ osszek¨ otni, akkor -1-et kell ki´ırni. Minta: Bemenet (a / jel sort¨ or´est helyettes´ıt) 6 / 0 2 / 0 3 / 1 0 / 1 3 / 2 2 / 2 3

Kimenet 7

Korl´ atok: a koordin´ at´ ak abszol´ ut ´ert´eke legfeljebb 1000 ´es N  30. Id˝olimit: 0,5 mp. ´ ekel´es: a pontok 30%-a kaphat´ Ert´ o, ha az x koordin´ ata 0 vagy 1 ´ert´eke eset´en a program helyes kimenetet ad. Bek¨ uldend˝o egy s161.zip t¨ om¨ or´ıtett a´llom´anyban a megfelel˝oen dokument´ alt ´es kommentezett forr´ asprogram, amely tartalmazza a megold´ as l´ep´eseit, valamint megadja, hogy a program melyik fejleszt˝ oi k¨ ornyezetben futtathat´ o.

 A feladatok megold´asai regisztr´aci´o ut´an a k¨ovetkez˝o c´ımen t¨olthet˝ok fel: https://www.komal.hu/munkafuzet Beku ¨ld´esi hat´arid˝o: 2022. m´ajus 15.

A kimenet egyetlen sor´ aban egy sz´ am szerepeljen: maximum h´ any ad´ otornyot tudunk telep´ıteni a felt´eteleknek megfelel˝ oen. Ha ez nem lehets´eges, ´ırjunk ki -1-et. 231

3

Magyar´ azat: telep´ıts¨ unk ad´ otornyot az 1, 3, 5 index˝ u v´ arosokba. 5 Korl´ atok: 1  N, M  10 . Id˝ olimit: 0,4 mp. ´ Ert´ekel´es: a pontok 50%-a kaphat´ o, ha az 1  N, M  10 felt´etelek eset´en a program helyes kimenetet ad.

A bemenet els˝ o sor´ aban az N ´es M sz´ amok tal´alhat´ ok sz´ok¨ozzel elv´ alasztva. A k¨ ovetkez˝ o M sor mindegyike k´et sz´ amot tartalmaz: egy adott v´ arosp´ ar indexeit, amelyeket u ´t k¨ ot ¨ ossze.

K¨ oz´ episkolai Matematikai ´ es Fizikai Lapok, 2022/4

Kimenet

 232

K¨ oz´ episkolai Matematikai ´ es Fizikai Lapok, 2022/4

M´er´esi feladat megold´asa

M. 410. Ha egy kis m´eret˝ u, er˝ os m´ agnes ´es egy v´ızszintes helyzet˝ u gemkapocs k¨ oz´e egy k´ artyalapot helyez¨ unk, akkor a k´ arty´ an´ al fogva m´eg fel tudjuk emelni a gemkapcsot. M´erj¨ uk meg, h´ any darab egym´ asra rakott k´ artyalap kell ahhoz, hogy m´ ar ne tudjuk felemelni a gemkapcsot! Mekkora ezen egym´ asra helyezett lapok vastags´ aga? Csatlakoztassunk egym´ ashoz k´et ugyanolyan kis m´ agnest, ´es vizsg´ aljuk meg, h´ any k´ artyalap sz¨ uks´eges ahhoz, hogy a gemkapcsot m´ ar ne tudjuk felemelni! (6 pont)

K¨ozli: Sz´ asz Kriszti´ an, Budapest

Megold´as. 1. A m´er´es elve. A feladatunk az volt, hogy meghat´arozzuk, h´ any k´ artyalap eset´en nem tudja m´ar felemelni a m´agnes az ac´el gemkapcsot, amelyet egy´ebk´ent mag´ ahoz vonz. Tulajdonk´eppen azt a t´avols´ agot hat´ aroztuk meg, amelyn´el a m´ agneses vonz´ oer˝ ot ´eppen legy˝ ozi a gravit´ aci´ os er˝o. (Elvileg leveg˝oben is ez a t´avols´ ag ad´odna, mert a k´ arty´ ak nem befoly´asolj´ ak a m´agneses mez˝ ot.) aval v´egezt¨ uk a m´er´est, 2. A m´er´es kivitelez´ese. Francia k´artya k´etf´ele paklij´ egy nagyobb ´es egy kisebb m´eret˝ uvel. Gemkapocsb´ ol is k´et m´eret¨ unk volt. Mivel nagyon er˝os m´agnessel dolgoztunk, ez´ert nem egyes´evel adtuk hozz´ a a lapokat, hanem m´as m´odszert alkalmaztunk. Kerest¨ unk olyan a´llapotot, amin´el m´ ar nem tudja megemelni a gemkapcsot a m´ agnes, majd innen indulva cs¨okkentett¨ uk a lapok sz´ am´at addig, am´ıg felemelte, ´es v´eg¨ ul visszatett¨ unk m´eg egy lapot a pakliba. Ezut´ an k¨ ovetkezett a kez¨ unkben l´ev˝ o k´ artyapakli vastags´ ag´anak meghat´aroz´ asa. Ehhez egy sarokba szor´ıtottuk” a paklit, prec´ızen hozz´arendezve a falakhoz, ” majd egy der´eksz¨ og˝ u vonalz´ ot illesztett¨ unk mell´e, v´eg¨ ul lef´enyk´epezt¨ uk, ´es a kinagy´ıtott felv´etelr˝ ol olvastuk le n´eh´ any tizedmillim´eter pontoss´ aggal a pakli magass´ag´ at. 3. Tov´ abbi m´er´eseink: agnes, m´ asik paklival. Ism´et azt a m´odszert – Egy nagy gemkapocs, egy m´ alkalmaztuk, mint az els˝o m´er´esn´el. Megkerest¨ uk azt az a´llapotot, amin´el m´eg nem emeli fel a m´agnes a gemkapcsot, majd azt, ahol m´ ar felemeli. A m´asik pakli haszn´ alat´ an´ al t¨ obb k´ artyalapra volt sz¨ uks´eg¨ unk a megfelel˝ o ´allapot el´er´es´ehez, mint az els˝ o m´er´esn´el, de a magass´ aguk csak 0,4 millim´eterrel t´ert el egym´ast´ ol. Mivel k´et k¨ ul¨ onb¨ oz˝ o gy´ art´ ot´ ol sz´armaztak a k´ arty´ ak, ez okozhatta a lapok vastags´ag´ anak elt´er´es´et. – Egy nagy gemkapocs, k´et m´ agnes, k´et paklival. Enn´el a m´er´esn´el j´ oval t¨obb k´ artyalapra volt sz¨ uks´eg¨ unk, mint amikor csak egy m´agnest haszn´ altunk, ugyanis a k´et m´ agnes nagyobb t´ avols´ agb´ ol is k´epes volt felemelni a gemkapcsot, mert egy¨ utt uk a k´et k´ artyapaklit, mert ha er˝ osebb m´ agneses mez˝ ot hoztak l´etre. Az´ert vegy´ıtett¨ a k¨ or¨ ulbel¨ ul 100-100 db-os paklikb´ol csak az egyikkel pr´ob´alkoztunk, a m´ agnesek b˝ oven felemelt´ek a gemkapcsot 100 lapon kereszt¨ ul is. K¨ oz´ episkolai Matematikai ´ es Fizikai Lapok, 2022/4

233

– Egy kis gemkapocs, egy m´ agnes, k´et paklival. A gemkapocs m´erete (´es ´ıgy a t¨omege is) kisebb volt, teh´ at a m´ agnes vonz´ oerej´enek kisebb neh´ezs´egi er˝ ot kellett legy˝oznie. Emiatt vastagabb k´ artyar´etegen kereszt¨ ul is siker¨ ult felemeln¨ unk a gemkapcsot. Mivel egy pakliban nem volt elegend˝o sz´ am´ u k´ artyalap, ez´ert itt is k´et paklival kellett dolgoznunk. – Egy kis gemkapocs, k´et m´ agnes, k´et paklival. Mivel az m´ ar egyetlen m´agnesn´el is l´ atszott, hogy nem lesz el´eg egy pakli, nyilv´ anval´ o volt, hogy r¨ ogt¨ on k´et ´ paklival kezdj¨ uk az emel´esi k´ıs´erletet. Erthet˝ oen ez lett a legt¨ obb k´ artyalapot fel” em´eszt˝ o” m´er´es a maga 145 lapj´aval. 4. M´er´esi eredm´enyek: 1 db m´agnes

2 db m´agnes

gemkapcsok t¨ omege

kis gemkapocs 115 lap; 33,1 mm vegyes pakli 145 lap 42,0 mm vegyes pakli 0,35 g

nagy gemkapocs 99 lap 83 lap 27,5 mm 27,9 mm nagy m´eret˝ u pakli kis m´eret˝ u pakli 121 lap 36,3 mm vegyes pakli 0,90 g

5. Tapasztalataink: K´et m´ agnes eset´en a m´ agneses mez˝ o er˝ oss´ege (a m´ agnesekt˝ ol bizonyos t´ avols´ agban) nagyobb, mint egyetlen m´ agnes t´erer˝ oss´ege. Mindk´et m´eret˝ u (t¨omeg˝ u) gemkapocs eset´en azt tapasztaltuk, hogy nem pontosan k´etszer akkora vastags´ ag´ u k´ artyapaklit (k´etszer annyi k´artyalapot) kellett a gemkapocs ´es a k´et m´agnes k¨ oz´e helyezz¨ unk, hanem kevesebb is elegend˝ o volt, hogy a m´agnesek vonz´ oerej´et ugyanakkor´ ara cs¨ okkents¨ uk, mint egyetlen m´ agnesn´el volt. Pontosab42,0 ban: kis gemkapocs eset´eben a t´ avols´ agar´ any 33,1 = 1,27 ≈ 1,3, nagy gemkapocsn´al 36,3

pedig 27,7 = 1,31 ≈ 1,3 volt. Ebb˝ ol arra k¨ ovetkeztethet¨ unk, hogy a m´agneses vonz´oer˝ o nem ford´ıtottan ar´ anyos a t´ avols´ aggal, hanem valamilyen m´ as o ugg´es ¨sszef¨ (tal´an valamilyen hatv´ anyv´anyf¨ uggv´eny) szerint v´ altozik. (Ezt a t´ avols´ agf¨ ugg´est sokf´ele m´eret˝ u, vagyis sokf´ele t¨ omeg˝ u gemkapocs emel´es´evel lehetne tanulm´anyozni.) 6. M´er´esi hibalehet˝ os´egek: – A hat´ areset megkeres´es´en´el l´ephet fel ±1 lapnyi hiba.

– A k´ artyapakli vastags´ ag´ anak a vonalz´ or´ ol t¨ ort´en˝ o leolvas´asa eredm´enyezhet ±0,2 mm-es m´er´esi hib´ at.

– A m´ agnes m´erete l´enyegesen kisebb volt, mint a nagyobb m´eret˝ u k´ artya. Igyekezt¨ unk a legfels˝ o k´ artya k¨ oz´ep´en tartani a m´ agnest, ´es a gemkapcsot is a m´asik oldalon pr´ ob´ altuk u ´gy elhelyezni, hogy az ´eppen a m´agnes alatt legyen, de ez nem mindig siker¨ ult elegend˝ o pontoss´aggal. Kiss Benedek (Sopron, Berzsenyi D. Ev. (L´ıceum) Gimn. ´es Koll., 9. ´evf.) ´es ´ am(Sopron, Berzsenyi D. Ev. (L´ıceum) Gimn. ´es Koll., 9. ´evf.) S´ os Ad´ m´er˝ op´ ar m´er´esi jegyz˝ ok¨ onyve alapj´ an 234

K¨ oz´ episkolai Matematikai ´ es Fizikai Lapok, 2022/4

7 m´er´esi jegyz˝ ok¨ onyv ´erkezett. 6 pontot kapott Daniel Fodor ´es Sterling Kocher ´ am (m´er˝ (m´er˝ op´ ar), valamint Kiss Benedek ´es S´ os Ad´ op´ ar). Kicsit hi´ anyos (4–5 pont) 4, hi´ anyos (2 pont) 1 dolgozat.

G. 760. Alum´ıniumb´ ol k´esz¨ ult, 10 cm magas, k´ up alak´ u testet a cs´ ucs´ ahoz r¨ ogz´ıtett fon´ al seg´ıts´eg´evel lassan kiemel¨ unk egy t´eglatest alak´ u akv´ ariumb´ ol. Kezdetben a k´ up a 10 cm ´ atm´er˝ oj˝ u alapk¨ or´en ´ all az akv´ arium alj´ an, ´es a v´ız teljesen ellepi. Az akv´ arium t´erfogata sokkal nagyobb, mint az alum´ıniumk´ up´e. ´ Abr´ azoljuk a fonalat fesz´ıt˝ o er˝ ot a k´ up elmozdul´ as´ anak f¨ uggv´eny´eben! (4 pont)

Fizika gyakorlatok megold´asa

A K¨ oMaL Ny´ari T´ abor m´er´esi feladata nyom´an

Megold´as. Az alum´ıniumk´ up magass´ aga M = 10 cm, alapk¨ or´enek sugara R = d/2 = 5 cm. Az akv´ ariumban a v´ız valamekkora h magass´ agban ´all, ´es M < h, hiszen kezdetben a v´ız teljesen ellepi a f´emk´ upot.

G. 755. A 80 kg t¨ omeg˝ u akci´ oh˝ os olyan ejt˝ oerny˝ ot haszn´ al, amivel nyitott ´ allapotban 8 m/s sebess´eggel s¨ ullyed. Egy jelenetben a 60 kg t¨ omeg˝ u h˝ osn˝ ot a leveg˝ oben elkapja, majd ezut´ an nyitja az erny˝ ot. Mekkora sebess´eggel ´er f¨ oldet az ¨ osszekapaszkodott p´ ar? Milyen magass´ agb´ ol t¨ ort´en˝ o leugr´ as eset´en ´ern´enek szabadon esve ugyanekkora sebess´eggel a f¨ oldre?

olj¨ uk a k´ up alapk¨ or´enek az akv´ arium alj´at´ ol sz´am´ıtott elmozdul´as´ at s-sel. Jel¨ A k´ up t´erfogata

(4 pont)

az alum´ıniumk´ upra hat´ o neh´ezs´egi er˝ o

Megold´as. Az ejt˝oerny˝ os addig gyorsul, am´ıg a l´egellen´all´asb´ ol sz´armaz´ o er˝ o kisebb, mint az emberre hat´o neh´ezs´egi er˝ o, a´lland´ osult es´esi sebess´egn´el pedig a k´et er˝ o egyenl˝ o. Ugyanakkora ejt˝ oerny˝ o eset´eben a l´egellen´ all´as a sebess´eg n´egyzet´evel ar´ anyos. A k´et esetet ¨osszehasonl´ıtva: v2 m1 g = 12 , m2 g v2 ahol m1 = 80 kg, m2 = (80 + 60) kg, v1 = 8 m/s, v2 pedig az ¨osszekapaszkodott p´ ar keresett sebess´ege. A fenti egyenletb˝ol  140 m m ·8 = 10,58 . v2 = 80 s s

Vk´up =

G = mg = Al Vk´up g = 2700

kg N = 6,94 N. · (2,62 · 10−4 m3 ) · 9,81 m3 kg

Az alum´ıniumk´ up kiemel´es´en´el n´egy szakaszt k¨ ul¨ onb¨ oztethet¨ unk meg. (i) Am´ıg a k´ up az akv´ arium alj´ an nyugszik (s = 0), a neh´ezs´egi er˝ ovel a k´ upra hat´ o felhajt´oer˝ o (Ffelh. ) ´es az aljzat nyom´oereje tart egyens´ ulyt. A fonalat fesz´ıt˝ o er˝ o upra hat´ o felhajt´oer˝ o megegyezik a k´ up a´ltal kiszor´ıtott v´ız s´ uly´ aval: Ffon´al = 0. A k´ Ffelh. = Gv´ız = mv´ız g = v´ız Vk´up g = 1000 · (2,62 · 10−4 ) · 9,81 N = 2,57 N. (ii) Ha lassan, egyenletes sebess´eggel elkezdj¨ uk felfel´e h´ uzni a k´ uphoz r¨ ogz´ıtett fonalat, a k´ upra hat´ o er˝ ok tov´abbra is egyens´ ulyban maradnak:

Egy szabadon es˝ o test 10,58 m v2 s = 1,08 s t= = g 9,81 sm2 id˝ o alatt ´eri el a v2 nagys´ ag´ u sebess´eget, mik¨ ozben m 9,81 g s2 (1,08 s)2 ≈ 5,7 m h = t2 = 2 2 utat tesz meg. Az akci´ oh˝ os ´es a h˝ osn˝ o f¨ oldet ´er´esi sebess´ege kb. 6 m´eter magasr´ ol t¨ ort´en˝ o leugr´ as v´egsebess´eg´enek felelne meg, ha szabadon esn´enek.

1 2 R πM = 262 cm3 = 2,62 · 10−4 m3 , 3

G = Ffon´al + Ffelh. , ahonnan Ffon´al = G − Ffelh. = (6,94 − 2,57) N = 4,37 N. A fonalat fesz´ıt˝ o er˝ o addig ennyi, am´ıg a k´ up cs´ ucsa el nem ´eri a v´ız felsz´ın´et, teh´ at am´ıg s  h − M . (iv) Amikor s  h, akkor k´ up teljesen kiemelkedett a v´ızb˝ ol, a fonalat fesz´ıt˝ o er˝ o megegyezik a k´ upra hat´ o neh´ezs´egi er˝ ovel:

T¨ or¨ ok Hanga (Budapest, Fasori Evang´elikus Gimn., 10. ´evf.)

Ffon´al = G = 6,94 N.

Megjegyz´es. A megold´ as sor´ an felt´etelezt¨ uk, hogy a k¨ ozegellen´ all´ asi er˝ o teljes eg´esz´eben az ejt˝ oerny˝ ot˝ ol sz´ armazik, ami mellett az emberre (emberekre) hat´ o l´egellen´ all´ as elhanyagolhat´ oan kicsi.

(Mivel az akv´arium t´erfogata sokkal nagyobb, mint az alum´ıniumk´ up´e, a v´ızszint v´altoz´ as´at nem kell figyelembe venn¨ unk.)

39 dolgozat ´erkezett. Helyes 27 megold´ as. Hi´ anyos (1–2 pont) 7, hib´ as 3, nem versenyszer˝ u 2 dolgozat.

(iii) Ha h − M < s < h, az alum´ıniumk´ up egy r´esze m´ar kiemelkedett a v´ızb˝ ol, m´ıg egy m´ asik r´esze m´eg a v´ızszint alatt van. A k´ upra hat´ o felhajt´ oer˝ o a k´ up

K¨ oz´ episkolai Matematikai ´ es Fizikai Lapok, 2022/4

235

236

K¨ oz´ episkolai Matematikai ´ es Fizikai Lapok, 2022/4

v´ızbe mer¨ ul˝ o r´esze a´ltal kiszor´ıtott v´ız s´ uly´ aval egyenl˝o. A kiszor´ıtott v´ız t´erfogata megegyezik a v´ız alatt l´ev˝ o csonkak´ up t´erfogat´ aval, amelyet u ´gy kapunk meg, hogy a k´ up teljes t´erfogat´ ab´ ol kivonjuk a v´ız feletti, kisebb k´ up (cs´ ucsi r´esz) t´erfogat´ at. A cs´ ucsi r´esz alapk¨ or´enek sugara r, a magass´ aga x, ahol x = s + M − h. Hasonl´ o R r R up der´eksz¨ og˝ u h´ aromsz¨ ogekb˝ ol k¨ ovetkezik, hogy M = x , vagyis r = M x. A csonkak´ t´erfogata 2  R 1 x R2 π 3 x π = Vk´up − ·x . Vcsonkak´up = Vk´up − Vcs´ucs = Vk´up − r2 πx = Vk´up − 3 M 3 3M 2

Megold´as. Egy t´enylegesen elv´egzett k´ıs´erlettel pr´ ob´ altam kider´ıteni, hogy ´ a HATULJA sz´ ot hogyan kell fel´ırni ahhoz, hogy a lapot megford´ıtva a t¨ uk¨ orben balr´ol jobbra el tudjuk olvasni. Az els˝ o k´epen l´athat´ o annak a lapnak a h´ atulja, amelyet megford´ıtva a t¨ uk¨ or el´e tettem. A m´ asodik k´epen l´athat´ o, hogy a t¨ uk¨ orben az als´ o sort lehet balr´ol jobbra kiolvasni. Ebb˝ ol meg´allap´ıthatjuk, hogy a K¨ oMaL ´ felirat h´atulj´ ara a HATULJA sz´ ot t¨ uk¨ or´ır´ assal kell fel´ırni.

Az ismert adatok behelyettes´ıt´ese ut´ an ezt kapjuk: Vcsonkak´up = 2,62 · 10−4 m3 − 0,26 x3 . A fonalat fesz´ıt˝ o er˝ o



−4

Ffon´al = G − v´ız gVcsonkak´up = 6,94 N − 1000 · 9,81 · 2,62 · 10

x3 − 0,26 3 m



N. F¨ oldi Albert (Szolnok, Varga Katalin Gimn., 9. ´evf.)

Figyelembe v´eve, hogy

52 dolgozat ´erkezett. Helyes 42 megold´ as. Kicsit hi´ anyos (2 pont) 1, hib´ as 5, nem versenyszer˝ u 4 dolgozat.

x = s − h + M = s − h + 0,1 m,

G. 764. Egy nyugalmi ´ allapotb´ ol indul´ o, szabadon es˝ o test mozg´ as´ anak utols´ o m´ asodperc´eben ugyanakkora utat tett meg, mint az els˝ o h´ arom m´ asodperc alatt. Milyen magasr´ ol esett le a test? (Hanyagoljuk el a l´egellen´ all´ ast.)

a fon´ aler˝ o a k´ up elmozdul´ as´ anak f¨ uggv´eny´eben: Ffon´al =

(4 pont)

3 N . = 4,37 N + 2551(s − h + 0,1 m) m3

Megold´as. M´erj¨ uk az id˝ot m´asodperc, a megtett utat pedig m´eter egys´egekben, ´es sz´ amoljunk g kerek´ıtett, 10 sm2 ´ert´ek´evel.

Mik¨ ozben az s elmozdul´ as h − M -r˝ ol h-ra n˝ o, a fonalat fesz´ıt˝ o er˝ o egy harmadfok´ u f¨ uggv´eny szerint 4,37 N-r´ ol 6,94 N-ra n˝ o.

g

Egy szabadon es˝ o test (ha a l´egelen´all´ ast elhanyagoljuk) t id˝ o alatt d = 2 t2 utat tesz meg. Az els˝o h´ arom m´asodpercben ez az u ´t: d=

Hruby Laura (Budapest, Veres P´ aln´e Gimn., 10. ´evf.) 25 dolgozat ´erkezett. Helyes 5 megold´ as. Kicsit hi´ anyos (3 pont) 9, hi´ anyos (1–2 pont) 5, hib´ as 4, nem versenyszer˝ u 2 dolgozat.

10 · 32 = 45 m. 2

A t ideig tart´ o es´es utols´ o m´ asodperc´eben is 45 m´etert tesz meg a test, teh´ at 2

10 (t − 1) 10 t2 − = 45. 2 2

´ G. 761. Hogyan ´ırt´ ak a HATULJA sz´ ot a K¨ oMaL felirat h´ atulj´ ara: szok´ asos m´ odon vagy t¨ uk¨ or´ır´ assal?

Ennek az egyenletnek t = 5 a megold´ asa, vagyis az es´es magass´ aga: h=

g 2 10 2 t = 5 = 125 m. 2 2

Ha g-nek a pontosabb, 9,81 sm2 ´ert´ek´evel sz´ amolunk, akkor a h ≈ 123 m-es eredm´enyt kapjuk. ´ Nagy Csenge (Sz´ekelyudvarhely, Tam´ asi Aron Gimn., 9. ´evf.) 48 dolgozat ´erkezett. Helyes 30 megold´ as. Kicsit hi´ anyos (3 pont) 7, hi´ anyos (1–2 pont) 4, hib´ as 1, nem versenyszer˝ u 6 dolgozat.

(3 pont) K¨ oz´ episkolai Matematikai ´ es Fizikai Lapok, 2022/4

237

238

K¨ oz´ episkolai Matematikai ´ es Fizikai Lapok, 2022/4

a) A sebess´eget a gyorsul´ as–id˝ o grafikon alatti ter¨ uletk´ent kaphatjuk meg. opontban a sebess´eg: Az 1. a´br´ an s¨ot´eten jel¨ olt h´aromsz¨ ogr˝ ol leolvashat´o, hogy t1 id˝

Fizika feladatok megold´asa

v1 =

7,5 a 1 t1 = 2

m s2

· (3 s) m = 11,25 . 2 s

Hasonl´o m´ odon kapjuk, hogy tetsz˝oleges t id˝ opontban a sebess´eg: P. 5347. A kezdetben nyugv´ o, m = 2 kg t¨ omeg˝ u test s´ url´ od´ asmentesen mozoghat a v´ızszintes fel¨ uleten. Egy adott pillanatban a testre a fel¨ ulettel p´ arhuzamosan egy olyan a ´lland´ o ir´ any´ u F er˝ o kezd hatni, amelynek nagys´ aga egyenletesen v´ altozva 4 s alatt 0-r´ ol 20 N-ra n˝ o. ulva? a) Mekkora lesz a test sebess´ege t1 = 3 s m´ b) Mekkora utat tesz meg a test 3 s alatt, ha a t2 = 2 s alatt megtett u ´t 10 s2 = 3 m? (5 pont)

K¨ozli: Kotek L´ aszl´ o, P´ecs

I. megold´as. Ha a 2 kg t¨ omeg˝ u testre hat´ o er˝ o id˝oben egyenletesen n¨ovekszik, ol a0 = 10 sm2 -re akkor a test gyorsul´ asa is egyenletesen v´ altozik, ´es t0 = 4 s alatt 0-r´ n˝ o (1. ´ abra). Algebrai ¨ osszef¨ ugg´essel: a(t) = 2,5 ´es a k´erd´eses t1 =

m · t, s3

3 t0 = 3 s 4

id˝ opontban a gyorsul´as: a1 =

v(t) =

m a(t) · t = 1,25 3 t2 . 2 s

b) A sebess´eg–id˝ o grafikon a 2. ´ abr´ an l´ athat´ o. A t id˝ o alatt megtett s(t) u ´t a parabola 0 ´es t id˝ opontok k¨ oz¨ otti ´ıve alatti ter¨ ulettel egyezik meg. Feladatunk a s¨ot´etebben jel¨ olt ter¨ ulet nagys´ ag´ anak meghat´aroz´ asa. Tudjuk, hogy a megtett utat egyenletes mozg´as eset´en a v0 t, egyenletesen gyoran sz´amolhatjuk. Sejthet˝ o, hogy egyenletesen sul´o mozg´asn´ al az a20 t2 k´eplet alapj´ v´altoz´ o gyorsul´ as eset´en a megtett u ´t (ha a kezd˝ osebess´eg nulla) az eltelt id˝ o k¨ ob´e3 or nagyobb, mint a 2 s alatt vel ar´ anyos, ´es emiatt a 3 s alatt megtett u ´t (3/2) -sz¨ megtett 10 m´eteres u ´t. 3 Hogyan l´ athat´ o be ezen sejt´es helyess´ege? Ha a 2. ´ abr´ an l´athat´ o parabo2 anyl´at a t tengely ment´en 2/3 ar´ anyban, a v tengely ment´en pedig (2/3) ar´ ban ¨osszezsugor´ıtjuk, akkor a g¨ orbe tov´ abbra is parabola marad, de az A pont a B pontba ker¨ ul. Mivel a g¨ orbe alatti ter¨ ulet a k´et transzform´ aci´ o egy¨ uttes hat´ a3 anyban cs¨ okken, fel´ırhatjuk, hogy s´ara (2/3) ar´  3 3 27 10 · m = 11,25 m. s(t = 3 s) = · s(t = 2 s) = 2 8 3 II. megold´as. A feladatot a differenci´ al- ´es integr´ alsz´ am´ıt´ as ¨ osszef¨ ugg´eseinek felhaszn´ al´as´aval is megoldhatjuk. Eset¨ unkben v´ altoz´ o gyorsul´ as´ u mozg´assal van dolgunk. A gyorsul´as egys´egnyi id˝ o alatt v´egbemen˝ o v´ altoz´ as´ at r´ andul´asnak” ne” vezz¨ uk ´es j-vel jel¨ olj¨ uk. A feladatban szerepl˝ o mozg´as r´ andul´ asa id˝ oben a´lland´ o, nagys´ aga 10 m/s2 m Δa = = 2,5 3 . j= Δt 4s s

3 m a0 = 7,5 2 . 4 s

A test r´ andul´asa a gyorsul´as–id˝ o f¨ uggv´eny deriv´ altja: j0 =

da(t) , dt

teh´ at

a(t) = j0 t;

a test gyorsul´ asa a sebess´eg–id˝ o f¨ uggv´eny deriv´ altja: j0 t =

dv(t) , dt

teh´ at

v(t) = j0

t2 ; 2

v´eg¨ ul a test sebess´ege az u ´t–id˝ o f¨ uggv´eny deriv´ altja: 1. a ´bra K¨ oz´ episkolai Matematikai ´ es Fizikai Lapok, 2022/4

j0

2. a ´bra

239

240

t2 ds(t) = , 2 dt

teh´ at

s(t) = j0

t3 . 6

K¨ oz´ episkolai Matematikai ´ es Fizikai Lapok, 2022/4

(Kihaszn´ altuk, hogy kezdetben, t = 0 pillanatban a test gyorsul´asa, a sebess´ege ´es az elmozdul´asa is nulla.) Ezek szerint a test sebess´ege t = 3 s m´ ulva 2

v(3 s) = 2,5

vagyis (1)

F =

A r´ udra hat´ o er˝ ok v´ızszintes ir´any´ u komponenseinek egyens´ ulya:

m (3 s) m = 11,25 , s3 2 s

(2)

a megtett u ´tja pedig

T (α) = F sin α =

m (3 s) = 11,25 m. s3 6

K + F cos α = mg,

Nem haszn´ altuk fel a 2 m´ asodperc alatt megtett u ´t megadott ´ert´ek´et, de ellen˝ orizhetj¨ uk, hogy az helyes-e:

ahonnan (1) felhaszn´al´as´ aval K(α) = mg −

3

m (2 s) 10 s(2 s) = 2,5 3 = m. s 6 3

T (α)  μK(α),

84 dolgozat ´erkezett. Helyes 51 megold´ as. Kicsit hi´ anyos (4 pont) 13, hi´ anyos (1–3 pont) 14, hib´ as 1, nem versenyszer˝ u 5 dolgozat.

vagyis

P. 5357. V´ızszintes asztallapon fekszik egy homog´en t¨ omegeloszl´ as´ u r´ ud. Ezt a rudat lassan f¨ ugg˝ oleges helyzetbe hozzuk az egyik v´eg´ere hat´ o, a r´ udra mindenkor mer˝ oleges er˝ ovel. Legal´ abb mekkora a r´ ud ´es az asztallap k¨ oz¨ otti tapad´ asi s´ url´ od´ asi egy¨ utthat´ o, ha a r´ ud nem cs´ uszik meg fel´ all´ıt´ as k¨ ozben?

amit ´ıgy is fel´ırhatunk: (3)

Amerikai feladat nyom´an

I. megold´as. Mivel a rudat lassan emelj¨ uk, ez´ert a r´ ud minden pillanatban nyugalmi helyzet˝ unek tekinthet˝ o. Emiatt a r´ ud minden helyzet´eben a r´ a hat´ o er˝ok osszege ´es a forgat´ onyomat´ekok ¨ osszege is nulla. ¨ Legyen a r´ udra mindenkor mer˝ oleges er˝ o nagys´ aga F , az asztal a´ltal kifejtett f¨ ugg˝oleges ir´ any´ u k´enyszerer˝ o legyen K, a v´ızszintes ir´any´ u tapad´ asi s´ url´od´ asi er˝ o olj¨ uk -lel, t¨ opedig T . A r´ ud hossz´at jel¨ meg´et m-mel, a v´ızszintessel bez´art sz¨ ool 90◦ -ig) lege (ami lassan v´altozik 0◦ -t´ gyen α (l´ asd az 1. ´ abr´ at). A forgat´ onyomat´ekok egyens´ uly´ at (statikus esetben) a r´ ud b´armely pontj´ ara fel´ırhatjuk. Legyen ez a pont a r´ udnak az asztallal ´erintkez˝o v´egpontja. Ekkor

μ

T sin α cos α = , K 2 − cos2 α

μ

sin α cos α . 2 sin2 α + cos2 α

B˝ov´ıts¨ uk (3) jobb oldal´ an ´ all´ o t¨ ortet μ

(4)

1 cos2 α -val:

1 tg α = . 2 tg2 α + 1 2 tg α + tg1α

Ennek az o¨sszef¨ ugg´esnek tetsz˝ oleges tg α-ra teljes¨ ulnie kell, hiszen a r´ ud semelyik helyzet´eben nem cs´ uszik meg. Alkalmazzuk (4) jobb oldal´ anak nevez˝oj´ere a sz´amtani ´es a m´ertani k¨ ozepekre vonatkoz´ o egyenl˝ otlens´eget:  √ 1 1 2 tg α +  2 2 tg α · = 8, tg α tg α ezzel a (4) felt´etel: 1 μ  √ ≈ 0,35. 8

1. a ´bra

Ha ez teljes¨ ul, akkor a r´ ud a fel´ all´ıt´ asa sor´ an nem cs´ uszik meg.

F = mg(cos α) , 2 K¨ oz´ episkolai Matematikai ´ es Fizikai Lapok, 2022/4

mg cos2 α . 2

Az α sz¨ogben megemelt r´ ud akkor nem cs´ uszik meg az asztalon, ha

Waldhauser Mikl´ os (Szegedi Radn´oti M. K´ıs´erleti Gimn., 11. ´evf.) dolgozata alapj´an

(5 pont)

mg sin α cos α, 2

a f¨ ugg˝oleges ir´ any´ u komponensek´e pedig

3

s(3 s) = 2,5

mg cos α. 2

Beke B´ alint (Budapest, ELTE Ap´ aczai Csere J, Gyak. Gimn., 11. ´evf.) 241

242

K¨ oz´ episkolai Matematikai ´ es Fizikai Lapok, 2022/4

ahonnan

II. megold´as. Az I. megold´ as gondolatmenet´et k¨ ovetve a meg nem cs´ usz´ as felt´etel´ere ezt kapjuk: μ

(1)

sin α cos α ≡ f (α). 2 − cos2 α

Ha ´abr´ azoljuk az f (α) f¨ uggv´eny grafikonj´ at (pl. a https:// 2. a ´bra www.geogebra.org/classic?lang=hu vagy a https://www.wolframalpha.com/ alkalmaz´as seg´ıts´eg´evel), arr´ ol (l´asd a 2. ´ abr´ at) leolvashatjuk, hogy f (α) legnagyobb ´ert´eke kb. 0,35. Ha a tapad´ o s´ url´od´ asi egy¨ utthat´ o enn´el a sz´ amn´ al nagyobb, akkor a r´ ud f¨ ugg˝oleges helyzetbe hozhat´ o an´elk¨ ul, hogy megcs´ uszna az asztalon. T¨ obb dolgozat alapj´ an 44 dolgozat ´erkezett. Helyes 23 megold´ as. Kicsit hi´ anyos (4 pont) 3, hi´ anyos (1–3 pont) 10, hib´ as 6, nem versenyszer˝ u 2 dolgozat.

P. 5364. Sima, v´ızszintes, s´ url´ od´ asmentes s´ıkon nyugszik egy R sugar´ u, m t¨ omeg˝ u f´elhenger, dombor´ u fel´evel felfel´e. A f´elhenger tetej´er˝ ol nyugalmi helyzetb˝ ol indul el s´ url´ od´ as n´elk¨ ul egy kis m´eret˝ u, de ugyancsak m t¨ omeg˝ u test. Milyen hossz´ u utat tesz meg ez a test a f´elhengeren, miel˝ ott elv´ alik t˝ ole? (5 pont)

K¨ozli: Sz´ asz Kriszti´ an, Budapest

Megold´as. A kis testre a neh´ezs´egi er˝ o hat lefel´e, valamint a f´elhenger a´ltal kifejtett nyom´oer˝ o a f´elhenger felsz´ın´ere mer˝ olegesen. A f´elhengerre a gravit´ aci´ os er˝ o hat lefel´e, az al´ at´ amaszt´ as ´ altal kifejtett nyom´oer˝ o f¨olfel´e, ´es a kis testre kifejtett nyom´ oer˝ o ellenereje. V´ızszintes ir´anyban k¨ uls˝ o er˝ o nem hat, a rendszer v´ızszintes ´gy lehets´eges, ir´ any´ u lend¨ ulete a´lland´o, v´egig nulla, mint a kezdeti pillanatban. Ez u ha a henger sebess´ege ´es a kis test sebess´eg´enek v´ızszintes komponense azonos nagys´ ag´ u ´es ellenkez˝ o ir´ any´ u (1. a ´bra). Az energiamegmarad´ as t¨ orv´enye szerint  1  1 mgR = mgR cos ϕ + mvx2 + m vx2 + vy2 , 2 2

A f´elhengerrel egy¨ utt mozg´o vonatkoztat´ asi rendszerben a f´elhenger a´ll, a kis test pedig egy k¨ or ment´en, a k¨ or ´erint˝oj´enek ir´ any´ aban mozog. A kis test sebess´eg´enek v´ızszintes komponense 2vx . K´enyszerfelt´etel, hogy a k¨ orp´ alya ´erint˝oj´ere mer˝ oleges sebess´egkomponens nulla: vy cos ϕ = 2vx sin ϕ. Ezt (1)-be helyettes´ıtve kapjuk, hogy 1 − cos ϕ vx2 = gR . 1 + 2 tg2 ϕ okkenhet, hiszen Am´ıg a kis test a f´elhenger felsz´ın´en mozog, addig vx nem cs¨ a k¨or felsz´ın´ere mer˝ oleges k´enyszerer˝ o v´ızszintes komponense a f´elhenger sebess´eg´evel azonos ir´ anyba mutat, a neh´ezs´egi er˝ onek ´es a talaj nyom´ oerej´enek pedig nincs v´ızszintes komponense. A kis test akkor v´alik el a k¨ or felsz´ın´er˝ ol, amikor a f´elhenger a´ltal kifejtett k´enyszerer˝ o null´ av´ a v´ alik, vagyis amikor vx ´ert´eke a legnagyobb. Ez ott k¨ovetkezik be, ahol az f (ϕ) =

K¨ oz´ episkolai Matematikai ´ es Fizikai Lapok, 2022/4

1 − cos ϕ 1 + 2 tg2 ϕ

f¨ uggv´enynek maximuma van a [0; π/2] intervallumon. Sz´els˝ o´ert´eksz´ am´ıt´ assal, vagy f (ϕ) grafikus a´br´ azol´ as´ aval bel´ athat´ o, hogy ϕ = ϕmax ≈ 0,75(rad) ≈ 43◦ sz¨ ogn´el van a maximum, ´ıgy a kis testnek a f´elhenger felsz´ın´en megtett u ´tja: s = Rϕmax ≈ 0,75 R. D´ ora M´ arton (Budapest, ELTE Ap´ aczai Csere J. Gyak. Gimn., 12. ´evf.) 50 dolgozat ´erkezett. Helyes 16 megold´ as. Kicsit hi´ anyos (4 pont) 1, hi´ anyos (1-3 pont) 21, hib´ as 7, nem versenyszer˝ u 5 dolgozat.

P. 5366. Ide´ alis g´ az ´ alland´ o nyom´ ashoz, illetve ´ alland´ o t´erfogathoz tartoz´ o fajh˝ oinek h´ anyadosa κ. a) A g´ az adiabatikusan t´ agul. Mekkora a g´ az munk´ aj´ anak ´es a bels˝ o energia megv´ altoz´ as´ anak ar´ anya? b) A g´ az izotermikusan ¨ osszenyom´ odik. Mekkora a g´ az munk´ aj´ anak ´es a felvett h˝ onek az ar´ anya? c) A g´ azt izob´ ar folyamatban meleg´ıtj¨ uk. Mekkora a g´ az munk´ aj´ anak ´es a felvett h˝ onek az ar´ anya? (4 pont)

1. a ´bra

2gR(1 − cos ϕ) = 2vx2 + vy2 .

K¨ ozli: Zsigri Ferenc, Budapest

Megold´as. a) Ha a g´ az adiabatikusan t´agul, akkor a g´ azon v´egzett munka aznak a´tadott h˝ o (W ) negat´ıv (azaz a g´ az v´egez munk´ at: W = −Wg´az ), ´es a g´ Q = 0. Az I. f˝ ot´etel szerint a bels˝ o energia v´ altoz´ asa ΔU = Q + W . Mivel Q = 0, anya −1. ΔU = W = −Wg´az , ´ıgy Wg´az ´es ΔU ar´

2. a ´bra

243

244

K¨ oz´ episkolai Matematikai ´ es Fizikai Lapok, 2022/4

b) Ha a g´ az izotermikusan ¨ osszenyom´ odik, akkor a h˝ om´ers´eklet-v´ altoz´ as ΔT = 0. Ide´ alis g´azra f ΔU = nRΔT, 2 ahol f a szabads´ agi fokok sz´ ama, n a g´ az mennyis´ege molban ´es R a g´ az´ allan´ d´ o. Alland´ o h˝ om´ers´ekleten a bels˝ o energia a´lland´ o, teh´at ΔU = 0. Mivel ΔU = = Q + W = 0, ´ıgy Q = −W = Wg´az , teh´at a g´ az munk´aj´anak ´es a felvett h˝ onek az ar´anya +1. c) Ha a g´ azt izob´ ar folyamatban meleg´ıtj¨ uk, akkor a nyom´ asa ´alland´ o, azaz Δp = 0. A bels˝ o energia v´ altoz´ asa: ΔU =

f pΔV. 2

P. 5368. Egy R = 30 cm sugar´ u, f´emhuzalb´ ol k´eolt´est adunk, majd a k¨ osz¨ ult karik´ anak Q = 6 · 10−6 C t¨ z´eppontj´ an ´ atmen˝ o, a s´ıkj´ ara mer˝ oleges tengely k¨ or¨ ul ω = 520 1/s sz¨ ogsebess´eggel megforgatjuk v´ akuumban. Egy adott pillanatban egy elektron ´eppen a karika k¨ oz´eppontj´ an rep¨ ul ´ at v = 120 m/s nagys´ ag´ u, a karika s´ıkj´ aba es˝ o sebess´eggel. Mekkora az elektron p´ aly´ aj´ anak g¨ orb¨ uleti sugara a karika k¨ oz´eppontj´ aban, ha ott a F¨ old m´ agneses tere ´eppen az elektron sebess´eg´enek ir´ any´ aba mutat? (5 pont)

K¨ ozli: Holics L´ aszl´ o, Budapest

Megold´as. A karik´ aval egy¨ utt forg´o t¨ olt´esek

Kihaszn´ altuk az egyetemes g´azt¨ orv´enyt, ami szerint eset¨ unkben

I=

n R ΔT = Δ(pV ) = pΔV.

Q Q ΔQ Qω = = 2π = Δt T 2π ω

M´ asr´eszt tudjuk, hogy

er˝ oss´eg˝ u ´aramot hoznak l´etre. A karik´ at teh´at tekinthetj¨ uk egy R sugar´ u k¨ orvezet˝onek, amelyben I er˝ oss´eg˝ u´ aram folyik.

f Q + W = ΔU = pΔV, 2

Az elektron pillanatnyi hely´en, vagyis a k¨ orvezet˝ o k¨ oz´eppontj´aban a m´agneses indukci´ ovektor nagys´ aga I Qω = μ0 , |B| = μ0 2R 4πR ir´ anya pedig mer˝ oleges a karika s´ıkj´ara. A (−e) t¨olt´es˝ u, B-re mer˝ oleges sebess´eg˝ u elektronra hat´o Lorentz-er˝ o nagys´ aga

tov´ abb´ a hogy a g´ az ´ altal v´egzett munka: Wg´az = pΔV. Ezek szerint Q + W = Q − Wg´az = azaz Q=



f Wg´az , 2

  μ0 evQω F = (−e)(v × B) = evB = . 4π R

 f + 1 Wg´az . 2

Az ide´alis g´az ´ alland´ o nyom´ ashoz, illetve a´lland´ o t´erfogathoz tartoz´ o fajh˝oinek h´ anyadosa a szabads´ agi fokokkal kifejezve: κ=

f +2 , f

ahonnan

f=

2 κ−1

(Mivel a f¨oldi m´ agneses t´er ir´anya ´eppen p´ arhuzamos az elektron sebess´eg´evel, ´ıgy a Lorentz-er˝ o sz´ am´ıt´ as´ an´ al figyelmen k´ıv¨ ul hagyhat´ o.) Az m t¨omeg˝ u elektronra hat´o m´agneses er˝ o a r´eszecske p´ aly´ aj´at elg¨ orb´ıti. Newton mozg´asegyenlete szerint F =m

k¨ ovetkezik. ´Igy izob´ ar v´ altoz´ asn´ al a g´ az munk´aj´anak ´es a felvett h˝ onek az ar´anya: Wg´az κ−1 1 = =1− . Q κ κ

ahol r a p´alya g¨ orb¨ uleti sugara a karika k¨ oz´eppontj´aban. Innen kapjuk, hogy a keresett g¨orb¨ uleti sug´ar: r=

Csillingek csapat: ´ Isk. ´es Gimn., 9. ´evf.) ´es Csilling D´ aniel (Budapesti Fazekas M. Gyak. Alt. Csilling Katalin (Budapest, Szil´ agyi E. Gimn., 12. ´evf.) 61 dolgozat ´erkezett. Helyes 15 megold´ as. Kicsit hi´ anyos (3 pont) 11, hi´ anyos (1–2 pont) 21, hib´ as 9, nem versenyszer˝ u 5 dolgozat.

K¨ oz´ episkolai Matematikai ´ es Fizikai Lapok, 2022/4

245

v2 , r

mv 2 4π mvR (9,11 · 10−31 ) · 120 · 0,3 = = 107 m ≈ 66 cm. F μ0 eQω (1,6 · 10−19 ) · (6 · 10−6 ) · 520 Bencz Benedek (Budapest, Ba´ar-Madas Ref. Gimn., 9. ´evf.)

25 dolgozat ´erkezett. Helyes 11 megold´ as. Kicsit hi´ anyos (4 pont) 7, hi´ anyos (1–3 pont) 4, hib´ as 1, nem versenyszer˝ u 2 dolgozat.

246

K¨ oz´ episkolai Matematikai ´ es Fizikai Lapok, 2022/4

P. 5370. Egy r¨ ovidl´ at´ o ember szem´enek k¨ ozelpontja 8 cm-re van a szem´et˝ ol szem¨ uveg n´elk¨ ul. Mekkora lesz a k¨ ozelpontj´ anak a t´ avols´ aga, ha felveszi −5 dioptri´ as szem¨ uveg´et? (4 pont)

K¨ ozli: Honyek Gyula, Veresegyh´az

Megold´as. Az egyszer˝ us´eg kedv´e´ert tekints¨ uk a szemet optikai szempontb´ ol egyetlen v´ekony lencs´enek, j´ ollehet a t´enyleges helyzet enn´el bonyolultabb. Jel¨ ol´esek: okuszt´ avols´ aga, – f1 a szem f´ – D1 = f1 a szem dioptri´aja, 1 uveg f´ okuszt´ avols´ aga, – f2 a szem¨ uveg dioptri´ aja, – D2 = f1 a szem¨ 2 ozelpont ´es a szemlencse t´ avols´ aga szem¨ uveg n´elk¨ ul, – t1 a k¨ ozelpont ´es a szemlencse t´ avols´ aga szem¨ uveggel, – t2 a k¨ – k a szemlencse ´es a retina t´ avols´ aga (a szem m´erete). Megjegyz´es. A szem f1 f´ okuszt´ avols´ ag´ at bizonyos hat´ arok k¨ oz¨ ott k´epesek vagyunk v´ altoztatni, legkisebb ´ert´eke a k¨ ozelpontban l´ev˝ o t´ argyt´ avols´ agnak felel meg. Ez az adat is f¨ uggetlen a szem¨ uveg visel´es´et˝ ol.

A m´eterben m´ert f´ okuszt´ avols´ ag reciproka (a dioptria) megegyezik a k¨ozelpontt´ avols´ ag reciprok´anak ´es a k´ept´ avols´ ag reciprok´anak ¨osszeg´evel: D1 =

1 1 + . t1 k

P. 5371. A tau-r´eszecske (τ ) elektromos t¨ olt´ese ugyanakkora, mint az elektron´e. T¨ omege 3470-szer akkora, mint az elektron´e ´es 1,89-szer akkora, mint a proton´e. ofordulhat, hogy a protonnal Nagyon r¨ ovid az ´elettartama (3 · 10−13 s), m´egis el˝ k¨ ot¨ ott rendszert alkot. Ebben az esetben a k´et r´eszecske a k¨ oz¨ os t¨ omegk¨ oz´eppont k¨ or¨ ul k¨ orp´ aly´ an kering, ´es a rendszer teljes perd¨ ulete n (n = 1, 2, . . .). a) Adjuk meg a τ -proton atom ´es a H-atom sz´ınk´epeiben a megfelel˝ o hull´ amhosszak ar´ any´ at! b) Mekkora a τ -proton atom k¨ ot´esi energi´ aja? (5 pont)

K¨ ozli: Simon P´eter, P´ecs

Megold´as. A τ – p atom (tau-hidrog´en) energiaszintjeit az e – p rendszer (k¨ oz¨ons´eges H-atom) energiaszintjeihez hasonl´ o m´ odon kaphatjuk meg, alkalmazva a Bohr-f´ele atommodell feltev´eseit. A k´et eset k¨ oz¨ otti k¨ ul¨ onbs´eg az, hogy a τ r´eszecske t¨omege ¨ osszem´erhet˝ o a proton t¨ omeg´evel, emiatt c´elszer˝ u merev testk´ent kezelni ezt a rendszert. Legyen a k´et r´eszecske t´ avols´ aga r, a t¨ omegk¨ oz´eppontjukt´ ol m´ert t´ avols´ agok pedig mτ mp r, illetve rτ = r. rp = mτ + m p mτ + m p A τ -proton merev test” tehetetlens´egi nyomat´eka a r´eszecsk´ek mozg´ asi s´ıkj´ara ” mer˝ oleges, a t¨ omegk¨ oz´epponton ´atmen˝ o tengelyre: Θ = mp rp2 + mτ rτ2 =

(1)

mp m τ 2 r ≡ m∗ r 2 . mτ + m p

A szem¨ uveg ´es a szemlencse nagyon k¨ ozel van egym´ ashoz, ´ıgy a dioptri´ajuk ¨osszead´ odik, teh´ at a szem¨ uveges ember szeme D1 + D2 dioptri´asnak tekinthet˝ o. A k´ep szem¨ uveggel ´es an´elk¨ ul is a szem h´ atulj´ an”, teh´at ugyanott keletkezik, ” ´ıgy fenn´ all, hogy 1 1 D1 + D2 = + . t2 k

(Az m∗ mennyis´eget a k´et r´eszecske reduk´ alt t¨ omeg´enek nevezik.) Ha a k´et r´eszecske (mint merev test) ω sz¨ ogsebess´eggel forog a k¨ oz¨ os t¨ omegk¨ oz´eppont k¨or¨ ul, akkor a perd¨ ulete (impulzusnyomat´eka) a Bohr-f´ele kvantumfelt´etel szerint

A fenti k´et egyenlet k¨ ul¨ onbs´eg´eb˝ ol D2 = t1 − t1 ad´odik, vagyis (m´eter egys´egekkel 2 1 1 . Innen sz´ amolva): −5 = t1 − 0,08

ahol  = h/(2π) ´es n pozit´ıv eg´esz sz´ am. Mindk´et r´eszecske k¨ ormozg´ as´ at a Coulomb-er˝ o biztos´ıtja:

2

1 = −5 + 12,5, t2

azaz

(2)

mp rp ω 2 = mτ rτ ω 2 = k

t2 = 0,133 m.

Teh´ at a r¨ ovidl´ at´ o ember k¨ ozelpontj´ anak t´ avols´ aga kb. 13 cm-re lesz a szem´et˝ ol, ha felveszi a szem¨ uveg´et. Marozsi Lenke S´ ara (Kecskem´eti Katona J. Gimn., 11. ´evf.) ´es Vig Zs´ ofia (Szeged, SZTE Gyak. Gimn., 11. ´evf.) dolgozata alapj´an

247

e2 , r2

ahonnan m∗ rω 2 = k

(3)

e2 . r2

Az (1), (2) ´es (3) ¨ osszef¨ ugg´esb˝ ol kisz´am´ıthatjuk, hogy

67 dolgozat ´erkezett. Helyes 39 megold´ as. Kicsit hi´ anyos (3 pont) 7, hi´ anyos (1–2 pont) 8, hib´ as 2, nem versenyszer˝ u 11 dolgozat.

K¨ oz´ episkolai Matematikai ´ es Fizikai Lapok, 2022/4

Θω = n,

ω=

248

m∗ (ke2 ) (n)

3

2

2

,

illetve

r=

(n) . m∗ (ke2 )

K¨ oz´ episkolai Matematikai ´ es Fizikai Lapok, 2022/4

A rendszer teljes energi´ aja a kinetikus ´es a potenci´ alis energia ¨osszege:

Megold´as. A kapcsol´o z´ art a´ll´ as´ aban az ´aramk¨ or ered˝o ellen´ all´ asa

E = Ekin. + Epot. , ahol Ekin. =

1 m∗ (ke2 ) Θω 2 = 2 2n2 2

Re = 2

I= m∗ (ke2 ) ke2 =− r n2 2

12 V U = 0,6 A = Re 20 Ω

er˝ oss´eg˝ u ´aram folyik.

ami a reduk´ alt t¨ omeggel ar´ anyos. Hasonl´ıtsuk ¨ ossze a τ -proton ´es az elektron-proton rendszer k¨ot´esi energi´ aj´at, vagyis sz´am´ıtsuk ki a reduk´alt t¨ omegek ar´ any´ at. A hidrog´enatomn´ al a reduk´alt t¨ omeg gyakorlatilag az elektron t¨ omege (me ), hiszen me  mp . A megadott t¨omegar´anyok szerint a reduk´alt t¨ omegek ar´ anya:

K¨ozvetlen¨ ul a kapcsol´o kinyit´ asa ut´ an a tekercsen ´atfoly´ o´ aram er˝oss´ege m´eg mindig 0,6 A lesz. (Ha az ´aramer˝ oss´eg nagyon r¨ ovid id˝ o alatt v´eges ´ert´ekkel megv´ altozna, akkor az ´ aramer˝ oss´eggel ar´ anyos m´ agneses fluxus v´ altoz´ asi sebess´ege nagyon nagy lenne, ami nagyon nagy fesz¨ ults´eget induk´ alna a tekercsben.) Az I er˝ oss´eg˝ u ´ aram most csak a 30 Ω-os ellen´all´ ason folyik kereszt¨ ul, azon teh´at 0,6 A · 30 Ω = 18 V lesz a fesz¨ ults´eg. Ez nagyobb, mint az a´ramforr´ as kapocsfesz¨ ults´ege, a hi´ anyz´o” 6 V fesz¨ ults´eg teh´at a tekercsben fog induk´al´odni. ” Josepovits G´ abor (Budapest, Szerb Antal Gimn., 11. ´evf.) 22 dolgozat ´erkezett. Helyes 15 megold´ as. Hi´ anyos (1–2 pont) 6, hib´ as 1 dolgozat.

m∗τ −p m p mτ mp mτ mτ 3470 1,89 = · ≈ 1200. = · · = m∗e−p me (mτ + mp ) mτ me mτ + m p 1,89 2,89

Ny´ari fizikat´abor

a) A τ -proton rendszer energiaszintjeinek abszol´ ut ´ert´eke 1200-szor nagyobb, mint a hidrog´enatom megfelel˝o energiaszintjeinek abszol´ ut ´ert´eke. A Bohr-modell szerint a kisug´arzott fotonok energi´aja az energiaszintek k¨ ul¨onbs´eg´evel egyezik meg. Az elektrom´agneses hull´ amok frekvenci´aja (a hf = ΔE ¨osszef¨ ugg´es szerint) ugyancsak egyenesen ar´anyos a reduk´ alt t¨ omeggel, a megfelel˝ o hull´amhosszak teh´at a τ -proton sz´ınk´ep´eben kb. 1200-szor r¨ ovidebbek, mint a H-atom´eban. b) A k¨ ot´esi energia (az n = 1-es a´llapot ioniz´al´as´ahoz sz¨ uks´eges energia) a τ protonn´ al 1200-szor nagyobb, mint a H-atom 13,6 eV k¨ot´esi energi´ aja, teh´at mintegy 16,3 keV, azaz kb. 2,6 · 10−15 J.

T´egl´ as Panna (R´evkom´arom, Selye J´ anos Gimn., 12. ´evf.)

15 dolgozat ´erkezett. Helyes 7 megold´ as. Kicsit hi´ anyos (4 pont) 1, hi´ anyos (1–3 pont) 5, hib´ as 2 dolgozat.

P. 5378. Az ´ abr´ an l´ athat´ o´ aramk¨ or K kapcsol´ oja hossz´ u ideje z´ arva van. Egyszer csak a kapcsol´ ot kinyitjuk. Mekkora a tekercsben induk´ al´ od´ o fesz¨ ults´eg nagys´ aga k¨ ozvetlen¨ ul a kapcsol´ o kinyit´ asa ut´ an? P´eldat´ ari feladat nyom´ an

K¨ oz´ episkolai Matematikai ´ es Fizikai Lapok, 2022/4

= 20 Ω.

2

pedig a rendszer potenci´ alis (Coulomb-) energi´aja. Leolvashatjuk, hogy az o¨sszenergia 2 m∗ (ke2 ) E=− , 2n2 2

(5 pont)

+ 301 Ω

A f˝o´agban, vagyis a tekercsen kereszt¨ ul

a mozg´asi (kinetikus) energia, Epot. = −

1 1 60 Ω

249

2022. j´ unius 24. ´es j´ unius 30. k¨ oz¨ ott

Ism´et megrendezz¨ uk a t¨ obb ´evtizedes hagyom´anyokkal rendelkez˝ o fizikat´ abort ¨ ul˝ Domb´ ov´ar-Gunaras Ud¨ ofaluban, az apartman h´azakban ´es a hozz´ ajuk tartoz´ o z¨oldter¨ uleten. A t´ aborba v´ arjuk olyan fizika ir´ant nyitott tanul´ ok jelentkez´es´et a 9–11. ´evfolyamokr´ ol, akik tudn´ak v´ allalni az akt´ıv t´ abori r´eszv´etelt a vele j´ ar´ o utaz´ asi viszontags´ agokkal egy¨ utt. Els˝ osorban a K¨ oMaL feladatmegold´oit v´arjuk, de korl´ atozott sz´ amban m´as – a fizika ir´ant hat´arozottan ´erdekl˝ od˝ o – di´ ak is r´eszt vehet a t´ aborban, ha valamilyen versenyeredm´enye, vagy a fizikatan´ ar´ anak aj´anl´ asa ezt al´ at´ amasztja. A t´aborban (k¨ ul¨ on tan´ arokkal ´es programmal) r´eszt vesz a nemzetk¨ ozi mateasokat (nemmatikai di´ akolimpi´ara k´esz¨ ul˝ o matematikus csapat” is, ´es az esti el˝oad´ ” zetk¨ozileg is ismert el˝ oad´ okkal) k¨ oz¨ osen hallgathatj´ atok meg. A t´aborba olyan hat´aron t´ uli magyar k¨ oz´episkol´ asokat is v´ arunk, akik akt´ıv K¨ oMaL versenyz˝ok, vagy a fizika ir´ant elk¨ otelezett, m´as versenyeken eredm´enyesen szerepl˝ o di´ akok. A Nemzeti Tehets´egprogram keret´eben p´aly´ azott ´es elnyert ¨ osszeg 2 860 000 Ft, mely p´ aly´ azati forr´ asb´ ol biztos´ıtja a MATFUND Alap´ıtv´ any a t´ abor k¨ olts´eg´enek egy r´esz´et (sz´ all´ as + napi h´aromszori ´etkez´es, f¨ urd˝obel´ep˝ o, jutalmak, el˝oad´ ok tiszteletd´ıja stb.). 250

K¨ oz´ episkolai Matematikai ´ es Fizikai Lapok, 2022/4

G. 780. Zavarja-e a halakat, ha a partt´ ol 2 m´eter t´ avols´ agban besz´elgetnek a horg´aszok? (A hang terjed´esi sebess´ege leveg˝ oben 340 m/s, v´ızben pedig 1500 m/s.) (4 pont) P. 5400. A kis herceg egyik g¨ omb alak´ u bolyg´ oja olyan gyorsan forog a tengelye k¨or¨ ul, hogy az egyenl´ıt˝ oj´en nulla a neh´ezs´egi gyorsul´ as. Milyen ir´anyban n˝onek a f´ ak a bolyg´on? (4 pont)

M´ ajus k¨ ozep´en email-ben k¨ uldj¨ uk el a di´ akoknak a t´ abori felh´ıv´ ast ´es a jelentkez´esi lapot, melyet azoknak kell elektronikusan visszak¨ uldeni¨ uk, akik a ny´ ari K¨ oMaL Fizikat´abor r´esztvev˝ oi akarnak lenni. A jelentkez´esi hat´ arid˝o: m´ajus 31. T´ uljelentkez´es eset´en a pontverseny pillanatnyi a´ll´asa, illetve a be´erkezett jelentkez´esek sorrendje lesz a m´ervad´o.

P. 5401. Egy kicsiny (pontszer˝ unek tekinthet˝ o), de neh´ez testet k´et egyforma hossz´ u, k¨ozel azonos teherb´ır´ as´ u fon´ alon tartunk. A fonalak fels˝o v´eg´et egy v´ızszintes egyenes ment´en lassan elt´ avol´ıtjuk egym´ast´ ol. Amikor a fonalak 2α sz¨ oget z´arnak be egym´assal, az egyik fon´al elszakad, ´es a test a m´asik fon´ al r¨ ogz´ıtettnek tekinthet˝ o v´ege k¨ or¨ ul ingak´ent lengeni kezd. Mekkora lehetett α, ha a m´ asik fon´ al a leng´esek sor´ an nem szakad el? (5 pont)

adig P´eter, V´ acduka K¨ ozli: Gn¨

(5 pont)

P. 5402. Egy R sugar´ u, elhanyagolhat´ o t¨ omeg˝ u, v´ekony hengeres abroncsra egy m t¨ omeg˝ u, pontszer˝ u nehez´eket er˝ os´ıtett¨ unk. Az abroncs az ´ abr´ an l´athat´ o labilis egyens´ ulyi helyzet´eb˝ ol kimozdul, ´es akkor cs´ uszik meg a talajon, amikor k¨ oz´eppontj´anak elmozdul´asa ´eppen R. Mekkora a tapad´ asi s´ url´od´asi egy¨ utthat´ o az abroncs ´es a v´ızszintes talaj k¨ oz¨ ott? K¨ ozli: Balogh P´eter, G¨ od¨ oll˝ o

Fizik´ab´ ol kit˝ u z¨ ott feladatok

M. 413. M´erj¨ uk meg az ´etolaj t¨ or´esmutat´ oj´at! (6 pont)

K¨ozli: Gn¨ adig P´eter, V´acduka

G. 777. Szobah˝ om´ers´eklet˝ u pressz´ok´ av´et felg˝ oz¨ol´essel” szeretn´enk felmeleg´ı” teni. Becs¨ ulj¨ uk meg, hogy mennyit romlik ek¨ ozben a k´ av´e min˝ os´ege”, vagyis a t¨o” m´enys´ege! (4 pont) G. 778. Az ´ abra pocsoly´ an a´thalad´ o biciklikerekek vizes nyom´anak egy r´eszlet´et mutatja a sz´ araz aszfalton. Balr´ ol jobbra vagy jobbr´ol balra mozgott a bicikli? Melyik az els˝o kerek´enek a nyoma, ´es melyik a h´ats´ o´e?

P. 5403. Valaki egy olyan mobilg´ at ´ep´ıt´es´et javasolta, ami egy v´ızszintes ´es hozo vaslapb´ ol ´ all. A k´et f´emlapnak a k¨ oz¨ os ´elre mer˝ oleges z´a 45◦ -os sz¨ogben csatlakoz´ √ aga 2 cm. Ezt a szerkezetet m´erete 1 m´eter, illetve 2 m´eter, a f´emlapok vastags´ ” nemcsak a vaslapok s´ ulya, de m´eg a v´ız nyom´ asa is a talajhoz szor´ıtja” – ´ervelt a feltal´ al´o. a) Legal´abb mekkora (tapad´ asi) s´ url´od´asi egy¨ utthat´ ora van sz¨ uks´eg a mobilg´ at ´es a talaj k¨oz¨ott, hogy a v´edelem m´eg a maxim´ alis v´ızmagass´ ag eset´en is biztons´agos legyen?

(3 pont)

G. 779. Ha a Hold felsz´ın´et ´ oce´ anok ´es sz´ arazulatok bor´ıtan´ ak, akkor lenne-e a Holdon ap´ aly ´es dag´ aly? (3 pont) K¨ oz´ episkolai Matematikai ´ es Fizikai Lapok, 2022/4

251

252

K¨ oz´ episkolai Matematikai ´ es Fizikai Lapok, 2022/4

b) Hol lehet az ilyen alak´ u mobilg´ atakon a talaj ´altal kifejtett f¨ ugg˝oleges nyom´ oer˝ o t´ amad´aspontja, ha 2 cm-n´el vastagabb, vagy v´ekonyabb vaslapot alkalmazunk? Felborulhat-e a mobilg´ at a legmagasabb v´ız´all´asn´ al (vagyis amikor a v´ız ´eppen a ferde vaslap tetej´eig ´er)? Vizsg´aljuk meg a mobilg´ at elhelyez´es´enek k´etf´ele lehet˝ os´eg´et:

´ Aprilisi p´otfeladat.∗ A K¨ oMaL minden sz´am´at a nyomd´ aba ad´ as el˝ott ketten is elolvass´ ak. A mostani sz´ amban az egyik lektor 60, a m´asik 40 hib´at tal´ alt, ´es ezek k¨oz¨ott 35 volt olyan, amelyet mindketten ´eszrevettek. Becs¨ ulj¨ uk meg, hogy h´any hiba maradhatott ezek ut´ an a k´eziratban!



(i) a ferde fel¨ ulet a v´ız fel´e d˝ ol; (ii) a ferde fel¨ ulet a v´edend˝ o ter¨ ulet fel´e d˝ ol. (5 pont)

Beku ¨ld´esi hat´arid˝o: 2022. m´ajus 15. Elektronikus munkafu ¨zet: https://www.komal.hu/munkafuzet K¨ozli: Simon P´eter, P´ecs

u h˝otart´ alyok P. 5404. Egy ide´alis Carnot-g´ep T1 ´es T2 (T2 < T1 ) h˝om´ers´eklet˝ seg´ıts´eg´evel (izotermikus ´es adiabatikus a´llapotv´ altoz´ asokon kereszt¨ ul) ciklusonk´ent W hasznos munk´ at tud v´egezni. Hogyan m´odosul a h˝ oer˝ og´ep hat´ asfoka, ha a munkahenger dugatty´ uj´ anak kicsiny s´ url´od´asa miatt ciklusonk´ent 2q h˝o fejl˝odik (q  W ), ´es ez a h˝ o fele-fele ar´ anyban megosztva visszaker¨ ul a h˝ otart´ alyokba? (5 pont)

aszl´ o, Budapest K¨ozli: Holics L´

P. 5406. Maxim´alisan mekkora potenci´ alk¨ ul¨ onbs´eg hozhat´ o l´etre egy U fesz¨ ults´eg˝ u telep ´es k´et egyforma kondenz´ ator seg´ıts´eg´evel? A kondenz´ atorok felt¨olt´es¨ uk ut´ an szabadon ´atrendezhet˝ ok ´es u ´jra bek¨ othet˝ ok egy h´al´ozatba. (5 pont)

P´eldat´ ari feladat nyom´an

P. 5407. A CERN egyik line´ aris gyors´ıt´ oj´aban kezdetben a´ll´onak tekinthet˝o protonokat gyors´ıtanak L = 30,0 m hossz´ uu ´ton U = 500 MV fesz¨ ults´eggel. Feltehetj¨ uk, hogy a gyors´ıt´ oban az elektromos t´er homog´en. Mennyi id˝ o alatt teszik meg a protonok az L t´ avols´ agot? (5 pont)

Sv´ajci versenyfeladat

P. 5408. A v´ızszintes fel¨ uleten l´ev˝ o, tart´ooszo, M = 3m t¨ omeg˝ u kiskocsira egy loppal rendelkez˝ fon´ aling´at szerel¨ unk. Az inga hossza L = 50 cm, a v´eg´en l´ev˝ o, pontszer˝ unek tekinthet˝ o goly´ o t¨ omege m = 0,15 kg. Kezdetben a testek nyugalomban vannak. Az ing´at a fon´ al feszes ´allapot´ aban v´ızszintes helyzet´eig kit´er´ıtj¨ uk, ´es kezd˝ osebess´eg n´elk¨ ul elengedj¨ uk. A s´ url´od´ as mindenhol elhanyagolhat´o. a) Mekkora a kocsi sebess´ege, amikor a fon´ al α = 60◦ -os sz¨oget z´ ar be a f¨ ugg˝ oleges ir´ annyal? b) Mekkora er˝ o fesz´ıti ekkor a fonalat? (6 pont)

K¨ oz´ episkolai Matematikai ´ es Fizikai Lapok, 2022/4

MATHEMATICAL AND PHYSICAL JOURNAL FOR SECONDARY SCHOOLS (Volume 72. No. 4. April 2022) Problems in Mathematics

K¨ ozli: Wiedemann L´ aszl´ o, Budapest

P. 5405. K´et k¨ ul¨ on´ all´ o ellen´ all´ ason o oss´eg˝ u ´aram folyik ´at. Igazol¨sszesen I er˝ juk, hogy a k´et ellen´ all´ asra es˝ o¨ osszteljes´ıtm´eny akkor minim´alis, ha a k´et ellen´ all´asra es˝ o fesz¨ ults´eg megegyezik! (4 pont)



K¨ozli: Kotek L´ aszl´ o, P´ecs

253

New exercises for practice – competition C (see page 226): Exercises up to grade 10: C. 1714. The integers 1 to 22 are written on a blackboard. In each move, a pair of numbers is selected, erased and replaced with the absolute value of their difference. Prove that the last number added to the board is odd. (German problem) C. 1715. A circle k1 of radius 8 cm lies in the interior of a circle k. Both circles intersect the circle k2 of radius 15 cm, as shown in the figure. What is the radius of k if the shaded area inside k but outside k1 is equal to the total area of the shaded regions in the interior of k2 ? Exercises for everyone: C. 1716. In factorial representation, the place values of the digits are not the powers of a base: the nth place value is n factorial. Thus the digit in the first place is to be multiplied by 1, the digit in the second place is multiplied by 2, that in the third place is multiplied by 6, and so on. For example, the number 3310! in factorial representation corresponds to the number 3 · 4! + 3 · 3! + 1 · 2! = 92 in decimal notation. (If there is a number of more than one decimal digits in a certain place then it is indicated by using brackets.)† It is observed that one third of 111! is 11! , one third of 111 111! is 22 011! , and one third of 111 111 111! is 33 022 011! . Determine the factorial representation of one third of the number that consists of 3n digits of 1, also given in factorial representation. (Based on the idea of I. L´en´ art, Budapest) C. 1717. Let x1 and x2 denote the two real roots of the x x 1 1 equation 15x2 − 21x + 7 = 0. Find the exact value of the expression x1 + x2 + x + x . 2 1 1 2 C. 1718. Eight unit cubes are placed on a plane, and five further unit cubes are placed on top of them as shown in the figure. Find the lengths of the sides of triangle ABC. Exercises upwards of grade 11: C. 1719. In the interior of a regular triangle ABC consider the points P where side AB subtends an angle of 135◦ . Prove that the line segments P A, P B, P C can always form a triangle, and one angle of such triangles is always the same, independently of the position of point P . C. 1720. The elements of a given set of 10 elements are all at most two-digit positive integers. Is it true that such a set will always have two disjoint subsets in which the sum of the elements is equal?

∗ A p´ otfeladat megold´ asa bek¨ uldhet˝ o a [email protected] c´ımre, de nem sz´ am´ıt bele a pontversenybe. † It can be shown that the representation is unique, that is, every positive integer has a single factorial representation. See the Informatics problems I. 553. of the January issue.

254

K¨ oz´ episkolai Matematikai ´ es Fizikai Lapok, 2022/4

New exercises – competition B (see page 227): B. 5238. Solve the following equation over the set of positive integers: (k + n)! = k3 + n3 + (k + n)(3kn − 1). (3 points) (Proposed by M. Szalai, Szeged) B. 5239. The sides a, b and c of a triangle, in this order, form an arithmetic sequence. Show that the centre of the inscribed circle divides the angle bisector drawn to side b in a 1 : 2 ratio. (3 points) B. 5240. Show that every positive integer n has a multiple in which the sum of the digits is n. (4 points) (Proposed by Cs. S´ andor, Budapest) B. 5241. In a triangle ABC, the centre of the circumscribed circle is O, and ∠ABC > 90◦ . The tangent drawn to the circumscribed circle at C intersects line AB at point P , and the perpendicular drawn from P to BC intersects line OC at Q. Prove that AB is perpendicular to AQ. (4 points) (Proposed by Z. L. Nagy, Budapest) B. 5242. Let m and n denote arbitrary positive integers. Consider those lattice points (x; y) in the Cartesian coordinate plane for which 1  x  m and 1  y  n. What is the maximum possible number of points that can be selected out of these mn lattice points such that no four points selected should form a non-degenerate parallelogram? (6 points) (Proposed by E. F¨ uredi, Budapest) B. 5243. In a triangle ABC, ∠CAB = 48◦ and ∠ABC = 54◦ . D is an interior point of the triangle such that ∠CDB = 132◦ and ∠BCD = 30◦ . Prove that the line segments forming the polygon ACDB cannot form a triangle. (5 points) B. 5244. Determine all integers n > 4 such that (k, n) > 1 for all composite numbers k less than n. (5 points) (Proposed by S. R´ oka, Ny´ıregyh´ aza) B. 5245. a) Prove that there exist infinitely many, pairwise non-similar triangles in which the lengths of the sides are integers, and one angle is 3 times as large as another. b) Is there a triangle with the above property in which the lengths of the sides are all at most 10? (6 points) (Based on the idea of M. Hujter, Budapest) April puzzle∗ . Place on a chessboard six white chess pieces selected from two chess sets such that however a black piece is placed on a vacant field, it can be eliminated immediately. New problems – competition A (see page 229): A. 824.  An infinite set S of positive numbers is called thick, if in every interval of the form 1/(n + 1), 1/n (where n is an arbitrary positive integer) there is a number which is the difference of two elements from S. Does there exist a thick set such that the sum of its elements is finite? (Proposed by G´ abor Sz˝ ucs, Sziksz´ o) A. 825. Find all functions f : Z+ → R+ that satisfy f (nk2 ) = f (n)f 2 (k) for all positive integers n and k, furthermore lim

n→∞

f (n+1) = 1. A. 826. An antelope is a chess f (n)

piece which moves similarly to  (x1 , y1 ) and (x2 , y2 ) are joined by an  the knight: two cells antelope move if and only if |x1 − x2 |, |y1 − y2 | = {3, 4}. The numbers from 1 to 1012 are placed in the cells of a 106 × 106 grid. Let D be the set of all absolute differences of the form |a − b|, where a and b are joined by an antelope move in the arrangement. How many arrangements are there such that D contains exactly four elements? (Proposed by Nikolai Beluhov, Bulgaria)

Problems in Physics (see page 251) M. 413. Measure the refractive index of cooking oil. G. 777. We would like to heat room temperature espresso coffee by “steaming”. Estimate how much the “quality” of the coffee deteriorates, that is, how much the coffee concentration decreases. G. 778. The figure shows the wet track of a bicycle on a dry asphalt after the bicycle passed a puddle. Did the bike move from the left to the right or ∗

Out of competition. A possible solution will be shown on the cover of the May issue.

K¨ oz´ episkolai Matematikai ´ es Fizikai Lapok, 2022/4

255

from the right to the left? Which trail was made by the front wheel and which one was made by the rear one? G. 779. Would there be tides on the Moon if its surface was covered by oceans and landmasses? G. 780. Are fish disturbed by fishermen talking 2 metres away from the shore? (The speed of sound is 340 m/s in air and 1500 m/s in water.) P. 5400. One of the little prince’s spherical planets is spinning so fast that the gravitational acceleration on its equator is zero. In which direction do the trees on the planet grow? P. 5401. A small (considered point-like) but heavy body is held by two ropes having the same length and approximately the same working load limit. The upper ends of the ropes are slowly moved apart along a horizontal line. When the angle between the ropes becomes 2α, one of the ropes breaks and the body begins to swing as a pendulum about the fixed end of the other rope. What could the value of α be if the other rope does not break during the motion? P. 5402. A point-like weight m is attached to a thin ring of radius R and of negligible mass The ring moves out of its unstable equilibrium position shown in the figure, and it starts slipping on the ground when the displacement of its centre is just R. What is the coefficient of friction between the ring and the horizontal ground? P. 5403. Someone suggested building a mobile dam from iron sheets: a horizontal one and another which is attached to it and is tilted at an angle of 45◦ . The length of the edges of the sheets which √ are perpendicular to the common intersection line of the two sheets are 1 metre and 2 metres, and the width of the iron sheets is 2 cm. “This structure is pressed against the ground not only by the weight of the iron plates but also by the pressure of the water” – argued the inventor. a) What is the least value of the coefficient of static friction between the mobile dam and the ground in order that the dam should be able to protect safely even if the water level is maximum (that is: the level of the water reaches the top edge of the slant iron sheet)? b) Where is the point of application of the vertical normal force exerted by the ground on the mobile barrier of this shape, if we use iron sheets having a thickness less than 2 cm or more than 2 cm? Can the mobile dam overturn in the case of the highest water level (i.e. when the water just to the top of the tilted iron plate)? Investigate the two options for installing the mobile dam: (i) the tilted surface slopes towards the water; (ii) the tilted surface slopes towards the area to be protected (see the figure). P. 5404. An ideal Carnot heat engine, with the help of heat reservoirs of temperatures T1 and T2 (T2 < T1 ), can perform W work in each cycle (through isothermal and adiabatic processes). How will the efficiency of the heat engine change if the small friction between the piston in the working cylinder causes that 2q heat in each cycle is released (q  W ), and this heat is absorbed evenly by the two heat reservoirs? P. 5405. The sum of the currents through two resistors is I. Prove that the total dissipated power in the two resistors is minimum if the voltages across the two resistors are equal. P. 5406. What is the maximum potential difference that can be created with the help of two alike capacitors and a battery of electromotive force U ? The capacitors can be rearranged and connected into a circuit again after they were charged. P. 5407. Protons initially considered stationary in a linear accelerator at CERN are accelerated along a path of L = 30.0 m through a voltage of U = 500 MV. The electric field in the accelerator can be considered uniform. How long does it take for the protons to travel the distance L? P. 5408. A simple pendulum is attached to the top of a rod mounted to a trolley of mass M = 3m. The trolley is on a horizontal surface. The length of the thread of the pendulum is L = 50 cm, and the mass of the point-like bob at its end is m = 0.15 kg. Initially the objects are at rest. Then the pendulum bob is displaced such that its thread is tight and horizontal, and then released without initial speed. Friction is negligible everywhere. a) What is the speed of the trolley, when the angle between the thread and the vertical is α = 60◦ ? b) What is the tension in the thread at this position?

72. ´evfolyam 4. sz´am

K¨oMaL

Budapest, 2022. ´aprilis

´ ´ FIZIKAI LAPOK ¨ EPISKOLAI KOZ MATEMATIKAI ES ˝ ´ITVE INFORMATIKA ROVATTAL BOV ´ ALAP´ITOTTA: ARANY DANIEL 1894-ben 72. ´evfolyam 5. sz´am

Kedves Olvas´ oink! Budapest, 2022. m´ajus

´ Megjelenik ´evente 9 sz´amban, janu´art´ol m´ajusig ´es szeptembert˝ol decemberig havonta 64 oldalon. ARA: 1050 Ft

´ TARTALOMJEGYZEK Kedves Olvas´oink! . . . . . . . . . . . . . . . . . . . . . . . . . . . . . . .

258

Ericsson-d´ıj – t´aj´ekoztat´o . . . . . . . . . . . . . . . . . . . . . . . .

258

Nasz´odi M´arton: Konvex testbe ´ırhat´o tetra´eder d-dimenzi´oban . . . . . . . . . . . . . . . . . . . . . . . . . . . . . . . .

258

Kiss Melinda Fl´ora, Baran Zsuzsa, Janzer Lili: EGMO 2022 . . . . . . . . . . . . . . . . . . . . . . . . . . . . . . . . . .

262

Az EGMO 2022 feladatai . . . . . . . . . . . . . . . . . . . . . . . .

263

EGMO besz´amol´o . . . . . . . . . . . . . . . . . . . . . . . . . . . . . . .

264

J´ocsik Csilla: Megold´asv´azlatok a 2022/4. sz´am emelt szint˝u matematika gyakorl´o feladatsor´ahoz . . . . . . . . . . . . . . . . . . . . . . . . . . . . . . . . . . . . . . .

266

Matematika feladatok megold´asa (5164., 5178., 5187., 5206., 5209., 5214., 5229., 5235.) . . . . . . . .

277

A C pontversenyben kit˝uz¨ott gyakorlatok (1721– 1727.) . . . . . . . . . . . . . . . . . . . . . . . . . . . . . . . . . . . . . . . .

289

A B pontversenyben kit˝uz¨ott feladatok (5246– 5253.) . . . . . . . . . . . . . . . . . . . . . . . . . . . . . . . . . . . . . . . .

290

Az A pontversenyben kit˝uz¨ott nehezebb feladatok (827–829.) . . . . . . . . . . . . . . . . . . . . . . . . . . . . . . . . . . . .

291

Informatik´ab´ol kit˝uz¨ott feladatok (565–567., 63., 162.) . . . . . . . . . . . . . . . . . . . . . . . . . . . . . . . . . . . . . . . . .

292

K¨onyvismertet´es . . . . . . . . . . . . . . . . . . . . . . . . . . . . . . . .

297

Fizika gyakorlatok megold´asa (771., 772.) . . . . . . . .

299

Fizika feladatok megold´asa (5359., 5382., 5384., 5385., 5387., 5388., 5389.) . . . . . . . . . . . . . . . . . . . . .

302

Fizik´ab´ol kit˝uz¨ott feladatok (414., 781–784., 5409–5417.) . . . . . . . . . . . . . . . . . . . . . . . . . . . . . . . . . .

313

Problems in Mathematics . . . . . . . . . . . . . . . . . . . . . . . .

318

Problems in Physics . . . . . . . . . . . . . . . . . . . . . . . . . . . . .

319

K¨ oz´ episkolai Matematikai ´ es Fizikai Lapok, 2022/5

´ EVA ´ F˝oszerkeszt˝o: RATKO ´ ¨ Fizikus szerkeszt˝o: GNADIG PETER ´ ILDIKO ´ M˝uszaki szerkeszt˝o: MIKLOS Bor´ıt´o: BURGHARDT ZSUZSA ´ Kiadja: MATFUND ALAP´ITVANY ´ RITA Alap´ıtv´anyi k´epvisel˝o: KOS Felel˝os kiad´o: KATONA GYULA Nyomda: OOK-PRESS Kft. ´ Felel˝os vezet˝o: SZATHMARY ATTILA INDEX: 25 450 ISSN 1215-9247 A matematika bizotts´ag vezet˝oje: ´ HERMANN PETER ´ BALINT, ´ ´ Tagjai: B´IRO GYENES ZOLTAN, ´ ´ KISS HUJTER BALINT, IMOLAY ANDRAS, ´ ´ GEZA, ´ ´ RITA, KOZMA GEZA, KOS KOS ´ MATOLCSI DAVID, ´ KATALIN ABIGEL, ´ ´ PACH PETER ´ ´ V´IGH ¨ ORDI ¨ OK PETERN E, PAL, VIKTOR A fizika bizotts´ag tagjai: ´ ´ ´ BARANYAI KLARA, HOLICS LASZL O, ´ ´ HONYEK GYULA, OLOSZ BALAZS, SZASZ ´ SZECHENYI ´ ´ KRISZTIAN, GABOR, VIGH ´ E, ´ VLADAR ´ KAROLY, ´ MAT WOYNAROVICH FERENC Az informatika bizotts´ag vezet˝oje: ´ ´ SCHMIEDER LASZL O ´ E, ´ FARKAS CSABA, FODOR Tagjai: BUSA MAT ´ ´ NIKOLETT, LOCZI ´ ZSOLT, LASZL O LAJOS, ´ ´ ´ SIEGLER GABOR, SZENTE PETER, TOTH ´ TAMAS ´ ANDREA, TASNADI ´ ANIKO ´ Ford´ıt´ok: GROF ´ ´ ¨ Szerkeszt˝os´egi titk´ar: TRASY GYORGYN E A szerkeszt˝os´eg c´ıme: 1117 Budapest, P´azm´any P´eter s´et´any 1/C III. emelet 3.405. Telefon: 372-2850 A lap megrendelhet˝o az Interneten: www.komal.hu/megrendelolap/reszletek.h.shtml. El˝ofizet´esi d´ıj egy ´evre: 8800 Ft K´eziratokat nem ˝orz¨unk meg ´es nem k¨uld¨unk vissza. Minden jog a K¨oMaL tulajdonosai´e. E-mail: [email protected] Internet: http://www.komal.hu This journal can be ordered from the Editorial office: P´azm´any P´eter s´et´any 1/C III. emelet 3.405. 1117–Budapest, Hungary telephone: +36 (1) 372-2850 or on the Postal address H–1518 Budapest 112, P.O.B. 32, Hungary, or on the Internet: www.komal.hu/megrendelolap/reszletek.e.shtml. A Lapban megjelen˝o hirdet´esek tartalm´a´ert felel˝oss´eget nem v´allalunk.

257

A k¨ovetkez˝ o tan´evre sz´ ol´o megrendel´esr˝ ol sz´ol´o inform´ aci´ ok v´ arhat´ oan m´ ajus k¨ozep´en ker¨ ulnek fel a honlapra. A Kiad´o

Ericsson-d´ıj – t´aj´ekoztat´ o

A k¨ovetkez˝ o ´evi Ericsson-d´ıj v´ arhat´ oan ny´ ar elej´en ker¨ ul ki´ır´asra. K´erj¨ uk, k¨ovess´ek figyelemmel honlapunkat ´es/vagy facebook oldalunkat. MATFUND Alap´ıtv´any

Konvex testbe ´ırhat´o tetra´eder d-dimenzi´ oban∗

1. Egy egyszer˝ u feladat: s´ıkidom h´aromsz¨ ogek k¨ oz´e szendvicselve” ” Adott egy korl´atos konvex s´ıkidom, K, amely z´ art, teh´ at a hat´arol´ o g¨ orb´et is tartalmazza. Vegy¨ uk az ¨ osszes K ´ altal tartalmazott h´ aromsz¨ oget. Bel´ athat´ o (ezt hidd el), hogy ezek k¨ oz¨ ott van (esetleg t¨ obb) maxim´ alis ter¨ ulet˝ u, legyen S ilyen h´aromsz¨og. Igazold, hogy ha S-et az s s´ ulypontj´ ab´ ol (−2)-szeres´ere nagy´ıtjuk (teh´ at k¨oz´eppontosan t¨ ukr¨ ozz¨ uk s-re, majd vessz¨ uk a k´ep´et a 2 ar´ any´ u, s k¨ oz´eppont´ u aromsz¨ og tartalmazza K-t. Ne olvass hasonl´ os´agn´ al), akkor az ´ıgy kapott S  h´ tov´ abb, a megold´ as k¨ ovetkezik! Megold´as. Vegy¨ unk egy S  -n k´ıv¨ uli p pontot. Ekkor S  valamelyik oldalegyeol. Jel¨ olje a1 ´es a2 az S  h´ aromsz¨ og ezen egyenesen fekv˝ o nese elv´ alasztja p-t S  -t˝ aromsz¨ og megfelel˝o cs´ ucsait. K¨ onny˝ u l´atni, hogy k´et cs´ ucs´ at, ´es a1 , illetve a2 az S h´ ´ ´ Nemzeti A cikk az Innov´ aci´ os ´es Technol´ ogiai Miniszt´erium UNKP-20-5 k´ odsz´ am´ u Uj Kiv´ al´ os´ ag Programj´ anak a Nemzeti Kutat´ asi, Fejleszt´esi ´es Innov´ aci´ os Alapb´ ol finansz´ırozott szakmai t´ amogat´ as´ aval k´esz¨ ult. ∗

258

K¨ oz´ episkolai Matematikai ´ es Fizikai Lapok, 2022/5

az a1 a2 p h´ aromsz¨ ognek nagyobb a ter¨ ulete, mint S-´e (azonos alap, nagyobb magass´ ag). Ez´ert S v´ alaszt´ as´ab´ ol ad´od´oan p nem lehet K pontja, amivel az a´ll´ıt´ ast bel´ attuk.  Ennek a cikknek a c´elja, hogy kimondjuk ´es bel´assuk a fenti feladat d-dimenzi´ os analogonj´ at. Ehhez el˝osz¨ or r¨ oviden bevezetj¨ uk a d-dimenzi´ os t´er fogalm´ at, ´es geometri´aj´anak n´eh´ any alapelem´et. 2. Magasabb dimenzi´ o – Bevezet´es Hogy n´ez ki a k´erd´es a h´aromdimenzi´ os t´erben? Hasonl´ o ´ervel´essel bel´ athat´ o, hogy ha egy korl´ atos konvex testben, amely z´art, teh´ at a hat´arol´ o fel¨ uletet is tartalmazza, vesz¨ unk egy legnagyobb t´erfogat´ u tetra´edert, akkor annak a s´ ulypontj´ ab´ol vett (−3)-szoros nagy´ıtottja tartalmazza a testet. Ehhez k´et dolgot kell tudni. Egyr´eszt azt, hogy ha a t´er egy pontja nincs a tetra´ederben, akkor annak egyik laps´ıkja a tetra´edert a pontt´ ol elv´alasztja. M´asr´eszt azt, hogy a tetra´eder t´erfogata egyenesen ar´ anyos az alapter¨ ulet ´es a magass´ ag szorzat´aval (az ar´ anyoss´ agi t´enyez˝ o 13 , de ez itt nem fontos). Mi t¨ ort´enik h´ aromn´ al magasabb dimenzi´ oban? Ehhez egy-k´et dolgot tiszt´azni kell, legink´abb azt, hogy mit jelent a magasabb dimenzi´ o. Err˝ol r´eszletesen ´ırtunk a K¨ oMaL 2018 szeptemberi sz´ am´aban [1]. R¨ oviden a d-dimenzi´ os eukl´ıdeszi geometria fel´ep´ıt´es´et ´ıgy (is) lehet kezdeni: Vegy¨ uk a val´ os rendezett sz´am d-esek halmaz´ at (d r¨ ogz´ıtett pozit´ıv eg´esz), teh´ at azt a halmazt, melynek elemei d koordin´ at´ ab´ol ´alank ponthalmaza, ´es Rd -vel jel¨olj¨ uk. Egy l´ o vektorok, (x1 , x2 , . . . , xd ), ez geometri´ ilyen vektort tudok szorozni val´ os sz´ammal (minden koordin´ at´ at megszorzok vele), ´es megint egy vektort kapok, tov´ abb´ a¨ ossze is tudok adni k´et ilyen vektort a szokott m´odon, koordin´ at´ ank´ent. Tudom most defini´ alni k´et pont (teh´ at vektor) ´altal arozott meghat´ arozott szakaszt. Az (x1 , . . . , xd ) ´es (y1 , . . . , yd ) pontok a´ltal meghat´ z´ art szakasz az (1 − λ)(x1 , . . . , xd ) + λ(y1 , . . . , yd ) pontok halmaza, ahol λ befutja a [0, 1] val´os intervallumot. Gondold meg (t´enyleg!), hogy d = 2, illetve 3 eset´en val´oban ´ıgy kapom meg a szakasz pontjait. Most m´ar k¨onny˝ u defini´alni Rd konvex halmazait: azon ponthalmazok, amelyek tetsz˝ oleges k´et pontjukra az azok a´ltal meghat´ arozott szakaszt is tartalmazz´ ak. avols´ ag´ at a s´ık- ´es t´erbeli esetek A p = (p1 , . . . , pd ) ´es q = (q1 , . . . , qd ) pontok t´ term´eszetes ´altal´ anos´ıt´ asak´ e nt a  2

2

2

d(p, q) = (p1 − q1 ) + (p2 − q2 ) + . . . + (pd − qd ) k´eplettel defini´ aljuk. ´Igy, hogy t´ avols´ agfogalmunk is van, defini´ alhatjuk a z´ art halmazt. Egy Rd -beli K halmazt z´ artnak h´ıvunk, ha tartalmazza minden torl´ od´ asi pontj´ at, azaz minden olyan p pontot, amelyhez tetsz˝ oleges ε > 0 t´avols´ agra van p-t˝ol legfeljebb ε t´avol l´ev˝ o K-beli pont. Gondold meg, hogy p´eld´ aul a s´ıkon egy k¨orlemez a hat´ arol´ o k¨ orvonal n´elk¨ ul nem z´ art, azzal egy¨ utt viszont z´ art halmaz. 3. Konvex burok, hipers´ık, szimplex

Mi a tetra´eder d-dimenzi´ os megfelel˝oje? Ehhez ´erdemes megismerkedni a konvex burok ´es a hipers´ık fogalm´ aval. K¨ oz´ episkolai Matematikai ´ es Fizikai Lapok, 2022/5

259

Feladat. Rd -ben tetsz˝ olegesen sok konvex halmaz metszete is konvex. R¨ogz´ıts¨ unk egy tetsz˝ oleges H ponthalmazt Rd -ben. Vegy¨ uk az ¨ osszes H-t tartalmaz´ o konvex halmaz metszet´et, jel¨ olj¨ uk K-val. A fenti feladat szerint K konvex halmaz, amely tartalmazza H-t. R´ aad´ asul K a legkisebb a H-t tartalmaz´ o konvex halmazok k¨ oz¨ ott, teh´ at minden H-t tartalmaz´ o konvex halmaz tartalmazza K-t is. Ezt a K-t h´ıvjuk H konvex burk´ anak. P´eld´ aul a s´ıkon h´ arom nem egy egyenesre es˝ o pont konvex burka az a h´aromsz¨ oglemez, amelynek cs´ ucsai az adott h´ arom pont. Hogyan lehet a konvex burok pontjait el˝o´ all´ıtani? Legyen H v´eges ponthalmaz, o indexek itt nem koordin´ at´ akat jel¨ olnek, hanem H = {p1 , p2 , . . . , pn } ⊂ Rd (az als´ a vektorok sz´ amoz´ as´at). Feladat. L´assuk be, hogy H konvex burka a (1)

λ1 p 1 + λ2 p 2 + . . . + λ n p n

alak´ u pontok halmaza, ahol λ1 , λ2 , . . . , λn  0 ´es λ1 + λ2 + . . . + λn = 1. Azt kell teh´at bel´atni, hogy az (1) alak´ u pontok halmaza tartalmazza H-t (ez k¨ onny˝ u), konvex, ´es tetsz˝oleges K-t tartalmaz´ o konvex halmaz tartalmaz minden (1) alak´ u pontot. A s´ıkon h´ arom pont lehet egy egyenesen, vagy nem, ut´obbi esetben azt mondhatjuk, hogy ´ altal´ anos helyzet˝ uek. Ekkor a konvex burkuk egy h´ aromsz¨ og. A t´erben n´egy pont eshet egy s´ıkba, vagy nem, ut´obbi esetben, megintcsak a´ltal´ anos helyzet˝ unek mondjuk ˝oket, ´es ´eszrevehetj¨ uk, hogy konvex burkuk egy (nem felt´etlen szab´ alyos) tetra´eder. Hogy megy¨ unk tov´ abb a dimenzi´oval? d uk azon (x1 , . . . , xd ) ∈ Rd pontok halmaz´ at, Az R t´erben hipers´ıknak nevezz¨ amelyek kiel´eg´ıtik az α1 x 1 + . . . + αd x d = α0 line´aris egyenletet valamely α0 , α1 , . . . , αd ∈ R sz´ amokra, ahol az α1 , . . . , αd sz´ amok nem mindegyike nulla. Gondold meg, hogy a s´ıkon minden egyenes ilyen alak´ u, a 3-dimenzi´ os t´erben pedig minden s´ık ilyen alak´ u. Ha a fenti egyenletben az egyenl˝os´egjelet , illetve  jelre cser´elj¨ uk, akkor az egyenl˝ otlens´eget kiel´eg´ıt˝ o pontok halmaza a hipers´ık ´ altal hat´ arolt egyik, illetve m´ asik z´ art f´elt´er. V´egre megv´ alaszolhatjuk, mi a tetra´eder d-dimenzi´ os megfelel˝oje. Ha p1 , p2 , altal´ anos helyzet˝ u pontok Rd -ben, teh´ at nem tartalmazza o˝ket . . . , pd+(x1 ,...,xd )1 ´ hipers´ık, akkor ezen d + 1 pont konvex burk´ at szimplexnek nevezz¨ uk. 3.1. Szimplex mint f´elterek metszete. Egy konvex ¨ otsz¨ oget a s´ıkon tekinthet¨ unk u ´gy, mint az ¨ ot cs´ ucs´ anak a konvex burka. De u ´gy is n´ezhet¨ unk r´ a, mint az ¨ ot oldalegyenes a´ltal meghat´ arozott z´ art f´els´ıkok metszete. Hasonl´oan, egy kock´ at a t´erben megkapunk mint a 8 cs´ ucs´ anak a konvex burka, de u ´gy is, hogy vessz¨ uk a 6 laps´ıkj´at ´es az azok ´altal meghat´ arozott (megfelel˝ o ir´ any´ u) 6 f´elt´er metszet´et. Nem l´ atjuk be, de nem meglep˝ o a k¨ ovetkez˝ o´ all´ıt´ as. 1. ´all´ıt´as. Legyenek p1 , p2 , . . . , pd+1 ´ altal´ anos helyzet˝ u pontok Rd -ben. Tudjuk, hogy minden i = 1, . . . , d + 1 indexre a {p1 , p2 , . . . , pd+1 } \ {pi } ponthalmaz egy 260

K¨ oz´ episkolai Matematikai ´ es Fizikai Lapok, 2022/5

hipers´ıkot hat´ aroz meg. Jel¨ olje Hi az ezen hipers´ık ´ altal hat´ arolt azon z´ art f´elteret, amely tartalmazza pi -t. Ekkor a p1 , p2 , . . . , pd+1 pontok konvex burkak´ent kapott szimplex megegyezik a H1 , . . . , Hd+1 f´elterek metszet´evel. 4. T´erfogat Az utols´ o fogalom, amellyel m´eg ad´ osak vagyunk, ha a cikk elej´en feltett k´erd´est d-dimenzi´ oban is fel akarjuk tenni, d-dimenzi´ os konvex halmaz t´erfogata. K¨ oz´episkol´ aban azt mondj´ ak, hogy az a oldalhossz´ u n´egyzet ter¨ ulete a2 , ´es minden konvex s´ıkidomot k¨ ozel´ıt¨ unk” v´eges sok p´ aronk´ent nem a´tfed˝ o (mondjuk ” tengely-p´ arhuzamos) n´egyzet uni´ oj´aval. Ahogy a k¨ozel´ıt´es egyre pontosabb, u ´gy tart a n´egyzetek o sszter¨ u lete egy sz´ a mhoz. Ezt a sz´ a mot h´ ıvjuk a konvex s´ ıkidom ¨ ter¨ ulet´enek. Ez egy j´o fel´ep´ıt´ese a ter¨ uletnek, de az´ert tudni kell, hogy itt n´eh´ any lukat majd csak az egyetemen (pl. matematika szakon) fognak bet¨omni: mit jelent az, hogy sok kis n´egyzet uni´ oja k¨ ozel van a s´ıkidomhoz? Ahogy finomodik a k¨ozel´ıt´es, mi´ert tart a n´egyzetek ¨ osszter¨ ulete egy sz´ amhoz? Ez a munka sz´epen elv´egezhet˝ o, de mi most ezt nem tessz¨ uk meg, mi is hitelbe” dolgozunk: hidd el, kedves Olvas´ o, ” hogy egy d-dimenzi´ os konvex testet ugyan´ıgy lehet kis kock´ ak uni´ojak´ent k¨ozel´ıteni, ´es ezen uni´ ok t´erfogata, ahogyan a k¨ ozel´ıt´es egyre pontosabb, tart egy sz´ amhoz, os t´erfogat´ anak h´ıvunk. amelyet a konvex test d-dimenzi´ 4.1. G´ ula t´erfogata. Legyen A az Rd t´er egy hipers´ıkj´anak egy konvex r´eszul es˝ o pont. Ekkor az A ∪ {p} halmaz halmaza, p ∈ Rd pedig ezen hipers´ıkon k´ıv¨ konvex burk´ at A alap´ u, p cs´ ucs´ u g´ ul´ anak h´ıvjuk. A hipers´ıkot azonos´ıthatjuk Rd−1 gyel, ´es ´ıgy tudunk A-nak a (d − 1)-dimenzi´os t´erfogat´ ar´ ol besz´elni. A hipers´ık ´es p t´avols´ aga (teh´at az o˝ket o sszek¨ o t˝ o legr¨ o videbb szakasz hossza) a g´ ula magass´ aga. ¨ Bel´ athat´ o, hogy (2) g´ ula d-dimenzi´ os t´erfogata =

Feladat. L´assuk be, hogy S-nek az s k¨ oz´eppont´ u (−d)-ar´ any´ u k¨ oz´eppontos abb, seg´ıts´eg k¨ ovetkezik! nagy´ıtottja, S  , tartalmazza S-et. Ne olvass tov´ ´ ıtsuk el˝ Seg´ıts´eg. All´ o a pi pontot pi = λ1 p1 + . . . + λi−1 pi−1 + λi+1 pi+1 + . . . + utthat´ ok nemnegat´ıvak, ´es az o uk 1. + λd+1 pd+1 alakban, ahol a λj egy¨ ¨sszeg¨ V´eg¨ ul kimondhatjuk a cikk bevezet˝ oj´eben szerepl˝ o s´ıkbeli feladat d-dimenzi´ os v´altozat´ at. art, korl´ atos, konvex halmaz, K. Vegy¨ uk az ¨ osszes K 2. t´etel. Adott Rd -ben egy z´ altal tartalmazott szimplexet. Bel´ ´ athat´ o (ezt hidd el), hogy ezek k¨ oz¨ ott van (esetleg t¨ obb) maxim´ alis d-dimenzi´ os t´erfogat´ u, legyen S ilyen szimplex. Ekkor, ha S-et az s s´ ulypontj´ ab´ ol (−d)-szeres´ere nagy´ıtjuk, akkor az ´ıgy kapott S  szimplex tartalmazza K-t. Bizony´ıt´as. A t´etel bizony´ıt´ asa az eddigi el˝ok´esz¨ uletekkel ugyanaz, mint a s´ıkbeli eset´e. Jel¨olje S cs´ ucsait a1 , . . . , ad+1 , az S  cs´ ucsait a1 , . . . , ad+1 . Ekkor 1. ´all´ıt´ as ul es˝ o p ponthoz van egy i ∈ {1, . . . , d + 1} index, hogy S  szerint minden S  -n k´ıv¨ az {a1 , . . . , ad+1 } \ {ai } pontok a´ltal meghat´ arozott hipers´ık egyik oldal´an tal´ alhat´o, m´ıg p a m´asikon. A (2) k´eplet alapj´an k¨ onny˝ u l´atni, hogy az   {a1 , . . . , ad+1 } \ {ai } ∪ {p}

halmaz konvex burkak´ent kapott szimplex t´erfogata nagyobb S t´erfogat´ an´ al, ez´ert p nem lehet K pontja.  Feladat. L´assuk be, hogy S-nek az s k¨ oz´eppont´ u (d + 1)-ar´ any´ u k¨ oz´eppontos nagy´ıtottja tartalmazza K-t. Hivatkoz´as [1] Nasz´ odi M.: Polit´ opok ´es a g¨ omb d-dimenzi´ oban, K¨ oMaL 68. ´evf. 9. sz. (2018).

 1  × alap (d − 1)-dimenzi´os t´erfogata × magass´ ag. d

Nasz´ odi M´arton

A k´eplet d = 2 ´es 3 esetben ismer˝ os lehet. Most nem l´ atjuk be (2)-t, ehhez egy kicsit kell tudni integr´ alni. 5. Szimplexek k¨ oz¨ ott

EGMO 2022

Ha egy h´aromsz¨ oget a s´ ulypontja k¨ or¨ ul (−2)-szeres´ere nagy´ıtunk, akkor egy ot tartalmaz´o h´ ˝ aromsz¨ oget kapunk. Ha egy tetra´edert a s´ ulypontja k¨or¨ ul (−3)szoros´ ara nagy´ıtunk, akkor egy o˝t tartalmaz´ o tetra´edert kapunk. Most v´egiggondoljuk mindezt d dimenzi´ oban. Legyen S a p1 , . . . , pd+1 ´ altal´ anos helyzet˝ u Rd -beli pontok konvex burkak´ent kapott szimplex. A s´ ulypontja az 1 (p1 + . . . + pd+1 ) s= d+1 pont. K¨ oz´ episkolai Matematikai ´ es Fizikai Lapok, 2022/5

261

Az idei Eur´ opai L´anyok Matematikai Olimpi´aj´at (EGMO-t) Magyarorsz´ag szervezte 2022. ´aprilis 6–12. k¨ oz¨ ott Egerben. A verseny weboldala: https://egmo2022.hu/. Erre a megm´erettet´esre szervez˝o orsz´ agk´ent k´et csapatot is deleg´alhattunk. A versenyen a Nemzetk¨ ozi Matematikai Di´ akolimpi´ahoz (IMO) hasonl´oan k´et versenynap van, mindk´et nap 3–3 feladatot kell megoldani 4,5 o´ra alatt. Minden feladat 7 pontot ´er. A feladatt´ıpusok a k¨ ovetkez˝ ok voltak: geometria, sz´ amelm´elet, 262

K¨ oz´ episkolai Matematikai ´ es Fizikai Lapok, 2022/5

algebra (els˝ o nap), algebra, kombinatorika, geometria (m´ asodik nap). A feladatsorok ´es megold´ asok a verseny hivatalos honlapj´an el´erhet˝oek: https://www.egmo.org/egmos/egmo11/.

3. Egy pozit´ıv eg´eszekb˝ ol ´ all´ o, v´egtelen a1 , a2 , . . . sorozatot kock´ asf¨ ul˝ unek nevez¨ unk, ha (1) a1 teljes n´egyzet, ´es

Id´en 57 orsz´ agb´ ol 222 r´esztvev˝ o oldotta meg a feladatokat. A magyar l´ anyok kiv´ al´o eredm´enyt ´ertek el. A HUN” csapat a hivatalos ” eur´ opai list´an a 31 eur´opai orsz´ag k¨ oz¨ ott az 5. helyet szerezte meg 96 ponttal. Az ¨ osszes (57) r´esztvev˝ o orsz´ agot tekintve a HUN” csapat 8., a HUNB” csapat ” ” pedig (nem hivatalos eur´ opai csapatk´ent) a 19. lett. Az egy´eni eredm´enyek: Fu op Csilla: 32 pont, eur´opai 3. hely, ¨ osszes´ıtett 12. hely, arany´erem; ¨ l¨ Kercs´ o-Moln´ar Anita: 25 pont, ¨ osszes´ıtett 35. hely, ez¨ ust´erem; P´ah´an Anita Dalma: 24 pont, eur´opai 24. hely, ¨ osszes´ıtett 40. hely, ez¨ ust´erem; Somogyi Dalma: 22 pont, ¨ osszes´ıtett 51. hely, ez¨ ust´erem; Sztrany´ak Gabriella: 21 pont, eur´opai 33. hely, ¨ osszes´ıtett 56. hely, bronz´erem; Nagy Leila: 19 pont, eur´opai 44. hely, ¨ osszes´ıtett 74. hely, bronz´erem; ´ Ung´ar Eva: 16 pont, ¨osszes´ıtett 97. hely, bronz´erem; Beinschroth Ninett: 15 pont, ¨ osszes´ıtett 118. hely.

(2) minden n  2 eg´eszre az an a legkisebb pozit´ıv eg´esz sz´ am, amire

teljes n´egyzet. Bizony´ıtsuk be, hogy minden kock´asf¨ ul˝ u a1 , a2 , . . . sorozathoz l´etezik olyan ul minden n  k eg´esz eset´en. pozit´ıv eg´esz k sz´ am, amire an = ak teljes¨ M´asodik nap 4. Adott n  2 pozit´ıv eg´esz sz´ amra hat´ arozzuk meg a legnagyobb pozit´ıv eg´esz ´gy, hogy N sz´ amot, amire l´etezik N + 1 val´os sz´am a0 , . . . , aN u 1 (1) a0 + a1 = − n , ´es (2) (ak + ak−1 )(ak + ak+1 ) = ak−1 − ak+1 minden 1  k  N − 1-re.

A v´egs˝ o eredm´enylista a https://www.egmo.org/egmos/egmo11/scoreboard/ oldalon tanulm´anyozhat´ o. ¨ ome Alap´ıtv´ K¨ osz¨ onj¨ uk a Morgan Stanley ´es A Gondolkod´ as Or¨ any t´ amogat´as´ at. A j¨ ov˝o ´evi verseny 2023. ´aprilis 13–19. k¨ oz¨ ott Portoroˇzban lesz. Rem´elj¨ uk, hogy j¨ ov˝ore is hasonl´oan sok lelkes l´annyal tal´alkozhatunk a felk´esz´ıt´es folyam´ an ´es a v´ alogat´ oversenyeken. Kiss Melinda Fl´ ora, Baran Zsuzsa, Janzer Lili az EGMO felk´esz´ıt˝o csapat nev´eben

Az EGMO 2022 feladatai Els˝ o nap 1. Legyen ABC egy olyan hegyessz¨ og˝ u h´ aromsz¨og, ahol BC < AB ´es BC < < CA. A P pont az AB szakaszon, a Q pont az AC szakaszon helyezkedik el u ´gy, hogy P = B, Q = C ´es BQ = BC = CP . Legyen T az AP Q h´ aromsz¨og k¨or¨ ul´ırt k¨ or´enek k¨ oz´eppontja, H az ABC h´ aromsz¨ og magass´ agpontja, valamint S a BQ ´es CP egyenesek metsz´espontja. Bizony´ıtsuk be, hogy a T , H ´es S pontok egy egyenesre esnek. 2. Jel¨ olje Z+ = {1, 2, 3, . . . } a pozit´ıv eg´esz sz´ amok halmaz´at. Keress¨ uk meg uggv´enyt, amire tetsz˝ oleges pozit´ıv eg´esz a, b sz´ aaz ¨ osszes olyan f : Z+ → Z+ f¨ mokra az al´ abbi k´et felt´etel mindegyike teljes¨ ul: (1) f (ab) = f (a)f (b), ´es (2) az f (a), f (b) ´es f (a + b) sz´ amok k¨ oz¨ ul legal´ abb kett˝ o egyenl˝ o.

K¨ oz´ episkolai Matematikai ´ es Fizikai Lapok, 2022/5

na1 + (n − 1)a2 + . . . + 2an−1 + an

263

5. Tetsz˝ oleges pozit´ıv eg´esz n, k sz´ amokra jel¨ olje f (n, 2k) azt a sz´ amot, ah´ anyf´elek´eppen egy n × 2k-as t´abla teljesen lefedhet˝o nk darab 2 × 1-es domin´ oval. (P´eld´ aul f (2, 2) = 2 ´es f (3, 2) = 3.) Keress¨ uk meg az ¨ osszes olyan pozit´ıv eg´esz n sz´ amot, amire minden pozit´ıv eg´esz k sz´ am eset´en f (n, 2k) p´ aratlan. 6. Legyen ABCD egy h´ urn´egysz¨ og, a k¨ or¨ ul´ırt k¨ or´enek k¨ oz´eppontj´at jel¨ olj¨ uk O-val. Az A ´es B pontokb´ ol h´ uzott bels˝ o sz¨ ogfelez˝ ok metsz´espontja legyen X, a B ´es C pontokb´ ol h´ uzott bels˝ o sz¨ ogfelez˝ ok metsz´espontja legyen Y , a C ´es D pontokb´ ol h´ uzott bels˝ o sz¨ ogfelez˝ ok metsz´espontja legyen Z, valamint a D ´es A pontokb´ ol h´ uzott bels˝ o sz¨ ogfelez˝ ok metsz´espontja legyen W . Tov´ abb´ a, az AC ´es BD egyenesek metsz´espontja legyen P . Tegy¨ uk fel, hogy az X, Y , Z, W , O ´es P pontok mind k¨ ul¨onb¨oz˝ oek. Bizony´ıtsuk be, hogy az O, X, Y , Z ´es W pontok pontosan akkor fekszenek egy k¨or¨on, ha a P , X, Y , Z ´es W pontok egy k¨ or¨ on fekszenek.

EGMO besz´amol´ o Az EGMO-ra egy szerdai napon ´erkezt¨ unk meg, egy¨ utt m´ asik 40 orsz´ag csapat´ aval. Mivel id´en Magyarorsz´ag rendezte a versenyt, a szok´ asos 4 helyett 8 versenyz˝ ovel indulhattunk. M´asnapra m´ ar a legt´ avolabbr´ol ´erkez˝ ok is ott voltak, ´es kezd˝ odhetett az esem´eny. A cs¨ ut¨ ort¨ oki napot a megnyit´oval ind´ıtottuk, aminek a m˝ usorai megalapozt´ ak a j´o hangulatot, ´es l´ athattuk az online r´esztvev˝ o csapatokat is. A verseny 264

K¨ oz´ episkolai Matematikai ´ es Fizikai Lapok, 2022/5

Megold´asv´azlatok a 2021/4. sz´am emelt szint˝ u matematika gyakorl´o feladatsor´ahoz

szempontj´ ab´ ol ez a nap a feladatsorok ford´ıt´ as´ar´ ol sz´olt, ´ıgy ameddig a leaderek k´esz´ıtett´ek nek¨ unk a feladatokat, mi egy v´ arosn´ez˝ o kincsvad´ aszaton vehett¨ unk r´eszt csapatonk´ent. Itt a v´ aros minden pontj´an l´ev˝ o´ allom´asokon kellett k¨ ul¨onb¨oz˝ o vicces feladatokat megoldani, mint a min´el magasabb jenga torony ´ep´ıt´ese. Ezeket a szuper guide csapat szervezte meg ´es bonyol´ıtotta le, akik igaz´ an kitettek maguk´ert az eg´esz verseny alatt. A harmadik ´es a negyedik nap d´elel˝ ottjein voltak a versenyek, ahol 3-3 feladaton gondolkozhattunk. A versenynapokat este egy-egy csapatmegbesz´el´es el˝ ozte meg, ahol sok j´otan´ acsot ´es b´ ator´ıt´ ast (¨ olel´est) kaptunk a csapatvezet˝ oinkt˝ ol. A verseny ut´ an k¨ ul¨ onb¨ oz˝ o workshopokon vett¨ unk r´eszt, p´eld´ aul a Jane Street szervez´es´eben k¨ ul¨ onb¨ oz˝ o dolgok sz´am´at/hossz´ at tippelhett¨ uk meg, lehetett sportolni, uveskedni. Este pedig egy karaoke-estet tartottunk, ahol a fiv´ arost n´ezni ´es k´ezm˝ atal szervez˝okkel ´es a t¨ obbi versenyz˝ovel egy¨ utt a legnagyobb sl´agerekre (pl. Katy Perry Hot’n’cold-j´ara) tomboltunk, ´es az ´enekhangunkt´ ol zengett a hotel, mik¨ozben a koordin´ atorok a megold´ asaink jav´ıt´as´ aval bajl´ odtak. Vas´ arnap vonattal elment¨ unk a Szalajka-v¨ olgybe, ahol egy gyors kisvonatoz´ as ut´ an a Medve Matek a´ltal szervezett programon vett¨ unk r´eszt. K¨ ul¨onb¨oz˝ o helysz´ıneken kellett feladatokat megoldanunk, majd egy¨ utt visszas´et´ altunk a vas´ ut´ allom´ asra. M´ıg mi j´ol ´erezt¨ uk magunkat az erd˝ oben, a koordin´atorok ´es a csapatvezet˝ ok befejezt´ek a feladatok jav´ıt´ as´at, ´es a nap v´eg´ere m´ ar meg is lettek az eredm´enyek. Est´ere kv´ız ´es k¨ oz¨ os ´enekl´es volt tervezve, majd t´arsasoztunk t¨obb csapattal. M´ asnap d´elel˝ ott Budapestre ment¨ unk v´ arost n´ezni, ahol a k¨otelez˝o program ult beiktatnunk egy gofriz´ ast is :). D´elut´ an a z´ar´ ou mell´e siker¨ ¨nneps´eggel hivatalosan is v´eget ´ert a verseny r´esze az esem´enynek. Miut´ an megkaptuk az 1 arany-, 3 ez¨ ust- ´es 3 bronz´ermet, a k¨ oz¨ os vacsor´aval kezdt¨ uk az u ¨nnepl´est. Ut´ana rengeteget t´ ancoltunk, nagyon j´ ol ´erezt¨ uk magunkat. J´ o volt a t´arsas´ ag, egy¨ utt buliztak a versenyz˝ ok, a guidok, a csapatvezet˝ ok ´es a koordin´ atorok is. Az EGMO nem j¨ ohetett volna l´etre a lelkes szervez˝ ok, a csod´ as csapatvezet˝ oink ´es a szuper guideok n´elk¨ ul. Minden nap tele volt programmal, sose unatkoztunk egy percig sem. K´os G´ez´ anak szem´elyes k¨ osz¨ onet a kock´ asf¨ ul˝ u nyulak´ert, akik a verseny alatt is t´ amogattak, ´es akik miatt kiharcoltuk, hogy egy helyett k´et pl¨ usst is be lehessen vinni a terembe. Melind´ anak, Lilinek ´es Zsuzs´ anak k¨osz¨onj¨ uk a sok t´ amogat´ ast ´es tan´ acsot. Az EGMO nagy hat´assal volt r´ ank, eml´ekezetess´e tette m´eg az ismerked˝os bingo j´ at´ek ´es a szuper, o tletes feladatok, p´eld´ aul az 5-¨os kombinatorika. Nagy ¨ ´elm´eny volt sz´ amunkra, hogy k¨ ozel 40 orsz´agb´ ol ´erkez˝o, hasonl´o gondolkod´ as´ u, okos, matekos l´anyokkal ismerkedhett¨ unk meg, akikkel osztozhattunk a matek ´es a tudom´anyok szeretet´eben. A versenyz˝ ok nem csak Eur´op´ ab´ ol, hanem a vil´ ag minden t´aj´ar´ ol ´erkeztek, besz´elgett¨ unk pl. amerikaiakkal, n´emetekkel, costa ricaiakkal, ´ırekkel, izraeliekkel, indiaiakkal ´es azerbajdzs´ aniakkal is. Szuper ´elm´eny volt m´ as kult´ ur´ akkal megismerkedni, ´es ´eletre sz´ ol´o bar´ ats´ agokat k¨ otni a magyar ´es k¨ ulf¨oldi l´ anyokkal. A magyar csapat (Vica, K-M. Anita, P. Anita, Leila, Gabi, Ninett, Dalma, Csilla) K¨ oz´ episkolai Matematikai ´ es Fizikai Lapok, 2022/5

265

I. r´esz 1. Oldjuk meg a k¨ ovetkez˝ o egyenl˝ otlens´egeket a val´ os sz´ amok halmaz´ an: 2 a) a − 5a + 4  0, (3 pont) b) log 1 (5x+1 − 25x )  −2.

(8 pont)

2

u kifejez´es gy¨ okei: a1 = 1, a2 = 4. A paMegold´as. a) Az a2 − 5a + 4 m´asodfok´ rabola ´abr´ azol´ asa ut´ an leolvashat´o, hogy az [1; 4] intervallumon teljes¨ ul az egyenl˝otlens´eg. ulnie b) A logaritmus defin´ıci´ oja ´ertelm´eben az 5x+1 − 25x > 0 felt´etelnek teljes¨ x uggv´eny szigor´ uan monoton n˝ o, ha kell. Emiatt 0 < 5 < 5. Az exponenci´alis f¨ az alapja 1-n´el nagyobb. ´Igy a kifejez´es ´ertelmez´esi tartom´anya D = ]−∞; 1[. Logaritmusos kifejez´esk´ent ´ırhatjuk fel a jobb oldalt: log 1 (5x+1 − 25x )  log 1 4. 2

2

A logaritmusf¨ uggv´eny szigor´ uan monoton cs¨ okken, ha az alapja kisebb, mint 1. Rendezve a m´ asodfok´ u egyenl˝ otlens´eget: 0  52x − 5 · 5x + 4. ´ v´altoz´ Uj o vezethet˝ o be 5x helyett, legyen h´ at a = 5x , ´es ´ıgy az el˝ oz˝ o r´eszx an, feladat eredm´eny´et felhaszn´ alhatjuk: 1  5  4, amelyb˝ol visszahelyettes´ıt´es ut´ az exponenci´alis f¨ uggv´eny monoton n¨ oveked´es´ere hivatkozva, az ´ertelmez´esi tartom´annyal ¨osszevetve ad´ odik a megold´ as: x ∈ [0; log5 4]. 2. A bolth´ al´ ozat rakt´ ar´ aban 14 500 doboz u ot t´ arolnak. A r¨ ovid szavatoss´ agi ¨d´ıt˝ id˝ o miatt a tulajdonos szeretn´e a teljes k´eszletet 29 napon bel¨ ul eladni. Az els˝ o napon 150 darabot siker¨ ult ´ert´ekes´ıteni. a) Legal´ abb mennyivel kell naponta n¨ ovelni az eladott u ok sz´ am´ at, hogy ¨d´ıt˝ a terv siker¨ ulj¨ on? (3 pont) A bolth´ al´ ozat n¨ oveked´ese miatt sz¨ uks´egess´e v´ alt egy u ´j rakt´ ar´ep¨ ulet megv´ as´ arl´ asa, amelyet a tulajdonos hitel felv´etel´evel k´ıv´ an megval´ os´ıtani. A bankt´ ol kapott 30 milli´ o forint k¨ olcs¨ ont 5 ´even ´ at 5 egyenl˝ o r´eszletben kell visszafizetnie. A 2021. olcs¨ onre a bank minden ´ev v´eg´en 3,5%-os kamatot sz´ am´ıt fel. janu´ arj´ aban felvett k¨ A t¨ orleszt´est a tulajdonos a k¨ ovetkez˝ o ´ev janu´ arj´ aban kezdi meg. b) Mekkor´ ak az egyes t¨ orleszt˝ or´eszletek ezer forintra kerek´ıtve? (5 pont) A bolt gazdas´ agi vezet˝ oje azt tan´ acsolta, hogy ´evente maximum 4 milli´ o forintot k¨ olts¨ on a tulajdonos az u ´j rakt´ ar´ep¨ uletre felvett k¨ olcs¨ on t¨ orleszt´es´ere, az el˝ oz˝ ovel megegyez˝ o banki kamat mellett. c) H´ any ´ev alatt tudja ´ıgy a tulajdonos visszafizetni a k¨ olcs¨ ont? (5 pont) 266

K¨ oz´ episkolai Matematikai ´ es Fizikai Lapok, 2022/5

Megold´as. a) A sz´ amtani sorozat els˝ o tagja 150; a tagok sz´ama 29; az els˝ o 29 tag ¨ osszege 14 500. Behelyettes´ıtve a sz´ amtani sorozat ¨osszegk´eplet´ebe: 14 500 =

29 · (300 + 28d) . 2

Az egyenlet megold´ asa ut´ an d = 25, teh´at legal´abb 25 darabbal kell n¨ovelni a naponta eladott mennyis´eget. b) A bankt´ ol felvett hitel: a0 = 30 000 000 Ft; az ´eves kamat: p = 3,5%, ez´ert q = 1,035; a t¨ orleszt˝ or´eszlet: x Ft. 2026 janu´arj´aban, az 5. t¨ orleszt˝ or´eszlet kifizet´ese ut´ an 0 Ft-ra cs¨okken a tartoz´ as: a0 · q 5 − x · q 4 − x · q 3 − x · q 2 − x · q − x = 0. Rendezz¨ uk az egyenletet: a0 · q 5 = x · q 4 + x · q 3 + x · q 2 + x · q + x. A jobb oldalon kiemelhet¨ unk x-et, ´ıgy: x=

q4

+

a0 · q 5 = 6 644 441,2. + q2 + q + 1

q3

6 644 000 Ft-ot kell ´evente t¨ orleszteni. c) Ha ´evente maxim´ alisan y = 4 000 000 Ft-ot tud t¨orleszteni, akkor az n-edik t¨ orleszt˝ or´eszlet kifizet´ese ut´ an 0 Ft-ra cs¨ okken a tartoz´ as. Ez´ert: n

a0 · q − y · q

n−1

−y·q

n−2

2

− . . . − y · q − y · q − y = 0.

Rendezz¨ uk az egyenletet, emelj¨ unk ki y-t, ´es haszn´ aljuk a m´ertani sorozat els˝ o n elem´enek ¨ osszeg´ere vonatkoz´ o¨ osszef¨ ugg´est: a0 · q n = y ·

a) Mennyi lehet n legkisebb ´es legnagyobb ´ert´eke, ha az A ∩ B ∩ C halmaz elemsz´ ama 3? (3 pont) b) H´ any olyan sz´ am tal´ alhat´ o 1 ´es 200 k¨ oz¨ ott, amelyek A, B ´es C halmazok k¨ oz¨ ul pontosan kett˝ onek elemei? (5 pont) c) Hat´ arozzuk meg a k¨ ovetkez˝ o ´ all´ıt´ asok logikai ´ert´ek´et. V´ alaszainkat minden esetben indokoljuk. o sz´ amjegye 5 vagy 6. (i) A B \ (A ∪ C) halmaz elemeinek utols´ (ii) B \ (A ∪ C) = B \ A. (iii) A C \ (A ∪ B) halmaz elemeinek szorzata 10 darab 0-ra v´egz˝ odik n = 200 eset´en. (6 pont) Megold´as. a) A ∩ B ∩ C a 60-nal oszthat´ o sz´ amok halmaza, ennek elemsz´ama 3, azaz A ∩ B ∩ C = {60; 120; 180}. Ez´ert n legkisebb ´ert´eke 180; legnagyobb ´ert´eke 239 lehet. b) Az A ∩ B halmaz a 30-cal oszthat´o 200-n´ al nem nagyobb sz´amok; elemsz´ ama 6. al nem nagyobb sz´amok; elemsz´ ama 3. Az A ∩ C halmaz a 60-nal oszthat´o 200-n´ Az B ∩ C halmaz a 60-nal oszthat´o 200-n´ al nem nagyobb sz´amok; elemsz´ ama 3.

Az A ∩ B ∩ C halmaz a 60-nal oszthat´o 200-n´ al nem nagyobb sz´amok; elemsz´ ama 3. Pontosan k´et halmaznak elemei: |A ∩ B| + |B ∩ C| + |A ∩ C| − 3 · |A ∩ B ∩ C| = 3. Megjegyz´es. Ezek az elemek: 30; 90; 150.

c) (i) Igaz, mert azok a sz´ amok, amelyek 15-tel oszthat´ oak, de 6-tal ´es 20-szal nem, azok nem p´arosak, de oszthat´ oak 5-tel, ez´ert 5-re v´egz˝ odnek. (ii) Igaz, Venn-diagramon ´abr´ azolva (B ∩ C) \ A = ∅.

qn − 1 . q−1

Behelyettes´ıtve a megadott ´ert´ekeket: 30 000 000 · 1,035n = 4 000 000 ·

3. Legyen az U alaphalmaz az els˝ o n pozit´ıv eg´esz sz´ am, amelynek h´ arom r´eszhalmaza: A : 6-tal oszthat´ o sz´ amok, B : 15 t¨ obbsz¨ or¨ osei, C : olyan eg´eszek, amelyeknek oszt´ oja a 20.

(iii) Hamis, C \ (A ∪ B) = {20; 40; 80; 100; 140; 160; 200}, az elemek szorzata 9 db null´ara v´egz˝ odik.

1,035n − 1 , 1,035 − 1

4. Tekints¨ uk az ABCD paralelogramm´ at, amelynek AB oldala 16 cm-rel hosszabb, mint az AD oldala, valamint hegyessz¨ oge DAB = α. a) Igazoljuk, hogy a paralelogramma sz¨ ogfelez˝ oi a ´ltal meghat´ arozott P QRS n´egysz¨ og t´eglalap. (3 pont)

1,05 · 1,035n = 4 · 1,035n − 4, 80 = 1,035n , 59 n = log1,035

80 ≈ 8,85. 59

b) Igazoljuk, hogy a P QRS t´eglalap ter¨ ulete cm2 -ben m´erve T = 128 · sin α. (7 pont)

Teh´at 9 ´ev alatt tudja a tulajdonos visszafizetni a k¨olcs¨ont. K¨ oz´ episkolai Matematikai ´ es Fizikai Lapok, 2022/5

267

268

K¨ oz´ episkolai Matematikai ´ es Fizikai Lapok, 2022/5

c) Mekkora a P QRS t´eglalap a ´tl´ oj´ anak hossza?

(3 pont)

Megold´as. a) Az ASB h´aromsz¨ og A cs´ ucs´ an´al l´ev˝ o sz¨oge a sz¨ogfelez´es miatt α2 ; B cs´ ucs´ an´ al l´ev˝ o sz¨ oge: α 180◦ − α = 90◦ − . 2 2 A h´ aromsz¨ og S cs´ ucs´ an´ al l´ev˝ o sz¨ oge ´ıgy α  α  180◦ − + 90◦ − = 90◦ . 2 2

II. r´esz 5. Almasz¨ uret ut´ an v´eletlenszer˝ uen kiv´ alasztottak 50 darab alm´ at, megm´ert´ek az ´ atm´er˝ oj¨ uket, ezt mutatja az al´ abbi oszlopdiagram.

Hasonl´oan igazolhat´ o, hogy a P , Q, R cs´ ucsokn´ al l´ev˝ o sz¨ogek szint´en der´eksz¨ogek. ´Igy a P QRS n´egysz¨ og t´eglalap. b) Legyen a paralelogramma AD oldala a, ´ıgy az AB oldal hossza a + 16. α 2 α AP = a · cos 2

P D = a · sin

α 2 α AS = (a + 16) · cos 2

a) Mennyi az alm´ ak ´ atm´er˝ oj´enek ´ atlaga ´es sz´ or´ asa?

BS = (a + 16) · sin

(4 pont)

A k¨ ovetkez˝ o ´evben u ´jfajta t´ apoldatot is haszn´ altak az almaf´ ak ¨ ont¨ oz´es´ere. Az u ´jabb almasz¨ uret ut´ an u ´jra v´ alasztottak 50 darabot, amelyeknek megm´ert´ek az ´ atm´er˝ oj´et. Az el˝ oz˝ o ´evi mint´ aval ¨ osszehasonl´ıtva azt tapasztalt´ ak, hogy a 8 cm-n´el kisebb gy¨ um¨ olcs¨ ok ´ atm´er˝ oje 24%-kal n˝ ott, ha m´eret¨ uket eg´esz cm-re kerek´ıtj¨ uk. b) Mekkora lesz az u ´j minta medi´ anja, illetve a minta medi´ ant´ ol val´ oa ´tlagos abszol´ ut elt´er´ese? (5 pont)

A t´eglalap oldalai: α α α − a · cos = 16 · cos , 2 2 2 α α α P Q = QD − P D = BS − P D = (a + 16) · sin − a · sin = 16 · sin . 2 2 2 P S = AS − AP = (a + 16) · cos

olcsb˝ ol almalevet pr´eselnek. A leszedett alm´ at megh´ aA sz¨ ureten szedett gy¨ um¨ mozz´ ak, kiszedik a magj´ at, ez´ altal 12%-ot vesz´ıt a t¨ omeg´eb˝ ol. A pr´esel´eskor 8 kg pucolt alm´ ab´ ol ´ atlagosan 5 liter levet k´esz´ıtenek. c) H´ any kg alm´ at szedtek a sz¨ uret alkalm´ aval, ha abb´ ol ¨ osszesen 3 000 liter almal´e k´esz¨ ult? (3 pont)

A t´eglalap ter¨ ulete: α α · cos . 2 2

Az alm´ at 200 Ft/kg egys´eg´ aron tudja eladni a gazda. Az almal´e ´ ar´ at u ´gy szeretn´e meghat´ arozni, hogy azzal 20%-kal t¨ obb bev´etele legyen.

Haszn´ aljuk a k´etszeres sz¨ ogekre vonatkoz´ o add´ıci´ os t´etelt:

d) Mennyibe ker¨ ulj¨ on 1 liter almal´e? (A v´ alaszt eg´esz forintra kerek´ıtve adjuk meg.) (4 pont)

T = P S · P Q = 162 · sin

T = 162 · sin

α α α α · cos = 128 · 2 · sin · cos = 128 · sin α. 2 2 2 2

Megold´as. a) Az alm´ak ´ atm´er˝ oj´et tekintve az a´tlag

Ezt kellett bizony´ıtanunk. x=

c) A P SQ h´ aromsz¨ ogben fel´ırhatjuk a Pitagorasz-t´etelt:  α 2  α 2 QS 2 = 16 · sin + 16 · cos , 2 2  α α QS 2 = 162 · sin2 + cos2 . 2 2

sz´ or´ asa: D(x) =

√ 2,1444 = 1,464 cm.

b) A r´egi 5 cm a´tm´er˝ oj˝ u alm´ab´ol a t´ apoldat hat´as´ ara 6 cm ´ atm´er˝ oj˝ u lett. A 6 cm ´atm´er˝ oj˝ u 7 cm-esre n˝ott, a 7 cm helyett 9 cm a´tm´er˝ oj˝ uek lettek az alm´ak. A t¨obbi alma m´erete nem v´ altozott. T´ abl´ azatba foglalva a t´ apoldat hat´as´ ara kapott u ´j minta adatait: ´tm´er˝ a o (cm) darab

Haszn´ alhatjuk a trigonometrikus Pitagorasz-t´etelt, ´ıgy a z´ ar´ ojelben szerepl˝o kifejez´es ´ert´eke pontosan 1. A t´eglalap a´tl´ oj´anak hossza teh´at 16 cm. K¨ oz´ episkolai Matematikai ´ es Fizikai Lapok, 2022/5

367 8 · 5 + 5 · 6 + . . . + 2 · 11 = = 7,34 cm, 50 50

269

270

6 8

7 5

8 16

9 19

10 –

11 2

K¨ oz´ episkolai Matematikai ´ es Fizikai Lapok, 2022/5

A medi´ ant az adatok nagys´ ag szerinti sorrendj´eben a 25. ´es 26. adat a´tlaga adja: 8 cm. A medi´ant´ ol val´ o´ atlagos abszol´ ut elt´er´es: 46 8 · |6 − 8| + 5 · |7 − 8| + . . . + 2 · |11 − 8| = = 0,92 cm. 50 50

hogy pontosan 7; 8; 9; . . . b´erletlemond´ as t¨ ort´enik. Ezek ¨ osszege 12 b´erletlemond´asig meghaladja a 0,65-¨ ot, ´ıgy igaz lesz Tam´ as ´ all´ıt´ asa. Az X val´ osz´ın˝ us´egi v´ altoz´ o jelentse a b´erletlemond´ asok sz´am´at:

c) x kg alm´ ab´ ol 0,88x kg h´ amozott alma lesz. Egyenes ar´ anyoss´ ag van a h´ amozott alma t¨ omege ´es a bel˝ ole k´esz¨ ul˝ o almal´e k¨ oz¨ ott. A k´et mennyis´eg h´ anyadosa 0,88x alland´ ´ o. 85 = 3000 , amelyb˝ ol x = 5454,54. Teh´ at 5454,54 kg alm´ ab´ ol lesz 3000 l almal´e. d) y kg alma elad´ asa eset´en 200y Ft bev´etel sz´ armazik. Enn´el szeretn´enk 20%kal t¨ obbet: 200y · 1,2 Ft bev´etel legyen az almal´e ´ar´ ab´ol. y kg alm´ ab´ol 0,88y · 58 liter almal´e k´esz¨ ul. Legyen az almal´e literenk´enti a´ra z Ft. ´Igy a bev´etel¨ unk: 0,88y · 5 · z. Fel´ırhat´o a

P (X = 8) =



 196 · 0,058 · 0,95188 = 0,1184, 8

P (X = 10) = 0,128 15, P (X = 11) = 0,114 05,

5 200y · 1,2 = 0,88y · · z 8

P (X = 12) = 0,0925.

egyenlet, amelyb˝ ol z = 436,36. Az almalevet 436 Ft-´ert kell eladni. 6. A sz´ınh´ az szervez´esi oszt´ aly´ an 196 b´erletet adtak el az aktu´ alis ´evadra. A b´erletes el˝ oad´ asok el˝ ott – a b´erlettulajdonosok elfoglalts´ aga miatt – ´ atlagosan 5% b´erletlemond´ as t¨ ort´enik. A sz´ınh´ az p´enzt´ ar´ aban az ilyen esetekre u ´gynevezett l´epcs˝ ojegyeket adnak el, amelyek nem helyre sz´ olnak, hanem a b´erletlemond´ as miatt meg¨ uresedett helyeket t¨ olthetik fel a n´ez˝ ok. A 12. ´evfolyam tanul´ oi 7 db l´epcs˝ ojegyet v´ as´ aroltak.

¨ Osszeg¨ uk: P (7  X  12) = 0,6783. c) A barna bor´ıt´ekok sz´ ama 4x darab, ezekb˝ ol 10% kedvezm´enyt tartalmaz: 4x · 0,5, 15% kedvezm´eny: 4x · 0,5. S´ arga sz´ın˝ u bor´ıt´ek: 5x darab, ezekb˝ ol 15% kedvezm´eny: 5x · 13 , 20% kedvezm´eny: 5x · 23 . Lila bor´ıt´ek: 5x darab ezek mindegyik´eben 15% kedvezm´eny tal´ alhat´ o. Annak val´ osz´ın˝ us´ege, hogy az ¨ osszes bor´ıt´ek k¨ oz¨ ul egy olyat tal´ al R´eka, amelyben 15% kedvezm´eny van: P (15%) =

(2 pont)

Tam´ as azt mondja: Legal´ abb 65% a val´ osz´ın˝ us´ege, hogy mindenkinek jut u o¨l˝ ” hely a sz´ınh´ azban.” b) Igazoljuk Tam´ as ´ all´ıt´ as´ at. (6 pont) A sz´ınh´ azban a sz´ek alj´ ara barna, s´ arga ´es lila sz´ın˝ u bor´ıt´ekokat ragasztottak 4 : 5 : 5 ar´ anyban, amelyekben v´ as´ arl´ asi kedvezm´enyre jogos´ıt´ o kuponok tal´ alhat´ oak. A barna bor´ıt´ekok fel´eben 10%-os, a t¨ obbiben 15%-os kupon tal´ alhat´ o. A s´ arga sz´ın˝ uek harmad´ aban 15%-os, a t¨ obbiben 20%-os kupon van. A lila bor´ıt´ekokba egys´egesen 15%-os kupont tettek. R´eka 15%-os kedvezm´enyt tal´ alt a saj´ at bor´ıt´ekj´ aban. c) Mekkora a val´ osz´ın˝ us´ege, hogy R´eka s´ arga sz´ın˝ u bor´ıt´ekot kapott? asa: Van olyan di´ak, akinek nem jut u ohely Megold´as. a) Az ´all´ıt´ as tagad´ ¨l˝ a sz´ınh´ azban. b) Ahhoz, hogy mindenkinek jusson u ohely a sz´ınh´azban, legal´ abb 7 ember¨l˝ nek le kell mondania a b´erletet. Binomi´alis eloszl´ ast haszn´ alhatunk 196 emberre, 0,05 lemond´asi val´ osz´ın˝ us´eg mellett. Kezdj¨ uk el kisz´ amolni annak a val´ osz´ın˝ us´eg´et, 271

4x · 0,5 + 5x · 13 + 5x 13 = . 4x + 5x + 5x 21

A megold´ as felt´eteles val´osz´ın˝ us´eg kisz´am´ıt´ as´ aval ad´odik: 5x · 13 P (s´ arga · 15%) 5 P (s´ arga | 15%) = = 14x = = 0,1923. P (15%) 26 13 21 Teh´at 0,1923 annak val´ osz´ın˝ us´ege, hogy ha R´eka 15%-os kupont tal´ alt a saj´ at bor´ıt´ekj´aban, akkor s´ arga sz´ın˝ u bor´ıt´ekot kapott.

(8 pont)

K¨ oz´ episkolai Matematikai ´ es Fizikai Lapok, 2022/5

 196 · 0,057 · 0,95189 = 0,095, 7

P (X = 9) = 0,1302,

8

Simon azt mondja: Mind a 7 di´ aknak jut u ohely a sz´ınh´ azban.” ¨l˝ ” a) Fogalmazzuk meg Simon a ´ll´ıt´ as´ anak tagad´ as´ at.

P (X = 7) =



7. a) Oldjuk meg a k¨ ovetkez˝ o egyenletet a val´ os sz´ amok halmaz´ an: 2 sin x = |x − 1| + |x − 3|.

(7 pont)

b) Mekkora az f (x) = 2 sin x, g(x) = |x − 1| + |x − 3| g¨ orb´ek ´es az y tengely altal hat´ ´ arolt korl´ atos s´ıkidom ter¨ ulet´enek pontos ´ert´eke? (9 pont) 272

K¨ oz´ episkolai Matematikai ´ es Fizikai Lapok, 2022/5

Megold´as. a) Az egyenlet megold´ as´ ahoz el˝osz¨ or tekints¨ uk az abszol´ ut ´ert´ekek defin´ıci´ oj´at: ⎧ ⎨x − 1, ha x  1, |x − 1| = ⎩−(x − 1), ha x < 1; |x − 3| =

⎧ ⎨x − 3,

⎩−(x − 3),

π

T =

2, ⎪ ⎪ ⎩ 2x − 4,



 g(x) − f (x) dx =

1 0

π

(−2x + 4 − 2 sin x) dx +

1

2 1

(2 − 2 sin x) dx =

π

= [−x2 + 4x + 2 cos x]0 + [2x + 2 cos x]12 =  π − (2 + 2 cos 1) = π − 1. = (−1 + 4 + 2 cos 1) − (0 + 2 cos 0) + π + 2 cos 2

ha x < 3.

⎧ ⎪ ⎪ ⎨−2x + 4,

2 0

ha x  3,

Ezeket felhaszn´ alva a jobb oldalon szerepl˝ o f¨ uggv´eny:

g(x) = |x − 1| + |x − 3| =

Ter¨ ulet´et integr´ alsz´ am´ıt´ as seg´ıts´eg´evel sz´ am´ıthatjuk ki:

8. A 20 cm oldalhossz´ us´ ag´ u n´egyzet alak´ u feh´er lapb´ ol a kezd˝ o origami szakk¨ or¨ on tetra´edert hajtogatnak a gyerekek u ´gy, hogy az ´ abr´ an jel¨ olt AC, AB ´es BC szakaszok ment´en hajtj´ ak meg a lapot.

ha x < 1, ha 1  x < 3, ha x  3.

´ azoljuk k¨ Abr´ oz¨ os koordin´ ata-rendszerben a jobb ´es bal oldalon szerepl˝o f¨ uggv´enyeket:

a) Mekkora a tetra´eder legnagyobb ´es legkisebb ter¨ ulet˝ u lapj´ anak hajl´ assz¨ oge a hajtogat´ as ut´ an? (7 pont)

A k´et f¨ uggv´eny ´ert´ekk´eszlet´enek egyetlen k¨ oz¨ os eleme a 2. Ezt az f (x) = 2 sin x ol az eredeti egyenletnek csak f¨ uggv´eny x = π2 + k · 2π helyeken veszi fel. Ezekb˝ π asa a f¨ uggv´eny-grafikonok alapj´an. Ellen˝ orz´essel meggy˝ oz˝ odheaz x = 2 a megold´ t¨ unk a megold´ as helyess´eg´er˝ ol.

b) Mekkora a tetra´eder t´erfogata? (3 pont) Az elk´esz´ıtett tetra´edereket a gyerekek meger˝ os´ıtik az ´elek hossz´ ara ragasztott sz´ınes sz´ıv´ osz´ alak seg´ıts´eg´evel. Majd a legnagyobb ter¨ ulet˝ u lapj´ ara ´ all´ıtva leteszik egy asztalra egyform´ an igaz´ıtva a testeket. c) H´ any k¨ ul¨ onb¨ oz˝ o meger˝ os´ıtett tetra´eder k´esz´ıthet˝ o piros, z¨ old, s´ arga ´es k´ek sz´ın˝ u sz´ıv´ osz´ allal, ha az egy cs´ ucsban tal´ alkoz´ o sz´ıv´ osz´ alak nem lehetnek egyforma sz´ın˝ uek? (6 pont) Megold´as. a) A n´egyzet oldala 20 cm, a B ´es C pontok az adott oldalak felez˝opontjai. A tetra´eder lapjainak ter¨ ulete:

b) A keresett korl´ atos s´ıkidomot az ´ abra mutatja: TAF B = TAEC = TBDC =

20 · 10 = 100 cm2 , 2

10 · 10 = 50 cm2 , 2

ez a legkisebb,

TABC = 202 − 2 · 100 − 50 = 150 cm2 ,

K¨ oz´ episkolai Matematikai ´ es Fizikai Lapok, 2022/5

273

274

ez a legnagyobb.

K¨ oz´ episkolai Matematikai ´ es Fizikai Lapok, 2022/5

A H pont legyen a BC szakasz felez˝opontja. A tetra´eder ADH s´ıkmetszet´eben kapjuk a legnagyobb ´es legkisebb ter¨ ulet˝ u lapok hajl´assz¨og´et. Ebben a h´ aromsz¨ogben:

Ebben a konkr´et esetben 8 lehet˝os´eg van. Figyelembe v´eve, hogy az AD ´elre van os´eg, majd az AB ´elre ezut´ an 3 lehet˝os´eg, ´ıgy ¨ osszesen 4 · 3 · 8 = 96 ¨osszesen 4 lehet˝ lehet˝ os´eg van.

AD = 20 cm, √ DH = 5 2 cm, √ √ √ AH = 20 2 − 5 2 = 15 2 cm.

Teh´ at 96 k¨ ul¨ onb¨ oz˝ o meger˝ os´ıtett tetra´eder k´esz´ıthet˝ o.

Az ADH h´ aromsz¨ og H cs´ ucs´ an´al l´ev˝ o sz¨ oge adja a v´alaszt. Fel´ırva a koszinuszt´etelt: √ √  √ 2  √ 2 202 = 15 2 + 5 2 − 2 · 15 2 · 5 2 · cos α,

cos α =

1 , 3

α = 70,53◦ .

b) A tetra´eder magass´ aga az ADH h´ aromsz¨ og AH oldal´ ahoz tartoz´o magass´ aga (M ). M sin α = √ , 5 2

M=

1 2 9. A koordin´ atarendszerben megrajzoltuk a p : y = 12 x − 13 x + 10 egyenlet˝ u 3 parabol´ at. a) ´ Irjuk fel a parabola ´es az y-tengely metsz´espontj´ aban a parabol´ ahoz h´ uzott ´erint˝ o egyenlet´et. (6 pont) Fizika tanulm´ anyok alapj´ an ismert, hogy a parabolat¨ uk¨ or a tengely´evel p´ arhuzamos f´enysugarakat a f´ okuszponton kereszt¨ ul veri vissza.

b) A parabola belsej´eben f´enysug´ ar ´erkezik az y-tengely ment´en. Mi a visszaver˝ od˝ o f´enysug´ ar egyenes´enek egyenlete? (7 pont) Fizik´ aban bees´esi mer˝ olegesnek h´ıvj´ ak a bees´esi pontban a parabola ´erint˝ oj´ere all´ıtott mer˝ ´ olegest.

20 cm. 3

c) Mekkora az y-tengely ment´en bees˝ o f´enysug´ ar ´es a bees´esi mer˝ oleges ´ altal bez´ art sz¨ og? (3 pont)

A tetra´eder t´erfogata: V =

N´ezz¨ uk azokat a konkr´et eseteket, amikor az AD ´el piros, az AB ekkor 3-f´ele lehet m´eg (z¨old, s´ arga vagy k´ek). Ha az AB ´el z¨ old, akkor a t¨ obbi ´el lehets´eges sz´ınei a 2. ´ abr´ an l´ athat´ ok.

at´ ait meghaMegold´as. a) A parabola ´es az y-tengely k¨ oz¨ os pontj´ anak koordin´ t´arozhatjuk, felhaszn´ alva, hogy az y-tengely pontjainak abszcissz´aja x = 0. A met. ´Igy M = (0; 10 . sz´espont ordin´ at´ aja behelyettes´ıt´es ut´ an y = 10 3 3 )

150 · 20 TABC · M 3 = 333,3 cm3 . = 3 3

A metsz´espontba h´ uzott ´erint˝o meredeks´ege az

Megjegyz´es. Az ADH h´ aromsz¨ og D cs´ ucs´ an´ al der´eksz¨ og van, ´ıgy a t´erfogat a BCD h´ aromsz¨ ogre mint alapter¨ uletre ´es az AD oldalra mint magass´ agra t´ amaszkodva is kisz´ am´ıthat´ o.

c) A tetra´eder ´elei 4-f´ele sz´ınnel sz´ınezhet˝ ok: piros, z¨old, s´ arga ´es k´ek. Vegy¨ uk sorra az ´eleket: AD, AB, AC, CD, BC ´es v´eg¨ ul BD (1. ´ abra).

f (x) =

10 1 1 · x2 − x + 12 3 3

f¨ uggv´eny deriv´ altj´ anak x0 = 0 pontban sz´ am´ıtott helyettes´ıt´esi ´ert´eke: f  (x) =

1 1 x− , 6 3

m = f  (0) =

1 1 1 ·0− =− . 6 3 3

Az ´erint˝o egyenlete: y − 10 = − 13 · x. Rendezve az egyenletet: x + 3y = 10. 3 b) A parabola tengelypontj´ anak koordin´at´ ait a parabola egyenlet´enek teljes n´egyzett´e alak´ıt´ asa ut´ an tudjuk megadni. y=

1. a ´bra K¨ oz´ episkolai Matematikai ´ es Fizikai Lapok, 2022/5

1 2 · (x − 2) + 3, 12

y−3=

1 2 · (x − 2) , 12

´ıgy a tengelypont koordin´ at´ ai: T (2; 3), a parabola param´etere p = 6. Megadhat´ o 6 a parabola f´okuszpontja: F (2; 3 + 2 ) = (2; 6). A visszaver˝ od˝ o f´enysug´ ar egyenlet´enek fel´ır´ as´ ahoz haszn´aljuk, hogy a f´enysug´ ar −−→ 8 ◦ egyenes´enek ir´ anyvektora az M F = (2; 3 ). A vektort 90 -kal elforgatva a f´enysug´ ar

2. a ´bra

275

276

K¨ oz´ episkolai Matematikai ´ es Fizikai Lapok, 2022/5

norm´ alvektor´ at kapjuk: n = ( 83 ; −2). Haszn´ aljuk ezt a norm´ alvektort ´es a parabola f´okuszpontj´ at. A visszavert f´enysug´ ar egyenlete:

A k˝ o A pap´ır A oll´ o

16 8 x − 2y = − 12. 3 3

c) A bees´esi mer˝oleges meredeks´eg´enek ´es az a) r´eszben szerepl˝ o ´erint˝o meredeks´eg´enek szorzata −1. m b · m = mb · −

1 3



= −1,

f (a, b) =

mb = 3.

 1  1  1 f (a − 1, b) + 1 + f (a, b − 1) + 1 + f (a, b) + 1 , 3 3 3

f (a, b) =

J´ocsik Csilla Gy˝or

 3 1 + f (a − 1, b) + f (a, b − 1) . 2 2

Tov´abb´ a nyilv´ an f (a, b) = f (b, a), ´es a = 0 vagy b = 0 eset´en f (a, b) = 0. Ez m´ar el´eg ahhoz, hogy kisz´ amoljuk f (a, b)-t minden a, b-re. Azonban a = b = 3-ra ink´abb c´elszer˝ u konkr´et sz´ amokkal v´egigsz´ amolni, mint meghat´ arozni f explicit alakj´ at. V´agjunk is bele: f (1, 1) =

Matematika feladatok megold´asa

 3 3 1 + f (0, 1) + f (1, 0) = , 2 2 2

  3 9 = , 2 4   1 9 9 15 + , = 2 4 4 4   1 9 21 , = 2 4 8   1 21 15 75 + , = 2 8 4 16   1 75 75 99 + . = 2 16 16 16

 3 1 3 1 f (2, 1) = + f (1, 1) + f (2, 0) = + 2 2 2 2 f (2, 2) =

B. 5164. K´et j´ at´ekos 3 gy˝ ozelemig tart´ o k˝ o-pap´ır-oll´ o p´ arbajt j´ atszik. Tegy¨ uk fel, hogy mindketten minden menetben v´eletlenszer˝ uen (egym´ ast´ ol ´es a kor´ abbi alasztj´ ak ki, hogy mit mutatnak. Adjuk mutat´ asokt´ ol f¨ uggetlen¨ ul), 13 : 13 : 13 es´ellyel v´ meg a menetek sz´ am´ anak v´ arhat´ o ´ert´ek´et.

f (3, 1) =

(5 pont)

f (3, 2) =

I. megold´as. Legyen f (a, b) a menetek sz´ am´anak v´ arhat´ o ´ert´eke, ha a gy˝ozelemhez az els˝o j´at´ekosnak (akit A-val jel¨ ol¨ unk a k´es˝obbiekben) a, a m´ asodiknak (˝ ot pedig B-vel) b menetet kell nyernie (ez az (a, b)-vel jel¨olt p´arbaj). Amikor az (a, b) p´ arbajban az els˝ o menet lezajlott, h´arom dolog lehets´eges. Ha az els˝o j´at´ekos nyert, akkor neki m´ar csak a − 1 menetet kell nyernie, vagyis az (a − 1, b) p´ arbaj alakult ki. Ha a m´ asodik nyert, akkor (a, b − 1) p´arbaj alakult ki. Ha pedig d¨ ontetlen, akkor tov´abbra is (a, b) p´ arbajr´ ol van sz´ o. Az al´abbi t´ abl´ azatb´ ol ul 1/3 es´ellyel alakul ki. leolvashat´ o, hogy minden eset a fenti h´arom k¨ oz¨

K¨ oz´ episkolai Matematikai ´ es Fizikai Lapok, 2022/5

B oll´ o A nyert B nyert =

 2 1 f (a, b) = 1 + f (a − 1, b) + f (a, b − 1) , 3 3

A bees´esi mer˝oleges ir´ anysz¨ oge: tg α = = 3, α = 71,57◦ . Az y-tengely ment´en bees˝ o f´enysug´ ar ´es a bees´esi mer˝oleges a´ltal bez´ art sz¨ og a fent sz´ am´ıtott sz¨ og p´ otsz¨ oge, ´ıgy β = 90◦ − α = 18,43◦ .

B pap´ır B nyert = A nyert

Ebb˝ ol nyerhet¨ unk egyenletet f (a, b)-re. Ha A nyert, akkor a h´ atral´ev˝ o menetek v´arhat´ o sz´ama f (a − 1, b), ha B nyert, akkor f (a, b − 1). Ha d¨ ontetlen alakult ki, akkor a h´ atral´ev˝ o menetek v´arhat´ o sz´ ama f (a, b). Minden esetben egy menet m´ar lement, ´ıgy 1-et kell adni a fenti ´ert´ekekhez. Mivel a v´ arhat´ o ´ert´ek addit´ıv, ebb˝ ol

Rendezve az egyenletet: 4x − 3y = −10.



B k˝ o = A nyert B nyert

277

f (3, 3) =

 3 3 1 + f (1, 2) + f (2, 1) = + 2 2 2

 3 3 1 + f (2, 1) + f (3, 0) = + 2 2 2  3 3 1 + f (3, 1) + f (2, 2) = + 2 2 2  3 3 1 + f (2, 3) + f (3, 2) = + 2 2 2

Lovas M´ arton (Budapest, B´ek´ asmegyeri Veres P´eter Gimn., 10. ´evf.) II. megold´as. Osszuk fel a p´ arbajt meccsekre. Egy meccs addig tart, ameddig valamelyik j´at´ekos meg nem nyer egy menetet. Teh´ at egy meccs egy vagy t¨ obb ´ menetb˝ol ´all, melyek k¨ oz¨ ul az utols´ o kiv´etel´evel mindegyik menet d¨ ontetlen. Ertelemszer˝ uen az nyeri a p´ arbajt, aki el˝ obb nyer meg h´ arom meccset. 278

K¨ oz´ episkolai Matematikai ´ es Fizikai Lapok, 2022/5

Minden menet 13 es´ellyel lesz d¨ ontetlen, f¨ uggetlen¨ ul az el˝oz˝ o menett˝ ol. ´Igy n−1 annak a val´ osz´ın˝ us´ege, hogy egy meccs pontosan n menetb˝ol ´all, mn = ( 13 ) · 23 . Teh´ at az egy meccsen bel¨ uli menetek sz´am´anak v´ arhat´ o ´ert´eke ´ıgy sz´ am´ıthat´o ki:  n−1 ∞ ∞   1 2 M= = = nmn = n 3 3 n=1 n=1 =



1 1+2· +3 3

  2  3 1 1 9 2 3 2 +4 + ... · = · = . 3 3 3 4 3 2

(Val´ osz´ın˝ us´egsz´ am´ıt´ asban k´epzett megold´ ok azt is mondhatj´ak erre, hogy az egy meccsen bel¨ uli menetek sz´ama geometriai eloszl´ as´ u, p = 23 param´eterrel, teh´ at v´ ar1 3 hat´ o ´ert´eke p = 2 ). Mivel a j´at´ek szimmetrikus, ´ıgy mindegyik meccset 12 es´ellyel nyeri A j´at´ekos at´ekos. A k¨ ul¨ onb¨ oz˝ o meccsek eredm´enyei pedig f¨ uggetlenek egy´es es´ellyel B j´ m´ ast´ ol. Ezt felhaszn´ alva hat´ arozzuk meg a p´ arbaj eld¨ont´es´ehez sz¨ uks´eges meccsek sz´ am´anak eloszl´as´at.

• E(X1 ) = E(X2 ) = E(X3 ) = M , hiszen az els˝o h´ arom meccs biztosan megval´osul. • E(X4 ) = M (1 − p3 ), hiszen X4 ´ert´eke p3 es´ellyel 0, m´ıg 1 − p3 es´ellyel lesz 4. meccs, ´ıgy ha n = 1, akkor  n−1 1 2 P (X4 = n) = (1 − p3 ) · . 3 3 • E(X5 ) = M p5 , hiszen X5 ´ert´eke p3 + p4 es´ellyel 0, m´ıg p5 es´ellyel megval´ osul ez a meccs, ´ıgy  n−1 1 2 · . P (X5 = n) = p5 3 3 Teh´at a feladat k´erd´es´ere a v´alasz   99 3 3 3 3 33 M (3 + (1 − p3 ) + p5 ) = = . 1+1+1+ + = · 2 4 8 2 8 16

1 2

• Akkor el´eg 3 meccs, ha az els˝ o h´ arom meccset ugyanaz nyeri. Ennek es´elye  3 1 1 = . p3 = 2 · 2 4 • Akkor d˝ ol el a 4. meccs v´eg´en a p´arbaj, ha az els˝ o n´egy menet gy˝ ozteseinek sorozata a k¨ ovetkez˝ o 6 sorozat valamelyike: AABA, ABAA, BAAA (ilyenkor A nyer), ABBB, BABB, BBAB (ilyenkor B nyer). Egy-egy ilyen sorozat 4 es´elye ( 12 ) , teh´ at annak az es´elye, hogy 4 meccsb˝ ol ´all a p´ arbaj:

 4 1 3 = . p4 = 6 · 2 8 • Legk´es˝ obb az 5. meccs v´eg´ere el kell d˝ olj¨ on a p´arbaj (a skatulyaelv alapj´an az 5 meccsb˝ol legal´abb 3-at ugyanannak az embernek kell nyerni), ´ıgy annak az es´elye, hogy 5 meccsb˝ol ´ all a p´ arbaj: p5 = 1 − p 3 − p 4 =

3 . 8

Legyen most Xi az a val´osz´ın˝ us´egi v´ altoz´ o, amelyik megadja az i. meccsen bel¨ uli menetek sz´am´at. Ha a 4., illetve 5. meccs nem val´ osul meg, akkor X4 = 0 (illetve X5 = 0). ´Igy a teljes meccs meneteinek sz´ ama ´eppen X1 + X2 + X3 + X4 + X5 , ennek v´ arhat´ o ´ert´eke (felhaszn´ alva, hogy a v´arhat´ o ´ert´ek addit´ıv): E(X1 ) + E(X2 ) + E(X3 ) + E(X4 ) + E(X5 ). Az egyes E(Xi ) ´ert´ekeket egyenk´ent meg tudjuk mondani: K¨ oz´ episkolai Matematikai ´ es Fizikai Lapok, 2022/5

279

Megjegyz´esek. 1. Ha Y -nal jel¨ olj¨ uk a meccsek sz´ am´ at jel¨ ol˝ o val´ osz´ın˝ us´egi v´ altoz´ ot, akkor 3 3 33 1 , E(Y ) = 3 · + 4 · + 5 · = 4 8 8 8 azaz ´eppen teljes¨ ul, hogy: 99 E(X1 ) · E(Y ) = . 16 ´Igy is kisz´ am´ıthat´ o lenne a feladat v´egeredm´enye? uggetlen, teljes¨ ul E(X1 ) · E(Y ) = E(X1 Y ), de X1 Y ´ altal´ aban nem Mivel X1 ´es Y f¨ egyezik meg a menetek sz´ am´ aval (b´ ar n´emi k¨ oze van hozz´ a). Teh´ at erre nem tudunk hivatkozni. Val´ oj´ aban ez a kisz´ am´ıt´ asi m´ od egy m´ely ´es ´ altal´ anos o ugg´es, az u ´gynevezett ¨sszef¨ Wald-azonoss´ ag (ld. https://en.wikipedia.org/wiki/Wald’s_equation) egy egyszer˝ u speci´ alis esete. ´ Az azonoss´ ag feltal´ al´ oj´ ar´ ol, Wald Abrah´ amr´ ol ´erdemes elolvasni ezt a t¨ ort´enetet is: https://ematlap.hu/konyvespolc-2017-03/443-hogy-ne-tevedjunk-wald-abrahames-a-hianyzo-lovedeknyomok. 2. A leggyakoribb hiba a m´ asodik (honlapon is k¨ oz¨ olt) megold´ as v´egi megjegyz´esnek megfelel˝ o hiba volt. 3. Emellett t¨ obb olyan versenyz˝ o is volt, aki nem volt tiszt´ aban a v´ arhat´ o ´ert´ek fogalm´ aval, ´es azt hitte, hogy a feladat azt k´erdezi, hogy melyik a legnagyobb val´ osz´ın˝ us´eg˝ u (eg´esz) menetsz´ am. 4. A feladat els˝ o r´esz´et sokan a honlapihoz hasonl´ oan oldott´ ak meg, de a m´ asodik as, mint r´eszben gyakoribb volt a megjegyz´esk´ent ´ırt hib´ as indokl´ ast alkalmaz´ o megold´ az ott le´ırt helyes megold´ as. T¨ obben is fel´ırt´ ak az egyes menetsz´ amokhoz tartoz´ o val´ osz´ın˝ us´egeket a menetsz´ amok n f¨ uggv´eny´eben, ´es ezt helyettes´ıtett´ek be a v´ arhat´ o ´ert´ek megfelel˝ o k´eplet´ebe. ´Igy egy igen bonyolult ¨ osszeghez jutottak, melynek kisz´ am´ıt´ as´ aval hossz´ u oldalakon ´ at vesz˝ odtek. Szerencs´ere enn´el t¨ obben alkalmazt´ ak a Markov-l´ ancokkal t¨ ort´en˝ o megold´ ast, mely seg´ıts´eg´evel sokkal bar´ ats´ agosabb sz´ amol´ as ut´ an k¨ onnyed´en eljutottak a helyes eredm´enyhez.

280

K¨ oz´ episkolai Matematikai ´ es Fizikai Lapok, 2022/5

54 dolgozat ´erkezett. 5 pontot kapott 25 versenyz˝ o: Beinschroth Ninett, Csizmadia Mikl´ os, Diaconescu Tashi, Duchon M´ arton, Fekete Rich´ ard, Han Ziying, Heged˝ us D´ aniel, Kolesz´ ar Domonkos, Kov´ acs Alex, K¨ ok´enyesi M´ ark P´eter, Lenkey Gy¨ ongyv´er, Lovas ´ M´ arton, M´ acsai D´ aniel, Metzger Abris Andr´ as, Moln´ ar-Szab´ o Vilmos, Nagy Levente, N´emeth M´ arton, Simon L´ aszl´ o Bence, Szanyi Attila, Sztrany´ ak Gabriella, Terj´ek Andr´ as J´ ozsef, Varga Boldizs´ ar, Velich N´ ora, Wiener Anna, Z¨ ombik Barnab´ as. 4 pontos 10, 3 pontos 3, 2 pontos 4, 1 pontos 6, 0 pontos 5 dolgozat.

B. 5178. Legyen x pozit´ıv val´ os sz´ am. Mutassuk meg, hogy    √ √ √ √ 3 8 . 6x + 9 + 16x + 64  x+ √ x+ √ x x (4 pont)

ami ´eppen a bizony´ıtand´ o.

´ Isk., 8. ´evf.) Varga Boldizs´ ar (Ver˝ oce, G´eza Fejedelem Reform´atus Alt. II. megold´as. Mivel x pozit´ıv val´os sz´am, ez´ert mindent ´ertelmez¨ unk, ´es mindk´et oldal pozit´ıv. Bontsuk fel a z´ ar´ ojelet az egyenl˝ otlens´eg jobb oldal´ an: √ √ 24 + 11. 6x + 9 + 16x + 64  x + x

Javasolta: Szoldatics J´ ozsef (Budapest)

I. megold´as. El˝ osz¨ or is jegyezz¨ uk meg, hogy egyenl˝otlens´eg¨ unk minden pozit´ıv val´os sz´amra ´ertelmes (a negat´ıvokra nem, de a feladat ezt kiz´arta). Ezut´ an szorozzuk meg x-szel az egyenl˝ otlens´eg mindk´et oldal´ at, a kapott egyenl˝ otlens´eg ekvivalens√ lesz az eredetivel, ´ e s mivel x pozit´ ıv, nem fordul meg  √ ´ uk, a rel´aci´ ojel. Ezt kapjuk: x 6x + 9 + 16x + 64  (x + 3)(x + 8). Eszrevehetj¨ hogy a bal oldalon a gy¨ okjel alatti tagok fel´ırhat´ ok k´et sz´ am n´egyzet´enek k¨ ul¨onbs´e2 2 altal a bizony´ıtand´ o gek´ent: 6x + 9 = (x + 3) − x2 , m´ıg 16x + 64 = (x + 8) − x2 , ez´ egyenl˝ otlens´eg ekvivalens az    2 2 x (x + 3) − x2 + (x + 8) − x2  (x + 3)(x + 8) egyenl˝ otlens´eggel.

Vegy¨ unk fel egy olyan h´ aromsz¨ oget, amelynek k´et oldala x + 3 ´es x + 8, a harmadikhoz tartoz´ o magass´ ag pedig x. Meg kell jegyezz¨ uk, hogy ilyen h´aromsz¨ og mindig l´etezik, ugyanis megkaphat´ o k´et olyan der´eksz¨ og˝ u h´ aromsz¨ og uni´ojak´ent, amelyek egyik befog´ oja x, ´ atfog´ oik pedig x + 3 ´es x + 8, m´arpedig der´eksz¨og˝ u h´aromsz¨oget ak´ armilyen 1-n´el kisebb befog´o-´atfog´ o ar´ anyb´ol lehet szerkeszteni. Tov´abb´ a az x-re vonatkoz´ o pozitivit´ast haszn´ alva az is egyszer˝ uen l´ athat´ o, hogy ezek a hosszok pozit´ıvak lesznek. El˝osz¨ or sz´amoljuk ki a harmadik oldalt. Mivel a magass´ ag k´et der´eksz¨og˝ u h´ aromsz¨ ogre bontja a nagy h´ aromsz¨ oget, haszn´ alhatjuk mindkett˝oben a Pitagoraszt´etelt, azt kapjuk, hogy a k´et m´ asik befog´o rendre   2 2 (x + 3) − x2 , illetve (x + 8) − x2 ,   2 2 azaz a harmadik oldal hossza (x + 3) − x2 + (x + 8) − x2 . Legyen az ezzel szemk¨ ozti sz¨ og α. Ennek seg´ıts´eg´evel ´ırjuk fel k´etf´elek´eppen a h´ aromsz¨og ter¨ ulet´et az ismert ter¨ uletk´epletekkel. R¨ ogt¨ on a k´etszeres ter¨ uletre t´erve:    2 2 2t = (x + 3) − x2 + (x + 8) − x2 ) x = (x + 3)(x + 8) sin α. K¨ oz´ episkolai Matematikai ´ es Fizikai Lapok, 2022/5

Mivel egy sz¨og szinusza legfeljebb 1, ez´ert    2 2 (x + 3) − x2 + (x + 8) − x2 x, (x + 3)(x + 8)  2t =

281

Mindk´et oldal pozit´ıv, ez´ert a n´egyzetre emel´es ekvivalens a´talak´ıt´ as: 

384x2 + 2112x + 2304 + 22x + 73  x2 + 22x + 169 +

528 576 + 2 , x x

 528 576 + 2 . 2 96x2 + 528x + 576  x2 + 96 + x x

A pozit´ıv x2 -tel szorozva:  2x2 96x2 + 528x + 576  x4 + 96x2 + 528x + 576.

Legyen a := 96x2 + 528x + 576, hogy egyszer˝ ubb legyen az egyenlet¨ unk: √ 2x2 a  x4 + a.

Mivel mindk´et oldal pozit´ıv, ism´et emelj¨ unk n´egyzetre: 4x4 a  x8 + 2ax4 + a2 , 0  x8 − 2ax4 + a2 , 2

0  (x4 − a) , ami nyilv´an teljes¨ ul. Mivel v´egig ekvivalens a´talak´ıt´ asokat v´egezt¨ unk, ez´ert az eredeti egyenl˝ otlens´eg is minden pozit´ıv x eset´en fenn´ all. Csizmadia Mikl´ os (Budapest XIV. Ker¨ uleti Szent Istv´an Gimn´azium, 11. ´evf.) 52 dolgozat ´erkezett. 4 pontos 46, 3 pontos 5, 0 pontos 1 dolgozat.

B. 5187. Az S = {1, 2, . . . , n} halmaz egy r´eszhalmaza primit´ıv, ha nincs benne k´et olyan elem, melyek k¨ oz¨ ul az egyik oszt´ oja a m´ asiknak. Mutassuk meg, hogy ha egy A ⊆ S primit´ıv halmazhoz nem lehet u ´gy hozz´ avenni u ´jabb S-beli elemet, hogy primit´ıv maradjon, akkor vagy A = {1}, vagy A m´erete legal´ abb annyi, mint n-ig a pr´ımek sz´ ama. (6 pont) 282

Javasolta: S´ andor Csaba (Budapest) K¨ oz´ episkolai Matematikai ´ es Fizikai Lapok, 2022/5

Megold´as. Ha 1 ∈ A, akkor A-nak nem lehet m´as eleme, vagyis teljes¨ ul a feladat felt´etele, a tov´ abbiakban felt´etelezz¨ uk, hogy 1 ∈ / A. Hajtsuk v´egre a k¨ ovetkez˝ o algoritmust: kezdetben a B halmaz legyen u ¨res, a C halmazban pedig legyenek benne a pr´ımek 1-t˝ ol n-ig. Ism´etelj¨ uk, ameddig lehet, a k¨ ovetkez˝ ot: ha l´etezik olyan p ∈ C ´es y ∈ A, hogy alkalmas j pozit´ıv any (nem felt´etlen¨ ul p´ aronk´ent k¨ ul¨onb¨oz˝ o) B-beli eg´esszel y = pj · k, ahol k n´eh´ sz´ am szorzata, akkor a p pr´ımet vegy¨ uk ki C-b˝ ol, ´es rakjuk a´t B-be. (Kezdetben teh´at rakjuk a´t B-be azokat a pr´ımeket, amelyeknek van hatv´ anya A-ban, majd az olyanokat, amelyeknek valamely hatv´ anya el˝o´all egy A-beli elem ´es n´eh´ any B-beli pr´ım szorzat´ anak h´ anyadosak´ent, stb.) Ha m´ar nem tudunk t¨obb ilyen l´ep´est v´egezni, a k¨ ovetkez˝ o esetek a´llhatnak fenn: – A C halmaz u ol n-ig}| – ´ıgy teljes¨ ul a feladat ¨res. Ekkor |A|  |{pr´ımek 1-t˝ felt´etele – mivel minden l´ep´es sor´ an olyan A-beli elemet v´alasztottunk ki, amelyet osszesen |{pr´ımek 1-t˝ ol n-ig}| l´ep´es t¨ort´ent). Tegy¨ uk fel ugyanis, kor´ abban nem (´es ¨ hogy az y ∈ A sz´ amot az i-edik ´es a j-edik l´ep´esben is kiv´ alasztottuk, ahol i < j. Jel¨ olje B1 ´es C1 , illetve B2 ´es C2 a B ´es C halmaz aktu´alis ´allapot´ at k¨ozvetlen¨ ul az i-edik, illetve a j-edik l´ep´es v´egrehajt´ asa el˝ ott. Indirekt feltev´es¨ unk szerint ekkor os alak egy´ertelm˝ us´ege miatt p2 oszt´ oja k1 y = pj11 · k1 ´es y = pj22 · k2 . A pr´ımt´enyez˝ at p2 a B1 ´es a C1 halmaznak is eleme, nek, ´ıgy p2 ∈ B1 , m´asr´eszt p2 ∈ C2 ⊆ C1 . Teh´ ami lehetetlen, hiszen a B ´es C halmazok az elj´ar´ as sor´ an v´egig diszjunktak. – C = {p}. Ekkor A-ban nincs p-vel oszthat´ o elem, teh´at m´eg hozz´ avehetj¨ uk a halmazhoz p-t, ´es ez ellentmond´ as. – Minden m´ as esetben vegy¨ uk a legkisebb p ∈ C elemet. Mivel A primit´ıv, van p-vel oszthat´o eleme; legyen z ∈ A olyan, hogy a p pr´ımt´enyez˝ o hatv´ anya z-ben maxim´alis az A elemei k¨ oz¨ ul. Tudjuk, hogy van olyan q ∈ C, q > p pr´ım, amelyre q | z, m´ ask¨ ul¨ onben m´eg egy l´ep´est elv´egezve p-t ´ atrakhattuk volna B-be. amot. Nyilv´ an s < z  n, ´es s-nek nincs oszt´oja A-ban, Tekints¨ uk az s := zq · p sz´ mivel az z-nek is oszt´ oja lenne, vagy nagyobb lenne benne a p kitev˝ oje, mint z-ben. Tov´abb´ a s-nek nincs t¨obbsz¨ or¨ ose sem A-ban, mert abban nagyobb lenne p kitev˝ oje, mint z-ben. Teh´ at s-et hozz´ a lehetne venni A-hoz, ami ellentmond´ as. N´emeth M´ arton (Nagykanizsa, Batthy´ any L. Gimn., 11. ´evf.) 72 dolgozat ´erkezett. 6 pontos 34, 5 pontos 7, 4 pontos 5, 3 pontos 2, 2 pontos 3, 1 pontos 12, 0 pontos 7 dolgozat.

most el˝ osz¨or a legnagyobb sz´ amjegyet. Ezut´an haladjunk nagys´ ag szerint lefel´e. Mindegyik sz´ amjegyet a k´esz¨ ul˝ o n-jegy˝ u sz´ amnak vagy a bal, vagy a jobb, vagy mindk´et oldalal´ ara ´ırhatjuk le. Ez a m´odszer nyilv´ an mindig k¨ ul¨ onb¨ oz˝ o, ´es helyes hegyszer˝ u sz´ amot ad; m´asr´eszt pedig megkapunk minden lehets´eges hegyszer˝ u sz´ amot az adott sz´ amjegyekb˝ ol k´epezve. Ez azt jelenti, hogy k sz´ amjegy eset´en elrendeos´eg van. Ha b´ armelyik kiv´ alaszt´ as b´ armelyik sorrendj´ehez z´es¨ ukre 3k−1 -f´ele lehet˝ hozz´ avessz¨ uk m´eg a null´ at (amely mindig csak az utols´ o helyre ker¨ ulhet), akkor k´esz´ıtett¨ unk egy u ´jabb hegyszer˝ u sz´ amot, teh´ at az 1–9-es sz´ amjegyekb˝ ol k´epzett hegyszer˝ u sz´amok sz´ama az ¨ osszes k´esz´ıthet˝ o sz´ am fele. Az eddigiek alapj´an a legnagyobb sz´ amjegy szerint ¨ osszegezve:            9 9 9 9 9 · 31 + · 32 + . . . + · 37 + · 38 · 2 = 174 762. · 30 + 2 3 8 9 1 ´ Isk. ´es Gimn., 11. ´evf.) Kov´ acs M´ oric (Budapesti Fazekas M. Gyak. Alt. dolgozata alapj´an Megjegyz´esek. 1. Ha 0 sz´ amot is hegyszer˝ unek tekintj¨ uk, akkor eggyel nagyobb ¨ osszeget kapunk. 2. A szerepl˝ o binomi´ alis egy¨ utthat´ ok val´ osz´ın˝ us´ıtik, hogy ez az ¨ osszeg a binomi´ alis t´etel alapj´ an is kisz´ am´ıthat´ o. Val´ oban, ´ırjuk fel a 49 -t (1 + 3)9 -k´ent:           9 9 9 9 9 0 1 2 8 ·3 + ·3 + · 3 + ... + ·3 + · 39 . 4 = (1 + 3) = 0 1 2 8 9 9

9

Ha ebb˝ ol az ¨ osszegb˝ ol elhagyjuk az els˝ o tagot – az 1-et –, majd a fennmarad´ o tagokat 3-mal osztjuk, akkor ´eppen a feladat v´egeredm´eny´eben szerepl˝ o z´ ar´ ojeles kifejez´est kapjuk. Teh´ at a hegyszer˝ u sz´ amok sz´ ama pontosan: 262 144 − 1 49 − 1 ·2= · 2 = 174 762. 3 3 ¨ Osszesen 112 dolgozat ´erkezett. 3 pontot kapott 75, 2 pontot 11 tanul´ o. 1 pontos 17, 0 pontos 5 versenyz˝ o dolgozata. Nem versenyszer˝ u 4 dolgozat.

B. 5209. Egy 2022 elem˝ u, eg´eszekb˝ ol ´ all´ o halmaznak legfeljebb h´ any olyan k´etelem˝ u r´eszhalmaza lehet, melyre a k´et elem ¨ osszege szint´en a halmazhoz tartozik? (5 pont)

B. 5206. Egy n-jegy˝ u a1 a2 a3 . . . an sz´ amot hegyszer˝ unek nevez¨ unk, ha van uan monoton n¨ ovekv˝ o, m´ıg olyan 1  k  n eg´esz, amelyre a1 , a2 , . . . , ak szigor´ uan monoton cs¨ okken˝ o sorozat. (P´eld´ aul az 1, 121, 1231 sz´ aak , ak+1 , . . . , an szigor´ mok hegyszer˝ uek, de az 1442 ´es az 12313 nem hegyszer˝ uek.) H´ any hegyszer˝ u sz´ am van? (3 pont) Megold´as. Ahhoz, hogy egy hegyszer˝ u sz´ amot tudjunk k´esz´ıteni, el˝osz¨or v´alasszuk ki, hogy mely sz´amjegyek fognak benne szerepelni (a 0-s sz´ amjegyet egyel˝ ore ´ ne v´ alasszuk ki). Rendezz¨ uk sorba nagys´ ag szerint ezeket a sz´amjegyeket. Irjuk le K¨ oz´ episkolai Matematikai ´ es Fizikai Lapok, 2022/5

283

Megold´as. A null´ at felt´etlen¨ ul ´erdemes belevenn¨ unk a halmazba, mert azt b´armely m´asik sz´ ammal ¨ osszeadva ism´et az elemet kapjuk. Tekints¨ uk el˝ osz¨ or csak a pozit´ıv elemeket. Legyenek ezek x1 < x2 < . . . < xn . Vizsg´aljuk meg mindegyik elem eset´eben, hogy legfeljebb h´ any darab n´ala kisebb p´ arja lehet (a p´ar itt azt jelenti, hogy ¨ osszeadva o˝ket ism´et a halmaz egy elem´et kapjuk). ´Igy meg fogjuk kapni az ¨ osszes p´ art. Mivel az xn a legnagyobb elem, ahhoz unk, nem rendelhet¨ unk tov´ abbi pozit´ıv elemet. Az xn−1 -hez csak egyet rendelhet¨ hiszen csak egy n´ala nagyobb eleme van a halmaznak. L´ atjuk, hogy egyik sz´amhoz sem rendelhet¨ unk sem a n´ ala nagyobb elemek sz´ am´an´al t¨ obb, sem a n´ ala kisebb 284

K¨ oz´ episkolai Matematikai ´ es Fizikai Lapok, 2022/5

pozit´ıv elemek sz´ am´an´ al t¨ obb darab elemet. (Az elm´eleti maximum pl. n = 4-re 0 + 1 + 1 + 0 = 2.) Ezt az elm´eleti maximumot el is lehet ´erni u ´gy, ha 1-t˝ ol n-ig vessz¨ uk a pozit´ıv eg´esz sz´ amokat. Ez a m´odszer a negat´ıv elemek k¨ oz¨ ott is ugyan´ıgy m˝ uk¨odik. Tekints¨ uk most a k¨ ul¨ onb¨ oz˝ o el˝ ojel˝ u sz´ amok o sszegeit. Legyenek a pozit´ıv sz´a¨ mok 1-t˝ ol n-ig, a negat´ıvak pedig (−1)-t˝ ol (−k)-ig. Az eddigiek alapj´ an n + k = = 2021. Ha b´ armelyik negat´ıv elemhez b´ armelyik pozit´ıvat p´ aros´ıtjuk az biztosan megfelel˝o lesz, mert az o oz´e fog esni. Ilyen p´ arb´ ol n · k darabot tu¨sszeg −k ´es n k¨ ul az egyik p´ aros, a m´ asik dunk megadni. A pozit´ıv ´es negat´ıv elemek sz´ama k¨oz¨ p´ aratlan. A szimmetria miatt feltehetj¨ uk tov´ abb´ a, hogy a pozit´ıvak sz´ama p´aratlan, n = 2u + 1, k = 2021 − 2u − 1 = 2020 − 2u.

Ha pedig a sz´amrendszer alapj´ anak 3-as marad´eka 2, akkor a marad´ekok sorban a k¨ovetkez˝ ok: 1, 2, 1, 2 . . . , mert 1-nek 1, ezt 2-vel (a 3-as marad´ekkal) szorozva 2 a marad´ek, ezt megint 2-vel szorozva 4, melynek 3-as marad´eka 1, ´es ´ıgy tov´ abb. Ez esetben az im´ent eml´ıtett 3 darab 1-esnek azokon a helyi´ert´ekeken kell elhelyezkednie, amelyek 3-mal osztva ugyanazt a marad´ekot adj´ ak. Az 101010 sz´am a fenti felt´etelek mindegyik´enek megfelel, ez´ert van ilyen sz´am. BLG-s kobakok (Nagykanizsa, Batthy´ any L. Gimn., 10. ´evf.) dolgozata alapj´an II. megold´as. A 101010 sz´ am ilyen, n alap´ u sz´ amrendszerben (n > 1 eg´esz) ennek a sz´ amnak a val´ odi ´ert´eke

A negat´ıv p´ arok sz´ama: (1010 − u − 1)(1010 − u), a pozit´ıv p´ arok sz´ama (u − 1)u + u, v´eg¨ ul a vegyes p´ arok sz´ama: (2020 − 2u)(2u + 1). Ezek ¨ osszeg´et vizsg´aljuk. Az u m´ asodfok´ u polinomj´at kapjuk:

n5 + n3 + n = n(n4 + n2 + 1). Ha n oszthat´ o 3-mal, akkor az (n4 + n2 + 1)-szerese is.

1009 · 1010 − 2019u + u2 + u2 + 4040u − 4u2 + 2020 − 2u = = −2u2 + 2019u + 1010 · 1011.

Ha n nem oszthat´ o 3-mal, akkor, mivel minden 3-mal nem oszthat´ o n´egyzet2 sz´ am 3-mal osztva 1 marad´ekot ad, ez´ert az 1, az n2 ´es az n4 = (n2 ) is. Ilyenkor az ¨osszeg¨ uk 3-mal oszthat´ o, ´ıgy annak n-szerese is.

Ez a f¨ uggv´eny maximum´ at a m´ asodfok´ u f¨ uggv´enyekre vonatkoz´ o ismert elj´ ar´ as b alapj´an u = − 2a ≈ 505 eset´en veszi fel. Ha az u = 505-h¨oz tartoz´o helyettes´ıt´esi ´ert´ekhez hozz´ aadjuk m´eg a 0-val k´epzett 2021 p´ art, kapjuk az el´erhet˝o maximumot:

Bel´ attuk, hogy az n-es sz´ amrendszerben fel´ırt 101010 sz´am b´ armilyen n-re oszthat´ o 3-mal, teh´at van ilyen sz´am.

−2 · 5052 + 2019 · 505 + 1010 · 1011 + 2021 = 1 532 676.

¨ Osszesen 133 dolgozat ´erkezett. 3 pontos 86, 2 pontos 34, 1 pontos 6 dolgozat. 0 pontot 1 versenyz˝ o kapott. Nem versenyszer˝ u 6 dolgozat.

J´ anosik M´ at´e (Gy˝ or, R´evai Mikl´ os Gimn., 12. ´evf.)

´ Isk. ´es Gimn., 10. ´evf.) L´ aszl´ o Anna (Budapesti Fazekas M. Gyak. Alt. dolgozata alapj´an ¨ Osszesen 65 dolgozat ´erkezett. 5 pontot kapott 30, 4 pontot 4 tanul´ o. 3 pontos 9, 2 pontos 12, versenyz˝ o dolgozata. 1 pontra ´ert´ekelt 6, 0 pontra 4 versenyz˝ o dolgozata.

B. 5214. A 110 egy olyan sz´ amjegysorozat, amelyet b´ armilyen 1-n´el nagyobb pozit´ıv eg´esz alap´ u sz´ amrendszerben tekintve p´ aros sz´ amot kapunk. Van-e olyan 1-esekb˝ ol ´es 0-kb´ ol ´ all´ o sz´ amjegysorozat, amelyet b´ armilyen 1-n´el nagyobb pozit´ıv eg´esz alap´ u sz´ amrendszerben tekintve 3-mal oszthat´ o pozit´ıv eg´esz sz´ amot kapunk?

B. 5229. Az a = 0 val´ os sz´ amra ´es az f : R → R f¨ uggv´enyre

teljes¨ ul minden x, y ∈ R eset´en. Bizony´ıtsuk be, hogy f addit´ıv, vagyis f (x + y) = = f (x) + f (y) minden x, y ∈ R eset´en. (6 pont)

Javasolta: George Stoica (Saint John, New Brunswick, Kanada)

Megold´as.

(3 pont) I. megold´as. Olyan sz´ amot keres¨ unk, amely legal´ abb egy darab 1-est tartalmaz, hiszen pozit´ıv; viszont nem v´egz˝ odhet 1-re, mert akkor a 3-as sz´ amrendszerben 1 marad´ekot adna 3-mal osztva, teh´ at az utols´o sz´ amjegye 0. Ha a sz´amrendszer alapja oszthat´ o 3-mal, akkor a nem utols´ o helyi´ert´ekek mindegyike 0 marad´ekot ad 3-mal osztva, mert tartalmaznak 3-as pr´ımt´enyez˝ ot, ´ıgy ez esetben b´ arhol lehet 1-es a keresett sz´ amban. Azokban a sz´ amrendszerekben, amelyekben az alap 3-as marad´eka 1, minden helyi´ert´ek 3-as marad´eka 1, mert ha ¨ osszeszorozzuk a marad´ekokat, az mindig 1 · 1 = 1. Ekkor annyi 1-es sz´ amjegy sz¨ uks´eges, hogy a sz´amuk 3-mal oszthat´ o legyen, azaz legal´abb 3 darab 1-est tartalmaz a sz´ am. K¨ oz´ episkolai Matematikai ´ es Fizikai Lapok, 2022/5

  f x + f (y) = f (x) + f (y) + ay

(1)

285

El˝ osz¨or a feladatban szerepl˝ o egyenlet (a tov´ abbiakban (1)) mindk´et oldal´ ahoz adjunk x-et, majd vegy¨ uk mindk´et oldal f szerinti k´ep´et:     f (x + f x + f (y) ) = f x + f (x) + f (y) + ay .

Az (1) alapj´an bontsuk ki a bal oldali kifejez´est:

      f (x + f x + f (y) ) = f (x) + f x + f (y) + a x + f (y) =

= f (x) + f (x) + f (y) + ay + ax + af (y),

286

K¨ oz´ episkolai Matematikai ´ es Fizikai Lapok, 2022/5

majd a jobb oldal kifejez´est is:

B. 5235. Mutassuk meg, hogy a Fibonacci-sorozatban minden 3-n´ al nagyobb pr´ımsz´ am 4k + 1 alak´ u.

  f x + f (x) + f (y) + ay = f (x + f (x) + ay) + f (y) + ay =

(5 pont)

= f (x + ay) + f (x) + ax + f (y) + ay.

Az egyenl˝os´eg fenn´ all, ´ıgy: f (x) + f (x) + f (y) + ay + ax + af (y) = f (x + ay) + f (x) + ax + f (y) + ay, (2)

f (x) + af (y) = f (x + ay),

amely egyenletb˝ol x = y = 0 helyettes´ıt´essel megkapjuk f (0)-t: f (0) + af (0) = f (0), ´ıgy af (0) = 0. Ebb˝ ol a = 0 miatt k¨ ovetkezik, hogy f (0) = 0.

Ha most (1)-be x hely´ere 0-t helyettes´ıt¨ unk ´es felhaszn´aljuk, hogy f (0) = 0, akkor az f f¨ uggv´eny u ´jabb tulajdons´ ag´ahoz jutunk: (3)

  f f (y) = f (0) + f (y) + ay = f (y) + ay.

olj¨ uk. Vizsg´ aljuk meg a Megold´as. Az n-edik Fibonacci-sz´amot Fn -nel jel¨ Fibonacci-sz´ amok n´egyes marad´ek´ at, az el˝ oz˝ o k´et sz´ am ¨ osszege a k¨ ovetkez˝ o (a sorozat defin´ıci´oj´ab´ ol ad´od´ oan). Az els˝ o k´et sz´ am marad´eka 1, a harmadik´e 1 + 1 = 2, a negyedik´e 2 + 1 = 3, az ¨ ot¨ odik´e 3 + 2 = 5 ≡ 1, a hatodik´e 1 + 3 = 4 ≡ 0, a hetedik´e 1 + 0 = 1, a nyolcadik´e 1 + 0 = 1 ´es innent˝ ol ism´etl˝odik, mivel a k¨ ovetkez˝ o sz´ am csak az el˝oz˝ o kett˝ ot˝ ol f¨ ugg. L´athatjuk, hogy hatos peri´odusonk´ent ism´etl˝odnek a sz´amok, a 0 ´es a 2 marad´ek´ u biztosan p´ aros, ez´ert nem lehet 3-n´al nagyobb pr´ım. Az egyetlen 4k + 3 alak´ u ebben a negyedik, teh´ at a sorozatban az ilyen sz´ auak, ahol l nemnegat´ıv eg´esz. Mivel 6l + 4 p´ aros, ´ıgy fel´ırhat´ o 2n mok az F6l+4 alak´ alakban, ahol n = 3l + 2, ahol n  2. A Fibonacci-sorozattal kapcsolatban sok k¨ ozismert azonoss´ ag l´etezik (l´asd ´ kor´ abbi cikk¨ unket: Enekes B´ela – K´ os G´eza: N´eh´ any ´erdekess´eg a Fibonacci- ´es a Fibonacci-t´ıpus´ u sorozatokr´ ol, amely ezen a linken ´erhet˝ o el: http://db.komal.hu/KomalHU/cikk.phtml?id=200117, illetve http://db.komal.hu/KomalHU/showpdf.phtml?tabla=Cikk&id=200117). Az ismert t´etelek k¨ oz¨ ul most a teljes indukci´ oval k¨ onnyen bel´ athat´ o

Most pedig helyettes´ıts¨ unk (2)-be x hely´ere f (y)-t:

    f f (y) + af (y) = f f (y) + ay ,

Fn+k−1 = Fn Fk + Fn−1 Fk−1

amib˝ ol a jobb oldalon (1), a bal oldalon (3) alkalmaz´ as´aval f (y) + ay + af (y) = f (ay) + f (y) + ay,

azonoss´ agot alkalmazzuk (a cikkben a 4. oldalon (6)-os sz´ammal tal´ alhat´ o meg). Alkalmas (k − 1 = n) helyettes´ıt´essel kapjuk, hogy

af (y) = f (ay).

F2n = Fn Fn+1 + Fn−1 Fn = Fn (Fn+1 + Fn−1 ),

Ezt (2)-be visszahelyettes´ıtve az f (x) + f (ay) = f (x + ay) egyenlethez jutunk, ami ´eppen az additivit´ ast jelenti, hiszen ay = z eset´en z tetsz˝ oleges val´ os sz´am, mivel a = 0 ´es y tetsz˝ oleges val´ os sz´am. Ekkor teh´ at x ´es z tetsz˝ oleges val´ os sz´amokra teljes¨ ul, hogy f (x) + f (z) = f (x + z). ´ Eppen ezt akartuk megmutatni, ´ıgy a bizony´ıt´ asunkat befejezt¨ uk. Fazok´ an Marcell (Debreceni Fazekas Mih´ aly Gimn., 10. ´evf.) dolgozata alapj´an ¨ Osszesen 42 dolgozat ´erkezett. 6 pontos 20, 5 pontos 2, 4 pontos 4 dolgozat. 3 pontot 5, 2 pontot 3, 1 pontot 3 versenyz˝ o kapott. 5 dolgozat 0 pontos lett.

K¨ oz´ episkolai Matematikai ´ es Fizikai Lapok, 2022/5

287

ami azt jelenti, hogy Fn | F2n , vagyis, ha F2n pr´ımsz´ am, akkor Fn = 1 vagy obbi ellentmond´ as, hiszen a Fibonacci-sorozat n  2-re szigor´ uan moFn = F2n . Ut´ noton n¨oveked˝o. Ha pedig Fn = 1, akkor n = 1 vagy n = 2, azonban az n  2 felt´eat val´oban tel miatt csak az n = 2 esetet kell vizsg´alnunk. Ekkor F2n = F4 = 3, teh´ 4k + 3 alak´ u pr´ımsz´ amot kaptunk. T¨ obb eset nincs, ´ıgy bel´ attuk, hogy a Fibonaccisz´ amok k¨oz¨ ul az egyetlen 4k + 3 alak´ u pr´ım a 3, azaz a Fibonacci-sorozatban szerepl˝o, 3-n´ al nagyobb pr´ımsz´ amok sz¨ uks´egk´eppen 4k + 1 alak´ uak. Ezzel a bizony´ıt´ast befejezt¨ uk. ´ Isk. ´es Gimn., 10. ´evf.) B´enyei Borisz (Budapesti Fazekas M. Gyak. Alt. dolgozata alapj´an ¨ Osszesen 66 dolgozat ´erkezett. 5 pontos 26, 4 pontos 23, 3 pontos 10 dolgozat. 2 pontot 6, 1 pontot 1 versenyz˝ o kapott.

288

K¨ oz´ episkolai Matematikai ´ es Fizikai Lapok, 2022/5

C. 1727. F´ urjunk ´at egy R sugar´ u t¨ om¨ or g¨ omb¨ ot egy, a g¨ omb k¨ oz´eppontj´an a´tmen˝ o egyenes ment´en egy r sugar´ u hengeres f´ ur´ oval, ahol r < R. Fejezz¨ uk ki a keletkezett marad´ektest t´erfogat´ at a marad´ektest m magass´ ag´ anak f¨ uggv´eny´eben.

A C pontversenyben kit˝ u z¨ ott gyakorlatok (1721–1727.)

Javasolta: Szab´ o Bertalan (Miskolc, 1986) Beku unius 10. ¨ld´esi hat´arid˝o: 2022. j´ Elektronikus munkafu ¨zet: https://www.komal.hu/munkafuzet

Feladatok 10. ´evfolyamig



C. 1721. Bogl´arka fel´ırt sorban egym´as ut´an 2022 darab sz´ amot u ´gy, hogy a m´asodik sz´ amot elosztva az els˝ ovel, a h´anyados ´eppen a harmadik sz´ ammal lett egyenl˝ o, ´es ´ıgy tov´ abb, p´eld´ aul a hetedik sz´am egyenl˝o a hatodik ´es az ¨ot¨odik sz´ am h´ anyados´ aval. Melyik sz´ amot ´ırta fel utolj´ ara Bogl´arka, ha az els˝ o a 20, a m´ asodik pedig a 22 volt? C. 1722. Az ABCD n´egysz¨ ogr˝ ol tudjuk, hogy a AD oldal ugyanolyan hossz´ u, mint a DC oldal, valamint hogy a DAB sz¨ oget α-val jel¨olve ABC = 2α, BCD = = 3α ´es CDA = 4α. Bizony´ıtsuk be, hogy az AB oldal k´etszer olyan hossz´ u, mint az AD oldal. (N´emet versenyfeladat)

A B pontversenyben kit˝ uz¨ott feladatok (5246–5253.)

B. 5246. 14 ember u or¨ ul, mindenki k´ek vagy s´ arga p´ ol´oban. ¨l egy asztal k¨ Legfeljebb h´ any emberre teljes¨ ulhet, hogy a k´et szomsz´edja k¨ ul¨ onb¨ oz˝ o sz´ın˝ u p´ ol´oban van? (3 pont)

Feladatok mindenkinek C. 1723. Hat´ arozzuk meg mindazon, csupa k¨ ul¨onb¨oz˝ o sz´amjegyb˝ ol ´all´o, legfeljebb n´egyjegy˝ u abcd sz´ amokat (ahol a = 0 is megengedett), amelyekre 9 · abcd = = acbcd. Javasolta: Siposs Andr´ as (Budapest)

B. 5247. Egy k¨ ot´el k´et v´egpontj´ at a talajhoz r¨ ogz´ıtett¨ uk u ´gy, hogy a k´et v´egpont t´ avols´ aga kisebb a k¨ ot´el hossz´ an´al. A k¨ ot´el k¨ oz´eps˝ o pontj´ at 150 cm magass´ agra felemelve a k¨ot´el megfesz¨ ul. A k¨ ot´el egyik v´eg´et˝ ol 90 cm-re l´ev˝ o pontj´ at felemelve a k¨ot´el 90 cm magasan fesz¨ ul meg. Milyen hossz´ u a k¨ ot´el? (3 pont) B. 5248. Oldjuk meg az al´abbi egyenletrendszert a val´ os sz´amok halmaz´an:

C. 1724. Az ABC h´ aromsz¨ ogben CAB = 30◦ . Mekkor´ ak a h´aromsz¨og ismear be retlen sz¨ ogei, ha a h´ aromsz¨ og C pontb´ ol indul´o s´ ulyvonala 45◦ -os sz¨oget z´ az AB egyenessel?

x2 y2 8 + +x+y = , y x xy x(x + 1) + y(y + 1) = 6.

C. 1725. Legyen p pozit´ıv pr´ımsz´ am. Tudjuk, hogy az x2 − px − 580p = 0 egyenlet gy¨ okei eg´esz sz´ amok. Hat´ arozzuk meg p ´ert´ek´et. Javasolta: Szalai M´ at´e (Szeged) Feladatok 11. ´evfolyamt´ol C. 1726. Mutassuk meg, hogy ha x, y, z olyan val´os sz´amok, amelyekre x y z + + = 1, y+z z+x x+y

akkor

B. 5249. Jel¨ olj¨ uk az ABC h´ aromsz¨ og be´ırt k¨ or´enek ´erint´esi pontjai ´altal meghat´ arozott h´ aromsz¨ og ter¨ ulet´et T0 -lal, a hozz´ a´ırt k¨ or¨ ok k¨ oz´eppontjai a´ltal meghat´arozott h´ aromsz¨ og ter¨ ulet´et pedig T1 -gyel. Mutassuk meg, hogy T0 ´es T1 m´ertani k¨ozepe megegyezik az ABC h´ aromsz¨ og ter¨ ulet´evel. (5 pont)

Javasolta: B´ artfai P´ al

B. 5250. Igazoljuk, hogy minden nemnegat´ıv eg´esz n sz´ amra

x2 y2 z2 + + = 0. y+z z+x x+y

22 (5 pont)

Adjunk meg a felt´etelt teljes´ıt˝ o val´os sz´amokat. K¨ oz´ episkolai Matematikai ´ es Fizikai Lapok, 2022/5

(4 pont)

289

290

n

(n−2)+n+2

 (2n )!  22

n

(n−1)+1

.

Javasolta: Blahota Istv´ an (Ny´ıregyh´ aza) K¨ oz´ episkolai Matematikai ´ es Fizikai Lapok, 2022/5

B. 5251. Vegy¨ uk fel azt az ABCD t´eglalapot a koordin´ atarendszerben, amelynek cs´ ucsai A(0, 0), B(2022, 0), C(2022, 2), D(0, 2). Tekints¨ uk azokat az egys´egnyi ter¨ ulet˝ u h´ aromsz¨ ogeket, amelyek mindh´ arom cs´ ucsa a t´eglalap hosszabbik oldalp´arj´anak egy-egy r´ acspontja. Ezeket a h´aromsz¨ ogeket szeretn´enk megsz´ınezni u ´gy, hogy azonos sz´ın˝ u h´ aromsz¨ ogeknek nem lehet k¨ oz¨ os bels˝o pontjuk. Legal´ abb h´any sz´ınre van ehhez sz¨ uks´eg¨ unk? (5 pont)

Javasolta: Nagy Zolt´ an L´ or´ ant (Budapest)

B. 5252. Adott egy hat cs´ ucs´ u ABCA1 B1 C1 poli´eder, amelynek ABC ´es aromsz¨oglapja, tov´abb´ a az AA1 , BB1 ´es CC1 ´elei p´ arhuzamosak. A1 B1 C1 k´et h´ oinak metsz´espontjai P , Q Az AA1 B1 B, BB1 C1 C ´es CC1 A1 A trap´ezlapok ´atl´ ´es R. Mutassuk meg, hogy az ABCP QR ´es A1 B1 C1 P QR poli´ederek t´erfogata megegyezik. (6 pont)

n

Javasolta: Kocsis Szilveszter (Budapest)

A. 828. Az ABC h´ aromsz¨ og be´ırt k¨ or´enek k¨ oz´eppontja I, hozz´ a´ırt k¨ orei pedig ol az ΩA k¨ orh¨ oz ΩA , ΩB ´es ΩC . Legyen A az az egyenes, amely ´atmegy az I pontb´ oan vannak defini´alva. h´ uzott ´erint˝ok ´erint´esi pontjain. Az B ´es C egyenesek hasonl´ arozott h´ aromsz¨ og Bizony´ıtsuk be, hogy az A , B ´es C egyenesek a´ltal meghat´ magass´ agpontja megegyezik az ABC h´ aromsz¨ og Nagel-pontj´aval. (Ha egy h´ aromsz¨ og cs´ ucsait ¨ osszek¨ otj¨ uk a szemk¨ ozti oldalhoz hozz´ a´ırt k¨ or¨ ok ´erint´esi pontjaival, a kapott h´arom szakasz k¨ oz¨ os pontja a h´aromsz¨ og Nagel-pontja.) Javasolta: Nikolai Beluhov (Bulgaria) A. 829. Legyen G egy n cs´ ucs´ u egyszer˝ u gr´ af, melynek van legal´abb egy ulyoz´ asait, melyekre ´ele, ´ e s tekints¨ u k a gr´ a f cs´ u csainak azon S : V (G) → R0 s´  S(v) = 1. Legyen tov´ abb´ a v∈V (G)

f (G) = max S

B. 5253. Igaz-e, hogy ha k p´ aros, akkor az n elem˝ u S halmaz k elem˝ u r´eszhalmazai p´ arokba rendezhet˝ ok u ´gy, hogy az egy p´arba tartoz´ o r´eszhalmazok szimmetrikus differenci´ aja mindig 2 elem˝ u? (6 pont)

S(v)S(w),

ahol S v´egigfut az ¨ osszes lehets´eges s´ ulyoz´ ason. 1 ul, ha G cs´ ucsai Bizony´ıtsuk be, hogy f (G) = n2 akkor ´es csak akkor teljes¨ lefedhet˝ ok ´elek ´es p´ aratlan k¨ or¨ ok diszjunkt uni´oj´aval. (V (G) a G gr´ af cs´ ucsait, E(G) a G gr´ af ´eleit jel¨ oli.)



Beku unius 10. ¨ld´esi hat´arid˝o: 2022. j´ Elektronikus munkafu ¨zet: https://www.komal.hu/munkafuzet

Beku unius 10. ¨ld´esi hat´arid˝o: 2022. j´ Elektronikus munkafu ¨zet: https://www.komal.hu/munkafuzet





Az A pontversenyben kit˝ u z¨ ott nehezebb feladatok (827–829.)

Informatik´ab´ ol kit˝ uz¨ott feladatok

A. 827. Legyen n > 1 eg´esz sz´ am. Egy pakliban n-f´ele sz´ın˝ u ´es n-f´ele ´ert´ek˝ u k´ artya van, minden sz´ın ´es ´ert´ek p´ arb´ ol pontosan egy, azaz ¨osszesen n2 darab. A paklit megkeverj¨ uk, ´es kiosztjuk n j´at´ekos k¨ oz¨ ott u ´gy, hogy mindenki n darab k´ arty´ at kapjon. A j´ at´ekosok azt akarj´ak megcsin´alni, hogy egy ´altaluk v´ alasztott sorrendben le¨ ulnek egy k¨ or alak´ u asztalhoz, ´es az els˝o j´at´ekost´ ol kezdve sorban leraknak egy-egy lapot, m´ıg v´eg¨ ul mindenki lerakta az ¨osszes lapj´ at u ´gy, hogy mindig olyan k´arty´ at kell rakni, amely sem sz´ınben, sem ´ert´ekben nem egyezik meg a k¨ ozvetlen¨ ul el˝ otte lerakott k´ arty´ aval (az els˝onek lerakott k´ artya b´ armi lehet). Mely n-ekre lehets´eges, hogy u ´gy lett kiosztva a pakli, hogy a j´ at´ekosok ezt nem tudj´ak megcsin´alni? (A j´at´ekosok egy¨ uttm˝ uk¨ odnek egym´ assal, ´es l´atj´ ak egym´as lapjait.) Javasolta: Kocsis Anett (Budapest) K¨ oz´ episkolai Matematikai ´ es Fizikai Lapok, 2022/5

min

(v,w)∈E(G)

291

I. 565. Az el˝oz˝ o havi sz´ amunkban megjelent tr´ef´as fejt¨ or˝ o a k¨ ovetkez˝ o volt: Helyezz¨ unk el hat feh´er b´ abut egy sakkt´ abl´ ara k´et vil´agos k´eszletb˝ol u ´gy, hogy ” egy s¨ot´et b´ abut let´eve b´ armely szabad mez˝ore, az biztosan u ¨thet˝o legyen.” K´esz´ıts¨ unk programot, amely egy fejt¨ or˝ o megold´ as´ at ellen˝ orzi, teh´at megadja, hogy az elhelyez´esben val´ oban minden szabadon maradt mez˝ ot u ¨t´esben tartanak-e a vil´agos b´ abuk. A program a standard bemenet nyolc sor´ ab´ol olvasson be egy elhelyez´est. A sakkb´ abuk bet˝ ujele: vez´er = V, b´ astya = B, husz´ar = H, fut´o = F, kir´ aly = K, gyalog = G. Az u o mez˝ oket egy-egy sz´ ok¨ oz jel¨ oli. ¨resen a´ll´ Ha az elhelyez´es megfelel˝ o, akkor a program az OK u ¨zenetet jelen´ıtse meg a standard kimeneten. Ha az elhelyez´es nem j´o, akkor a program a standard kimenet nyolc sor´ aba ´ırja ki a sakkt´ abl´ at, jel¨ olve a vil´ agos b´ abukat, illetve jelenjen meg egyon, amelyek nincsenek u egy X karakter azokon a mez˝ok¨ ¨t´esben. 292

K¨ oz´ episkolai Matematikai ´ es Fizikai Lapok, 2022/5

4. Az A5:A34 tartom´ anyban adjuk meg az els˝ o 30 pozit´ıv eg´esz sz´ amot ´es a B5:B19 tartom´ anyban az els˝o 15 eg´esz sz´ am faktori´ alis´ at. 5. Az F5:F34 tartom´ anyban adjuk meg a B2 cell´ aban tal´alhat´ o sz¨ oveg karaktereit egym´ as alatt u ´gy, hogy fentr˝ ol lefel´e a faktori´ alis sz´ amrendszerben adott sz´ am karaktereit jobbr´ ol balra tudjuk kiolvasni. A sz¨ ovegn´el hosszabb karakterhelyeket jel¨ ol˝o cell´ akban 0 ´ert´ek jelenjen meg.

P´elda: Bemenet (a sz´ ok¨ oz¨ ok hely´et pont jel¨ oli) ........ ..V..... .....V.. .B...... .......B ........ ......B. B.......

Kimenet ....X... ..V..... .....V.. .B...... .......B ...XX... ......B. B.......

Bek¨ uldend˝o egy t¨ om¨ or´ıtett i565.zip ´ allom´anyban a program forr´ ask´odja ´es r¨ ovid dokument´ aci´ oja, amely megadja, hogy a forr´ as´allom´any melyik fejleszt˝oi k¨ ornyezetben ford´ıthat´ o. ´ A 2022. janu´ari sz´ I. 566 (E). amban megjelent I. 553. feladatban egy t´ızes sz´ amrendszerben adott sz´ amot kellett megadni faktori´ alis sz´amrendszerben. A faktori´ alis sz´ amrendszerben megadott sz´ am n-edik helyi´ert´ek´en a´ll´o sz´ amjegy´et n! ´eram faktori´ alis sz´amrendszerbeli alakja t´ek´evel kell szorozni. P´eld´ aul a 23510 sz´ 14301! , azaz 1 · 1 + 3 · 6 + 4 · 24 + 1 · 120. Amennyiben a faktori´alis sz´amrendszerben 9-esn´el nagyobb sz´ amjegy fordul el˝o, akkor az ott szerepl˝ o sz´ amot z´ ar´ ojelbe tessz¨ uk. am m´asodik legmagasabb helyi´ert´ek˝ u sz´amjegye 10, P´eld´ aul az 1(10)406010111! sz´ ´es a sz´ am t´ızes sz´ amrendszerben 77 686 68910 . Ebben a feladatban t´abl´ azatkezel˝ o seg´ıts´eg´evel kell elk´esz´ıten¨ unk az ´atv´ alt´ ast: egy faktori´ alis sz´ amrendszerben fel´ırt eg´esz sz´ am t´ızes sz´amrendszerbeli alakj´ at kell megadni. A faktori´alis sz´ amrendszerben fel´ırt sz´am legf¨oljebb 15 sz´amjegy˝ u, a nyit´ o ´es z´ar´ o z´ar´ ojelekkel egy¨ utt legf¨ oljebb 30 karakterb˝ ol ´all. • • •



6. A G5:G34 tartom´ any cell´aiban adjunk meg olyan k´epletet, amely megsz´ amolja a faktori´ alis sz´ amban jobbr´ol balra haladva az adott karakterhely el˝ott el˝ ofordul´o nyit´o z´ ar´ ojeleket. 7. A H5:H34 tartom´ any cell´aiban adjunk meg olyan k´epletet, amely megsz´ amolja a faktori´ alis sz´ amban jobbr´ol balra haladva az adott karakterhely el˝ott el˝ ofordul´o z´ ar´ o z´ ar´ ojeleket. 8. Az I5:I34 tartom´ any cell´aiban hat´arozzuk meg a faktori´ alis sz´ am megfelel˝o ojeleket tartalmaz´ o sorokban 0 ´ert´eket adjunk meg vagy sz´ amjegy´et. A z´ ar´ hagyjuk u o ´es z´ ar´ o z´ ar´ ojelek k¨ oz¨ otti sorokban a´ll´ıtsuk el˝o a t¨ obbje¨resen, a nyit´ gy˝ u sz´am sz´amjegyei alapj´an a t¨ obbjegy˝ u sz´ am ´ert´ek´et, ami az utols´ o z´ ar´ ojelen bel¨ uli sz´ am sor´ aban jelenjen meg.

Minta:

A megold´ as sor´ an vegy¨ uk figyelembe a k¨ ovetkez˝ oket: Amennyiben lehets´eges, ´es a feladat m´ast nem mond, akkor a megold´ as sor´ an k´epletet, f¨ uggv´enyt, hivatkoz´ ast haszn´ aljunk. A megold´ ashoz seg´edsz´ am´ıt´ asokat a K oszlopt´ ol jobbra v´egezhet¨ unk. A r´eszfeladatok k¨oz¨ ott van olyan, amely egy kor´ abbi k´erd´es eredm´eny´et haszn´ alja fel. Ha a kor´abbi r´eszfeladatot nem siker¨ ul teljesen megoldani, akkor as´ at u ´gy, ahogy van, vagy helyettes´ıts¨ uk megfelel˝ o ´ert´ekhaszn´ aljuk a megold´ kel, ´es azzal dolgozzunk tov´ abb. ´Igy ugyanis pontokat kaphatunk erre a r´eszfeladatra is. A megold´ ashoz program vagy makr´o nem haszn´ alhat´ o, kiz´ar´ olag a t´abl´ azatkezel˝ o be´ep´ıtett f¨ uggv´enyeivel dolgozzunk.

1. Alak´ıtsuk ki a lenti mint´ anak megfelel˝o t´ abl´ azatszerkezetet ´es ments¨ uk a t´ abl´ azatot faktor10 n´even a t´ abl´ azatkezel˝ o alap´ertelmezett form´ atum´ aban. 2. K´esz´ıts¨ uk el az els˝ o ´es a negyedik sorban l´ atott fejl´ecet, ahol sz¨ uks´eges egyes´ıts¨ uk a cell´akat, illetve igaz´ıtsuk v´ızszintesen k¨oz´epre a cell´ ak tartalm´at. 3. Az A2 cell´ aba ´ırjunk be egy faktori´ alis sz´ amrendszerben megadott sz´ amot sz¨ovegk´ent. A sz´ am t´ızes sz´amrendszerben adott alakja a C2-es cell´aban jelenjen meg. K¨ oz´ episkolai Matematikai ´ es Fizikai Lapok, 2022/5

293

294

K¨ oz´ episkolai Matematikai ´ es Fizikai Lapok, 2022/5

9. A J5:J34 tartom´ any cell´aiban adjuk meg, hogy az el˝ oz˝ o oszlopban szerepl˝o sz´ amjegy a kisebb helyi´ert´ekekt˝ ol indulva h´anyadik sz´ amjegye a faktori´alis sz´ amrendszerbeli sz´ amnak. Amennyiben az I oszlopban nem egy sz´ amjegy ´ert´eke van, akkor ott az el˝ oz˝ o sorban l´ev˝ o sz´amot adjuk meg. Egy sz´am a minta szerint t¨ obbsz¨ or is megjelenhet, de sorrendben az els˝o sz´ am adja meg, hogy az adott sorban az I oszlopban kapott ´ert´ek h´ anyadik sz´ amjegy´et adja a faktori´ alis sz´ amrendszerbeli sz´amnak. 10. A C5:C19 tartom´ anyban adjuk meg a J oszlopban meghat´ arozott sorsz´ am alapj´an a faktori´alis sz´ amrendszerbeli sz´ am megfelel˝o helyi´ert´ek´en ´all´o sz´amjegyet t´ızes sz´ amrendszerben, vagy 0 ´ert´eket. 11. A D5:D19 tartom´ anyban adjuk meg a B ´es C oszlopban l´ev˝ o megfelel˝ o ´ert´ekek szorzat´ at, vagy 0 ´ert´eket. anyban kisz´am´ıtott sz´ amok ¨osszeg´et. 12. A C2 cell´ aban adjuk meg a D5:D19 tartom´ 13. Form´azzuk a t´ abl´ azatot a mint´ anak megfelel˝oen. Bek¨ uldend˝o egy t¨ om¨ or´ıtett i566.zip ´ allom´anyban a megold´ ast tartalmaz´ o munkaf¨ uzet ´es a megold´ as r¨ ovid le´ır´ as´ at bemutat´ o dokument´ aci´ o. I. 567. Vizsg´aljuk meg k´et t¨ omegpont mozg´ as´at, amelyek egy s´ıkban mozognak, ´es mozg´ asukat csak a k¨ oz¨ ott¨ uk l´ev˝ o t¨ omegvonz´ as befoly´asolja. A t¨omegvonz´ ast a gravit´ aci´ os er˝ot¨ orv´enyb˝ ol sz´am´ıtsuk. A k´et t¨ omeg ´ert´eke, kezdeti helyzet¨ uk, valamint a kezdeti sebess´eg¨ uk legyen a megold´ asban param´eterk´ent megadhat´ o. A k´et o mozg´as´ ahoz a kezdeti sebess´egek is egy s´ıkba esnek. test egy s´ıkban t¨ ort´en˝ A megold´ as sor´ an a folyamatot bontsuk kis Δt id˝ ointervallumokra, melyekben a test sebess´ege ´es gyorsul´asa a´lland´ o ´ert´eknek tekinthet˝ o. Minden id˝ointervallum sor´ an sz´am´ıtsuk ki a testek k¨ oz¨ otti vonzer˝ o alapj´an a testek gyorsul´as´anak koordin´ at´ ait, majd ezek seg´ıts´eg´evel m´ odos´ıtsuk a testek sebess´eg´et, illetve a sebess´eget ismerve adjuk meg a test elmozdul´as´ at. Ezek alapj´ an minden id˝ ointervallum eltelte ut´ an tegy¨ uk az elmozdul´asnak megfelel˝o helyre a k´et testet u ´gy, hogy mozg´asuk p´aly´ aja l´athat´ o legyen. A feladat tetsz˝oleges online grafikus rendszerben vagy szimul´aci´ os k¨ornyezetben megoldhat´ o, illetve alkalmaz´ oi program k´esz´ıthet˝ o a versenyben haszn´ alhat´ o programoz´ asi nyelveken. A megold´ as mutassa be a k´et t¨omegpont mozg´ as´at, teh´at folyamatosan rajzolja ki a helyzet¨ uket. ast tartalmaz´ o Bek¨ uldend˝ok egy i567.zip t¨ om¨ or´ıtett ´allom´anyban a megold´ forr´ as´allom´anyok vagy a forr´ asok el´erhet˝ os´eg´et mutat´ o hivatkoz´ as, illetve egy r¨ovid dokument´ aci´ o, amely haszn´alati u ´tmutat´ ot ad a megold´ ashoz, illetve sz¨ uks´eg eset´en tartalmazza a programoz´ asi nyelv grafikus kieg´esz´ıt´es´enek m´odj´ at. I/S. 63. Adott egy N sorb´ ol ´es N oszlopb´ ol ´ all´o n´egyzetr´ acs (teh´ at ¨osszesen acspontot tartalmaz). Egy egyenest r´ acsegyenesnek nevez¨ unk, ha legal´ abb N 2 r´ k´et r´acsponton a´thalad. Adjuk meg, hogy N ´ert´ek´et˝ ol f¨ ugg˝oen h´ any k¨ ul¨onb¨oz˝ o r´acsegyenes van. A bemenet egyetlen sor´aban az N sz´ am tal´ alhat´ o.

P´eld´ ak:

295

Kimenet 6 20

Korl´ atok: 1  N  1000. Id˝ olimit: 0,4 mp. ´ Ert´ekel´es: a pontok 50%-a kaphat´ o olyan programra, amely megfelel˝o kimenetet ad, ha 1  N  50. S. 162. J´ on´ asnak van egy N cs´ ucs´ u teljes bin´aris f´ aja. A fa gy¨ okere az 1-es sorsz´ am´ u cs´ ucs. Ha az i-edik cs´ ucs nem lev´el, akkor a k´et gyereke a 2i ´es 2i + 1 sorsz´ am´ u cs´ ucs. Minden cs´ ucshoz hozz´arendelt¨ unk egy pozit´ıv eg´esz sz´ amot, ez a cs´ ucs s´ ulya. A s´ ulyok k¨ oz¨ ott nincs k´et egyforma. J´on´as megpr´ ob´ alta a f´ at maximum-kupacc´ a alak´ıtani. Azt szeretn´e, hogy minden cs´ ucs s´ ulya nagyobb legyen, mint a k´et gyerek´enek a s´ ulya. Ehhez a k¨ ovetkez˝ o m˝ uveletet hajtotta v´egre Q-szor: kiv´ alasztott egy cs´ ucsot, majd megkereste, melyik a legnagyobb s´ uly az ¨ osszes alatta l´ev˝ o cs´ ucsban. Ezut´ an a kiv´alasztott cs´ ucs s´ uly´at kicser´elte a legnagyobb s´ ullyal. Sz´ am´ıtsuk ki, mennyire siker¨ ult J´ on´as terve, azaz h´ any olyan sz¨ ul˝ o-gyerek p´ ar van, amire teljes¨ ul a kupac felt´etel. Bemenet: az els˝ o sor tartalmazza a cs´ ucsok N sz´ am´at. A k¨ ovetkez˝ o sorban a cs´ ucsok s´ ulya szerepel sorsz´ am szerint n¨ ovekv˝ o sorrendben. A harmadik sorban a cser´ek Q sz´ ama szerepel. A negyedik sorban Q sz´ am szerepel: a kiv´alasztott cs´ ucsok sorsz´amai a kiv´alaszt´ as sorrendj´eben. Kimenet: a kimenet els˝ o ´es egyetlen sor´ aba azon sz¨ ul˝ o-gyerek p´ arok sz´am´at kell ´ırni, melyben a sz¨ ul˝ o s´ ulya nagyobb, mint a gyerek s´ ulya. Minta: Bemenet (a / jel sort¨ or´est helyettes´ıt) 7 / 1 4 6 9 2 3 7 3 / 3 2 1

Kimenet 4

Magyar´ azat: a 3-as cs´ ucs s´ ulya 6, ezt a 7-essel cser´elj¨ uk meg. A 2-es cs´ ucs s´ ulya 4, ezt a 9-essel cser´elj¨ uk meg. Az 1-es cs´ ucs s´ ulya 1, ezt a 9-essel cser´elj¨ uk. A cs´ ucsok s´ ulya ezut´ an: 9, 1, 7, 4, 2, 3, 6. ucsok s´ ulya legfeljebb 109 . Id˝ olimit: 1 mp. Korl´ atok: 3  N = 2k − 1 < 105 . A cs´ ´ Ert´ekel´es: a pontok 50%-a kaphat´ o arra a programra, amely helyes kimenetet ad, ha N · Q  106 .

Bek¨ uldend˝o egy s162.zip t¨ om¨ or´ıtett a´llom´anyban a megfelel˝oen dokument´ alt ´es kommentezett forr´ asprogram, amely tartalmazza a megold´ as l´ep´eseit, valamint megadja, hogy a program melyik fejleszt˝ oi k¨ ornyezetben futtathat´ o.

 A feladatok megold´asai regisztr´aci´o ut´an a k¨ovetkez˝o c´ımen t¨olthet˝ok fel: https://www.komal.hu/munkafuzet Beku unius 15. ¨ld´esi hat´arid˝o: 2022. j´

A kimenet egyetlen sor´ aban egy sz´ am szerepeljen: a k¨ ul¨onb¨oz˝ o r´acsegyenesek sz´ ama. K¨ oz´ episkolai Matematikai ´ es Fizikai Lapok, 2022/5

Bemenet 2 3

296

K¨ oz´ episkolai Matematikai ´ es Fizikai Lapok, 2022/5

K¨ onyvismertet´es John D. Barrow: 100 alapvet˝ o dolog a matematik´ar´ ol ´es a m˝ uv´eszetekr˝ol, ˝ NEM TUDTUK, hogy nem tudjuk AMIROL J. D. Barrow (1952–2020) angol matematikus, elm´eleti fizikus, kozmol´ogus leny˝ ug¨ oz˝ oen ´erdekes k¨ onyv´et mintha kimondottan a K¨oMaL olvas´ oinak ´es versenyz˝ oinek ´ırta volna. A 100 r¨ ovid (egy-k´et oldalas) ´ır´ as – mindegyik k¨oz´erthet˝ o – kalandozik a matematika, a fizika, a csillag´aszat, a nyelv´eszet, a tudom´anyt¨ort´enet ´es a m˝ uv´eszetek hat´ arter¨ uletein. Kedvcsin´al´ok´ent – a teljess´eg ig´enye n´elk¨ ul – megeml´ıt¨ unk n´eh´ any gy¨ ongyszemet”. ” – A balerin´ ak egy-egy nagyobb ugr´as k¨ ozben mintha a gravit´aci´ ot legy˝ozve lebegn´enek a leveg˝ oben. Vajon hogyan egyeztethet˝o o¨ssze ez a (filmfelv´etelekkel is dokument´ alt) jelens´eg a t¨ omegvonz´ as t¨ orv´eny´evel? – K´epzelj¨ uk magunkat egy nagy k´ept´ ar biztons´agi f˝ on¨ok´enek. A termek falait rengeteg ´ert´ekes k´ep bor´ıtja, meglehet˝osen alacsonyra akasztva, hogy szemmagass´agb´ ol is j´ ol l´athat´ ok legyenek, ´ıgy viszont v´edeni kell ezeket a tolvajok ´es a rong´ al´ok el˝ol. A k´ept´ ar k¨ ul¨ onf´ele alak´ u ´es m´eret˝ u termekb˝ ol ´ all. Legal´ abb h´ any terem˝orre van sz¨ uks´eg¨ unk, hogy minden egyes k´epet folyamatosan szemmel tarthassanak? Sajnos ez nem k¨ onny˝ u, s˝ot u ´gynevezett neh´ez” sz´ am´ıt´ asi feladat, amelynek az elv´egz´es´e” hez sz¨ uks´eges id˝ o megdupl´az´ odhat, valah´ anyszor a gal´eria egy u ´jabb fallal b˝ov¨ ul. A matematikusok sokf´ele helyzetet tanulm´ anyoztak, olyanokat is, amikor az o˝r¨ok mozoghatnak, vagy t¨ ukr¨ ok seg´ıthetik o˝ket az eldugott sarkok szemmel tart´as´ aban. – Korunk egyik legfontosabb ´ert´eke a hat´ekonys´ag, ´es ennek szellem´eben hajlamosak vagyunk el´ıt´elni a k´esleked´est. M´egsem nyilv´ anval´o, hogy a halogat´as mindig nemk´ıv´ anatos. Ha a v´ allalkoz´ asunknak egy nagy feladat elv´egz´es´et kell kifizetnie, amelynek a r´eszfeladataihoz haszn´ alt elj´ ar´ asok egyre olcs´ obbak lesznek, akkor tal´ an meg´eri egy kicsit k´enyelmesebbre venni a temp´ ot, ´es k´es˝ obb hozz´ akezdeni, hiszen ´ıgy a teljes k¨ olts´eg kisebb lesz. A sz´ am´ıt´ og´epes feldolgoz´ asban ´eppen ez a helyzet. A h´ıres Moore-t¨ orv´eny kimondja, hogy egy adott p´enz¨osszeg´ert megv´ as´arolhat´ o sz´am´ıt´ og´epes teljes´ıtm´eny mintegy 18 havonta megdupl´ az´ odik. Megmutathat´ o, hogy a 18/ ln 2 ≈ 26 h´ onapn´ al jelenleg t¨ obb id˝ ot ig´enyl˝ o sz´ am´ıt´ astechnikai feladatokat ´erdemes k´es˝ obb elkezdeni, m´ıg az enn´el r¨ ovidebb idej˝ u projektekbe c´elszer˝ u a lehet˝o leghamarabb belekezdeni, mert a technol´ ogia jelenlegi fejl˝od´ese mellett azokat nem lehet hamarabb elv´egezni. – Van olyan hull´amvas´ ut, amelynek p´aly´ aja hurok alak´ u, ´es a hurok tetej´en´el ul sem esn´enk fejjel lefel´e utazunk, akkora sebess´eggel, hogy m´eg biztons´agi o¨v n´elk¨ ki. Vajon milyen alak´ u g¨ orbe a hurok? Aki azt gondolja, hogy k¨or, t´eved. Kisz´am´ıthat´ o (a K¨ oMaL-ban t¨obbsz¨ or kit˝ uz¨ ott feladatk´ent is szerepelt), hogy ha a motor n´elk¨ ul halad´ o szerelv´enyben a k¨ orp´ alya legmagasabb pontj´an´al ´eppen s´ ulytalanok” ” vagyunk, akkor a k¨ or legals´ o pontj´an´ al a norm´al tests´ ulyunk 6-szoros´at ´erz´ekelj¨ uk. A legt¨ obb utas ett˝ ol elvesz´ıten´e az eszm´elet´et, mert t´ ul kev´es oxig´en jutna az agy´ a-

ba. Emiatt a val´odi hull´ amvasutakn´ al a p´alyag¨ orbe nem k¨ or, hanem pl. klotoid (m´ as n´even Corn` u-f´ele spir´ al), amelynek g¨ orb¨ ulete a megtett u ´ttal ar´ anyosan cs¨ okken. – Mi´ert toj´ asdad alak´ u az igazi toj´ as? Milyen biol´ ogiai el˝onyei vannak a g¨ ombt˝ol (´es az ellipszoidt´ol, vagy a r¨ ogbilabda alakt´ ol) elt´er˝ o alak´ u mad´artoj´ asoknak? – Az egyszer˝ u matematikai szab´alyok k¨ oz¨ ul az egyik legmeglep˝ obb az u ´gynevezett Benford-t¨ orv´eny. Frank Benford amerikai m´ern¨ okr˝ ol nevezt´ek el, aki 1938ban ´ırt cikket r´ ola. Megfigyelte (ahogy azt m´ ar f´el ´evsz´ azaddal kor´ abban Simon Newcomb kanadai–amerikai asztrofizikus is ´eszrevette), hogy sok, els˝ore v´eletlenszer˝ unek gondolt sz´ amhalmaz (tavak ter¨ ulete, baseball-eredm´enyek, egy magazinban el˝ ofordul´o o sszes sz´ a m, a csillagok poz´ ıci´ oja, ´arjegyz´ekek, fizikai a´lland´ ok vagy ¨ o sz´ amjegye (meglehet˝ os pontoss´ aggal, a m´ert´ekegys´egt˝ ol f¨ ugk¨onyvel´esi t´etelek) els˝ getlen¨ ul) nagyon ´erdekes val´ osz´ın˝ us´egi eloszl´ast k¨ ovet. Furcsa m´ odon az 1, 2, 3, . . . , 9 sz´ amjegyek nem egyforma, hanem egyre cs¨ okken˝ o es´ellyel fordulnak el˝ o az els˝ o sz´ amjegy hely´en (az 1-es val´ osz´ın˝ us´ege 30%, a 9-es´e mind¨ossze 5%!) A Newcomb–Benford-t¨ orv´eny megd¨ obbent˝ o ter¨ uleteken is alkalmaz´ asra tal´ alt. Egy amerikai doktorandusz, Mark Nigrini 1992-ben azt vetette fel, hogy hamis k¨onyvel´esi adatok kisz˝ ur´es´ere is alkalmas lehet az els˝ osz´ amjegy-t¨ orv´eny. Ha egy ad´ oalany kital´alt, vagy ´eppen v´eletlensz´ am-gener´ atorral gy´ artott sz´amokkal t¨olti ki a mez˝ oket ´es nem val´ os t´enyeket le´ır´ okkal, ezek a sz´ amok nem k¨ ovetik a Newcomb–Benford t¨ orv´enyt. onyv egyik ´ır´ asa (H´ any sz´ ot ismert Shakespeare? ) adta az ¨ otletet a – A k¨ K¨oMaL m´ ult havi sz´ am´aban megjelent, pontversenyen k´ıv¨ uli feladathoz is. ´ Aprilisi p´otfeladat. A K¨ oMaL minden sz´ am´ at a nyomd´ aba ad´ as el˝ ott ketten is elolvass´ ak. A mostani sz´ amban az egyik lektor 60, a m´ asik 40 hib´ at tal´ alt, ´es ezek k¨ oz¨ ott 35 volt olyan, amelyet mindketten ´eszrevettek. Becs¨ ulj¨ uk meg, hogy h´ any hiba maradhatott ezek ut´ an a k´eziratban! Megold´as. Ha az egyik lektor p, a m´asik q val´ osz´ın˝ us´eggel veszi ´eszre a hib´akat, ´es a hib´ ak sz´ama N , akkor fenn´all, hogy p · N = 60, q · N = 40 ´es pq · N = 35, an kapjuk, hogy ahonnan p ´es q kik¨ usz¨ ob¨ol´ese ut´ N=

60 · 40 = 68,6 ≈ 69. 35

A fel nem fedezett hib´ ak v´elhet˝ o sz´ ama: n = 69 − (60 + 40 − 35) = 4. ´ Altal´aban, ha az egyik lektor N1 , a m´ asik N2 hib´ at tal´ alt, ´es ezek k¨ oz¨ ul N(1,2) -t vettek mindketten ´eszre, akkor n=

N1 N 2 − N1 − N2 + N(1,2) , N(1,2)

amit ´ıgy is fel´ırhatunk:   N1 − N(1,2) N2 − N(1,2) . n= N(1,2) 

(G. P.) K¨ oz´ episkolai Matematikai ´ es Fizikai Lapok, 2022/5

297

298

K¨ oz´ episkolai Matematikai ´ es Fizikai Lapok, 2022/5

a megfelel˝ o fesz¨ ults´egek: UA = UC = 13 U ´es UB = 23 U . A teljes´ıtm´enyek rendre 19 P , 4 P ´es 19 P . 9

Fizika gyakorlatok megold´asa

G. 771. Az ´ abr´ an l´ athat´ o kapcsol´ asban a fogyaszt´ ok azonos R ellen´ all´ as´ uak, ´es U fesz¨ ults´eg eset´en a teljes´ıtm´eny¨ uk P . Mekkora az egyes fogyaszt´ ok teljes´ıtm´enyfelv´etele a kapcsol´ ok nyitott (ny), illetve z´ art (z) ´ all´ as´ an´ al? T¨ olts¨ uk ki a t´ abl´ azatot!

1. ny ny z z

2. ny z ny z

A

B

C

3. a ´bra

4. a ´bra

4. eset: Mindk´et kapcsol´ o z´ art (4. a ´bra). Az A ´es a C fogyaszt´ o fesz¨ ults´ege U , a teljes´ıtm´eny¨ uk pedig P . A B fogyaszt´ on kereszt¨ ul nem folyik a´ram, a teljes´ıtm´enye teh´ at nulla. A kit¨olt¨ott t´ abl´ azat ´ıgy n´ez ki: 1.

(4 pont)

K¨ozli: Zsigri Ferenc, Budapest 2

Megold´as. Az egyes fogyaszt´ok teljes´ıtm´enye P  = UR , ahol U  a fogyaszt´ on m´erhet˝ o fesz¨ ults´eg. 1. eset: Mindk´et kapcsol´ o nyitott (1. ´ abra). (Az a´br´ an az ´aram u ´tj´at vastagabb, az a´rammentes szakaszokat pedig szaggatott vonal jel¨oli.) ults´eg jut, ´ıgy A sorosan kapcsolt A ´es B fogyaszt´ o mindegyik´ere U  = 12 U fesz¨ 1 o´e pedig nulla. a teljes´ıtm´eny¨ uk 4 P , a harmadik fogyaszt´

2.

A

B

C

ny

ny

1 P 4

1 P 4

0

ny

z

P

0

0

z

ny

1 P 9

4 P 9

1 P 9

z

z

P

0

P

Bir´ o Kata (Miskolc, F¨ oldes F. Gimn., 9. ´evf.) 28 dolgozat ´erkezett. Helyes 14 megold´ as. Kicsit hi´ anyos (3 pont) 5, hi´ anyos (1–2 pont) 3, hib´ as 6 dolgozat.

G. 772. A gyerekek k¨ orj´ at´ekot j´ atszanak a mez˝ on. Szerencs´etlen m´ odon a k¨ or k¨ ozep´en ´ all´ o t´ arsuk dar´ azsf´eszekbe l´ep, ´es a m´erges darazsak sz´etrep¨ ulnek. A mez˝ on keleti ir´ anyb´ ol 4,5 m/s sebess´eg˝ u sz´el f´ uj, a gyerekek 6 m/s nagys´ ag´ u sebess´eggel sug´ arir´ anyban menek¨ ulnek. A tud´ osok vizsg´ alata szerint ezek a darazsak sz´elcsendben 8 m/s sebess´eggel tudnak rep¨ ulni. Becs¨ ulj¨ uk meg, hogy a gyerekek h´ any sz´ azal´eka menek¨ ul meg biztosan a dar´ azscs´ıp´esekt˝ ol! A v´ alasz megad´ as´ ahoz haszn´ alhatunk ak´ ar vonalz´ ot, k¨ orz˝ ot ´es sz¨ ogm´er˝ ot is. (4 pont) 1. a ´bra

2. a ´bra

2. eset: Az 1. kapcsol´ o nyitott, a 2. z´ arva van (2. ´ abra). Most csak az A fogyaszt´ on folyik ´at ´ aram, a r´ a es˝ o fesz¨ ults´eg U  = U , ´ıgy a teljes´ıtm´enye P . 3. eset: Az 1. kapcsol´ o z´ art, a 2. nyitott a´ll´ asban van (3. ´ abra). A p´ arhuzamosan kapcsolt A ´es C fogyaszt´ o ered˝ o ellen´ all´ asa fele a B fogyaszt´ o R ellen´ all´as´ anak, ´ıgy K¨ oz´ episkolai Matematikai ´ es Fizikai Lapok, 2022/5

299

I. megold´as. Az 1. ´ abr´ an a folytonos vonallal jel¨ olt elmozdul´ asvektorok a daazsf´eszekbe l´ep´es ut´ an valamennyi (mondjuk 1 m´ asodrazsak helyzet´et jel¨ olik a dar´ percnyi) id˝o eltelt´evel. A szaggatott vonallal h´ uzott nyilak a darazsakt´ ol megmenek¨ ul˝ o gyerekek elmozdul´ asvektorait jel¨ olik. A k´et vektorsereg egy 8 egys´eg, illetve 6 egys´eg sugar´ u k¨ or ker¨ uleti pontjaiba mutat; a k¨ or¨ ok k¨ oz´eppontja k¨ oz¨ otti t´ avols´ ag a sz´el sebess´eg´enek megfelel˝oen 4,5 egys´eg. 300

K¨ oz´ episkolai Matematikai ´ es Fizikai Lapok, 2022/5

amib˝ol (2)

v12 + v22 + v02 + 2v2 v0 = 64.

A (2) egyenletb˝ol (1)-et kivonva kapjuk, hogy v02 + 2v2 v0 = 28,



v2 =

m 28 − 20,25 = 0,86 , 9 s

tov´ abb´ a v12 + v22 = 36 alapj´ an v1 = 1. ´ abra

4,52 = 62 + 82 − 2 · 6 · 8 cos α, ahonnan

36 + 64 − 20,25 = 0,83, 96 majd a szinuszt´etel szerint sin α 4,5 = , sin β 6



sin β = 0,742,

α = 33,83◦ ,





β = 47,92◦ .

A gyerekek k¨ oz¨ ul azok menek¨ ulnek meg biztosan a dar´ azscs´ıp´est˝ ol, akik az a´br´ an s¨ ot´etebben jel¨ olt 2(α + β) = 163,5◦ -os sz¨ og˝ u tartom´ any ir´ any´ aban kezdenek el szaladni. Ez a sz¨ og a teljes, 360◦ -nak 0,454-ed r´esze, teh´ at a gyerekek kb. 45%-a menek¨ ul meg a darazsakt´ ol. Team cucu: Esztinka Anna Karolina, Braun J´ ulia, Ludvig Csenge Lilla (Budapest, V´arosmajori Gimn., 10. ´evf.) II. megold´as. Bontsuk fel a sebess´egeket sz´elir´ anyra mer˝ oleges ´es a sz´ellel ellent´etes ir´ any´ u komponensekre. Tekints¨ uk azt a gyereket, akit ´eppen nem ´ernek utol a darazsak. Az o˝ sebess´e g´enek komponensei legyeut ´ert´eke v12 + v22 = 6 m/s, amib˝ ol nek v1 ´es v2 , abszol´ (m/s egys´egekkel sz´ amolva) (1)

m m = 5,94 . s s

Megjegyz´es. A fenti megfontol´ asok nem vett´ek figyelembe a gyerekek alkotta k¨ or at. Ez a m´eret nyilv´ an null´ at´ ol k¨ ul¨ onb¨ oz˝ o, ellenkez˝ o esetben a darazsak r¨ ogt¨ on r0 sugar´ megcs´ıphetik az o o dar´ azs a gyerekn´el r0 -lal hosszabb ¨sszes gyereket. A gyereket utol´er˝ utat kell megtegyen, ez az u ´tk¨ ul¨ onbs´eg azonban az u oz´esi id˝ o n¨ ovekedt´evel a dar´ azs ¨ld¨ altal megtett u ´ ´thoz viszony´ıtva egyre jelent´ektelenebb´e v´ alik. A gyereket ´eppen utol´er˝ o ozel´ıt´es dar´ azs rep¨ ul´esi ideje viszonylag nagy, teh´ at a megold´ as sor´ an alkalmazott r0 ≈ 0 k¨ jogos. (G. P.) 21 dolgozat ´erkezett. Helyes 9 megold´ as. Kicsit hi´ anyos (3 pont) 7, hi´ anyos (1–2 pont) 4, nem versenyszer˝ u 1 dolgozat.

Fizika feladatok megold´asa

v12 + v22 = 36.

2. ´ abra Legyen a sz´el sebess´ege v0 = 4,5 m/s. Ekkor a gyereket u oz˝ o ´es o˝t majdnem utol´er˝ o dar´ azs talajhoz viszony´ıtott rep¨ ul´esi sebess´eg´e¨ld¨ oh¨ oz k´epest m´ert sebess´ege v1 ´es v2 + v0 komponek komponensei v1 ´es v2 , a leveg˝ nens˝ u, a nagys´ aga  2

v12 + (v2 + v0 ) = 8 m/s,

K¨ oz´ episkolai Matematikai ´ es Fizikai Lapok, 2022/5

36 − 0,862

A darazsak el˝ ol ´eppen megmenek¨ ul˝ o gyerek a keleti sz´elir´ anyhoz k´epest 5,94 ◦ ´ ogben szalad. Igy azok a gyerekek menek¨ ulnek meg, akikα = arctg 0,86 = 82 -os sz¨ 82 nek sebess´ege legfeljebb 82◦ -kal t´er el a keleti ir´anyt´ol, ˝ok a csoport 180 ≈ 0,45 h´anyada, vagyis a 45%-a. ´ Isk. ´es Gimn., 9. ´evf.) Csilling D´ aniel (Budapesti Fazekas M. Gyak. Alt.

Az ´ abr´ an l´athat´ o kis h´ aromsz¨ ogre alkalmazva a koszinuszt´etelt:

cos α =



301

P. 5359. Egy kocka ´elei k´etf´ele ellen´ all´ asb´ ol ´ep¨ ulnek fel. Valamelyik k´et szemk¨ ozti laphoz tartoz´ o 8 db ´el ellen´ all´ as´ anak ´ert´eke r, m´ıg az ezekre mer˝ oleges 4 db ´elt alkot´ o ellen´ all´ asok ´ert´eke R. Hat´ arozzuk meg a h´ al´ ozat ered˝ o ellen´ all´ as´ at az egyik R ellen´ all´ ast k¨ ozrefog´ o, k´et szomsz´edos cs´ ucspont k¨ oz¨ ott! (4 pont)

K¨ ozli: Szekeres B´ela, Budapest

Megold´as. A vizsg´ alt k´et szomsz´edos cs´ ucspont legyen az 1. ´ abr´ an l´ athat´ oA ´es E pont. A szimmetria miatt a D ´es a B, valamint az F ´es a H pontok azonos abr´ an l´ athat´ ora egyszer˝ us¨ odik. potenci´ al´ uak. Ezt kihaszn´ alva a kapcsol´as 2. ´ 302

K¨ oz´ episkolai Matematikai ´ es Fizikai Lapok, 2022/5

P. 5382. Egy r´egi szalagos magnetofon gyors ´ attekercsel´eskor a szed˝ oors´ ot a ´lland´ o fordulatsz´ ammal forgatja. A k´et ors´ o bels˝ o´ atm´er˝ oje 5 cm, a k¨ uls˝ o´ atm´er˝ oj¨ uk pedig 15 cm. A teljesen teli ors´ or´ ol a magn´ oszalag a ´tcs´ev´el´esi ideje 3 perc. A szalagot az u ora tekercselik ´ at. Ind´ıt´ ast´ ol sz´ am´ıtva mennyi id˝ o m´ ulva lesz a k´et ¨res ors´ ors´ on ´eppen egyenl˝ o hossz´ us´ ag´ u magn´ oszalag? (4 pont)

K¨ ozli: Holics L´ aszl´ o, Budapest

Megold´as. Az ´ attekercsel´es kezdetekor a szed˝ oors´ on a magn´oszalag tekercs´enek ulva pedig r2 = 7,5 cm. A szalag hossz´ uk¨ uls˝ o sugara r1 = 2,5 cm, T = 3 perc m´ s´aga az ors´on l´ev˝ o r´esz keresztmetszet´enek ter¨ ulet´evel ar´ anyos. Legyen a tekercs ult a szed˝ oors´ ora. A m´ar k¨ uls˝ o sugara r3 akkor, amikor a szalag fele ´eppen a´tker¨ ´attekercselt r´esz keresztmetszet´enek ter¨ ulete ekkor megegyezik az a´ttekercseletlen r´esz ter¨ ulet´evel: r32 π − r12 π = r22 π − r32 π, ahonnan r3 = 1. a ´bra

2. a ´bra

K´et egyforma nagys´ ag´ u ellen´ all´ as p´ arhuzamos ered˝ oje feleakkora, mint az egyik¨ uk ellen´ all´ asa (pl. BD ´es A k¨ oz¨ ott r/2). A p´ arhuzamosan, illetve sorosan kapcsolt ellen´ all´asokra ´erv´enyes ¨osszef¨ ugg´esek szerint a BD ´es HF k¨ oz¨ otti ellen´ all´ as: 1 RBD−HF

=

A szed˝ oors´ o ´ alland´ o fordulatsz´ ama miatt a rajta l´ev˝ o tekercs k¨ uls˝ o sugara id˝ oben egyenletesen n¨ ovekszik. Ha a szalag fel´enek a´ttekercsel´es´ehez t id˝ ore van sz¨ uks´eg, akkor fenn´ all: r3 − r1 t = , r2 − r1 T

t=

teh´ at a BD ´es HF pontok k¨ oz¨ ott l´ athat´ o r´esz ellen´ all´asa: R(R + r) . 3R + 2r

RA−E

1

= r+

R(R+r) 3R+2r

2

+

2

3R + 2r 1 4R + 2r + 6Rr 1 = 2 + = , 2 R 2r + R + 4Rr R R(2r2 + R2 + 4Rr)

´ Isk. 11. ´evf.) Vigh Zs´ ofia (Szeged, SZTE Gyak. Gimn. ´es Alt. 36 dolgozat ´erkezett. Helyes 19 megold´ as. Kicsit hi´ anyos (3 pont) 3, hi´ anyos (1–2 pont) 11, hib´ as 1, nem versenyszer˝ u 2 dolgozat.

P. 5384. Egy v´ekony, homog´en, f¨ ugg˝ oleges p´ alca tetej´en kicsiny goly´ o helyezkedik el. A p´ alca t¨ omeg´ehez k´epest a goly´ o t¨ omege elhanyagolhat´ o, ´es a p´ alca s´ url´ od´ asmentesnek tekinthet˝ o asztalon ´ all. A p´ alca egyszer csak eld˝ ol. Mikor csap´ odik a goly´ o nagyobb sebess´eggel az asztallapra, ha a p´ alca tetej´ere van ragasztva, vagy ha egyszer˝ uen csak r´ atett¨ uk a p´ alc´ ara, ahonnan nagyon k¨ onnyen lebillenhet?

ahonnan a k´erd´eses ered˝o ellen´ all´ as RA−E = R

R2 + 2r2 + 4Rr . 4R2 + 2r2 + 6Rr

Yokota Adan (G¨ od¨ oll˝o, T¨or¨ok Ign´ ac Gimn., 12. ´evf.)

(A merev testek forg´ omozg´ as´ ar´ ol r¨ ovid cikk olvashat´ o a K¨ oMaL honlapj´ an.∗ ) (4 pont)

47 dolgozat ´erkezett. Helyes 14 megold´ as. Kicsit hi´ anyos (3 pont) 13, hi´ anyos (1–2 pont) 6, hib´ as 11, nem versenyszer˝ u 3 dolgozat.



303

304

K¨ oz´ episkolai Matematikai ´ es Fizikai Lapok, 2022/5

5,6 − 2,5 · (3 min) = 1,85 min. 7,5 − 2,5

Megjegyz´es. L´ athat´ o, hogy szalag fel´enek ´ attekercsel´esi ideje nem a teljes tekercsel´esi id˝ o fele, hanem ann´ al kicsit t¨ obb.

Hasonl´oan kapjuk, hogy az A ´es az E pontok k¨oz¨otti ered˝ o ellen´ all´asra ´erv´enyes: 1

r12 + r22 = 5,6 cm. 2

ahonnan a keresett id˝ o:

2 3R + 2r 1 + = , (r/2 + r/2 + R) R R(R + r)

RBD−HF =



K¨ ozli: Honyek Gyula, Veresegyh´ az

https://www.komal.hu/cikkek/cikklista.h.shtml.

K¨ oz´ episkolai Matematikai ´ es Fizikai Lapok, 2022/5

Megold´as. I. eset. A goly´ o r¨ ogz´ıtve van a p´ alca v´eg´ehez. A p´ alca ´es a goly´ o tekinthet˝ o egyetlen merev testnek. Mivel a goly´ o t¨omege elhanyagolhat´o, a teljes tehetetlens´egi nyomat´ek megegyezik az m t¨omeg˝ u,  hossz´ us´ag´ u homog´en p´ alca saj´ at (a t¨ omegk¨ oz´eppontj´ara vonatkoztatott) tehetetlens´egi nyomat´ek´ aval: (1)

Θ=

mg

1  1 = mv 2 + Θω 2 . 2 2 2

 v − ω = 0. 2

Q

Megjegyz´es. A h˝ ovezet´esi t¨ orv´eny alakilag nagyon hasonl´ıt az elektromos vezet˝ okre alkalmazott Ohm-t¨ orv´enyre. A h˝ o´ aram megfelel˝ oje az elektromos ´ aramer˝ oss´eg, a h˝ om´ermennyis´egnek pedig s´eklet-k¨ ul¨ onbs´eg megfelel˝ oje a potenci´ alk¨ ul¨ onbs´eg (fesz¨ ults´eg), az Aκ d az elektromos ellen´ all´ as reciproka (a vezet˝ ok´epess´eg) felel meg.

(1)

u ¨ veg

(2)

IQ

(1) IQ

II. eset. Az m0 t¨ omeg˝ u goly´ o nincs r¨ ogz´ıtve a p´ alc´ ahoz, ez´ert annak megmozdul´ asa ut´ an leesik a p´alc´ ar´ ol ´es  magass´ agb´ ol szabadon esik. A becsap´od´as all: v2 sebess´eg´ere fenn´ v2 =

leveg˝ o

A k´erd´eses h´ anyados:

  v1 = v + ω = ω = 3g. 2



κu¨veg . du¨veg

A k´etr´eteg˝ u” ablak (ami az u oz¨ otti leveg˝ovel egy¨ utt tulajdonk´eppen h´ a¨vegek k¨ ” romr´eteg˝ u) h´arom sorosan kapcsolt ellen´ all´ asnak” is tekinthet˝ o, ´es ´ıgy ” 1 (2) . IQ = AΔT 2d dleveg˝o u ¨ veg + κ κ

3g . ω=  A goly´ o becsap´ od´ asi sebess´ege a p´ alca v´egpontj´ anak sebess´eg´evel egyezik meg:

azaz

Aκ , d

ahol IQ = t az id˝oegys´egenk´ent ´ atadott h˝ o, ΔT = Tszoba − Tk¨uls˝o a h˝ om´ers´ekletk¨ ul¨onbs´eg, A a fel¨ ulet nagys´ aga, d a r´eteg vastags´ aga, κ pedig a h˝ o´ atad´ ast biztos´ıt´ o anyag h˝ ovezet´esi t´enyez˝ oje.

IQ = AΔT



1 m0 g = m0 v22 , 2

IQ = ΔT

Az egyr´eteg˝ u ablak eset´eben a h˝ o´ aram

Az (1), (2) ´es (3) egyenletekb˝ol kapjuk, hogy  2 ω 1 1 1  m2 ω 2 , + · mg = m 2 2 2 2 12   1 1 g = ω2  + , 4 12

teh´ at

P´eldat´ ari feladat nyom´an

Megold´as. A h˝ ovezet´es t¨ orv´enye szerint

Tudjuk m´eg, hogy a p´ alca elcs´ usz´ o v´eg´enek f¨ ugg˝oleges ir´ any´ u sebess´ege mindv´egig nulla, teh´ at a p´ alca becsap´ od´ as´anak pillanat´aban is (3)

(4 pont)

1 m2 . 12

A s´ url´od´ asmentess´egnek k¨ osz¨ onhet˝ oen a p´ alca t¨omegk¨oz´eppontja f¨ ugg˝olegesen lefel´e mozog, a p´alca als´ o v´egpontja pedig v´ızszintesen elcs´ uszik. Ha a becsap´ od´as pillanat´ aban a t¨ omegk¨ oz´eppont sebess´ege v, a p´ alca sz¨ogsebess´ege pedig ω, akkor – az energiamegmarad´ as t¨ orv´enye szerint – fenn´ all: (2)

P. 5385. H´ anyadr´esz´ere cs¨ okken az ablakon kisz¨ ok˝ o h˝ o´ aram, ha az egyr´eteg˝ u, ol k´esz¨ ult ablakot ugyanilyen u abl´ ab´ ol k´esz¨ ult, k´etr´etedu¨veg = 3 mm vastag u ¨vegb˝ ¨vegt´ or´es van? A leveg˝ u ablakra cser´elj¨ uk, melynek u oz¨ ott dleveg˝o = 7 mm-es leveg˝ ¨vegei k¨ g˝ o ´es az u ovezet´esi t´enyez˝ oje κleveg˝o = 0,025 W/(m K) ´es κu¨veg = 1,2 W/(m K). ¨veg h˝

2+

dleveg˝o κu¨veg du¨veg κleveg˝o

=

1 2+

7 3

·

1,2 0,025

=

1 1 = . 2 + 112 114

A k´etr´eteg˝ u ablak teh´at nem k´etszer, hanem 114-szer jobb h˝ oszigetel˝ o, mint az egyr´eteg˝ u. Elekes Dorottya (Budapest-Fasori Evang´elikus Gimn., 9. ´evf.) dolgozata alapj´an

2g.

L´athat´ o, hogy v1 > v2 , teh´ at az els˝ o esetben csap´ odik a goly´ o nagyobb sebess´eggel az asztallapra.

1

=

36 dolgozat ´erkezett. Helyes 15 megold´ as. Kicsit hi´ anyos (3 pont) 7, hi´ anyos (1–2 pont) 10, hib´ as 3, nem versenyszer˝ u 1 dolgozat.

Kov´ acs Kinga (Kecskem´et, Katona J. Gimn., 11. ´evf.)

ar´ asi fesz¨ ults´eg˝ u, Rb bels˝ P. 5387. Egy U0 u o ellen´ all´ as´ u telepre k¨ ul¨ onb¨ oz˝ o ¨resj´ R nagys´ ag´ u k¨ uls˝ o ellen´ all´ asokat kapcsolunk.

42 dolgozat ´erkezett. Helyes 22 megold´ as. Kicsit hi´ anyos (3 pont) 8, hi´ anyos (1–2 pont) 8, hib´ as 2, nem versenyszer˝ u 2 dolgozat.

a) Mekkora maxim´ alis hasznos” (a k¨ uls˝ o ellen´ all´ asra jut´ o) teljes´ıtm´enyt ny´ ujt” hat ez a telep? Milyen R eset´en ´erhetj¨ uk el a legnagyobb Pmax teljes´ıtm´enyt?

K¨ oz´ episkolai Matematikai ´ es Fizikai Lapok, 2022/5

305

306

K¨ oz´ episkolai Matematikai ´ es Fizikai Lapok, 2022/5

b) Mutassuk meg, hogy b´ armely, Pmax -n´ al kisebb P hasznos teljes´ıtm´eny k´et ag´ u k¨ uls˝ o ellen´ all´ as eset´en is megval´ osulhat. Mekkora k¨ ul¨ onb¨ oz˝ o, R1 = R2 nagys´ amtani, illetve m´ertani k¨ oz´ep´ert´eke? az R1 ´es R2 sz´ c) Mekkora a fenti k´et esetben m´erhet˝ o kapocsfesz¨ ults´egek ¨ osszege? d) Mekkora az R1 , illetve R2 ellen´ all´ ason foly´ o´ aramok o sszege? ¨ e) Az energialead´ as hat´ asfok´ at a hasznos teljes´ıtm´eny ´es a telep ´ altal leadott osszes teljes´ıtm´eny h´ anyadosak´ent ´ertelmezz¨ uk. Mekkora a fenti k´et eset hat´ as¨ fok´ anak o ¨sszege? (4 pont)

K¨ozli: Siposs Andr´ as, Budapest

Megold´as. a) Bel´atjuk, hogy a hasznos teljes´ıtm´eny akkor a legnagyobb, amikor a k¨ uls˝ o ellen´ all´ asra jut´ o fesz¨ ults´eg (a kapocsfesz¨ ults´eg) megegyezik a bels˝o ellen´ all´as fesz¨ ults´eg´evel, vagyis Uk = Ub = U0 /2. ´ azoljuk az Uk kapocsfest¨ Abr´ ults´eget a terhel˝o a´ram (I) f¨ uggv´eny´eben! Mivel Uk = U0 − IRb ,

azaz

I=

teh´at

U02 = Pmax . 4Rb Az egyenl˝os´eg akkor teljes¨ ul, ha Uk = U0 − Uk , vagyis ha a terhel˝ o ellen´ all´ asra jut´ o kapocsfesz¨ ults´eg megegyezik a bels˝o ellen´ all´ asra jut´ o fesz¨ ults´eggel, teh´ at ha IR = IRb , azaz R = Rb . ´ azoljuk a hasznos teljes´ıtm´enyt a kapocsfesz¨ b) Abr´ ults´eg f¨ uggv´eny´eben. Az (1) ugg´es szerint ¨osszef¨ 1 2 U0 Uk P =− U + , Rb k Rb aminek grafikonja egy lefel´e nyitott parabola Uk = U0 ´es Uk = 0 z´erushelyekkel (2. ´ abra). P 

U0 − Uk , Rb

a f¨ uggv´eny grafikonja egy egyenes (az u ´n. munkaegyenes), amelynek tengelymetU0 , vagyis rendre az u asi fesz¨ ults´eg ´es a r¨ovidz´ ar´ asi a´ram szetei: U0 ´es Imax = R ¨resj´ar´ b (1. ´ abra). 2. ´ abra

L´athatjuk, hogy b´armely – a maxim´alisn´ al kisebb – hasznos teljes´ıtm´eny k´et k¨ ul¨onb¨oz˝ o kapocsfesz¨ ults´eg eset´en is megval´ osulhat. Jel¨ olj¨ uk ezt a k´et fesz¨ ults´eget at) k¨ ul¨ onb¨ oz˝ o I1 ´es I2 ´ aramer˝ oss´eg Uk,1 -gyel ´es Uk,2 -vel. Ezekhez (l´asd az 1. a´br´ ul¨ onb¨ oz˝ oek lesznek, hiszen tartozik. A megfelel˝ o terhel˝o ellen´ all´ asok (R1 ´es R2 ) is k¨ P = I12 R1 = I22 R2 miatt  2 I2 R1 = = 1. R2 I1 A terhel˝o ellen´ all´ as ´es a teljes´ıtm´eny k¨ oz¨ otti kapcsolat: 2  U0 R, P = R + Rb

1. ´ abra

A k¨ uls˝ o ellen´ all´ asra jut´ o teljes´ıtm´eny ´altal´ anos esetben: (1)

P = IUk =

Uk (U0 − Uk ) , Rb

amit algebrai ´ atalak´ıt´ asok ut´an ´ıgy is fel´ırhatunk:  2  U0 R2 − − 2Rb R + Rb2 = 0. P

ami az ´abr´ an l´athat´ o t´eglalap ter¨ ulet´evel egyezik meg. Mivel Rb adott ´ert´ek, a hasznos teljes´ıtm´eny akkor a legnagyobb, amikor amtani k¨ oz´ep ´es a m´ertani k¨oz´ep k¨oz¨otti ¨osszef¨ ugg´es Uk (U0 − Uk ) maxim´alis. A sz´ szerint  U0 Uk + (U0 − Uk ) = , Uk (U0 − Uk )  2 2 K¨ oz´ episkolai Matematikai ´ es Fizikai Lapok, 2022/5

307

Adott P eset´en a fenti egyenlet R-re n´ezve m´ asodfok´ u, melynek gy¨ okei R1 ´es R2 . A gy¨ok¨ok ´es az egy¨ utthat´ ok k¨ oz¨ otti ¨ osszef¨ ugg´es szerint R1 + R2 = 308

U02 − 2Rb , P

illetve

R1 R2 = Rb2 ,

K¨ oz´ episkolai Matematikai ´ es Fizikai Lapok, 2022/5

vagyis a k´et esetben m´erhet˝ o kapocsfesz¨ ults´egek ¨ osszege ´eppen az U0 u ¨resj´arati fesz¨ ults´eggel egyezik meg. d) Ugyancsak a 4. ´abr´ ar´ ol olvashatjuk le, hogy

vagyis a k´et k¨ ul¨ onb¨ oz˝ o terhel˝o ellen´ all´ as sz´amtani k¨oz´ep´ert´eke A=

U2 R1 + R2 = 0 − Rb , 2 2P

I1 I2 + = 1, Imax Imax

a m´ertani k¨ oz´ep´ert´ek¨ uk pedig G=



R1 R2 = Rb .

teh´at az ugyanakkora hasznos teljes´ıtm´enyhez tartoz´ o´ aramer˝ oss´egek ¨ osszege a r¨ ovidz´ar´ asi a´rammal egyenl˝ o:

Az ismert A  G egyenl˝ otlens´eg szerint U02 − Rb  Rb , 2P

azaz

I1 + I2 = Imax =

U2 P  0 = Pmax , 4Rb

ahogy ezt m´ar kor´ abban bel´ attuk. c) T´erj¨ unk vissza a kapocsfesz¨ ults´eg ´es az a´ramer˝ oss´eg 1. a´br´ an l´athat´ o kapcsolat´ahoz. Ha adott U0 ´es Rb mellett k´etf´ele m´odon is megval´osul a hasznos teljes´ıtm´enyek egyenl˝ os´ege, akkor a 3. ´ abr´ an l´ athat´ o OABC ´es OA B  C  t´eglalapok ter¨ ulete megegyezik.

U0 . Rb

e) A hasznos teljes´ıtm´eny ´es a telep a´ltal leadott teljes´ıtm´eny h´ anyadosa (a hat´asfok): Uk IUk = . η= IU0 U0 A k´et eset hat´ asfok´ anak ¨ osszege (a c) k´erd´esre adott v´ alaszt felhaszn´ alva): η1 + η2 =

Uk,1 Uk,2 + = 1. U0 U0

Schmercz Blanka (Budapest, ELTE Ap´ aczai Csere J. Gyak. Gimn., 11. ´evf.) dolgozat´anak felhaszn´al´as´ aval 26 dolgozat ´erkezett. Helyes 19 megold´ as. Hi´ anyos (2–3 pont) 5, hib´ as 2 dolgozat.

P. 5388. Egy 15 mW-os l´ezer λ = 632,8 nm hull´ amhossz´ u, line´ arisan polariz´ alt f´enye egy 2 mm ´ atm´er˝ oj˝ u k¨ ork¨ or¨ os apert´ ur´ an l´ep ki a l´ezer doboz´ ab´ ol. a) Mekkora az elektromos t´erer˝ oss´eg maxim´ alis ´ert´eke a l´ezernyal´ abban? b) Mekkora az impulzusa a l´ezernyal´ ab 1 m´eter hossz´ u darabj´ anak? (4 pont)

P´eldat´ ari feladat nyom´an

Megold´as. a) A l´ezernyal´ab s = 1 m hossz´ us´ ag´ u utat 3. a ´bra

4. a ´bra

Ha mind az ´aramer˝ oss´eget, mind pedig a kapocsfesz¨ ults´eget elosztjuk a maxim´ alis ´ert´ek¨ ukkel, ´es az ezek k¨ oz¨ otti kapcsolatot a´br´ azoljuk (4. ´ abra), akkor a munkaegyenes meredeks´ege −1 lesz, mik¨ ozben a k´et t´eglalap ter¨ ulete tov´ abbra is egyenl˝o marad, hiszen Uk,1 I1 Uk,2 I2 P · = · = . U0 Imax U0 Imax U0 Imax A 4. a´bra szimmetri´ aj´ab´ ol k¨ ovetkezik, hogy az egyforma ter¨ ulet˝ u OP QR ´es az OP  Q R t´eglalap egybev´ ag´ o, egym´asb´ ol 90◦ -os elforgat´ assal kaphat´ o meg. Ezek szerint Uk,1 Uk,2 + = 1, U0 U0 K¨ oz´ episkolai Matematikai ´ es Fizikai Lapok, 2022/5

309

t=

1m s = = 3,33 · 10−9 s c 3 · 108 m/s

id˝ o alatt tesz meg. Ennyi id˝ o alatt a P teljes´ıtm´eny˝ u l´ezer     W = P t = 15 · 10−3 W · 3,33 · 10−9 s = 5 · 10−11 J

elektrom´ agneses energi´ at bocs´at ki. A nyal´ab 1 m´eter hossz´ u, 3,14 mm2 kereszt−6 3 metszet˝ u darabj´anak t´erfogata V0 = 3,14 · 10 m . A l´ezernyal´ab energias˝ ur˝ us´ege (t´erfogategys´egre jut´ o energi´ aja) w= 310

W J 5 · 10−11 J = = 1,59 · 10−5 3 . −6 3 V0 3,14 · 10 m m K¨ oz´ episkolai Matematikai ´ es Fizikai Lapok, 2022/5

Az energias˝ ur˝ us´eg kifejezhet˝ o a line´arisan polariz´ alt f´eny elektromos t´erer˝ oss´eg´enek maxim´alis ´ert´ek´evel is: 1 2 . w = ε0 Emax 2

P. 5389. Egy (pontszer˝ unek tekinthet˝ o) l´egy rep¨ ul a ´lland´ o v sebess´eggel az f f´ okuszt´ avols´ ag´ u lencse optikai tengely´evel p´ arhuzamosan, att´ ol d t´ avols´ agra. Legal´ abb mekkora nagys´ ag´ u a l´egy ´es a l´egy k´ep´enek relat´ıv sebess´ege? ´ Esztorsz´ agi versenyfeladat nyom´ an

(5 pont)

Megjegyz´es. Az elektrom´ agneses s´ıkhull´ am energias˝ ur˝ us´ege egy adott r helyen w(r) =

Megold´as. T¨ obbf´ele lehet˝ os´eget fogunk vizsg´alni. Kezdj¨ uk azzal, amikor gy˝ ujt˝ olencs´enk van, ´es a l´egy (L) a lencse B pontj´ at´ ol indulva v sebess´eggel rep¨ ul jobbra (1. a ´bra). Ameddig a l´egy a f´ okuszt´ avols´ agn´ al k¨ ozelebb van m´eg a lencs´ehez, az A k´ep l´ atsz´ olagos (virtu´ alis), ´es az F  B egyenesen mozog ugyancsak jobb fel´e. Jel¨ olj¨ uk az A pont sebess´eg´enek az optikai tengellyel p´ arhuzamos komponens´et u-val. Mivel az F  B egyenes meredeks´ege

1 1 ε0 E 2 (r) + ε0 c2 B 2 (r) = ε0 E 2 (r). 2 2

A line´ arisan polariz´ alt s´ıkhull´ amban az elektromos t´erer˝ oss´eg E(r) = Emax · sin

2π x λ

m´ odon v´ altozik, ´ıgy az ´ atlagos ´ert´eke:   2π 1 2 2 w´atlag = ε0 Emax · sin2 = ε0 Emax . x λ ´atlag 2 Ha a l´ezerf´eny cirkul´ arisan pol´ aros, abban az elektromos t´erer˝ oss´eg nagys´ aga a nyal´ ab ment´en nem v´ altozik, ´ıgy az energias˝ ur˝ us´eg a fenti ´ert´ek k´etszerese lenne.

A nyal´ abban az elektromos t´erer˝ oss´eg maxim´ alis ´ert´eke a fentiek szerint:  V 2w N Emax = = 1,9 . = 1,9 · 103 ε0 C mm

Efoton

=

v rel = v − v A = (v − u, −u tg α). Keress¨ uk azt az u-t, amire a relat´ıv sebess´eg nagys´ aga, vagyis  2 2 vrel = |v rel | = (v − u) + (u tg α)

a lehet˝o legkisebb. Ezt deriv´ al´assal k¨ onnyen meg lehet kapni, de elemi m´ odszerekkel is c´elhoz ´erhet¨ unk, ha vrel helyett

5 · 10−11 J = 1,59 · 108 . 3,14 · 10−19 J

2

kg m h = 1,05 · 10−27 λ s impulzussal rendelkezik, ´ıgy a nyal´ ab 1 m´eter hossz´ u darabj´anak ¨osszimpulzusa: Pfoton =

kg m . s

´ Isk., 11. ´evf.) ´es Albert M´ at´e (Szeged, SZTE Gyak. Gimn. ´es Alt. Szab´ o M´ arton (Szeghalom, P´eter Andr´ as Gimn. ´es Koll., 11. ´evf.) dolgozata alapj´an 13 dolgozat ´erkezett. Helyes 6 megold´ as. Kicsit hi´ anyos (3 pont) 2, hi´ anyos (1–2 pont) 4, hib´ as 1 dolgozat.

K¨ oz´ episkolai Matematikai ´ es Fizikai Lapok, 2022/5

2

2 = (v − u) + (u tg α) ≡ (1 + tg2 α)u2 − 2vu + v 2 vrel

Minden foton

P¨osszes = n · Pfoton = 1,66 · 10−19

d BC = , F C f

A l´egy sebess´egvektora: v = (v, 0), a k´ep´e pedig v A = (u, u tg α). A l´egy ´es a l´egy k´ep´enek relat´ıv sebess´ege

A nyal´ ab 1 m´eter hossz´ u darabj´aban l´ev˝ o fotonok sz´ama: W

tg α =

az A pont sebess´egvektor´ anak az optikai tengelyre mer˝ oleges ir´ any´ u komponense u tg α.

b) A l´ezerf´eny fotonjainak energi´aja:    6,63 · 10−34 Js 3 · 108 m/s hc = = 3,14 · 10−19 J. Efoton = λ 632,8 · 10−9 m

n=

1. ´ abra

311

legkisebb ´ert´ek´et keress¨ uk meg. Ez u-nak m´asodfok´ u kifejez´ese, aminek minimuma teljes n´egyzett´e alak´ıt´ assal is meghat´ arozhat´ o. Mivel tg α = d/f , ezt kapjuk:    2 2 f 2 + d2 vf d2 d 2 2 − +v 2  v , vrel = u f f + d2 f 2 + d2 f 2 + d2  vagyis a l´egynek ´es a virtu´ alis k´ep´enek relat´ıv sebess´ege legal´abb vd/ f 2 + d2 .

Amikor a l´egy m´ ar t´ avolabb ker¨ ult a lencs´et˝ ol, mint annak f´ okuszt´ avols´ aga, az A k´ep val´odiv´ a v´ alik (2. ´ abra). Az A pont sebess´eg´et most is (u, u tg α) alakban ´ırhatjuk fel, ´eppen u ´gy, mint a l´ atsz´ olagos k´ep eset´eben. A relat´ıv sebess´eget ´es annak legkisebb ´ert´ek´et is ugyanazok az ¨ osszef¨ arozz´ ak meg, ´es ´ıgy ebben ugg´esek hat´ a tartom´ anyban is vrel legkisebb ´ert´eke vd/ f 2 + d2 . 312

K¨ oz´ episkolai Matematikai ´ es Fizikai Lapok, 2022/5

G. 781. Forraljunk vizet egy nagy l´abosban a t˝ uzhelyen. Tegy¨ unk egy v´ekonyfal´ u poh´ arba csapvizet, majd mer´ıts¨ uk a forr´ asban l´ev˝ o v´ızbe u ´gy, hogy az sehol se ´erintkezzen a l´abos fal´ aval. Felforr-e a poh´ arban a v´ız, ha elegend˝ oen hossz´ u ideig v´arunk? (3 pont)

2. ´ abra

Vizsg´aljuk meg most a sz´ or´olencse eset´et (3. ´ abra)! A l´egy k´epe – a l´egy helyzet´et˝ ol f¨ uggetlen¨ ul – mindig l´atsz´ olagos, ´es a sebess´egvektora ´ıgy adhat´ o meg: v A = (u, −u tg α),

(4 pont)

Vermes Mikl´ os feladata nyom´ an

(3 pont)

v rel = v − v A = (v − u, u tg α),

vrel

(A hodogr´ afr´ ol r¨ ovid cikk olvashat´ o a K¨ oMaL honlapj´an.∗ )

G. 783. Egy homog´en, n t¨ or´esmutat´ oj´ u, R sugar´ uu omb k¨ oz´eppontj´aban ¨vegg¨ pontszer˝ u f´enyforr´ as helyezkedik el. A g¨ omb¨ ot k´ıv¨ ulr˝ ol n´ezz¨ uk. Hol l´ atjuk a f´enyforr´ as k´ep´et?

ha v = (v, 0).

A relat´ıv sebess´eg ebben az esetben

3. ´ abra

G. 782. Egy ker´ekp´ ar egyenletesen, 3 m/s sebess´eggel halad v´ızszintes u ´ton. ´ azoljuk a ker´ek k¨ Kerekeinek a´tm´er˝ oje 70 cm. Abr´ ul¨ onb¨ oz˝ o helyzeteiben az egyik ker¨ uleti pont sebess´egvektorait ´es gyorsul´asvektorait egy-egy k¨ oz¨ os pontb´ ol indulva, azaz k´esz´ıts¨ uk el a sebess´eg- ´es gyorsul´ashodogr´ afokat.

G. 784. Az al´ abbi ´ abr´ an egyszer˝ u g´epek kavalk´ adj´ at l´athatjuk. A s´ url´od´as, valamint a csig´ak ´es emel˝ ok t¨ omege elhanyagolhat´ o. Melyik ir´anyba indul el a legals´ o test?

aminek abszol´ ut ´ert´eke:  2 2 = (v − u) + (u tg α) .

Ennek a minimum´at keress¨ uk u f¨ uggv´eny´eben. L´ atjuk, hogy ez a kifejez´es ugyanaz, mint amit agy˝ ujt˝ olencs´en´el kaptunk, ´ıgy a legkisebb ´ert´eke is ugyanakkora, nevezetesen vd/ f 2 + d2 . ´ Isk. ´es Gimn., 11. ´evf.) G´ abriel Tam´ as (Budapesti Fazekas M. Gyak. Alt.

(4 pont)

13 dolgozat ´erkezett. Helyes Antal´ oczy Szabolcs, Biebel Botond, G´ abriel Tam´ as, Kert´esz Bal´ azs ´es T´egl´ as Panna megold´ asa. Kicsit hi´ anyos (4 pont) 1, hi´ anyos (2–3 pont) 6, nem versenyszer˝ u 1 dolgozat.

Fizik´ab´ ol kit˝ u z¨ ott feladatok P. 5409. A fenti ´ abr´ an egyszer˝ u g´epek kavalk´ adj´ at l´athatjuk. A s´ url´od´ as, valamint a csig´ak ´es emel˝ ok t¨ omege elhanyagolhat´ o. Mekkora er˝ o ´ebred a fonalakban? M. 414. M´erj¨ uk meg a cs´ usz´ asi s´ url´od´ asi egy¨ utthat´ o ´ert´ek´et t¨obb, k¨ ul¨onb¨oz˝ o finoms´ ag´ u csiszol´ opap´ır ´es egy fahas´ ab k¨ oz¨ ott! (6 pont) K¨ oz´ episkolai Matematikai ´ es Fizikai Lapok, 2022/5

(4 pont)

K¨ozli: Vigh M´ at´e, Biatorb´ agy



313

314

Holics L´ aszl´ o feladata nyom´ an

https://www.komal.hu/cikkek/cikklista.h.shtml

K¨ oz´ episkolai Matematikai ´ es Fizikai Lapok, 2022/5

P. 5410. A v´ andors´ olyom sz´ arnycsap´ asok n´elk¨ ul is k´epes megtenni nagyobb t´avols´ agokat. Ilyenkor a mozg´ asa k´et r´eszb˝ ol a´ll. Az els˝ o r´eszben kiterjesztett sz´arnyakkal k¨or¨ ozve emelkedik egy f¨ olfel´e ´araml´ o meleg leveg˝ooszlopugg˝oleges sebess´egban (termikben) v1 f¨ gel. A m´ asodik r´eszben a termiket elhagyva a v´ızszintessel α sz¨ oget bez´ arva alland´ ´ o sebess´eggel siklik a k¨ ovetkez˝ o, L t´avols´ agra l´ev˝ o termikig. A v2 sikl´asi sebess´eg j´ o k¨ ozel´ıt´essel egyenesen ar´anyos a sikl´ as v´ızszintessel bez´art α sz¨ og´enek szinusz´aval: v2 = k sin α, ahol k egy ismert a´lland´ o. a) Legal´ abb milyen magasra kell a mad´ arnak emelkednie a termikben, hogy egy emelked´esb˝ ol ´es egy sikl´ asb´ ol ´ all´ o mozg´as a legr¨ovidebb ideig tartson? b) Legal´ abb mennyi id˝ ore van sz¨ uks´ege a v´ andors´olyomnak, hogy az egyik termik alj´at´ ol eljuthasson a m´asik termik alj´ aig? c) Hat´ arozzuk meg az optim´alis menetidej˝ u mozg´ashoz tartoz´ o sikl´asi sz¨oget! m Adatok: v1 = 2 m , k = 10 , L = 2 km. s s (5 pont)

(L´asd m´eg A g˝ oz, g´ az ´es a kritikus h˝ om´ers´eklet” c. r¨ ovid cikket a K¨ oMaL ” honlapj´an∗ .) (3 pont)

K¨ozli: Simon P´eter, P´ecs

P. 5411. A F¨ old k¨ or¨ ul egy m˝ uhold c/a = e numerikus excentricit´ as´ u ellipszisp´ aly´ an kering, kering´esi ideje T . Mennyi id˝ o alatt ´er a m˝ uhold az ´ abr´ an jel¨ olt A pontb´ ol a B pontba?

P. 5413. Egy 20 cm f´ okuszt´ avols´ ag´ u gy˝ ujt˝olencs´et az ´ abra szerint egy dombor´ u g¨ ombt¨ uk¨ orre helyez¨ unk. Mekkora legyen a t¨ uk¨ or g¨ orb¨ uleti sugara, hogy a lencs´ere f¨ ugg˝olegesen ´erkez˝ o, p´ arhuzamos f´enynyal´ ab a rendszerr˝ol val´o visszaver˝od´es ut´ an is p´ arhuzamos maradjon? (4 pont)

(4 pont) K¨ ozli: Sz´ asz Kriszti´ an, Budapest

P´eldat´ ari feladat nyom´ an

P. 5414. F´emdr´ otb´ ol egy R sugar´ u k¨ ort form´aztunk, ´es ugyanebb˝ ol a dr´ otb´ ol az egyik ´atm´er˝ ot is elk´esz´ıtett¨ uk. Mekkora legyen az AB = AC ´ıvek hossza, hogy az A ´es B pontok k¨oz¨ ott m´erhet˝ o ered˝ o ellen´ all´ as megegyezzen oz¨ ott m´erhet˝ o ered˝ o ellen´ all´ assal? a B ´es C pontok k¨

P. 5412. Ha egy g´azt (´ alland´ o nyom´ as mellett) leh˝ ut¨ unk, akkor elegend˝ oen alacsony h˝om´ers´ekleten a g´ az ´ altal´ aban cseppfoly´ osodik (kondenz´al´odik, lecsap´odik). Ez azonban csak bizonyos nyom´ astartom´ anyban t¨ ort´enik ´ıgy. Az ´ abra a sz´en-dioxid f´azisdiagramj´ at” mutatja. Legal´ abb, illetve legfeljebb mekkora nyom´as mellett t¨or” t´enik meg a cseppfoly´ osod´ as a fenti m´odon? Mi t¨ ort´enik, ha a h˝ ut´est enn´el a tartom´ anyn´al magasabb, illetve alacsonyabb nyom´ ason v´egezz¨ uk?

(4 pont)



K¨ oz´ episkolai Matematikai ´ es Fizikai Lapok, 2022/5

K¨ ozli: Honyek Gyula, Veresegyh´ az

315

316

K¨ ozli: G´ asp´ ar Merse El˝ od, Budapest

https://www.komal.hu/cikkek/cikklista.h.shtml

K¨ oz´ episkolai Matematikai ´ es Fizikai Lapok, 2022/5

MATHEMATICAL AND PHYSICAL JOURNAL FOR SECONDARY SCHOOLS (Volume 72. No. 5. May 2022)

P. 5415. Egy elhanyagolhat´ o ellen´ all´ as´ u, szigetel´es n´elk¨ uli huzalb´ ol, a v´ızszintes s´ıkban elhelyezked˝ o, α = 45◦ -os sz¨oget bez´ ar´ o, V alakot hajl´ıtunk. Ezt az elrendez´est olyan m´agneses mez˝ obe helyezz¨ uk, melynek B indukci´ ovektora mer˝ oleges uga v´ızszintes s´ıkra, ´es nagys´ aga a B(t) = B0 /t0 · t ¨osszef¨ ok. g´es szerint v´ altozik az id˝ oben, ahol B0 ´es t0 ismert a´lland´ A V alak´ u vezet˝ ore szigetel´es n´elk¨ uli, kezdetben r¨ ogz´ıtett f´emrudat helyez¨ unk az ´ abr´ anak megfelel˝o m´ odon. A r´ ud egys´egnyi hossz´ us´ag´ u darabj´anak ellen´all´ asa r.

Problems in Mathematics

a) Mennyi h˝ o fejl˝ odik a f´emr´ udban t0 id˝ o alatt? b) A bekapcsol´ ast´ ol (t = 0 id˝ opillanat) sz´ am´ıtott t0 id˝ opillanatban a m´agneses indukci´ o v´ altoz´ asa megsz˝ unik. Ebben a pillanatban az eddig r¨ogz´ıtett f´emrudat uk. Meka v´ızszintes s´ıkban, a f´emr´ udra mer˝ olegesen v0 sebess´eggel mozgatni kezdj¨ kora legyen ez a sebess´eg, hogy a r´ udban foly´ o´ aram er˝oss´ege ne v´ altozzon? c) H´anyszor t¨ obb h˝ o fejl˝ odik a f´emr´ udban a mozgat´ as sor´ an, mint a r¨ogz´ıtett u ideig mozgatjuk? helyzetben, ha a f´emrudat 2t0 hossz´ (5 pont)

K¨ozli: Kotek L´ aszl´ o, P´ecs

P. 5416. Egy 1,1 nm hossz´ us´ag´ u, hozz´ a k´epest elhanyagolhat´ o sz´eless´eg˝ u ´es vastags´ ag´ u t´err´eszben ¨ ot elektron van. Ebben a t´err´eszben a potenci´alis energia nulla, ezen k´ıv¨ ul nagyon nagy. (Az elektronok egym´assal val´ o k¨olcs¨onhat´ as´at´ ol eltekinthet¨ unk.) a) Mekkora a rendszer elektronjainak gerjeszt´es´ehez sz¨ uks´eges minim´ alis energia? b) Mekkora hull´ amhossz´ us´ag´ u elektrom´ agneses hull´ am k´epes ezt a gerjeszt´est l´etrehozni? Hol a helye ennek az elektrom´ agneses hull´ amnak a spektrumban? (5 pont)

K¨ozli: Zsigri Ferenc, Budapest

P. 5417. V´ızszintes talajon ´all´ o R sugar´ u, elhanyagolhat´ o t¨omeg˝ u keskeny henunk (de nem er˝ os´ıtj¨ uk geres abroncs tetej´ere kis m´eret˝ u, m t¨ omeg˝ u nehez´eket helyez¨ hozz´ a), ´es a rendszert a labilis egyens´ ulyi helyzet´eb˝ ol kimozd´ıtjuk. Az egyre gyorsabban gurul´o abroncsr´ ol a kis test valahol lerep¨ ul. a) Legal´ abb mekkora az ´erintkez˝o fel¨ uletek k¨ oz¨ott a tapad´ asi s´ url´od´asi egy¨ utthat´ o, ha a mozg´ as sor´ an sem a kis test az abroncson, sem az abroncs a talajon nem cs´ uszik meg? b) Hol fog f¨ oldet ´erni a lerep¨ ul˝ o kis test? (6 pont)

K¨ozli: Balogh P´eter, G¨od¨oll˝o

 Beku unius 15. ¨ld´esi hat´arid˝o: 2022. j´ Elektronikus munkafu ¨zet: https://www.komal.hu/munkafuzet



K¨ oz´ episkolai Matematikai ´ es Fizikai Lapok, 2022/5

New exercises for practice – competition C (see page 289): Exercises up to grade 10: C. 1721. Bonnie listed 2022 numbers such that the ratio of the second number divided by the first number equals the third number on the list, and so on, for example, the seventh number equals the ratio of the sixth number divided by the fifth. What is the last number on Bonnie’s list if the first number is 20, and the second number is 22? C. 1722. In a quadrilateral ABCD, sides AD and DC are equal in length. If α denotes the angle DAB then ∠ABC = 2α, ∠BCD = 3α and ∠CDA = 4α. Prove that side AB is twice as long as side AD. (German competition problem) Exercises for everyone: C. 1723. Determine all at most four-digit numbers abcd of distinct digits (allowing a = 0, too) for which 9 · abcd = acbcd. (Proposed by A. Siposs, Budapest) C. 1724. In a triangle ABC, ∠CAB = 30◦ . Find the measures of the other angles of the triangle, given that the median drawn from vertex C encloses an angle of 45◦ with line AB. C. 1725. Let p denote a positive prime number. Given that the roots of the equation x2 − px − 580p = 0 are integers, find the value of p. (Proposed by M. Szalai, Szeged) Exercises upwards of y x z grade 11: C. 1726. Prove that if x, y, z are real numbers such that y+z + z+x + x+y = 1, x2

y2

z2

then y+z + z+x + x+y = 0. Find all real numbers satisfying this condition. C. 1727. In a solid sphere of radius R, a cylindrical bore of radius r < R is made along a line passing through its centre. Express the volume of the remaining solid in terms of the height m of the remaining solid. (Proposed by B. Szab´ o, Miskolc, 1986) New exercises – competition B (see page 290): B. 5246. There are 14 people sitting around a table. Each of them is wearing either a blue shirt or a yellow shirt. What is the maximum possible number of people who have adjacent neighbors with shirts of different color? (3 points) B. 5247. The ends of a rope are fixed to the ground at two points separated by a distance shorter than the length of the rope. The rope will become taut if its midpoint is raised to a height of 150 cm. The rope will also become taut if a point of the rope 90 cm from one end is raised to a height of 90 cm. How long is the rope? (3 points) B. 5248. Solve the following simultaneous equations over the set x2

y2

8

of real numbers: y + x + x + y = xy , x(x + 1) + y(y + 1) = 6. (4 points) B. 5249. Let T0 denote the area of the triangle formed by the points of tangency of the inscribed circle of triangle ABC on the sides, and let T1 denote the area of the triangle formed by the centres of the escribed circles. Show that the geometric mean of T0 and T1 equals the area of triangle ABC. (5 points) (Proposed by P. B´ artfai) B. 5250. Prove that for n n all non-negative integers n, 22 (n−2)+n+2  (2n )!  22 (n−1)+1 . (5 points) (Proposed by I. Blahota, Ny´ıregyh´ aza) B. 5251. The vertices of a rectangle ABCD in the coordinate plane are A(0, 0), B(2022, 0), C(2022, 2), D(0, 2). Consider those triangles of unit area that have all three vertices at lattice points lying on the longer sides of the rectangle. These triangles are to be coloured so that no triangles of the same colour have an interior point in common. What is the minimum number of colours needed? (5 points) (Proposed by Z. L. Nagy Budapest) B. 5252. A polyhedron ABCA1 B1 C1 has six vertices. Faces ABC and A1 B1 C1 are triangles. The edges AA1 , BB1 and CC1 are parallel. Faces AA1 B1 B, BB1 C1 C and CC1 A1 A are trapeziums in which the diagonals intersect at points P , Q and R, respectively. Show that the volumes of polyhedra ABCP QR and A1 B1 C1 PQR  are equal. (6 points) (Proposed by Sz. Kocsis, Budapest) B. 5253. Is it true that if

317

318

n k

K¨ oz´ episkolai Matematikai ´ es Fizikai Lapok, 2022/5

is even, then the k-element subsets of an n-element set S can be paired up so that the symmetric difference of every pair should have exactly 2 elements? (6 points) New problems – competition A (see page 291): A. 827. Let n > 1 be a given integer. In a deck of cards the cards are of n different suites and n different values, and for each pair of a suite and a value there is exactly one such card. We shuffle the deck and distribute the cards among n players giving each player n cards. The players’ goal is to choose a way to sit down around a round table so that they will be able to do the following: the first player puts down an arbitrary card, and then each consecutive player puts down a card that has a different suite and different value compared to the previous card that was put down on the table. For which n is it possible that the cards were distributed in such a way that the players cannot achieve their goal? (The players work together, and they can see each other’s cards.) (Proposed by Anett Kocsis, Budapest) A. 828. Triangle ABC has incenter I and excircles ΩA , ΩB , and ΩC . Let A be the line through the feet of the tangents from I to ΩA , and define lines B and C similarly. Prove that the orthocenter of the triangle formed by lines A , B , and C coincides with the Nagel point of triangle ABC. (The Nagel point of triangle ABC is the intersection of segments ATA , BTB , and CTC , where TA is the tangency point of ΩA with side BC, and points TB and TC are defined similarly.) (Proposed by Nikolai Beluhov, Bulgaria) A. 829. Let G be a simple graph on n vertices with at least one edge, and let  us consider S(v) = 1. those S : V (G) → R0 weighings of the vertices of the graph for which v∈V (G)

Furthermore define f (G) = max S

min

(v,w)∈E(G)

S(v)S(w), where S runs through all possible

1 weighings. Prove that f (G) = n2 if and only if the vertices of G can be covered with

a disjoint union of edges and odd cycles. (V (G) denotes the vertices of graph G, E(G) denotes the edges of graph G.)

Problems in Physics (see page 313) M. 414. Measure the coefficient of kinetic friction between several sheets of sandpaper with different grit sizes and a wooden block. G. 781. Boil water in a large pot on the stove. Put some cool water in a thin-walled glass, then immerse the glass of water into the boiling water so that it does not touch the walls of the pot. Will the water in the glass boil if we wait for a long enough time? G. 782. A bicycle is moving uniformly along a horizontal path at a speed of 3 m/s. Its wheels have a diameter of 70 cm. Choose an arbitrary point on the circumference of the wheel and at different positions of the wheel draw the velocity vectors and the acceleration vectors of this point starting from one common point for each quantity, that is draw the velocity and acceleration hodographs. G. 783. There is a point-like light source at the centre of a uniform glass ball of radius R and of refractive index n. The sphere is observed from the outside. Where do we see the image of the light source? G. 784. The figure shows a whole range of simple machines. Friction and the masses of pulleys and levels are negligible. Into which direction will the lowermost object start moving? P. 5409. The figure shows a whole range of simple machines. Friction and the masses of pulleys and levels are negligible. What are the values of the tension in the threads? P. 5410. The peregrine falcon can travel long distances without flapping its wing. Doing so, its movement has two parts. In the first part, it circles with its wings extended and rises in an upward flowing column of warm air (thermals) at a vertical speed of v1 . In

K¨ oz´ episkolai Matematikai ´ es Fizikai Lapok, 2022/5

319

the second part, it leaves the thermal at an angle of α with respect to the horizontal and glides at a constant speed to the next thermal at a distance of L. The glide speed v2 is approximately directly proportional to the sine of the angle α (the direction of glide with the horizontal): v2 = k sin α, where k is a known constant. a) To what minimum height must a peregrine rise in the thermal so that the time of its rising and gliding motion should be the shortest possible? b) At least how much time is needed for the peregrine to move from the bottom of a thermal to the bottom of the next thermal? c) Determine the m glide angle which belongs to the motion with the optimal flight time. Data: v1 = 2 s , m

k = 10 s , L = 2 km. P. 5411. A satellite orbits the Earth in an elliptical orbit of numerical eccentricity c/a = e, with a period of T . How long does it take for the satellite to go from point A to point B, shown in the figure? P. 5412. If a gas is cooled (at constant pressure), then at a sufficiently low temperature the gas will usually liquefy (condense). However, this only happens over a certain pressure range. The figure shows the “phase diagram” of carbon dioxide. What are the values of the minimum and the maximum pressure at which this condensation can occur as described above? What happens if cooling is carried out at pressures higher or lower than this range? (Translation of the labels in the figure is as follows: l´egnem˝ u = gas, folyad´ek = liquid, szil´ ard = solid, h´ armaspont = triple point, kritikus pont = critical point, szuperkritikus ´ allapot = supercritical fluid.) P. 5413. A converging lens with a focal length of 20 cm is placed on a convex spherical mirror as shown in the figure. What should the radius of curvature of the mirror be in order that a vertical parallel beam of light incident on the lens remain parallel after reflection from the system? P. 5414. We formed a circle of radius R from a piece of metal wire and from the same wire we made one of the diameters of the circle as well. What should the length of the arcs AB = AC be in order that the equivalent resistance between points A and B be the same as the equivalent resistance between points B and C? P. 5415. From a piece of wire of negligible resistance a V shaped figure was bent. The wire has no insulation, the angle between the two parts of the V is α = 45◦ . It is placed horizontally into a magnetic field whose induction B is perpendicular to the plane of the wire. The magnitude of this induction B changes with time according to B(t) = B0 /t0 · t, where B0 and t0 are known constants. A metal rod, also without insulation, is placed onto the V shaped wire, initially it is fixed, as shown in the figure. The resistance of a unit length of the rod is r. a) How much heat is produced in the metal rod in a time of t0 ? b) At time t0 from the moment of switching on (time t = 0), the change in magnetic induction ceases. At this instant we begin to move the metal rod (which was fixed till this time) in the horizontal plane and perpendicularly to the rod at a constant speed of v0 . What should this speed be in order that the value of the current in the rod should not change? c) By what factor will the heat produced in the moving rod be greater than that produced in a static rod, if the rod is moved for a time of 2t0 ? P. 5416. There are five electrons in a region of length 1.1 nm with respect to which the width and thickness of the region is negligibly small. The potential energy in this region is zero, and outside it is very big. We can neglect the interaction of the electrons with each other. a) What is the minimum energy required to excite the electrons in the system? b) What is the wavelength of the electromagnetic wave that can produce this excitation? Where is this electromagnetic wave in the spectrum? P. 5417. A small body of mass m is placed (but not fixed) onto the top of a thin cylindrical ring of radius R and of negligible mass being at rest on the horizontal ground. The system is displaced from its unstable equilibrium position. As the ring rolls faster and faster, the small body flies off somewhere. a) What is the least value of the coefficient of static friction between the surfaces in contact if neither the small body on the ring nor the ring on the ground is skidding during the motion? b) Where will the small object hit the ground?

72. ´evfolyam 5. sz´am

K¨oMaL

Budapest, 2022. m´ajus

´ ´ FIZIKAI LAPOK ¨ EPISKOLAI KOZ MATEMATIKAI ES ˝ ´ITVE INFORMATIKA ROVATTAL BOV

Besz´amol´ o a 63. Nemzetk¨ozi Matematikai Di´akolimpi´ar´ ol

´ ALAP´ITOTTA: ARANY DANIEL 1894-ben 72. ´evfolyam 6. sz´am

Budapest, 2022. szeptember

´ Megjelenik ´evente 9 sz´amban, janu´art´ol m´ajusig ´es szeptembert˝ol decemberig havonta 64 oldalon. ARA: 1100 Ft

´ TARTALOMJEGYZEK Frenkel P´eter: Besz´amol´o a 63. Nemzetk¨ozi Matematikai Di´akolimpi´ar´ol. . . . . . . . . . . . . . . . . . . . . . . .

322

A 63. Nemzetk¨ozi Matematikai Di´akolimpia feladatai . . . . . . . . . . . . . . . . . . . . . . . . . . . . . . . . . . . . . . . . Olimpiai el˝ok´esz´ıt˝o szakk¨or¨ok . . . . . . . . . . . . . . . . . . . .

323 324

Kiss Melinda Fl´ora, Baran Zsuzsa: EGMO 2022/2023 felh´ıv´as . . . . . . . . . . . . . . . . . . . . . . . . . . . .

325

T´othm´er´esz Lilla: N´egysz´ın-sejt´es I: A sejt´es sz¨ulet´ese ´es egy bizony´ıt´asi k´ıs´erlet. . . . . . . . . . . . . . . .

326

Erd˝os G´abor: Gyakorl´o feladatsor emelt szint˝u matematika ´eretts´egire . . . . . . . . . . . . . . . . . . . . . . . . T´aj´ekoztat´o a foly´oirat el˝ofizet´es´er˝ol . . . . . . . . . . . . . Versenyki´ır´as a K¨oMaL pontversenyeire . . . . . . . . . . Matematika feladatok megold´asa (5220., 5231.) . . .

332 334 334 345

A K pontversenyben kit˝uz¨ott gyakorlatok (729– 733.) . . . . . . . . . . . . . . . . . . . . . . . . . . . . . . . . . . . . . . . . .

351

A C pontversenyben kit˝uz¨ott gyakorlatok (732– 733., 1728–1732.) . . . . . . . . . . . . . . . . . . . . . . . . . . . . . A 2021–2022-es pontversenyek v´egeredm´enye . . . . .

352 I

A B pontversenyben kit˝uz¨ott feladatok (5254– 5261.) . . . . . . . . . . . . . . . . . . . . . . . . . . . . . . . . . . . . . . . . K¨ursch´ak-verseny . . . . . . . . . . . . . . . . . . . . . . . . . . . . . . .

353 354

Az A pontversenyben kit˝uz¨ott nehezebb feladatok (830–832.) . . . . . . . . . . . . . . . . . . . . . . . . . . . . . . . . . . . .

354

Informatik´ab´ol kit˝uz¨ott feladatok (568–570., 64., 163.) . . . . . . . . . . . . . . . . . . . . . . . . . . . . . . . . . . . . . . . . .

355

Sarkadi Tam´as, Tasn´adi Tam´as: Sz´ep szerepl´es az 52. Nemzetk¨ozi Fizikai Di´akolimpi´an . . . . . . . .

361

A Nemzetk¨ozi Csillag´aszati ´es Asztrofizikai Di´akolimpi´ar´ol ´es v´alogat´overseny´er˝ol, az Athletica Galactic´ar´ol . . . . . . . . . . . . . . . . . . . . . . . . . . . . . . . . . . Kunfalvi Rezs˝o Olimpiai V´alogat´overseny 1. elm´eleti fordul´o . . . . . . . . . . . . . . . . . . . . . . . . . . . . . . . . . . . Fizika gyakorlat megold´asa (773.) . . . . . . . . . . . . . . . .

368 369

Fizika feladatok megold´asa (5392., 5393., 5397., 5398., 5402.) . . . . . . . . . . . . . . . . . . . . . . . . . . . . . . . . . .

370

Fizik´ab´ol kit˝uz¨ott feladatok (415., 785–788., 5418–5426.) . . . . . . . . . . . . . . . . . . . . . . . . . . . . . . . . . . E¨otv¨os-verseny . . . . . . . . . . . . . . . . . . . . . . . . . . . . . . . . . . Problems in Mathematics . . . . . . . . . . . . . . . . . . . . . . . . Problems in Physics . . . . . . . . . . . . . . . . . . . . . . . . . . . . .

378 381 382 383

365

K¨ oz´ episkolai Matematikai ´ es Fizikai Lapok, 2022/6

´ EVA ´ F˝oszerkeszt˝o: RATKO ´ ¨ Fizikus szerkeszt˝o: GNADIG PETER ´ ILDIKO ´ M˝uszaki szerkeszt˝o: MIKLOS Bor´ıt´o: BURGHARDT ZSUZSA ´ Kiadja: MATFUND ALAP´ITVANY ´ RITA Alap´ıtv´anyi k´epvisel˝o: KOS Felel˝os kiad´o: KATONA GYULA Nyomda: OOK-PRESS Kft. ´ Felel˝os vezet˝o: SZATHMARY ATTILA INDEX: 25 450 ISSN 1215-9247 A matematika bizotts´ag vezet˝oje: ´ HERMANN PETER ´ BALINT, ´ ´ Tagjai: B´IRO GYENES ZOLTAN, ´ ´ KISS HUJTER BALINT, IMOLAY ANDRAS, ´ ´ GEZA, ´ ´ RITA, KOZMA GEZA, KOS KOS ´ MATOLCSI DAVID, ´ KATALIN ABIGEL, ´ ´ PACH PETER ´ ´ V´IGH ¨ ORDI ¨ OK PETERN E, PAL, VIKTOR A fizika bizotts´ag tiszteletbeli eln¨oke: ´ ´ HOLICS LASZL O ´ Tagjai: BARANYAI KLARA, HONYEK GYULA, ´ ´ KRISZTIAN, ´ OLOSZ BALAZS, SZASZ ´ ´ ´ E, ´ VLADAR ´ SZECHENYI GABOR, VIGH MAT ´ KAROLY, WOYNAROVICH FERENC Az informatika bizotts´ag vezet˝oje: ´ ´ SCHMIEDER LASZL O ´ E, ´ FARKAS CSABA, FODOR Tagjai: BUSA MAT ´ ´ ZSOLT, LOCZI LAJOS, SIEGLER GABOR, ´ ´ ´ SZENTE PETER, TOTH TAMAS ´ ANDREA, TASNADI ´ ANIKO ´ Ford´ıt´ok: GROF ´ ´ ¨ Szerkeszt˝os´egi titk´ar: TRASY GYORGYN E A szerkeszt˝os´eg c´ıme: 1117 Budapest, P´azm´any P´eter s´et´any 1/C III. emelet 3.405. Telefon: 372-2850 A lap megrendelhet˝o az Interneten: www.komal.hu/megrendelolap/reszletek.h.shtml. El˝ofizet´esi d´ıj egy ´evre: 9200 Ft K´eziratokat nem ˝orz¨unk meg ´es nem k¨uld¨unk vissza. Minden jog a K¨oMaL tulajdonosai´e. E-mail: [email protected] Internet: http://www.komal.hu This journal can be ordered from the Editorial office: P´azm´any P´eter s´et´any 1/C III. emelet 3.405. 1117–Budapest, Hungary telephone: +36 (1) 372-2850 or on the Postal address H–1518 Budapest 112, P.O.B. 32, Hungary, or on the Internet: www.komal.hu/megrendelolap/reszletek.e.shtml. A Lapban megjelen˝o hirdet´esek tartalm´a´ert felel˝oss´eget nem v´allalunk.

321

Az idei Nemzetk¨ ozi Matematikai Di´ akolimpi´at j´ ulius 6. ´es 16. k¨ oz¨ ott Norv´egia rendezte meg Osl´oban, k´et j´arv´ anys´ ujtotta ´ev ut´ an v´egre szem´elyes jelenl´ettel. A versenyen 104 orsz´ag 589 di´akja vett r´eszt. A versenyen, szok´ as szerint, mindk´et napon n´egy ´es f´el ´ora alatt h´ arom-h´ arom feladatot kellett megoldani. A feladatok sz¨ oveg´et al´ abb k¨ oz¨ olj¨ uk. Mindegyik feladat helyes megold´ as´a´ert 7 pont j´art, ´ıgy egy versenyz˝ o maxim´ alis teljes´ıtm´ennyel 42 pontot szerezhetett. A versenyz˝ok pontsz´ama a koordin´ atorok ´es a csapatvezet˝ ok k¨oz¨otti egyeztet´es r´ev´en alakult ki. A verseny befejez´ese ut´ an meg´allap´ıtott ponthat´ arok szerint arany´ermet a 34–42 pontot el´er˝ o, ez¨ ust´ermet a 29–33 pontos, m´ıg bronz´ermet a 23–28 ponttal rendelkez˝o tanul´ ok szereztek. A szokatlanul magas ponthat´ arok f˝ok´ent annak tudhat´ ok be, hogy az 1. ´es 4. feladaton k´ıv¨ ul id´en unek bizonyult. a 2. feladat is igen k¨ onny˝ ´ A magyar csapatot a Budapesti Fazekas Mih´aly Gyakorl´ o Altal´ anos Iskola ´es Gimn´ azium hat tanul´ oja alkotta. Kov´acs Tam´as (12. oszt.) 30 ponttal ez¨ ust´ermet nyert. Seres-Szab´ o M´arton (11. oszt.) 28 ponttal, N´ador Benedek (11. oszt.) 27 ponttal, Moln´ar-Szab´ o Vilmos (11. oszt.) 25 ponttal, Terj´ek Andr´as J´ ozsef (12. oszt.) 25 ponttal ´es N´emeth M´arton (11. oszt.) 23 ponttal bronz´ermet kapott. Frenkel P´eter (ELTE TTK Algebra ´es Sz´ amelm´elet Tansz´ek; R´enyi Int´ezet) a magyar csapat vezet˝ojek´ent, Dobos S´ andor (Budapesti Fazekas Mih´aly Gyakorl´ o ´ Altal´ anos Iskola ´es Gimn´ azium) a magyar csapat helyettes vezet˝ojek´ent, KunszentiKov´ acs D´ avid (ELTE TTK Alkalmazott Anal´ızis Tansz´ek; R´enyi Int´ezet) a Di´akolimpi´ at ir´any´ıt´ o¨ ottag´ u T´ abla, az Etikai Bizotts´ ag ´es a Feladatkiv´ alaszt´ o Bizottos s´ag tagjak´ent, valamint f˝ okoordin´atork´ent, Borb´enyi M´ arton (ELTE TTK) ´es K´ G´eza (ELTE TTK Anal´ızis Tansz´ek; SZTAKI) a Feladatkiv´ alaszt´ o Bizotts´ag tagjak´ent ´es koordin´ atork´ent, Beke Csongor, Csah´ ok T´ımea, Csap´ o Hajnalka, Frakn´ oi ´ am, Gerencs´er Bal´ ´ Ad´ azs, Gyarmati M´ at´e, Gy˝ orffy Agoston, Hansel Soma, Imolay Andr´ as, Jank´ o Zsuzsanna, Kerekes Anna, Kiss Melinda Fl´ ora, Kl´ asz Vikt´ oria, Kocsis Anett, Kov´ acs Benedek, Lenger D´ aniel, Nagy Kartal, V´ ali Benedek, V´ arkonyi ´ Zsombor ´es Z´ ahorsk´y Akos pedig koordin´ atork´ent m˝ uk¨ od¨ ott k¨ ozre az olimpi´ an. Az orsz´agok nem-hivatalos pontverseny´eben Magyarorsz´ag a r´esztvev˝ o 104 orsz´ ag k¨oz¨ott a 32.-33. helyen v´egzett. A csapatverseny ´elmez˝ ony´enek sorrendje ´ıgy alakult (megszerzett pontsz´ amaikkal): 1. K´ına 252, 2. D´el-Korea 208, 3. USA 207, 4. Vietn´am 196, 5. Rom´ania 194, 6. Thaif¨old 193, 7. N´emetorsz´ ag 192, 8-9. Ir´an ´es Jap´ an 191, 10-11. Izrael ´es Olasz322

K¨ oz´ episkolai Matematikai ´ es Fizikai Lapok, 2022/6

orsz´ ag 188, 12. Lengyelorsz´ag 183, 13. Egyes¨ ult Kir´ alys´ag 179, 14-15. Kanada ´es Tajvan 178, 16. Bulg´ aria 177, 17-18. Kazahszt´ an ´es Ukrajna 174, 19-21. Braz´ılia, Hongkong ´es Peru 173, 22. Sza´ ud-Ar´ abia 168, 23. Mexik´o 167, 24-25. India ´es Szin¨ enyorsz´ gap´ ur 165, 26-28. Orm´ ag, G¨ or¨ ogorsz´ ag ´es T¨or¨okorsz´ ag 163, 29-30. Ausztr´alia ´es Mong´ olia 162, 31. Belarusz 160, 32-33. Franciaorsz´ ag ´es Magyarorsz´ag 158. Az o o orsz´ ag ´es versenyz˝o neve ´es eredm´enye megtal´alhat´ o ¨sszes r´esztvev˝ az https://www.imo-official.org/ honlapon. Szeretn´ek k¨ osz¨ onetet mondani a versenyz˝ ok tan´ arainak. A k¨ozponti olimpiai felk´esz´ıt˝ o szakk¨ or vezet˝oje a helyettes csapatvezet˝ o, Dobos S´ andor volt. A felk´esz´ıt´es r´esz´et k´epezte egy egyhetes t´ aboroz´as j´ unius v´eg´en, Dobos S´ andor ´es Kiss G´eza ´ (Budapesti Fazekas Mih´aly Gyakorl´o Altal´ anos Iskola ´es Gimn´ azium), valamint Imolay Andr´ as ´es Kov´ acs Benedek (ELTE TTK) vezet´es´evel. A felk´esz´ıt´esben ´es a v´ alogat´ oversenyek dolgozatainak jav´ıt´ as´ aban a tan´ev sor´ an sokan m´asok is r´eszt ´ am R´eka ´es Fazakas vettek. A 11. oszt´ alyos versenyz˝ ok tan´ arai Dobos S´ andor, Ad´ T¨ unde, a 12. oszt´ alyosok´e Gyenes Zolt´ an, Hujter B´ alint ´es Juh´ asz P´eter voltak. N´emeth M´ arton tan´ ara volt m´eg Erd˝ os G´ abor, N´ador Benedek´e pedig Kov´ acs Benedek ´es Sz˝ ucs G´ abor. Az olimpi´an voltak matematikai ´es kultur´ alis-turisztikai jelleg˝ u k´ıs´er˝ o programok is. A r´esztvev˝ ok a zsongl˝ ork¨ od´es matematik´ aj´ar´ ol hallgathattak el˝ oad´ast, o m´ uzeumokba ´es kalandparkba l´atogathattak el. A teljes program megtal´alhat´ a https://www.imo2022.org/imo/Programme honlapon. T¨ obb u ´j tiszts´egvisel˝ ot is megv´ alasztottak az olimpi´ an, k¨ozt¨ uk egy magyart is: K´ os G´eza a T´ abla tagja lett. A k¨ ovetkez˝ o matematikai di´akolimpi´at Jap´ an rendezi Csiba v´aros´ aban, 2023. j´ ulius 2–13. k¨ oz¨ ott. Frenkel P´eter

A 63. Nemzetk¨ ozi Matematikai Di´akolimpia feladatai Els˝ o nap 1. feladat. Oslo bankja k´etf´ele t´ıpus´ u ´erm´et bocs´ at ki: alum´ıniumot (jele A) ´es bronzot (jele B). Mariann el˝ ott n alum´ınium´erme ´es n bronz´erme van egy sorban elrendezve valamilyen tetsz˝ oleges kezdeti sorrendben. L´ ancnak nevezz¨ uk egym´ ast k¨ ozvetlen¨ ul k¨ ovet˝o, azonos t´ıpus´ u ´erm´ek tetsz˝oleges sorozat´ at. R¨ogz´ıtett k  2n pozit´ıv eg´esz sz´ am mellett Mariann ism´etelten v´egrehajtja a k¨ovetkez˝ o m˝ uveletet: meghat´arozza a leghosszabb olyan l´ancot, amely tartalmazza a balr´ ol sz´am´ıtott kadik ´erm´et, ´es az ezen l´anchoz tartoz´o ¨ osszes ´erm´et ´atteszi a sor bal sz´el´ere. P´eld´ aul, ha n = 4 ´es k = 4, akkor az AABBBABA elrendez´esb˝ ol kiindul´o folyamat: AABBBABA → BBBAAABA → AAABBBBA → BBBBAAAA →

3. feladat. Legyen k pozit´ıv eg´esz, ´es legyen S p´ aratlan pr´ımsz´ amoknak egy v´eges halmaza. Bizony´ıtand´ o, hogy (elforgat´ast´ ol ´es t¨ ukr¨ oz´est˝ ol eltekintve) legfeljebb egyf´elek´eppen lehet az S elemeit egy k¨ or ment´en elrendezni u ´gy, hogy b´ armely k´et u legyen valamilyen pozit´ıv eg´esz x-szel. szomsz´edosnak a szorzata x2 + x + k alak´ M´asodik nap 4. feladat. Legyen ABCDE olyan konvex ¨ otsz¨ og, hogy BC = DE. Tegy¨ uk fel, hogy az ABCDE ¨ otsz¨ og belsej´eben l´ev˝ o T pontra T B = T D, T C = T E ´es ABT  = T EA. Messe az AB egyenes a CD ´es CT egyeneseket a P , illetve Q pontban. Tegy¨ uk fel, hogy a P, B, A, Q pontok az egyenes¨ uk¨ on ebben a sorrendben helyezkednek el. Messe az AE egyenes a CD ´es DT egyeneseket az R, illetve S pontban. Tegy¨ uk fel, hogy az R, E, A, S pontok az egyenes¨ uk¨ on ebben or¨ on a sorrendben helyezkednek el. Bizony´ıtand´ o, hogy a P , S, Q, R pontok egy k¨ vannak. 5. feladat. Hat´ arozzuk meg mindazon, pozit´ıv eg´eszekb˝ ol ´ all´ o (a, b, p) sz´ amh´ armasokat, amelyekre p pr´ım ´es ap = b! + p. 6. feladat. Legyen n pozit´ıv eg´esz. Skandin´ av n´egyzet egy n × n m´eret˝ u t´ abla, osszes eg´esz sz´ amot tartalmazza u ´gy, hogy minden mez˝oben amely 1-t˝ ol n2 -ig az ¨ pontosan egy sz´ am ´ all. K´et k¨ ul¨ onb¨ oz˝ o mez˝ ot szomsz´edosnak tekint¨ unk, ha van k¨ oz¨os oldaluk. Ha egy mez˝ onek minden szomsz´edj´aban nagyobb sz´am a´ll, mint ˝obenne, akkor v¨ olgynek nevezz¨ uk. Kaptat´ o egy sorozat, amely egy vagy t¨ obb mez˝ ob˝ ol ´all u ´gy, hogy (i) a sorozat els˝ o mez˝ oje egy v¨ olgy, (ii) a sorozat minden tov´abbi mez˝ oje szomsz´edos az ˝ot k¨ ozvetlen¨ ul megel˝ oz˝ o mez˝ ovel, ´es (iii) a sorozat mez˝ oiben ´all´ o sz´ amok n¨ ovekv˝ o sorrendben vannak. Adott n eset´en hat´ arozzuk meg egy skandin´ av n´egyzetben l´ev˝ o kaptat´ ok sz´am´anak legkisebb lehets´eges ´ert´ek´et.

Olimpiai el˝ok´esz´ıt˝ o szakk¨or¨ok a 2022/2023. tan´evben A Bolyai J´ anos Matematikai T´arsulat ´altal szervezett K¨ ozponti olimpiai szakk¨ori felk´esz¨ ul´es az al´ abbiak szerint t¨ ort´enik:

→ BBBBAAAA → · · · .

Hat´ arozzuk meg mindazon, 1  k  2n tulajdons´ ag´ u (n, k) p´ arokat, amelyekre minden kiindul´ asi elrendez´es eset´en lesz olyan pillanat a folyamat sor´ an, hogy a balr´ ol sz´ am´ıtott els˝o n ´erme mind azonos t´ıpus´ u. K¨ oz´ episkolai Matematikai ´ es Fizikai Lapok, 2022/6

2. feladat. Jel¨ olje R+ a pozit´ıv val´ os sz´amok halmaz´at. Hat´ arozzuk meg mind+ + uggv´enyeket, amelyekre minden x ∈ R+ eset´en pontosan egy azon f : R → R f¨ olyan y ∈ R+ l´etezik, hogy xf (y) + yf (x)  2.

323

Budapest: az els˝ o alkalom szeptember 23-´ an (p´enteken) lesz a Budapesti Faze´ kas Mih´aly Gyakorl´ o Altal´ anos Iskola ´es Gimn´ aziumban (Budapest VIII. ker¨ ulet, Horv´ ath M. t´er 8.) 14:30 ´es 17:00 k¨ oz¨ ott, szakk¨ orvezet˝ o: Dobos S´ andor. 324

K¨ oz´ episkolai Matematikai ´ es Fizikai Lapok, 2022/6

Csongr´ ad-Csan´ ad megye: az els˝ o alkalom szeptember 15-´en (cs¨ ut¨ort¨ok¨on) lesz, ut´ ana k´ethetente a Szegedi Tudom´ anyegyetem Bolyai Int´ezet´eben (Szeged, Aradi v´ertan´ uk tere 1., I. emelet, Riesz terem), 15:00 ´es 17:00 k¨oz¨ott, szakk¨orvezet˝ o: Kosztol´ anyi J´ ozsef. Erd˝ os P´ al Matematikai Tehets´eggondoz´ o Iskola veszpr´emi foglalkoz´ asai 9–12. ´evfolyamosok sz´ am´ ara. Az egyes foglalkoz´ asokra a jelentkez´est a di´ akok egy´enileg v´egezhetik el 2022. szeptember 17-ig az Erd˝os Iskola honlapj´an∗ . Az idei els˝o foglalkoz´ as Veszpr´emben szeptember 30. ´es okt´ ober 2. k¨oz¨ott lesz.

EGMO 2022/2023 felh´ıv´as

2023. a´prilis 13. ´es 19. k¨ oz¨ ott Portoroˇzban ker¨ ul megrendez´esre a tizenkettedik Eur´ opai L´any Matematikai Di´akolimpia, az EGMO (https://egmo2023.dmfa.si/, https://www.egmo.org/). Orsz´agunk a versenyen egy n´egyf˝ os csapattal k´epviseltetheti mag´at, melynek ¨ osszet´etele 2023 elej´en der¨ ul ki. A v´ alogat´ as szempontjai: v´ alogat´ oversenyek (2022 ˝osz´en ´es 2023 elej´en) – kis m´ert´ekben az elm´ ult ´evi is –, orsz´ agos kifejt˝os egy´eni versenyek (matematika OKTV, K¨ ursch´ ak J´ ozsef Matematikai Tanul´ overseny, Arany D´ aniel Matematikaverseny), a K¨oMaL (A ´es) B pontversenyei, valamint az ´evk¨ozi munka. A versenyen val´ o sikeres szerepl´eshez, illetve a kiutaz´o csapatba ker¨ ul´eshez is alapvet˝ oen n´elk¨ ul¨ ozhetetlen az alapos felk´esz¨ ul´es, ezt t¨obbf´elek´eppen is szeretn´enk ´ k¨ unk az ´erdekl˝ od˝oknek (teseg´ıteni. Ev ozben k¨ or¨ ulbel¨ ul havi rendszeress´eggel k¨ uld¨ matikus) feladatsorokat; az ezekre k¨ uld¨ ott megold´ asokra szem´elyesen is visszajelz¨ unk. Emellett az ˝oszi v´alogat´ oig legeredm´enyesebb di´ akok r´eszt vehetnek a t´eli brit-magyar k¨ oz¨ os IMO felk´esz´ıt˝ o t´ aborban. A k´et v´ alogat´ o o¨sszes´ıtett eredm´enye alapj´an a legeredm´enyesebb di´ akok r´eszt vehetnek az intenz´ıv EGMO felk´esz´ıt˝o h´etv´eg´en is. A felk´esz¨ ul´esbe ´erdemes min´el el˝ obb, ak´ ar m´ar kilencedikesk´ent bekapcsol´odni. Minden l´ any jelentkez´es´et szeretettel v´ arjuk, akit ´erdekel a versenyr´eszv´etel lehet˝ os´ege ´es nem riad vissza att´ ol, hogy ez´ert komolyabb munk´ at fektessen bele. Aki szeretne r´eszt venni a v´ alogat´ asban ´es felk´esz¨ ul´esben, vagy b´ armilyen k´erd´ese van, ´ırjon min´el el˝ obb az [email protected] c´ımre. Kiss Melinda Fl´ora, Baran Zsuzsa

N´egysz´ın-sejt´es I: A sejt´es szu ¨let´ese ´es egy bizony´ıt´asi k´ıs´erlet∗ Francis Guthrie, egy londoni di´ ak, 1852-ben Anglia megy´einek t´erk´ep´et n´ezegetve r´ aj¨ott, hogy ki tudja sz´ınezni a megy´eket 4 sz´ınnel u ´gy, hogy szomsz´edos megy´ek sehol se kapjanak azonos sz´ınt. Meglepte, hogy egy ilyen sok megy´eb˝ ol ´ all´ o, l´atsz´olag teljesen szab´ alytalan t´erk´epet ilyen kev´es sz´ınnel ki lehet sz´ınezni. Ez´ert azt kezdte sejteni, hogy ez egy ´altal´ anos tulajdons´ ag lehet, ´es tal´ an minden t´erk´epet ki lehet sz´ınezni 4 sz´ınnel u ´gy, hogy szomsz´edos tartom´ anyok ne kapjanak azonos sz´ınt. Miut´an nem siker¨ ult bebizony´ıtania, hogy ´ıgy lenne, de olyan t´erk´epet sem siker¨ ult rajzolnia, ahol t¨ obb, mint 4 sz´ınre lenne sz¨ uks´eg, ´ırt ¨ occs´enek, Frederiknek, aki ekkoriban a University College Londonban tanult matematik´ at. Frederik is ´erdekesnek tal´ alta a sejt´est, viszont neki sem siker¨ ult sem bizony´ıt´ ast, sem ellenp´eld´ at tal´ alnia. Ez´ert elmondta a k´erd´est tan´ ar´ anak, a neves matematikus Augustus De Morgannek. De Morgan hasonl´ oan j´ art, a k´erd´est nagyon izgalmasnak abbi matematikus tal´ alta, megoldani azonban nem tudta, ez´ert feltette a k´erd´est tov´ ismer˝ oseinek. ´Igy indult u ´tj´ara a h´ıres n´egysz´ın-sejt´es, melyet v´eg¨ ul 1976-ban siker¨ ult bebizony´ıtania Appelnek ´es Hakennek, de a bizony´ıt´ asuk annyiban szokatlan volt, hogy rengeteg esetet sz´ am´ıt´ og´eppel kellett leellen˝ orizni, teh´at nem egy emberi ´esszel k¨onnyen a´tl´ athat´ o bizony´ıt´ asr´ ol volt sz´o. K´es˝ obb Robertson, Sanders, Seymour ´es Thomas adott egy bizony´ıt´ast, amiben kevesebb esetet kell sz´ am´ıt´ og´eppel ellen˝ orizni, de m´eg az o˝ bizony´ıt´ asukhoz is sz¨ uks´eg van a sz´ am´ıt´ og´epre. R¨ ovid” ” bizony´ıt´as az´ota sem ismert a t´etelre. A sejt´es felvet´ese ´es bizony´ıt´asa k¨ oz¨ ott eltelt t¨obb, mint 120 ´ev alatt a n´egysz´ın-sejt´es v´egig nagyon sok embert foglalkoztatott, ast eredm´enyeztek. A k¨ o´es a megold´ as´ara tett er˝ ofesz´ıt´esek rengeteg hasznos tud´ vetkez˝ o cikksorozat ebb˝ol szeretne egy kis ´ızel´ıt˝ ot adni. Az els˝o r´eszben bemutatjuk az els˝ o bizony´ıt´ asi k´ıs´erletet a n´egysz´ın-sejt´esre. Ezt Kempe publik´ alta 1879-ben. Ezut´ an 11 ´evig megoldottnak hitt´ek a n´egysz´ınsejt´est, azonban 11 ´ev ut´ an Heawood ´eszrevett egy hib´ at a bizony´ıt´ asban. Sajnos az der¨ ult ki, hogy ez nem egy k¨ onnyen kijav´ıthat´ o hiba. Ennek ellen´ere a Kempe-f´ele bizony´ıt´as sok hasznos ¨ otletet szolg´altatott. Heawoodnak a Kempe-f´ele bizony´ıt´ ast m´odos´ıtva siker¨ ult bel´atnia az ¨ otsz´ın-t´etelt, azaz hogy b´ armely t´erk´ep j´ ol sz´ınezhet˝ o 5 sz´ınnel. Valamint a n´egysz´ın-t´etel majdnem 100 ´evvel k´es˝obbi Appel–Haken-f´ele bizony´ıt´asa is ´ep´ıt Kempe gondolatmenet´ere. Izgalmas ´es tanuls´ agos feladat megkeresni a hib´at Kempe ´ervel´es´eben. N´ezz¨ uk meg, hogyan is sz´ol ez az ´ervel´es. A sejt´es jelent˝ os´ege. Miel˝ott bemutatn´ ank Kempe ´ervel´es´et, hadd mondjunk m´eg p´ ar sz´ot arr´ ol, hogy mi´ert ´erdekes a n´egysz´ın-sejt´es. Hiszen nincs igaz´ an ´ ´ Nemzeti Az ´ır´ as az Innov´ aci´ os ´es Technol´ ogiai Miniszt´erium UNKP-20-5 k´ odsz´ am´ u Uj Kiv´ al´ os´ ag Programj´ anak a Nemzeti Kutat´ asi, Fejleszt´esi ´es Innov´ aci´ os Alapb´ ol finansz´ırozott szakmai t´ amogat´ as´ aval k´esz¨ ult. ∗



https://erdosiskola.mik.uni-pannon.hu/

K¨ oz´ episkolai Matematikai ´ es Fizikai Lapok, 2022/6

325

326

K¨ oz´ episkolai Matematikai ´ es Fizikai Lapok, 2022/6

praktikus jelent˝ os´ege. Tal´ an kicsit eszt´etikusabb egy t´erk´epet 4 sz´ınnel sz´ınezni, mint ha 6 sz´ınnel lenne sz´ınezve, de ha csak az a c´elunk, hogy a megy´eket j´ol el tudjuk k¨ ul¨ on´ıteni egym´ast´ ol, val´oj´aban 6 sz´ın is ugyanolyan j´o, mint 4. Hogy m´egis 120 ´ev matematikusai lelkesedtek a k´erd´es ir´ant, az az´ert van, mert ez egy meglep˝o jelens´eg, aminek nem ´ertj¨ uk az ok´ at. Ha pedig valami meglep˝ o dolog igaz, akkor az ember azt felt´etelezi, hogy az annak a h´ atter´eben a´ll´o t¨orv´enyszer˝ us´eget meg´ertve sokkal jobban fogjuk ´erteni a vil´agot, ´es ez sok m´ as jelens´eg meg´ert´es´eben is seg´ıteni fog.

ja ki, hogy tetsz˝oleges ¨ osszef¨ ugg˝o s´ıkgr´ afra cs +  = e + 2, azaz a cs´ ucsok ´es lapok ama 2-vel nagyobb, mint az ´elek sz´ ama. Ezt a sz´ep formul´ at sokf´elek´eppen be ¨osszsz´ lehet bizony´ıtani, ´erdemes az olvas´onak ¨ on´ all´ oan megpr´ ob´ alni. Az´ert mi is adunk itt egy bizony´ıt´ ast. ´ amra vonatkoz´ Elsz´ o indukci´ ot haszn´alunk. Az alapeset az, ha a gr´afunknak nincs ´ele. Mivel minden cs´ ucsb´ ol mindenhova el kell tudnunk jutni ´eleken kereszt¨ ul, ekkor cs´ ucsa is csak 1 lehet a gr´ afnak. Lap pedig szint´en 1 lesz (a v´egtelen lap). Teh´at ekkor val´oban cs +  = 2 = e + 2.

Megjegyezz¨ uk, hogy b´ ar a t´erk´epek sz´ınez´ese nem egy gyakorlatban ´erdekes k´erd´es, sok praktikusan ´erdekes probl´em´ at le lehet ford´ıtani a´ltal´ anosabb sz´ınez´esi k´erd´esekre.

Tegy¨ uk fel most, hogy e = k > 0, ´es hogy k-n´ al kevesebb ´el˝ uo ugg˝o s´ıkg¨sszef¨ r´afokra m´ ar tudjuk, hogy teljes¨ ul a t´etel. Vegy¨ unk egy tetsz˝ oleges ´elt, mely az u ucsokat k¨ oti o os´eg van. Az ´el k´et oldal´ an vagy k´et k¨ ul¨ onb¨ oz˝ o ´es v cs´ ¨ssze. K´et lehet˝ lap van, vagy pedig ugyanaz a lap (mindkett˝ore mutatunk egy-egy p´eld´ at az 1. ´ abr´ an). Ha az ´el k´et oldal´ an k´et k¨ ul¨ onb¨ oz˝ o, L1 ´es L2 lap van, akkor az ´elet elhagyva is o¨sszef¨ ugg˝o marad a gr´ af. Ugyanis az uv ´elen val´ o´ athalad´ ast tudjuk helyettes´ıteathalad´ assal. (Ha az ´el k´et oldal´ an ugyanaz a lap ni az L1 lap marad´ek ´ıv´en val´o ´ szerepel, akkor ezt nem tudjuk megtenni, mert ilyenkor a lap hat´ ar´ an k´etszer is szerepel az uv ´el, ´ıgy nem tudunk a marad´ek hat´ aron kereszt¨ ul eljutni u-b´ ol v-be.) Teh´at ha az uv ´el k´et oldal´ an k´et k¨ ul¨ onb¨ oz˝ o lap van, akkor hagyjuk el az ´elt. Ekkor az el˝obbiek miatt ¨ osszef¨ ugg˝o marad a gr´ af, ´es az ´elsz´am e − 1 = k − 1 lesz. Az L1 ´es L2 lapok ¨osszeolvadnak egy lapp´ a, ez´ert a lapok sz´ ama  − 1 lesz. A cs´ ucsok sz´ ama nem v´altozik. Mivel az u ´j gr´ af k − 1 ´el˝ u, erre az indukci´os feltev´es miatt m´ ar teljes¨ ul, hogy cs + ( − 1) = (e − 1) + 2. Vagyis val´ oban, cs +  = e + 2.

Fogalmazzuk ´at a k´erd´est, hogy megszabaduljunk a f¨ol¨osleges adatokt´ol. El˝oab´ol nyilsz¨or is, fogalmazzuk ´at egy kicsit a n´egysz´ın-sejt´est. A sz´ınez´es szempontj´ v´ an mindegy, hogy egy megy´enek pontosan milyen az alakja, csak az a fontos, hogy mely m´as megy´ekkel szomsz´edos. ´Igy el´eg, ha csak a szomsz´eds´ agi viszonyokat jegyezz¨ uk meg. Azaz rajzoljunk minden megy´ere egy pontot (p´eld´ aul a megyesz´ekhelyre), ´es k¨ oss¨ uk ¨ ossze a szomsz´edos megy´ek sz´ekhelyeit az o˝ket elv´ alaszt´ o hat´ aron kereszt¨ ul. ´Igy egy gr´afot kapunk. M´egpedig egy el´eg speci´ alis gr´ afot: a cs´ ucsai egy s´ıkra vannak rajzolva, az ´elei pedig nem metszik egym´ast. Az ilyen gr´ afokat s´ıkgr´afoknak nevezz¨ uk. Mindig fel fogjuk azt is tenni, hogy nincs hurok´el, azaz egy ´el k´et v´egpontja mindig k¨ ul¨ onb¨ oz˝ o. (A t´erk´epekb˝ ol kapott gr´ afokra ez nyilv´ an igaz.) A t´erk´ep sz´ınez´es´et u ´gy lehet leford´ıtani a gr´ af nyelv´ere, hogy a s´ıkgr´af cs´ ucsaot¨ ott cs´ ucsok k¨ ul¨onb¨oz˝ o sz´ınt kapnak. it szeretn´enk sz´ınezni u ´gy, hogy ´ellel ¨ osszek¨ Az ilyen sz´ınez´eseket innent˝ol j´o sz´ınez´eseknek nevezz¨ uk. Azt is meg lehet gondolni, hogy a s´ıkgr´ afok sz´ınez´ese nem a´ltal´ anosabb feladat a t´erk´epek sz´ınez´es´en´el, mert minden hurok´elmentes s´ıkgr´afhoz lehet rajzolni egy t´erk´epet, ahol pontosan azok a megy´ek lesznek szomsz´edosak, amelyeknek megfelel˝ o cs´ ucsok ¨ossze voltak k¨otve. (Vigy´ azat, egy ponton ´erintkez˝o megy´eket nem tekint¨ unk szomsz´edosnak.) Teh´ at a n´egysz´ın-sejt´es azt mondja ki, hogy egy (hurok´elmentes) s´ıkgr´ af pontjainak l´etezik 4 sz´ınnel j´ o sz´ınez´ese.

1. ´ abra. Bal oldalon: Az uv ´el k´et oldal´ an k´et k¨ ul¨ onb¨ oz˝ o, L1 ´es L2 lap van. Jobb oldalon: Az uv ´el k´et oldal´ an azonos lap van

Megjegyezz¨ uk, hogy a s´ıkgr´ afok el´eg speci´ alis gr´ afok. Egy ´altal´ anos gr´af cs´ ucsainak j´o sz´ınez´es´ehez tetsz˝olegesen sok sz´ınre lehet sz¨ uks´eg¨ unk. Ha p´eld´ aul vesz¨ unk ucsot, ´es mindegyik cs´ ucsot ¨ osszek¨ otj¨ uk az ¨ osszes t¨obbivel, akkor k´enytelenek n cs´ vagyunk n sz´ınt haszn´ alni egy j´ o sz´ınez´eshez. De egy ilyen gr´ af n > 4 eset´en persze nem rajzolhat´ o le a s´ıkra u ´gy, hogy az ´elek ne mess´ek egym´ ast.

Azt az esetet kell m´eg kezeln¨ unk, ha az ´el k´et oldal´ an azonos lap szerepel. Olvasszuk ¨ ossze az u ´es v cs´ ucsokat egy cs´ uccs´ a, ´es t¨ or¨ olj¨ uk ki a kiv´ alasztott ´el¨ unket. A t¨ obbi u-b´ ol vagy v-b˝ ol indul´o ´elt hagyjuk meg, csak most m´ ar az ¨osszeolvasztott cs´ ucsb´ ol fognak indulni. ´Igy tov´ abbra sem fogj´ ak ´elek metszeni egym´ ast. A cs´ ucsok ´es az ´elek sz´ ama 1-gyel cs¨ okken, a lapok sz´ama pedig v´ altozatlan. Most is k − 1 ´el˝ u s´ıkgr´ afot kaptunk, teh´ at az indukci´os felt´etel miatt (cs − 1) +  = (e − 1) + 2, azaz cs +  = e + 2. Ezzel bel´attuk az Euler-formul´ at. Nevezz¨ unk egy gr´ afot egyszer˝ unek, ha b´ armely k´et cs´ ucs k¨ oz¨ ott legfeljebb 1 ´el halad, ´es nincsenek hurok´elek, azaz b´ armely ´el k´et v´egpontja k¨ ul¨ onb¨ oz˝ o. Az Eulerformul´ ab´ol levezethetj¨ uk, hogy egy egyszer˝ u s´ıkgr´ afnak nincsen t´ ul sok ´ele a cs´ ucssz´ am´ahoz k´epest:

A s´ıkgr´afok n´eh´any fontos tulajdons´aga. El˝osz¨or l´assunk be n´eh´ any dolgot a s´ıkgr´ afokr´ ol. Ezek k¨oz¨ ul az els˝ o Leonhardt Euler h´ıres formul´ aja. (Ebben az alfejezetben m´eg nem fogunk csalni”.) ” Vegy¨ unk egy ¨ osszef¨ ugg˝o s´ıkgr´ afot. Az ¨ osszef¨ ugg˝o itt azt jelenti, hogy minden cs´ ucsb´ol minden m´asik cs´ ucsba el tudunk jutni a gr´af ´elein kereszt¨ ul. A cs´ ucsok ´es az ´elek tartom´ anyokra osztj´ ak a s´ıkot. Ezek k¨ oz¨ ul lesz egy v´egtelen tartom´ any, obbi pedig korl´ atos. Nevezz¨ uk ezeket a tartom´anyokat a s´ıkgr´ af lapjainak (a v´ega t¨ telen tartom´ anyt is bele´ertve). A sz´ amukat jel¨ olj¨ uk -el. Emellett jel¨olj¨ uk cs-vel a gr´ af cs´ ucsainak sz´am´at, e-vel pedig az ´eleinek sz´ am´at. Euler formul´ aja azt mondK¨ oz´ episkolai Matematikai ´ es Fizikai Lapok, 2022/6

327

1. ´all´ıt´as. Egyszer˝ u s´ıkgr´ afban e  3cs − 6. 328

K¨ oz´ episkolai Matematikai ´ es Fizikai Lapok, 2022/6

Bizony´ıt´as. Mivel a s´ıkgr´ afunkban nincsenek hurkok ´es t¨obbsz¨or¨os ´elek, ez´ert minden lapot legal´abb 3 ´el hat´ arol. Ha van olyan lap, amit t¨obb, mint 3 ´el hat´ arol, akkor h´ uzzunk be egy a´tl´ ot. Ezzel tov´ abbra is s´ıkgr´ afunk van. A cs´ ucsok sz´ ama v´altozatlan maradt, az ´elek sz´am´at pedig csak n¨ ovelt¨ uk. Ezzel a m´ odszerrel el´erhetj¨ uk, hogy a s´ıkgr´ af minden lapj´at pontosan 3 ´el hat´ arolja. Jel¨olj¨ uk ennek a kib˝ ov´ıtett am´at pedig  -vel. Azt ´all´ıtjuk, hogy 3 = 2e . gr´ afnak az ´elsz´am´at e -vel, a lapsz´ Val´ oban, ha minden lapra megsz´ amoljuk az o˝t hat´ arol´ o ´eleket, akkor 3 ´elt fogunk sz´ amolni. Viszont ´ıgy minden ´elt k´etszer sz´ amoltunk, a k´et oldala ment´en. os´eget, Fel´ırva az Euler formul´at, cs +  = e + 2. Behelyettes´ıtve az  = 23 e egyenl˝ cs = 13 e + 2, azaz e = 3cs − 6. Mivel az eredeti gr´afra e  e , megkapjuk, hogy e  3cs − 6. 

v-nek 4 szomsz´edja van ´es ezek 4 sz´ınt haszn´alnak. Tegy¨ uk fel, hogy a v szomsz´edai az ´oramutat´o j´ar´ asa szerint rendre piros, s´ arga, k´ek ´es z¨ old sz´ın˝ uek. Sz´ınezz¨ uk ´ at a v cs´ ucs piros szomsz´edj´at k´ekre. Ha ez (G − v)-nek egy j´o sz´ınez´ese, akkor k´eszen vagyunk, mert most m´ ar lehet a G-ben a v cs´ ucs sz´ıne piros. Ha az u ´j sz´ınez´es nem j´o, akkor a k´ekre a´tsz´ınezett cs´ ucsnak eredetileg volt (1 vagy t¨obb) k´ek szomsz´edja. Ezeket a cs´ ucsokat most sz´ınezz¨ uk ´ at pirosra. Ha ezeknek voltak tov´ abbi piros sz´ın˝ u szomsz´edai, azokat sz´ınezz¨ uk a´t k´ekre stb. Egy p´eld´ at l´atunk erre a 2. ´ abr´ an. K¨ onny˝ u meggondolni, hogy ezt a szab´ alyt k¨ ovetve o ut´an el fognak fogyni egy cs´ ucsot legfeljebb egyszer sz´ınez¨ unk a´t. Teh´ at egy id˝ az ´atsz´ınezend˝ o cs´ ucsaink, ´es kapunk egy j´ o sz´ınez´est a G − v gr´ afra.

1. k¨ ovetkezm´eny. Tetsz˝ oleges egyszer˝ u s´ıkgr´ afban l´etezik legfeljebb 5 fok´ u cs´ ucs. Bizony´ıt´as. Indirekt tegy¨ uk fel, hogy minden foksz´am legal´abb 6. Ekkor a foksz´ amok ¨ osszege legal´ abb 6cs. B´ armely gr´afban a foksz´ amok ¨osszege az ´elek sz´am´anak dupl´ aja, hiszen minden ´elt mindk´et v´egpontj´ an´al megsz´amolunk. Teh´ at az ´elek sz´ ama legal´abb 3cs. Ez ellentmond az el˝ obbi a´ll´ıt´ asnak, teh´ at hamis volt az indirekt feltev´es¨ unk. 

2. ´ abra. A bal oldalon egy v cs´ ucs kiv´etel´evel j´ ol sz´ınezett s´ıkgr´ af l´ athat´ o. A sz´ınezetlen (feh´er) v cs´ ucs piros szomsz´edj´ at k´ekre a ´tsz´ınezve, majd a sz¨ uks´eges tov´ abbi szomsz´edokat a ´tsz´ınezve a jobb oldali sz´ınez´est kapjuk

Kempe ´ervel´ese Ennyi el˝ ok´esz¨ ulet ut´ an most m´ ar el tudjuk mondani, hogy sz´olt Kempe hib´as bizony´ıt´ asa a n´egysz´ın-sejt´esre. Arra biztatjuk az olvas´ ot, hogy pr´ob´alja meg megtal´ alni a hib´ at. Vegy¨ uk ´eszre, hogy el´eg, ha egyszer˝ u s´ıkgr´ afokat tudunk 4 sz´ınnel j´ol sz´ınezni. Ugyanis ha egy u ´es v pont k¨ oz¨ ott t¨ obb ´el is fut, az ugyan´ ugy csak annyit jelent, hogy az u-t ´es a v-t nem lehet azonos sz´ınre sz´ınezni, mint ha egy ´el futna k¨ozt¨ uk. Teh´ at ha van t¨ obbsz¨ or¨os ´el, akkor hagyjunk el annyi ´elt, hogy csak egyszeres legyen. (Ezzel nyilv´anval´ oan a s´ıkbarajzolts´ agot sem rontjuk el.) Ha az ´ıgy kapott egyszer˝ u gr´ afot tudjuk 4 sz´ınnel j´ ol sz´ınezni, akkor az eredetit is. Egyszer˝ u s´ıkgr´ afokra cs´ ucssz´amra vonatkoz´ o indukci´oval l´atjuk be a t´etelt. Tegy¨ uk fel, hogy minden k-n´ al kisebb cs´ ucssz´am´ u egyszer˝ u s´ıkgr´ afot tudunk 4 sz´ınnel j´ol sz´ınezni. Meg kell mutatnunk, hogy ekkor minden k cs´ ucs´ u egyszer˝ u s´ıkgr´afot is j´ol lehet sz´ınezni 4 sz´ınnel. Legyen G egy k cs´ ucs´ u egyszer˝ u s´ıkgr´af. az 1. k¨ovetkezm´eny szerint a G gr´ afban van legfeljebb 5 fok´ u cs´ ucs. Vegy¨ unk egy ilyen cs´ ucsot, ´es jel¨ olj¨ uk v-vel. T¨ or¨ olj¨ uk a gr´ afb´ ol v-t ´es a r´ a illeszked˝ o ´eleket, ´es nevezz¨ uk a kapott gr´ afot (G − v)-nek. Ekkor tov´ abbra is egy s´ıkbarajzolt egyszer˝ u gr´afunk lesz, de m´ ar k − 1 ponttal. Teh´ at ezt a gr´ afot az indukci´os feltev´es szerint j´ ol lehet sz´ınezni 4 sz´ınnel. Sz´ınezz¨ uk ki (j´ ol) 4 sz´ınnel. Most rajzoljuk vissza a v pontot ´es a r´ a illeszked˝ o ´eleket. Kellene a v-nek is tal´alni egy sz´ınt, ami k¨ ul¨onb¨ozik a szomsz´edai sz´ın´et˝ ol. Ha v szomsz´edai legfeljebb 3 sz´ınt haszn´ alnak a 4 sz´ın k¨oz¨ ul, akkor v megkaphatja a negyedik sz´ınt, ´es k´eszen vagyunk. Gond akkor van, ha v szomsz´edai mind a 4 sz´ınt haszn´ alj´ ak. Ez csak akkor lehet, ha v-nek 4 vagy 5 szomsz´edja van. N´ezz¨ uk meg alaposabban ezeket az eseteket. K¨ oz´ episkolai Matematikai ´ es Fizikai Lapok, 2022/6

329

K´et lehet˝ os´eg¨ unk van. Ha a v szomsz´edai k¨ oz¨ ul a k´eket nem sz´ınezt¨ uk ´ at pirosra, akkor most a v lehet piros, ´es k´eszen vagyunk. A 2. a´br´ an p´eld´ aul egy ilyen esetet l´atunk. Ha viszont v eredetileg k´ek szomsz´edj´at ´ atsz´ınezt¨ uk pirosra, akkor l´atsz´olag nem nyert¨ unk semmit, tov´ abbra is mind a 4 sz´ın ott van v szomsz´edai k¨oz¨ott. (Egy ilyen p´eld´ at l´atunk a 3. a ´br´ an.) Mit tudunk most mondani? Az´ert valamit ´ıgy is tanultunk a gr´ afr´ ol: m´egpedig hogy (az eredeti sz´ınez´est n´ezve) a v cs´ ucs piros ´es k´ek szomsz´edai k¨ oz¨ ott van egy u ´t, amin felv´ altva piros ´es k´ek cs´ ucsok vannak. Vegy¨ uk ´eszre hogy ez az u ´t a v-vel egy¨ utt m´ ar egy k¨ ort ad, ´es arga szomsz´edja ´es z¨ old szomsz´edja k¨ oz¨ ul az egyik a k¨ or belsej´eben van, a v s´ a m´asik pedig a k¨ ulsej´eben. Mi´ert hasznos nek¨ unk ez az inform´aci´ o? Pr´ob´ aljuk meg most ugyanezt elj´atszani a s´ arga ´es z¨ old sz´ınekkel. Azaz a v s´ arga szomsz´edj´at ´atsz´ınezz¨ uk z¨oldre. Ha az eredetileg s´arga cs´ ucsnak voltak z¨ old szomsz´edai, akkor ´atsz´ınezz¨ uk ˝oket s´ arg´ara stb. Ez az a´tsz´ınez´es m´ ar nem ´erhet v´eget u ´gy, hogy a v z¨old szomsz´edj´at a´tsz´ınezz¨ uk s´ arg´ara. Akkor ugyanis lenne a v s´ arga ´es z¨ old

3. ´ abra. A bal oldali sz´ınez´esben a v piros szomsz´edj´ at k´ekre a ´tsz´ınezve a k¨ oz´eps˝ o sz´ınez´est kapjuk. A v k´ek szomsz´edja pirosra sz´ınez˝ od¨ ott. A piros-k´ek k¨ ort sz¨ urke vonallal jel¨ olt¨ uk. A jobb oldali a ´bra mutatja a v s´ arga szomsz´edj´ anak z¨ oldre sz´ınez´es´evel kapott sz´ınez´est

330

K¨ oz´ episkolai Matematikai ´ es Fizikai Lapok, 2022/6

szomsz´edai k¨ oz¨ ott egy u ´t, ami csak s´arga ´es z¨ old cs´ ucsokat tartalmaz. Ennek az u ´tnak a piros-k´ek k¨ or belsej´eb˝ ol el kellene jutnia a k¨ ulsej´ebe, azt viszont csak u ´gy tudn´ a megtenni, ha a gr´ af k´et ´ele metszen´e egym´ ast. Mivel s´ıkgr´afr´ ol van sz´ o, ez nem lehet. Teh´at tal´ alunk egy a´tsz´ınez´est, ahol a v-nek nincs s´ arga szomsz´edja, azaz a v-t sz´ınezhetj¨ uk s´ arg´ara. Ezzel befejezt¨ uk az indukci´ os l´ep´es bizony´ıt´as´at abban az esetben ha a v-nek 4 szomsz´edja van.

Hivatkoz´asok [1] https://web.stonehill.edu/compsci/LC/Four-Color/Four-color.htm [2] Jeremy L. Martin, The Notorious Four-Color Problem, KU Mini College el˝oad´as, https://jlmartin.ku.edu/MiniCollege2013/slides.pdf [3] Alfred Bray, Kempe’s “proof ” of the four-color theorem, MATH horizons, 2002. https://mathweb.ucsd.edu/~ssam/old/19W-154/kempe.pdf

v-nek 5 szomsz´edja van ´es ezek 4 sz´ınt haszn´alnak. H´ atra van m´eg az az eset, ha v-nek 5 szomsz´edja van, ´es ezek haszn´ alj´ ak mind a 4 sz´ınt. Ekkor biztos, hogy valamelyik sz´ın k´et szomsz´edn´ al van haszn´alva, a m´asik 3 sz´ın pedig 1-1 szomsz´edn´ al. Tegy¨ uk fel, hogy mondjuk a k´ek sz´ın szerepel k´etszer (ez nyilv´ an feltehet˝o, utt kicser´elhetj¨ uk hiszen ha mondjuk a s´arga sz´ın szerepel k´etszer, akkor minden¨ a s´arga ´es a k´ek sz´ıneket). K´et lehet˝ os´eg van: Vagy egym´ as ut´an k¨ovetkezik a k´et k´ek szomsz´ed v k¨ or¨ ul, vagy pedig nem. Ha a k´et k´ek cs´ ucs egym´as ut´an k¨ovetkezik, akkor ugyanazt a gondolatmenetet el lehet mondani, mint amikor v-nek 4 szomsz´edja volt. (Ennek ellen˝orz´es´et az olvas´ ora b´ızzuk, de meg´ıg´erj¨ uk hogy a hiba nem ebben a r´eszben van.) Ha a k´et k´ek cs´ ucs nem egym´ast k¨ oveti, akkor feltehet˝o, hogy az ´oramutat´o j´ar´asa szerint ´ıgy n´eznek ki v szomsz´edain a sz´ınek: k´ek, piros, k´ek, s´ arga, z¨old. (Megint csak, ha pl. a s´ arga sz´ın lenne a k´et k´ek k¨ oz¨ ott, akkor cser´elj¨ uk ki mindenhol a s´ arga ´es a piros sz´ıneket stb.). Nevezz¨ uk is el ezeket a cs´ ucsokat K1, P , K2, S, Z-nek. Csin´aljuk a k¨ ovetkez˝ ot: Pr´ ob´aljuk P -t ´ atsz´ınezni s´ arg´ara (majd ennek s´arga szomsz´edait pirosra stb.). Ha S nem sz´ınez˝ odik a´t pirosra, akkor v lehet piros, ´es k´eszen vagyunk. Ha S ´ atsz´ınez˝ odik pirosra, akkor van egy piros-s´ arga u ´t P -b˝ ol S-be, ami v-vel egy¨ utt m´ ar egy k¨ or, ´es ez a k¨ or elv´alasztja a K1 ´es Z cs´ ucsokat a K2-t˝ ol. Ekkor pr´ ob´aljuk meg P -t z¨ oldre a´tsz´ınezni (´es szomsz´edait pirosra stb). Ha Z nem sz´ınez˝ odik a´t pirosra akkor v megint csak lehet piros ´es k´eszen vagyunk. Ha Z ´ atsz´ınez˝ odik pirosra, akkor van egy felv´altva piros-z¨old sz´ın˝ uu ´t P -b˝ ol Z-be, utt m´ ar egy k¨ or, ´es ami elv´ alasztja K1-et a K2 ´es S cs´ ucsokt´ ol. ami v-vel egy¨ Ha ez a helyzet, teh´at a P cs´ ucsot sem s´ arg´ara, sem z¨oldre nem siker¨ ult a´tsz´ınezni, akkor ink´abb hagyjuk a P cs´ ucsot, ´es pr´ ob´aljuk meg a k´ek sz´ınt szabadd´ a tenni. M´egpedig a K1 cs´ ucsot sz´ınezz¨ uk a´t s´ arg´ara. Mivel a piros-z¨old k¨or elv´alasztja K1-et S-t˝ ol, ez´ert ha K1-et s´ arg´ara sz´ınezz¨ uk, akkor S nem fog ´atsz´ınez˝ odni k´ekre. A K2 cs´ ucsot pedig sz´ınezz¨ uk ´ at z¨ oldre. Mivel a piros-s´ arga k¨or elv´alasztja at a v szomsz´edai a K2-t Z-t˝ ol, ez´ert ekkor a Z nem fog ´atsz´ınez˝ odni k´ekre. Teh´ k¨ oz¨ ul el tudtuk t¨ untetni a k´ek sz´ınt. ´Igy most a v cs´ ucsot kisz´ınezhetj¨ uk k´ekre. Ezzel minden esetben tal´ altunk megfelel˝ o sz´ınt a v cs´ ucsnak, azaz a bizony´ıt´ assal k´eszen vagyunk. Hol lehet a hiba? Arra biztatjuk az olvas´ ot, hogy keresse meg a hib´ at a fenti ´ervel´esben. Annyit el´arulunk, hogy az utols´ o esettel van a gond, teh´ at amikor v-nek 5 szomsz´edja van, ´es a k´et azonos sz´ınt kap´ o szomsz´ed nem egym´ as ut´an k¨ovetkezik v k¨ or¨ ul. A k¨ ovetkez˝ o r´eszben mi is le´ırjuk majd, hogy hol a hiba, ´es hogy ha a n´egysz´ınt´etel nem is j¨ on ki, hogyan lehet m´egis Kempe ´ervel´ese seg´ıts´eg´evel bel´ atni az ¨otsz´ınt´etelt. K¨ oz´ episkolai Matematikai ´ es Fizikai Lapok, 2022/6

331

T´ othm´er´esz Lilla ELTE

Gyakorl´o feladatsor emelt szint˝ u matematika ´eretts´egire I. r´esz 1. Oldjuk meg a k¨ ovetkez˝ o egyenleteket a val´ os sz´amok halmaz´an: a) x · (1 − lg 5) = 2 · lg(2x − 2), b) 1 + 2 · cos2 x = sin(2x).

(7 pont) (6 pont)

2. Az e egyenes egyenlete 4x − 3y = 15. Mennyi a sugara annak a k¨ ornek, amely ´erinti az e egyenest, tov´ abb´ a az orig´ oban ´erinti az y tengelyt? (12 pont) 3. A F´ ab´ol Vaskarika Kft. log´oj´an l´athat´ o ABCD n´egyzet oldalai 4 cm hossz´ uak, a BE k¨ or´ıv k¨ oz´eppontja a D pont, az ED k¨ or´ıv k¨ oz´eppontja pedig a B pont. A log´ o mind a n´egy r´esz´et pirosra, k´ekre, s´ arg´ara vagy z¨ oldre festik u ´gy, hogy ha k´et r´esz ker¨ ulet´enek van k¨ oz¨ os szakasza, akkor azok k¨ ul¨ onb¨ oz˝ o sz´ın˝ uek lesznek. a) H´ any n´egyzetcentim´eter a lefestend˝ o ter¨ ulet? (6 pont) b) H´any k¨ ul¨ onb¨ oz˝ o kifest´es lehets´eges? (8 pont) 4. Az iskolai focicsapat edz´es´en az edz˝o megk´erdezett minden jelenl´ev˝ o di´akot, hogy h´ any oszt´ alyt´arsuk tagja a csapatnak. Ketten 1-et, hatan 2-t, egyvalaki 3-at, arman 5-¨ ot v´ alaszoltak a k´erd´esre. ¨oten 4-et ´es h´ a) Mennyi az elhangzott v´alaszok m´ odusza, medi´anja, ´atlaga ´es terjedelme? (7 pont) b) Miut´an a matematika–testnevel´es szakos edz˝o nagyon elcsod´alkozott a v´ alaszokat hallva, a csapat tagjai el´arult´ ak neki, hogy n´eh´ any csapattag matematikaversenyre ment, ez´ert nem tudott elj¨ onni a mai edz´esre. Legal´ abb h´ anyan hi´ anyoztak? (5 pont) II. r´esz 5. A tavasszal h´ arom nagy sportversenyt rendeztek a nekeresdfalvi ´altal´ anos iskol´aban. Az asztalitenisz bajnoks´ agban 120 gyerek indult. A focilig´aba 8 csapat 332

K¨ oz´ episkolai Matematikai ´ es Fizikai Lapok, 2022/6

nevezett, mindegyik csapatban 9 j´at´ekos l´epett p´ aly´ ara. Az u ´sz´ oversenyen 81-en ¨ teljes´ıtett´ek a t´ avot. 23 gyerek mind a h´ arom sport´agban versenyzett. Osszesen 45-en voltak azok, akik egyn´el t¨ obb sz´ amban is indultak.

a) H´any olyan sz´am¨ ot¨ os van, amelyben a sz´ amok sz´amtani sorozatot alkotnak? (6 pont) b) H´any olyan sz´am¨ ot¨ os van, amelyben a sz´ amok m´ertani sorozatot alkotnak? (10 pont)

a) H´ anyan vettek r´eszt legal´ abb az egyik sport´ ag verseny´en? (6 pont) A focilig´aban mindegyik csapat mindegyik m´asik csapattal egyszer j´ atszott. Az els˝o h´eten o sszesen 13 m´ e rk˝ o z´ e sre ker¨ u lt sor. ¨ b) Bizony´ıtsuk be, hogy volt olyan csapat, amely az els˝o h´eten legal´ abb n´egyszer l´epett p´ aly´ ara. (4 pont) Az asztalitenisz bajnoks´agban n´egy olyan gyerek jutott be a legjobb nyolc k¨ oz´e, aki a Futrinka utc´ aban lakik. A versenyz˝okb˝ ol sorsol´assal n´egy p´ art alkottak, akik megk¨ uzdhettek egym´ assal a legjobb n´egy k¨ oz´e jut´ as´ert. c) Mennyi a val´ osz´ın˝ us´ege, hogy nem volt olyan p´ar, ahol mindk´et gyerek a Futrinka utc´ aban lakik? (6 pont) 6. a) Egy szab´alyos hatsz¨ og alap´ u egyenes has´ ab alap´elei 6 cm, oldal´elei 5 cm hossz´ uak. Milyen hossz´ u test´ atl´ oi vannak a has´ abnak, ´es melyik fajt´ab´ol h´ any darab? (8 pont) osz´ın˝ us´ege, hogy egy szab´ alyos n oldal´ u has´ ab cs´ ub) Legyen pn annak a val´ csai k¨ oz¨ ul kett˝ ot v´eletlenszer˝ uen kiv´ alasztva, azok egy lap´atl´ o v´egpontjai lesznek. Hat´ arozzuk meg a k¨ ovetkez˝ o hat´ ar´ert´eket: lim pn .

n→∞

(8 pont)

7. Vizsg´aljuk az an = n2 + 4n + 9 sorozatot. a) Bizony´ıtsuk be, hogy a sorozat szigor´ uan monoton n¨ov˝ o. (3 pont) b) Bizony´ıtsuk be, hogy a sorozatnak egyik tagja sem n´egyzetsz´am. (5 pont) c) H´ anyf´elek´eppen lehet a sorozat els˝ o 100 tagja k¨oz¨ ul kiv´ alasztani k´et k¨ ul¨onb¨ oz˝ ot u ´gy, hogy azok ¨osszege oszthat´ o legyen 5-tel? (8 pont) 8. Egy kis tehersz´ all´ıt´ o haj´ o 100 kilom´eteren 0,1 · v 2 liter g´azolajat fogyaszt, ha a sebess´ege v km/h. Egy liter g´ azolaj a´ra 500 forint, a leg´enys´eg fizet´ese pedig or´ ´ ank´ent ¨ osszesen 27 000 forint. a) Milyen sebess´eggel haladjon a haj´ o, hogy a lehet˝o legolcs´ obban tudja teljes´ıteni az 500 km hossz´ u t´ avot? (10 pont) b) A haj´ o t¨ obbek k¨ oz¨ ott 15 birk´ at sz´all´ıt. Sajnos, az egyik birk´at betegen vitt´ek fel a fed´elzetre. Tudjuk, hogy ez a beteg birka b´ armelyik eg´eszs´eges birk´ at 10% val´osz´ın˝ us´eggel fert˝ ozi meg. Az ´ıgy megfert˝ oz¨ ott birk´ ak m´ar nem fert˝oznek tov´ abb. Mennyi a val´ osz´ın˝ us´ege, hogy legfeljebb 2 tov´abbi birka kapja el a fert˝oz´est? (6 pont) 9. A hagyom´anyos, u ´gynevezett ¨ ot¨ os lott´ on a szelv´enyen 1-t˝ ol 90-ig szerepelnek a sz´ amok, ´es ezek k¨ oz¨ ul kell kiv´alasztani azt az 5 sz´ amot, amit a sorsol´ ason kih´ uznak. K¨ oz´ episkolai Matematikai ´ es Fizikai Lapok, 2022/6

333

Erd˝os G´abor Nagykanizsa

T´aj´ekoztat´ o a foly´oirat el˝ ofizet´es´er˝ ol

A K¨oz´episkolai Matematikai ´es Fizikai Lapok megrendelhet˝o a kiad´ on´ al, a MATFUND Alap´ıtv´ anyn´al a szerkeszt˝os´eg c´ım´en; valamint a k¨ ovetkez˝ o c´ımen: http://www.komal.hu/megrendelolap/reszletek.h.shtml. El˝ofizet´esi d´ıj a 2022–2023-as tan´evre (2022 szeptember´et˝ ol 2023 m´ ajus´aig) 9200 Ft. Azonos c´ımre k¨ uldend˝o, 9-n´el nagyobb p´eld´ anysz´am´ u megrendel´es eset´en a csoportos el˝ofizet´esi d´ıj r´eszletes a´rai a fenti oldalon olvashat´ ok. Csekket ´es sz´ aml´ at a szeptemberi sz´ ammal egy¨ utt k¨ uld¨ unk, a fizet´es csak ezut´an t¨ ort´enhet. Lapunk el˝ ofizet˝ oi az el˝ ofizetett p´eld´ any c´ımlapj´ an l´ athat´ o el˝ ofizet˝ oi azonos´ıt´ o seg´ıts´eg´evel a kit˝ uz¨ ott feladatainkhoz m´ ar a lap nyomtatott v´ altozat´ anak megjelen´es´evel egyidej˝ uleg hozz´ af´erhetnek. o kedA Bolyai J´ anos Matematikai T´ arsulat (BJMT) tagjai ´ altal ig´enybevehet˝ vezm´enyekr˝ ol k´erj¨ uk, olvassa el a T´ arsulat honlapj´ an a Tags´ agi inform´ aci´ ok”-at: ” www.bolyai.hu. Azok, akik az id´en k´erik felv´etel¨ uket a Bolyai J´ anos Matematikai T´arsulatba, felv´eteli k´erelm¨ uk elb´ır´ al´asa ut´ an (legk¨ ozelebb v´arhat´ oan okt´ oberben) ´ertes´ıt´est ´es tagd´ıjbefizet´esi csekket kapnak, ez´ert k¨ ul¨ on nem sz¨ uks´eges el˝ obb jelentkezni¨ uk. A K¨oz´episkolai Matematikai ´es Fizikai Lapok p´eld´ anyonk´enti a´ra 1100 Ft. K´erj¨ uk versenyz˝ oinket, hogy a K¨ oMaL 2022–2023-as tan´evi matematika, fizika ´es informatika pontverseny´enek versenyki´ır´ as´ at figyelmesen olvass´ak el!

Versenyki´ır´as∗ a K¨oMaL 2022–2023. ´evi pontversenyeire Kedves Versenyz˝ onk! Matematik´ ab´ol, fizik´ ab´ ol ´es informatik´ ab´ ol k¨ ul¨ onf´ele neh´ezs´eg˝ u pontversenyeket ind´ıtunk. A 2021–2022-es tan´evt˝ol kezdve csapatban is lehet versenyezni, mely∗

K´erj¨ uk, hogy azok is figyelmesen olvass´ ak el a versenyki´ır´ ast, akik tavaly m´ ar r´eszt vettek valamelyik verseny¨ unkben.

334

K¨ oz´ episkolai Matematikai ´ es Fizikai Lapok, 2022/6

nek r´eszletei a tov´ abbiakban olvashat´ ok. A versenyek 9 h´ onapon kereszt¨ ul, 2022 szeptember´et˝ ol 2023. j´ unius elej´eig tartanak. Minden h´ onapban u ´j feladatokat t˝ uz¨ unk ki, ´es a megold´ asokat a k¨ ovetkez˝ o h´ onap elej´eig k¨ uldheted be. A verseny v´egeredm´eny´et 2023. szeptemberi sz´ amunkban hirdetj¨ uk ki. A d´ıjakat j¨ov˝ o o˝sszel, a K¨ oMaL Ifj´ us´agi Ank´eton adjuk a´t. Pontversenyeinkben a r´eszv´etel a 2022/2023-as tan´evben is t´er´ıt´esmentes. K´erj¨ uk azonban versenyz˝oink sz¨ uleit, hozz´ atartoz´ oit, vagy az o˝ket t´ amogat´ o int´ezm´enyeket, c´egeket, hogy el˝ ofizet´es¨ ukkel ´es adom´ anyaikkal seg´ıts´ek Lapunk fennmarad´as´ at. Nevez´es a versenyre Versenyeinkben minden a´ltal´ anos iskol´as ´es k¨oz´episkol´as kor´ u tanul´ o r´eszt vehet. ´ uAz Eur´ opai Uni´ o Altal´ anos Adatv´edelmi Rendelete (GDPR) ´ertelm´eben sz¨ l˝oi enged´ely sz¨ uks´eges a 16 ´evesn´el fiatalabb versenyz˝ oink adatainak nyilv´ antart´ as´ ahoz. Az o˝ eset¨ ukben egy sz¨ ul˝ oi nyilatkozatra is sz¨ uks´eg van, melyet a regisztr´ aci´ o sor´ an lehet megadni. Amennyiben a sz¨ ul˝ oi nyilatkozat nem ´erkezik meg, a versenyz˝ o nem szerepelhet az eredm´enylist´ aban. Adatkezel´esi szab´alyzatunk a https://www.komal.hu/info/adatkezeles.h.shtml c´ımen olvashat´ o. Regisztr´aci´ o Ha m´eg soha nem vett´el r´eszt a K¨ oMaL versenyeiben, az els˝ o l´ep´es a regisztr´ aci´ o a honlapunkon (https://www.komal.hu/u?a=reg). A regisztr´ aci´ o sor´ an alapvet˝ o adatokat (n´ev, sz¨ ulet´esi d´ atum, iskola, oszt´ aly, e-mail c´ım) k´er¨ unk. A k´es˝obbi bejelentkez´eshez sz¨ uks´eges jelszavadat e-mailben k¨ uldj¨ uk el. A nagyon gyakori csal´ adnev˝ u versenyz˝ oknek (Horv´ ath, Kiss, Varga stb.) javaasodik vezet´ekn´evsoljuk, hogy v´ alasszanak egy h´aromjegy˝ u jelz˝ osz´amot, amit m´ k´ent haszn´ alnak (pl. Kiss 349 Anna, Szab´o 344 P´eter). K´erj¨ uk, hogy mind a regisztr´ aci´ okor, mind pedig a tan´ev sor´ an bek¨ uld¨ ott dolgozataidon is minden esetben az ´ıgy kib˝ ov´ıtett nevet haszn´ald. A sikeres regisztr´aci´ o ut´ an adhatod meg tov´ abbi adataidat (pl. felk´esz´ıt˝otan´ arok neve; levelez´esi c´ım: ide szoktuk k¨ uldeni az ´eretts´egizettek oklevel´et), ´es nyilatkozhatsz a r´eszletes pontsz´amok nyilv´anoss´ ag´ ar´ ol vagy egyes konkr´et versenyekben val´o r´eszv´etelr˝ ol. Ha kor´ abban m´ ar regisztr´alt´ al, akkor nincs sz¨ uks´eg u ´jabb regisztr´ aci´ ora; a tavalyi jelszavadat tov´abbra is haszn´alhatod; ugyanakkor sz¨ uks´eges lesz a szem´elyes be´all´ıt´ asaid a´ttekint´ese, fel¨ ulvizsg´alata. Az egyes pontversenyekre az els˝ o dolgozat bek¨ uld´es´evel nevezhetsz be. A versenyekbe a tan´ev sor´ an k´es˝ obb is be lehet kapcsol´ odni. FONTOS! A versenyben csak a regisztr´aci´ o ut´an, a Munkafu ¨zetbe be´ırt vagy felt¨ olt¨ ott megold´asokat ´ert´ekelju o n´elku ¨k! A regisztr´aci´ ¨l, post´an vagy e-mailben beku ott megold´asokat ut´olag sem vesszu ¨ld¨ ¨k figyelembe! Az oszt´alyok sz´amoz´asa A K¨ oMaL versenyeiben az oszt´alyokat 5-t˝ ol 12-ig sz´ amozzuk. Lehet, hogy a sz´ amoz´ as nem azonos az iskol´ aban haszn´alt sz´ ammal. Azok sz´am´ıtanak 12. oszt´ alyosnak, akik most kezdik az ´eretts´egi vizsga el˝otti utols´ o ´evet. 11. ´es 10. oszt´ alyosnak K¨ oz´ episkolai Matematikai ´ es Fizikai Lapok, 2022/6

335

sz´ am´ıtanak azok, akik v´arhat´ oan 2023-ban, illetve 2024-ben fejezik be a k¨ oz´episkol´at. Azok, akik 8 + 5 ´eves k´epz´esben vesznek r´eszt, p´eld´ aul a nyelvi el˝ok´esz´ıt˝ o oszt´alyok tanul´oi, k´et egym´ as ut´ani ´evben is 9. oszt´ alyosnak sz´am´ıtanak. K´erj¨ uk, ha az oszt´ alyod sorsz´ ama nem 1-gyel n˝ ott tavalyhoz k´epest, ezt jelezd a szerkeszt˝os´egnek e-mailben. A regisztr´aci´ o m´ odos´ıt´asa A regisztr´ aci´ o ut´ an az azonos´ıt´ ashoz sz¨ uks´eges adataidat (n´ev, iskola, oszt´ aly, e-mail c´ım) ¨on´ all´ oan nem m´odos´ıthatod. Ha ezek megv´ altoztak, k´erj¨ uk, hogy fordulj e-mailben a szerkeszt˝os´eghez. Mindenkit o´vunk a regisztr´ aci´ o¨ onk´enyes megism´etl´es´et˝ ol, a t¨ obbsz¨ or¨ os regisztaci´ o seg´ıtene; csak m´eg r´aci´ ot´ ol. Nincs olyan helyzet, amikor a t¨ obbsz¨ or¨ os regisztr´ nagyobb zavart okoz. (Ugye nem szeretn´el k´etszer szerepelni a pontversenyben, feleakkora pontsz´ ammal?) Arck´epek Ha szeretn´ed, hogy f´enyk´eped megjelenjen honlapunkon a pontverseny eredm´eny´eben, k¨ uldd el a szerkeszt˝ os´egnek e-mailben. Ha lehet, v´ alassz vil´ agos, egysz´ın˝ u h´atteret. A k´epeket t¨ obbnyire a´tm´eretezz¨ uk ´es megfelel˝ o m´eret˝ ure v´ agjuk, ez´ert ´erdemes nagyobb felbont´ast haszn´ alni. Csapatversenyek A 2021/22-es tan´evt˝ ol kezdve csapatok sz´am´ara is meghirdet¨ unk t¨ obb pontversenyt a hagyom´ anyos egy´eni pontversenyek mellett. V´ arjuk 2-3 f˝ os csapatok jelentkez´es´et a C ´es B matematika, az I informatika, a G ´es P fizika, tov´ abb´ a 2 f˝ os csapatok nevez´es´et az M fizika m´er´esi pontversenyekre. A csapatversenyek a´ltal´ anos szab´alyai megegyeznek az egy´eni nevez´es˝ u hagyom´anyos versenyek szab´ alyaival (a versenyek le´ır´ as´ at l´asd lentebb), feladatai megegyeznek az egy´eni verseny feladataival. A csapattagoknak egy´enileg is kell regisztr´ alniuk, ha kor´ abban nem regisztr´altak. Ezut´ an lehet csapatot regisztr´ alni. A tagok lehetnek k¨ ul¨ onb¨ oz˝ o iskol´ ab´ ol ´es k¨ ul¨onb¨oz˝ o ´evfolyamokr´ ol is. Egy csapat abban a kateg´ori´aban fog versenyezni, ami az ´evfolyam szerinti legid˝osebb tagj´anak a kateg´ori´aja. Egy szem´ely t¨ obb csapatnak is tagja lehet, illetve indulhat egy´eni versenyben is, de egy pontversenyben pontosan egyszer vehet r´eszt. Nem lehet versenyezni egyszerre a C csapatversenyben ´es a K, B vagy A egy´eni pontversenyben, illetve a G csapatversenyben ´es a P egy´eni versenyben. A C ´es B csapatversenyeket k´et kateg´ ori´aban: a 9–10. ´evfolyamosok, illetve 11–12. ´evfolyamosok; az I, M ´es P csapatversenyeket egy-egy kateg´ ori´aban: a 9–12. ´evfolyamosok; tov´abb´ a a G csapatversenyt egy kateg´ ori´aban: a 9–10. ´evfolyamosok sz´ am´ara hirdetj¨ uk meg. Matematika versenyek ovekv˝ o neh´ezs´egi sorrendben K, C, B ´es A kateN´egyf´ele versenyt ind´ıtunk, n¨ g´ori´aban. Egy tanul´o t¨ obb pontversenyben is indulhat, de az egy´eni K ´es B pontversenyek k¨oz¨ ul csak az egyiket v´alaszthatja. Ha kilencedik oszt´ alyos vagy, akkor 336

K¨ oz´ episkolai Matematikai ´ es Fizikai Lapok, 2022/6

a szem´elyes be´ all´ıt´ asaid k¨ oz¨ ott nyilatkozhatsz, hogy melyik versenyben szeretn´el r´eszt venni. Mindegyik verseny¨ unkre ´erv´enyes, hogy egy feladatra csak egy megold´ast ´ert´ekelu ¨nk. Term´eszetesen ¨ or¨ommel v´ arunk a´ltal´ anos´ıt´ asokat, megjegyz´eseket, m´asfajta megold´ asi vagy kit˝ uz´esre tett javaslatokat, ezeket sz´ıvesen k¨oz¨olj¨ uk, s˝ ot, a pontversenyen k´ıv¨ ul k¨ ul¨ ond´ıj form´ aj´aban is elismerj¨ uk. Versenyz˝oink akkor kapnak pontot az ´ altaluk javasolt feladatra, ha annak megold´ as´ at – a t¨obbi feladat megold´ as´ ahoz hasonl´ oan – felt¨ oltik a Munkaf¨ uzetbe. K-jel˝ u matematika feladatok – az ABACUS ´es a K¨oMaL K¨oz¨os pontversenye Kilencedikes Kezd˝oknek A K-pontversenyben csak kilencedik oszt´ alyosok indulhatnak. Azoknak aj´ anljuk, akik m´eg csak most ismerkednek a K¨ oMaL-lal. Szeptembert˝ol m´ajusig kilenc fordul´ oban, havonta ¨ ot feladat jelenik meg; ezek k¨oz¨ ul szeptembert˝ ol m´arciusig h´ arom feladat az ABACUS pontverseny´evel k¨ oz¨ os, mely feladatokat az ABACUS matematikai lapok bocs´ atja a K¨ oMaL rendelkez´es´ere. Mindegyik feladat teljes megold´asa 5 pontot ´er. Az ABACUS pontverseny´eben tov´ abbra is az a´ltal´ anos iskol´ak 3–8. oszt´ alyos tanul´ oi vehetnek r´eszt. Azok a 9-edikesek, akik K-ban indulnak, nem lehetnek tagjai C pontversenybe nevezett csapatnak. C-jel˝ u matematika gyakorlatok A C-pontverseny gyakorlatait azoknak az olvas´ oinknak aj´anljuk, akik t´ ul neh´eznek vagy szokatlannak tal´alj´ ak a B ´es az A kateg´ oria feladatait. Itt rendszeresen k¨ ozl¨ unk az iskolai tananyaghoz szorosabban kapcsol´od´o gyakorlatokat, azok tal´ alhatnak itt kedv¨ ukre val´ot, akik valamivel – de nem sokkal – szeretn´enek t´ ull´epni az iskolai matematika keretein, vagy emelt szint˝ u ´eretts´egit k´ıv´ annak tenni matematik´ ab´ ol. A C pontverseny els˝ o k´et feladata megegyezik a K pontverseny utols´ o asra kapott pontsz´ amok mindk´et pontverk´et feladat´aval, jel¨ ol´esu ¨k K/C. A megold´ senybe besz´ am´ıtanak – hasonl´oan az I/S informatika feladatok pontsz´am´ıt´ as´ahoz. A K pontversenyben tov´ abbra is csak 9. ´evfolyamos, illetve nyelvi el˝ ok´esz´ıt˝o ´evfolyamra j´ ar´ o tanul´ ok vehetnek r´eszt, ugyanakkor versenyezhetnek egyszerre mind a K, mind a C pontversenyekben – az ut´ obbiban a 10-edikesekkel egy kateg´ori´aban.

A gyakorlatok egy r´esze ´ altal´ anos iskol´asoknak is aj´ anlhat´ o, m´as r´esz¨ uk azonban a 11–12. ´evfolyam tanulm´anyaira t´ amaszkodik. Minden h´ onapban h´et gyakorlatot t˝ uz¨ unk ki, ebb˝ ol az 1–5. gyakorlatokra a legfeljebb 10. ´evfolyamosok, a 3–7. gyakorlatokra pedig a 11–12. ´evfolyamosok k¨ uldhetnek be megold´ ast. Minden dolgozatra legfeljebb 5 pont kaphat´ o. A C-pontversenyt egy´eniben h´ arom korcsoportban: 5–8., 9–10., illetve 11–12. oszt´ alyosok, csapatban pedig k´et korcsoportban ´ert´ekelj¨ uk: 5–10., illetve 11–12. oszt´ alyosok. Aki a C csapatversenyben indul, nem indulhat egy´enileg sem a K, sem a C, sem a B versenyben (de indulhat a B csapatversenyben). B-jel˝ u matematika feladatok A B-pontversenyben havonta ¨ osszesen nyolc feladatot t˝ uz¨ unk ki, de havonta mindenkinek legfeljebb hat megold´ as´ at sz´am´ıtjuk be a pontversenybe (amelybe azonban el˝osz¨or a nem versenyszer˝ ueket sz´ am´ıtjuk be, l´asd lentebb). Az eredm´enyes versenyz´eshez teh´ at nincs sz¨ uks´eg valamennyi feladat megold´ as´ ara; ki-ki gondolja v´egig, mely p´eld´ akkal foglalkozna sz´ıvesen, hogyan ´erhetn´e el a legt¨ obb pontot. ul A B-feladatok sorrendje megfelel az iskolai tananyagnak: egy feladatsoron bel¨ az alacsonyabb sorsz´ am´ uakat aj´ anljuk a fiatalabbaknak. A feladatok – sz´ and´ekaink szerinti – neh´ezs´eg´et a k¨ oz¨ olt pontsz´ am jelzi (t¨ obbnyire 3–6). Az egy´eni B-pontverseny eredm´eny´et ¨ ot korcsoportban tartjuk nyilv´ an: a 8. ´evfolyamig, a 9., 10., 11., illetve 12. ´evfolyamokban; a csapatverseny´et pedig k´et korcsoportban ´ert´ekelj¨ uk: 5–10., illetve 11–12. oszt´ alyosok. Aki a B csapatversenyben indul, nem indulhat egy´enileg a B versenyben. A-jel˝ u nehezebb matematika feladatok Az A-pontverseny a legfelk´esz¨ ultebb di´akok sz´am´ara jelent kih´ıv´ ast. Azoknak aj´anljuk, akik tudom´ anyos kutat´ o p´ aly´ ara vagy nemzetk¨ ozi versenyekre k´esz¨ ulnek. Havonta k´et vagy h´ arom A-feladatot t˝ uz¨ unk ki, mindegyik feladatra legfeljebb 7 pont kaphat´ o. Az A-verseny r´esztvev˝ oit nem k¨ ul¨ on´ıtj¨ uk el ´evfolyamonk´ent, mindannyian egy¨ utt versenyeznek. Fizika versenyek ori´aban. Egy tanul´o t¨ obb H´aromf´ele fizika versenyt ind´ıtunk: M, G ´es P kateg´ pontversenyben is indulhat, de az egy´eni G ´es P pontversenyek k¨ oz¨ ul csak az egyiket v´alaszthatja. A legfeljebb 10. oszt´ alyosoknak honlapunkon, a szem´elyes be´ all´ıt´ asaik k¨oz¨ott kell nyilatkozniuk, hogy a P- ´es G-versenyek k¨ oz¨ ul melyikben k´ıv´ annak r´eszt venni. Term´eszetesen ¨ or¨ ommel v´ arunk a´ltal´ anos´ıt´ asokat, megjegyz´eseket, m´asfajta megold´ asi vagy kit˝ uz´esre tett javaslatokat, ezeket sz´ıvesen k¨ oz¨ olj¨ uk, s˝ ot, a pontversenyen k´ıv¨ ul k¨ ul¨ ond´ıj form´ aj´aban is elismerj¨ uk. Versenyz˝oink akkor kapnak pontot az a´ltaluk javasolt feladatra, ha annak megold´ as´ at – a t¨ obbi feladat megold´ as´ahoz hasonl´ oan – felt¨ oltik a Munkaf¨ uzetbe. M-pontverseny – fizika m´er´esi feladatok Havonta egy m´er´esi feladatot t˝ uz¨ unk ki, valamennyi koroszt´ aly sz´ am´ara k¨ oz¨ osen. A feladatok megold´ as´ aval 6-6 pontot lehet szerezni. A m´er´es elv´egz´es´ehez egy´eni versenyz˝ ok´ent is szabad egy szem´ely (csal´ adtag, oszt´ alyt´ ars, bar´ at) seg´ıts´eg´et is ig´enybe venni. A seg´ıt˝ o szem´ely adatait a m´er´esi

K¨ oz´ episkolai Matematikai ´ es Fizikai Lapok, 2022/6

337

338

K¨ oz´ episkolai Matematikai ´ es Fizikai Lapok, 2022/6

jegyz˝ ok¨ onyv elej´en a versenyz˝o adatai mellett t¨ untess´etek fel. Lehet k´etf˝os csapatban is indulni a versenyen, azaz az egy´eni versenyz˝ok ´es a csapatok egy k¨oz¨os versenyen indulnak. Aki egy csapat tagjak´ent indul az M versenyben, nem versenyezhet egy´enileg is, csak m´ asik versenyben. G-jel˝ u fizika gyakorlatok A G-pontversenyben legfeljebb 10. oszt´ alyosok vehetnek r´eszt. Azoknak az olvas´ oinknak aj´anljuk, akik t´ ul neh´eznek vagy szokatlannak tal´alj´ ak a P-feladatokat. T¨ obbnyire az iskolai tananyaghoz szorosabban kapcsol´ od´o gyakorlatokat tal´ alnak a versenyz˝ ok, ´ıgy azok is es´ellyel indulhatnak, akik m´eg nem rendelkeznek feladatmegold´ o rutinnal, de a gyakorlatok megold´as´ aval ´es bek¨ uld´es´evel felk´esz¨ ulhetnek arra, hogy a k¨ ovetkez˝ o ´evekben eredm´enyesen szerepelhessenek a P-pontversenyben. Minden h´ onapban n´egy G-gyakorlatot t˝ uz¨ unk ki, az el´erhet˝ o pontsz´ amokat a feladatok ut´an felt¨ untetj¨ uk. Mindenki szabadon v´alaszthat a kit˝ uz¨ott gyakorlaas´ at (el˝osz¨or a nem versenytok k¨ oz¨ ul, de havonta legfeljebb h´arom feladat megold´ szer˝ ueket) sz´ am´ıtjuk be a pontversenybe. Az egy´eni G-pontverseny eredm´eny´et h´ arom korcsoportban tartjuk nyilv´ an: a 8. ´evfolyamig, a 9. ´es 10. ´evfolyamokban; a csapatverseny´et pedig egyetlen korcsoportban: 5–10. oszt´alyosok. Aki a G csapatversenyben indul, nem indulhat egy´enileg sem a G, sem a P versenyben (de indulhat a P csapatversenyben). P-jel˝ u fizika feladatok Havonta nyolc (esetenk´ent kilenc) elm´eleti feladatot t˝ uz¨ unk ki, nem neh´ezs´egi, hanem az ´eletkornak megfelel˝o sorrendben. A pontsz´ amokat a feladatok ut´an felt¨ untetj¨ uk. Mindenki szabadon v´alaszthat a kit˝ uz¨ott elm´eleti feladatok k¨oz¨ ul. Az 5–8. ´evfolyamosoknak havonta legfeljebb h´arom, a 9–12. ´evfolyamosoknak legfeljebb n´egy megold´as´at sz´ am´ıtjuk be a pontversenybe (azonban el˝osz¨or a nem versenyszer˝ ueket). Az elm´eleti versenyt egy´enileg koroszt´ alyonk´ent (8. ´evfolyamig, 9., 10., 11., 12. uk ´es ´ert´ekelj¨ uk, a csapatverseny´et pedig egyetlen ´evfolyam) k¨ ul¨ on-k¨ ul¨ on ¨ osszes´ıtj¨ korcsoportban: 5–12. oszt´alyosok. Aki a P csapatversenyben indul, nem indulhat egy´enileg a P versenyben. Informatika versenyek I-pontverseny – informatika alkalmaz´ asi ´es programoz´ asi feladatok Havonta h´ arom I jel˝ u ´es egy I/S jel˝ u feladatot t˝ uz¨ unk ki valamennyi koroszt´ aly sz´ am´ara k¨ oz¨ osen. Mindegyik feladat 10 pontot ´er. A feladatok egy r´esze a´ltal´ anos iskol´asoknak is aj´ anlhat´ o, nagyobb r´esze azonban a k¨oz´episkolai tanulm´anyokra t´ amaszkodik. Alapvet˝ o c´elunk, hogy e feladatok seg´ıts´ek a felk´esz¨ ul´est az informatika versenyekre ´es az emelt szint˝ u ´eretts´egire. Minden h´ onapban a n´egy kit˝ uz¨ott feladatb´ ol a h´ arom legmagasabb pontsz´ amot el´ert feladat pontsz´am´at sz´am´ıtjuk be az I-pontversenybe. Az I jel˝ u feladatok programoz´asi ´es informatika alkalmaz´ oi feladatok. A feladatok egyike jelleg´eben ´es form´ aj´aban is l´enyeg´eben megegyezik az ´eretts´egin kit˝ uz¨ott ´ bet˝ feladatokkal, ezt az (E) uvel jelezz¨ uk a feladat sorsz´ama mellett. Versenyz˝oink ezen feladatok megold´ as´ aval a vizsg´ara val´ o felk´esz¨ ul´est is gyakorolhatj´ ak. K¨ oz´ episkolai Matematikai ´ es Fizikai Lapok, 2022/6

339

Az I/S jel˝ u feladatok az I jel˝ u programoz´ asi feladatokn´ al nehezebb, de az S jel˝ uekn´el k¨onnyebb programoz´ asi feladatok. A megold´ ashoz sz¨ uks´eges ismeretek ´es algoritmusok megtal´ alhat´ ok a http://tehetseg.inf.elte.hu/nemes ´es a https:// www.oktatas.hu/pub_bin/dload/kozoktatas/tanulmanyi_versenyek/oktv/ oktv2022_2023_vk/116_informatika_2223.pdf oldalakon. Aki az I csapatversenyben indul, nem indulhat egy´enileg sem az I, sem az S versenyben. S-pontverseny – nehezebb programoz´ asi feladatok Az S-pontverseny egy S jel˝ u nehezebb programoz´ asi feladatb´ ol ´es az I-pontversenyben is r´esztvev˝ o I/S feladatb´ ol a´ll. A feladatokra legfeljebb 10 pont kaphat´ o. Mindk´et feladat a programoz´ asi versenyekre val´ o felk´esz¨ ul´est szolg´ alja. A megold´ ashoz sz¨ uks´eges ismeretek ´es aj´ anlott algoritmusok k¨ or´et a Nemzetk¨ ozi Informatikai Di´akolimpi´akon alkalmazott angol nyelv˝ u le´ır´ as (IOI Syllabus) tartalmazza, l´ asd https://people.ksp.sk/~misof/ioi-syllabus/. Az S ´es I/S feladatok ´ert´ekel´eul azt is figyelembe vessz¨ uk, hogy az algoritmusok s´en´el az eredm´eny helyess´eg´en k´ıv¨ mennyire hat´ekonyak, nagym´eret˝ u bemen˝ o adatok eset´en is lefutnak-e a megadott id˝ okorl´aton bel¨ ul. Az S pontversenyt egy kateg´ ori´ aban (5–12. ´evfolyam) ´ert´ekelj¨ uk. A feladatok megjelen´ese ´ feladatokat havonta, szeptembert˝ol m´ajusig t˝ Uj uz¨ unk ki. A feladatokat megtal´ alod nyomtatott sz´ amunkban ´es honlapunkon. Honlapunkon a feladatokat, szeptember kiv´etel´evel, az adott h´ onap 28. napj´an hozzuk nyilv´ anoss´ agra. El˝ ofizet˝ oink azonban az adott t´ıpus´ u feladat bek¨ uld´esi hat´ aridej´et k¨ ovet˝o napt´ ol el´erhetik a k¨ ovetkez˝ o havi feladatok sz¨ oveg´et, ´es elkezdhetik a munk´at. Amennyiben el˝ ofizett´el a K¨ oMaL-ra, a szem´elyes be´ all´ıt´ asaid k¨ oz¨ ott add meg el˝ ofizet˝oi k´ ododat. Az el˝ ofizet˝ oi azonos´ıt´ ot megtal´alod a szeptemberi sz´ am c´ımlapj´ara r´ aragasztott c´ımk´en. Azok az el˝ofizet˝ oink, akik (p´eld´ aul ´eletkorukn´ al fogva) nem versenyz˝oink, regisztr´ aci´ o ´es az el˝ofizet˝ oi k´ od megad´asa ut´ an, a versenyz˝okkel egy¨ utt szint´en el´erhetik a feladatok sz¨ oveg´et. Egy el˝ ofizet˝ oi k´ odot csak egy szem´ely haszn´ alhat. A dolgozatok tartalma uk, tanulm´ anyozd a kor´ abbi sz´ amainkban ´es honlapunkon megjelent megK´erj¨ old´ asokat, ezek sokat seg´ıthetnek annak meg´ert´es´eben, hogy milyen form´ at ´es r´eszletess´eget v´ arunk el a bek¨ uld¨ ott megold´ asokt´ ol. Matematika ´es fizika elm´eleti megold´asok A megold´ as le´ır´ asa azt jelenti, hogy az olvas´ ot v´egigvezeted a megold´ asod l´ep´esein. T¨orekedj a r¨ ovid, olvashat´ o le´ır´ asra. Pr´ ob´ald alaposan ´atgondolni a l´ep´esek 340

K¨ oz´ episkolai Matematikai ´ es Fizikai Lapok, 2022/6

sorrendj´et, ´es ler¨ ovid´ıteni a megold´ ast. A gondos le´ır´ as sok id˝ot ig´enyel; ne hagyd az utols´o pillanatra. Maxim´alis pontsz´ am csak teljes megold´as´ert j´ar; puszta eredm´enyk¨ozl´es´ert nem adunk pontot. A kimondott a´ll´ıt´ asokat igazolni kell. Levezet´es ´es hivatkoz´ as n´elk¨ ul csak a k¨ oz´episkolai tananyagban szerepl˝ o t´eteleket fogadjuk el. K¨ozismert t´etelekre (pl. Menelaosz-t´etel, H¨ older-egyenl˝otlens´eg stb.) elegend˝ o a nev¨ ukkel hivatkozni, egy´eb esetekben ki kell mondani a felhaszn´ alt t´etelt, ´es fel kell t¨ untetni az id´ezett forr´ ast (c´ım, oldalsz´am vagy internet-c´ım). T´etelekre val´ o hivatkoz´ askor azt is meg kell mutatni, mi´ert teljes¨ ulnek a t´etel felt´etelei, ´es hogyan k¨ovetkezik a t´etel a´ll´ıt´ as´ab´ol a bizony´ıt´ as gondolatmenet´enek k¨ovetkez˝ o l´ep´ese. T¨ obbsz¨ or el˝ofordult m´ar, hogy egy-egy feladat szerepelt valamely p´eldat´ arban, vagy megtal´alt´ ak az interneten. Arra is l´attunk p´eld´ at, hogy egy foly´ oiratcikkben, vagy ´eppen a K¨ oMaL egy kor´abbi feladat´ aban a feladatban kit˝ uz¨ottel l´enyeg´eben ekvivalens, vagy ann´ al a´ltal´ anosabb ´all´ıt´as bizony´ıt´asa szerepelt. C´elunk tov´abbra is versenyz˝ oink probl´emamegold´ o k´epess´eg´enek fejleszt´ese, nem pedig a keres˝ oprogramok tesztel´ese, ez´ert nem adunk pontot azokra a dolgozatokra, amelyek csak a megold´as hely´et k¨ ozlik, vagy azt mutatj´ak meg, hogy a feladat egy nehezebb t´etel speci´alis esete vagy trivi´alis k¨ ovetkezm´enye; a v´egeredm´enyhez vezet˝ o megold´ast r´eszletesen le kell ´ırni. Ha a megold´ ashoz k¨ onyvekben vagy az interneten tal´alt ´ır´asokat haszn´alsz fel, ´es ezekb˝ol id´ezel, t¨ untesd fel a felhaszn´ alt forr´ asokat. A fizika feladatokn´al el˝ ofordulhat, hogy a feladat sz¨ovege nem tartalmaz a numerikus megold´ ashoz sz¨ uks´eges minden konkr´et inform´aci´ ot, p´eld´ aul bizonyos anyau ´ert´ekeit. Ilyengi ´ alland´ okat, f¨ oldrajzi vagy csillag´aszati mennyis´egek sz´amszer˝ kor vagy a N´egyjegy˝ u f¨ uggv´enyt´ abl´ azatokban, vagy az interneten kereshetj¨ uk meg a sz¨ uks´eges adatokat. A hosszabb, ¨ osszetettebb gondolatmeneteket ´erdemes tagolni, r´eszekre bontani; haszn´ alj, bekezd´eseket, c´ımeket ´es alc´ımeket. A k¨ ul¨onb¨oz˝ o seg´ed´ all´ıt´ asokra, k´epletekre ´es ´abr´ akra k¨ onnyebb hivatkozni, ha megsz´ amozod.

M´er´esi feladatok A m´er´es le´ır´ asa (m´er´esi jegyz˝ ok¨ onyv) felt´etlen¨ ul tartalmazza a m´er´es elv´enek a´ttekinthet˝ o le´ır´ as´ at (a m´er´esi elrendez´es v´ azlatos rajz´ aval, esetleg fot´okkal), megfelel˝o sz´ am´ u ´es pontoss´ ag´ u m´er´esi adatot (´ attekinthet˝o t´ abl´ azatban, a m´ert´ekegys´egeket is megadva), a m´er´esi adatok ki´ert´ekel´es´et (lehet˝ oleg grafikusan ´abr´ azolva), ´es a hiba nagys´ agrendj´enek becsl´es´et. A m´ert ´es sz´ am´ıtott mennyis´egeket ne adjuk meg indokolatlanul sok tizedesjeggyel, hanem csak a becs¨ ult hib´ aval o ¨sszhangban ´all´o pontoss´aggal. A m´er´esi jegyz˝ ok¨ onyv legyen viszonylag t¨ om¨ or, de annyira a´ttekinthet˝ o, hogy annak alapj´an b´ arki meg tudja ism´etelni a le´ırt m´er´est. Nagyon sok o azoknak csak egy reprezentat´ıv” r´esz´et (50-n´el t¨obb) m´er´esi adat eset´en elegend˝ ” bek¨ uldeni ´es a t¨ obbinek csak az ´atlag´ at k¨ oz¨ olni. A 6 oldaln´ al hosszabb jegyz˝ok¨ onyv tartalmazzon egy r¨ ovid (kb. 1/2 oldalas) o sszefoglal´ a st. ¨ Informatika megold´asok Az I-jel˝ u programoz´ asi feladatok megold´ as´ at Basic, C++, C#, Java, Pascal vagy Python nyelvek egyik´en kell elk´esz´ıtened. A fejleszt´eshez b´ armilyen fejleszt˝ ok¨ornyezetet haszn´ alhatsz, javasoljuk az Oktat´ asi Hivatal honlapj´an el´erhet˝o emelt szint˝ u ´eretts´egi szoftverlista fejleszt˝ oeszk¨ ozeit. Az I-pontversenyben kit˝ uz¨ ott alkalmaz´ oi feladatok megold´ as´ ahoz szint´en az el˝obbi szoftverlista eszk¨ ozeit javasoljuk. Az alkalmaz´ oi feladatokat a list´ an szerepl˝o alkalmaz´ asokkal fogjuk ´ert´ekelni. Az egy´eb haszn´ alhat´ o alkalmaz´ asokat egyegy feladat le´ır´ asa tartalmazza, ezek j´ or´eszt szabadon f¨ olhaszn´ alhat´ o programok. u feladatok megold´ as´ at C, C++, Pascal vagy Java nyelvek vaAz I/S ´es S-jel˝ lamelyik´en kell elk´esz´ıtened. A megold´ ashoz dokument´ aci´ ot kell ´ırnod ´es a forr´ ask´odot kommentekkel kell kieg´esz´ıtened. A k¨ ul¨ on´all´ o dokument´ aci´ oban a megold´ as elvi menet´enek, algoritmus´ anak ismertet´es´et v´ arjuk. A forr´ ask´ od kommentez´es´enek l´enyege, hogy seg´ıts´eg´evel – a dokument´ aci´ o ismeret´eben – k¨ onnyen meg´erthet˝ o legyen az egyes k´odsorok, k´ odr´eszletek feladata, szerepe a megold´ as menet´eben. Az I/S ´es S-jel˝ u programoz´ asi feladatok megold´ as´ at ellen˝ orizd a http:// ideone.com vagy a http://onlinegdb.com tesztk¨ ornyezetben a feladathoz el´erhet˝o bemenetekkel. Ezeknek a feladatoknak az ´ert´ekel´ese r´eszben automatikusan t¨ort´enik, ez´ert fontos, hogy a program az el˝o´ır´ as szerinti form´aban adjon kimenetet.

A geometria feladatok megold´ as´ anak fontos r´eszei az ´abr´ ak, amelyeken k¨ovetni ´es ellen˝ orizni lehet a megold´ as l´ep´eseit. Mindig rajzolj ´abr´ at, az ´abra n´elku ¨li, vagy nem megfelel˝o ´abr´at tartalmaz´ o megold´asokat nem tekintju k teljesnek. A gondolat¨ meneted azon l´ep´eseire, amelyekhez nincs mell´ekelve a szu ¨ks´eges ´abra, nem kapsz pontot. Bonyolultabb ´abr´ ak eset´en az egyes geometriai objektumokat sz¨ovegesen is uk¨ ork´epe az e egyenesre”). Elektronikus bek¨ uld´es defini´ald (pl. legyen P  a P pont t¨ ” eset´en u o felbont´ asra. A felbont´as akkor megfelel˝o, ha a sz´ am´ıt´og´ep ¨gyelj a megfelel˝ k´eperny˝ oj´en elf´er, ´es a fontos r´eszletek is j´ok kivehet˝ oek. A j´ o ´abra m´erete t¨obbnyire 500–1000 pixel k¨ oz¨ ott lehet.

A matematika ´es fizika dolgozatokat honlapunkon megszerkesztheted vagy k´esz f´ajl form´aj´aban felt¨ oltheted. Az informatika feladatok megold´as´ at csak felt¨ olteni tudod.

A matematika p´eld´ ak megold´ asak´ent sz´ am´ıt´ og´epes programokkal – bele´ertve az olyan online szolg´altat´ asokat is, mint p´eld´ aul a Wolfram Alpha – kisz´am´ıtott eredm´enyeket nem fogadunk el. Ha harmincn´ al t¨ obb esetet vizsg´alsz, pedig l´enyegesen le lehetett volna sz˝ uk´ıteni az esetek sz´am´at, azt is u ´gy tekintj¨ uk, mintha programot ´ırt´al volna.

Azokat a dolgozatokat, amelyek t¨ obb feladat megold´ as´ at tartalmazz´ak egy f´ajlban, vagy k¨ ulalakjuk miatt ´ert´ekelhetetlenek, nem versenyszer˝ unek tekintj¨ uk. Nem versenyszer˝ u tov´ abb´ a az olyan megold´ as, ahol rendes k´epletek helyett nehezen ´ertelmezhet˝ o karaktersorozatok vannak, pl. x2+((1+5+2sqrt(5)x2)/4 vagy (1+gy¨ ok5)/2*x.

K¨ oz´ episkolai Matematikai ´ es Fizikai Lapok, 2022/6

341

A megold´asok elk´esz´ıt´ese ´es beku ¨ld´ese Megold´asodat a Munkafu ¨ldd be. ¨zetben ku

342

K¨ oz´ episkolai Matematikai ´ es Fizikai Lapok, 2022/6

A megold´asok online szerkeszt´ese az Elektronikus Munkafu ¨zetben Az Elektronikus Munkaf¨ uzet a honlapunk r´esze. Webes fel¨ ulet, amely lehet˝ os´eget ad a megold´ as k¨ozvetlen be´ır´ as´ ara, szerkeszt´es´ere. A megold´ asaidat m´ odos´ıthatod, a´tszerkesztheted a bek¨ uld´esi hat´ arid˝ oig. K´epletek szerkeszt´es´ehez a TEX rendszert haszn´ aljuk. Javasoljuk, hogy honlapunkon j´ard v´egig a TEX tanfolyamot (https://www.komal.hu/mf?a=tk). K´esz f´ajlok felt¨ olt´ese Megold´ asaidat az otthoni vagy iskolai sz´ am´ıt´ og´epeken is elk´esz´ıtheted, ´es a k´esz f´ajlt honlapunkon felt¨ oltheted. Matematika ´es fizika feladatok megold´ asa eset´en a t¨ obbf´ele oper´ aci´ os rendszerben olvashat´o PDF form´atumot haszn´ ald. A dokumentum elej´en legyen ott az u ´n. fejl´ec: a feladat sz´ama pirossal, n´ev, oszt´ aly, v´ aros, iskola. K´ez´ır´assal k´eszu ¨li, feh´er pa¨lt megold´asodat vonalaz´as ´es n´egyzeth´al´o n´elku p´ırra ´ırd, majd megfelel˝o min˝ os´egben, egy darab pdf f´ajlk´ent t¨oltsd fel a Munkafu ¨zetbe. ¨ Ugyelj arra, hogy a k´ep j´ ol olvashat´o legyen, ´es a felbont´ as ne legyen se t´ ul nagy, se t´ ul alacsony. Ha f´enyk´epezel, ´erdemes t¨ obb k´epet k´esz´ıteni sz´ ort (term´eszetes) f´enyn´el, ´es a legjobban siker¨ ult k´epet haszn´ alni. A k´epet ford´ıtsd a´ll´o helyzetbe, a sz´el´et v´ agd k¨ orbe, hogy csak a megold´ as maradjon a k´epen, v´eg¨ ul m´eretezd a´t. F´enyk´epek feldolgoz´ as´ara sokf´ele k´epmanipul´ al´o programot ´es telefonos applik´aci´ ot haszn´alhatsz. Mi a CamScannert aj´ anljuk legink´ abb, mert ezzel k¨onnyen k´esz´ıthetsz egy darab megfelel˝o pdf f´ ajlt. Az informatika megold´asok beku ¨ld´ese Az informatika feladatok megold´asait kiz´ar´ olag k´esz f´ajlk´ent tudod felto¨lteni a Munkafu as t¨ obb f´ajlb´ol a´ll, u ´gy egy, a f´ajlok ¨zetbe. Amennyiben a megold´ mindegyik´et ´es a dokument´ aci´ ot is tartalmaz´o, a feladat sorsz´am´aval egyez˝o nev˝ u ¨ mapp´ at kell ZIP t¨ om¨or´ıt´essel becsomagolva egyetlen f´ ajlk´ent bek¨ uldened. Ugyelj arra, hogy a t¨ om¨ or´ıtett a´llom´anyokba futtathat´ o f´ajlok (pl. a fejleszt´eskor l´etrej¨ov˝ o .exe ´ allom´any) ne ker¨ uljenek. A programoz´ asi feladatokn´ al a forr´ ask´ od els˝ o soraiban megjegyz´esk´ent szerepeljen • • • •

a feladat sz´ama; a versenyz˝ o teljes neve (jelz˝osz´ ammal) ´es oszt´ alya; az iskola neve v´arosn´evvel egy¨ utt; az alkalmazott ford´ıt´ oprogram neve ´es verzi´osz´ ama.

K´erj¨ uk, hogy a programoz´asi feladatokn´ al a program be- ´es kimenete mindig uks´eg, mert a bek¨ ula feladatban megadott m´ odon val´ osuljon meg. Erre az´ert van sz¨ d¨ ott programokat sokf´ele tesztadatra lefuttatjuk, ´es ezt igyeksz¨ unk automatiz´ alni. Az informatika feladatokkal kapcsolatos b´arminem˝ u k´erd´eseket, esetleges reklam´aci´ okat az [email protected] c´ımre v´ arjuk. A beku ¨ld´esi hat´arid˝o A bek¨ uld´esi hat´ arid˝ o matematik´ ab´ol a lap megjelen´es´et k¨ovet˝o h´onap 10., fizik´ ab´ ol ´es informatik´ ab´ ol a 15. napja; szombat, illetve munkasz¨ uneti nap eset´en K¨ oz´ episkolai Matematikai ´ es Fizikai Lapok, 2022/6

343

a k¨ovetkez˝ o munkanap. Vedd figyelembe az internet esetleges hib´ait ´es a bek¨ uld´esi hat´ arid˝o id˝o el˝otti o´r´ akban a szerver g´ep¨ unk esetleges t´ ulterhelts´eg´et; ilyen okokra hivatkozva sem fogadunk el k´esedelmes dolgozatokat. ´ ekel´es Ert´ A pontversenyek ´all´ as´ at ´es versenyz˝oink r´eszletes eredm´enyeit a honlapunkon folyamatosan k¨ oz¨ olj¨ uk. Versenyz˝oinket e-mailben is ´ertes´ıtj¨ uk a pontsz´amok v´ altoz´asair´ ol. Jav´ıt´ oink a pontsz´ amon k´ıv¨ ul sz¨ oveges ´ert´ekel´est is k¨ uldhetnek, p´eld´ aul felh´ıvhatj´ ak a figyelmedet a dolgozatod hi´anyoss´ agaira. Ez azonban nem k¨ otelez˝ o, ugyanis a jav´ıt´oknak nem ritk´an sz´azas nagys´ agrend˝ u dolgozatot kell kijav´ıtani, amit r´ aad´asul az egyetemi tanulm´anyaik mellett tesznek. Reklam´aci´ ok A dolgozatok ´ert´ekel´ese ut´ an az Elektronikus Munkaf¨ uzetben r¨ ovid k´erd´est vagy u uldhetsz a jav´ıt´ onak, o˝ pedig ugyanott v´ alaszolhat. A k¨ ul¨ onb¨ oz˝ o ¨zenetet k¨ feladatokat k¨ ul¨ onb¨ oz˝ o jav´ıt´ ok jav´ıtj´ ak, ez´ert mindig csak az adott feladatr´ ol k´erdezz. ¨ Ugyelj az udvarias hangv´etelre. Olyan m´odon k´erdezz, amit szemt˝ol-szemben, ak´ar a tan´araiddal vagy a sz¨ uleiddel szemben is helyesnek tartan´ al. Eld¨ontetlen vita, reklam´aci´ o eset´en a szerkeszt˝ os´eghez fordulhatsz. Reklam´aci´okat a feladat ´ert´ekel´ese ut´ an k´et h´etig fogadunk el a [email protected] c´ımen. Szab´alytalan versenyz´es FONTOS! Akik egy´enileg versenyeznek egy adott pontversenyben, azoknak uz¨ ott feladatokat a be¨on´all´oan kell elk´esz´ıteniu ¨k a p´eld´ak megold´asait. Tilos a kit˝ k¨ uld´esi hat´ arid˝ o el˝ ott m´ asokkal megvitatni, m´ asokt´ ol seg´ıts´eget k´erni vagy elfogadoz¨ osen k´esz´ıtett vagy m´ asolt dolgozatokat – beni a feladatok megold´ as´ ahoz. A k¨ le´ertve az eredeti szerz˝o´et is – nem versenyszer˝ unek ´ert´ekelj¨ uk. Egy csapat tagjai egym´ assal megbesz´elhetik, megvitathatj´ ak az adott verseny feladatait, majd minden feladatra egy k¨ oz¨ os megold´ ast adnak be. A csoportosan m´asolt dolgozatokat visszak¨ uldj¨ uk az oszt´ alyt tan´ıt´ o tan´ arnak. S´ ulyosabb, az eg´esz pontversenyt vesz´elyeztet˝ o esetekben (pl. a feladatok megt´ argyal´asa internetes f´ orumokon) az ´erintett versenyz˝ oket ´es csapatokat kiz´arjuk a versenyb˝ ol. A v´egeredm´eny k¨ ozz´et´etele A versenyek v´egeredm´enye az ¨ osszes dolgozat kijav´ıt´ asa ut´ an, v´ arhat´ oan augusztus elej´en a honlapunkon, majd a 2023. szeptemberi sz´ amunkban jelenik meg. A legeredm´enyesebb versenyz˝ ok arck´ep´et 2023. decemberi sz´ amunkban k¨ oz¨ olj¨ uk. A legjobbak a MATFUND K¨ oz´episkolai Matematikai ´es Fizikai Alap´ıtv´ any p´ alyaagi Ank´et rendezv´eny´en. d´ıjait ´es t´ argyjutalmakat kapnak a 2023. ´evi K¨ oMaL Ifj´ us´ Az okleveleket post´an k¨ uldj¨ uk el. N´eh´any megjegyz´es A versenyben r´esztvev˝ o hozz´ aj´arul a dolgozat´ anak n´ev n´elk¨ uli, valamint a szerkesztett v´ altozat n´evvel t¨ ort´en˝ o k¨ ozl´es´ehez. ¨ ommel fogadunk feladatjavaslatokat, cikkeket, szakk¨ Or¨ ori munk´ ar´ ol sz´ol´o besz´ amol´okat, k¨ ozl´esre alkalmas iskolai p´ alyamunk´akat. Javaslataikat, k¨ ozlem´enyeiket post´ an vagy e-mailben juttathatj´ak el szerkeszt˝os´eg¨ unkbe. Sz´ep, ´erdekes ´es nem 344

K¨ oz´ episkolai Matematikai ´ es Fizikai Lapok, 2022/6

k¨ ozismert feladatokat b´ arki javasolhat kit˝ uz´esre. A javasolt feladatokat (megold´ asokkal egy¨ utt) a szerkeszt˝ os´eg c´ım´ere k¨ uldj´ek el. A di´ akok elfogadott feladatjavaslatai k¨ oz¨ ul a legszebbeket k¨ ul¨ ond´ıjban r´eszes´ıtj¨ uk. Versenyz˝oink akkor kapnak pontot az ´ altaluk javasolt feladatra, ha annak megold´ as´ at – a t¨obbi feladat megold´ as´ ahoz hasonl´ oan – felt¨ oltik a Munkaf¨ uzetbe. Szeretn´enk, ha a kit˝ uz¨ ott k´erd´esek nem z´ aruln´ anak le v´eglegesen a bek¨ uld´esi hat´ arid˝ovel, a k¨ oz¨ olt megold´ assal. Erre teremt lehet˝os´eget az internetes K¨oMaL f´orum. B´armely, a lapunkban megjelent feladathoz, cikkhez kapcsol´ od´o megjegyz´est, altal´ ´ anos´ıt´ ast sz´ıvesen l´atunk ´es alkalomadt´ an k¨ oz¨ olj¨ uk. an V´egezet¨ ul mindenkinek eredm´enyes tan´evet ´es sikeres versenyz´est k´ıv´ a Szerkeszt˝ os´eg

Matematika feladatok megold´asa

R´at´er¨ unk annak a bizony´ıt´as´ ara, hogy a sz´oban forg´o n´egyzetsz´amokb´ ol k´epamok k¨ oz¨ ul zett ¨osszes r´esz¨ osszeg k¨ ul¨ onb¨ oz˝ o, azaz az a21 , . . . , a2k (1  k  n eg´esz) sz´ ak´arhogyan v´ alasztunk ki n´eh´ anyat, azok ¨ osszege csakis akkor egyenl˝ o, ha pontosan ugyanazokat a sz´ amokat v´ alasztjuk ki. Most is k-ra vonatkoz´ o teljes indukci´ ot alkalmazunk. Az a´ll´ıt´ as trivi´ alis k = 1-re, hiszen az u osszegen k´ıv¨ ul maga a21 ¨res ¨ az egyetlen r´esz¨ osszeg. Az indukci´ os l´ep´esben feltessz¨ uk, hogy az a21 , . . . , a2k sz´ amokb´ ol k´epzett ¨ osszes r´esz¨ osszeg k¨ ul¨ onb¨ oz˝ o; majd tekintj¨ uk az a21 , . . . , a2k+1 sz´ao r´esz¨ osszegek mokb´ ol k´epzett r´esz¨ osszegeket. Ezek k¨ oz¨ ul az a2k+1 -t nem tartalmaz´ nem lehetnek egyenl˝ok, hiszen ezek az el˝ oz˝ o l´ep´esben is r´esz¨ osszegek voltak, ´ıgy hivatkozhatunk az indukci´os felt´etelre. Ha k´et r´essz¨ osszeg k¨ oz¨ ul mindkett˝ o tartalmazok, hiszen mindk´et oldalr´ ol elhagyva za a2k+1 -t, akkor szint´en nem lehetnek egyenl˝ o esetet kapjuk. T´etelezz¨ uk fel v´eg¨ ul, hogy egy a2k+1 -t tartalmaz´ o ´es a2k+1 -t az el˝oz˝ ol S2 egy ezt nem tartalmaz´ o r´esz¨ osszeg egyenl˝ o, p´eld´ aul S1 = S2 + a2k+1 , amelyb˝ nemnegativit´ asa miatt S1  a2k+1

(1)

B. 5220. Legyen n pozit´ıv eg´esz sz´ am. Mutassuk meg, hogy megadhat´ o 1-t˝ ol am u ´gy, hogy k¨ oz¨ ul¨ uk ak´ arh´ any k¨ ul¨ onb¨ oz˝ ot ¨ osszeadva (bele´ert2n+2 -ig n n´egyzetsz´ ve az egytag´ u o osszes sz´ am ¨ osszeg´et is) csupa k¨ ul¨ onb¨ oz˝ o sz´ amot ¨sszegeket ´es az ¨ kapjunk.∗ Javasolta: Freud R´ obert (Budapest)  √ 2ai−1 , Megold´as. Legyen a1 = 1 ´es tetsz˝oleges 2  i  n-re legyen ai = ahol a az a val´os sz´am fels˝o eg´eszr´esz´et jel¨ oli, azaz azt a legkisebb eg´esz sz´ amot, amely nem kisebb, mint a. Ekkor a rekurz´ ıvan defini´ a lt a , . . . , a sz´ a mok pozit´ ıv 1 n √  √ √ 2ai−1  2ai−1 > ai−1 . Mivel eg´eszek ´es k¨ ul¨ onb¨ oz˝ oek, mert 2 > 1, ´ıgy ai = ul¨onb¨oz˝ o k¨ ul¨ onb¨ oz˝ o sz´ amok n´egyzete k¨ ul¨ onb¨ oz˝ o, ez´ert a21 , . . . , a2n pontosan n darab k¨ n´egyzetsz´am.

ad´odik. Indirekt feltev´es¨ unk szerint S1 egy, az a21 , . . . , a2k sz´ amokb´ ol k´epezhet˝ o k  r´esz¨osszegk´ent is el˝o´all, aminek minden tagja pozit´ıv, ´ıgy S1  a2i , amelyre i=1

a lemm´at alkalmazva azt kapjuk, hogy S1 

(6 pont)

amok n´egyzeteire igaz, hogy Lemma. Az el˝ oz˝ oekben defini´ alt ai sz´ k 

a2i < a2k+1 .

i=1

A lemma bizony´ıt´asa. Az ´all´ıt´ ast k-ra vonatkoz´ o teljes indukci´ oval bizony´ıtjuk. Az ´ all´ıt´ as igaz k = 1-re, mert ekkor az ¨ osszeg a21 = 1, amely kisebb a22 = 4-n´el. Az indukci´ os l´ep´esben az mondhat´ o el, hogy ha k-ra igaz az ´all´ıt´ as, akkor k + 1-re is, hiszen k+1  i=1

a2i =

k 

a2i + a2k+1 < a2k+1 + a2k+1 = 2a2k+1 =

i=1

Ezzel a lemma bizony´ıt´as´ at befejezt¨ uk. ∗

 2 √ 2 √ 2ak+1  2ak+1 = a2k+2 .

ol kaphat´o r´esz¨ osszegek k¨ oz¨ ul ami ellentmond (1)-nek, ez´ert az a21 , . . . , a2k+1 sz´amokb´ semelyik kett˝o sem lehet egyenl˝ o. Ezzel a teljes indukci´ o v´eg´ere ´ert¨ unk, ´es k = n-re ´epp azt kaptuk, hogy a kiv´ alasztott n n´egyzetsz´amb´ ol k´epzett minden r´esz¨ osszeg k¨ ul¨onb¨oz˝ o. amok 2n+2 -n´el kisebbek. KoM´ar csak azt kell bel´atnunk, hogy az a21 , . . . , a2n sz´ 2 n+2 . r´abban l´attuk, hogy 0 < a1 < · · · < an , ´ıgy el´eg azt megmutatnunk, hogy a√ n 2 · 1 = 2 miatt a3 = 2 2 = 3 (hiszen  √ 2 at a = 1, a = 2, a3 = 3. A tov´ abbi ´ert´ekeket fel¨ ulr˝ ol be2 2 = 8 < 32 ), teh´ √ 1  √ 2 cs¨ ulj¨ uk. Mivel ai+1 = 2ai < 2ai + 1, ez´ert a b1 = 1, b2 = 2, b3 = 3, i  4-re √ bi = 2bi−1 + 1 sorozat fel¨ ulr˝ ol becs¨ uli az ai sorozatot. Ez teljes indukci´oval bizony´ıthat´ o, hiszen a1  b1 , a2  b2 , a3  b3 , ´es ha ai  bi (i  4), akkor ai+1
CD, a trap´ez k¨ oz´epvonala az AC ´atl´ ot az E, a BD ´ atl´ ot az F pontban metszi. A CD szakasz hossza az AB ´es EF szakaszok hossz´anak a) sz´amtani, b) m´ertani k¨ ozepe. AB ar´ any Hat´ arozzuk meg, hogy a k´et eset k¨ oz¨ ul melyikben lesz nagyobb az CD ´ert´eke.

C. 1732. Legyen U a 337-n´el nagyobb ´es 733-n´ al nem nagyobb pr´ımsz´ amok halmaza. H´any olyan 4-elem˝ u r´eszhalmaza van U -nak, amelynek a 467 vagy a 499 eleme?



Beku ober 10. ¨ld´esi hat´arid˝o: 2022. okt´ Elektronikus munkafu ¨zet: https://www.komal.hu/munkafuzet

Beku ober 10. ¨ld´esi hat´arid˝o: 2022. okt´ Elektronikus munkafu zet: https://www.komal.hu/munkafuzet ¨



K¨ oz´ episkolai Matematikai ´ es Fizikai Lapok, 2022/6

351

352

K¨ oz´ episkolai Matematikai ´ es Fizikai Lapok, 2022/6

A B pontversenyben kit˝ u z¨ ott feladatok (5254–5261.)

B. 5254. Bizony´ıtsuk be, hogy b´armely k´et, 3-mal nem oszthat´ o p´aratlan sz´ am n´egyzet´enek k¨ ul¨ onbs´ege oszthat´ o 24-gyel. (3 pont)

B. 5261. Kezd˝ o ´es M´ asodik a 100 cs´ ucs´ u teljes gr´ af ´elein j´atszanak. Felv´ altva l´epnek, Kezd˝ o minden l´ep´esn´el egy m´eg ki nem sz´ınezett ´elt pirosra, M´ asodik pedig minden l´ep´esn´el egy (m´eg ki nem sz´ınezett) ´elt k´ekre sz´ınez. A j´ at´ek akkor ´er v´eget, ha vagy van 4 olyan cs´ ucs, melyek k¨ oz¨ ott mind a 6 ´el piros, ekkor Kezd˝ o nyer; vagy van 4 olyan cs´ ucs, melyek k¨ oz¨ ott mind a 6 ´el k´ek, ekkor M´ asodik nyer; vagy pedig egyik sem teljes¨ ul, ´es nincs t¨ obb besz´ınezhet˝ o ´el, ekkor az eredm´eny d¨ ontetlen. Kinek van nyer˝ o strat´egi´ aja? (6 pont)

(Mennyis´egtani ´es Term´eszettudom´ anyi Didaktikai Lapok, 1943)

 Beku ober 10. ¨ld´esi hat´arid˝o: 2022. okt´ Elektronikus munkafu ¨zet: https://www.komal.hu/munkafuzet

B. 5255. T¨ ukr¨ ozz¨ uk k¨ oz´eppontosan az ABC h´ aromsz¨og A cs´ ucs´ at B-re, B cs´ ucs´at C-re, ´es C cs´ ucs´ at A-ra, ´ıgy kapjuk rendre a C1 , A1 ´es B1 pontokat. Mutassuk us´ag´ u oldalakkal h´ aromsz¨og szerkeszthet˝o. meg, hogy AA1 , BB1 ´es CC1 hossz´



(3 pont) B. 5256. Andr´as egy ´ev mind az 52 het´eben egy-egy ugyan´ ugy kit¨olt¨ott szelv´ennyel j´ atszik az ¨ ot¨ oslott´ on. Bea ellenben az ´ev utols´ o sorsol´ asa el˝ ott vesz 52 szelv´enyt, ´es azokat (p´aronk´ent k¨ ul¨ onb¨ oz˝ of´elek´eppen kit¨oltve) egyszerre j´ atssza meg. Igaz-e, hogy ugyanannyi es´elye van Andr´ asnak ´es Be´ anak arra, hogy legyen telital´ alatos szelv´eny¨ uk?

Figyelem! Az idei K¨ ursch´ ak J´ ozsef Matematikai Tanul´ overseny 2022. okt´ober 7-´en, p´enteken 14 ´ orakor ker¨ ul megrendez´esre. A verseny helysz´ıneir˝ ol ´es lebonyol´ıt´ as´ ar´ ol sz´ol´o inform´ aci´ okat k´es˝ obb tessz¨ uk k¨ ozz´e a https://www.bolyai.hu/versenyek-kurschak-jozsef-matematikaitanuloverseny/ oldalon.

(4 pont)



B. 5257. Az ABC hegyessz¨ og˝ u h´ aromsz¨ ogben a magass´ agok AA1 , BB1 , illetve opontja F . A k k¨ or ´ atmegy az F ´es a C1 pontokon, valamint CC1 , az AB oldal felez˝ uli meghosszabb´ıt´as´at a P , illetve a Q pontban az A1 C1 ´es B1 C1 szakaszok C1 -en t´ metszi. Igazoljuk, hogy A1 P = B1 Q.

Az A pontversenyben kit˝ uz¨ott nehezebb feladatok (830–832.)

(4 pont) B. 5258. Igaz-e, hogy minden pozit´ıv eg´esznek van olyan pozit´ıv t¨obbsz¨or¨ose, amelynek t´ızes sz´ amrendszerbeli alakj´ aban a sz´ amjegyek ¨osszege legfeljebb 2022? (5 pont)

Javasolta: S´ andor Csaba (Budapest)

B. 5259. Oldjuk meg a k¨ ovetkez˝ o egyenletrendszert a val´os sz´amok halmaz´an: x2 − 3y + 4 = z, y 2 − 3z + 4 = w,

A. 830. Ha H ⊂ Z ´es n ∈ Z, legyen hn a H azon v´eges r´eszhalmazainak a sz´ ama, melyekben a sz´amok ¨ osszege n. Van-e olyan H ⊂ Z, melyre 0 ∈ / H, ´es minden n ∈ Zaros sz´am? (Az u osszege 0.) re hn egy (v´eges) p´ ¨res halmaz elemeinek ¨ Javasolta: Beke Csongor (Cambridge)

2

z − 3w + 4 = x, w2 − 3x + 4 = y. (4 pont)

Bencze Mih´ aly (Brass´ o) javaslata alapj´an

B. 5260. A k k¨ or AB h´ urj´anak G ´es H pontjaira AG = GH = HB = 1. A k¨or egyik AB ´ıv´enek felez˝ opontja legyen F . Az F H ´es F G szel˝ok a k¨ort m´ asodszor a C, illetve D pontban metszik. Mutassuk meg, hogy CD = BC 2 . (6 pont)

Javasolta: Kocsis Szilveszter (Budapest)

K¨ oz´ episkolai Matematikai ´ es Fizikai Lapok, 2022/6

353

A. 831. Az ABC h´ aromsz¨ og BC oldal´ anak F a felez˝ opontja. Az A-n ´athalad´ o, BC-t F -ben ´erint˝o k¨ or messe az AB ´es AC oldalakat rendre az M ´es N pontokban. A CM ´es BN szakaszok metsz´espontja legyen X. A BM X ´es CN X h´ aromsz¨ ogek k¨or´e´ırt k¨oreinek m´asodik metsz´espontja legyen P . Igazoljuk, hogy A, F ´es P egy egyenesre illeszkednek. A. 832. Tegy¨ uk fel, hogy minden embernek egym´ast´ ol f¨ uggetlen¨ ul 0, 1, . . . vagy osz´ın˝ us´ege, hogy ´eppen i gyermeke sz¨ uletik, n gyermeke sz¨ ulethet, ´es annak a val´ ´n. Galton–Watson folyamat.) pi , ahol p0 + p1 + . . . + pn = 1 ´es pn = 0. (Ez az u 354

K¨ oz´ episkolai Matematikai ´ es Fizikai Lapok, 2022/6

Mely n pozit´ıv eg´esz ´es p0 , p1 , . . . , pn val´ osz´ın˝ us´egek eset´en lesz maxim´ alis annak a val´ osz´ın˝ us´ege, hogy egy adott ember ut´ odai ´eppen a tizedik gener´ aci´ oban halnak ki? Beku ober 10. ¨ld´esi hat´arid˝o: 2022. okt´ Elektronikus munkafu ¨zet: https://www.komal.hu/munkafuzet



t´ an m´ ar csak a m´ asik fajta dob´ ast kell tov´ abb ism´etelnetek, am´ıg az is 12 nem lesz, de ne felejts´etek, csak 100 k´ıs´erletet v´egezhettek. V´eg¨ ul csak k´et adatra lesz sz¨ uks´eg¨ unk, a dob´ ododeka´ederrel ´es a dob´ okockap´ arral v´egzett k´ıs´erletek sz´ am´ ara. Rem´elem, minden vil´ agos, Andris, gyere, seg´ıts az eszk¨ oz¨ oket kiosztani. K¨ ozben lehet tippelni arra, hogy mekkora lesz a dob´ assz´ amok ´ atlag´ anak ar´ anya. – Nyilv´ an azonos lesz nagyj´ ab´ ol, hisz a dodeka´eder pont olyan, mint a k´et dob´ okocka – kiab´ alt be Miki. Sokan helyesl˝oen b´ ologattak. – No, majd kider¨ ul – v´ alaszolta Piroska n´eni. – Kezdj´etek a k´ıs´erleteket! Seg´ıts¨ unk a 9.a oszt´ alynak! 1. Hozzuk l´etre a dodeka´ eder munkaf¨ uzetet a t´ abl´ azatkezel˝ o alap´ertelmezett f´ajlform´ atum´ aban. 2. K´esz´ıts¨ uk fel a munkalapot a 30 tanul´o legfeljebb 100 k´ıs´erlet´enek dokument´al´as´ara:

Informatik´ab´ ol kit˝ u z¨ ott feladatok

I. 568. Az ¨ oreg kir´ aly ¨ osszeh´ıvta tan´ acskoz´ asra f´erfi lesz´ armazottjait. ´Igy elj¨ ottek a fiai, unok´ ai, d´edunok´ ai stb. A kir´aly mem´ ori´aja m´ar nem a legjobb, ´es a tan´ acskoz´ as elej´en szeretn´e tudni mindenkir˝ ol, hogy h´ any gener´aci´ os t´ avols´ agban van t˝ ole. K´esz´ıts¨ unk programot, amely az N tag´ u csal´ad minden jelenl´ev˝ oj´er˝ ol megadja, hogy h´ any gener´aci´ ora van a kir´alyt´ ol az ´ırnok feljegyz´esei alapj´an. A standard bemenet (az ´ırnok adatai) els˝o sor´ aban a jelenl´ev˝ ok N (2  N   50) sz´ama van. Az ezt k¨ ovet˝o N − 1 sor mindegyike egy sz´amp´ art tartalmaz: az A apa ´es F fia (1  A, F  N ) sorsz´ am´at sz´ok¨ ozzel elv´ alasztva. obe a kir´ aly sorsz´ am´at, a m´aA standard kimenetre k´et sort ´ırjunk ki: az els˝ sodikba pedig emelked˝ o sorsz´ am szerint mindenkinek a gener´ aci´ os t´ avols´ ag´at a kir´alyt´ ol. P´elda bemenet (a / jel sort¨ or´est helyettes´ıt) 9 3 1 / 3 2 / 7 3 / 5 6 / 4 5 / 7 4 / 8 9 /7 8

Kimenet 7 2 2 1 1 2 3 0 1 2

3. A C1:CX1 tartom´ anyt t¨ olts¨ uk fel az 1, 2, . . . , 99, 100 ´ert´ekekkel. Az A2:A61 tartom´anyt pedig egyetlen, m´asolhat´ o f¨ uggv´eny seg´ıts´eg´evel az 1, 1, 2, 2, . . . , 29, 29, 30, 30 ´ert´ekekkel. A B2:B61 tartom´ anyt felv´altva a k´et dob´ okocka”´es a do” ” b´ ododeka´eder” sz¨ ovegekkel t¨ olts¨ uk fel. V´eg¨ ul k´esz´ıts¨ uk el az A1:CX1 tartom´ any szeg´elyez´es´et a minta szerint. 4. Ezek ut´an modellezz¨ uk a k´ıs´erletet.

Bek¨ uldend˝o egy t¨om¨ or´ıtett i568.zip ´ allom´anyban a program forr´ ask´odja, valamint a program r¨ ovid dokument´ aci´ oja, amely tartalmazza a megold´ as r¨ovid le´ır´ as´ at, ´es megadja, hogy a forr´ as´allom´any melyik fejleszt˝oi k¨ornyezetben ford´ıthat´ o.

am-gener´ atort haszn´ al´o olyan f¨ uggv´eny, a. A C2 cell´ aba ker¨ ulj¨ on a v´eletlensz´ amely a k´et f¨ uggetlen kockadob´ as ¨ osszeg´enek ´ert´ek´et jelen´ıti meg a cell´aban. b. A C3 cell´ aba ker¨ ulj¨ on hasonl´o f¨ uggv´eny, amely a dob´ ododeka´ederrel v´egzett dob´ as ´ert´ek´et jelen´ıti meg a cell´aban.

I. 569. Az egyik nap Piroska n´eni, a matektan´ ar egy dobozzal a kez´eben l´epett be a 9.a oszt´alyterm´ebe. A tan´ ora elej´en megszokott esem´enyek ut´ an felnyitotta a kor´ abban a tan´ ari asztalra letett dobozt. – N´ezz´etek csak, micsoda csod´ as valamit hoztam! – emelt ki a dobozb´ ol egy dodeka´edert. – Ez az ¨ ot szab´ alyos test egyike, 12 darab szab´ alyos ¨ otsz¨ oglapja van, dodeka´edernek h´ıvj´ ak. Ha a lapokra fel´ırjuk a sz´ amokat 1-t˝ ol 12-ig, akkor a dob´ okocka k´etszeres´et k´esz´ıthetj¨ uk el. Mivel harmincan vagytok, hoztam is 60 dob´ okock´ at ´es 30 dob´ ododeka´edert. Andris, ha elmondtam a feladatot, seg´ıts kiosztani, l´egy sz´ıves. Mindh´ arom testtel maximum 100 alkalommal kell dobnotok. A f¨ uzetbe jegyezz´etek akkal dobott sz´ amok ¨ osszeg´et, mell´e a dodeka´ederrel dofel minden dob´ asn´ al a kock´ bott ´ert´eket! A k´et kock´ aval ´es a dodeka´ederrel dob´ ast csak addig kell ism´etelgetni, am´ıg a k´et kock´ an kapott ¨ osszeg, vagy a dodeka´ederen kij¨ ott ´ert´ek 12 nem lesz. AzK¨ oz´ episkolai Matematikai ´ es Fizikai Lapok, 2022/6

355

c. A C2:C3 tartom´ any k´epleteit m´ asoljuk le a C4:C61 tartom´ anyra. d. A D2 cell´ aba ker¨ ulj¨ on a C2 cell´ a´ehoz hasonl´o f¨ uggv´eny, de, ha a C2 cella ´ert´eke 12, akkor a D2 cella legyen u arjunk el hasonl´ oan a D3, C3 ¨res. J´ cellap´ arral. e. A D2:D3 tartom´ any k´epleteit m´ asoljuk le a D4:D61 tartom´ anyra. aba ker¨ ulj¨ on a D2 cell´ a´ehoz hasonl´o f¨ uggv´eny, de, ha a D2 cella f. Az E2 cell´ ´ert´eke u arjunk el hasonl´ oan ¨res vagy 12, akkor az E2 cella legyen u ¨res. J´ az E3, D3 cellap´ arral. g. A D2:D3 tartom´ any k´epleteit m´ asoljuk le a D2:CX61 tartom´ anyra. 356

K¨ oz´ episkolai Matematikai ´ es Fizikai Lapok, 2022/6

5. A CY2:CY61 tartom´ any cell´aiba ker¨ ulj¨ on olyan f¨ uggv´eny, amely megadja az adott sor dob´asainak sz´ am´at a 12-es ´ert´ek el´er´es´eig. 6. A C62 ´es a C63 cell´ akba ker¨ ulj¨ on a dob´okock´ aval, illetve dob´ododeka´ederrel tanul´ onk´ent v´egzett dob´ asok ´atlaga eg´eszre kerek´ıtve. 7. Vizsg´aljuk meg a kapott ´ert´ekp´ art, ´es ´ırjunk a tapasztaltakr´ ol magyar´azatot a dokument´ aci´ oba. A megold´ asban saj´ at f¨ uggv´eny vagy makr´o nem haszn´ alhat´ o! Bek¨ uldend˝o egy t¨om¨ or´ıtett i569.zip ´ allom´anyban a t´abl´ azatkezel˝ o munkaf¨ uzet (dob´ ododeka´ eder.xlsx, dob´ ododeka´ eder.ods, . . . ), illetve egy r¨ovid dokument´ aci´ o (dob´ ododeka´ eder.txt, dob´ ododeka´ eder.pdf, . . . ), amelyben szerepel a megold´ askor alkalmazott t´ abl´ azatkezel˝ o neve, verzi´osz´ ama ´es a v´alasz a 7. feladatra. ´ Tavaly o˝sszel Paolo Panzani megh´ I. 570 (E). azasodott, elvette magyar bar´ atn˝ oj´et, Budapesten folytatj´ak ´elet¨ uket. Paolo pizzas¨ ut˝ o mester, ´ıgy 2022 ´aprilis´aban beindult a Pizza Panzani – online pizzam˝ uhely. A finom pizz´ak h´ıre gyorsan terjedt a neten, ny´ arra m´ ar k¨ ozel ezer v´ as´ arl´o regisztr´ alt oldalukon. Rendel´est leadni 10 ´or´ at´ ol este 22 ´or´ aig lehet. A belv´ arosban a´ll´ o m˝ uhely¨ ukb˝ ol kiz´ar´ olag a bels˝ o ker¨ uletekbe sz´all´ıtanak a m˝ uhely saj´ at fut´ar´ aval. A rendel´esi rendszert u ´gy alkott´ ak meg, hogy amennyiben egy rendel´eskor t¨ obbf´ele (ak´ ar m´eret, ak´ ar fajta szerint) pizz´ at is rendelnek, azokat a rendszer k¨ ul¨ on rendel´esk´ent kezeli, a megrendel˝o azool. nos´ıt´ oja ´es a d´ atum a´rulkodik a t¨obbf´ele t´etel rendel´esr˝

Erika, Paolo feles´ege a j´ uliusi rendel´esek alapj´an szeretn´e a v´allalkoz´ ast tov´ abbfejleszteni akci´ok bevezet´es´evel (akci´ os a´r; egyet fizet, kett˝ ot kap; boldog ´ora; a h´ onap minden negyedik rendel´ese f´el´ aron). Az informatik´aban j´artas b´ atyja, Alex egy adatb´azist k´esz´ıtett, a k¨ ovetkez˝ o t´abl´ akkal: Pizza (pid, n´ev) pid a pizzafajta azonos´ıt´ oja (sz´ aml´ al´o) ez a kulcs; n´ev a pizzafajta megnevez´ese. M´ eret (mid, ´atm) mid a m´eret azonos´ıt´ oja (sz´ aml´ al´o) ez a kulcs; ´atm a pizza a´tm´er˝ oje cm-ben (sz´am). ´ (ptip, m´er, ´ar) Ar ptip a pizzafajta azonos´ıt´ oja (sz´ am) ez az ¨ osszetett kulcs r´esze; am) ez az ¨osszetett kulcs r´esze; m´er a pizza m´eret´enek azonos´ıt´ oja (sz´ e´ar a pizza egys´eg´ ara Ft-ban (sz´ am).

K¨ oz´ episkolai Matematikai ´ es Fizikai Lapok, 2022/6

Rendel´ es rid nap id˝o mrid pit pm pdb

(rid, nap, id˝ o, mrid, pit, pm, pdb) a rendel´es azonos´ıt´ oja (sz´ aml´ al´o) ez a kulcs; a h´ onap napja (sz´am); a rendel´es napon bel¨ uli ideje [10:00–22:00] (id˝ o); a rendel´est lead´ o azonos´ıt´ oja (sz´ am); a pizzafajta azonos´ıt´ oja (sz´ am); a pizza m´eret´enek azonos´ıt´ oja (sz´ am); a rendelt darabsz´am (sz´ am).

Hozzuk l´etre adatb´ azist panzani n´even! Import´ aljuk az adatb´ azisba a sz¨ ovegf´ajlok nev´evel megegyez˝o t´ abl´ akba a pizza.txt, a m´ eret.txt, az ´ ar.txt ´es a rendel´ es.txt sz¨ ovegf´ ajlok tartalm´ at! A f´ ajlok UTF-8 k´ odol´as´ uak, tabul´ atorral tagoltak, els˝ o sorukban a mez˝onevek szerepelnek. abl´ ak kapcsolat´ at a k¨ ovetkez˝ o´ abra mutatja: A t´

A k¨ovetkez˝ o k´erd´esek megv´ alaszol´ asakor ´es feladatok megold´ as´an´al a lek´erde¨ z´eseket ´es a jelent´est a z´ ar´ ojelben olvashat´ o n´even ments¨ uk. Ugyelj¨ unk arra, hogy a megold´ asban pontosan a k´ıv´ ant mez˝ok szerepeljenek. Gondoskodjunk arr´ ol, hogy a lek´erdez´esek ´ertelmes mez˝ onevei a mez˝ o tartalm´ ara utaljanak ´es hogy az adatok ´ert´ekei mindenhol teljes sz´eless´eg¨ ukben legyenek olvashat´ok. 1. Jelen´ıts¨ uk meg a 24 cm-es pizz´ ak nev´et ´es egys´eg´ ar´ at n¨ ovekv˝ o egys´eg´ ar, azon be¨ ul a pizza neve szerint alfabetikus rend szerint. (1´ arak24) 2. Melyik a hat legn´epszer˝ ubb pizza, azaz amelyekb˝ ol a legt¨ obbet rendeltek? A pizza nev´et ´es a rendelt darabsz´amot szeretn´enk l´ atni. (2nyero) 3. Kik azok a visszat´er˝ o v´ as´ arl´ok, akik t¨ obb, mint h´ arom napon is rendeltek a h´onapban? (3tobb_szor) uk, hogy egy 4. Melyik napon kapt´ ak a legt¨ obb darabra a megrendel´est? Feltehetj¨ ilyen nap volt. (4maxdb) 5. Melyik pizz´akb´ ol h´ anyszor rendeltek egyszerre t¨ obb darabot? (5tobbdb) ´ 6. Atlagosan melyik ´ or´ aban ´erkezett a h´ onapban a legt¨ obb rendel´es? (6ora) 7. Mekkora volt a bev´etel az egyes pizzafajt´akb´ ol? A lek´erdez´es sorai bev´etel szerint cs¨ okken˝ o sorrendben legyenek rendezettek. (7bevetel)

357

358

K¨ oz´ episkolai Matematikai ´ es Fizikai Lapok, 2022/6

8. Melyik m´eret˝ u pizz´ ab´ ol mennyi fogyott a h´ onap sor´ an? (8meret) 9. K´esz´ıts¨ uk jelent´est, amely megadja, hogy aznap, amikor a legt¨obb rendel´est kapt´ ak, melyik pizz´ ab´ ol mennyit rendeltek az egyes m´eretekb˝ ol. A jelent´es az al´abbi minta elrendez´es´et k¨ ovesse. (9legjobbnap)

Bek¨ uldend˝o egy t¨om¨ or´ıtett i570.zip ´ allom´anyban a panzani adatb´ azis, vagy az adatb´ azis t´ abl´ ait l´etrehoz´ o, valamint a feladatok megold´as´at ad´o SQL-parancsok egy sz¨ oveges a´llom´anyban, ´es egy r¨ ovid dokument´ aci´ o, amely tartalmazza az alkalmazott adatb´ azis-kezel˝ o program nev´et ´es verzi´osz´ am´at. I/S. 64. Budapest belv´ aros´ aban innovat´ıv fut´arc´eget alap´ıtottak. A belv´ arost a c´eg egy N sorb´ol ´es M oszlopb´ ol ´all´ o n´egyzetr´ acsk´ent k´epzeli el. A n´egyzetr´ acs nedik sor´ aban ´es m-edik oszlop´ aban tal´alhat´ o egys´egn´egyzetet T [n][m]-el jel¨olj¨ uk. A c´eg sz´ekhelye a T [a][b] mez˝ on helyezkedik el. A sorokat ´es oszlopokat 1-t˝ol indexelj¨ uk. Egy nap rendel´est kapnak a T [p][q] mez˝or˝ ol, ahova a fut´ ar szeretne a lehet˝o leggyorsabban eljutni. Sajnos a fut´ ar m´eg csak p´alyakezd˝ o, ez´ert csak a sakkj´at´ek fut´ ora vonatkoz´ o szab´ alyai szerint tud mozogni Budapest belv´ aros´ aban (csak atl´ ´ osan mozoghat, de egy ir´anyban tetsz˝olegesen sokat, ha a belv´arosban marad). Adjuk meg, hogy legkevesebb h´any l´ep´es alatt juthat el a fut´ ar a T [a][b] mez˝or˝ ol a T [p][q] mez˝ oig, ha egy l´ep´esben a´tl´ osan l´ephet tetsz˝ olegesen sokat (a belv´ arosban maradva). Ha a fut´ ar sehogy sem tudja el´erni a T [a][b] mez˝ot, akkor ´ırjunk ki −1-et. A bemenet els˝ o sor´ aban az N ´es M sz´ amok szerepelnek sz´ok¨ozzel elv´ alasztva. alasztva. A m´asodik sorban az a, b, p, q sz´ amok szerepelnek sz´ok¨ozzel elv´ A kimenet egyetlen sor´ aban egy sz´ am szerepeljen, a minim´alis l´ep´essz´ am (vagy −1, ha nem lehets´eges el´erni a c´elmez˝ ot).

Korl´ atok: 2  N, M  109 , 1  a, p  N , 1  b, q  M . Id˝ okorl´at: 0,4 mp. ´ Ert´ekel´es: a pontok 50%-a kaphat´ o, ha a program az N, M  1000 tesztesetekre helyes megold´ ast ad. S. 163. Manaps´ ag t¨ obb olyan sakkmotor (sakkoz´ o robot) l´etezik, mely k´epes a legjobb sakknagymestereket is legy˝ ozni. A probl´ema neh´ezs´eg´et szeretn´enk illusztr´alni ezzel a feladattal, melyben jelent˝ osen k¨ onny´ıt¨ unk a felt´eteleken. Adott egy hagyom´anyos sakkt´ abla, melyen egy vil´agos gyalog ´es a k´et kir´ aly tal´ alhat´ o. Az esetek nagyj´ab´ ol 2/3-´aban vil´agos mattot tud adni a s¨ ot´et kir´ alynak, a figur´ ak elhelyezked´es´et˝ ol f¨ ugg˝oen azonban a kimenetel d¨ ontetlen is lehet (s¨ ot´et nem nyerhet). K´esz´ıts¨ unk sz´am´ıt´ og´epes programot, amely megadja, hogy mi lesz a j´at´ek kimenetele egy ilyen ´all´ as eset´en. Bemenet: a bemenet els˝ o ´es egyetlen sora egy sakk´ all´ ast tartalmaz a Forsyth– all´ asok jel¨ ol´es´ere. Figyelem! Edwards jel¨ol´es (FEN) szerint. Ez egy standard sakk´ A standard azt is megadja, hogy melyik j´ at´ekosnak kell l´epni. Ez befoly´asolhatja a kimenetet. Felt´etelezhetj¨ uk, hogy az 50 l´ep´es sz´ aml´ al´oja null´ar´ ol indul, ´ıgy a l´ep´essz´ am miatt nem lesz d¨ ontetlen az eredm´eny, ha vil´ agos nyerni tud. Kimenet: a kimenet els˝ o ´es egyetlen sor´ aba a W karaktert kell ´ırni, ha vil´agos nyerhet, ´es a B karaktert, ha s¨ ot´et kik´enyszer´ıtheti a d¨ ontetlent (mindk´et f´el optim´alisan j´atszik). P´elda: Bemenet 7k/4PK2/8/8/8/8/8/8 b - - 0 0

Magyar´ azat: ebben az a´ll´ asban s¨ ot´et l´ep, majd feh´er h´ arom l´ep´esben mattot ad. Id˝ okorl´ at: 1 mp. ´ Ert´ekel´es: A pontok 50%-a kaphat´ o, ha a program helyes eredm´enyt ad olyan bemenetre, amikor a gyalog a 7. sorban van. Seg´ıts´eg: j´at´ek´ all´ asokat kipr´ob´ alhatunk ´es tesztelhet¨ unk p´eld´ aul a https://www.chess.com/analysis ´es a https://syzygy-tables.info/ oldalakon. Bek¨ uldend˝o egy s163.zip t¨ om¨ or´ıtett a´llom´anyban a megfelel˝oen dokument´alt ´es kommentezett forr´ asprogram, amely tartalmazza a megold´ as l´ep´eseit, valamint megadja, hogy a program melyik fejleszt˝ oi k¨ ornyezetben futtathat´ o. A dokument´ aci´ o tartalmazza a megold´ as elm´eleti h´ atter´et, az esetleg felhaszn´alt forr´ asokat. Ne tartalmazzon k´ odr´eszleteket, azok magyar´azata k´ odkommentek form´aj´aban a forr´ asprogramban szerepeljen.



P´eld´ ak:

Bemenetek (a / jel sort¨ or´est helyettes´ıt) 5 4 / 2 2 4 4 12 4 / 1 1 11 3

K¨ oz´ episkolai Matematikai ´ es Fizikai Lapok, 2022/6

Kimenet W

A feladatok megold´asai regisztr´aci´o ut´an a k¨ovetkez˝o c´ımen t¨olthet˝ok fel: https://www.komal.hu/munkafuzet Beku ober 15. ¨ld´esi hat´arid˝o: 2022. okt´

Kimenetek 1 4

 359

360

K¨ oz´ episkolai Matematikai ´ es Fizikai Lapok, 2022/6

Sz´ep szerepl´es az 52. Nemzetk¨ ozi Fizikai Di´akolimpi´an

Az 52. Nemzetk¨ ozi Fizikai Di´akolimpia (IPhO) 2022. j´ ulius 10. ´es 17. k¨oz¨ott ker¨ ult megrendez´esre online form´ aban. A magyar csapat tagjai egy arany-, egy ez¨ ust- ´es h´ arom bronz´ermet szereztek, ´es ezzel az igen el˝ okel˝o 16-17. helyet ´ert´ek ¨ el az orsz´agok k¨ ozti nem hivatalos ´eremt´abl´ azatban. (Osszpontsz´ am szerint haz´ank a mintegy 70 r´esztvev˝ o orsz´ ag k¨ oz¨ ott a 19. helyen v´egzett.) Az ´ermek sorrendje szerint legel˝okel˝obb helyen v´egzett orsz´ agok ´ermeit ´es ¨osszpontsz´ am´at a k¨ovetkez˝ o t´ abl´ azat mutatja: ´ Erem´es pontt´ abl´ azat a 2022. ´evi 52. Nemzetk¨ ozi Fizikai Di´ akolimpi´ an orsz´ag 1. 2. 3. 4. 5. 6. 7. 8. 9. 10. 11. 12. 13. 14. 15. 16. 17. 18. 19. 20.

K´ına D´el-Korea Rom´ania USA Vietn´am Tajvan N´emetorsz´ ag Egyes¨ ult Arab Em´ırs´egek India Kazahszt´ an Szingap´ ur Indon´ezia Ausztr´ alia Izrael Gr´ uzia Bulg´aria Magyarorsz´ag Thaif¨ old Hongkong Jap´ an

arany´erem 5 4 4 3 3 2 2 2 1 1 1 1 1 1 1 1 1 0 0 0

ezu ¨st´erem 0 1 1 2 1 3 1 0 4 4 4 3 2 2 2 1 1 5 4 3

bronz´erem 0 0 0 0 1 0 2 3 0 0 0 1 2 2 2 3 3 0 1 2

dics´eret

pontsz´am

0 0 0 0 0 0 0 0 0 0 0 0 0 0 0 0 0 0 0 0

212,3 134,7 130,45 119,0 106,25 119,2 100,8 93,15 111,35 103,7 99,7 89,95 91,7 87,1 80,4 83,0 81,5 93,2 101,95 87,6

a magyar fizikai di´akolimpiai szakk¨ or¨ ok honlapj´an1 , majd egy h´et m´ ulva az IPhO ” 2 at bemutat´ o videHungary” YouTube-csatorn´ ara feltett´ek a feladatok megold´as´ ´ot. Rem´elhet˝ oleg ezek ´es a kor´ abban m´ ar felt¨ olt¨ ott vide´ ok is hat´ekonyan seg´ıtik az ´erdekl˝ od˝ o di´ akok egy´eni tanul´as´ at, versenyekre val´ o felk´esz¨ ul´es´et. A csapat kiv´ alaszt´ asa az el˝ oz˝ o ´evhez hasonl´oan h´ arom v´ alogat´ ok¨ orben t¨ ort´ent. akolimpiai szakk¨ or¨ ok honlapj´an meghirdetett els˝ o, A K¨oMaL3 -ban ´es a fizikai di´ online fordul´ ora m´ arcius k¨ ozep´en ker¨ ult sor. Ezen b´ arki indulhatott, el´eg volt a szakk¨ori honlapon k¨ ozz´etett feladatsor megold´as´ at szkennelve bek¨ uldeni. A h´ arom ´es f´el ´or´ as versenyen kit˝ uz¨ ott feladatokat nagyj´ab´ ol 20 di´ak k¨ uldte be. A be´erkezett dolgozat pontsz´ ama alapj´an a legjobb 11 di´ak kapott lehet˝os´eget a v´alogat´ as m´asodik k¨ or´eben val´ o r´eszv´etelre. A kiv´ alasztottak k¨ ozel egy h´ onapon kereszt¨ ul heti 3 alkalommal online felk´esz´ıt˝ o foglalkoz´ asokon ´es hetente egy vill´ amversenyen vettek r´eszt. Itt statisztikus termodinamika, fizikai optika, kvantumfizika, relativit´aselm´elet t´emak¨ or¨ okr˝ ol esett sz´o. A harmadik v´ alogat´ ok¨ ort az elm´ ult ´evekben megszokott, k´etnapos Kunfalviverseny jelentette. Az a´prilis v´eg´en megrendezett verseny ¨ ot ´ or´ as elm´eleti, h´arom ´or´ as m´er´esi ´es h´ arom ´or´ as sz´am´ıt´ og´epes szimul´ aci´ os r´eszb˝ ol a´llt. Az elm´eleti feladatok olimpiai st´ılus´ uak (hossz´ uak ´es sok alk´erd´esb˝ ol a´ll´ ok) voltak. A v´ alogat´ as h´ arom k¨ or´eben szerzett ¨ osszpontsz´ am alapj´ an kialakult az ¨ otf˝ os csapat, amely az 52. Nemzetk¨ ozi Fizikai Di´akolimpi´an k´epviselte haz´ ankat: Bencz Benedek, 9. oszt., Budapest, Ba´ar–Madas Reform´ atus Gimn´ azium, tan´ ara: Horv´ ath Norbert; Gurz´ o J´ozsef, 12. oszt., Budapest, Fazekas Mih´aly Gimn´ azium, tan´ ara: Nagy Piroska M´ aria; Kert´esz Bal´azs, 12. oszt., Debrecen, DRK D´ oczy Gimn´ aziuma, tan´ ara: T´ ofalusi P´eter; Kov´acs Bal´azs Csaba, 12. oszt., Hatvan, Bajza J´ ozsef Gimn´ azium, tan´ arai: Maruzsin´e Sevella Judit, Kov´ acs L´ aszl´ o; Toronyi Andr´as, 12. oszt., Budapest, Ba´ar–Madas Reform´ atus Gimn´ azium, tan´ ara: Horv´ ath Norbert. Megjegyezz¨ uk, hogy a v´ alogat´ oversenyen val´o szerepl´ese alapj´an beker¨ ult az olimpiai keretbe Moln´ar-Szab´ o Vilmos (11. oszt., Budapest, Fazekas Mih´aly Gimn, tan´ ara: Nagy Piroska M´ aria). Vilmos az IPhO r´eszv´etelr˝ ol lemondott, mert a verseny idej´en zajl´ o Nemzetk¨ ozi Matematikai Di´ akolimpi´an er˝os´ıtette a magyar csapatot egy bronz´eremmel. A csapat sz´ am´ara az els˝ o nemzetk¨ ozi er˝ opr´ oba a m´ ajus v´eg´en Szlov´eni´aban, Ljubljan´aban megrendezett Eur´ opai Fizikai Di´ akolimpia 4 (EuPhO) volt, amely a vil´ag m´asodik legnagyobb k¨ oz´episkol´ as fizikaversenye. (Ezen a versenyen Kert´esz Bal´ azs helyett Moln´ar-Szab´o Vilmos k´epviselte orsz´ agunkat.) Az EuPhO-n a ma-

A csapattagok kiv´ alaszt´ asa ´es felk´esz´ıt´ese most is az orsz´ ag ¨ot v´ aros´ aban (Budapesten, Miskolcon, Szegeden, Sz´ekesfeh´erv´ aron ´es P´ecsen) m˝ uk¨od˝o fizikai di´ akolimpiai szakk¨ or¨ ok¨ on zajlott. A budapesti szakk¨ or ebben a tan´evben is online m´ odban m˝ uk¨ od¨ ott; a szakk¨ orvezet˝ ok heti rendszeress´eggel feladatsorokat tettek k¨ozz´e

1

361

362

K¨ oz´ episkolai Matematikai ´ es Fizikai Lapok, 2022/6

https://ipho.elte.hu https://www.youtube.com/c/IPhOHungary 3 https://www.komal.hu 4 https://eupho.ee/eupho-2022 2

K¨ oz´ episkolai Matematikai ´ es Fizikai Lapok, 2022/6

gyar csapat egy arany´ermet (Kov´ acs Bal´ azs Csaba), egy ez¨ ust´ermet (Gurz´ o J´ ozsef), k´et bronz´ermet (Bencz Benedek, Toronyi Andr´as) ´es egy dics´eretet (Moln´ ar-Szab´ o Vilmos) szerzett. Az Eur´ opai Fizikai Di´akolimpi´ar´ ol sz´ol´o besz´ amol´o az okt´ oberi sz´ amunkban olvashat´o majd. A csapat tr´eningben tart´asa a ny´ ar elej´en is folytat´ odott; rendszeresen k¨ uldt¨ unk a csapattagoknak egy-egy r´egebbi IPhO elm´eleti feladatsort, amit a csapattagok ¨ on´ all´ oan megoldottak. N´eh´ any nap m´ ulva elk¨ uldt¨ uk a hivatalos megold´ ast is, majd kicsit k´es˝ obb online megbesz´elt¨ uk a feladatsor probl´em´ asabb r´eszeit, illetve r´eszletesebben k¨ orbej´artunk egy-egy k´erd´est. Az eredeti tervek szerint az 52. Nemzetk¨ ozi Fizikai Di´akolimpi´at Feh´eroroszorsz´ ag szervezte volna, de a nemzetk¨ ozi helyzetre val´ o tekintettel a IPhO Nemzetk¨ozi Bizotts´ aga ´ aprilisban t¨ or¨ olte az esem´enyt, ´es Sv´ ajc v´allalta a´t a verseny online foras´ at. K¨ osz¨ onet illeti a rendez˝oket ´es a nemzetk¨ozi seg´ıt˝oket, m´ aban val´ o lebonyol´ıt´ hogy ilyen r¨ ovid id˝o alatt siker¨ ult megszervezni ´es z¨okken˝ omentesen lebonyol´ıtani a versenyt! A versenyz˝ok t¨ obbs´ege saj´ at haz´ aj´aban, sz´ am´ıt´og´epes kapcsolaton kereszt¨ ul vett r´eszt az esem´enyen, de voltak olyan helysz´ınek is, ahol n´eh´ any orsz´ ag deleg´aci´ oja ¨ osszej¨ott, ´es egy k¨ oz¨ os helysz´ınen szervezt´ek meg a r´eszv´etelt, ezzel valamennyire visszaid´ezve a helysz´ıni szervez´es˝ u di´akolimpi´ak hangulat´ at. A versenyen 368 tanul´ o m´erte ¨ ossze tud´ as´ at, akik a t´ız egy´eni indul´o mellett mintegy 70 orsz´agot k´epviseltek. A magyar deleg´aci´ ot az ¨ ot csapattagon t´ ul Sarkadi Tam´ as (BME Fizikai Int´eabor zet) ´es Tasn´ adi Tam´ as (BME Matematikai Int´ezet) csapatvezet˝ ok, Sz´echenyi G´ (ELTE Fizikai Int´ezet) megfigyel˝o, Sz´ asz Kriszti´ an (BME Fizikai Int´ezet) ´es Vigh M´ at´e fel¨ ugyel˝ ok alkott´ ak, akik j´ ulius 10-t˝ ol 15-ig Balatonf¨ uredre, egy hotelbe k¨olt¨ oztek a verseny idej´ere. Az el˝ok´esz´ıt˝ o munk´ ak nagy r´esze ´es a verseny lebonyol´ıt´ asa a hotelt˝ ol t´ız percre lev˝o, u ´jonnan megny´ılt BME Tud´ascentrumban zajlott. A csapatvezet˝ ok ´es a megfigyel˝ o v´egezt´ek a ford´ıt´ast, jav´ıt´ast, moder´ al´ast, a fel¨ ugyel˝ ok gondoskodtak a di´ akok ell´at´ as´ar´ ol ´es a verseny szab´alyszer˝ u lebonyol´ıt´as´ar´ ol, valamint ki´ep´ıtett´ek a rendez˝ okkel a folyamatos vide´ okamer´as fel¨ ugyeletet. Az IPhO els˝ o versenynapj´ an az ¨ ot´ or´ as k´ıs´erleti fordul´ ora ker¨ ult sor. Az IPhO or a versenyz˝ok nem val´odi, hanem szimul´ alt k´ıs´erleteket v´et¨ ort´enet´eben el˝ osz¨ geztek el ´es ´ert´ekeltek ki. Ez azt jelenti, hogy a m´er´esi eredm´enyeket nem val´ odi eszk¨ oz¨ okkel, manu´ alisan elv´egzett m´er´esek szolg´altatt´ ak, hanem egy egyszer˝ u, sz¨oveges ablakban fut´ o programba kellett be´ırni n´eh´ any bemen˝o param´etert, ´es ennek f¨ uggv´eny´eben a program ki´ırt n´eh´ any m´er´esi eredm´enyt. (A programokat a fel¨ ugyel˝ ok el˝ozetesen telep´ıtett´ek a versenyz˝ok ´ altal haszn´ alt laptopokra.) A szimul´ aci´ os program gondoskodott m´eg arr´ ol is, hogy az eredm´enyeket megfelel˝ o v´eletlen hiba terhelje. A m´er´es t¨ obbi r´esze, az adatok felv´etele, ki´ert´ekel´ese m´ ar ugyan´ ugy zajlott, mint egy val´ odi m´er´esn´el. A versenyz˝ ok k´et m´er´esi feladatot kaptak. Az els˝ o feladatban egy ismeretlen ol kellett bolyg´ on k¨ ul¨ onb¨ oz˝ o param´eter˝ u goly´ ok (szimul´alt) leejt´es´enek eredm´enyeib˝ a bolyg´onak ´es l´egk¨ or´enek adataira k¨ ovetkeztetni. A m´asodik feladatban a versenyz˝ ok egy hengeres v´akuumdi´od´at vizsg´altak, a geometriai param´eterek ´es a di´ od´ara kapcsolt fesz¨ ults´eg f¨ uggv´eny´eben m´erhett´ek” az a´ramot, ´es a di´oda karakterisztik´ a” j´aban szerepl˝o param´eterek meghat´ aroz´ asa volt a feladat. Mindk´et m´er´esi feladat K¨ oz´ episkolai Matematikai ´ es Fizikai Lapok, 2022/6

363

a szok´ asos IPhO feladatokn´al nyitottabb” volt; a versenyz˝ oknek maguknak kel” lett r´ aj¨onni¨ uk, hogy egy-egy param´eter meghat´ aroz´ as´ ahoz milyen be´ all´ıt´ as mellett milyen adatsort ´erdemes f¨ olvenni. Az elm´eleti versenynapon a di´ akok h´ arom feladatot oldottak meg ¨ ot ´ ora alatt. Az els˝o feladatot a j´at´ekboltokban kaphat´ o, kicsiny, nagyon er˝os m´agneses goly´ okb´ ol ¨osszerakott NeoCube kocka motiv´alta. A feladatban m´ agneses dip´ olusok k¨ozti k¨olcs¨onhat´ ast vizsg´ alt´ ak a versenyz˝ok, t¨ obbek k¨ oz¨ ott olyan elrendez´esekben is, amik a j´at´ekm´agnesekkel megval´ os´ıthat´ ok. A m´asodik feladat a nemr´eg p´ aly´ ara ´all´ıtott James Webb u ˝rteleszk´ oppal kapcsolatos ´erdekes probl´em´akr´ ol sz´olt; volt a feladatban optika, termodinamika ´es modern fizika is. Remek volt az id˝ oz´ıt´es; a t´avcs˝o ´eppen az elm´eleti fordul´o napj´ an k¨ uldte el els˝ o sikeres felv´eteleit. A harmadik k´erd´esben n´egy (nem csak hatv´ anyf¨ uggv´enyeken alapul´ o) sk´alat¨ orv´enyekkel megoldand´ o feladatot t˝ uztek ki. A probl´em´ ak a fizika k¨ ul¨ onb¨ oz˝ o ter¨ uleteihez, mechanik´ ahoz, hidrodinamik´ ahoz, relativit´aselm´elethez kapcsol´odtak, s˝ ot, volt olyan k´erd´es is, ahol a relev´ ans ter¨ ulet meghat´ aroz´ asa is fejt¨ or´est ig´enyelt (vizes homok szil´ards´aga). A feladatok helyes megold´ as´ ahoz t¨ obb k¨ ul¨ onb¨ oz˝ o gondolatmenettel is ¨ el lehetett jutni. Osszess´ eg´eben, az elm´eleti fordul´o feladatai nagyon sz´epek, nehezek voltak, melyek megold´ as´ ahoz ¨ otletekre, fizikai gondolkod´ asra ´es intu´ıci´ ora volt sz¨ uks´eg. A feladatok ´es a megold´ asok megtekinthet˝ ok az idei verseny hivatalos honlapj´an5 . Az eg´esz verseny sz´ınvonal´ at, a probl´em´ ak neh´ezs´eg´et j´ ol mutatja, hogy a maxim´alisan megszerezhet˝o 50 pontb´ ol (20 a m´er´esre, 30 az elm´eletre) 23,70 pont el´er´es´evel m´ar arany´ermet, 16,05 ponttal ez¨ ust¨ ot ´es 11,65 ponttal bronz´ermet lehetett szerezni, a dics´eret als´ o hat´ ara pedig 7,15 pont volt. A magyar versenyz˝ok eredm´enye a k¨ ovetkez˝ o: Kov´acs Bal´azs Csaba: arany´erem (24,00 pont); Gurz´ o J´ozsef: ez¨ ust´erem (16,70 pont); Toronyi Andr´as: bronz´erem (14,70 pont); Kert´esz Bal´azs: bronz´erem (14,10 pont); Bencz Benedek: bronz´erem (12,00 pont). Gratul´alunk a csapatnak a sz´ep eredm´enyhez! Szeretn´enk k¨ osz¨ onetet mondani a di´akok k¨oz´episkolai tan´ arainak, valamint sok sikert ´es hasonl´ oan tehets´eges tan´ıtv´ anyokat k´ıv´ anunk nekik a tov´ abbiakban. K¨ osz¨ onet az ¨ ot magyarorsz´agi olimpiai ok´esz´ıt˝o szakk¨ or vezet˝oinek a sok ´even a´t´ıvel˝ o, kitart´o munk´ ajuk´ert. K¨ ul¨ on k¨ osz¨ oel˝ net illeti tov´ abb´ a Sz´ asz Kriszti´ ant, Sz´echenyi G´ abort, Vank´ o P´etert ´es Vigh M´ at´et a csapat felk´esz´ıt´es´eben ny´ ujtott seg´ıts´eg´ert. K¨ osz¨ onet illeti a Budapesti M˝ uszaki ´es Gazdas´ agtudom´anyi Egyetemet, mert rendelkez´es¨ unkre bocs´ atotta Balatonf¨ ureden az u ´j Tud´ascentrum´at, ´es k¨ osz¨ onettel tartozunk az anyagi t´ amogat´ as´ert az Emberi Er˝ oforr´ asok Miniszt´erium´anak. B´ar az online versenyek hangulata utol sem ´erheti egy helysz´ıni rendez´es˝ u IPhO var´ azs´ at, Balatonf¨ uredr˝ ol is maradand´ o, sz´ep eml´ekekkel t´ert¨ unk haza. Sokat s´eunk egy haj´ okir´ andul´ asra, l´ athattuk a K´ekszalag t´altunk a Tagore s´et´ anyon, elment¨ 5

364

https://ipho2022.com

K¨ oz´ episkolai Matematikai ´ es Fizikai Lapok, 2022/6

Balatonker¨ ul˝ o vitorl´ asverseny rajtj´ at, s˝ ot, m´eg aznap d´elut´ an a Tihanyi Benc´es Ap´ ats´ ag alatt egy cukr´aszd´ aban fagyizva figyelhett¨ uk, ahogy a f´elsziget perem´en felt˝ unnek a leggyorsabb katamar´ an rohan´ ok fekete vitorl´ai, ´es a haj´ok befutnak a f¨ uredi c´elba. o A k¨ ovetkez˝ o, 2023. ´evi di´akolimpi´at Jap´ an rendezi Toki´oban6 . A versenyre val´ felk´esz¨ ul´est a n´egy vid´eki ´es a budapesti szakk¨ or seg´ıti:

rendezte az esem´enyt, ahol a Csillag´ aszati ´es F¨ oldtudom´ anyi Kutat´ ok¨ ozpont is r´eszt vett a szervez´esben. Az ´evek m´ ul´ as´ aval egyre n´epszer˝ ubb´e v´ alt a verseny. N˝ott a jelentkez˝ok sz´ama, ezzel p´ arhuzamosan a szakmai felk´esz´ıt˝ ok is egyre t¨ obben csatlakoztak a jobb eredm´enyek el´er´es´enek ´erdek´eben. Ennek ok´ an sz¨ uletett meg tavaly, a Kolumbia ´altal szervezett di´ akolimpi´an az els˝o magyar IOAA arany´erem.

Budapest: Sz´ asz Kriszti´ an (BME Fizikai Int´ezet, Budafoki u ´t 8.), Miskolc: Z´ amborszky Ferenc (F¨ oldes Ferenc Gimn´azium, H˝os¨ok tere 7.),

Ez a n¨oveked´es adott okot arra, hogy legyen egy beazonos´ıthat´ o elnevez´ese a versenynek. Megsz¨ uletett az Athletica Galactica, amely Magyarorsz´ agon egyed¨ ul´all´o megm´erettet´es k¨ oz´episkol´ asok sz´am´ara. A fizika ´es matematika ter´en kiemelked˝o, a csillag´aszat ´es az u ˝rkutat´ as ir´ant ´erdekl˝ od˝o di´ akok egy h´aromfordul´ os torn´an vesznek r´eszt, amelynek a v´eg´en az orsz´agos v´ alogat´ okon tov´ abbjutott 25 di´ ak k¨ oul kiv´ alaszt´ asra ker¨ ul a legjobb 10 versenyz˝o. Az ˝ o di´akolimpiai felk´esz´ıt´es¨ uket z¨ szakmai csapatunk veszi a´t. A felk´esz´ıt´es h´ arom felk´esz´ıt˝ o h´etv´eg´eb˝ ol, heti feladatsorokb´ ol, valamint egy felk´esz´ıt˝o ´es v´ alogat´ o t´ aborb´ol a´ll, amelyek sor´ an a di´ akok mind elm´eleti, mind gyakorlati tekintetben olimpiai szintre fejl˝ odhetnek. Ezenfel¨ ul pedig a nagy megm´erettet´est megel˝ ozi egy miniolimpia, amelyet szomsz´edos orsz´agokkal forg´o rendszerben tartunk minden ´evben. Id´en Szlov´enia adott otthont ennek az esem´enynek, j¨ov˝ore pedig v´ arhat´ oan Horv´atorsz´ ag szervezi meg.

P´ecs: P´ alfalvi L´ aszl´ o (P´ecsi Tudom´anyegyetem Fizikai Int´ezet, Ifj´ us´ag u ´tja 6.), Szeged: Sarl´ os Ferenc ´es Cs´ anyi S´ andor (Szegedi Tudom´anyegyetem, D´om t´er 9.), ´ Sz´ekesfeh´erv´ar: Orosz Tam´ as (Obudai Egyetem Alba Regia M˝ uszaki Kar, Budai u ´t 45.). A szakk¨ or¨ okkel kapcsolatos tov´abbi tudnival´ok, el´erhet˝ os´egek, aktualit´asok ´es a felk´esz¨ ul´est seg´ıt˝ o anyagok a fizika di´akolimpiai szakk¨or¨ok hivatalos honlapj´an ul els˝ osorban ¨on´all´o olvashat´oak: http://ipho.elte.hu. A fenti szakk¨or¨ok¨on k´ıv¨ munk´ aval, a K¨ oMaL elm´eleti ´es m´er´esi feladatainak rendszeres megold´ as´aval ´es a hazai fizikaversenyeken val´ o rendszeres r´eszv´etellel lehet k´esz¨ ulni a j¨ov˝o ´evi fizikai di´ akolimpi´ara. Eredm´enyes felk´esz¨ ul´est k´ıv´ anunk! Sarkadi Tam´as ´es Tasn´adi Tam´as, csapatvezet˝ ok

A Nemzetk¨ ozi Csillag´aszati ´es Asztrofizikai Di´akolimpi´ar´ ol ´es v´alogat´ overseny´er˝ ol, az Athletica Galactic´ar´ ol Napjainkban egyre n´epszer˝ ubb a titokzatos u ˝r felfedez´ese, a csillagok f¨ urk´esz´ese. A k¨ ul¨ onb¨ oz˝ o technikai fejleszt´esek, t´ avcs¨ ovek, a vil´agh´al´o ny´ ujtotta t´er kiterjeszt´ese mind seg´ıtik ennek a vil´agnak a megismer´es´et. A csillag´aszat a felfedez´es tudom´ anya. Ahogy az ´egbolt, u ´gy a lehet˝ os´egeink is hat´ artalanok. Az Athletica Galactica verseny kiv´ al´o alkalom ´es lehet˝ os´eg arra, hogy megtal´alja a j¨ ov˝o k¨ oz´episkol´ as kozmikus tehets´egeit, akiknek a kisujj´aban van a fizika, a matematika ´es az informatika. A mozgalom nem u ´j kelet˝ u, m´ ar t¨obb mint 10 ´eve van jelen haz´ankban. A kezdetekt˝ ol a Bajai Obszervat´orium Alap´ıtv´ any ´egisze alatt futott a felk´esz´ıt´es. Olimpikonjaink ´evr˝ ol ´evre egyre eredm´enyesebben szerepeltek a Nemzetk¨ozi Csillag´aszati ´es Asztrofizikai Di´akolimpi´an (IOAA). Azt´ an 2019-ben m´ar haz´ ank 6

K¨ uldet´es¨ unknek ´erezz¨ uk, hogy seg´ıts´eget biztos´ıtsunk a di´akok ´es tan´ arok sz´am´ara ismereteik b˝ ov´ıt´es´ehez, gyakorl´ashoz. Tessz¨ uk ezt egy kor´ abbi olimpikonunk, D´ alya Gergely ´ altal ´ırt, m´ar az egyetemek ´altal is haszn´ alt k¨ onyvvel. Ez a k¨otet az olimpia t´emak¨ oreit fel¨ olel˝ o elm´eleti ¨ osszefoglal´ as mintafeladatokkal, laikusoknak ´es a szakm´ anak egyar´ ant. Hamarosan nyomtat´ asban is megjelenik emell´e egy feladatgy˝ ujtem´eny megold´ okulccsal ´es magyar´ azatokkal. T´ avcs˝ohaszn´ alati ´es -ismereti tanfolyamainkkal pr´ob´ alunk hozz´ aj´arulni ahhoz, hogy a di´akok b´ atran tudjanak tan´araikhoz fordulni seg´ıts´eg´ert. Ehhez a kezdem´enyez´eshez az orsz´ ag t¨obb pontj´an megtal´alhat´ o partnereink is csatlakoztak. Az Athletica Galactic´ ara, valamint mag´ara az IOAA-ra sz¨ uks´eges elm´eleti ´es gyakorlati tud´ as elsaj´ at´ıt´ as´at di´akolimpiai szakk¨ ori h´ al´ozat seg´ıti, amely orsz´ agszerte, k¨ ul¨onb¨ oz˝ o szinteken foly´ o munk´aj´aval ´epp´ ugy helyet ad a teljesen kezd˝o, mint a m´ar nem el˝osz¨ or versenyz˝o di´akoknak. S˝ ot a r´esztvev˝ ok egy remek, motia is v´alnak. v´alt, o¨sszetart´ o ´es vid´ am csapat tagjaiv´ IOAA Olimpiai Szakk¨ or, Budapesten ´es online Az Olimpiai Szakk¨ or 2013 o´ta alkotja a szakk¨ ori h´ al´ozat gerinc´et, felk´esz´ıtve a matematik´ ab´ ol ´es fizik´ ab´ ol tehets´eges, 10–12. oszt´ alyos di´ akokat az Athletica Galactica d¨ont˝ oj´enek szintj´ere. A szakk¨ ort volt di´ akolimpikonok tartj´ak, akik a d¨ ont˝o ut´ ani keretfelk´esz´ıt´esben is r´eszt vesznek, Kalup Csilla, az ELTE Csillag´aszati Tansz´eke doktorandusz´ anak szakk¨ orvezet´es´evel. ´ A szakk¨or¨ ok az ELTE TTK l´ agym´anyosi campus Eszaki T¨ ombj´eben folynak szeptembert˝ ol a d¨ ont˝ oig, kb. k´ethetente, szombatonk´ent, 09:00 ´es 14:00 k¨ oz¨ ott, de online csatlakoz´asra is lehet˝ os´eg van. Az els˝ o h´ arom szakk¨ or id˝opontja szeptember 17., okt´ ober 1. ´es okt´ ober 22. A r´eszv´etel ingyenes, de regisztr´aci´ ohoz k¨ ot¨ ott.

https://ipho2023.jp

K¨ oz´ episkolai Matematikai ´ es Fizikai Lapok, 2022/6

365

366

K¨ oz´ episkolai Matematikai ´ es Fizikai Lapok, 2022/6

Budapesti alapoz´ o szakk¨ or ´es vid´eki szakk¨ or¨ ok ´ Az Ujpesti K¨ onyves K´ alm´an Gimn´aziumban a matematikai, fizikai ´es csillag´ aszati tud´ ast megalapoz´ o szakk¨ ort tart Udvardi Imre matematika-fizika szakos tan´ ar 8-10. oszt´ alyosoknak. A szakk¨ ori foglalkoz´ asok p´entekenk´ent 15:00-t˝ ol 18:00ig vannak, amelyeket 22:00-ig t´ avcs¨ ovez´es k¨ ovet az iskola Kulin Csillagd´aj´aban. Els˝ o alkalom: okt´ober 7. A fentiek mellett sz´amos vid´eki szakk¨ or is felk´esz¨ ul´esi lehet˝ os´eget k´ın´ al. Tov´ abbi inform´ aci´ o a szakk¨ or¨ okr˝ ol, valamint jelentkez´esi inform´aci´ oa https://athleticagalactica.hu/orszagos-szakkori-halozat c´ımen tal´ alhat´ o. Aki kedvet ´erez mag´ aban, hogy r´eszese legyen egy kiv´al´o csapatnak, vagy csak foglalkoztatja, hogy mi mindent rejt a csillagos ´eg, a csod´ alatos univerzum, csatlakozzon a megadott el´erhet˝ os´egeken. Akik ki szeretn´ek pr´ob´ alni magukat a hazai v´ alogat´ o versenysorozaton, hogy felm´erj´ek tud´ asukat, azok pedig al´ abb olvashatj´ ak, hogy mik´ent tudnak jelentkezni. ´ FELH´IVAS az Athletica Galactica K´arp´at-medencei K¨ oz´episkolai Csillag´aszati ´es Asztrofizikai Versenyre

A Csillag´aszati ´es F¨ oldtudom´ anyi Kutat´ ok¨ ozpont K´ arp´ at-medence magyar ajk´ u k¨ oz´episkol´ as di´akjai sz´ am´ara orsz´ agos versenyt hirdet. Ism´et keress¨ uk azokat a k¨ oz´episkolai pedag´ogusokat, akik a fizika ´es matematika ter´en kiemelked˝o, a csillag´aszat ´es az u ˝rkutat´ as ir´ant ´erdekl˝ od˝o k¨ oz´episkol´asokat ¨oszt¨onzik a nevez´esre, s felk´esz¨ ul´es¨ uket seg´ıtik a h´ aromfordul´ os verseny sor´an. Tan´ ari t´ amogat´ asukkal hozz´ aj´arulnak a hazai d¨ ont˝ oben val´ o megm´erettet´esre, ´ıgy lehet˝ os´eget adva a legtehets´egesebb di´ akok sz´am´ara a Nemzetk¨ ozi Csillag´aszati ´es Asztrofizikai Di´akolimpi´an (IOAA, International Olympiad on Astronomy and Astrophysics) val´o r´eszv´etelre, amelyet Lengyelorsz´ag szervez 2023. augusztus v´eg´en. ulnek ki a magyar nemzeti di´akolimpiai A d¨ ont˝ obe jut´o di´akok legjobbjaib´ol ker¨ keret tagjai. A 2022-es Athletica Galactica verseny fordul´ oinak id˝ opontjai: I. (iskolai) fordul´o: 2022. november 16. (szerda), 15.00 o´ra (id˝ otartam: 120 perc); II. (iskolai) fordul´o: 2022. december 14. (szerda), 15.00 o´ra (id˝ otartam: 120 perc); III. (iskolai, sz´ am´ıt´ og´epes) fordul´ o: 2023. janu´ ar 11. (szerda), 15.00 ´ora (id˝ otartam: 120 perc); Orsz´ agos d¨ ont˝ o (terv): 2023. m´arcius 24–26. Jelentkezni honlapunkon (www.athleticagalactica.hu), ´erdekl˝ odni az al´ abbi el´erhet˝ os´egeken lehet: Vincze Nikolett +36 30 683 6154 [email protected] K¨ oz´ episkolai Matematikai ´ es Fizikai Lapok, 2022/6

367

Kunfalvi Rezs˝ o Olimpiai V´alogat´ overseny 1. elm´eleti fordul´o 2022. m´arcius 21. 15:00 Figyelem!∗ A versenyen nem-grafikus sz´ amol´ og´epen, ´ır´ o- ´es rajzeszk¨ oz¨ ok¨ on k´ıv¨ ul semmilyen m´ as seg´edeszk¨ oz (pl. k¨ onyv, f¨ uzet, t´ abl´ azatok, internet) nem haszn´ alhat´ o. A feladatok megold´ as´ at k´ez´ır´ assal, pap´ırra kell elk´esz´ıteni, minden feladat megold´ asa u ´j oldalon kezd˝ odj¨ on. Az els˝ o oldalon szerepeljen a versenyz˝ o neve, ´evfolyama, felk´esz´ıt˝ o tan´ arainak ´es iskol´ aj´ anak neve. T¨ orekedni kell a j´ ol a ´ttekinthet˝ o k¨ ulalakra, az olvashat´ o k´ez´ır´ asra, a megold´ asok fizikai alapjainak ismertet´es´ere, valamint a magyaros, vil´ agos ´es t¨ om¨ or fogalmaz´ asra. Minden feladat azonos pontsz´ amot ´er. A verseny id˝ otartama 3 ´ ora, amelynek lej´ arta ut´ an tov´ abbi 30 perc ´ all rendelkez´esre a megold´ asok digitaliz´ al´ as´ ara ´es elk¨ uld´es´ere. A megasokat egyetlen pdf-dokumentumban a verseny napj´ an (2022. m´ arcius 21.) 18:30-ig kell old´ elk¨ uldeni az [email protected] c´ımre. A k´es˝ on ´erkezett dolgozatokat nem tudjuk elfogadni. A pdf-dokumentum k´esz¨ ulhet p´eld´ aul mobiltelefonos alkalmaz´ assal vagy szkennerrel.

F1. Az ´ abr´ an l´ athat´ o k´et pontszer˝ u, szabad elmozdul´asra k´epes test k¨ oz¨ ott csak a t¨ omegvonz´ as hat. Kezdetben a testek t´avols´ aga d, az egyik m t¨ omeg˝ u test nyugalombanvan, a m´asik (ugyancsak m t¨ omeg˝ u) test abr´ an l´ atpedig v0 = k γm/d sebess´eggel mozog az ´ hat´ o ir´anyban, ahol k param´eter, γ pedig a gravit´ aci´ os ´alland´ o. a) Legal´ abb mekkora k ´ert´eke, ha a testek hossz´ u id˝o ut´ an egym´ast´ ol nagyon messzire ( v´egtelen” t´ avolra) ker¨ ulnek? ” b) Mekkora a testek k¨ oz¨ otti legnagyobb t´ avols´ ag k = 1 eset´en?

F2. Egy szigetel˝oanyagb´ ol k´esz¨ ult, R sugar´ u, L hossz´ us´ ag´ u (L R), v´ekony fal´ u, M t¨ omeg˝ u cs˝ o egyenletesen fel van t¨ oltve σ fel¨ uleti t¨ olt´ess˝ ur˝ us´eggel. A cs˝ o r¨ ogz´ıtett szimmetriatengelye k¨ or¨ ul s´ url´od´ asmentesen foroghat. A cs˝ ore fonalat cs´ev´el¨ unk, melynek v´eg´ere m t¨ omeg˝ u kis testet r¨ ogz´ıt¨ unk. Mekkora gyorsul´ assal mozog a kis test, ha elengedj¨ uk?



A versenyz˝ oknek sz´ ol´ o technikai inform´ aci´ ok itteni k¨ ozl´es´evel a k´es˝ obbi, hasonl´ o st´ılusban megrendezend˝ o versenyek r´esztvev˝ oit szeretn´enk seg´ıteni. (A Szerk.)

368

K¨ oz´ episkolai Matematikai ´ es Fizikai Lapok, 2022/6

Ezek szerint fenn´all:

F3. Az ´ abr´ an l´ athat´ o´ aramk¨ orben a telepek, a di´da ´es a tekercs ide´alis. A K kapcsol´o hossz´ o u ideje z´ arva van. Adatok: L = 150 mH, C = 200 nF, R = 500 Ω, U0 = 9,0 V. a) Mekkora maxim´ alis Umax fesz¨ ults´egre t¨ olt˝ odik fel a kondenz´ ator, miut´an a kapcsol´ot kinyitjuk? b) A kapcsol´ o kinyit´ asa ut´ an mennyi id˝ ovel ´eri el a kondenz´ ator fesz¨ ults´ege az Umax ´ert´eket?

n 27,32 + n = , 365,26 27,32

´es ennek az egyenletnek a megold´asa: n = 2,21 ≈ 2. Teh´at val´oban kb. k´et nappal t¨ obb id˝ o telik el k´et holdt¨ olte k¨ oz¨ ott, mint amennyi a Hold kering´esi ideje.

F4. A pozitron egyszeresen pozit´ıv t¨ olt´es˝ u elemi r´eszecske, melynek t¨omege omeg´evel. Egy a´ll´ onak tekinthet˝o elektronnak 3m0 c2 mozegyenl˝ o az elektron m0 t¨ g´asi energi´ aj´ u pozitront u oztet¨ unk, melynek k¨ ovetkezt´eben annihil´aci´ o k¨ovetkezik ¨tk¨ be ´es az energia k´et foton form´ aj´aban sug´arz´ odik sz´et. a) Felt´eve, hogy a k´et foton ellent´etes ir´ anyban rep¨ ul sz´et, mekkora a fotonok hull´amhossz´ anak ar´anya? b) Mekkora a k´et foton kirep¨ ul´esi ir´ anya a´ltal bez´ art sz¨og lehets´eges legkisebb ´ert´eke?

Feh´erv´ ari Don´ at (Miskolc, F¨ oldes Ferenc Gimn., 10. ´evf.) Megjegyz´es. Egyszer˝ us´ıtett – ak´ ar fejben is elv´egezhet˝ o – sz´ am´ıt´ as a k¨ ovetkez˝ o. A F¨ old 1

ozel´ıt˝ oleg 1 h´ onap ( holdnap”). kering´esi ideje 12 h´ onap, a Hold´e ennek kb. 12 -ed r´esze, k¨ ” A Hold teh´ at az ´ all´ ocsillagokhoz viszony´ıtott kering´esi idej´ehez k´epest annak m´eg kb. 1 -ed r´esze, vagyis hozz´ avet˝ olegesen 2 nappal k´es˝ obb ker¨ ul ism´et a Nap–F¨ old egyenesre. 12

12 dolgozat ´erkezett. Helyes 7 megold´ as. Kicsit hi´ anyos (3 pont) 1, hi´ anyos (1 pont) 1, hib´ as 2, nem versenyszer˝ u 1 dolgozat.

Fizika feladatok megold´asa Fizika gyakorlat megold´asa

G. 773. A F¨ old–Hold rendszer a k´et ´egitest k¨ oz¨ os t¨ omegk¨ oz´eppontja k¨ or¨ ul kering 27,32 napos kering´esi id˝ ovel a t´ avoli a ´ll´ ocsillagokhoz k´epest. Ehhez k´epest t¨ obb mint k´et nappal hosszabb id˝ o, ´ atlagosan 29,53 nap telik el k´et egym´ ast k¨ ovet˝ o holdt¨ olte k¨ oz¨ ott. Magyar´ azzuk meg a k´etf´ele peri´ odusid˝ o k¨ oz¨ otti k¨ ul¨ onbs´eget, ´es egyszer˝ us´ıtett sz´ am´ıt´ assal mutassuk meg, hogy val´ oban nagyj´ ab´ ol k´et nap az elt´er´es! (4 pont) Megold´as. Holdt¨oltekor a Hold, a F¨ old ´es a Nap (k¨ozel´ıt˝oleg) egy egyenesbe oesik. K´et telihold k¨ oz¨ott a Nap–F¨ old egyenes valamekkora sz¨oggel elfordul az ´all´ ul a Hold csillagokhoz k´epest. Ugyanennyivel t¨ obbet kell elforduljon a 360◦ -on fel¨ a F¨ old k¨ or¨ ul, hogy megint a Nap–F¨ old egyenes´en legyen rajta. Legyen az egym´ ast k¨ ovet˝o k´et holdt¨ olte k¨ oz¨ ott eltelt id˝ o 27,32 + n nap. Mivel or¨ ul, 27,32 + n nap alatt a Nap–F¨old a F¨ old 365,26 nap alatt fordul 360◦ -ot a Nap k¨ egyenes elfordul´ asa 27,32 + n · 360◦ . α= 365,26 A Hold 27,32 nap alatt tesz meg egy teljes fordulatot a F¨old k¨or¨ ul, n nap alatt pedig a F¨ old–Hold egyenes m´eg elfordul α sz¨ oggel. Az id˝o egyenesen ar´anyos az elfordul´as sz¨ og´evel, teh´ at n · 360◦ . α= 27,32 K¨ oz´ episkolai Matematikai ´ es Fizikai Lapok, 2022/6

369

P. 5392. Egy sz¨ ok˝ ok´ ut k¨ oz´eps˝ o ny´ıl´ as´ an f¨ ugg˝ olegesen ki´ araml´ o v´ekony v´ızsug´ ar H magass´ agig jut el. A v´ızsug´ ar v´ızhozama”, azaz az id˝ oegys´egenk´ent ki´ araml´ o v´ız ” . Milyen h magass´ agban lebeg egy m t¨ omeg˝ u labda, ha a v´ızsug´ art´erfogata: Φ = ΔV Δt ba helyezz¨ uk? (Felt´etelezhetj¨ uk, hogy a v´ızsug´ ar teljes keresztmetszete el´eri a labd´ at, ´es arr´ ol v´ızszintes ir´ anyban spriccel sz´et.) (5 pont)

A Kvant nyom´ an

Megold´as. A v´ızsug´ar b´ armelyik keresztmetszet´en Δt id˝ o alatt ΔV = ΦΔt t´erfogat´ u, teh´ at ρΦΔt t¨ omeg˝ u v´ız ´aramlik a´t (ρ a v´ız s˝ ur˝ us´ege). Ennyi v´ıznek Δp = ρΦΔt · v nagys´ ag´ u, f¨ ugg˝olegesnek tekinthet˝ o impulzusa van, ahol v az ´ araml´o v´ız sebess´ege az adott magass´ agban. A labd´ anak csap´od´o v´ızsug´ar f¨ ugg˝oleges ir´ any´ u o alatt null´ ara cs¨ okken, teh´ at – Newton 2. t¨ orv´enye szerint – a v´ız impulzusa Δt id˝ F =

Δp = ρΦv Δt

er˝ ot fejt ki a labd´ara. Tudjuk, hogy a ny´ıl´ason ki´araml´ o, valamekkora M t¨ omeg˝ u v´ız kezdeti v0 sebess´ege ´es a maxim´alis H emelked´esi magass´ aga k¨ oz¨ otti ¨ osszef¨ ugg´es az energiamegmarad´ as t¨orv´enye szerint M 370

v02 = M gH, 2

azaz

v0 =



2gH.

K¨ oz´ episkolai Matematikai ´ es Fizikai Lapok, 2022/6

Hasonl´oan kapjuk, hogy h magass´ agban a v´ız sebess´ege:  v(h) = 2g(H − h).

Ahhoz, hogy az m t¨ omeg˝ u labda lebegjen, a r´ a hat´ o er˝ ok ered˝oj´enek null´anak kell lennie:  ρΦ 2g(H − h) = mg,

Megold´as. a) Az M t¨ omeg˝ u testre hat´ o fon´ aler˝ o legyen K1 , az m t¨ omeg˝ ure oz´es el˝ otti pillanatban az M t¨ omeg˝ u goly´ o hat´ o er˝ o pedig K2 . Tudjuk, hogy az u ¨tk¨ 0, a m´ a sik goly´ o sebess´ e ge pedig (az energiamegmarad´ a s t¨ o rv´ e nye sebess´ege v1 = √ szerint) v2 = 2gL . Az u otti pillanatban a nagyobb t¨ omeg˝ u test gyorsul´ asa nulla, a kisebb ¨tk¨oz´es el˝ abra). Newton II. t¨ orv´enye t¨omeg˝ u test (centripet´alis) gyorsul´ asa pedig v22 /L (1. ´ szerint a goly´ ok mozg´asegyenlete:

ahonnan a lebeg´es magass´ aga:

h=H−

K1 − M g = 0,

m2 g . 2ρ2 Φ2

illetve K2 − mg = m

v22 , L

ahonnan a fon´ aler˝ ok: K1 = K2 = 3mg.

A fenti egyenlet azt is megmondja, hogy legfeljebb mekkora t¨omeg˝ u labd´at lehet lebegtetni. Nyilv´ an h > 0, vagyis  m < mmax = ρΦ 2H/g.

ag´ u, f¨ ugg˝olegesen A felf¨ uggeszt´esi pontot ebben a pillanatban K1 + K2 = 6mg nagys´ lefel´e ir´anyul´o er˝ o terheli.

Csillingek csapat: ´ Isk. ´es Gimn., 9. ´evf.) ´es Csilling D´ aniel (Budapesti Fazekas M. Gyak. Alt. Csilling Katalin (Budapest, Szil´ agyi E. Gimn., 12. ´evf.)

31 dolgozat ´erkezett. Helyes 19 megold´ as. Kicsit hi´ anyos (4 pont) 3, hi´ anyos (1–3 pont) 6, hib´ as 1, nem versenyszer˝ u 2 dolgozat.

P. 5393. Egy m t¨ omeg˝ u ´es egy M = = 3m t¨ omeg˝ u, kicsiny goly´ ohoz fonalakat er˝ os´ıt¨ unk, melyek m´ asik v´eg´et a bal oldali a´bra szerint azonos magass´ agban r¨ ogz´ıtj¨ uk. A goly´ ok k¨ oz´eppontja ekkor a felf¨ uggeszt´es alatt L m´elys´egben van. A kisebb t¨ omeg˝ u goly´ ot felemelj¨ uk u ´gy, hogy a hozz´ a kapcsol´ od´ o fon´ al v´ızszintes legyen (jobb oldali ´abra), majd a goly´ ot elengedj¨ uk. A k´et goly´ o t¨ ok´eletesen rugalmasan ´es egyenesen u ozik. ¨tk¨

1. a ´bra

b) ´Irjuk fel a t¨ ok´eletesen rugalmas u oz´esre az energia- ´es a lend¨ uletmegma¨tk¨ olj¨ uk (´es rad´ as t¨orv´eny´et! Ha az u oz´es ut´ ani sebess´egeket u1 -gyel ´es u2 -vel jel¨ ¨tk¨ a 2. ´ abr´ anak megfelel˝oen az u oz´esi pontt´ol t´ avolod´ o ir´ anyban tekintj¨ uk ezeket ¨tk¨ pozit´ıvnak), a megmarad´ asi t¨ orv´enyek szerint mv2 = M u1 − mu2 ,

a) Az u oz´es el˝ otti pillanatban mekkora egy¨ uttes er˝ ovel terheli a k´et fon´ al ¨tk¨ a felf¨ uggeszt´est? b) Mekkora a terhel´es az u oz´es ut´ ani pillanatban? ¨tk¨ c) Az els˝ o ´es a m´ asodik u oz´es k¨ oz¨ ott mekkora a k´et fon´ al ´ altal bez´ art legna¨tk¨ gyobb sz¨ og? d) A c) esetben mekkora nagys´ ag´ u, ´es milyen ir´ any´ u az egy¨ uttes terhel´es? e) Mekkora sz¨ oget z´ arnak be a fonalak a f¨ ugg˝ olegessel, amikor bek¨ ovetkezik a m´ asodik u oz´es? ¨tk¨ (5 pont) K¨ oz´ episkolai Matematikai ´ es Fizikai Lapok, 2022/6

2. a ´bra

K¨ozli: Zsigri Ferenc, Budapest 371

illetve

1 1 1 mv22 = mu22 + M u21 . 2 2 2

Behelyettes´ıtve az ismert adatokat, majd m-mel egyszer˝ us´ıtve ezt kapjuk:  2gL = 3u1 − u2 , 2gL = u22 + 3u21 .

Innen u2 -t kik¨ usz¨ ob¨ olve a

12u21 − 6u1



2gL = 0

asa: m´asodfok´ u egyenlethez jutunk. Mivel nyilv´an u1 = 0, az egyenlet megold´  gL . u1 = u2 = 2 372

K¨ oz´ episkolai Matematikai ´ es Fizikai Lapok, 2022/6

A k´et goly´ o teh´ at az u oz´es ut´ an azonos nagys´ ag´ u, de ellent´etes ir´ any´ u sebess´eggel ¨tk¨ fog elindulni. aler˝oket ism´et Az u oz´es ut´ ani pillanatban fell´ep˝ o (K1 -vel ´es K2 -vel jel¨olt) fon´ ¨tk¨ Newton II. t¨ orv´eny´eb˝ ol kaphatjuk meg: K2 − mg = m K2 =

gL 1 u22 =m , L 2 L

d) Legyen a fonalakat fesz´ıt˝ o er˝ o a fonalak sz´els˝ o helyzet´eben F1 ´es F2 . Ebben a helyzetben mindk´et goly´ o pillanatnyi sebess´ege nulla, teh´ at a fon´ alir´any´ u (centripet´ alis) gyorsul´ asuk is nulla kell hogy legyen. Ezek szerint mindk´et testre igaz, hogy az ered˝o er˝ o fon´ alir´any´ u komponense nulla, vagyis a fon´ aler˝ o a neh´ezs´egi er˝ o fon´ alir´any´ u komponens´evel egyenl˝o:

3 mg, 2

´es hasonl´ ok´epp:

F1 = M g cos ϕ =

9 3 M g = mg, 4 4

F2 = mg cos ϕ =

3 mg. 4

A felf¨ uggeszt´esi pontban hat´o ered˝ o er˝ o a koszinuszt´etel szerint gL 1 u2 K1 − M g = M 1 = M , L 2 L

F 2 = F12 + F22 − 2F1 F2 cos(180◦ − 2ϕ) =

teh´ at

= K1 =

9 3 M g = mg. 2 2

45 2 2 27 2 2 45 2 2 27 2 2 387 2 2 m g + m g cos(2ϕ) = m g + m g (2 cos2 ϕ − 1) = m g , 8 8 8 8 64

vagyis

Mivel mindk´et fon´ aler˝ o f¨ ugg˝oleges, ez´ert a felf¨ uggeszt´esi pontn´al hat´ o (f¨ ugg˝olegesen lefel´e ir´ anyul´o) terhel´es:

F =

√ 3 43 mg ≈ 2,46 mg. 8

K1 + K2 = 6mg.

Az egy¨ uttes terhel˝oer˝ o ir´ any´ at a szinuszt´etel seg´ıts´eg´evel sz´ am´ıthatjuk ki. A 3. a´br´ ar´ ol leolvashat´o, hogy

Megjegyz´es. Az u oz´es sor´ an mindk´et fon´ aler˝ o nagys´ aga hirtelen v´eges ´ert´ekkel ¨tk¨ megv´ altozik, az ¨ osszeg¨ uk azonban ugyanakkora marad, mint amennyi az u oz´es el˝ ott ¨tk¨ volt.

sin α F2 2 = =√ , sin(180◦ − 2ϕ) F 43 ahonnan

 3 7 ≈ 0,303, vagyis α ≈ 17,6◦ . sin α = 4 43 A felf¨ uggeszt´esi pontn´al hat´ o terhel˝oer˝ o teh´at a v´ızszintessel 90◦ − ϕ + α = 66,2◦ -os sz¨oget z´ ar be. e) A fonalak mozg´asa nem f¨ ugg a rajtuk l´ev˝ o t¨ omegek nagys´ ag´ at´ ol, ez´ert a m´ asodik u oz´es akkor fog bek¨ ovetkezni, amikor mindk´et fon´ al ´epp f¨ ugg˝oleges. ¨tk¨ ´ Isk. ´es Gimn., 11. ´evf.) G´ abriel Tam´ as (Budapesti Fazekas M. Gyak. Alt. dolgozata alapj´an

c) Mivel a k´et test u oz´es ut´ ani kezd˝ o¨tk¨ sebess´eg´enek nagys´ aga megegyezik, valamint minden helyzetben a gyorsul´ asuk is egyforma nagys´ ag´ u, ez´ert azonos id˝ o alatt mindk´et goly´ o ugyanolyan magasra fog eljutni. Az emelked´es h magass´ ag´at az energiamegmarad´asb´ ol k¨ onnyen megkaphatjuk: mgh =

1 m u22 = mgL, 2 L 4

42 dolgozat ´erkezett. Helyes 21 megold´ as. Kicsit hi´ anyos (4 pont) 8, hi´ anyos (1–3 pont) 9, hib´ as 3, nem versenyszer˝ u 1 dolgozat.

vagyis h = L/4. Jel¨ olj¨ uk a fonalaknak a f¨ ugg˝olegessel bez´ art legnagyobb sz¨ og´et ϕ-vel. A 3. a ´br´ ar´ ol leolvashatjuk, hogy L − 14 L 3 cos ϕ = = , L 4

P. 5397. Egy Q = 10−9 C t¨ olt´es˝ u kicsiny testet egy nagy m´eret˝ u, f¨ oldelt f´emlemezt˝ ol d = 10 cm t´ avols´ agban szigetel˝ o´ allv´ anyon r¨ ogz´ıtett¨ unk. a) Mekkora a f´emlemez fel¨ uleti t¨ olt´ess˝ ur˝ us´ege a kicsiny testhez legk¨ ozelebb es˝ o P pontj´ aban? 3. a ´bra

b) Milyen messze van P -t˝ ol az a pont, ahol a f´emlemez fel¨ uleti t¨ olt´ess˝ ur˝ us´ege a maxim´ alis ´ert´eknek egyharmada? (4 pont)

azaz ϕ = 41,4◦ , ´es ´ıgy a k´et fon´ al ´ altal bez´ art legnagyobb sz¨og: 2ϕ = 82,8◦ . K¨ oz´ episkolai Matematikai ´ es Fizikai Lapok, 2022/6

373

374

K¨ ozli: Holics L´ aszl´ o, Budapest K¨ oz´ episkolai Matematikai ´ es Fizikai Lapok, 2022/6

Megold´as. A Q pozit´ıv t¨ olt´es negat´ıv t¨ olt´eseket vonz a f¨oldelt f´emlemezre. A t¨ olt´es ´es a f´emlemez elektromos mez˝ oje, a f´emlemez azon oldal´ an, ahol a Q t¨olt´es is van, ugyanolyan, mintha a lemez m´ asik oldal´ an, att´ ol d t´avols´ agban lenne egy −Q t¨ uk¨ ort¨ olt´es”, ´es nem lenne ott a f´emlemez. ” Tekints¨ unk egy, a P pontt´ ol r t´ avols´ agban l´ev˝ o kicsiny, ΔA nagys´ ag´ u fel¨ uletdarabk´ at a f´emlemez s´ıkj´aban (1. ´ abra). Erre a kis fel¨ uletre alkalmazva a Gauss-f´ele fluxust¨ orv´enyt: σ(r) ΔA = −E(r)ΔA, ε0 ahol E(r) az ered˝ o (a f´emfel¨ uletre mer˝ oleges ir´ any´ u) elektromos t´erer˝ oss´eg nagys´ aga a k´erd´eses pontban, σ(r) pedig az egys´egnyi fel¨ uletre jut´ o elektromos t¨olt´es, vagyis a f´emlemez fel¨ uleti t¨ olt´ess˝ ur˝ us´ege. Eszerint a keresett (negat´ıv) t¨olt´ess˝ ur˝ us´eg: σ(r) = −ε0 E(r).

a) A P pontban σ(r = 0) = −

Q C = 1,59 · 10−8 2 . 2 2πd m

Ez a t¨olt´ess˝ ur˝ us´eg maxim´ alis ´ert´eke. b) Ha a P pontt´ ol x t´ avols´ agban a fel¨ uleti t¨ olt´ess˝ ur˝ us´eg a legnagyobb ´ert´ek harmada, akkor d Q Q =− σ(x) = − 2π (d2 + x2 )3/2 6πd2 teljes¨ ul. Innen a k´erd´eses t´ avols´ ag  x = d 32/3 − 1 = 0,104 m.

D´ ora M´ arton (Budapest, ELTE Ap´ aczai Csere J´ anos Gyak. Gimn., 12. ´evf.)

20 dolgozat ´erkezett. Helyes D´ ora M´ arton, G´ abriel Tam´ as, Schmercz Blanka ´es T´egl´ as Panna megold´ asa. Kicsit hi´ anyos (3 pont) 2, hi´ anyos (1–2 pont) 5, hib´ as 9 dolgozat.

P. 5398. Digit´ alis f´enyk´epez˝ og´epen 35 mm gy´ ujt´ ot´ avols´ ag´ u objekt´ıv tal´ alhat´ o, melynek k¨ ozelpontja 25 cm. A k¨ ozelpont az a szenzort´ ol m´ert legkisebb t´ avols´ ag, ahonnan az objekt´ıv m´eg k´epes f´ okusz´ alni. a) Hogyan v´ altozik meg a k¨ ozelpont t´ avols´ aga, ha az objekt´ıv ´es a f´enyk´epez˝ og´ep k¨ oz´e egy k¨ ozgy˝ ur˝ ut helyez¨ unk, melynek hat´ as´ ara az objekt´ıv 12 mm-rel messzebbre ker¨ ul a szenzort´ ol? b) K´esz´ıts¨ unk egy k¨ ozelpontba helyezett t´ argyr´ ol felv´etelt k¨ ozgy˝ ur˝ uvel ´es an´elk¨ ul. Hogyan ar´ anylik egym´ ashoz ezen k´et k´ep nagys´ aga? 1. a ´bra

2. a ´bra

A k´et t¨ olt´es elektromos t´erer˝ oss´eg´enek nagys´ aga a kis fel¨ uletdarabn´ al: E 1 = E2 =

K¨ ozli: Sz´echenyi G´ abor, Budapest

(5 pont)

Megold´as. Ismerj¨ uk, hogy az objekt´ıv f´okuszt´avols´ aga: f = 3,5 cm, a k¨ ozelpont t´avols´ aga a szenzort´ ol:  = 25 cm ´es a k¨ ozgy˝ ur˝ u vastags´ aga: d = 1,2 cm. a) A feladat sz¨ ovege szerint a k¨ ozelpontban l´ev˝ o t´ argyr´ ol az objekt´ıv m´eg ´eppen k´epes ´eles k´epet alkotni, vagyis a szok´ asos jel¨ ol´esekkel

1 Q , 4πε0 s2

ahol s a vizsg´ alt fel¨ uletdarabk´ anak a Q t¨ olt´est˝ ol, illetve a −Q t¨ uk¨ort¨olt´est˝ ol m´ert t´ avols´ aga (2. ´ abra). Az ered˝o E t´erer˝ oss´eget a t¨ olt´es ´es a t¨ uk¨ ort¨ olt´es elektromos ter´enek szuperpoz´ıci´ oj´ab´ ol kaphatjuk meg. Ennek nagys´aga: |E| = |E 1 + E 2 | = ahol cos α = d/s ´es s = r t´ avols´ agban

1 2Q cos α, 4πε0 s2

 = t + k. A lek´epz´esi t¨orv´enyb˝ ol: 1 1 1 = + . f t −t

Ez t-re n´ezve m´ asodfok´ u egyenlet:

√ d2 + r2 . ´Igy teh´ at a f´emlemez t¨olt´ess˝ ur˝ us´ege: a P pontt´ ol amelynek megold´asa

d Q σ(r) = − . 2π (d2 + r2 )3/2 K¨ oz´ episkolai Matematikai ´ es Fizikai Lapok, 2022/6

t2 − t + f = 0,

t1 = 20,8 cm, 375

376

illetve

t2 = 4,2 cm.

K¨ oz´ episkolai Matematikai ´ es Fizikai Lapok, 2022/6

Eset¨ unkben fizikai ´ertelme a nagyobb (t = t1 ) gy¨ oknek van, hiszen a lek´epez´eskor t > 2f = 7 cm esetben kapunk – a f´enyk´epez˝ og´epekt˝ol elv´arhat´ o m´odon – kicsiny´ıtett k´epet. A k´ept´ avols´ ag: k =  − t1 = 4,2 cm. A k¨ ozgy˝ ur˝ u behelyez´ese ut´ an a k´ept´ avols´ ag a kor´abbi esethez k´epest d-vel n˝ o, o t´ argyt´ avols´ ag: t = 9,9 cm. vagyis k  = k + d = 5,4 cm lesz, ´es az ehhez tartoz´ A keresett k¨ ozelpontt´ avols´ ag teh´at a k¨ ozgy˝ ur˝ u behelyez´ese ut´ an

Mivel az abroncs t¨ omege (´es ´ıgy a tehetetlens´egi nyomat´eka is) elhanyagolhat´ o, a rendszer t¨ omegk¨ oz´eppontja mindig az m t¨ omeg˝ u test pillanatnyi helyzet´en´el, a P pontn´ al lesz. A nehez´ek pontszer˝ unek tekinthet˝ o, ´ıgy a tehetetlens´egi nyomat´eka null´anak vehet˝ o. A forg´omozg´as alapegyenlete szerint az abroncsra ´es a nehez´ekre hat´ o k¨ uls˝ o er˝ ok ered˝o forgat´ onyomat´eka a P pontra: 

 = t + k  = 15,3 cm. b) K¨ ozgy˝ ur˝ u n´elk¨ ul egy T nagys´ ag´ u t´ argy k´ep´enek m´erete: K1 = T kt , k¨ozgy˝ ur˝ uvel pedig K2 = T

k . t

A keresett ar´ any teh´at Innen kapjuk, hogy K2 5,4 · 20,8 k t ≈ 2,7. =  = K1 tk 9,9 · 4,2



M = ΘP β = 0, M = N R sin α − SR(1 + cos α) = 0.

μ0 =

sin α S = ≈ 0,55. N 1 + cos α

T´egl´ as Panna (R´evkom´arom, Selye J´ anos Gimn., 12. ´evf.)

Nemesk´eri D´ aniel (Budapest, ELTE Ap´ aczai Csere J´ anos Gimn., 11. ´evf.)

9 dolgozat ´erkezett. Helyes G´ abriel Tam´ as, Kert´esz Bal´ azs, K¨ urti Gergely, Toronyi Andr´ as ´es T´egl´ as Panna megold´ asa. Hi´ anyos (1–2 pont) 4 dolgozat.

23 dolgozat ´erkezett. Helyes Beke B´ alint, Bencz Benedek, D´ ora M´ arton, Kert´esz Bal´ azs, Nemesk´eri D´ aniel ´es Toronyi Andr´ as megold´ asa. Kicsit hi´ anyos (4 pont) 2, hi´ anyos (1–3 pont) 10, hib´ as 4, nem versenyszer˝ u 1 dolgozat.

P. 5402. Egy R sugar´ u, elhanyagolhat´ o t¨ omeg˝ u, v´ekony hengeres abroncsra egy m t¨ omeg˝ u, pontszer˝ u nehez´eket er˝ os´ıtett¨ unk. Az abroncs az ´ abr´ an l´ athat´ o labilis egyens´ ulyi helyzet´eb˝ ol kimozdul, ´es akkor cs´ uszik meg a talajon, amikor k¨ oz´eppontj´ anak elmozdul´ asa ´eppen R. Mekkora a tapad´ asi s´ url´ od´ asi egy¨ utthat´ o az abroncs ´es a v´ızszintes talaj k¨ oz¨ ott? (5 pont)

Fizik´ab´ ol kit˝ uz¨ott feladatok

K¨ozli: Balogh P´eter, G¨od¨oll˝o Megold´as. Mivel az abroncs tiszt´ an g¨ ord¨ ul, mialatt a talajon megtesz R t´avols´ agot, a geometriai k¨oz´eppontj´anak elmozdul´ asa is ugyanennyi, ´ıgy a sz¨ogelfordul´ asa (l´ asd az ´ abr´ at) α = 1 rad ≈ 57,3◦ . Jel¨ olj¨ uk a talaj a´ltal az abroncsra kifejtett v´ızszintes s´ url´od´asi er˝ ot S-sel, a f¨ ugg˝ oleges nyom´ oer˝ ot pedig N -nel. Tiszta g¨ord¨ ul´es eset´en S  μ0 N

(6 pont)

K¨ ozli: Horv´ ath Norbert, Budapest

G. 785. Felh˝ os id˝oben vagy esik az es˝o, vagy nem. Mit˝ol f¨ ugg, hogy a felh˝oben l´ev˝ o es˝ ocseppek (vagy j´egkrist´ alyok) lepotyognak a gravit´ aci´ o hat´ as´ ara, vagy benne maradnak a felh˝oben? (3 pont) G. 786. Egy decemberi ´es egy j´ uniusi napon, Ecuadorban, d´elben, v´ed˝ oszemu ¨vegben arccal a Nap fel´e fordulunk. Mit l´atunk, merre mozog a Nap az ´egen, jobbra vagy balra? (3 pont)

(ahol μ0 a k´erdezett tapad´ asi s´ url´od´ asi egy¨ utthat´ o), ´es a megcs´ usz´ as pillanat´aban S = μ0 N. K¨ oz´ episkolai Matematikai ´ es Fizikai Lapok, 2022/6

M. 415. A ny´ ar meleg´et m´eg kihaszn´ alva m´erj¨ uk meg, hogy milyen t´ avols´ agra jut el egy talajszinten l´ev˝ o t¨ oml˝ ob˝ ol ind´ıtott v´ızsug´ar a v´ızhozamt´ ol ´es sz¨ og´ all´ ast´ ol f¨ ugg˝oen!

377

G. 787. Internetes kutakod´assal a´llap´ıtsuk meg, hogy mekkora a v´ız eset´eben oz¨ otti h˝ om´ers´eklet-tartom´anyban a legnagyobb sz´ azal´ekos elt´er´es a 0 ◦ C ´es 100 ◦ C k¨ 378

K¨ oz´ episkolai Matematikai ´ es Fizikai Lapok, 2022/6

a k¨ ovetkez˝ o fizikai mennyis´egek vizsg´ alatakor: s˝ ur˝ us´eg, hangsebess´eg, fel¨ uleti fesz¨ ults´eg, fajh˝ o! H´ any Celsius-fokhoz tartozik ezeknek a mennyis´egeknek a legnagyobb ´es legkisebb ´ert´eke? (A sz´azal´ekos elt´er´est mindig a legnagyobb ´ert´ekhez viszony´ıtsuk.) T¨ untess¨ uk fel az adatok forr´ as´ at! (3 pont) G. 788. Egy fi´ u cs´ onakj´ aval a´tevez egy foly´ on a pontosan szemben l´ev˝ o m´ol´ ohoz, majd azonnal megfordul ´es visszaevez a kiindul´ asi pontba. A 288 m sz´eles foly´ o viz´enek sebess´ege 1 m/s, a cs´ onak v´ızhez viszony´ıtott sebess´ege 2,6 m/s. A fi´ u azt is kipr´ ob´ alja, hogy a foly´ on felfel´e tesz meg 288 m´etert, majd a visszautat is ugyan´ ugy evezve teszi meg. Sz´am´ıtsuk ki a cs´ onak k´etf´ele mozg´as´ anak idej´et! (4 pont)

(4 pont)

P. 5418. K´et k¨ ul¨ onb¨ oz˝ o sz¨ ogben, de megegyez˝o kezd˝osebess´eggel elr´ ugott labda oldet. A magasabb p´ aly´ an halad´ o labda k´etszer annyi ideig azonos t´ avols´agban ´ert f¨ rep¨ ult, mint a m´ asik. Hogyan ar´anylik egym´ashoz a k´et p´ alya cs´ ucsmagass´ aga? Milyen sz¨ ogek alatt r´ ugt´ ak el a labd´ at? (4 pont)

P´eldat´ ari feladat nyom´ an

P. 5419. V´ızszintes s´ıkon egyenletesen, 6 m/s nagys´ag´ u sebess´eggel mozg´o pontszer˝ u test gyorsul´as´ anak nagys´ aga a´lland´ o. A test p´aly´ aj´anak A ´es B pontja k¨ oz´e es˝ o u ´tja 1,2-szerese az elmozdul´ asvektor nagys´ ag´anak. Ezt az utat a test 2 m´ asodperc alatt teszi meg. Mekkora a gyorsul´asa? (4 pont)

P´eldat´ ari feladat nyom´ an

P. 5423. Egy hossz´ u, egyenes vezet˝ore mer˝ oleges s´ıkban egy t´eglalap alak´ u dr´ otkeretet helyez¨ unk el. A vezet˝ oh¨oz legk¨ ozelebb, d t´ avols´ agra a keret egyik ´el´enek k¨oz´eppontja tal´ alhat´ o. Vonzza vagy tasz´ıtja egym´ ast a k´et vezet´ek, ha az egyenes vezet˝ oben I1 , oss´eg˝ u´ aram folyik? a vezet˝ o keretben pedig I2 er˝ (4 pont)

P´eldat´ ari feladat nyom´ an

K¨ozli: Holics L´ aszl´ o, Budapest

P. 5420. a) Milyen α sz¨ og eset´en van egyens´ ulyban az ´ abr´ an l´athat´ o rendszer, ha a lejt˝ on nincs s´ url´od´ as? b) Milyen α sz¨ogek eset´en vannak egyens´ ulyban a testek, ha a lejt˝ on a s´ url´od´ asi egy¨ utthat´ o μ = 0,2? c) Mekkora gyorsul´assal ´es merre mozognak a testek, ha α = 35◦ ´es a s´ url´od´ asi egy¨ utthat´ o μ = 0,15? Mekkora ebben az esetben a k´et k¨ ot´eler˝ o ar´ anya? (5 pont)

P. 5422. Egy z´ art, henger alak´ u, L = 40 cm hossz´ us´ ag´ u, h˝ ovezet˝o fal´ u tart´ alyt egy v´ekony dugatty´ u oszt k´et r´eszre, amelyekben ide´ alisnak tekinthet˝ o g´ az tal´alhat´ o. Kezdetben a tart´ aly tengelye f¨ ugg˝oleges, a dugatty´ u pedig egyens´ ulyi ´allapot´ aban ´eppen a tart´ aly fel´en´el helyezkedik el. Ezut´ an a tart´aly szimmetriatengely´et 90◦ -kal lassan elforgatjuk, melynek eredm´enyek´epp a dugatty´ u 10 cm t´ avols´ aggal mozdul el. Mennyivel mozdult volna el a dugatty´ u, ha 90◦ helyett 180◦ -kal forgattuk volna el a tart´ alyt? A h˝ om´ers´eklet mindv´egig a´lland´ o.

K¨ozli: Siposs Andr´ as, Budapest

P. 5424. Az ´ abr´ an l´athat´ o kondenz´ atorrendszert 5 darab n´egyzet alak´ u, A ter¨ ulet˝ u, t¨ oltetlen f´emlemezb˝ol a´ll´ıtottuk ¨ ossze. A lemezek t´avols´ aga , illetve 2, a sz´eleffektusok 2 A miatt elhanyagolhat´ oak. A lemezek k¨ oz¨ ott leveg˝ o, illetve a barna sz´ınnel jel¨ olt t´err´eszekben εr relat´ıv dielektromos a´lland´ oj´ u szigetel˝ o anyag van. A dielektrikum mindk´et esetben az adott helyen l´ev˝ o kondenz´ artorlemezek k¨ oz¨ otti t´erfogat fel´et t¨ olti ki. Mekkora az elrendez´es ered˝o kapacit´ asa?

P. 5421. A tengerfen´eken, 150 m m´elys´egben fekszik egy els¨ ullyedt, f˝oleg ac´elb´ ol k´esz¨ ult, egykoron 1000 tonna v´ızkiszor´ıt´as´ u haj´ o, amelyet szeretn´enek a felsz´ınre hozni. Ennek ´erdek´eben a b´ uv´ arok bolt´ıves r´eszeket alak´ıtanak ki a haj´ oban, amelyek al´ a hossz´ u cs¨oveken kereszt¨ ul a felsz´ınr˝ ol l´egk¨ori leveg˝ ot juttatnak kompresszorok seg´ıts´eg´evel. Legal´ abb mekkora munk´ at kell v´egezni¨ uk a kompresszoroknak, hogy a haj´ o megemelkedjen a tengerfen´ekr˝ ol! Felt´etelezhetj¨ uk, hogy a tenger u. ´es a leveg˝ o is 15 ◦ C h˝om´ers´eklet˝

P. 5425. F´enysug´ ar leveg˝ob˝ ol n = 1,33 t¨ or´esmutat´ oj´ u v´ızbe l´ep ´ at. Mekkora a bees´esi sz¨og, ha a f´enyt¨ or´es sor´ an a f´enysug´ar sebess´eg´enek a hat´ arfel¨ uletre mer˝ oleges komponense nem v´ altozik meg?

(5 pont)

(4 pont)

K¨ oz´ episkolai Matematikai ´ es Fizikai Lapok, 2022/6

K¨ozli: T´ ofalusi P´eter, Debrecen 379

(5 pont)

380

K¨ ozli: Balogh P´eter, G¨ od¨ oll˝ o

P´eldat´ ari feladat nyom´ an K¨ oz´ episkolai Matematikai ´ es Fizikai Lapok, 2022/6

P. 5426. A fotonrak´eta olyan elk´epzelt rak´eta, amelynek hajt´ om˝ uve az u ¨zemanyagot fotonokk´ a alak´ıtja, majd azokat egyir´anyban, p´ arhuzamosan kil¨ovelli. Egy hossz´ ut´ av´ uu ˝rutaz´ as sor´ an a rak´eta nyugalomb´ ol indulva egyenes p´ aly´ an haladva felgyorsul valamekkora sebess´egre, majd a hajt´ om˝ uv´et az ellenkez˝o ir´anyban u ¨zemeltetve az u ´tic´elig f´ekezve meg´ all. Ezalatt a rak´eta t¨omege negyed´ere cs¨okken. Mekkora volt a rak´eta maxim´ alis sebess´ege?

MATHEMATICAL AND PHYSICAL JOURNAL FOR SECONDARY SCHOOLS (Volume 72. No. 6. September 2022)

(A relativisztikus dinamik´ ar´ ol r¨ ovid cikk olvashat´ o a K¨oMaL honlapj´an.∗ ) (6 pont)

K¨ozli: Vigh M´ at´e, Biatorb´ agy

 Beku ober 15. ¨ld´esi hat´arid˝o: 2022. okt´ Elektronikus munkafu zet: https://www.komal.hu/munkafuzet ¨



E¨ otv¨ os-verseny

Az idei E¨ otv¨ os-versenyt 2022. okt´ ober 14-´en, p´enteken d´elut´ an 15h -t´ ol 20h -ig rendezi meg az E¨ otv¨os Lor´ and Fizikai T´arsulat. A versenyen azok a di´ akok vehetnek r´eszt, akik vagy k¨oz´episkolai tanul´ok, vagy a verseny ´ev´eben fejezt´ek be k¨ oz´episkolai tanulm´anyaikat. Nemcsak magyar allampolg´ ´ ars´ ag´ u versenyz˝ ok indulhatnak, hanem Magyarorsz´agon tanul´ o k¨ ulf¨oldi di´ akok, valamint k¨ ulf¨old¨ on tanul´ o, de magyarul ´ert˝ o di´akok is. A megold´ asokat magyar nyelven kell elk´esz´ıteni, a rendelkez´esre a´ll´o id˝o 300 perc. Minden ´ırott vagy nyomtatott seg´edeszk¨ oz haszn´alhat´ o, de hagyom´anyos (nem programozhat´ o) zsebsz´amol´og´epen k´ıv¨ ul minden elektronikus eszk¨oz haszn´alata tilos. El˝ ozetesen jelentkezni nem kell, elegend˝ o egy szem´elyazonoss´ ag igazol´as´ara szolg´al´o okm´ annyal (szem´elyi igazolv´ any, di´akigazolv´ any vagy u ´tlev´el) megjelenni a verseny valamelyik helysz´ın´en. A helysz´ınek ´es a versennyel kapcsolatos minden tov´abbi inform´ aci´ o megtal´ alhat´ o a verseny honlapj´an: http://eik.bme.hu/~vanko/fizika/eotvos.htm. Versenybizotts´ag ∗

https://www.komal.hu/cikkek/cikklista.h.shtml.

K¨ oz´ episkolai Matematikai ´ es Fizikai Lapok, 2022/6

381

Problems in Mathematics New exercises for practice – competition K (see page 351): K. 729. What angle is enclosed by the hands of the tower hall clock 2022 minutes before it strikes midnight? (Based on the idea of K.A. Kozma, Gy˝ or) K. 730. Eight chords are drawn in a circle, such that they have the largest possible number of intersections. Into how many regions will the eight chords divide the disc bounded by the circle? K. 731. A 4 × 6 rectangle is to be covered, without overlaps, with tiles congruent to the L-shape shown in the figure. The L-shaped tiles may be rotated or turned over as needed. Are there at least 36 different arrangements possible? K/C. 732. Four mathematics teachers are all younger than 70 years, and the age of each of them is a prime number of years. How old is the youngest if their average age is 60 and all their ages are different? K/C. 733. What is the area of the smallest rectangle that has an inscribed parallelogram in which one angle is 60◦ , sides are 4 cm and 6 cm long, and two sides lie on two sides of the rectangle? New exercises for practice – competition C (see page 352): Exercises up to grade 10: K/C. 732. See the text at Exercises K. K/C. 733. See the text at Exercises K. Exercises for 1 1 everyone: C. 1728. Find the exact solutions of the equation − 6 x + 2 = {x}. ({x} denotes the fractional part of x, that is, the difference between x and the greatest integer not greater than x.) C. 1729. Semicircles k1 and k2 , respectively, are drawn outside a square ABCD, over the sides BC ´es CD as diameters. The midpoints of the semicircular arcs are E and F , and the midpoints of line segments DE and AF are P and Q, respectively. Show that P lies on diagonal AC and Q lies on diagonal BD of the square. C. 1730. Find all decimal numbers of the form 0.abc where a, b, c are digits, a = 0, and 0.abc = a . (Croatian problem) Exercises upwards of grade 11: C. 1731. The parallel sides a+b+c of a trapezium ABCD are AB > CD. The midline of the trapezium intersects diagonal AC at E and diagonal BD at F . The length of line segment CD is the a) arithmetic b) geometric mean of line segments AB and EF . In which of the two cases will the ratio AB have a larger value? C. 1732. Let U denote the set of prime numbers greater than CD 337 but not greater than 733. How many 4-element subset does U have that contain 467 or 499 as an element? New exercises – competition B (see page 353): B. 5254. Prove that the difference of the squares of any two odd numbers not divisible by 3 is divisible by 24. (3 points) (Journal of Mathematics and Science Didactics, 1943) B. 5255. Vertex A of a triangle ABC is reflected about vertex B, B is reflected about C, and C is reflected about A. The reflections are points C1 , A1 and B1 , respectively. Show that there exists a triangle with sides of lengths AA1 , BB1 and CC1 . (3 points) B. 5256. In the lottery game, five numbers are drawn out of ninety every week. Andrew fills out a single lottery ticket with the same five numbers in each of the 52 weeks of the year. Belle uses a different scheme. She plays only once a year with 52 tickets simultaneously: she fills them out in pairwise different ways. Is it true that both of them have the same chance of having a ticket with five correct numbers? (4 points) B. 5257. In an acute-angled triangle ABC, the heights are AA1 , BB1 , CC1 ,

382

K¨ oz´ episkolai Matematikai ´ es Fizikai Lapok, 2022/6

and the midpoint of side AB is F . A circle k passes through the points F and C1 , and intersects the extensions of line segments A1 C1 and B1 C1 beyond C1 at points P and Q, respectively. Prove that A1 P = B1 Q. (4 points) B. 5258. Is it true that every positive integer has a positive multiple in which the sum of the digits in decimal notation is at most 2022? (5 points) (Proposed by Cs. S´ andor, Budapest) B. 5259. Solve the following simultaneous equations over the set of real numbers: x2 − 3y + 4 = z, y 2 − 3z + 4 = w, o) z 2 − 3w + 4 = x, w2 − 3x + 4 = y. (4 points) (Based on the idea of M. Bencze, Brass´ B. 5260. G and H are points of chord AB of a circle k such that AG = GH = HB = 1. Let F denote the midpoint of one of the arcs AB. The secants F H and F G intersect the circle again at points C and D, respectively. Show that CD = BC 2 . (6 points) (Proposed by Sz. Kocsis, Budapest) B. 5261. Starting Player and Second Player are playing a game on the edges of a complete graph of 100 vertices. They take turns in colouring an edge of the graph that has not been coloured before. In each step, Starting Player colours his edge red, and Second Player colours his edge blue. The game terminates and Starting Player wins if there is a set of four vertices such that all the six connecting edges are red. The game terminates and Second Player wins if there is a set of four vertices such that all the six connecting edges are blue. The game terminates with a draw if there is no such set of four vertices but there remain no further vertices to colour. Who has a winning strategy? (6 points) New problems – competition A (see page 354): A. 830. For H ⊂ Z and n ∈ Z, let hn denote the number of finite subsets of H in which the sum of the elements is n. Does there exist H ⊂ Z, for which 0 ∈ / H, and hn is a (finite) even number for every n ∈ Z? (The sum of the elements of the empty set is 0.) (Submitted by Csongor Beke, Cambridge) A. 831. In triangle ABC let F denote the midpoint of side BC. Let the circle passing through point A and tangent to side BC at point F intersect sides AB and AC at points M and N , respectively. Let line segments CM and BN intersect in point X. Let P be the second point of intersection of the circumcircles of triangles BM X and CN X. Prove that points A, F and P are collinear. A. 832. Let us assume that the number of offsprings for every man can be 0, 1, . . . or n with probabilities p0 , p1 , . . . , pn independently from each other, where p0 + p1 + · · · + pn = 1 and pn = 0. (This is the so called Galton–Watson process.) Which positive integer n and probabilities p0 , p1 , . . . , pn will maximize the probability that the offsprings of a given man go extinct in exactly the tenth generation?

Problems in Physics (see page 378) M. 415. Taking advantage of the warm summer weather, measure how the horizontal range of a jet of water launched from a hose at the ground depends on the water flow and the angular position of the nozzle. G. 785. In cloudy weather, it either rains or it doesn’t. What determines whether the raindrops (or the ice crystals) in a cloud fall off due to gravity or stay in the cloud? G. 786. One day in December and one in June, in Ecuador, at noon, with solar eclipse glasses on, we face to the Sun. What do we see, which way does the Sun move in the sky, to the right or to the left? G. 787. Research the internet and find out in the case of water between the temperature values of 0 ◦ C and 100 ◦ C the largest percentage difference of the following quantities: density, speed of sound, surface tension, and specific heat. To what temperature values (in Celsius degree) do the maximum and minimum values of these quantities belong? (Always relate the difference in percent to the maximum value.) Also indicate the sources of your data. G. 788. A boy takes a boat across a river to the

K¨ oz´ episkolai Matematikai ´ es Fizikai Lapok, 2022/6

383

pier directly opposite, then immediately turns around and rows back to the starting point. The river is 288 m wide, the water flows at a speed of 1 m/s, and the speed of the boat relative to the water is 2.6 m/s. The boy also tries that he rows upstream 288 m and then rows back to the starting point. Calculate the times for the two movements of the boat. P. 5418. Two balls, kicked at different angles but with the same initial speed, land at the same distance. The ball with the higher trajectory flew twice as long as the other. What is the relationship between the peak heights of the two trajectories? At what angles were the balls kicked? P. 5419. The magnitude of the acceleration of a point-like body moving at a constant speed of 6 m/s in a horizontal plane is constant. The length of the path of the body between points A and B is 1.2 times the magnitude of the displacement vector. It takes 2 seconds for the body to cover this path. What is its acceleration? P. 5420. a) At what angle of α is the system in the figure in equilibrium if there is no friction on the slope? b) For what angles of α are the objects in equilibrium if the coefficient of friction on the slope is μ = 0.2? c) What is the acceleration and the direction of motion of the objects if α = 35◦ and the coefficient of friction is μ = 0.15? What is the ratio of the two tensions in this case? P. 5421. At the bottom of the sea, 150 m below the surface, lies a sunken, mainly steel ship, which once had a 1000 tons displacement. Divers would like to bring this ship the surface. To do this, the divers create arched sections in the ship, into which the atmospheric air from the surface is pumped through long tubes by means of compressors. At least how much work do the compressors have to do in order to raise the ship off the seabed? We can assume that both the sea and the air has a temperature of 15 ◦ C. P. 5422. A closed, cylindrical tank of length L = 40 cm, is made of heat-conducting walls and is divided into two parts with a thin piston. There is some ideal gas in both parts of the tank. Initially, the axis of the tank is vertical and the piston is in equilibrium, exactly at the middle of the tank. Then the tank is slowly turned such that its symmetry axis turns 90◦ , which causes the piston to move a distance of 10 cm. How much would the piston have moved if the tank had been rotated by 180◦ instead of 90◦ ? The temperature is constant all the time. P. 5423. A rectangular frame of wire is placed next to a long, straight conducting wire such that its plane is perpendicular to the wire, as it is shown in the figure. The midpoint of one of the edges of the frame is the closest to the wire, and it is at a distance of d from the wire. Do the two wires attract or repel each other if the straight wire carries a current of magnitude I1 and the conducting frame carries a current of magnitude I2 ? P. 5424. The capacitor system shown in the figure is made of 5 squareshaped uncharged metal plates of area A. The distance between the plates is  or 2, and the edge effects are negligible because 2  A. Between the plates, in the white regions there is air and in the brown regions there is some insulating material of relative dielectric constant εr . In both condensers which contain dielectric, the dielectric material fills half of the area between the plates of the condensers. What is the equivalent capacitance of the arrangement? P. 5425. A ray of light passes from air into water of refractive index n = 1.33. What is the angle of incidence if during refraction the component of the speed of light ray which is perpendicular to the boundary does not change? P. 5426. A photon rocket is an imaginary rocket whose engine converts fuel into photons, which are then ejected into one direction parallel to each other. During a long-duration space mission, the rocket, starting from rest and moving in a straight path, accelerates to some speed, then with its engine running in the opposite direction, it brakes to a stop at the end of its journey. During this time, the mass of the rocket is reduced to one-quarter of its original value. What was the maximum speed of the rocket?

72. ´evfolyam 6. sz´am

K¨oMaL

Budapest, 2022. szeptember

´ ´ FIZIKAI LAPOK ¨ EPISKOLAI KOZ MATEMATIKAI ES ˝ ´ITVE INFORMATIKA ROVATTAL BOV

A 63. Nemzetk¨ozi Matematikai Di´akolimpia feladatainak megold´asa I.

´ ALAP´ITOTTA: ARANY DANIEL 1894-ben 72. ´evfolyam 7. sz´am

Budapest, 2022. okt´ober

´ Megjelenik ´evente 9 sz´amban, janu´art´ol m´ajusig ´es szeptembert˝ol decemberig havonta 64 oldalon. ARA: 1100 Ft

´ TARTALOMJEGYZEK A 63. Nemzetk¨ozi Matematikai Di´akolimpia feladatainak megold´asa I.. . . . . . . . . . . . . . . . . . . . . . . .

386

T´othm´er´esz Lilla: N´egysz´ın-sejt´es II: Hol a hiba Kempe bizony´ıt´as´aban? . . . . . . . . . . . . . . . . . . . . . . .

389

Mikl´os Ildik´o: 61. R´atz L´aszl´o V´andorgy˝ul´es . . . . . .

392

A k¨oz´episkolai tan´arok verseny´enek feladatai . . . . .

393

A 2022. ´evi Beke Man´o Eml´ekd´ıjasok . . . . . . . . . . . . .

396

A 2022. ´evi Rem´enyi-d´ıjasok . . . . . . . . . . . . . . . . . . . . .

397

N´emeth L´aszl´o: Gyakorl´o feladatsor emelt szint˝u matematika ´eretts´egire . . . . . . . . . . . . . . . . . . . . . . . .

397

Erd˝os G´abor: Megold´asv´azlatok a 2022/6. sz´am emelt szint˝u matematika gyakorl´o feladatsor´ahoz . . . . . . . . . . . . . . . . . . . . . . . . . . . . . . . . . . . . . . .

399

Matematika C gyakorlatok megold´asa (1680., 1686.) . . . . . . . . . . . . . . . . . . . . . . . . . . . . . . . . . . . . . . . .

409

Matematika feladatok megold´asa (5193., 5218.) . . .

413

A K pontversenyben kit˝uz¨ott gyakorlatok (734– 738.) . . . . . . . . . . . . . . . . . . . . . . . . . . . . . . . . . . . . . . . . .

415

A C pontversenyben kit˝uz¨ott gyakorlatok (737– 738., 1733–1737.) . . . . . . . . . . . . . . . . . . . . . . . . . . . . .

416

A B pontversenyben kit˝uz¨ott feladatok (5262– 5269.) . . . . . . . . . . . . . . . . . . . . . . . . . . . . . . . . . . . . . . . .

418

Az A pontversenyben kit˝uz¨ott nehezebb feladatok (833–835.) . . . . . . . . . . . . . . . . . . . . . . . . . . . . . . . . . . . .

419

Informatik´ab´ol kit˝uz¨ott feladatok (571–573., 65., 164.) . . . . . . . . . . . . . . . . . . . . . . . . . . . . . . . . . . . . . . . . .

420

Sztrany´ak Gabriella: Ny´ari matematika- ´es fizikat´abor 2022. . . . . . . . . . . . . . . . . . . . . . . . . . . . . . . . . . . .

425

Sz´asz Kriszti´an, Vank´o P´eter: Besz´amol´o a 6. Eur´opai Fizikai Di´akolimpi´ar´ol . . . . . . . . . . . . . . . . . . .

426

Ifj´u Fizikusok Nemzetk¨ozi Versenye . . . . . . . . . . . . . .

431

Fizika gyakorlatok megold´asa (769., 779., 780.) . . .

433

Fizika feladatok megold´asa (5386., 5391., 5394., 5401., 5407.) . . . . . . . . . . . . . . . . . . . . . . . . . . . . . . . . . .

435

Fizik´ab´ol kit˝uz¨ott feladatok (416., 789–792., 5427–5435.) . . . . . . . . . . . . . . . . . . . . . . . . . . . . . . . . . .

442

Problems in Mathematics . . . . . . . . . . . . . . . . . . . . . . . .

445

Problems in Physics . . . . . . . . . . . . . . . . . . . . . . . . . . . . .

447

K¨ oz´ episkolai Matematikai ´ es Fizikai Lapok, 2022/7

´ EVA ´ F˝oszerkeszt˝o: RATKO ´ ¨ Fizikus szerkeszt˝o: GNADIG PETER ´ ILDIKO ´ M˝uszaki szerkeszt˝o: MIKLOS Bor´ıt´o: BURGHARDT ZSUZSA ´ Kiadja: MATFUND ALAP´ITVANY ´ RITA Alap´ıtv´anyi k´epvisel˝o: KOS Felel˝os kiad´o: KATONA GYULA Nyomda: OOK-PRESS Kft. ´ Felel˝os vezet˝o: SZATHMARY ATTILA INDEX: 25 450 ISSN 1215-9247 A matematika bizotts´ag vezet˝oje: ´ HERMANN PETER ´ BALINT, ´ ´ Tagjai: B´IRO GYENES ZOLTAN, ´ ´ KISS HUJTER BALINT, IMOLAY ANDRAS, ´ ´ GEZA, ´ ´ RITA, KOZMA GEZA, KOS KOS ´ MATOLCSI DAVID, ´ KATALIN ABIGEL, ´ ´ PACH PETER ´ ´ V´IGH ¨ ORDI ¨ OK PETERN E, PAL, VIKTOR A fizika bizotts´ag tiszteletbeli eln¨oke: ´ ´ HOLICS LASZL O ´ Tagjai: BARANYAI KLARA, HONYEK GYULA, ´ ´ KRISZTIAN, ´ OLOSZ BALAZS, SZASZ ´ ´ ´ E, ´ VLADAR ´ SZECHENYI GABOR, VIGH MAT ´ KAROLY, WOYNAROVICH FERENC Az informatika bizotts´ag vezet˝oje: ´ ´ SCHMIEDER LASZL O ´ E, ´ FARKAS CSABA, FODOR Tagjai: BUSA MAT ´ ´ ZSOLT, LOCZI LAJOS, SIEGLER GABOR, ´ ´ ´ SZENTE PETER, TOTH TAMAS ´ ANDREA, TASNADI ´ ANIKO ´ Ford´ıt´ok: GROF ´ ´ ¨ Szerkeszt˝os´egi titk´ar: TRASY GYORGYN E A szerkeszt˝os´eg c´ıme: 1117 Budapest, P´azm´any P´eter s´et´any 1/C III. emelet 3.405. Telefon: 372-2850 A lap megrendelhet˝o az Interneten: www.komal.hu/megrendelolap/reszletek.h.shtml. El˝ofizet´esi d´ıj egy ´evre: 9200 Ft K´eziratokat nem ˝orz¨unk meg ´es nem k¨uld¨unk vissza. Minden jog a K¨oMaL tulajdonosai´e. E-mail: [email protected] Internet: http://www.komal.hu This journal can be ordered from the Editorial office: P´azm´any P´eter s´et´any 1/C III. emelet 3.405. 1117–Budapest, Hungary telephone: +36 (1) 372-2850 or on the Postal address H–1518 Budapest 112, P.O.B. 32, Hungary, or on the Internet: www.komal.hu/megrendelolap/reszletek.e.shtml. A Lapban megjelen˝o hirdet´esek tartalm´a´ert felel˝oss´eget nem v´allalunk.

385

A hagyom´ anyoknak megfelel˝oen ebben az ´evben is k¨ oz¨ olj¨ uk a ny´ ari matematikai di´ akolimpia feladatainak a megold´ asait; l´enyeg´eben u ´gy, ahogyan a legillet´ekesebbek, a magyar csapat tagjai le´ırt´ak. K¨ ozrem˝ uk¨ od´es¨ uket k¨ osz¨ onj¨ uk ´es ez´ uton is gratul´ alunk eredm´enyeikhez. A szerkeszt˝ os´eg Els˝ o nap∗ 1. Oslo bankja k´etf´ele t´ıpus´ u ´erm´et bocs´ at ki: alum´ıniumot (jele A) ´es bronzot (jele B). Mariann el˝ ott n alum´ınium´erme ´es n bronz´erme van egy sorban elrendezve valamilyen tetsz˝ oleges kezdeti sorrendben. L´ancnak nevezz¨ uk egym´ ast k¨ ozvetlen¨ ul k¨ ovet˝ o, azonos t´ıpus´ u ´erm´ek tetsz˝ oleges sorozat´ at. R¨ ogz´ıtett k  2n pozit´ıv eg´esz sz´ am mellett Mariann ism´etelten v´egrehajtja a k¨ ovetkez˝ o m˝ uveletet: meghat´ arozza a leghosszabb olyan l´ ancot, amely tartalmazza a balr´ ol sz´ am´ıtott k-adik ´erm´et, ´es az ezen l´ anchoz tartoz´ o¨ osszes ´erm´et ´ atteszi a sor bal sz´el´ere. P´eld´ aul, ha n = 4 ´es ol kiindul´ o folyamat: k = 4, akkor az AABBBABA elrendez´esb˝ AABBBABA → BBBAAABA → AAABBBBA → BBBBAAAA → → BBBBAAAA → · · · .

Hat´ arozzuk meg mindazon, 1  k  2n tulajdons´ ag´ u (n, k) p´ arokat, amelyekre minden kiindul´ asi elrendez´es eset´en lesz olyan pillanat a folyamat sor´ an, hogy a balr´ ol sz´ am´ıtott els˝ o n ´erme mind azonos t´ıpus´ u. Terj´ek Andr´as J´ ozsef megold´asa. El˝ osz¨ or megmutatjuk, hogy ha k  n − 1 an sosem lesz az els˝o vagy k  32 n + 1, akkor van olyan kiindul´asi helyzet, amely ut´ n ´erme azonos t´ıpus´ u. Legyen el˝osz¨ or k  n − 1 ´es legyen az ´erm´ek elrendez´ese a k¨ ovetkez˝ o: n − 1 db A,

1 db B,

1 db A,

n − 1 db B.

Ekkor a kiv´ alasztott l´ anc mindig az els˝ o l´anc lesz, ´ıgy egy l´ep´es nem v´ altoztat semmit, teh´ at sosem lesz az els˝ o n ´erme azonos t´ıpus´ u. Ha pedig k  32 n + 1, tekints¨ uk a k¨ ovetkez˝ o elrendez´est: n n n n db A, db B, db A, db B. 2 2 2 2

Ekkor a kiv´ alasztott l´ anc mindig a legutols´ o lesz (mert k  32 n + 1 > 2 ·  n2  + at az ´ermesor v´ altoz´ asa egy 4 hossz´ u ciklust fog k¨ ovetni ´es sosem lesz +  n2 ). Teh´ n hossz´ u l´anc. ∗

386

A m´ asodik nap feladatainak megold´ as´ at a novemberi sz´ amban k¨ oz¨ olj¨ uk.

K¨ oz´ episkolai Matematikai ´ es Fizikai Lapok, 2022/7

Teh´at xf (x)  1 minden x ∈ R+ eset´en. Ezut´ an bel´atjuk, hogy

Vegy¨ uk ´eszre, hogy ha egy l´ep´es sor´ an olyan l´ancot v´ alasztunk ki, amely nem az els˝ o vagy utols´ o, akkor ezen l´ anc ´ athelyez´es´evel a k´et szomsz´edja egy l´ ancc´ a v´ alik, ´es nem keletkezik u ´j l´ anc, teh´ at cs¨ okken a l´ ancok sz´ ama az ´ermesorozatban. Nem lehet v´egtelen sok olyan l´ep´es, amely cs¨okkenti a l´ ancok sz´ am´at (mert nincs olyan l´ep´es, amelynek sor´an u ´j l´ anc keletkezik), ´ıgy egy id˝o ut´ an b´ armilyen kiindul´ ohelyzetb˝ ol mindig az els˝o vagy az utols´ o l´ancot v´ alasztjuk ki. k = n ´es k = n + 1 eset´en, ha legsz´els˝ o l´ancot v´ alasztunk ki, az csak n hossz´ u lehet, teh´ at ezt az elej´ere helyezve az els˝ o n ´erme azonos t´ıpus´ u. o ut´an mindig az utols´o l´ancot v´ alasztjuk, n + 1 < k < 32 n + 1 eset´en egy id˝ ´es ezt helyezz¨ uk a sorozat legelej´ere. Ilyen l´ep´esekkel el˝obb-ut´ obb minden l´ancot ki kell hogy v´alasszunk (mert egy l´anc, amely h´ atulr´ ol az -edik,  − 1 l´ep´es ut´an az utols´o l´anc lesz). Ez azt jelenti, hogy minden l´ anc hossza legal´ abb 2n − k + 1. at minden l´anc hossza nagyobb, Mivel k < 32 n + 1, ´ıgy 2n − k + 1 > n2 . Teh´ n at egy ´erm´eb˝ ol nem lehet 2 k¨ ul¨ onb¨ oz˝ o l´ anc. Vagyis az eg´esz sorozat egy mint 2 ; teh´ n hossz´ u A ´es egy n hossz´ u B l´ ancb´ ol a´ll. Teh´ at pontosan azok az (n, k) p´ arok felelnek meg, ahol nk
0. ´Igy minden x-n´el kisebb y-ra (ahol y ∈ R+ ) teljes¨ ulnie kell, hogy: xf (y) + yf (x) > 2 ´es f (y) 

1 . y

V´eg¨ ul legyen m´eg z = x − y. Vizsg´ alni fogjuk, hogy mi t¨ ort´enik, amikor z > 0 1 nagyon kis ´ert´eket vesz fel. xf (x − z) + (x − z)f (x) > 2, ekkor f (x − z)  x−z miatt 1 + (x − z) x x−z



1 −ε x



>2

is teljes¨ ul, ebb˝ ol z z + 1 > 2 + + xε, x−z x z z zε + > + xε, x−z x

3 n + 1. 2

zε + 1 +

2. Jel¨ olje R+ a pozit´ıv val´ os sz´ amok halmaz´ at. Hat´ arozzuk meg mindazon + uggv´enyeket, amelyekre minden x ∈ R+ eset´en pontosan egy olyan f : R → R+ f¨ y ∈ R+ l´etezik, hogy xf (y) + yf (x)  2. N´emeth M´arton megold´asa. A felt´etel szimmetri´aja miatt, ha x-hez y az egyetlen megfelel˝o ´ert´ek, akkor y-hoz az x, vagyis a felt´etel aroz egy olyan g :  meghat´  uci´ot, melyre b´armely x ∈ R+ eset´en g g(x) = x, ´es x-hez y = g(x) R+ → R+ invol´ az egyetlen ´ert´ek, mely kiel´eg´ıti a feladat egyenl˝otlens´eg´et. El˝ osz¨ or bel´ atjuk, hogy g(x) = x minden pozit´ıv val´os x-re. Tegy¨ uk fel indirekt, hogy nem, vagyis l´etezik x = y ∈ R+ , melyekre xf (y) + yf (x)  2.

xf (x) + xf (x) > 2

´es

yf (y) + yf (y) > 2,

xf (x) > 1

´es

yf (y) > 1.

2zxε +

z2 > x2 ε + z 2 ε, x

2zxε +

z2 > x2 ε, x

ami azonban nem teljes¨ ul, amikor z > 0 megfelel˝ oen kicsi, hiszen minden tag pozit´ıv, ´es a jobb oldal a´lland´ o marad. Ellentmond´ asra jutottunk, vagyis az indirekt ul. feltev´es nem igaz, teh´ at f (x) ≡ x1 teljes¨ V´eg¨ ul bel´ atjuk, hogy ez eleget tesz a felt´eteleknek. A sz´ amtani-m´ertani k¨ ozepek egyenl˝ otlens´eg´evel: xy x y + 2 = 2. y x xy y

Egyenl˝ os´eg akkor ´es csak akkor teljes¨ ul, amikor xy = x , teh´ at x = y > 0.

Ekkor az ´ert´ekek egy´ertelm˝ us´ege miatt a k¨ ovetkez˝ ok is igazak:

3. Legyen k pozit´ıv eg´esz, ´es legyen S p´ aratlan pr´ımsz´ amoknak egy v´eges halmaza. Bizony´ıtand´ o, hogy (elforgat´ ast´ ol ´es t¨ ukr¨ oz´est˝ ol eltekintve) legfeljebb egyf´elek´eppen lehet az S elemeit egy k¨ or ment´en elrendezni u ´gy, hogy b´ armely k´et szomu legyen valamilyen pozit´ıv eg´esz x-szel. sz´edosnak a szorzata x2 + x + k alak´

Haszn´ aljuk a sz´amtani-m´ertani k¨ ozepek egyenl˝ otlens´eg´et:   √    2  xf (y) + yf (x)  2 xyf (x)f (y) = 2 xf (x) · yf (y) > 2 1 · 1 = 2.

Kov´acs Tam´as megold´asa. El˝ osz¨ or azt fogom bel´atni, hogy a legnagyobb pr´ımnek csak k´etf´ele szomsz´edja lehet, azaz minden lehets´eges elrendez´esben ugyanaz a k´et pr´ım kell hogy mellette ´alljon. Legyen p a legnagyobb pr´ım, ´es az egyik elrendez´esben a szomsz´edai q ´es r; ´es legyen pq = a2 + a + k, ´es pr = b2 + b + k. Mivel

Ezzel ellentmond´ asra jutottunk, vagyis g(x) ≡ x. K¨ oz´ episkolai Matematikai ´ es Fizikai Lapok, 2022/7

1 . x

387

388

K¨ oz´ episkolai Matematikai ´ es Fizikai Lapok, 2022/7

p a legnagyobb sz´ am, p2 > pq, ´es p2 > pr, teh´at p > a, ´es p > b, ´es nyilv´an a = b, mert p = q. Ez azt jelenti, hogy a ´es b is megold´ asai a modulo p testen ´ertelmezett u egyenlet, teh´at legfeljebb x2 + x + k = 0 egyenletnek. Azonban ez egy m´asodfok´ k´et megold´ asa lehet. ´Igy nem lehet p-nek egy harmadik s szomsz´edja, mert akkor az ahhoz tartoz´ o c ´ert´ek is ugyan´ıgy egy b-t˝ ol ´es c-t˝ ol k¨ ul¨onb¨oz˝ o megold´ as lenne. M´ asodszor azt fogom bel´atni, hogy a legnagyobb pr´ım q ´es r szomsz´edaira qr osz¨ or vegy¨ uk ´eszre, hogy felt´eve, hogy q > r, ´es is el˝ o´ all x2 + x + k alakban. Ehhez el˝ ´ıgy a > b, (q − r)p = a2 + a + k − b2 − b − k = (a − b)(a + b + 1). Mivel 0 < b < a < p, az´ert p  a − b, ´es ´ıgy p | a + b + 1. Felhaszn´alva megint, hogy a, b < p, kapjuk, hogy 0 < a + b + 1 < 2p, teh´ at p = a + b + 1. Ekkor q − r = a − b, ´atrendezve a − q = b − r. os´egbe behelyettes´ıtve Nevezz¨ uk ezt a k¨ ul¨ onbs´eget d-nek. A pq = a2 + a + k egyenl˝ el˝osz¨ or a p = a + b + 1, majd az a = q + d, illetve b = q + r kifejez´est: 2

q(q + r + 2d + 1) = (q + d) + q + d + k,

Hol van a hiba a Kempe-f´ele bizony´ıt´asban? Ahogy azt m´ar az el˝oz˝ o r´eszben el´arultuk, a hiba az utols´ o esetben van, m´egpedig akkor, ha a P cs´ ucsot nem tudtuk sem s´ arg´ara, sem z¨ oldre ´ atsz´ınezni. Ekkor az volt a m´ odszer¨ unk, hogy a K1 cs´ ucsot ´atsz´ınezt¨ uk s´arg´ara, a K2 cs´ ucsot pedig z¨ oldre. Azt a´ll´ıtottuk, hogy a K1 s´ arg´ara sz´ınez´esekor az S cs´ ucs nem sz´ınez˝ odik a´t k´ekre. Ez val´oban ´ıgy igaz, hiszen a pirosz¨old k¨or elv´alasztja a K1-et S-t˝ ol. Viszont megt¨ ort´enhet, hogy az ´ atsz´ınez´es sor´ an a piros-s´ arga k¨ or n´eh´ any s´ arga cs´ ucs´ at is ´atsz´ınezz¨ uk, ´ıgy a K1 ´ atsz´ınez´ese ut´ an unik l´etezni. Ez´ert a K2 z¨ oldre sz´ınez´esekor m´ar nem tudja a piros-s´ arga k¨ or megsz˝ a piros-s´ arga k¨ or garant´ alni, hogy a Z cs´ ucs nem sz´ınez˝ odik a´t k´ekre. Hogy ez a probl´ema val´ oban bek¨ ovetkezhet, azt az 4. ´ abra mutatja. A v cs´ ucs feh´er, a t¨obbi cs´ ucs 4 sz´ınnel j´ ol van sz´ınezve. A piros szomsz´edot nem lehet sem s´arg´ara, sem z¨ oldre ´ atsz´ınezni (az ezt tan´ us´ıt´ o piros-s´ arga ´es piros-z¨ old utakat sz¨ urke vonal jel¨ oli).

q 2 + qr + 2qd + q = q 2 + 2qd + d2 + q + d + k, qr = d2 + d + k. Innen v´egtelen lesz´ all´ assal fogjuk befejezni a bizony´ıt´ast. Tekints¨ uk az S halmaz elemsz´ am´at, ´es tegy¨ uk fel, hogy egy n elem˝ u halmazra l´etezik k´et k¨ ul¨onb¨oz˝o konstrukci´ o. Azonban az eddigiek alapj´an tudjuk, hogy a legnagyobb pr´ımnek mindk´et konstrukci´ oban ugyanaz a k´et szomsz´edja, r´ aaad´asul ha ezek szomsz´edosak lenn´enek, teljes´ıten´ek a felt´etelt. Teh´ at ha az S halmazb´ ol kihagyjuk a legnagyobb elem´et, akkor egy n − 1 elem˝ u halmazt kapunk, amire szint´en van konstrukci´ o, hiszen ha az el˝ oz˝ o konstrukci´ ob´ol kivessz¨ uk a legnagyobb pr´ımet, akkor a k´et szomsz´edja egym´ as szomsz´edj´av´ a v´ alik. Ez teljes´ıti a felt´etelt, ´es ´ıgy minden sz´ amnak ugyannnyi szomsz´edja marad. A v´egtelen lesz´ all´ ast folytatva teh´ at tal´alhatn´ ank h´aromelem˝ u S  halmazt is, aminek van k´etf´ele fel´ır´asa. Ez azonban ellentmond´as, hiszen h´ arom sz´amnak minden lehets´eges fel´ır´ asa egym´ as elforgatottja vagy t¨ ukr¨oz´ese.

N´egysz´ın-sejt´es II: Hol a hiba Kempe bizony´ıt´as´aban?∗ Az el˝oz˝ o r´eszben le´ırtuk Kempe hib´ as bizony´ıt´ as´at a n´egysz´ın-sejt´esre, az olvas´ ora b´ızva, hogy megtal´alja a hib´at. Ezt a hib´at Heawood vette ´eszre 11 ´evvel azut´ an, hogy Kempe publik´alta a bizony´ıt´ as´at. Heawood j¨ott r´ a arra is, hogy Kempe ´ervel´ese seg´ıts´eg´evel annyit az´ert be lehet l´atni, hogy tetsz˝oleges (hurok´elmentes) s´ıkgr´ afot j´ol lehet sz´ınezni 5 sz´ınnel. Most mi is le´ırjuk, hogy hol van a hiba Kempe ´ervel´es´eben, ´es hogy hogyan lehet bel´atni az ¨ otsz´ın-t´etelt. ´ ´ Nemzeti Az ´ır´ as az Innov´ aci´ os ´es Technol´ ogiai Miniszt´erium UNKP-20-5 k´ odsz´ am´ u Uj Kiv´ al´ os´ ag Programj´ anak a Nemzeti Kutat´ asi, Fejleszt´esi ´es Innov´ aci´ os Alapb´ ol finansz´ırozott szakmai t´ amogat´ as´ aval k´esz¨ ult.

4. a ´bra

Az 5. ´ abra mutatja, hogy mi t¨ ort´enik, ha a K1 cs´ ucsot s´ arg´ara sz´ınezz¨ uk. Ekkor a piros-s´ arga u ´t egyik cs´ ucsa k´ekre sz´ınez˝ odik, teh´ at elt˝ unik a piros-s´arga ´ val´ u ´t. Es oban, ellen˝ orizhetj¨ uk, hogy ha ebben a sz´ınez´esben most a K2 cs´ ucsot z¨oldre sz´ınezz¨ uk, akkor m´ ar a Z cs´ ucs k´ekre fog sz´ınez˝ odni. Teh´at Kempe m´odszere nem m˝ uk¨ odik minden esetben, azaz nem siker¨ ult bel´atnunk a n´egysz´ın-sejt´est. Persze (ahogy az´ ota Appel ´es Haken megmutatta) minden hurok´elmentes s´ıkgr´ afot j´ol lehet sz´ınezni 4 sz´ınnel. De sajnos Kempe m´odszer´en´el agyaf´ urtabb m´ odszerekre van sz¨ uks´eg¨ unk, hogy egy ilyen 4 sz´ınnel val´o j´ o sz´ınez´est mindig megtal´aljunk. Viszont ha egy konkr´et s´ıkgr´ afot szeretn´enk 4 sz´ınnel j´ol sz´ınezni, akkor ´erdekes m´ odon Kempe m´odszere a gyakorlatban sokszor hat´ekony. Viszonylag ritk´ ak az olyan G − v gr´ afok ´es r´eszleges sz´ınez´esek, amiket nem lehet Kempe m´odszer´evel befejezni. (Nyilv´an ez´ert is tartott 11 ´evig, am´ıg valaki felfedezte a bizony´ıt´asban a hib´ at.) A 6-sz´ın t´etel ´es az 5-sz´ın t´etel



K¨ oz´ episkolai Matematikai ´ es Fizikai Lapok, 2022/7

389

5. a ´bra

Mikor Heawood megtal´ alta a hib´ at Kempe bizony´ıt´ as´ aban, arra is r´ aj¨ ott, hogy az ¨otsz´ın-t´etelt az´ert be lehet bizony´ıtani Kempe m´ odszer´evel. N´ezz¨ uk meg hogy 390

K¨ oz´ episkolai Matematikai ´ es Fizikai Lapok, 2022/7

is megy ez a bizony´ıt´ as, de el˝ otte jegyezz¨ uk meg, hogy a hatsz´ın-t´etelt eg´eszen meglep˝oen egyszer˝ u bebizony´ıtani. 1. t´etel. Tetsz˝ oleges hurok´elmentes s´ıkgr´ af j´ ol sz´ınezhet˝ o 6 sz´ınnel. Bizony´ıt´as. Ism´et el´eg a t´etelt egyszer˝ u s´ıkgr´ afokra bizony´ıtani, mert a t¨obbsz¨ or¨ os ´eleket egyszeres ´elekkel helyettes´ıtve a j´ o sz´ınez´esek ugyanazok maradnak. Egyszer˝ u s´ıkgr´ afokra pontsz´ amra vonatkoz´ o indukci´oval l´atjuk be a t´etelt. Ha legfeljebb 6 pontja van a gr´ afnak, akkor nyilv´ an ki tudjuk sz´ınezni 6 sz´ınnel, hiszen ak´ ar minden pontnak saj´at sz´ın jut. Tegy¨ uk fel, hogy minden legfeljebb k-pont´ u egyszer˝ u s´ıkgr´ afot ki lehet sz´ınezni 6 sz´ınnel. Vegy¨ unk egy (k + 1)-pont´ u egyszer˝ uG s´ıkgr´ afot. Mint az el˝ oz˝ o r´eszben megmutattuk, egy egyszer˝ u s´ıkgr´ afban mindig van olj¨ uk v-vel. A v t¨orl´es´evel legfeljebb 5-fok´ u pont. Vegy¨ unk egy ilyen pontot, ´es jel¨ kapott G − v tov´ abbra is s´ıkgr´ af, de m´ar k cs´ uccsal. Teh´at ezt a gr´ afot az indukci´os feltev´es szerint ki lehet sz´ınezni 6 sz´ınnel. Sz´ınezz¨ uk ki (j´ol) 6 sz´ınnel. Most rajzoljuk vissza a v pontot ´es a r´ a illeszked˝ o ´eleket. Mivel v-nek legfeljebb 5 szomsz´edja van, a szomsz´edai legfeljebb 5 sz´ınt haszn´alnak el. Mindenk´epp marad teh´at egy szabad sz´ın, amit v-nek adhatunk. ´Igy j´ ol sz´ınezt¨ uk G-t 6 sz´ınnel, teh´ at befejezt¨ uk az indukci´os l´ep´est.  Most l´ assuk be, hogy 5 sz´ın is el´eg a j´o sz´ınez´eshez!

Vegy¨ uk ´eszre, hogy az 5 sz´ınnel val´o sz´ınez´es megtal´ al´as´ aban eg´esz sok szabads´agi fokunk volt, p´eld´ aul az utols´o l´ep´esben a s´ arga szomsz´edot feket´ere is sz´ınezhett¨ uk volna, de kezdhett¨ uk volna u ´gy is, hogy a piros cs´ ucsot feket´ere sz´ınezz¨ uk a´t. Teh´at az ember u ´gy ´erzi, hogy 5 sz´ınnel j´o sz´ınez´est tal´ alni nem t´ uls´ agosan neh´ez dolog. 6 sz´ınnel j´ o sz´ınez´est tal´ alni pedig v´egk´epp nagyon egyszer˝ u volt. 4 sz´ınnel j´o sz´ınez´est tal´ alni m´egis olyan neh´ezz´e v´ alik, hogy 120 ´ev kellett a bizony´ıt´ashoz, ´es m´eg ebben a bizony´ıt´ asban is rengeteg gr´ afr´ ol sz´am´ıt´ og´eppel kellett ellen˝ orizni hogy van j´o sz´ınez´ese. A k¨ovetkez˝ o r´eszben egy m´ asik utat mutatunk be, amelyen kereszt¨ ul matematiasik u ´t a sz´ınez´esi polinom kusok a n´egysz´ın-sejt´est pr´ ob´ alt´ ak bebizony´ıtani. Ez a m´ m´odszere volt, ami arr´ ol sz´olt, hogy ahelyett, hogy csak megpr´ ob´ aljuk bizony´ıtani a 4 sz´ınnel val´o j´ o sz´ınez´esek l´etez´es´et, ink´ abb pr´ ob´ aljuk meg megsz´ amolni o˝ket. B´ar v´eg¨ ul nem a sz´ınez´esi polinom seg´ıts´eg´evel lett bebizony´ıtva a n´egysz´ın-t´etel, a sz´ınez´esi polinom kutat´asa nagyon sok ´erdekes dolgot felt´art, ´es valamennyire azt is megmutatja, hogy mi´ert olyan neh´ez a n´egysz´ın-t´etel. Hivatkoz´asok [1] https://web.stonehill.edu/compsci/LC/Four-Color/Four-color.htm [2] Alfred Bray, Kempe’s “proof ” of the four-color theorem, MATH horizons, 2002. https://mathweb.ucsd.edu/~ssam/old/19W-154/kempe.pdf T´ othm´er´esz Lilla ELTE

2. t´etel. Tetsz˝ oleges hurok´elmentes s´ıkgr´ af j´ ol sz´ınezhet˝ o 5 sz´ınnel. Bizony´ıt´as. Ugyan´ ugy, mint a hatsz´ın-t´eteln´el, most is el´eg, ha a t´etelt egyszer˝ u s´ıkgr´ afokra bizony´ıtjuk. Megint pontsz´ amra vonatkoz´ o indukci´ot haszn´alunk. Legfeljebb 5-pont´ u gr´afra a j´ o sz´ınezhet˝ os´eg vil´agos. Tegy¨ uk fel, hogy legfeljebb k-pont´ u gr´ afokra m´ ar tudunk 5 sz´ınnel j´o sz´ınez´est adni. Vegy¨ unk egy (k + 1)-pont´ u ´ G gr´ afot. Ujra vehet¨ unk egy legfeljebb 5-fok´ u v pontot. A v t¨orl´es´evel kapott gr´ afot az indukci´ os feltev´es szerint ki tudjuk j´ol sz´ınezni 5 sz´ınnel. N´ezz¨ uk, hogy ki tudjuk-e sz´ınezni a v cs´ ucsot, miut´ an visszarajzoltuk. Ha a v-nek legfeljebb 4 szomsz´edja van, vagy 5 szomsz´edja van, de ezek a G − v sz´ınez´esekor nem mind k¨ ul¨ onb¨ oz˝ o sz´ınt kaptak, akkor v szomsz´edai legfeljebb 4 sz´ınt haszn´ alnak el, ´es marad a v-nek szabad sz´ın. Probl´ema csak akkor van, ha a vnek pontosan 5 szomsz´edja van, ´es a G − v sz´ınez´es´eben ez az 5 szomsz´ed mind k¨ ul¨ onb¨ oz˝ o sz´ıneket kapott. Megmutatjuk, hogy ilyenkor Kempe m´odszer´evel meg lehet egy kicsit v´altoztatni a G − v sz´ınez´es´et u ´gy, hogy tov´ abbra is j´o sz´ınez´es legyen, de v szomsz´edai m´ar ne legyenek mind k¨ ul¨ onb¨oz˝ o sz´ın˝ uek. Tegy¨ uk fel, hogy v szomsz´edai az ´oramutat´o j´ar´ asa szerint piros, s´arga, k´ek, z¨ old ´es fekete sz´ıneket kaptak. Pr´ ob´ aljuk meg a piros szomsz´edot k´ekre a´tsz´ınezni, majd ennek k´ek szomsz´edait pirosra, stb. Ha v k´ek szomsz´edja nem sz´ınez˝ odik a´t pirosra, akkor v lehet piros. Ha a k´ek szomsz´ed ´atsz´ınez˝ odik, akkor van egy pirosutt egy k¨ort alkot. Ez k´ek u ´t a v piros ´es k´ek szomsz´edja k¨ oz¨ ott, ami v-vel egy¨ a k¨ or elv´alasztja a s´ arga szomsz´edot a z¨ old ´es fekete szomsz´edokt´ ol. Teh´ at ekkor a s´arga szomsz´edot ´atsz´ınezve p´eld´ aul z¨ oldre, a z¨ old szomsz´ed m´ ar nem fog s´arg´ara atsz´ınez˝ ´ odni. Vagyis v lehet s´ arga, ´es k´eszen vagyunk.  K¨ oz´ episkolai Matematikai ´ es Fizikai Lapok, 2022/7

391

61. R´atz L´aszl´ o V´andorgy˝ ul´es Eger, 2022. j´ ulius 5–8.

Eger m´ ar harmadik ´eve k´esz¨ ult megrendezni a v´ andorgy˝ ul´est, de a pand´emia k¨ozbesz´ olt: k´et ´eve elmaradt az esem´eny, tavaly pedig online rendezt´ek meg. ´Igy m´ar nagyon v´artuk, hogy v´egre u ´jra szem´elyesen vehess¨ unk r´eszt rajta, tal´ alkozhassunk a r´eg nem l´atott kedves ismer˝ os¨ okkel. A Bolyai J´ anos Matematikai T´arsulat oktat´ asi bizotts´aga nagyon sokat f´aradozott az´ert, hogy min´el t¨ obb fiatal, p´alyakezd˝o, vagy ´eppen m´eg tan´ arjel¨ olt hallgat´ o pedag´ogus vegyen r´eszt. Ebben nagy seg´ıts´eg lehetett a t´ arsulat ´altal ki´ırt p´ aly´ azat, amely pont a fiatal koroszt´ aly r´eszv´etel´et t´ amogatta. A megnyit´ on hagyom´anyosan a´tadt´ ak a Beke Man´ o-eml´ekd´ıjakat, ´es id´en el˝ osz¨or a Rem´enyi-d´ıjakat is. A Rem´enyi-d´ıj a Graphisoft-d´ıj ut´oda. Eger egy nagyon hangulatos v´aros, ak´ ar a templomait, a v´arat, vagy a Sz´epar a v´andorgy˝ ul´esnek otthont ad´o Eszterh´ azy K´ aroly Katoliasszony-v¨olgyet, ak´ kus Egyetem campus´ at tekintj¨ uk. A pazar szakmai programk´ın´ alat mellett egy´eb kult´ ur´ alis programokban is b˝ ovelkedt¨ unk: eg´esz ny´ aron zajlottak az Agria Ny´ ari ¨ J´at´ekok, illetve pont a v´ andorgy˝ ul´es napjaiban az Egri Bor Unnepe. 392

K¨ oz´ episkolai Matematikai ´ es Fizikai Lapok, 2022/7

´ A v´andorgy˝ ul´esr˝ ol hossz´ u besz´ amol´o olvashat´ o az Erint˝ o Elektronikus Ma1 oad´asok anyagai megtekinthet˝ok tematikai Lapok szeptemberi sz´ am´aban . Az el˝ a v´andorgy˝ ul´es honlapj´an2 . A 2023-as v´ andorgy˝ ul´esre ism´et nagy l´etsz´ amban v´arja a matematikatan´ arokat a Bolyai J´ anos Matematikai T´arsulat. Mikl´ os Ildik´ o A k¨oz´episkolai tan´arok3 verseny´enek feladatai 1. Az RLV 20 ´es RLV 22 alak´ u¨ otjegy˝ u sz´ amok ¨ osszege 123 442. Mennyi az R + L + V ´ert´eke? (A) 10; (B) 11; (C) 12; (D) 13; (E) 14. 2. A 16 384 legnagyobb pr´ımoszt´ oja a 2, mivel 16 384 = 214 . Mennyi a 16 383 legnagyobb pr´ımoszt´ oj´ aban a sz´ amjegyek ¨ osszege? (A) 3; (B) 7; (C) 10; (D) 12; (E) 22. 3. Egy bizonyos napon Gy˝ orben d´elel˝ ott n fokkal melegebb van, mint Szegeden an 16 ´ or´ ara Gy˝ orben a h˝ om´ers´eklet 5 fokkal cs¨ okkent, m´ıg Szegeden (n ∈ N+ ). D´elut´ 3 fokkal n˝ ott. Ekkor a k´et v´ aros h˝ om´ers´eklete 2 fokkal k¨ ul¨ onb¨ ozik egym´ ast´ ol. Mennyi az n ¨ osszes lehets´eges ´ert´ek´enek szorzata? (A) 10; (B) 30; (C) 60; (D) 100; (E) 120. 4. Mennyi az al´ abbi kifejez´es ´ert´eke?

(A) 0;

(B) 1;

(D) 2;

10. Mekkora azon t ´ert´ekek o ¨sszege (0◦  t◦  360◦ , t ∈ R), melyekre a s´ıkbeli koo ordin´ atarendszer (cos 40◦ ; sin 40◦ ), (cos 60◦ ; sin 60◦ ) ´es (cos t◦ ; sin t◦ ) pontjai egy egyenl˝ sz´ ar´ u h´ aromsz¨ og cs´ ucsai lehetnek? (A) 100; (B) 150; (C) 333; (D) 360; (E) 380. 11. Legyen n = 34 · 34 · 63 · 270. Mennyi az n p´ aratlan ´es p´ aros oszt´ oi o ¨sszeg´enek ar´ anya? (A) 1 : 16; (B) 1 : 14; (C) 1 : 8; (D) 1 : 4; (E) 1 : 2. 12. Egy bolha mozog a s´ıkbeli der´eksz¨ og˝ u koordin´ atarendszerben. A (0; 0) pontb´ ol indul, 5 egys´eg hossz´ us´ ag´ u ugr´ asokkal halad, ´es minden ugr´ asa v´eg´en eg´esz koordin´ at´ aj´ u pontokba ´erkezik. Legkevesebb h´ any ugr´ assal juthat el az (1; 0) pontba? (A) 2; (B) 3; (C) 5; (D) 7; (E) nem juthat el. 13. Az eg´esz sz´ amok halmaz´ an h´ any (x; y) megold´ asa van az al´ abbi egyenletnek? x2022 + y 2 = 2y. (A) 1;

(B) 2;

(C) 3;

(D) 4;

(E) v´egtelen sok.

14. Egy kalapba beletesz¨ unk 40 k´ arty´ at, amelyeken egy-egy sz´ am szerepel 1-t˝ ol 40-ig. Viki ´es D´ avid becsukott szemmel kih´ uz a kalapb´ ol egy-egy k´ arty´ at, ´es az azon szerepl˝ o sz´ amot nem mondj´ ak meg egym´ asnak, majd k¨ oz¨ ott¨ uk a k¨ ovetkez˝ o besz´elget´es zajlik le:

log2 160 log2 80 − . log40 2 log20 2 5 (C) 4 ;

¨ k¨ 9. Ot ul¨ onb¨ oz˝ o sz´ın˝ u korongot elhelyez¨ unk egym´ ast´ ol egyenl˝ o t´ avols´ agra egy k¨ or ker¨ ulet´en. El˝ osz¨ or Dia kiv´ alaszt k´et szomsz´edos korongot ´es felcser´eli azokat, majd ezut´ an t˝ ole f¨ uggetlen¨ ul testv´ere, Viki teszi meg ugyanezt. A k´et csere ut´ an mennyi lesz az eredeti hely¨ uket elfoglal´ o korongok sz´ am´ anak v´ arhat´ o (´ atlagos) ´ert´eke? (A) 1,6; (B) 1,8; (C) 2,0; (D) 2,2; (E) 2,4.

Viki: Nem tudom megmondani, hogy kett˝ onk k¨ oz¨ ul ki h´ uzott nagyobb sz´ amot. ´ viszont most m´ D´ avid: En ar ezt tudom. Viki: Pr´ımsz´ amot h´ uzt´ al? D´ avid: Igen. Viki: Nos, akkor a sz´ amodat 100-zal megszorozva, ´es az eredm´enyhez az eny´emet hozz´ aadva n´egyzetsz´ amot kapunk.

(E) log2 5.

5. Az ´ abr´ an l´ athat´ o ABCD ´es BF DE rombuszok hasonl´ ok. TABCD = 24, DAB = 60◦ ´es DE⊥AB. √ Mekkora 3 ; (C) 8; a BF DE rombusz ter¨ u lete? (A) 6; (B) 4 √ (D) 9; (E) 6 3 .

Mennyi a besz´elget´es alapj´ an a k´et kih´ uzott k´ arty´ an lev˝ o sz´ am o ¨sszege? (B) 37; (C) 47; (D) 57; (E) 67.

(A) 27;

a

6. Egy 8 eg´esz sz´ amb´ ol ´ all´ o adathalmaz ´ atlaga, medi´ anja, egyetlen m´ odusza ´es terjedelme is 8. Mekkora lehet legfeljebb az adathalmaz legnagyobb eleme? (A) 11; (B) 12; (C) 13; (D) 14; (E) 15. 7. Andris, Bal´ azs ´es Dani j´ atszanak. Mindannyian t¨ obbsz¨ or egym´ as ut´ an feldobnak egy-egy p´enz´erm´et. Mindegyik¨ uk addig teszi ezt, am´ıg meg nem kapja az els˝ o fejet. Ekkor befejezi a dob´ assorozat´ at. Mennyi a val´ osz´ın˝ us´ege, hogy mindh´ arman ugyanannyiszor 1 1 1 1 1 dobj´ ak fel az ´erm´ej¨ uket? (A) 8 ; (B) 7 ; (C) 6 ; (D) 4 ; (E) 3 .

◦ 8. Egy k¨ orbe ´ırhat´ o ABCD n´egysz¨ ogben CAB = 70◦ , BDA 4, √ = 40 , DA = √ √ 9 2 (B) 6; (C) 2 ; (D) 8 − 2 ; BC = 6. Mekkora az AC ´ atl´ o? (A) 3 + 5 ; (E) 7.

16. Egy ´ep´ıt´esz a v´ızszintes talajon kijel¨ olt ABCDEF szab´ alyos hatsz¨ og cs´ ucsaiban k¨ ul¨ onb¨ oz˝ o magass´ ag´ u, f¨ ugg˝ oleges oszlopokat ´ all´ıt, majd ezek tetej´ere er˝ os´ıt egy talajjal nem p´ arhuzamos hatsz¨ og alak´ u napelemet. A napelem billeg´es n´elk¨ ul illeszkedik a taraga rendre 12, √ 9 ´es t´ ooszlopokra. Ha az A, B, C pontokban elhelyezett oszlopok magass´ 10 m´e√ ter, akkor h´ any m´eter az ooszlop? (A) 9; (B) 6 3 ; √ E pontban elhelyezett tart´ (C) 8 3 ; (D) 17; (E) 12 3 . 17. Az a, b, c, d, e olyan pozit´ıv eg´esz sz´ amok, melyekre a + b + c + d + e = 2022. Legyen M az a + b, b + c, c + d, d + e ¨ osszegek maximuma. Mekkora M legkisebb lehets´eges ´ert´eke? (A) 674; (B) 675; (C) 807; (D) 808; (E) 809. 18. Egy matematikaversenyen az egyes iskol´ ak 3-3 f˝ os csapatokkal vettek r´eszt. Az egy´eni versenyben minden r´esztvev˝ o k¨ ul¨ onb¨ oz˝ o eg´esz pontsz´ amot kapott. A Bolyai Iskola tanul´ oi k¨ oz¨ ul R´eka ´erte el a legjobb eredm´enyt. Az ˝ o pontsz´ ama az ¨ osszes pont´ert´ek medi´ anja volt. R´eka csapatt´ arsai k¨ oz¨ ul Petra 37. helyezett, Vivien pedig 64. helyezett lett. H´ any iskola vett r´eszt a versenyen? (A) 22; (B) 23; (C) 24; (D) 25; (E) 26.

1

https://ematlap.hu/tanora-szakkor-2022-19/1220-sikeres-vandorgyulesegerben. 2 https://www.bolyai.hu/61-ratz-laszlo-vandorgyules-eger. 3 Az ´ altal´ anos iskolai tan´ arok verseny´enek feladatait k´es˝ obb k¨ oz¨ olj¨ uk.

K¨ oz´ episkolai Matematikai ´ es Fizikai Lapok, 2022/7

ul reduk´ alt t¨ ortet k¨ ul¨ onlegesnek, ha a ´es b pozit´ıv 15. Nevezz¨ uk az b nem felt´etlen¨ eg´esz sz´ amok ´es a + b = 15. H´ any k¨ ul¨ onb¨ oz˝ o eg´esz sz´ am ´ırhat´ o fel k´et nem felt´etlen¨ ul k¨ ul¨ onb¨ oz˝ o k¨ ul¨ onleges t¨ ort ¨ osszegek´ent? (A) 9; (B) 10; (C) 11; (D) 12; (E) 13.

393

394

K¨ oz´ episkolai Matematikai ´ es Fizikai Lapok, 2022/7

19. Egy egyfordul´ os k¨ orm´erk˝ oz´eses asztalitenisz torn´ an minden j´ at´ekos 10 m´erk˝ oz´est nyert meg ´es 10-et vesz´ıtett el. D¨ ontetlen m´erk˝ oz´es nem volt. H´ any olyan (A; B; C) tri´ o volt a versenyz˝ ok k¨ oz¨ ott, amelyekn´el A legy˝ ozte B-t, B legy˝ ozte C-t ´es C legy˝ ozte A-t? (A) 385; (B) 665; (C) 945; (D) 1140; (E) 1330. 20. Legyen P ´es Q rendre az ABCD n´egyzet DA ´es AB oldal´ anak egy-egy bels˝ o pontja. A P B ´es QC szakaszok mer˝ olegesek egym´ asra, ´es az R pontban metszik egym´ ast. Mekkora a n´egyzet ter¨ ulete, ha P R = 7 ´es BR = 6? (A) 85; (B) 96; (C) 100; (D) 117; (E) 136. asodfok´ u f¨ uggv´enyekn´el a f˝ oegy¨ utt21. A rendezett alakban megadott f (x) ´es g(x) m´ hat´ ok rendre 2 ´es −2. Mindk´et f¨ uggv´eny grafikonja ´ athalad a (16; 54), (20; 53) pontokon. Mennyi az f (0) + g(0) ¨ osszeg sz´ amjegyeinek ¨ osszege? (A) 8; (B) 10; (C) 12; (D) 14; (E) 16. 22. 10 ember ´ all k¨ or alakban, egym´ ast´ ol egyenl˝ o t´ avols´ agra. K¨ oz¨ ul¨ uk mindenki 3 m´ asikat ismer a t¨ obbiek k¨ oz¨ ul, a k´et szomsz´edj´ at ´es a vele szemk¨ oztit. H´ anyf´elek´eppen oszthatjuk el az embereket 5 p´ arba u ´gy, hogy az egyes p´ arok tagjai ismerj´ek egym´ ast? (A) 8; (B) 11; (C) 12; (D) 13; (E) 18. 23. Az R, L, V pozit´ıv val´ os sz´ amok teljes´ıtik az R+

1 = 4, L

L+

1 = 1, V

28. A 13 sorb´ ol ´es 17 oszlopb´ ol ´ all´ o n´egyzeth´ al´ os t´ abl´ azat minden mez˝ oj´ebe egyegy sz´ amot ´ırunk 1-t˝ ol 221-ig. El˝ osz¨ or soronk´ent fel¨ ulr˝ ol lefel´e haladva balr´ ol jobbra ´ırjuk be egyes´evel n¨ ovekedve a sz´ amokat. A m´ asodik kit¨ olt´es eset´en oszloponk´ent balr´ ol jobbra haladva fel¨ ulr˝ ol lefel´e ´ırjuk be egyes´evel n¨ ovekedve a sz´ amokat. Mennyi azoknak a sz´ amoknak az ¨ osszege, amelyek a k´etf´ele kit¨ olt´es eset´en ugyanabba a mez˝ obe ker¨ ulnek? (A) 222; (B) 333; (C) 444; (D) 555; (E) 666. 29. Egy k¨ ort 20 pont seg´ıts´eg´evel egyenl˝ o hossz´ us´ ag´ u ´ıvekre osztunk fel, majd a pontokhoz az egyikt˝ ol elindulva az o ´ramutat´ o j´ ar´ as´ anak megfelel˝ oen haladva az 1-t˝ ol 20-ig terjed˝ o sz´ amokat rendelj¨ uk. Ezut´ an berajzoljuk a k¨ or azon h´ urjait, amelyek olyan ponamok k¨ ul¨ onbs´ege egy pr´ımsz´ ammal tokat k¨ otnek o ajuk rendelt sz´ ¨ssze, amelyekn´el a hozz´ egyenl˝ o. Mennyi a berajzolt szakaszok a ´ltal meghat´ arozott h´ aromsz¨ ogek sz´ ama? (A) 20; (B) 36; (C) 40; (D) 72; (E) 96. 30. Bal´ azsnak, Be´ anak ´es Daninak az ´ev ugyanazon napj´ an van a sz¨ ulet´esnapja. Dani ma 1 ´eves, Bal´ azs pedig 1 ´evvel id˝ osebb Be´ an´ al. Ma van az els˝ o olyan sz¨ ulet´esnapja Daninak, amikor Bea ´eletkora t¨ obbsz¨ or¨ ose Dani´enak, ´es 100 ´eves kor´ aig ¨ osszesen a mai napot is besz´ am´ıtva 9 ilyen alkalom lesz. Mennyi lesz Bal´ azs ´eletkor´ aban a sz´ amjegyek osszege, amikor ´eletkora legk¨ ozelebb t¨ obbsz¨ or¨ ose lesz Dani´enak? (A) 7; (B) 8; (C) 9; ¨ (D) 10; (E) 11. A feladatsort Fony´ on´e N´emeth Ildik´ o ´es Fony´ o Lajos ´ all´ıtott´ ak ¨ ossze, ´es Kiss G´eza lektor´ alta.

A k¨ oz´episkolai tan´arok verseny´enek eredm´enye

7 1 = V + R 3 2

4

felt´eteleket. Mekkora az RLV szorzat ´ert´eke? (A) 3 ; (B) 1; (C) 3 ; √ 24. Egy g´epi forg´ acsol´ o szersz´ am k´ese egy 50 cm sugar´ u k¨ oralak´ u lemezb˝ ol k´esz¨ ult oly m´ odon, hogy annak egy darabj´ at kiv´ agt´ ak. Form´ aj´ at az ´ abra mutatja. Mekkora a B pontnak O-t´ ol val´ o t´ a√ vol(A) 3 2 ; s´ aga, √ ha AB = 6 cm, BC√= 2 cm ´es ABC = 90◦ ? (B) 2 6 ; (C) 5; (D) 26 ; (E) 6.

(D) 2;

7

(E) 3 .

25. Adottak a s´ıkbeli der´eksz¨ og˝ u koordin´ atarendszerben az O(0; 0) ponton ´ athalad´ oe ´es f egyenesek. Az e egyenes egyenlete 5x − y = 0. A P (−1; 4) pontot el˝ obb az e, majd az f egyenesre t¨ ukr¨ ozve rendre a P1 ´es P2 (4; 1) pontokhoz jutunk. Mi az f egyenes egyenlete? (A) −x + 5y = 0; (B) 2x − 3y = 0; (C) 3x + 2y = 0; (D) x − 3y = 0; (E) 5x − 3y = 0.

26. Egy zs´ akban kezdetben 1 piros ´es 1 k´ek goly´ o tal´ alhat´ o, a k¨ ozel´eben lev˝ o dobozban pedig nagysz´ am´ u ugyanilyen piros ´es k´ek goly´ o van. Vivi n´egyszer hajtja v´egre a k¨ ovetkez˝ o m˝ uveletsort: – v´eletlenszer˝ uen kih´ uz a zs´ akb´ ol egy goly´ ot, – majd kivesz egy ugyanilyen sz´ın˝ u goly´ ot a dobozb´ ol, – v´eg¨ ul mindkett˝ ot visszarakja a zs´ akba. Mennyi a val´ osz´ın˝ us´ege annak, hogy a v´eg´en a zs´ ak mindk´et sz´ın˝ u goly´ ob´ ol azonos 1 1 1 1 1 sz´ am´ ut tartalmaz? (A) 6 ; (B) 5 ; (C) 4 ; (D) 3 ; (E) 2 .

´ ´ ad Gimn.), 1. Koncz Levente (Budapest, Obudai Arp´ 2. Magyar Zsolt (Budapest, Szent Istv´ an Gimn.), 3. Fridrik Rich´ard (Szeged), 4. Baloghn´e Cseh Judit (Szolnok, Varga Katalin Gimn.), ´ Isk., Koll. ´es Ovoda), ´ 5. Kall´ os B´ela (Ny´ıregyh´ aza, Szent Imre Katolikus Gimn., Alt. 6. Moln´ar Istv´an (B´ek´escsaba, Andr´ assy Gyula Gimn. ´es Koll.), V´ertes Judit (Budapest VI. Ker¨ uleti K¨ olcsey Ferenc Gimn.), 8. Balga Attila (Budapest V. Ker¨ uleti E¨ otv¨ os J´ ozsef Gimn.), 9. Tulip´an (jelige).

A 2022. ´evi Beke Man´o Eml´ekd´ıjasok A Beke Man´ o Eml´ekd´ıj Bizotts´ag d¨ ont´ese alapj´ an 2022-ben a d´ıj m´ asodik fokozat´ aban r´eszes¨ ult Burom M´aria (Egri Szil´ agyi Erzs´ebet Gimn. ´es Koll.), ´ G¨oncfalvin´e Cseh Eva (Eger, Eszterh´ azy K´ aroly Egyetem Gyakorl´oisk.), Kiss Zolt´an (P´ecsi Le˝ owey Kl´ara Gimn.), Nagyn´e Lelkes Anik´ o (Kecskem´eti Kod´ aly ´ ´ Zolt´an Enek-zenei Alt. Isk., Gimn., Szakgimn. ´es Alapfok´ u M˝ uv´eszeti Isk.), ´ Sz´ant´ o Zsuzsanna (Budapest, Zugl´oi Herman Ott´o Tud´ ask¨ ozpont Alt. Isk.), ´ Isk. ´es Ovoda) ´ Sz´ekeli Andrea (Kecskem´et, Pet˝ ofi S´ andor Katolikus Alt. ´es T´oth Mariann (Debreceni Fazekas Mih´ aly Gimn.).

27. Egy k¨ orbe szab´ alyos 5, 6, 7, 8 oldal´ u soksz¨ oget ´ırunk u ´gy, hogy b´ armely k´et soksz¨ og cs´ ucsai p´ aronk´ent k¨ ul¨ onb¨ oz˝ oek, ´es nincs olyan pont, amely illeszkedne legal´ abb h´ arom soksz¨ og oldal´ ara. A k¨ or¨ on bel¨ ul h´ any k¨ oz¨ os pontja van az oldalaknak? (A) 52; (B) 56; (C) 60; (D) 64; (E) 68.

K¨ oz´ episkolai Matematikai ´ es Fizikai Lapok, 2022/7

395

A r´eszletes indokl´as a honlapunkon (www.komal.hu) olvashat´ o.

396

K¨ oz´ episkolai Matematikai ´ es Fizikai Lapok, 2022/7

A 2022. ´evi Rem´enyi-d´ıjasok ´ ´ am R´eka (Budapesti Fazekas Mih´aly Gyak. Alt. ´ Ad´ Isk. ´es Gimn.), Arki Tam´as (Gy˝ or, R´evai Mikl´ os Gimn. ´es Koll.), Bal´azs Ferenc (Kiskunf´elegyh´ azi Szent Benedek PG K´et Tan´ıt´ asi Nyelv˝ u Techn. ´es Koll.), Bal´azs Tivadar (Debreceni Fazekas Mih´aly Gimn.), Balga Attila (Budapest V. Ker¨ uleti E¨otv¨os J´ozsef Gimn.), ´ Bar´ati Akos (P´ecs, Ciszterci Rend Nagy Lajos Gimn. ´es Koll.), Cs. Nagy Andr´as (V´ aci SzC Boronkay Gy¨ orgy M˝ uszaki Techn. ´es Gimn.), Ga´al Istv´ann´e (Debreceni ´ Isk. Fazekas Mih´aly Gimn.), Gyenes Zolt´an (Budapesti Fazekas Mih´aly Gyak. Alt. ´es Gimn.), Holl´o G´abor (Budapest, B´ek´ asmegyeri Veres P´eter Gimn.), Horv´ath Orsolya (Budapest, B´ek´ asmegyeri Veres P´eter Gimn.), K¨ot´el Tam´as (Budapest, B´ek´ asmegyeri Veres P´eter Gimn.), L´anyi Veronika (P´ecsi Janus Pannonius Gimn.), ´ Lenger D´aniel (Budapesti Fazekas Mih´aly Gyak. Alt. Isk. ´es Gimn.), Szalahov M´aria Zsuzsanna (P´ecs, Ciszterci Rend Nagy Lajos Gimn. ´es Koll.), Sz´amad´on´e B´ek´essy Szilvia (Budapest, B´ek´ asmegyeri Veres P´eter Gimn.), Szlobodnikn´e Kiss Edit (V´ aci SzC Boronkay Gy¨ orgy M˝ uszaki Techn. ´es Gimn.), Tassy Gergely (Budapest, B´ek´ asmegyeri Veres P´eter Gimn.), T´oth Tibor (Miskolc, F¨oldes Ferenc Gimn.), Varga Attila (Balassagyarmat, Balassi B´ alint Gimn.).

b) Fogalmazzuk meg a k¨ ovetkez˝ o ´ all´ıt´ as megford´ıt´ as´ at: Ha P ´es Q, akkor ” nem R.” c) Tegy¨ uk fel, hogy Ha P ´es Q, akkor nem R.” Igaz-e most a Ha R, akkor ” ” nem P ´es nem Q.” ´ all´ıt´ as? V´ alaszunkat indokoljuk. (13 pont) 4. A 32 lapos magyar k´ artya egyik leg´erdekesebb j´ at´eka az ulti. (A magyar k´arty´ aban a sz´ınek: makk, piros, t¨ ok, z¨ old; sz´ınenk´ent ´ asz, kir´ aly, fels˝ o, als´ o, 10-es, 9-es, 8-as ´es 7-es alkotja a 32 lapot.) Ha pl. B´el´ anak a j´at´ek elej´en leosztott 10 lapj´ab´ol egy ¨ otlapos piros ultija van, ez azt jelenti, hogy n´ ala van a piros hetes ´es m´eg n´egy piros, tov´ abb´ a¨ ot m´asik, nem piros lap. a) Mennyi a val´ osz´ın˝ us´ege, hogy B´el´ anak o¨tlapos piros ultit osztottak a j´at´ek elej´en? K´epzelj¨ uk el, hogy 10 j´ ol megkevert magyar k´artyacsomag van el˝ ott¨ unk ´es mindegyikr˝ ol levessz¨ uk a legfels˝ o lapot. b) Mennyi a val´ osz´ın˝ us´ege annak, hogy a 10 lapb´ ol pontosan 5 piros? c) Ha az el˝ obbi h´ uz´ askor ¨ ot piros lapot h´ uztunk, mennyi a val´ osz´ın˝ us´ege, hogy k¨oz¨ott¨ uk legal´ abb egy hetes van? Minden v´egeredm´enyt n´egy tizedesjegyre kerek´ıtve adjunk meg. (14 pont) II. r´esz

Gyakorl´ o feladatsor emelt szint˝ u matematika ´eretts´egire

5. a) Vizsg´ aljuk meg monotonit´ as ´es korl´atoss´ ag szempontj´ab´ol az an =

I. r´esz (x2 −4x)(2−x)

1. Adott az f (x) = f¨ uggv´eny, amelynek ´ertelmez´esi tartom´anya x−2 Df = R \ {2}, ´es a g(x) = 3|x − 1| − 3 f¨ uggv´eny, amelynek ´ertelmez´esi tartom´anya Dg = R. a) Oldjuk meg az f (x) = g(x) egyenletet.

A K ´es L halmazokat ´ertelmezz¨ uk a k¨ ovetkez˝ ok´eppen: K := x | x ∈ Df ´es

f (x)  0 ; L := x | x ∈ Dg ´es g(x)  0 . b) Adjuk meg a K ∩ L, K \ L ´es (K ∪ L) \ K halmazokat. (12 pont) 2. Egy h´ aromsz¨ og cs´ ucsai: A(−2; −2), B(7; 1), C(5; 5).

a) Mekkora a h´ aromsz¨ og ter¨ ulete? b) Sz´ am´ıtsuk ki a h´ aromsz¨ og s´ ulypontja ´es magass´ agpontja t´avols´ ag´ anak pontos ´ert´ek´et. (12 pont) 3. a) Hat´ arozzuk meg az al´ abbi kijelent´esek logikai ´ert´ek´et, a´ll´ıt´asainkat indokoljuk. A) Minden pozit´ıv eg´esz sz´ amra teljes¨ ul, hogy az ¨osszes pozit´ıv oszt´ oj´anak atlaga kisebb a sz´ ´ am fel´en´el. B) Van olyan n cs´ ucs´ u teljes gr´ af, amelynek h´ aromszor annyi ´ele van, mint az n cs´ ucs´ u fagr´ afnak. K¨ oz´ episkolai Matematikai ´ es Fizikai Lapok, 2022/7

397

n2 + 3n + 2 , n ∈ Z+ 2

sorozatot. a

b) Hat´ arozzuk meg n+1 hat´ ar´ert´ek´et, ha n → +∞. an

2

ol a k¨ ovetkez˝ oket tudjuk: bn = n2 + x · n c) Mennyi az x, ha a bn sorozatr´ + 2 (16 pont) (x ∈ R, n ∈ Z ), valamint (bn+1 − bn ) = bn + bn+1 ? 6. a) Milyen sz´ amjeggyel kezd˝odik a 162022 a t´ızes sz´amrendszerben fel´ırva ´es mi az utols´ o k´et sz´ amjegye? o 43-mal. (16 pont) b) Igazoljuk, hogy 22023 + 1 oszthat´ 7. K´et k¨oz´episkola sakkbajnoks´ agot rendezett u ´gy, hogy a versenyz˝ok el˝osz¨ or a saj´at iskol´ajukon bel¨ ul lebonyol´ıtott h´ aziversenyen vettek r´eszt, melynek sor´ an mindenki mindenkivel egy partit j´ atszott. Ezut´ an ker¨ ult sor az iskol´ak egym´as elleni k¨ uzdelm´ere, ahol minden versenyz˝o a m´asik iskola mindegyik versenyz˝oj´evel egy m´erk˝oz´est v´ıvott. Az egym´ as elleni m´erk˝oz´esek sz´ ama ´eppen annyi volt, mint a k´et h´ aziversenyen ¨ osszesen. a) Iskol´ ank´ent h´ anyan vettek r´eszt a bajnoks´ agban, ha az egyikben k´etszer annyian indultak, mint a m´ asikban? K´et riv´alis asztalitenisz csapat u ´gy d¨ onti el, melyik¨ uk a jobb, hogy kiv´ alasztj´ak saj´ at maguk k¨ oz¨ ul a legjobbat, majd a k´et legjobb megk¨ uzd egym´assal a c´ım´ert. 398

K¨ oz´ episkolai Matematikai ´ es Fizikai Lapok, 2022/7

A csapaton bel¨ uli kiv´ alaszt´ as u ´n. egyenes kies´eses rendszerben t¨ort´enik, amelyben minden fordul´o el˝ ott p´ arokba sorsolj´ ak a r´esztvev˝ oket. A p´ar j´atszik egy m´erk˝oz´est, a gy˝ oztes tov´ abbjut a k¨ ovetkez˝ o fordul´oba, a vesztes kiesik. Akinek a sorsol´askor nem marad p´ar, m´erk˝oz´es n´elk¨ ul jut a k¨ ovetkez˝ o fordul´oba. A pingpongban nincs ¨ d¨ ontetlen, az utols´o m´erk˝ oz´es gy˝ oztese lesz a csapat legjobbja. Osszesen 24 m´erk˝oz´est j´ atszottak, mire kider¨ ult, hogy melyik csapat lett a gy˝ oztes.

illetve:

b) H´ any j´at´ekos nevezett a versenyre az egyik, illetve a m´ asik csapatb´ ol, ha az egyikben o asikban? (16 pont) ¨ttel kevesebben voltak, mint a m´

A logaritmusf¨ uggv´eny monotonit´ asa miatt:

´j ismeretlent, ´ıgy az eredeti egyenletet ilyen alakra hoztuk: Vezess¨ uk be az y = 2x u  2 lg y = lg (y − 2) . y = (y − 2)

8. Egy sz´ amtani sorozat els˝ o¨ ot tagj´anak ¨ osszege 30; a sorozat els˝ o, m´asodik ´es negyedik tagja egy m´ertani sorozat h´ arom szomsz´edos eleme. a) Mennyi a sz´ amtani sorozat els˝ o tagja ´es k¨ ul¨onbs´ege? Egy m´ertani sorozat tagjaira fenn´all, hogy a1 + a3 + a5 = 182 ´es a2 + a4 = −60. b) Hat´ arozzuk meg a sorozat els˝ o tagj´ at ´es h´ anyados´ at.

(16 pont)

oget z´ arnak be egym´assal. 9. Az ABCD t´eglalap a´tl´ oi 30◦ -os sz¨ a) Mekkora az AB ´es BC oldal, ha AC = 12 cm? b) Milyen messze van a B cs´ ucs az AC ´ atl´ ot´ ol? Egy KLM N t´eglalap LN ´ atl´ oja a t´eglalapot k´et der´eksz¨og˝ u h´aromsz¨ogre bontja. A KLN h´aromsz¨ og K-b´ ol indul´o magass´ aga, bels˝o sz¨ogfelez˝ oje ´es s´ ulyvonala legyen rendre m, f , s. c) Mekkora sz¨ oget z´ ar be egym´ assal az LN ´ atl´ o ´es a KL oldal, ha 3f 2 = 2ms? (16 pont) N´emeth L´aszl´o Fony´od

Megold´asv´azlatok a 2022/6. sz´am emelt szint˝ u matematika gyakorl´o feladatsor´ahoz I. r´esz 1. Oldjuk meg a k¨ ovetkez˝ o egyenleteket a val´ os sz´ amok halmaz´ an: a) x · (1 − lg 5) = 2 · lg(2x − 2), b) 1 + 2 · cos2 x = sin(2x).

(7 pont) (6 pont)

Megold´as. a) Mivel a logaritmusf¨ uggv´eny ´ertelmez´esi tartom´anya a pozit´ıv val´ os sz´amok halmaza, ez´ert 2x − 2 > 0, azaz (1)

2x > 2,

vagyis x > 1.

x · (1 − lg 5) = x · (lg 10 − lg 5) = x · lg K¨ oz´ episkolai Matematikai ´ es Fizikai Lapok, 2022/7

10 5



2

y 2 − 5y + 4 = 0, A m´asodfok´ u egyenlet gy¨ okei y = 1 ´es y = 4. Mivel (1) miatt y = 2x > 2, ez´ert unk az els˝o eset nem lehets´eges, vagyis y = 2x = 4, ahonnan x = 2. Minden l´ep´es¨ megford´ıthat´ o ´es a kapott gy¨ ok eleme az ´ertelmez´esi tartom´anynak. b) I. megold´ as. Haszn´ aljuk ki, hogy sin2 x + cos2 x = 1, illetve sin(2x) = 2 · sin x · · cos x. Az egyenlet rendez´ese ut´ an a k¨ ovetkez˝ o, u ´gynevezett homog´en egyenletet kapjuk: sin2 x − 2 · sin x · cos x + 3 cos2 x = 0. Ha cos x = 0, akkor az egyenlet alapj´ an sin x = 0, ekkor azonban sin2 x + cos2 x = 0, ami nem lehets´eges, teh´at ebben az esetben nem ad´odik megold´ as. Ezek szerint cos x = 0, ´ıgy az egyenlet mindk´et oldal´ at oszthatjuk cos2 x-szel: tg2 x − 2 · tg x + 3 = 0. A m´asodfok´ u egyenlet diszkrimin´ansa negat´ıv, teh´at nincs val´os gy¨ oke, vagyis az eredeti egyenletnek sincs val´ os gy¨ oke. II. megold´ as. Vizsg´aljuk meg az egyenlet k´et oldal´ an ´ all´ o kifejez´esek ´ert´ekk´eszlet´et. Tudjuk, hogy cos2 x  0, ez´ert 1 + 2 · cos2 x  1. Mivel sin(2x)  1, ez´ert az egyenl˝os´eg csak akkor teljes¨ ulhet, ha az egyenlet mindk´et oldal´ an ´ all´ o kifejez´es ´ert´eke egyenl˝o 1-gyel. am. Ha 1 + cos2 x = 1, akkor cos x = 0, azaz x = π2 + kπ, ahol k eg´esz sz´ Ekkor azonban sin(2x) = sin(π + 2kπ) = 0, vagyis ha az egyenlet bal oldal´an ´all´o kifejez´es ´ert´eke egyenl˝o 1-gyel, akkor a jobb oldali kifejez´es ´ert´eke nem egyenl˝ o 1-gyel, teh´at az egyenletnek nincs val´os gy¨ oke. 2. Az e egyenes egyenlete 4x − 3y = 15. Mennyi a sugara annak a k¨ ornek, amely ´erinti az e egyenest, tov´ abb´ a az orig´ oban ´erinti az y tengelyt? (12 pont) Megold´as. I. megold´ as. Ha a k¨ or az orig´oban ´erinti az y tengelyt, akkor k¨ oz´eppontja az x tengelyre illeszkedik, koordin´ at´ ai O(u; 0), ´es a k¨ or sugara r = |u|, ez´ert a k¨or egyenlete:

A logaritmus azonoss´ agait felhaszn´ alva:

 2 2 · lg(2x − 2) = lg (2x − 2) .

2

(x − u) + y 2 = u2 ,

= x · lg 2 = lg(2x ),

x2 + y 2 − 2ux = 0. 399

400

K¨ oz´ episkolai Matematikai ´ es Fizikai Lapok, 2022/7

Ha az egyenes ´erinti a k¨ ort, akkor az egyenes ´es a k¨ or egyenlet´eb˝ ol a´ll´o egyenletrendszernek egy megold´ asa van. Az egyenes egyenlet´eb˝ ol y-t kifejezve, a k¨or egyenlet´ebe helyettes´ıtve: 2

x +



2 4x − 5 − 2ux = 0, 3

3. A F´ ab´ ol Vaskarika Kft. log´ oj´ an l´ athat´ o ABCD n´egyzet oldalai 4 cm hossz´ uak, a BE k¨ or´ıv k¨ oz´eppontja a D pont, az ED k¨ or´ıv k¨ oz´eppontja pedig a B pont. A log´ o mind a n´egy r´esz´et pirosra, k´ekre, s´ arg´ ara vagy z¨ oldre festik u ´gy, hogy ha k´et r´esz ker¨ ulet´enek van k¨ oz¨ os szakasza, akkor azok k¨ ul¨ onb¨ oz˝ o sz´ın˝ uek lesznek. a) H´ any n´egyzetcentim´eter a lefestend˝ o ter¨ ulet? (6 pont) b) H´ any k¨ ul¨ onb¨ oz˝ o kifest´es lehets´eges? (8 pont)

25x2 − (18u + 120)x + 225 = 0. Az egyenletnek akkor van egy megold´ asa, ha diszkrimin´ ansa 0: 2

(18u + 120) − 22 500 = 0, |18u + 120| = 150. Ha 18u + 120 = 150, akkor u = 53 , ha pedig 18u + 120 = −150, akkor u = −15. K´et

k¨ or felel meg a feladat felt´eteleinek: az egyiknek a k¨oz´eppontja O1 ( 53 ; 0) ´es sugara

asiknak pedig a k¨ oz´eppontja O2 (−15; 0) ´es sugara r2 = 15. r1 = 53 , a m´ II. megold´ as. Ha a k¨ or az orig´oban ´erinti az y tengelyt, akkor k¨oz´eppontja az x tengelyre illeszkedik, koordin´ at´ ai O(u; 0). A f¨ uggv´enyt´abl´ azatban is megta(x ; y ) pont ´ e s az Ax + By + C = 0 egyenlet˝ u l´ alhat´ o o sszef¨ u gg´ e s alapj´ a n a P ¨ 0 0 0 e egyenes t´ avols´ aga ´ıgy sz´ am´ıthat´ o ki:    Ax0 + By0 + C  .  d(P0 ; e) =  √ A2 + B 2 

tBED

A lefestend˝o ter¨ ulet teh´ at 27,65 cm2 .

Az O(u; 0) pont egyenl˝o t´ avols´ agra van a 4x − 3y − 15 = 0 egyenlet˝ u egyenest˝ol ´es az y tengelyt˝ ol:    4u − 15      = |u|,   2 42 + (−3) |4u − 15| = |u|. 5

Ha 4u − 15 = −5u, akkor u = 53 , ha pedig 4u − 15 = 5u, akkor u = −15. K´et k¨or

felel meg a feladat felt´eteleinek: az egyiknek a k¨ oz´eppontja O1 ( 53 ; 0) ´es a sugara

b) A fest´esre minimum k´et sz´ınt fel kell haszn´ alni. Bontsuk sz´et az eseteket annak megfelel˝oen, hogy h´ any sz´ınt haszn´ alunk. Ha a sz´ınek sz´ama kett˝ o, akkor az ABC r´esz sz´ıne 4-f´ele lehet, ugyanilyen sz´ınnel viszont csak a CED r´esz sz´ınezhet˝ o. Az ACD r´esz sz´ın´ere 3 lehet˝ os´eg maradt, a BEC r´esz sz´ıne pedig ezzel megegyez˝o lesz. Ebben az esetben teh´at a lehet˝os´egek sz´ ama 4 · 3 = 12. Ha a sz´ınek sz´ama h´arom, akkor az egyik sz´ınt nem fogjuk haszn´ alni, ezt 4-f´elek´eppen v´ alaszthatjuk ki, egy m´asik sz´ınt viszont k´et r´esz fest´es´ere is haszn´alunk, err˝ ol a sz´ınr˝ ol 3-f´elek´eppen d¨ onthet¨ unk. Azt, hogy melyik k´et r´esz lesz egyforma, az el˝ oz˝ o esetn´el l´ atottak miatt 2-f´elek´eppen d¨ onthetj¨ uk el, mint ahogy az u os´eg van a megmaradt k´et sz´ın felhasz¨resen maradt r´eszek fest´es´ere is 2 lehet˝ as´aval. A lehet˝os´egek sz´ ama teh´ at: 4 · 3 · 2 · 2 = 48. n´al´ V´eg¨ ul, ha mind a n´egy sz´ınt felhaszn´ aljuk, akkor a lehet˝ os´egek sz´ ama 4! = 24.

asiknak pedig a k¨ oz´eppontja O2 (−15; 0) ´es a sugara r2 = 15. r1 = 53 , a m´

A lehets´eges kifest´esek sz´ ama ¨ osszesen 12 + 48 + 24 = 84.

Megjegyz´es. Egy, a m´ asodik megold´ ast´ ol nem l´enyegesen k¨ ul¨ onb¨ oz˝ o harmadik megold´ as gondolatmenete: a keresett k¨ or¨ ok k¨ oz´eppontjai illeszkednek az e egyenes ´es az y tengely ´ altal meghat´ arozott valamely sz¨ og sz¨ ogfelez˝ oj´ere. A sz¨ ogfelez˝ ok egyenlete fel´ırhat´ o t¨ obbf´ele m´ odszerrel, p´eld´ aul a m´ asodik megold´ asban haszn´ alt k´eplet seg´ıts´eg´evel. A k´et sz¨ ogfelez˝ o egyenlete: 3x − y = 5 ´es x + 3y = −15, a k¨ or¨ ok k¨ oz´eppontj´ at a sz¨ ogfelez˝ ok ´es az x tengely metsz´espontjai adj´ ak. K¨ oz´ episkolai Matematikai ´ es Fizikai Lapok, 2022/7

Megold´as. a) A k¨ or´ıvek at√ sugara a n´egyzet BD ´ l´ oja: BE = DE = BD = 4 2 . Ez azt is jelenti, hogy a BED h´ aromsz¨ og szab´alyos, ez´ert bels˝ o sz¨ ogei 60 fokosak. A BD ´ atl´ o az alakzatot k´et r´eszre osztja, ´ıgy az alakzat ter¨ ulete a r´eszek ter¨ ulet´enek o ¨sszege. Az egyik r´esz az ABD der´eksz¨ og˝ u h´ aromsz¨ og: tABD = 4·4 = 8 cm2 . Hat´ arozzuk meg a m´ asik r´esz ter¨ ulet´et. 2 oz´epponti sz¨ oHa a B k¨ oz´eppont´ u, BD sugar´ u, 60◦ k¨ g˝ u k¨ orcikk ter¨ ulet´ehez hozz´ aadjuk a D k¨ oz´eppont´ u, oz´epponti sz¨ og˝ u k¨ orcikk ter¨ ulet´et, akDB sugar´ u, 60◦ k¨ kor a BED h´ aromsz¨ og ter¨ ulet´et k´etszer sz´ amoltuk, ´ıgy ezt a kapott ¨ osszegb˝ ol ki kell vonni:  √ 2 √  √ 2 √ 4 2 · 3 4 2 ·π 32 − = π − 8 3 ≈ 19,65 cm2 . =2· 6 4 3

401

4. Az iskolai focicsapat edz´es´en az edz˝ o megk´erdezett minden jelenl´ev˝ o di´ akot, hogy h´ any oszt´ alyt´ arsuk tagja a csapatnak. Ketten 1-et, hatan 2-t, egyvalaki 3-at, oten 4-et ´es h´ arman 5-¨ ot v´ alaszoltak a k´erd´esre. ¨ a) Mennyi az elhangzott v´ alaszok m´ odusza, medi´ anja, ´ atlaga ´es terjedelme? (7 pont) 402

K¨ oz´ episkolai Matematikai ´ es Fizikai Lapok, 2022/7

b) Miut´ an a matematika–testnevel´es szakos edz˝ o nagyon elcsod´ alkozott a v´ alaszokat hallva, a csapat tagjai el´ arult´ ak neki, hogy n´eh´ any csapattag matematikaversenyre ment, ez´ert nem tudott elj¨ onni a mai edz´esre. Legal´ abb h´ anyan hi´ anyoztak? (5 pont) Megold´as. a) A m´ odusz 2, mert ezt mondt´ ak a legt¨obben. A medi´an meg´allap´ıt´ as´ahoz rendezz¨ uk az elhangzott v´alaszokat n¨ovekv˝o sorrendbe: 1; 1; 2; 2; 2; 2; 2; 2; 3; 4; 4; 4; 4; 4; 5; 5; 5. A 17 sz´ am k¨ oz¨ ul a k¨ oz´eps˝ o a 9-edik, ami 3, ´ıgy ennyi a medi´ an. A sz´ amok a´tlaga: 2 + 12 + 3 + 20 + 15 52 2·1+6·2+1·3+5·4+3·5 = = ≈ 3,06. 17 17 17 A terjedelem a legnagyobb ´es a legkisebb elhangzott v´alasz k¨ ul¨onbs´ege: 5 − 1 = 4. b) Ha egy oszt´ alyb´ ol n gyerek tagja a csapatnak, akkor az oszt´ aly mindegyik tagj´ anak n − 1 oszt´ alyt´ arsa tagja a csapatnak. Ha teh´ at mindenki ott lenne az edz´esen, akkor n-nel oszthat´ o lenne azoknak a sz´ ama, akik azt mondj´ ak, hogy n − 1 oszt´ alyt´ arsuk tagja a csapatnak. Ekkor azonban m´eg legal´ abb 3 olyan gyerek van, aki 3-at mondana, ´es legal´ abb 3 olyan gyerek, akinek 5 lenne a v´ alasza, teh´ at legal´abb 6 gyerek hi´anyzik az edz´esr˝ ol. Ez az eset lehets´eges, ha egy oszt´alyb´ ol 2, k´et oszt´ alyb´ ol 3-3, egy oszt´ alyb´ ol 4, egy oszt´alyb´ ol 5 ´es egy oszt´ alyb´ ol 6 gyerek lenne a csapat tagja. II. r´esz 5. A tavasszal h´ arom nagy sportversenyt rendeztek a nekeresdfalvi ´ altal´ anos iskol´ aban. Az asztalitenisz bajnoks´ agban 120 gyerek indult. A focilig´ aba 8 csapat nevezett, mindegyik csapatban 9 j´ at´ekos l´epett p´ aly´ ara. Az u ´sz´ oversenyen 81-en ¨ avot. 23 gyerek mind a h´ arom sport´ agban versenyzett. Osszesen 45-en teljes´ıtett´ek a t´ voltak azok, akik egyn´el t¨ obb sz´ amban is indultak. a) H´ anyan vettek r´eszt legal´ abb az egyik sport´ ag verseny´en? (6 pont) A focilig´ aban mindegyik csapat mindegyik m´ asik csapattal egyszer j´ atszott. Az els˝ o h´eten o sszesen 13 m´ e rk˝ o z´ e sre ker¨ u lt sor. ¨ b) Bizony´ıtsuk be, hogy volt olyan csapat, amely az els˝ o h´eten legal´ abb n´egyszer l´epett p´ aly´ ara. (4 pont) Az asztalitenisz bajnoks´ agban n´egy olyan gyerek jutott be a legjobb nyolc k¨ oz´e, aki a Futrinka utc´ aban lakik. A versenyz˝ okb˝ ol sorsol´ assal n´egy p´ art alkottak, akik megk¨ uzdhettek egym´ assal a legjobb n´egy k¨ oz´e jut´ as´ert. c) Mennyi a val´ osz´ın˝ us´ege, hogy nem volt olyan p´ ar, ahol mindk´et gyerek a Futrinka utc´ aban lakik? (6 pont) Megold´as. a) Ha ¨osszeadjuk a h´ arom sport´ agban indul´ok sz´am´at, akkor 45 tanul´ ot legal´abb k´etszer, 23 tanul´ot pedig pontosan h´aromszor sz´amoltunk, ´ıgy a legal´abb egy sz´amban indul´ok sz´ama 120 + 8 · 9 + 81 − 45 + 23 = 251 f˝o. K¨ oz´ episkolai Matematikai ´ es Fizikai Lapok, 2022/7

403

b) Indirekt m´ odszerrel v´egezz¨ uk el a bizony´ıt´ast. Tegy¨ uk fel, hogy nincs olyan csapat, amelyik az els˝ o h´eten legal´ abb n´egyszer j´ atszott. Ez azt jelenti, hogy mind a nyolc csapat legfeljebb h´ aromszor l´epett p´ aly´ ara. Mivel mindegyik m´erk˝ oz´esen k´et = 12 csapat l´ep a p´ aly´ ara, ´ıgy ebben az esetben a m´erk˝oz´esek sz´ ama legfeljebb 8·3 2 lett volna. Ellentmond´ ashoz jutottunk, mivel 13 m´erk˝ oz´esre ker¨ ult sor. Indirekt feltev´es¨ unk teh´ at hamisnak bizonyult, vagyis van olyan csapat, amelyik legal´ abb n´egyszer l´epett p´ aly´ ara. (Megjegyz´es. Felh´ıvjuk a figyelmet enn´el a bizony´ıt´ asn´ al egy tipikus ´es hib´ as gondolatmenetre. A bajnoks´ agokat u ´gy szokt´ ak szervezni, hogy egy fordul´ oban mindegyik csapat j´ atszik egy meccset. Mivel nyolc csapat van, ´ıgy mindegyik fordul´ oban n´egy meccset rendeznek. H´ arom fordul´ o eset´en ez 12 meccset jelent. Vagyis a 13. meccs m´ ar a negyedik fordul´ oban ker¨ ul megrendez´esre. Mi a hiba a gondolatmenetben? Az, hogy nem tudjuk, hogy ezt a bajnoks´ agot hogyan bonyol´ıtj´ ak le, ´es azt kell bel´ atnunk, hogy tetsz˝ oleges sorrendben rendezz¨ uk is meg a meccseket, az a ´ll´ıt´ as akkor is igaz lesz, nem csak egy konkr´et lebonyol´ıt´ asi forma eset´en.)

c) I. megold´ as. K´epzelj¨ uk el, hogyan zajlik a negyedd¨ ont˝ ok sorsol´asa. Az els˝ o p´ar kisorsol´asakor a nyolc j´ at´ekos k¨ oz¨ ul h´ uznak ki kett˝ ot, ´es a p´ aros´ıt´ as akkor lesz csak a v´eg´en j´ o, ha a n´egy Futrinka utcai k¨ oz¨ ul ´es a n´egy m´ ashol lak´o k¨ oz¨ ul is egyet h´ uznak, a p´ ar kih´ uz´ as´anak sorrendje nem sz´am´ıt. Hasonl´ oan folytatva a m´asodik ´es a harmadik p´ ar kih´ uz´ asakor, a keresett val´osz´ın˝ us´eg:

4 4 3 3 2 2 · · · 8 ≈ 0,229. p = 1 8 1 · 1 6 1 · 1 4 1 = 2

2

2

35

II. megold´ as. K´epzelj¨ uk el, hogyan zajlik a negyedd¨ ont˝ ok sorsol´asa. Az els˝ o n´ev kih´ uz´ as´an´al m´eg nem d˝ olt el semmi, n´ezz¨ uk a m´asodik h´ uz´ ast. Ha az els˝o kih´ uzott j´at´ekos Futrinka utcai, akkor a lehets´eges ellenfelek k¨ oz¨ ul 3 Futrinka utcai, 4 pedig nem. Ha az els˝o kih´ uzott j´ at´ekos m´ ashol lak´o, akkor a lehets´eges ellenfelek k¨ oz¨ ul 3 m´ashol lak´o, 4 pedig nem. Mivel mindegyik p´arba egy Futrinka utcai ´es egy m´ashol lak´o versenyz˝ onek kell ker¨ ulnie, ez´ert a m´asodik n´ev kih´ uz´ asakor 47 annak a val´osz´ın˝ us´ege, hogy az els˝ o p´ ar kisorsol´asa j´ ol siker¨ ult. Hasonl´ oan gondolkodva, uz´ asakor pedig 23 val´ osz´ın˝ us´eggel a negyedik n´ev kih´ uz´ asakor 35 , a hatodik n´ev kih´ siker¨ ul a feladat felt´eteleinek megfelel˝ oen a sorsol´ as. Mivel ekkor m´ ar csak egy Futrinka utcai ´es egy m´ ashol lak´o marad a negyedik p´arba, ´ıgy az eg´esz sorsol´ as a feladat felt´eteleinek megfelel˝ oen siker¨ ult. A keresett val´osz´ın˝ us´eg: p=

8 4 3 2 · · = ≈ 0,229. 7 5 3 35

6. a) Egy szab´ alyos hatsz¨ og alap´ u egyenes has´ ab alap´elei 6 cm, oldal´elei 5 cm hossz´ uak. Milyen hossz´ u test´ atl´ oi vannak a has´ abnak, ´es melyik fajt´ ab´ ol h´ any darab? (8 pont) b) Legyen pn annak a val´ osz´ın˝ us´ege, hogy egy szab´ alyos n oldal´ u has´ ab cs´ ucsai k¨ oz¨ ul kett˝ ot v´eletlenszer˝ uen kiv´ alasztva, azok egy lap´ atl´ o v´egpontjai lesznek. Hat´ arozzuk meg a k¨ ovetkez˝ o hat´ ar´ert´eket: lim pn .

n→∞

404

(8 pont)

K¨ oz´ episkolai Matematikai ´ es Fizikai Lapok, 2022/7

Megold´as. a) Legyenek a has´ ab alapjai az ABCDEF ´es az A B  C  D E  F  hatsz¨ ogek. Az ABCDEF hatsz¨ og felbonthat´o hat darab egybev´ ag´o szab´ alyos h´aromsz¨ ogre. Ezt felhaszn´ alva az AD ´ atl´ o hossza az alap´el hossz´ anak k´etszerese, vagyis 12 cm. Az AC ´es AE ´ atl´ ok hossza egyenl˝o, hiszen egym´ as t¨ uk¨ork´epei az AD ´atl´ ora n´ezve. Az AC ´ atl´ o hossza kisz´ amolhat´ o koszinuszt´etellel, vagy kihaszn´alva azt, hogy az ABCO rombusz ´atl´ oi mer˝ olegesen felezik egym´ ast. Azt kapjuk, hogy √ olegesek az alapokra, ´ıgy azok minden egyeneAC = AE = 6 3 . Az oldal´elek mer˝ aromsz¨ ogek der´eksz¨og˝ uek. Mivel mindk´et s´ere is, ez´ert az ACC  , ADD , AEE  h´ befog´ojuk hossza ismert, ´ıgy Pitagorasz-t´etellel az ´atfog´ ok, vagyis a has´ ab A cs´ ucsb´ ol indul´o test´ atl´ oinak hossza kisz´ amolhat´ o:  √  √ √ 2   6 3 + 52 = 108 + 25 = 133 ≈ 11,53 cm, AC = AE =  AD = 122 + 52 = 13 cm.

II. megold´ as. B´ armelyik cs´ ucsot is v´alasztjuk ki els˝ok´ent, minden esetben a m´ asodik cs´ ucs kiv´ alaszt´ asa d¨ onti el, hogy lap´ atl´ ot hat´ aroz meg a k´et cs´ ucs vagy sem. A m´asodik cs´ ucs kiv´ alaszt´ as´ara ¨ osszesen 2n − 1 lehet˝ os´eg van. Hat´ arozzuk meg, hogy ezek k¨oz¨ ul h´ any olyan van, amely az els˝ok´ent kiv´ alasztott cs´ ucsb´ ol h´ uzott valamelyik lap´atl´ onak a v´egpontja. Az n oldal´ u alapon n − 3 darab, m´eg az adott pontra illeszked˝ o k´et oldallapon tov´ abbi 1-1 darab lap´ atl´ o h´ uzhat´ o, ´ıgy az els˝ ok´ent kiv´alasztott cs´ ucsb´ ol h´ uzhat´ o lap´ atl´ ok sz´ama n − 3 + 2 = n − 1. Ezen lap´ atl´ oknak az els˝ok´ent kiv´ alasztott pontt´ ol k¨ ul¨ onb¨ oz˝ o v´egpontjai a megfelel˝ o v´ alaszt´ asok m´asodik pontk´ent, ez´ert a keresett val´osz´ın˝ us´eg: n−1 . pn = 2n − 1 A keresett hat´ ar´ert´ek pedig: 1 − n1 n−1 1 = lim = . n→∞ 2n − 1 n→∞ 2 − 1 2

lim pn = lim

n→∞

Szimmetria-okokb´ol az ABCDEF alap mindegyik cs´ ucs´ ab´ol k´et 11,53 cm hossz´ u ´es egy 13 cm hossz´ u test´ atl´ o h´ uzhat´ o, ´ıgy a has´ abnak ¨osszesen 12 darab 11,53 cm hossz´ u ´es 6 darab 13 cm hossz´ u test´ atl´ oja van.

n

7. Vizsg´ aljuk az an = n2 + 4n + 9 sorozatot. a) Bizony´ıtsuk be, hogy a sorozat szigor´ uan monoton n¨ ov˝ o.

(3 pont)

b) Bizony´ıtsuk be, hogy a sorozatnak egyik tagja sem n´egyzetsz´ am. (5 pont) c) H´ anyf´elek´eppen lehet a sorozat els˝ o 100 tagja k¨ oz¨ ul kiv´ alasztani k´et k¨ ul¨ onb¨ oz˝ ot u ´gy, hogy azok ¨ osszege oszthat´ o legyen 5-tel? (8 pont) 2

Megold´as. a) I. megold´ as. Mivel an = n2 + 4n + 9 = (n + 2) + 5, ez´ert a sorozat 2 uggv´eny grafikonj´ ara grafikonja a val´ os sz´amokon ´ertelmezett f (x) = (x + 2) + 5 f¨ illeszked˝ o pontokb´ ol ´ all. Az f f¨ uggv´eny grafikonja T (−2; 5) tengelypont´ u, norm´ al ´all´as´ u parabola, ez´ert az f f¨ uggv´eny szigor´ uan monoton n¨ ov˝ o, ha x > −2, ´ıgy minden n pozit´ıv eg´esz sz´ am eset´en n + 1 > n > −2, ez´ert an+1 = f (n + 1) > f (n) = uan monoton n¨ ov˝ o. = an , vagyis a sorozat szigor´ 2

II. megold´ as. Mivel an+1 = (n + 1) + 4(n + 1) + 9 = n2 + 6n + 14, ez´ert b) I. megold´ as. Sz´ amoljuk ¨ ossze az egy cs´ ucsb´ ol h´ uzhat´ o lap´ atl´ okat. Az n oldal´ u alapon n − 3 darab, m´ıg az adott pontra illeszked˝o k´et oldallapon tov´ abbi 1-1 darab lap´ atl´ o h´ uzhat´ o, ´ıgy az egy cs´ ucsb´ ol h´ uzhat´ o lap´atl´ ok sz´ama n − 3 + 2 = n − 1. Mivel a has´ abnak 2n cs´ ucsa van, ´es mindegyik lap´ atl´ ot k´etszersz´ amoljuk, ´ıgy 2n·(n−1) 2n . Mivel k´et cs´ ucsot ¨osszesen 2 -f´elek´eppen a lap´atl´ ok sz´ama ¨ osszesen 2 lehet kiv´ alasztani, ez´ert a keresett val´osz´ın˝ us´eg: 2n · (n − 1) n · (n − 1) n−1 2 pn = . = =

2n − 1 2n · (2n − 1) 2n 2 2

an+1 − an = (n2 + 6n + 14) − (n2 + 4n + 9) = 2n + 5 > 0 minden n pozit´ıv eg´esz sz´ amra, vagyis an+1 > an , teh´at a sorozat szigor´ uan monoton n¨ov˝o. b) I. megold´ as. Mivel n > 0, ez´ert 2

A sorozat n-edik tagja az n + 2 n´egyzet´en´el nagyobb, ´es az eggyel nagyobb sz´ am, az n + 3 n´egyzet´en´el kisebb, ´ıgy nincs olyan eg´esz sz´ am, amelynek a n´egyzet´evel egyenl˝ o lenne, teh´at nem n´egyzetsz´am. II. megold´ as. Tegy¨ uk fel, hogy an = k 2 , ahol k pozit´ıv eg´esz sz´ am. 2

(n + 2) + 5 = k 2 ,

A keresett hat´ ar´ert´ek pedig:

2

5 = k 2 − (n + 2) ,

1 − n1 n−1 1 = lim lim pn = lim = . n→∞ n→∞ 2n − 1 n→∞ 2 − 1 2

5 = (k + n + 2) · (k − n − 2).

n

K¨ oz´ episkolai Matematikai ´ es Fizikai Lapok, 2022/7

2

(n + 2) = n2 + 4n + 4 < n2 + 4n + 9 < n2 + 6n + 9 = (n + 3) .

405

406

K¨ oz´ episkolai Matematikai ´ es Fizikai Lapok, 2022/7

Mivel az els˝ o t´enyez˝ o pozit´ıv, ´es a szorzat is pozit´ıv, ez´ert a m´asodik t´enyez˝ onek is pozit´ıvnak kell lennie. Ez pedig csak u ´gy lehets´eges, ha k + n + 2 = 5 ´es k − n − 2 = = 1. A k´et egyenletb˝ ol ´ all´ o egyenletrendszernek egy megold´asa van: n = 0 ´es k = 3. Mivel a sorozat defin´ıci´ oja szerint n pozit´ıv eg´esz sz´ am, ´ıgy ez a megold´ as nem megfelel˝o, vagyis a sorozatnak egyik tagja sem n´egyzetsz´am. (Megjegyz´es: sok helyen elfogadott, hogy a sorozatnak legyen nulladik tagja, ebben az esetben egyetlen olyan tagja van a sorozatnak, amely n´egyzetsz´am, hiszen a0 = 9.) c) Ha n = 5h + m alak´ u, akkor   2 2 an = (5h + m + 2) + 5 = 5 · 5h2 + 2hm + 4h + 1 + (m + 2) . 2

ot¨os marad´eka, mint amennyi az (m + 2) ¨ot¨os marad´eka. Ha Teh´at annyi az an ¨ ot¨os marad´eka rendre 4, 4, 1, 0, 1. teh´at az n ¨ ot¨ os marad´eka 0, 1, 2, 3, 4, akkor az an ¨ A sorozat els˝o 100 tagj´ab´ ol ´ıgy 20 darab ¨ ot¨ os marad´eka 0, 40 darab ¨ot¨os marad´eka 1 ´es 40 darab tag ¨ ot¨ os marad´eka 4. K´etf´elek´eppen kaphatunk k´et kiv´ alasztott tag osszegek´ent 5-tel oszthat´ o sz´ amot: vagy k´et olyan sz´ amot adunk ¨ossze, melynek 0 ¨ az ¨ ot¨ os marad´eka, vagy k´et olyat, melyek k¨ oz¨ ul az egyik 1, a m´asik 4 marad´ekot ad 5-tel osztva. Az ¨ osszes eset sz´ ama teh´ at:

20 + 40 · 40 = 190 + 1600 = 1790. 2

8. Egy kis tehersz´ all´ıt´ o haj´ o 100 kilom´eteren 0,1 · v 2 liter g´ azolajat fogyaszt, ha a sebess´ege v km/h. Egy liter g´ azolaj ´ ara 500 forint, a leg´enys´eg fizet´ese pedig or´ ´ ank´ent o ¨sszesen 27 000 forint. a) Milyen sebess´eggel haladjon a haj´ o, hogy a lehet˝ o legolcs´ obban tudja teljes´ıteni az 500 km hossz´ u t´ avot? (10 pont) b) A haj´ o t¨ obbek k¨ oz¨ ott 15 birk´ at sz´ all´ıt. Sajnos, az egyik birk´ at betegen vitt´ek fel a fed´elzetre. Tudjuk, hogy ez a beteg birka b´ armelyik eg´eszs´eges birk´ at 10% val´ osz´ın˝ us´eggel fert˝ ozi meg. Az ´ıgy megfert˝ oz¨ ott birk´ ak m´ ar nem fert˝ oznek tov´ abb. Mennyi a val´ osz´ın˝ us´ege, hogy legfeljebb 2 tov´ abbi birka kapja el a fert˝ oz´est? (6 pont) , akkor a menetid˝ o 500 h. Mivel Megold´as. a) Ha a haj´ o sebess´ege v km h v 2 o az u olts´eg 5 · 0,1 · v · 500 Ft, ez´ert a teljes k¨olts´eget forintban a k¨ovetkez˝ ¨zemanyagk¨ f¨ uggv´eny adja meg (v > 0): 13 500 000 k(v) = 250 · v 2 + = 250 · v 2 + 13 500 000 · v −1 . v A f¨ uggv´enynek akkor lehet minimuma, ha els˝ o deriv´altja 0: k  (v) = 500 · v − 13 500 000 · v −2 = 0,

Mivel k  (v) = 500 + 27 000 000 · v −3 > 0, ha v > 0, ´ıgy a f¨ uggv´enynek v = 30 eset´en t´enyleg minimuma van, teh´ at az ide´alis sebess´eg 30 km/h. b) A 14 m´ asik birka mindegyike 0,1 val´osz´ın˝ us´eggel fert˝ oz˝ odik meg, ´es 0,9 val´ osz´ın˝ us´eggel marad eg´eszs´eges. A fert˝ oz´est elkap´ o birk´ ak sz´ama binomi´ alis eloszl´ ast alkot, ´es azt kell meghat´ aroznunk, hogy a sz´ amuk milyen val´osz´ın˝ us´eggel lesz 0, 1 vagy 2:



14 14 14 13 · 0,9 · 0,1 + · 0,912 · 0,12 ≈ 0,229 + 0,356 + 0,257 ≈ 0,842. 0,9 + 1 2 K¨or¨ ulbel¨ ul 84,2 sz´ azal´ek a val´ osz´ın˝ us´ege annak, hogy legfeljebb 2 u ´jabb birka fert˝ oz˝ odik meg. 9. A hagyom´ anyos, u ´gynevezett ¨ ot¨ os lott´ on a szelv´enyen 1-t˝ ol 90-ig szerepelnek a sz´ amok, ´es ezek k¨ oz¨ ul kell kiv´ alasztani azt az 5 sz´ amot, amit a sorsol´ ason kih´ uznak. a) H´ any olyan sz´ am¨ ot¨ os van, amelyben a sz´ amok sz´ amtani sorozatot alkotnak? (6 pont) b) H´ any olyan sz´ am¨ ot¨ os van, amelyben a sz´ amok m´ertani sorozatot alkotnak? (10 pont) Megold´as. a) Legyen a legkisebb kih´ uzott sz´am a, a sz´ amtani sorozat differenam, ekkor a legnagyobb kih´ uzott sz´ am a + 4d  90, ci´aja pedig a d pozit´ıv eg´esz sz´ teh´ at 4d < 90, ´ıgy d < 22,5. Sz´amoljuk ¨ ossze a lehets´eges eseteket u ´gy, hogy d ´ert´ek´et r¨ogz´ıtj¨ uk, ´es megsz´ amoljuk minden esetben a lehets´eges ´ert´ekeit. Ha d = 22, akkor a  90 − 4d = 90 − 4 · 22 = 2, vagyis a ´ert´eke 2-f´ele lehet. Ha d = 21, akkor a  90 − 4d = 90 − 4 · 21 = 6, vagyis a ´ert´eke 6-f´ele lehet. ´ Eszrevehetj¨ uk, hogy ha d ´ert´ek´et 1-gyel cs¨ okkentj¨ uk, akkor a lehets´eges ´ert´ekeu sz´ amtani sorozatot alkot. inek sz´ama 4-gyel n˝o, vagyis az esetek sz´ama egy 22 tag´ K´erd´es¨ unk ezen tagok ¨ osszege, ehhez hat´ arozzuk meg a 22-edik tagot: ha d = 1, akkor a  90 − 4d = 90 − 4 · 1 = 86, vagyis a ´ert´eke 86-f´ele lehet. Most m´ar elv´egezhetj¨ uk az ¨ osszegz´est, a megfelel˝o sz´ am¨ ot¨ os¨ ok sz´ama: S22 =

b) Legyen a legkisebb kih´ uzott sz´am a, a m´ertani sorozat h´ anyadosa pedig a q pozit´ıv racion´ a lis sz´ a m, ekkor a legnagyobb kih´ u zott sz´ a m a · q 4  90, teh´at √ 4 4 or azt az egyszer˝ ubb esetet, amikor q q  90, ´ıgy q  90 ≈ 3,08. Vizsg´aljuk el˝osz¨ eg´esz sz´ am, ekkor ´ert´eke csak 2 vagy 3 lehet. = 5,625, teh´at a ´ert´eke csak 1, 2, 3, 4, 5 lehet. Ha q = 2, akkor a  90 16 Ha q = 3, akkor a  90 , ami csak akkor teljes¨ ul, ha a = 1. 81

13 500 000 , 500 · v = v2

Eg´esz q eset´en teh´ at ¨ osszesen 6 esetet tal´altunk. Mi a helyzet, ha q = cb , ahol b ´es c relat´ıv pr´ım pozit´ıv eg´esz sz´ amok ´es b > c? Ekkor a legnagyobb sz´ am

v 3 = 27 000, v = 30.

K¨ oz´ episkolai Matematikai ´ es Fizikai Lapok, 2022/7

2 + 86 · 22 = 44 · 22 = 968. 2

a· 407

408

a · b4 b4 = 4  90. 4 c c K¨ oz´ episkolai Matematikai ´ es Fizikai Lapok, 2022/7

  Mivel ez a sz´am is eg´esz, ez´ert c4 oszt´ oja a · b4 -nek. Mivel azonban c4 ; b4 = 1, ez´ert az euklid´eszi lemma miatt c4 oszt´ oja a-nak. Ebb˝ ol az is k¨ovetkezik, hogy c4  a  90, ´ıgy c ´ert´eke csak 2 vagy 3 lehet.

Tekints¨ uk az ´ abr´ at. Az AO szakasz felezi a DAB-et, a BO szakasz pedig felezi az ABC-et, ez´ert OAB = 45◦ , illetve ABO = 30◦ . Ebb˝ ol azonnal k¨ ovetkezik, hogy az ABO h´ aromsz¨ ogben az a´bra α sz¨ og´ere α = = BOA = 105◦ . A DO szakasz felezi a CDA-et, ez´ert ODA = 60◦ , ´es ´ıgy a DAO h´ aromsz¨ ogben β = AOD = 75◦ . Eszerint

Ha c = 2, akkor a a c4 = 16 t¨ obbsz¨ or¨ ose. Ha a  2 · 16, akkor b > c = 2 miatt a · b4 32 · 34 32 · 81 = 162 > 90,  = c4 24 16 vagyis ellentmond´ ashoz jutottunk. Azt kaptuk, hogy a = 16 lehet csak. Ha b = 3, akkor ebb˝ol kapunk is egy megfelel˝ o sz´am¨ot¨ost: 16, 24, 36, 54, 81. Ha b  4, akkor viszont

α + β = 105◦ + 75◦ = 180◦ ,

a · b4 16 · 44  = 16 · 16 = 256 > 90, c4 24 vagyis nincs t¨ obb megfelel˝ o sz´ am¨ ot¨ os, ha c = 2. Mi a helyzet, ha c = 3? Ekkor a  81 ´es b  4 miatt a · b4 81 · 44  = 256 > 90, 4 c 34 vagyis ebben az esetben nem kapunk megfelel˝ o sz´ amokat. Tal´ altunk teh´ at ¨ot olyan sz´ am¨ ot¨ ost, ahol q = 2, egy olyat, ahol q = 32 ´es egy olyat, amelyben q = 3, ´ıgy ¨ osszesen 7 megfelel˝ o sz´ am¨ot¨os van. Erd˝os G´abor Nagykanizsa

ez pedig azt jelenti, hogy a B, O ´es D pontok egy egyenesen vannak, vagy m´ask´ent: a BDA ´es BDC der´eksz¨ og˝ u h´ aromsz¨ ogek k¨ oz¨ os ´ atfog´ oja a BD szakasz, amelyre illeszkedik az ABCD n´egysz¨ og be´ırt k¨ or´enek O k¨ oz´eppontja. A BDA ´es BDC der´eksz¨og˝ u h´aromsz¨ ogek megfelel˝ o sz¨ ogei megegyeznek, tov´ abb´ a a der´eksz¨ oggel szemben lev˝ o BD ´ atfog´ o k¨ oz¨ os benn¨ uk, ez´ert ez a k´et h´ aromsz¨ og egyr´eszt egybev´ ag´ o, m´ asr´eszt a BD egyenes´ere vonatkoz´oan egym´as t¨ uk¨ ork´epei. aromsz¨ og megszerkeszthet˝ o, hiszen AB adott ´es Ennek alapj´an az ABD h´ ogek szerkeszthet˝ok. a rajta fekv˝o 90◦ -os ´es 30◦ -os sz¨ Az A pontnak a BD ´ atl´ o egyenes´ere val´ o t¨ ukr¨ oz´es´evel megkapjuk a n´egysz¨ og hi´ anyz´o C cs´ ucs´ at. A szerkeszt´es a fentiek alapj´ an mindig v´egrehajthat´ o ´es csak egy megold´ as van. Bencsik Bendeg´ uz (Szeged, Radn´ oti Mikl´ os K´ıs´erleti Gimn´azium, 10. ´evf.) ´es m´ asok dolgozata alapj´an ¨ Osszesen 238 dolgozat ´erkezett. 5 pontos 71, 4 pontos 30, 3 pontos 39, 2 pontos 40, 1 pontos 43 dolgozat. 0 pontot 15 versenyz˝ o kapott.

C gyakorlatok megold´asa

C. 1680. Egy n´egysz¨ og egyik oldal´ anak hossza 5 cm, a rajta fekv˝ o k´et sz¨ og 90◦ ◦ ´es 60 . Tudjuk tov´ abb´ a, hogy a n´egysz¨ og h´ ur- ´es ´erint˝ on´egysz¨ og is. Hogyan lehet ezek alapj´ an megszerkeszteni a n´egysz¨ oget? ´ Irjuk le ´es indokoljuk a szerkeszt´es l´ep´eseit (az elemi szerkeszt´esi l´ep´eseket, mint pl. sz¨ og felez´ese, tengelyes t¨ ukr¨ oz´es, nem kell r´eszletezni).

C. 1686. Az ABC der´eksz¨ og˝ u h´ aromsz¨ og ´ atfog´ oja az AB szakasz. Az A cs´ ucsb´ ol kiindul´ o f bels˝ o sz¨ ogfelez˝ o a BC oldalt a D pontban metszi. Bizony´ıtsuk be, hogy az AB − BD ´es AC + CD szakaszok hossz´ anak m´ertani k¨ ozepe ´eppen az f = AD sz¨ ogfelez˝ o hossza. Javasolta: Zagyva Tiborn´e (Baja) I. megold´as. Jel¨ olj¨ uk a h´ aromsz¨ og oldalait a szok´ asos m´odon, legyen BC = a, CA = b ´es AB = c. A bels˝ o sz¨ ogfelez˝ o t´etele szerint

Javasolta: Zagyva Tiborn´e (Baja) Megold´as. Nem s´erti az ´altal´ anoss´ agot, ha feltessz¨ uk, hogy az ABCD n´egyurn´egysz¨ ogben AB = 5 cm ´es DAB = 90◦ , illetve ABC = 60◦ . Mivel ABCD h´ sz¨ og, ez´ert a szemben lev˝ o sz¨ ogeinek ¨ osszege p´ aronk´ent 180◦ , ´ıgy BCD = 90◦ ´es CDA = 120◦ . A felt´etel szerint ABCD ´erint˝on´egysz¨og is, ez´ert van be´ırt k¨ore, teh´ at bels˝o sz¨ ogfelez˝ oi egy pontban metszik egym´ast, ez a pont a be´ırt k¨or O k¨oz´eppontja. K¨ oz´ episkolai Matematikai ´ es Fizikai Lapok, 2022/7

409

BD AB c = = . DC CA b Eszerint van olyan λ > 0 val´ os sz´am, hogy a BD ´es DC szakaszok hossz´ara (1)

BD = λc,

DC = λb

teljes¨ ul. Tekints¨ uk az 1. ´ abr´ at. 410

K¨ oz´ episkolai Matematikai ´ es Fizikai Lapok, 2022/7

sz´ ar´ u der´eksz¨og˝ u h´ aromsz¨ og. A sz¨ ogfelez˝ o egy m´ asik tulajdons´ aga szerint a CAB uak. sz´ arain az AC ´es AC  szakaszok is egyenl˝o hossz´ A D ´es D pontok konstrukci´ oj´ab´ ol az el˝oz˝ oek szerint k¨ ovetkezik, hogy

Az ABC der´eksz¨og˝ u h´ aromsz¨ ogben fel´ırhatjuk, hogy cos 2α =

b , c

az ADC der´eksz¨ og˝ u h´ a-

romsz¨ ogben pedig cos α = fb , illetve sin α = λb . f

(2)

b2 λ 2 b2 b2 − λ 2 b2 b = 2− 2 = . c f f f2

AD = AC  + C  D = AC + CD;

(4)

Felhaszn´ aljuk a cos 2α = cos2 α − sin2 α trigonometrikus azonoss´ agot, ezzel

AD = AB − BD = AB − BD.

og, ´es ezzel annak k k¨ or¨ ul´ırt k¨ ore mindig l´etrej¨ on, ez csak akkor A DD D h´aromsz¨ nem lenne lehets´eges, ha a D ´es D pontok azonosak lenn´enek. Ez azonban azt ol (4) alapj´an az k¨ ovetkezne, hogy AC + CD = jelenten´e, hogy AD = AD , amib˝ = AB − BD, azaz AB = AC + CD + BD = AC + BC, de ez az ABC h´ aromsz¨ ogre fel´ırt h´aromsz¨og-egyenl˝ otlens´eg miatt lehetetlen.

1. a ´bra

A (2) egyenlet jobb oldal´ at ´ atalak´ıtjuk: (b − λb)(b + λb) b b(1 − λ)(b + λb) = = , c f2 f2 ahonnan b-vel val´ o oszt´ as ´es rendez´es ut´ an f 2 = (c − λc)(b + λb) k¨ovetkezik. Nyilv´anval´ o, hogy az ABD h´ aromsz¨ ogben a D cs´ ucsn´ al tompasz¨og van, ez´ert c > λc, vagyis c − λc > 0, m´asr´eszt b + λb > 0, teh´at  (3) f = (c − λc)(b + λb).

Mivel c − λc = AB − BD, illetve b + λb = AC + CD, ez´ert a (3) egyenlet ´eppen azt jelenti, amit bizony´ıtani akartunk:  f = (AB − BD)(AC + CD),

teh´ at az AB − BD ´es AC + CD szakaszok hossz´anak m´ertani k¨ozepe val´ oban az f sz¨ ogfelez˝ o hossza.

Sipeki M´ arton (Szolnok, Verseghy Ferenc Gimn´azium, 11. ´evf.) II. megold´as. Bocs´ assunk mer˝ olegest a D pontb´ ol az AB ´atfog´ ora, a mer˝oleges uk f¨ol a C  pontb´ ol kiindulva a B talppontja legyen C  . Az AB egyenes´ere m´erj¨ ol kiindulva az A pont fel´e a BD = BD ir´any´ aba a C  D = CD szakaszt, a B pontb´ szakaszt.

ogre δ = 45◦ . Bizony´ıtani fogjuk, hogy a 2. ´abr´ an jel¨ olt ADD  sz¨ Nyilv´anval´ o, hogy α ADC = 90◦ − , 2 a BDD h´aromsz¨ ogben pedig BD = BD , emiatt BDD  = BD D =

180◦ − β , 2

fel´ırhatjuk teh´ at, hogy 90◦ −

(5)

180◦ − β α +δ+ = 180◦ . 2 2

al´as´ aval A (5) egyenletben a m˝ uveletek elv´egz´es´evel ´es α + β = 90◦ felhaszn´ val´oban azt kapjuk, hogy δ = 45◦ . A k k¨orben a DD h´ urhoz 45◦ -os ker¨ uleti sz¨ og tartozik, a δ sz¨ og egyik sz´ ara  ´gy lehets´eges, hogy a δ sz¨ og AD sz´ ara a k k¨ or a DD szakasz, ´ıgy δ = 45◦ csak u ´erint˝oje. A k¨orh¨oz h´ uzott szel˝ o- ´es ´erint˝oszakaszok t´etel´eb˝ ol k¨ ovetkezik, hogy AD2 =   ol (4) szerint = AD · AD , amelyb˝ AD2 = (AC + CD) · (AB − BD), ebb˝ ol pedig n´egyzetgy¨ okvon´ as ut´an a feladat ´all´ıt´ asa ad´ odik. Megjegyz´es. A D pont az AB egyenesen a B ponton t´ ul is elhelyezkedhet, ez azonban a megold´ ast menet´et nem befoly´ asolja, mert a (2) ¨ osszef¨ ugg´es ebben az esetben is fel´ırhat´ o.

J´ armai Roland (Budapest, B´ek´ asmegyeri Veres P´eter Gimn., (10. ´evf.) dolgozata alapj´an III. megold´as. T¨ ukr¨ ozz¨ uk a D pontot a B cs´ ucsb´ ol indul´o BE bels˝ o sz¨ ogfelez˝ ore, a sz¨ogfelez˝ o tulajdons´aga miatt a D t¨ uk¨ ork´ep az AB ´ atfog´ o bels˝o pontja. Jel¨olj¨ uk meg tov´ abb´ a az AC egyenesen, a C ponton t´ ul azt a D pontot, amelyre  CD = CD.

2. ´ abra

Tekints¨ uk a 2. ´ abr´ at. Az AD sz¨ ogfelez˝ o minden pontja egyenl˝o t´avols´ agra o van a sz¨ ogsz´ arakt´ ol, ez´ert CD = C  D, ´es ´ıgy C  D = C  D, teh´at D DC  egyenl˝ K¨ oz´ episkolai Matematikai ´ es Fizikai Lapok, 2022/7

411

Mivel a t¨ ukr¨ oz´es miatt BD = BD , ez´ert egyr´eszt AD = AB − BD, m´ asr´eszt AD = AC + CD. 

412

K¨ oz´ episkolai Matematikai ´ es Fizikai Lapok, 2022/7

A 3. ´ abra jel¨ ol´eseivel 2α + 2β = 90◦ , ◦ uen bel´ athat´ o, hogy ´ıgy α + β = 45 . Egyszer˝ BD D = 90◦ − β, ´es ez´ert

Megold´as. El˝ osz¨ or azt fogjuk bizony´ıtani, hogy az ABG h´ aromsz¨ ognek BM s´ ulyvonala, majd pedig azt, hogy az M pont harmadolja a BM szakaszt.

AD D = 90◦ + β. A DD C egyenl˝ o sz´ ar´ u, der´eksz¨ og˝ u h´ aromsz¨ og, teh´at D DC = 45◦ , ´es mivel ADC = 90◦ − α, ez´ert ADD  = 90◦ − α + 45◦ , ebb˝ ol α + β = 45◦ alapj´an ADD  = 90◦ + β k¨ ovetkezik.

3. a ´bra

Az AD D ´es ADD h´ aromsz¨ ogek k´et-k´et sz¨ og´enek nagys´ aga α ´es 90◦ + β, ´ıgy a h´ aromsz¨ ogek harmadik sz¨ oge is nyilv´an megegyezik, ez pedig azt jelenti, hogy a k´et h´ aromsz¨ og hasonl´o. Hasonl´o h´ aromsz¨ ogekben a megfelel˝ o oldalak ar´anya megegyezik, ez´ert AD AD = , AD AD

2



Az ADC egyenl˝ o sz´ ar´ u der´eksz¨ og˝ u h´ aromsz¨ og, aromsz¨ ogben E-n´el de´ıgy DAC = 45◦ . Az AM E h´ og van, teh´at az AM E h´ ar´eksz¨ og, E-n´el 45◦ -os sz¨ romsz¨ og szint´en egyenl˝ o sz´ ar´ u ´es der´eksz¨ og˝ u, AE = = M E = x. Az eredeti ABC h´ aromsz¨ ogben C-n´el og van, ´ıgy az is azonnal ad´ odik, hogy a BEC 45◦ -os sz¨ h´ aromsz¨ og is egyenl˝o sz´ ar´ u ´es der´eksz¨ og˝ u. A feladatban szerepl˝o felt´etel alapj´ an CG = BM = y. Mivel y + x = BE = CE = y + EG, ez´ert EG = x. Az eddigiek alapj´an az ABG h´ aromsz¨ ogben az E pont az AG oldal felez˝ opontja, ez´ert BE a s´ ulyvonala, teh´at az M pont rajta van a s´ ulyvonalon. Azt kell m´eg bel´atni, hogy BM : M E = y : x = 2 : 1. Tudjuk, hogy AF : F B = 2 : 3. Legyen a sz´ amol´asainkhoz AF = 2z ´es F B = 3z. og hasonl´o a BEA der´eksz¨ og˝ u h´ aromsz¨ ogh¨ oz, A BF M der´eksz¨ og˝ u h´ aromsz¨ mert tov´ abbi egy sz¨ og¨ uk, a B-n´el fekv˝ o sz¨ og egybeesik. A megfelel˝ o oldalak ar´anya megegyezik:



azaz AD = AD · AD .

o eredm´eMivel AD = f , AD = AB − BD, illetve AD = AC + CD, el˝oz˝ ovetkezik, ez ´eppen a feladat a´ll´ıt´ asa, ny¨ unkb˝ ol f 2 = (AB − BD) · (AC + CD) k¨ hiszen ebb˝ ol a nyilv´anval´ oan pozit´ıv AB − BD ´es AC + CD sz´ amok m´ertani k¨ozepe:  f = (AB − BD) · (AC + CD).

(1)

⇐⇒

15z 2 = y(x + y).

Az ACF h´aromsz¨ og is hasonl´o az ABE h´ aromsz¨ ogh¨ oz, mert sz¨ ogeik megegyeznek. Az oldalak ar´ anya itt (2)

(A K¨ oMaL-honlapon is megtal´ alhat´ o a II. megold´ as)

¨ Osszesen 116 dolgozat ´erkezett. 5 pontos 67, 4 pontos 13, 3 pontos 7, 2 pontos 9 dolgozat. 1 pontot 7, 0 pontot 12 versenyz˝ o kapott. Nem versenyszer˝ u: 1 dolgozat.

3z y = 5z x+y

2x + y 2z = x 5z

⇐⇒

10z 2 = x(2x + y).

Az (1) ´es (2) egyenletekben szerepl˝ o sz´ amok mindegyike pozit´ıv, a megfelel˝ o oldalak oszt´ as´aval: y(x + y) 3 = , 2 x(2x + y) 6x2 + 3xy = 2y 2 + 2xy,

Matematika feladatok megold´asa

6x2 + xy − 2y 2 = 0.

B. 5193. Az ABC hegyessz¨ og˝ u h´ aromsz¨ ogben BCA = 45◦ , a magass´ agok talppontjai a BC, CA, AB oldalakon rendre D, E, F , a h´ aromsz¨ og magass´ agpontja M . Tudjuk, hogy az F pont az AB szakaszt AF : F B = 2 : 3 ar´ anyban osztja. Az AC oldalon megjel¨ olj¨ uk azt a G pontot, amelyre CG = BM . Mutassuk meg, hogy az ABG h´ aromsz¨ og s´ ulypontja M . (4 pont) K¨ oz´ episkolai Matematikai ´ es Fizikai Lapok, 2022/7

413

Szorzatt´ a alak´ıtva: (2x − y)(3x + 2y) = 0. Az x ´es y szakaszok hosszai, ´ıgy a m´asodik t´enyez˝ o biztosan pozit´ıv, teh´ at 2x − y = 0 ⇔ 2x = y. Ezzel bel´ attuk, hogy az M pont 2 : 1 ar´ anyban osztja kett´e a BE s´ ulyvonalat, vagyis az M pont az ABG h´ aromsz¨ og s´ ulypontja. F¨ ul¨ op Csilla (Szegedi Radn´ oti M. K´ıs´erleti Gimn., Szeged, 11. ´evf.) dolgozata alapj´an ¨ Osszesen 97 dolgozat ´erkezett. 4 pontot kapott 58 versenyz˝ o, 3 pontos 16, 2 pontos 13 tanul´ o dolgozata. 1 pontot kapott 8, 0 pontot 2 tanul´ o.

414

K¨ oz´ episkolai Matematikai ´ es Fizikai Lapok, 2022/7

B. 5218. Legfeljebb h´ any v´ alaszthat´ o ki az els˝ o 2022 pozit´ıv eg´esz sz´ am k¨ oz¨ ul u ´gy, hogy semelyik k´et kiv´ alasztott sz´ am k¨ ul¨ onbs´ege ne legyen pr´ımsz´ am? (5 pont) Megold´as. Seg´ed´ all´ıt´ as: Nyolc egym´ast k¨ ovet˝o eg´esz sz´ am k¨oz¨ ul legfeljebb kett˝ o v´ alaszthat´o ki u ´gy, hogy semelyik k´et kiv´alasztott sz´ am k¨ ul¨onbs´ege ne legyen pr´ım. Bizony´ıt´ as: B´ armely k´et kiv´ alasztott sz´ am k¨ ul¨onbs´ege 1, 4 vagy 6 (mivel 8 egym´ ast k¨ ovet˝o sz´ amr´ ol van sz´ o ´es a k¨ ul¨ onbs´eg nem lehet 2, 3, 5 vagy 7). Tegy¨ uk fel, hogy ki tudunk v´ alasztani h´ armat u ´gy, hogy b´ armelyik kett˝ o k¨ ul¨onbs´ege 1, 4 vagy 6. Legyenek ezek a sz´ amok a < b < c ´es legyenek x = b − a, y = c − b. Ilyenkor x, y, ´es x + y is az 1, 4, 6 sz´ amok valamelyike. Ez nem lehets´eges. Ellentmond´ asra jutottunk, teh´ at nem tudunk kiv´alasztani h´ arom sz´amot. Az els˝o 2022 pozit´ıv eg´esz sz´ amot feloszthatjuk 252 ilyen 8-as csoportra (1–8, 9–16, . . . , 2009–2016) ´es egy 6-os csoportra (2017–2022). Egyik csoportb´ ol sem v´ alaszthatunk ki 2-n´el t¨ obb sz´ amot. ´Igy ¨ osszesen nem v´ alaszthatunk ki 2 · 253 = = 506-n´ al t¨ obb sz´ amot. P´elda 506-ra: 4-gyel osztva 1 marad´ekot ad´ o sz´ amok (b´armely 2 k¨ ul¨onbs´ege oszthat´ o 4-gyel, ´ıgy nem lehet pr´ım). ´Igy legfeljebb 506 sz´amot v´ alaszthatunk ki. Kov´ acs Alex (Szegedi Radn´ oti Mikl´ os K´ıs´erleti Gimn., 12. ´evf.) Megjegyz´esek. 1. Egy tipikus hiba az volt, hogy a megold´ o a helyes konstrukci´ ob´ ol indult ki (4k + 1 alak´ u sz´ amok halmaza), ´es azt mondta, hogy t¨ obb elemet m´ ar nem lehet kiv´ alasztani ebb˝ ol a halmazb´ ol. Ez hib´ as ´ervel´es, mert pl. ha moh´ o algoritmussal elkezdj¨ uk kiv´ alasztani a legkisebb sz´ amot, amit m´eg bevehet¨ unk, akkor az {1, 2, 10, 11, . . .} halmazt kapjuk. ´Igy viszont l´enyegesen kevesebb elemet v´ alasztunk ki, mint 506, azonban tov´ abbi elemet m´ ar nem tudunk a list´ ankhoz adni. 2. Szint´en hasonl´ o hiba, hogy feltett´ek, hogy periodikusan kell kiv´ alasztani a sz´ amokat. 3. Egy m´ asik tipikus hiba, hogy l´enyegeben csak annyit mondtak, hogy ha van k´et szomsz´edos sz´ am, akkor ut´ ana van legal´ abb h´et nem kiv´ alaszthat´ o sz´ am. Ez az ´ervel´es azonban nem z´ arja ki azt az esetet, hogy a v´ alasztott sz´ amok halmaza az {1, 5, . . . , 2021, 2022}.

K. 735. A logikai k´eszletet Dienes Zolt´an fejlesztette ki. Peti kiveszi a k´eszletb˝ ol a piros ´es a z¨ old k¨ or¨ oket ´es n´egyzeteket, ¨ osszesen 16 darab s´ıkidomot. Ezen alakzatok mindegyike k¨ ul¨ onb¨ ozik valamiben a t¨ obbit˝ ol. Az alakzatok n´egy szempont alapj´an is k´et egyforma darabsz´am´ u csoportra oszthat´ ok: – kicsi vagy nagy, – piros vagy z¨ old, – k¨orlap vagy n´egyzet, – lyukas vagy sima. El tudja-e Peti helyezni a 16 s´ıkidomot egy k¨ or ment´en u ´gy, hogy a szomsz´edos alakzatok pontosan egy tulajdons´agban egyezzenek meg? K. 736. Egy c´egn´el 120 alkalmazott dolgozik: v´ızvezet´ek-szerel˝ ok, burkol´ok, k˝ om˝ uvesek ´es fest˝ ok. A v´ızvezet´ek-szerel˝ ok ´es a k˝ om˝ uvesek mindannyian rendelkeznek jogos´ıtv´ annyal, a t¨ obbiek pedig nem. A k˝ om˝ uvesek ´es a fest˝ ok a Pipacs utc´aban dolgoznak, a t¨obbiek a Kankalin utc´ aban. A jogos´ıtv´ annyal nem rendelkez˝ o alkalmazottak sz´ ama 64, a Kankalin utc´ aban dolgoz´ ok´e 84. A v´ızvezet´ek-szerel˝ ok sz´ama as´ u alkalmazottb´ ol h´ any dolgozik a fest˝ ok sz´am´anak k´etszerese. Melyik foglalkoz´ a c´egn´el? K/C. 737. Ha van k´et ismert hossz´ us´ag´ u zsin´ orunk, akkor lem´erhetj¨ uk ´es kijel¨olhetj¨ uk a k´et zsin´or hossz´anak ¨ osszeg´et, k¨ ul¨ onbs´eg´et, illetve egy zsin´ or hossz´at f´elbehajt´ assal felezhetj¨ uk. Van egy 2240 centim´eteres ´es egy 1760 centim´eteres v´ekony zsin´ orunk, ezek seg´ıts´eg´evel szeretn´enk kijel¨ olni egy 10 centim´eteres t´ avols´ agot csup´an egyetlen m´er´essel. (Az elj´ar´ as sor´ an teh´at f´elbehajt´ assal felez´est ak´ arh´ anyszor v´egrehajthatunk, de o sszeg vagy k¨ u l¨ o nbs´ e g lem´ e r´ e s´ e t csak egyetlen alkalommal ¨ tehetj¨ uk meg.) Adjunk meg egy megfelel˝o elj´ar´ ast. K/C. 738. A falinapt´ aron a h´onap napjait h´et oszlopba rendezve t¨ untetik fel. ana fentr˝ ol lefel´e haladva az egyes oszlopok a h´et napjainak Balr´ol jobbra ´es ut´ megfelel˝o napok sorsz´am´at tartalmazz´ak egym´as ut´an. Egy ilyen falinapt´ arban egy n × n-es n´egyzetes elrendez´esben tal´ alhat´ o napsorsz´ amok ¨ osszege 198. Mekkora lehet az ´erintett napsorsz´ amok k¨ oz¨ ott a legkisebb? Beku ¨ld´esi hat´arid˝o: 2022. november 10. Elektronikus munkafu ¨zet: https://www.komal.hu/munkafuzet

88 dolgozat ´erkezett. 5 pontos 45, 4 pontos 11, 3 pontos 8, 2 pontos 10, 1 pontos 12, 0 pontos 2 dolgozat.

A K pontversenyben kit˝ u z¨ ott gyakorlatok ABACUS-szal k¨ oz¨ os pontverseny 9. oszt´alyosoknak (734–738.)

A C pontversenyben kit˝ uz¨ott gyakorlatok (737–738., 1733–1737.)

K. 734. Sanyi ´es bar´ atai a focimeccsen az egyik h´eten 6 zacsk´ o szotyit ´es 4 zacsk´ o t¨ okmagot vettek, ez´ert ¨ osszesen 1900 Ft-ot fizettek. A k¨ovetkez˝ o h´eten 4 zacsk´ o szotyit ´es 2 zacsk´ o t¨ okmagot vettek, ´es ¨ osszesen 1100 Ft-ot fizettek. Egy zacsk´o szotyi ´es egy zacsk´ o t¨ okmag ´ ara mindek¨ ozben nem v´ altozott. H´ any forintba ker¨ ult egy zacsk´ o szotyi, illetve egy zacsk´ o t¨ okmag? K¨ oz´ episkolai Matematikai ´ es Fizikai Lapok, 2022/7

415

Feladatok 10. ´evfolyamig K/C. 737. A sz¨ oveg´et l´ asd a K feladatokn´ al. K/C. 738. A sz¨ oveg´et l´ asd a K feladatokn´ al. 416

K¨ oz´ episkolai Matematikai ´ es Fizikai Lapok, 2022/7

Feladatok mindenkinek C. 1733. Legfeljebb h´ any k¨ ul¨ onb¨ oz˝ o pozit´ıv pr´ımoszt´ oja lehet egy olyan h´ aromjegy˝ u sz´ amnak, amelynek a h´ arom sz´amjegye valamilyen sorrendben h´ arom, egym´ as ut´ani pozit´ıv eg´esz sz´ am?

A B pontversenyben kit˝ uz¨ott feladatok (5262–5269.)

Berk´ o Erzs´ebet (Szolnok) javaslata alapj´ an C. 1734. Az AB ´ atm´er˝ oj˝ u k k¨ or k¨ oz´eppontja O. Megrajzoljuk az OB ´atm´er˝ oj˝ u k1 k¨ ort, illetve a k1 k¨ort C pontban ´erint˝o, AB-vel p´ arhuzamos egyenest, amely arozzuk meg a COD1  ´es COD2  a k k¨ ort a D1 ´es D2 pontokban metszi. Hat´ sz¨ ogek nagys´ ag´ anak pontos ´ert´ek´et. Javasolta: B´ır´ o B´ alint (Eger)

(3 pont)

C. 1735. Oldjuk meg a val´ os sz´amp´ arok halmaz´an a √ √ x + y = 6,

Javasolta: Kozma Katalin Abig´el (Gy˝or)

B. 5263. Igazoljuk, hogy a h´ aromsz¨ og s´ ulyvonalainak n´egyzet¨ osszege nem kisebb a f´elker¨ ulet n´egyzet´en´el.

1 5 1 + = x y 16

(3 pont)

egyenletrendszert. (The Mathematical Association of America) Feladatok 11. ´evfolyamt´ol C. 1736. Az ABCD paralelogramma CD oldal´ an felvessz¨ uk a P bels˝ o pontot, a CD-vel p´ arhuzamos AB oldalon a Q bels˝ o pontot. A P A ´es QD szakaszok metsz´espontja M , a P B ´es QC szakaszok metsz´espontja N . Hat´ arozzuk meg annak a felt´etel´et, hogy M N  AB.

(Amerikai versenyfeladat o ol ) ¨tlet´eb˝

C. 1737. Alad´ ar a 32. sz¨ ulet´esnapj´ ara kapott k´et kock´ at. Az egyik kocka lapjait 1-t˝ ol 6-ig megsz´amozta, a m´asikra rendre a 0; 1; 2; 7; 8; 9 sz´amokat ´ırta. Ezekkel a kock´ akkal 10-t˝ ol kezdve ´eppen az ´eletkor´ aig, azaz 32-ig b´ armelyik pozit´ıv eg´esz sz´ amot kirakhatja, de a 33-at m´ar nem. K¨ ob¨ uki kock´ ak helyett szab´alyos okta´edereket haszn´ alt. Az okta´ederek lapjaira egy-egy sz´ amjegyet ´ırt, ´ıgy 10-t˝ ol kezdve a saj´at – ´evekben m´ert – ´eletkor´ aig az ¨ osszes eg´esz sz´ amot kirakta. H´any ´eves most K¨ ob¨ uki, ha ez egy ´ev m´ ulva m´ar nem siker¨ ulne? Javasolta: Kozma Katalin Abig´el (Gy˝or)



Javasolta: N´emeth L´ aszl´ o (Fony´ od)

B. 5264. Ketten a k¨ ovetkez˝ o j´at´ekot j´atssz´ak. El˝ osz¨ or Kezd˝o el˝ o´ırja egy 0–1 sorozat tetsz˝ oleges sz´ am´ u (ak´ ar v´egtelen sok) elem´et u ´gy, hogy v´egtelen sok elem o´ırja a sorozat legkisebb index˝ u m´eg nem m´eg ne legyen el˝o´ırva. Ezut´ an M´asodik el˝ el˝o´ırt elem´enek ´ert´ek´et. Majd ezeket a l´ep´eseket ism´etelgetik felv´ altva a v´egtelens´egig. Kezd˝o nyer, ha a kapott sorozat valahonnan kezdve periodikus, M´ asodik nyer, ha nem az. Van-e valakinek nyer˝ o strat´egi´ aja (´es ha igen, kinek)? (4 pont)

Javasolta: Pach P´eter P´ al (Budapest)

B. 5265. Nagy´ıtsuk k´etszeres´ere egy der´eksz¨ og˝ u h´ aromsz¨ og be´ırt k¨ or´et a der´eksz¨og˝ u cs´ ucsb´ ol. Mutassuk meg, hogy a kapott k¨ or ´erinti a h´aromsz¨ og k¨ or¨ ul´ırt k¨or´et. (4 pont)

Javasolta: V´ıgh Viktor (Szeged)

¨ B. 5266. N´eh´ any focista egy¨ utt nyaral. Osszesen k klubb´ ol ´es n nemzetb˝ol uk legal´ abb n − k + 1 olyan val´ok, ahol k < n. Bizony´ıtsuk be, hogy van k¨ oz¨ ott¨ focista, aki t¨obb klubt´ ars´ aval nyaral egy¨ utt, mint honfit´ ars´ aval. (5 pont) B. 5267. Adott egy p ´es egy q hossz´ us´ ag´ u szakasz, valamint egy ABC h´ aromsz¨og, amelynek oldalegyenesei a, b ´es c a szok´ asos bet˝ uz´es szerint. Szerkessz¨ uk meg anyban az ABC k¨or¨ ul´ırt k¨ or´en azt a P pontot, amire Pa a Pb Pc szakaszt p : q ar´ oleges vet¨ ulete az x oldalegyenesre. osztja, ahol Px a P pont mer˝

Beku ¨ld´esi hat´arid˝o: 2022. november 10. Elektronikus munkafu ¨zet: https://www.komal.hu/munkafuzet



K¨ oz´ episkolai Matematikai ´ es Fizikai Lapok, 2022/7

B. 5262. Lenke le´ırt egy lapra egy term´eszetes sz´ amot, amely nem tartalmaz 0-t, de tartalmaz legal´ abb k´et k¨ ul¨ onb¨ oz˝ o sz´amjegyet. Ezut´an a sz´ am al´ a le´ırta az ¨osszes olyan sz´ amot, amely az eredeti sz´ am jegyei sorrendj´enek megv´ altoztat´ as´aval l´etrehozhat´ o. Legfeljebb mekkora lehet a lapon szerepl˝ o sz´amok legnagyobb k¨oz¨os oszt´ oja?

(5 pont) 417

418

K¨ oz´ episkolai Matematikai ´ es Fizikai Lapok, 2022/7

B. 5268. Az ABC h´ aromsz¨ og be´ırt k¨ or´enek k¨ oz´eppontj´at jel¨olje I. Az ABC h´ aromsz¨ og belsej´eben, az ABI k¨ or¨ on vegy¨ unk fel egy P pontot. Az AP egyenes AI-re vett t¨ uk¨ ork´epe az ABI k¨ ort az A-n k´ıv¨ ul m´eg a Q pontban metszi. Bizony´ıtsuk be, hogy CP = CQ. (6 pont)

Javasolta: Kocsis Szilveszter (Budapest)

f

A. 835. Jel¨ olje f (n) (x) az f f¨ uggv´eny n-szeres iter´ altj´ at (azaz f (1) (x) = f (x), (n) (x) = f (f (x))).

(n+1)

Legyen p(n) egy adott eg´esz egy¨ utthat´ os polinom, amely pozit´ıv eg´esz n-ekre pozit´ıv eg´esz ´ert´eket vesz f¨ ol. Lehet-e az f (n) (n) = p(n) f¨ uggv´enyegyenletnek uggv´eny a megold´ asa? pontosan egy f : Z+ → Z+ f¨ Javasolta: Matolcsi D´ avid ´es Szab´ o Krist´ of (Budapest)

B. 5269. Legyen p  19 egy p´aratlan sz´ am. Sz´ınezz¨ uk ki a 0, 1, . . . , p − 1 sz´amokat k´et sz´ınnel. Legyen 1  i  p eset´en xi a {0, 1, . . . , p − 1} halmaz egy v´eletlenszer˝ uen v´ alasztott eleme (egyenletes eloszl´ as szerint, egym´ast´ ol f¨ uggetlenek a v´ aus´ege, hogy x1 , . . . , xp laszt´ asok). Igazoljuk, hogy legal´abb 3/(2p p) annak a val´osz´ın˝ egyforma sz´ın˝ uek ´es p | x1 + . . . + xp . (6 pont)

Beku ¨ld´esi hat´arid˝o: 2022. november 10. Elektronikus munkafu ¨zet: https://www.komal.hu/munkafuzet



Javasolta: Pach P´eter P´ al (Budapest)

 Beku ¨ld´esi hat´arid˝o: 2022. november 10. Elektronikus munkafu ¨zet: https://www.komal.hu/munkafuzet

Informatik´ab´ ol kit˝ uz¨ott feladatok

 I. 571. Sokan sz´ıvesen j´atszanak a pozit´ıv eg´esz sz´ amokkal ´es azok sz´amjegyeivel. Egy j´at´ekban a pozit´ıv eg´eszeket egyszer˝ us´ıtj¨ uk t¨ obb l´ep´esben a k¨ ovetkez˝ ok szerint: 1. Az egyjegy˝ u sz´ amokat nem egyszer˝ us´ıtj¨ uk tov´ abb.

Az A pontversenyben kit˝ u zo ¨tt nehezebb feladatok (833–835.) A. 833. A koordin´ ata-rendszer n´eh´ any r´ acspontj´at kisz´ınezz¨ uk pirosra, a t¨obbit feh´erre. Egy ilyen sz´ınez´est v´egesen univerz´ alisnak nevez¨ unk, ha tetsz˝ oleges v´eges, nem¨ ures A ⊂ Z eset´en l´etezik olyan k ∈ Z, hogy az (x, k) pont pontosan akkor van pirosra sz´ınezve, ha x ∈ A.

a) L´etezik-e olyan v´egesen univerz´ alis sz´ınez´es, hogy minden sorban v´eges sok r´ acspontot sz´ınez¨ unk pirosra, ´es minden sort k¨ ul¨onb¨oz˝ ok´eppen sz´ınez¨ unk meg, tov´ abb´ a a pirosra sz´ınezett r´ acspontok halmaza ¨ osszef¨ ugg˝o? b) L´etezik-e olyan v´egesen univerz´ alis sz´ınez´es, hogy minden sorban v´eges sok r´acspontot sz´ınez¨ unk pirosra, tov´abb´ a a pirosra ´es a feh´erre sz´ınezett r´ acspontok ugg˝o? halmaza is ¨osszef¨ A r´ acspontok egy H r´eszhalmaz´ at akkor nevez¨ unk o¨sszef¨ ugg˝onek, ha b´armely x, y ∈ H-ra l´etezik egy olyan r´acsvonalakon halad´ ou ´t, amely csak H-beli pontokon megy ´at, ´es x-et o oti y-nal. ¨sszek¨

2. A nem egyjegy˝ u, de p´ aros sz´am´ u sz´ amjegyb˝ ol ´ all´o sz´ amok eset´en megvizsg´aljuk, hogy a sz´ am utols´o sz´ amjegye oszt´ oja-e a sz´ am utols´o sz´ amjegy´enek elhagy´ as´aval keletkez˝ o sz´ amnak. Ha oszt´ oja, akkor a sz´amot egyszer˝ us´ıtj¨ uk arra sz´amra, amelyet az utols´ o sz´ amjegy elhagy´ as´ aval kapunk. 3. A nem egyjegy˝ u, de p´ aratlan sz´ am´ u sz´amjegyb˝ ol ´ all´ o sz´amok eset´en megvizsg´aljuk, hogy a sz´ am els˝ o sz´ amjegye oszt´ oja-e a sz´ am els˝ o sz´amjegy´enek elhauk arra gy´as´aval keletkez˝ o sz´ amnak. Ha oszt´ oja, akkor a sz´amot egyszer˝ us´ıtj¨ sz´ amra, amelyet az els˝o sz´ amjegy elhagy´ as´ aval kapunk. P´eld´ aul az 1208 eset´en a 120 oszt´ oja a 8, ´ıgy a sz´amot el˝osz¨ or a 120-ra egyszer˝ us´ıtj¨ uk, majd a 20-nak oszt´ oja az 1, ´ıgy a k¨ ovetkez˝ o l´ep´esben 20-ra egyszer˝ us´ıtj¨ uk, v´eg¨ ul a 2-nek nem oszt´oja a 0, vagyis tov´ abb nem egyszer˝ us´ıthet˝ o, ´ıgy az 1208 egyszer˝ us´ıt´es ut´ an 20 lesz. K´esz´ıts¨ unk programot, amely megadja az [a; b] intervallumba es˝ o (10  a < b   10 000 000) egyjegy˝ u sz´ amra egyszer˝ us´ıthet˝ o pozit´ıv eg´eszek sz´ am´at. A program a standard bemenet els˝ o sor´ ab´ ol olvassa be a ´es b ´ert´ek´et, majd a standard kimenet els˝ o sor´ aban adja meg a keresett eg´eszek sz´ am´at.

Javasolta: Kocsis Anett (Budapest)

P´eld´ ak:

urnyolcsz¨og, ´es i = 1, 2 . . . , 8 eset´en Bi = A. 834. Legyen A1 A2 . . . A8 konvex h´ = Ai Ai+3 ∩ Ai+1 Ai+4 (az indexek modulo 8 ´ertend˝ ok). Igazoljuk, hogy a B1 , . . . , B8 pontok egy k´ upszeleten vannak. K¨ oz´ episkolai Matematikai ´ es Fizikai Lapok, 2022/7

419

420

Bemenet 20 80 10000 20000 1000000 3000000

Kimenet 15 415 7831 K¨ oz´ episkolai Matematikai ´ es Fizikai Lapok, 2022/7

Bek¨ uldend˝o egy t¨om¨ or´ıtett i571.zip ´ allom´anyban a program forr´ ask´odja, valamint a program r¨ ovid dokument´ aci´ oja, amely tartalmazza a megold´ as r¨ovid le´ır´ as´ at, ´es megadja, hogy a forr´ as´allom´any melyik fejleszt˝oi k¨ornyezetben ford´ıthat´ o. I. 572. Sok sport´ ag lebonyol´ıt´ asi form´ aja r´eszben, vagy teljesen k¨orm´erk˝ oz´eses rendszerben t¨ ort´enik. Az ilyen versenyek sor´ an minden r´esztvev˝ o az ¨osszes t¨obbi r´esztvev˝ ovel egyszer megm´erk˝ ozik. K´esz´ıts¨ uk el n = 8 r´esztvev˝ ore a k¨ orm´erk˝ oz´es p´ aros´ıt´asait t´ abl´ azatkezel˝ o alkalmaz´ as seg´ıts´eg´evel a minta alapj´an. A mint´an l´athat´ o s´ arga mez˝ okbe szabad uggv´enyekkel a´ll´ıtsuk el˝ o. A mega csapatneveket beg´epelni, a t¨ obbi cella ´ert´ek´et f¨ old´ as ann´ al t¨ obb pontot ´er, min´el t¨ obb a f¨ uggv´enyek k¨oz¨ott a m´asolhat´ o.

¨ A t´ abl´ azat szerkezet´et, a cell´ak form´az´ as´ at ´ all´ıtsuk be a minta szerint. Ugyelj¨ unk a megfelel˝ o cell´ ak sz´eless´eg´ere, szeg´elyez´es´ere ´es a tartalom igaz´ıt´ as´ara. Seg´edsz´am´ıt´ asokat v´egezhet¨ unk az E oszlopt´ ol jobbra, amelyek ´ertelmez´es´et feliratokkal seg´ıts¨ uk el˝ o vagy a dokument´ aci´ oban ´ırjuk le. A megold´ asban saj´ at f¨ uggv´eny vagy makr´ o nem haszn´ alhat´ o. Bek¨ uldend˝o egy t¨ om¨ or´ıtett i572.zip ´ allom´anyban a megold´ ast tartalmaz´ o munkaf¨ uzet ´es a megold´ as r¨ ovid le´ır´ as´ at bemutat´ o dokument´ aci´ o. ´ Az atomok.txt egyszer˝ I. 573 (E). u sz¨ oveges a´llom´any az egyes atomok izoazat minden sor´aban egy atom egy izot´ opjainak t´abl´ azat´ at tartalmazza. A t´abl´ t´ opj´anak adatai szerepelnek az al´abbi minta szerint: ... 5 B 10 10,012 936 95(41) 0,199(7) 5 B 11 11,009 305 36(45) 0,801(7) 6 C 12 12,000 000 00(00) 0,9893(8) 6 C 13 13,003 354 835 07(23) 0,0107(8) 6 C 14 14,003 241 9884(40) 7 N 14 14,003 074 004 43(20) 0,996 36(20) 7 N 15 15,000 108 898 88(64) 0,003 64(20) ... Az els˝o sz´ am az elem rendsz´ama (Z), a m´asodik a vegyjele (jel¨olj¨ uk V-vel), a harmadik az elem izot´ opj´anak t¨ omegsz´ ama (A), a negyedik sz´ am az izot´op relat´ıv K¨ oz´ episkolai Matematikai ´ es Fizikai Lapok, 2022/7

421

atomt¨omege (m), az ¨ ot¨ odik sz´ am – amennyiben van – az izot´op el˝ofordul´as´ anak ar´ anya (jel¨olj¨ uk ea-val) egy adott elem eset´en. A z´ ar´ ojelben l´ev˝ o mennyis´egek az ´ert´ekek utols´ o egy-k´et sz´ amjegy´enek hib´ aj´at jelentik, de a feladatban ezekkel nem kell sz´ amolni. K´esz´ıts¨ unk sz´ am´ıt´ og´epes programot, amely beolvassa a t´ abl´ azatot tartalmaz´ o sz¨oveges a´llom´anyt, majd megoldja az al´ abbi feladatokat. A k´eperny˝ ore ´ır´ ast ig´enyl˝o r´eszfeladatok eset´en – a mint´ ahoz tartalm´aban hasonl´oan – ´ırjuk ki a k´eperny˝ore a feladat sorsz´ am´at (p´eld´ aul: 2. feladat:), ´es utaljunk a ki´ırt tartalomra is. Ha a felhaszn´al´ot´ ol k´er¨ unk be adatot, akkor jelen´ıts¨ uk meg a k´eperny˝ on, hogy milyen ´ert´eket v´arunk. Mindk´et esetben az ´ekezetmentes ki´ır´ as is elfogadott. 1. Olvassuk be az atomok.txt sz¨ oveges a´llom´anyb´ol az izot´opok adatait. A z´ ar´ ojelben l´ev˝ o mennyis´egeket a sz´ am´ıt´ as sor´ an nem fogjuk haszn´alni, ´ıgy azokat nem kell elt´ arolni. Az egy sorban l´ev˝ o ´ert´ekek k¨ oz¨ ott sz´ ok¨ oz az elv´ alaszt´ ojel. ¨ Ugyelj¨ unk arra, hogy a sz´ amok eg´esz- ´es t¨ ortr´esze k¨ oz¨ ott tizedesvessz˝ o szerepel, illetve a legt¨ obb esetben ezres tagol´ assal vannak megadva az ´ert´ekek, teh´ at itt is sz´ ok¨oz tal´ alhat´ o. Amennyiben egy izot´ opn´ al az el˝ofordul´ as ar´anya nincs megadva, u ´gy ott az ea ´ert´eket tekints¨ uk null´ anak. Helyes beolvas´ as eset´en a fenti mint´aban szerepl˝o izot´ opok sz´amadatai feldolgoz´ as ut´an ´ıgy n´eznek ki: ... 5 B 10 10.01293695 0.199 5 B 11 11.00930536 0.801 6 C 12 12.0 0.9893 6 C 13 13.003354835069999 0.0107 6 C 14 14.003241988400001 0.0 7 N 14 14.00307400443 0.99636 7 N 15 15.00010889888 0.00364 ... 2. K´erj¨ uk be a felhaszn´ al´ot´ ol egy elem vegyjel´et, ´es ´ırjuk ki az elem ¨ osszes izot´opj´anak adatait a k¨ ovetkez˝ o form´ aban: 2. feladat: K´ erem egy elem vegyjel´ et: C Z=6 A=12 m=12.0 ea=0.9893 Z=6 A=13 m=13.003354835069999 ea=0.0107 Z=6 A=14 m=14.003241988400001 ea=0.0 3. Adjuk meg, hogy melyik elemeknek szerepel a t´ abl´ azatban a legt¨ obb izot´ opjuk, ´es mennyi ez az ´ert´ek. 3. feladat: A legnagyobb izot´ opsz´ am: 10 Az elemek: Sn 4. K´esz´ıts¨ unk f¨ uggv´enyt tomeg n´even, amely megadja egy elem relat´ıv atomt¨ omeg´et figyelembe v´eve az izot´opjainak relat´ıv atomt¨ omeg´et ´es el˝ ofordul´ asok ar´ any´ at. 422

K¨ oz´ episkolai Matematikai ´ es Fizikai Lapok, 2022/7

A f¨ uggv´eny bemenete az elem vegyjele, visszaadott ´ert´eke egy val´ os sz´am legyen. P´eld´ aul a sz´en (C) eset´en az elem relat´ıv atomt¨omege a h´ arom izot´op adatai alapj´an (l´asd a mint´ at): 12,0 · 0,9893 + 13,003 354 835 069 999 · 0,0107 + 14,003 241 988 400 001 · 0,0 = 3 = 12,010 735 896 735 248. 5. K´esz´ıts¨ unk tomegek.txt n´even egyszer˝ u sz¨ oveges a´llom´anyt, amely azon elemek relat´ıv atomt¨ omeg´et tartalmazza, amelyekn´el legal´ abb egy izot´op ar´anya 0-t´ ol k¨ ul¨ onb¨ oz˝ o ´ert´ek. Egy sorban egy elem vegyjele ´es relat´ıv atomt¨omege szerepeljen. Az ´allom´anyban minden megfelel˝ o elem csak egyszer forduljon el˝o. Minta a tomegek.txt ´ allom´anyr´ol: H 1.0077091323512934 He 4.002601932120928 Li 6.94003660291572 Be 9.012183065 B 10.811028046410001 C 12.010735896735248 N 14.006703211445798 Bek¨ uldend˝o egy t¨om¨ or´ıtett i573.zip ´ allom´anyban a program forr´ ask´odja, valamint a program r¨ ovid dokument´ aci´ oja, amely tartalmazza a megold´ as r¨ovid le´ır´ as´ at, ´es megadja, hogy a forr´ as´allom´any melyik fejleszt˝oi k¨ornyezetben ford´ıthat´ o. I/S. 65. Adott egy N cs´ ucsb´ ol ´es M ´elb˝ ol ´ all´ o ir´any´ıtatlan gr´ af. A cs´ ucsokat 1-t˝ ol N -ig indexelj¨ uk. A gr´ afot az al´abbi m´ odon t¨ obb l´ep´esben b˝ ov´ıtj¨ uk ´elek hozz´ aad´ as´aval: egy l´ep´esben keres¨ unk egy olyan x ´es y cs´ ucsp´ art, amely nincs ¨osszek¨otve, de l´etezik olyan z cs´ ucs, ami ¨ ossze van k¨ otve x-szel ´es y-nal is (1  x, y, z  N ). Ha tal´ altunk ilyen x, y cs´ ucsp´art, akkor ¨ osszek¨ otj¨ uk ˝oket egy ´ellel, ha nem, akkor befejez˝od¨ ott a gr´ af b˝ ov´ıt´ese. Adjuk meg, hogy a gr´ afb˝ ov´ıt´es befejezt´evel h´any ´el van a gr´ afban. Mivel ez asi marad´ek´ at kell a sz´ am nagyon nagy is lehet, ez´ert a sz´ am (109 + 7)-tel vett oszt´ megadni. A standard bemenet els˝ o sor´ aban az N ´es M sz´ amok szerepelnek sz´ok¨ozzel elv´ alasztva. A tov´ abbi M sor mindegyike 2 sz´ amot tartalmaz sz´ ok¨ozzel elv´ alasztva: egy adott ´el k´et v´egpontj´ anak index´et. A standard kimenet egyetlen sor´ aban egy sz´ am szerepeljen: a gr´afb˝ ov´ıt´es befejezt´evel a gr´ afban lev˝o ´elek sz´ ama modulo 109 + 7. Bemenet (a / jel sort¨ or´est helyettes´ıt) 8 5 / 1 2 / 2 4 / 3 2 / 6 7 / 7 8

S. 164. A ford´ıt´ oprogramok nagyon alacsony szinten ´atrendezhetik a forr´ asprogramban kisz´ am´ıtand´ o kifejez´esben az elv´egzend˝ o m˝ uveletek sorrendj´et az´ert, hogy id˝ot vagy mem´ori´at sp´oroljanak. Ez sokat jav´ıthat egy programon, de nem v´altoztathatja meg annak jelent´es´et. Ebben a feladatban egy egyszer˝ u sz´ am´ıt´ as mem´oriaig´eny´et pr´ ob´ aljuk cs¨ okkenteni. Adott egy csak v´altoz´ okat, valamint ¨ osszead´ as ´es szorz´ as m˝ uveleteket tartalmaz´o kifejez´es. Az egyszer˝ us´eg kedv´e´ert ford´ıtott lengyel jel¨ ol´est haszn´ alunk. P´eld´ aul az (a + b) ∗ c kifejez´es lengyel form´ aban ab + c ∗ lesz. Ennek el˝onye, hogy a sz´ am´ıt´ og´ep ezt balr´ ol jobbra haladva egyszer˝ uen v´egre tudja hajtani. Kezdetben az ¨ osszes v´altoz´ o ´ert´eke bent van a mem´ ori´aban, mindegyik egy egys´egnyi t´ arhelyet foglal el. ´j (egys´egnyi) mem´ oEgy matematikai m˝ uvelet v´egrehajt´ asa ut´ an az eredm´eny egy u riater¨ uletre ker¨ ul. Az el˝ oz˝ o p´eld´ aban x = a + b, y = x ∗ c. A program ´ıgy 5 egys´egnyi mem´ori´at haszn´al, kezdetben a, b ´es c ´ert´ek´enek, majd x ´es y t´ arol´ as´ ara. A kifejez´es ki´ert´ekel´ese jav´ıthat´ o, ha a k´es˝ obb m´ar nem haszn´alt (egys´egnyi) mem´oriater¨ uleteket minden lehets´eges alkalommal fel¨ ul´ırjuk. A p´elda egy lehets´eges v´egrehajt´ asa ezzel a jav´ıt´ assal: a = a + b, b = a ∗ c. A program ´ıgy 3 egys´egnyi ul mem´ori´at haszn´al a kezdeti v´ altoz´ ok miatt. (Nem sz´am´ıt, hogy az eredm´eny v´eg¨ melyik v´ altoz´ oba ker¨ ul.) K´esz´ıts¨ unk programot, amely egy lengyelform´aban adott kifejez´es eset´en megadja, hogy h´ any egys´egnyi t´ arhely sz¨ uks´eges a kifejez´es ki´ert´ekel´es´ehez az eredeti ´es az optimaliz´alt m´odszer felhaszn´ al´as´ aval. A bemenet els˝ o ´es egyetlen sora egy sz´ am´ıt´ ast tartalmaz a ford´ıtott lengyel jel¨ol´es szerint. Ennek minden karaktere az angol abc egy kisbet˝ uje, illetve a +” ´es ” *” karakter lehet. ” A kimenet els˝ o sor´ aba a sz´ amol´as optimaliz´aci´ o n´elk¨ uli, a m´asodik sor´ aba az optimaliz´aci´ o ut´ ani mem´ oriaig´eny´et kell ´ırni. Minta: Bemenet

Id˝ olimit: 0,5 mp. ´ Ert´ekel´es: a pontok 20%-a kaphat´ o, ha a megold´ as a kimenet els˝o sor´ at helyesen hat´ arozza meg. Bek¨ uldend˝o egy s164.zip t¨ om¨ or´ıtett a´llom´anyban a megfelel˝oen dokument´ alt ´es kommentezett forr´ asprogram, amely tartalmazza a megold´ as l´ep´eseit, valamint megadja, hogy a program melyik fejleszt˝ oi k¨ ornyezetben futtathat´ o.



Kimenet

A feladatok megold´asai regisztr´aci´o ut´an a k¨ovetkez˝o c´ımen t¨olthet˝ok fel: https://www.komal.hu/munkafuzet Beku ¨ld´esi hat´arid˝o: 2022. november 15.

9

Korl´ atok: 2  N, M  105 . Id˝ olimit: 0,4 mp. ´ Ert´ekel´es: a pontok 50%-a kaphat´ o, ha a program az N, M  500 tesztesetekre helyes kimenetet ad. K¨ oz´ episkolai Matematikai ´ es Fizikai Lapok, 2022/7

Kimenet (a / jel sort¨ or´est helyettes´ıt) 8 / 4

ab+bc+*ca+*

423

 424

K¨ oz´ episkolai Matematikai ´ es Fizikai Lapok, 2022/7

k¨oz¨os t´ arsasoz´ assal ´es besz´elget´essel u ott¨ uk el az id˝ot. Sajnos ennek k¨ ovetkezt´e¨t¨ ben f´el szand´ allal a l´ abamon jutottam vissza hajnalban a matekos sz´ all´ asra, ´ıgy megtanultam ´ert´ekelni az als´ o eg´eszr´esz fontoss´ag´ at. Mindezek ellen´ere ¨ osszess´eg´eben m´egis nagyon pozit´ıv ´erz´esekkel t´ avoztam Domb´ ov´ arr´ ol, ahol u ´j bar´ atokat szereztem, csod´ as ´elm´enyekkel, ´es sok-sok tud´ assal gazdagodtam.

Ny´ari matematika- ´es fizikat´abor 2022. Domb´ ov´ar

Sztrany´ak Gabriella Budapest

2022. j´ unius v´eg´en ¨ osszesen 39 k¨ oz´episkol´as di´ak gy˝ ult ¨ossze a Domb´ov´ arGunaras u ul˝ ok¨ ozpontban. A t´ arsas´ ag egyik fele matematik´ aval, a m´asik fizik´ aval ¨d¨ foglalkozott, a szabadid˝ ot pedig k¨ oz¨ osen t¨ olt¨ ott´ek. A t´abort a K¨oz´episkolai Matematikai ´es Fizikai Lapokat kiad´o MATFUND Alap´ıtv´ any szervezte.

T´ amogat´ ok:

A Nemzeti Tehets´eg Program keret´eben fogadta el a Minisztereln¨oks´eg az NTP-TSZM-21-0144 p´aly´ azatot ( Mi´enk a t´er ´es az id˝o – a t¨omeg K¨oMaL t´aborba ” j¨ on”. A szervez˝ ok Ny´ari t´abor 2022. ´ id´en m´ Ha ny´ ar eleje, akkor K¨ oMaL t´abor! En asodszor vettem r´eszt a K¨oMaL uld˝oi ny´ ari t´ abor´aban, ahol a K¨ oMaL M, P ´es G pontverseny´enek eredm´enyes bek¨ egy kis fizik´aval ind´ıthatt´ ak a nyarat, a matek olimpiai csapatok pedig felk´esz¨ ulhettek a megm´erettet´esekre. Az els˝o k¨ oz¨ os programunk egy fizika el˝oad´ as volt, amit Honyek Gyula tan´ ar u ´r ´es Baranyai Kl´ ara tan´ arn˝ o tartottak. Az els˝ o t´ aboromban a rivaliz´al´as jegy´eben nem fogadtam kit¨ or˝ o ¨or¨ ommel, hogy t´ avol az iskol´at´ ol fizik´at kelljen hallgatnom, de az el˝oad´ as gy¨ okeresen megv´altoztatta a v´elem´enyem. Mindk´et alkalommal sz´ orakoztat´ o st´ılusban hallgathattuk meg k¨ ul¨ onb¨ oz˝ o fizikai k´ıs´erletek ´es jelens´egek magyar´azat´ at. A b´ atrabb v´allalkoz´ ok meggy´ ujthatt´ ak szappanos oldatba m´ artott tenyer¨ uket, vagy megk´ ostolhatt´ ak a sz´ ar´ıtott jeget.

Besz´amol´ o a 6. Eur´opai Fizikai Di´akolimpi´ar´ ol

K´et ´ev online rendez´es ut´ an ism´et hagyom´ anyos form´aban, 2022. m´ajus 20. ´es 24. k¨oz¨ott Ljublj´an´ aban, Szlov´enia f˝ov´ aros´ aban rendezt´ek meg a hatodik Eur´opai Fizikai Di´akolimpi´at (EuPhO). A versenyen 30 eur´opai ´es 7 Eur´op´ an k´ıv¨ uli orsz´ag ossze 12 arany´ermet osztottak ki, ¨osszesen 182 di´akja vett r´eszt. A versenyen mind¨ amelyb˝ ol az egyiket egy magyar di´ak, Kov´ acs Bal´ azs Csaba szerezte meg, aki az abszol´ ut 6. helyen v´egzett. A csapat ´es eredm´enyeik:

ob´alkozott a lehetetlen felAz el˝oad´ ast k¨ ovet˝oen a fizik´ asok egy r´esze megpr´ adattal, hogy megtan´ıtsanak bili´ ardozni, amit p´ ar k¨or forg´o k¨ovetett. Hab´ar az IMO–MEMO focimeccs id´en elmaradt, a matekosok ´es fizik´ asok sz´amos sportban m´erhett´ek ¨ ossze tud´ asukat. Mialatt mi az ´altalunk, illetve Kiss G´eza ´es Dobos S´ andor tan´ ar u ´r ´altal hozott feladatokkal pr´ob´ altunk megbirk´ ozni, a fizik´ asok krumpli´ agy´ uval l˝ ottek, m´er´eseket ´es becsl´eseket v´egeztek, ´es kis csapatokban versenyeztek nagy t´ abla csokik´ert. Mi, matekosok k´et egy´eni pr´ obaversenyt is ´ırtunk, m´ıg egy MEMO-jelleg˝ u pr´obacsapatversenyt is kipr´ob´ alhattunk. Hab´ ar nagy meglepet´esre senki nem tudta megoldani a k´ınai v´alogat´ o 6-os feladat´at, el´egedettek lehett¨ unk az eredm´enyeinkkel. Az ifik strandol´ as k¨ ozben jav´ıtgatt´ ak a dolgozatainkat, ´ıgy a medenc´eb˝ ol kil´epve betekint´est nyerhett¨ unk a jav´ıt´ asba, ´es reklam´ alhattunk is. A fizik´ asok az ´elm´enyf¨ urd˝oben uk, amiben igyekezt¨ unk seg´ıtsem pihenhettek, cs´ usz´ asok sebess´eg´et kellett m´erni¨ s´eg¨ ukre lenni.

Kov´acs Bal´azs Csaba (Hatvani Bajza J´ ozsef Gimn´ azium, 12. oszt.) arany´erem (34,4 pont), felk´esz´ıt˝ o tan´ arai: Maruzsin´e Sevella Judit ´es Kov´ acs L´ aszl´ o; ´ Gurz´ o J´ozsef (Budapesti Fazekas Mih´aly Gyakorl´ o Altal´ anos Iskola ´es Gimn´azium, 12. oszt.) ez¨ ust´erem (26,1 pont), felk´esz´ıt˝ o tan´ ara: Nagy Piroska M´ aria; ´ azium, Altal´ anos Bencz Benedek (Budapest, Ba´ar-Madas Reform´atus Gimn´ Iskola ´es Koll´egium, 9. oszt.) bronz´erem (18,6 pont), felk´esz´ıt˝ o tan´ ara: Horv´ ath Norbert; ´ Toronyi Andr´as (Budapest, Ba´ar-Madas Reform´atus Gimn´ azium, Altal´ anos Iskola ´es Koll´egium, 12. oszt.) bronz´erem (18,5 pont), felk´esz´ıt˝ o tan´ ara: Horv´ ath Norbert; ´ Moln´ar-Szab´ o Vilmos (Budapesti Fazekas Mih´aly Gyakorl´ o Altal´ anos Iskola ´es Gimn´ azium, 11. oszt.) dics´eret (14,9 pont), felk´esz´ıt˝ o tan´ ara: Nagy Piroska M´ aria.

Utols´ o este sajnos nem rakhattunk t´abort¨ uzet, mint tavaly, de a k¨oz¨os ´enekl´es ´ıgy is ´atmelengette sz´ıv¨ unket. Mikor az es˝ o a fizik´ asok fah´ azaiba sz´ am˝ uz¨ott minket,

A magyar csapat vezet˝oi Sz´ asz Kriszti´ an ´es Vank´ o P´eter voltak, Vigh M´ at´e pedig a zs˝ uriben, az els˝ o elm´eleti feladat szerz˝ ojek´ent k´epviselte haz´ ankat.

K¨ oz´ episkolai Matematikai ´ es Fizikai Lapok, 2022/7

425

426

K¨ oz´ episkolai Matematikai ´ es Fizikai Lapok, 2022/7

Az al´abbiakban k¨ oz¨ olj¨ uk a verseny feladatait, a megold´ asok a verseny honlapj´an (https://eupho.ee/eupho-2022/) ´erhet˝ ok el. K´ıs´erleti feladat: A megvil´ag´ıt´as fizik´aja Egy izz´ o u ´gy bocs´ at ki f´enyt, hogy kell˝ oen magas h˝ om´ers´ekletre hev´ıt egy volfr´ amsz´ alat, ´es ´ıgy a feketetest-sug´arz´as a spektrum l´athat´ o r´esz´ebe esik, de az energia jelent˝os r´esze elv´esz az infrav¨ or¨ os tartom´ anyban. Az emberi l´at´ as sz´am´ara ´erz´ekelhet˝ o f´eny mennyis´egi jellemz´es´ere olyan fotometrikus m´ert´ekegys´eget haszn´ alunk, amely figyelembe veszi az emberi szem hull´ amhosszf¨ ugg˝o ´erz´ekenys´eg´et. Egy forr´ as ´ altal minden ir´ anyban kibocs´ atott teljes l´athat´ o f´enymennyis´eget f´eny´aramnak nevezz¨ uk, ´es lumenben [lm] m´erj¨ uk. Egy fel¨ ulet egys´egnyi ter¨ ulete altal ´erz´ekelt l´ ´ athat´ o f´enymennyis´eget megvil´ag´ıt´asnak nevezz¨ uk, m´ert´ekegys´ege ovel m´erhetj¨ uk. a lux [lx = lm/m2 ], ´es f´enym´er˝ Ha a f´eny mennyis´eg´et az a´ltala sz´all´ıtott energia alapj´an jellemezz¨ uk, akkor radiometrikus egys´egeket haszn´ alunk, amelyek a teljes´ıtm´eny hagyom´ anyos egys´egeivel fejezhet˝ ok ki. A f´eny´ aram radiometrikus megfelel˝ oje a sug´arz´asi fluxus, m´ert´ekegys´ege a watt [W], a besug´arz´as [W/m2 ] pedig a megvil´ag´ıt´as megfelel˝oje. A feladatban a f´enyforr´ asok termikus ´es vil´ag´ıt´ astechnikai tulajdons´ agaival fogsz foglalkozni. A h´ arom feladat nagyr´eszt f¨ uggetlen egym´ ast´ ol. V´azold fel a m´er´esi elrendez´esedet mindegyik feladathoz! Az 1. ´es 2. feladatn´ al nem kell hibasz´am´ıt´ ast v´egezned.

A – Fekete ´es feh´er, 3 mm vastag m˝ uanyag lemez tart´ oval. Mindk´et lemez j´ ol elnyeli az infrav¨ or¨ os sug´arz´ ast. B – F´enym´er˝ o tart´ oval. A f´enym´er˝ o 6 perc ut´ an automatikusan kikapcsol – az on/off gomb hosszan tart´o benyom´ as´aval tudod u ´jra bekapcsolni. Figyelj az egys´egekre: (lx legyen, ne fc)! A HOLD gombbal tudod a kijelzett ´ert´eket r¨ ogz´ıteni.

K¨ oz´ episkolai Matematikai ´ es Fizikai Lapok, 2022/7

G – V¨or¨os, z¨ old ´es k´ek sz´ınsz˝ ur˝ o bor´ıt´ekban. (Ha sz´ınt´eveszt˝o vagy, k´erj a t´ abl´ addal seg´ıts´eget!) A sz˝ ur˝ok ´erz´ekenyek a h˝ ore. Tartsd t´avol azokat a f´enyforr´ast´ ol! H – T´apegys´eg. Nyomd meg t¨ obbsz¨ or a fesz¨ ults´eg/´aram gombot a be´all´ıtani k´ıv´ ant sz´ amjegy kiv´ alaszt´ as´ ahoz (a sz´amjegy alatt villog´o f´eny jelzi), ´es a gomb teker´es´evel a´ll´ıtsd be a sz´ am´ert´eket. P´ ar m´asodperc m´ ulva a villog´as abbamarad, ´es as´ ahoz a kijelz˝ o az aktu´ alis fesz¨ ults´eg/´aram´ert´eket mutatja. A f´enyforr´ as szab´alyoz´ az a´ramot v´ altoztasd. Ha a k´ıv´ ant a´ram nem ´erhet˝o el a fesz¨ ults´eglimit t´ ull´ep´ese n´elk¨ ul, akkor a t´apegys´eg ´ atv´ alt ´alland´ o fesz¨ ults´eg˝ u (fesz¨ ults´eggener´ ator) m´ odba, ´es korl´atozza az a´ramot. Csatlakoztasd a vezet´ekeket a megfelel˝ o negat´ıv (fekete) ´es pozit´ıv (v¨or¨ os) csatlakoz´oba a t´ apegys´egen. A z¨ old csatlakoz´ot ne haszn´ ald! A f´enyforr´as v´edelme ´erdek´eben el˝ osz¨ or ´all´ıtsd be a feszu ¨lts´eget a megengedett maxim´alis ´ert´ekre, az ´aramot pedig null´ara, miel˝ott csatlakoztatod a vezet´ekeket! Ha ki´eg a f´enyforr´ asod, k´erhetsz helyette m´ asikat. (Csak korl´atozott mennyis´eg ´all rendelkez´esre.) 1. Sz´ın ´es h˝ om´ers´eklet (4 pont) A feketetest-sug´arz´ as sz´ıne a h˝ om´ers´eklett˝ ol f¨ ugg. A csillag´aszatban a csillagok h˝om´ers´eklet´et a sz´ınindex¨ uk alapj´ an hat´ arozz´ ak meg, ami k´et k¨ ul¨ onb¨ oz˝ o sz´ınsz˝ ur˝ ovel m´ert megvil´ag´ıt´ as h´ anyadosa. T [K] 1570 1600 1610 1620 1630 1640 1660 1670 1700 1730 1780 1820 1880

M´er´esi eszk¨ oz¨ ok (l´asd az 1. ´abr´at!)

C – Kerek talp´ u tart´ o a f´enyforr´ asok r¨ ogz´ıt´es´ere, egy nehez´ek a stabiliz´al´as´ ara ´es k´et cser´elhet˝ o f´enymodul: egy izz´ol´ampa (maxim´alis feszu ¨lts´eg 12 V) ´es egy LED (maxim´alis feszu ¨lts´eg 3,0 V), ne l´epd t´ ul a 400 mA ´aramot). A modulok be´ekel´es´ehez haszn´ alhatod a fogpiszk´al´ot. A fekete pap´ırral v´edheted a szemed a m˝ uszer leolvas´ asakor. D – Infrav¨ or¨ os h˝om´er˝ o. A m´ert ´ert´ek a ravasz megnyom´ asa ut´ an kis k´es´essel jelenik meg. A m´er´eseknek lehet egy jelent˝ os, de ´alland´ o (szisztematikus) hib´ aja.

E – Nagy pap´ır, amin dolgozhatsz, t´ avols´ ag- ´es sz¨ ogbeoszt´asokkal. F – Sz¨ogm´er˝ o.

1. a ´bra. F´enyk´ep a k´ıs´erleti feladatok eszk¨ ozeir˝ ol. A sz¨ ogm´er˝ o ´es a fekete arny´ekol´ ´ o pap´ır nincs rajta a f´enyk´epen

427

v¨or¨ os [lx] 2 4 5 6 8 10 12 14 18 24 37 51 80

z¨ old [lx] 0 0 1 2 3 4 5 6 9 14 23 34 57

k´ek [lx] 0 0 0 0 0 0 0 0 1 3 7 11 21

T [K] 1940 2000 2060 2120 2160 2220 2260 2310 2350 2390 2460 2500 2540

v¨ or¨ os [lx] 120 165 230 310 379 484 586 753 888 1032 1292 1577 1811

z¨ old [lx] 91 130 194 274 348 460 570 748 929 1107 1452 1826 2198

k´ek [lx] 36 53 80 118 155 210 264 348 440 527 697 879 1078

1. t´ abl´ azat. Ismert h˝ om´ers´ekleten m˝ uk¨ od˝ o izz´ o megvil´ ag´ıt´ asa h´ arom k¨ ul¨ onb¨ oz˝ o sz´ın˝ u, a f´enyforr´ ast´ ol ´es a f´enym´er˝ ot˝ ol r¨ ogz´ıtett t´ avols´ agban l´ev˝ o sz´ınsz˝ ur˝ ovel megm´erve. A m´er´es pontoss´ aga ±2 lx

a) Az 1. t´ abl´ azat tartalmazza egy izz´ ol´ampa v¨ or¨ os, z¨ old ´es k´ek sz´ınsz˝ ur˝ on kereszt¨ ul m´ert megvil´ag´ıt´ as´ at a h˝ om´ers´eklet f¨ uggv´eny´eben. V´ alassz megfelel˝ o sz´ın428

K¨ oz´ episkolai Matematikai ´ es Fizikai Lapok, 2022/7

sz˝ ur˝ oket, ´es a t´ abl´ azat alapj´an k´esz´ıts egy kalibr´ aci´ os g¨orb´et, amely megadja a kiv´ alasztott sz´ınindexet a h˝ om´ers´eklet f¨ uggv´eny´eben! b) M´erd meg, milyen ¨ osszef¨ ugg´es van az izz´ osz´ alra kapcsolt elektromos telje´ azold eredm´enyedet a m´ert tartos´ıtm´eny ´es az izz´ osz´ al h˝om´ers´eklete k¨ oz¨ ott! Abr´ m´ anyban.

v´egez, mik¨ozben tengelye mindv´egig egybeesik az ed´eny tengely´evel. A folyad´ek felsz´ın´enek rezg´esi amplit´ ud´ oja A = 1 mm. Hat´ arozd meg a mozg´as T peri´ odusidej´et! A folyad´ek viszkozit´ as´ at hanyagold el. 2. H˝otani rezg´esek (10 pont)

2. F´enyhasznos´ıt´as (8 pont)

Egy ellen´ all´ as olyan anyagb´ol k´esz¨ ult, amelynek van egy f´ azis´ atalakul´asa olyan m´odon, hogy az ellen´all´ asa a k¨ oom´ers´eklete kivetkez˝ o k´et ´ert´ek valamelyike: R1 , ha a h˝ om´ers´eklete nagyobb, sebb, mint Tc , ´es R2 > R1 , ha a h˝ mint Tc .

Egy f´enyforr´ ast a f´enyhasznos´ıt´as jellemez, amit lumen/watt egys´egekben m´e¨ r¨ unk, azaz a f´eny´ aram ´es a felhaszn´ alt teljes´ıtm´eny h´ anyadosak´ent. Osszehasonl´ ıt´ ask´ent a Nap f´enyhasznos´ıt´ asa 93 lm/W. M´erd meg a f´enyhasznos´ıt´ ast a r´ akapcsolt elektromos teljes´ıtm´eny f¨ uggv´eny´e´ aas eset´eben a m´erhet˝ o f´enykibocs´at´ as tartom´ any´ aban! Abr´ ben mindk´et f´enyforr´ ´ zold eredm´enyeidet f´enyforr´ asonk´ent k¨ ul¨ on-k¨ ul¨ on grafikonon. Ird le az ¨osszes sz´ am´ıt´ asi l´ep´est, ´es add meg az ¨ osszes m´ert adatot. 3. Sug´arz´ o f˝ ut´es (8 pont) Ez a feladat id˝ oig´enyes. Ennek megfelel˝ oen tervezd meg az id˝ obeoszt´asodat! Ha egy testre f´eny esik, akkor annak egy r´esze elnyel˝ odik. Ha a test ´es a k¨ornyezet k¨ oz¨ ott nem t´ ul nagy a h˝ om´ers´eklet-k¨ ul¨ onbs´eg, a test h˝ olead´ as´at a k¨ornyezet fel´e a h h˝ o´atad´asi t´enyez˝ ovel jellemezhetj¨ uk, P/A = h(T − T0 ) form´aban, ahol T a feornyezet h˝ om´ers´eklete, ´es P/A jel¨oli az egys´egnyi fel¨ ulet l¨ ulet h˝ om´ers´eklete, T0 a k¨ h˝ olead´ as´at a k¨ ornyezet fel´e. a) Hat´ arozd meg a fekete m˝ uanyag h h˝ o´ atad´ asi t´enyez˝ oj´et ´es λ h˝ ovezet´esi egy¨ utthat´ oj´at, ´es v´egezz hibasz´ am´ıt´ ast! T´etelezd fel, hogy a test minden r´ aes˝ o f´enyt elnyel, ´es hogy az izz´ol´ampa minden teljes´ıtm´enyt elektrom´agneses sug´arz´ as form´aj´aban ad le. b) M´erd meg a feh´er m˝ uanyag albed´ oj´at (azt, hogy a be´erkez˝o sug´ arz´ as mekkora h´ anyada ver˝ odik vissza, ahelyett, hogy elnyel˝ odne), ´es v´egezz hibasz´ am´ıt´ast! Hasznos ¨ osszefu u g¨ omb¨ ov ter¨ ulete a θ1 ´es θ2 pol´arsz¨ogek ¨gg´es: Egy r sugar´ k¨ oz¨ ott (ahol 0  θ1  θ2  π) ΔA = 2πr2 (cos θ1 − cos θ2 ). ´ Utmutat´ o a t´apegys´eg haszn´alat´ahoz A m´er´esen haszn´ alt t´ apegys´eget fesz¨ ults´eggener´ ator ´es ´aramgener´ ator u ¨zemm´ odban is lehetett haszn´ alni. A fesz¨ ults´eg- ´es ´ aramlimitek be´ all´ıt´ as´anak m´odj´ at itt nem k¨ oz¨ olj¨ uk, de megtekinthet˝ o a fent megadott honlapon. Elm´eleti feladatok ´ 1. Usz´o henger (10 pont) Egy szil´ard, homog´en, h = 10 cm magas ´es s = 100 cm2 alapter¨ ulet˝ u henger ulet˝ u, folyad´ekkal t¨olt¨ott henu ´szik egy H = 20 cm magas ´es S = 102 cm2 alapter¨ geres ed´enyben. A henger ´es a folyad´ek s˝ ur˝ us´eg´enek h´ anyadosa γ = 0,70. A henger alja n´eh´ any centim´eterrel az ed´eny alja felett van. A henger f¨ ugg˝oleges rezg´eseket K¨ oz´ episkolai Matematikai ´ es Fizikai Lapok, 2022/7

429

Ezt az ellen´ all´ ast egy L induktivit´ as´ u tekercsen kereszt¨ ul egy fesz¨ ults´egforr´ asra k¨ otj¨ uk. Azt l´ atjuk, hogy ha a forr´as V fesz¨ ults´ege k´et kritikus ´ert´ek k¨ oz´e esik, 2. a ´bra as h˝ om´ers´eklete oszcill´ alni V1 < V < V2 , akkor az ellen´all´ kezd. Tegy¨ uk fel, hogy (i) az ellen´ all´ asb´ ol a k¨ ornyezetbe jut´ o P h˝ o´ aramot a P = α(T − T0 ) kifejez´es ornyezet adja meg, ahol α egy a´lland´ o, T az ellen´ all´ as h˝ om´ers´eklete, T0 pedig a k¨ h˝om´ers´eklete; (ii) az ellen´ all´ as geometriai m´erete olyan kicsi, hogy sokkal gyorsabban el´eri a termikus egyens´ ulyt, mint az L/R2 karakterisztikus id˝o. a) (2 pont) Fejezd ki V1 ´es V2 ´ert´ek´et a fent megadott param´eterek seg´ıts´eg´evel! b) (6 pont) Felt´etelezve, hogy V1 < V < V2 , v´ azold fel az ellen´ all´ as T h˝ om´ers´ekanyadost, let´et a t id˝ o f¨ uggv´eny´eben, ´es hat´ arozd meg a (Tmax − T0 )/(Tmin − T0 ) h´ om´ers´eklet maxim´ alis ´es minim´alis ´ert´ek´et jel¨ oli! ahol Tmax ´es Tmin a T h˝ √ c) (2 pont) Hat´ arozd meg az oszcill´aci´ o peri´ odusidej´et, ha V = V1 V2 ´es R2 = 16R1 ! 3. Dip´olus m´agneses mez˝ oben (10 pont) K´et kicsiny, egyenk´ent m t¨ omeg˝ u, rendre +q ´es −q t¨ olt´essel rendelkez˝o goly´ ot egy d hossz´ us´ ag´ u, merev r´ uddal ¨ osszek¨ ot¨ unk, ´es ´ıgy egy dip´ olust kapunk. A dip´ olus az XY s´ıkban helyezkedik el, ´es az XY -s´ıkra me m´ r˝ oleges ir´ any´ u, homog´en B agneses t´erben van. Kezdetben a dip´olus az X tengely menogsebest´en helyezkedik el, ´es ω0 kezdeti sz¨ s´eggel forog az XY -s´ıkban a 3. ´ abr´ an l´ atha3. a ´bra t´ o m´odon. A dip´ olus t¨ omegk¨ oz´eppontja kezo detben az orig´oban tal´alhat´ o, kezd˝osebess´ege v0 , ami szint´en az XY -s´ıkban fekv˝ vektor. Tekints¨ unk h´ arom k¨ ul¨ onb¨ oz˝ o esetet (a, b ´es c − d): a) (2 pont) Hat´ arozd meg ω0 ´ert´ek´et ´es v0 ir´ any´ at ahhoz, hogy a t¨ omegk¨ oz´eppont a´lland´ o v = v0 sebess´eggel mozogjon! 430

K¨ oz´ episkolai Matematikai ´ es Fizikai Lapok, 2022/7

b) (3 pont) Adott ω0 eset´en hat´ arozd meg azt a v0 vektort (ir´ anyt ´es nagys´agot), amelynek eredm´enyek´epp a t¨ omegk¨ oz´eppont k¨orp´ aly´ an fog mozogni! Hat´ aoz´eppontj´anak xc ´es yc koordin´ at´ ait! Nem kell rozd meg a k¨ orp´ alya Rc sugar´at ´es k¨ bizony´ıtanod, hogy csak egyetlen megold´ as l´etezik. c) (4 pont) Legyen v0 = 0. Hat´ arozd meg azt a legkisebb ω0 = ωmin sz¨ogsebess´eget, ami ahhoz sz¨ uks´eges, hogy a dip´olus ir´ anya ellent´etess´e v´aljon a mozg´asa sor´ an! d) (1 pont) Ha a dip´olus u ´gy indul, hogy v0 = 0 ´es a sz¨ogsebess´ege a c) omegk¨oz´eppont p´ aly´ aj´anak van egy r´eszben meghat´ arozott ω0 = ωmin , akkor a t¨ aszimptot´ aja. Hat´ arozd meg az aszimptota D t´ avols´ ag´ at az orig´ot´ ol! Hasznos vektorazonoss´ ag: a × ( b × c) = b ( a · c) − c ( a · b), ahol a ×” ´es a ·” rendre a vektori´alis, illetve a skal´ aris szorzatot jel¨oli. ” ” Sz´asz Kriszti´an, Vank´ o P´eter

sor´ an ny´ ujtott teljes´ıtm´eny alapj´ an 3 di´ak indulhat az osztr´ak AYPT versenyen, az 5 legjobb di´ ak pedig beker¨ ul a Pakiszt´anban megrendez´esre ker¨ ul˝ o 36. IYPT magyar csapat´aba. Jelentkez´es, a feladatok sz¨ ovege ´es tov´ abbi inform´ aci´ ok az hypt.elte.hu weboldalon, illetve az [email protected] email c´ımen. N´eh´ any p´elda a 2023-ra kit˝ uz¨ ott IYPT probl´em´ ak k¨ oz¨ ul: 3. Szir´ena. Ha leveg˝ot f´ ujunk egy forg´o, lyukakkal ell´atott lemezre, hang hallhat´ o. Magyar´azd meg ezt a jelens´eget, ´es vizsg´ ald meg, hogyan f¨ uggnek a hang tulajdons´agai a relev´ans param´eterekt˝ ol! 14. V´ızsug´ art¨ or´es. A f¨ ugg˝oleges v´ızsug´ar megt¨ orhet, ha egy ferde szit´ an halad kereszt¨ ul, melynek finom a h´ al´ozata. ´Irj fel egy t¨ orv´enyt, mely le´ırja a t¨ or´est, ´es vizsg´ald meg a relev´ans param´etereket! o s´ av. A homokkal teli m´elyed´es elnyelheti egy mozg´o j´arm˝ u kinetikus 17. F´ekez˝ energi´aj´at. Milyen hossz´ us´ ag´ u f´ekez˝ o s´ av k´epes egy passz´ıvan mozg´ o t´ argyat (pl. egy labd´ at) teljesen meg´all´ıtani? Milyen param´eterekt˝ ol f¨ ugg ez a hossz´ us´ ag? Ezu u fizikusaink Rom´ani´aban ¨st´ermesek lettek az ifj´

Ifj´ u Fizikusok Nemzetk¨ ozi Versenye Versenyfelh´ıv´as ´es besz´amol´ o Ha szereted a fizik´ at, a k´ıs´erletez´est, j´ ol besz´elsz angolul, ´es egy ´eletre sz´ ol´ o ´elm´enyre v´ agysz, akkor itt a helyed! A Fizika Vil´agbajnoks´ agnak is nevezett IYPT (Ifj´ u Fizikusok Nemzetk¨ozi Versenye, angolul International Young Physicists’ Tournament) egy angol nyelv˝ u, k´ıs´erleti fizikai csapatverseny, ahova a vil´ ag minden t´aj´ar´ ol (t¨obb mint 30 orsz´ agb´ol) ´erkeznek k¨ oz´episkol´ asok, hogy ¨ osszem´erj´ek tud´ asukat. Az IYPT a XXI. sz´ azad kih´ıv´ asainak megfelel˝ o k´eszs´egeket v´ ar el az indul´okt´ ol: nemcsak a fizik´aban kell j´artasnak lenni, hanem az eredm´enyeket prezent´ alni ´es megv´edeni is tudni kell. A r´esztvev˝ o di´ akok a versenyt megel˝oz˝ oen elv´egzett fizikai m´er´eseiket ´es kutat´ asaikat egy – angol nyelven el˝ oadott – tudom´ anyos prezent´ aci´ o form´aj´aban mutatj´ak be a riv´ alis csapatoknak. Az IYPT verseny magyarorsz´agi els˝ o fordul´oj´ara (Hungarian Young Physicists’ Tournament, HYPT) az hypt.elte.hu oldalon val´ o regisztr´ aci´ o hat´ arideje: 2022. november 1. ´ejf´el. ol 10 perces angol A jelentkez˝ o di´akoknak egy kiv´alasztott IYPT probl´em´ar´ nyelv˝ u el˝ oad´ ast kell k´esz´ıteni ´es felvenni, majd 2022. november 29-ig bek¨ uldeni. Ezen el˝ oad´ asok alapj´an a legjobb bek¨ uld˝ ok az ELTE TTK-n, december k¨ozep´en megrendez´esre ker¨ ul˝ o sz´obeli fordul´on vehetnek r´eszt. Az indul´o di´akoknak itt az ´ altaluk bek¨ uld¨ ott el˝ oad´ ast ´el˝ oben kell el˝oadniuk. A decemberi sz´ obeli fordul´ot k¨ ovet˝oen a 10 legmagasabb pontsz´amot el´er˝ o di´ ak az ELTE TTK Anyagfizikai Tansz´ek´en v´egezheti a tov´ abbi kutat´ asait. A felk´esz¨ ul´es K¨ oz´ episkolai Matematikai ´ es Fizikai Lapok, 2022/7

431

2022. j´ ulius 15–23. k¨ oz¨ ott ker¨ ult megrendez´esre 35. alkalommal az Ifj´ u Fizikusok Nemzetk¨ ozi Versenye (IYPT – International Young Physicists’ Tournament, https://www.iypt.org/). A koronav´ırus ut´ an v´egre ism´et sok orsz´ag tudott r´eszt venni. A magyar csapat 25 orsz´ag k¨ oz¨ ul v´eg¨ ul a 8. helyen v´egzett, ezzel ez¨ ust´ermet szerzett. 11 kutat´asi t´em´ aval ´erkezt¨ unk Temesv´ arra, melyb˝ ol 5-¨ ot mutattunk be a versenyen. A megm´erettet´esen a saj´ at kutat´asi eredm´enyek prezent´ al´asa mellett a t¨ obbi orsz´ ag eredm´enyeinek oppon´ al´asa ´es ´ert´ekel´ese (review) is a verseny r´esze. Tov´ abbi inform´ aci´ ok´ert l´ atogasd meg, ´es k¨ ovesd Facebook-oldalunkat: www.facebook.com/hypt.elte.hu, ahol a csapat r´eszletes ´elm´enybesz´ amol´oj´at, k´epeit ´es esem´enyeit tal´ alod, amelyek t¨ obbet mondanak minden sz´ on´ al. A magyar csapat tagjai: ´ Bodor Emma (Budapesti Fazekas Mih´aly Gyakorl´ o Altal´ anos Iskola ´es Gimn´azium, 10. ´evf.); Czehl´ar Gergely (B´ek´ asmegyeri Veres P´eter Gimn´ azium, 11. ´evf.); Kalocsai Zolt´an (Szombathely, Nagy Lajos Gimn´ azium, 12. ´evf.); ´ Pesti Patrik (ELTE Bolyai J´ anos Gyakorl´ o Altal´ anos Iskola ´es Gimn´ azium, 12. ´evf.); Simon Tam´as (Budapesti N´emet Iskola, 12. ´evf.). A versenyz˝ ok felk´esz´ıt´ese az ELTE Anyagfizikai Tansz´ek´en folyt egyetemi hallgat´ ok, oktat´ok ´es k¨ oz´episkolai tan´ arok vezet´es´evel. – Egyetemi ´es k¨ oz´episkolai oktat´ok: H¨ om¨ ostrei Mih´ aly (Budapesti N´emet Is´ kola, ELTE), Isp´ anovity P´eter (ELTE), Jenei P´eter (ELTE), Szeidemann Akos (E¨otv¨os J´ozsef Gimn´azium, Tata), Sz´echenyi G´ abor (ELTE), Vincze Mikl´ os (ELTEMTA). 432

K¨ oz´ episkolai Matematikai ´ es Fizikai Lapok, 2022/7

´ – Egyetemi hallgat´ok ´es doktoranduszok: B´ an´ oczki T´ımea (BME), Beregi Abel ´ am (ELTE), Lipovics D´ (University of Oxford), Kadlecsik Ad´ aniel (University College London), Penc Patrik (BME), Vavrik M´ arton (BME). A sok nevet´essel ´es kem´eny munk´ aval t¨ olt¨ ott ´ev ut´an most indul a felk´esz¨ ul´es a 2023-as megm´erettet´esre, mely Pakiszt´ anban ker¨ ul megrendez´esre. az HYPT szervez˝ok csapata

Fizika gyakorlatok megold´asa

G. 769. V´ızszintes u ´ton egyenletesen halad´ o aut´ o fogyaszt´ asm´er˝ oje 5 liter/100 km ´ert´eket mutat. Ha ugyanez az aut´ o ugyanezen az u ´ton gyorsulva mozog, akkor a fogyaszt´ asm´er˝ o 10 liter/100 km-t mutat abban a pillanatban, amikor a kocsi el´eri az egyenletes halad´ as sebess´eg´et. Ha az aut´ o ugyanekkora sebess´eggel egy emelked˝ on halad, akkor a fogyaszt´ asa 12 liter/100 km. Mit mutat a fogyaszt´ asm´er˝ o, ha az aut´ o ugyanezen az emelked˝ on az el˝ oz˝ oekben le´ırt gyorsul´ assal mozog felfel´e, ´es a sebess´ege is ´eppen megegyezik az el˝ oz˝ oekben le´ırtakkal? (3 pont) Megold´as. Az aut´o fogyaszt´ asa egyenesen ar´anyos a motor P = F v teljes´ıtm´eny´evel, ahol F a motor (´ att´eteleken kereszt¨ ul ´erv´enyes¨ ul˝ o) h´ uz´ oereje”, v pedig ” az aut´ o sebess´ege. Az F er˝ o egy r´esze az egyenletes mozg´asn´ al legy˝ozend˝ o er˝ok o gyors´ıt´ as´ahoz sz¨ uks´eges er˝ o. ered˝ oj´evel egyezik meg, a marad´ek pedig az aut´ Az aut´ o sebess´ege mind a n´egy esetben ugyanakkora, ´ıgy a sebess´egt˝ ol f¨ ugg˝o k¨ ozegellen´ all´ asi er˝ ok is ugyanakkora nagys´ ag´ uak. A k¨ovetkez˝ o egyenleteket ´ırhatjuk fel:

G. 779. Ha a Hold felsz´ın´et ´ oce´ anok ´es sz´ arazulatok bor´ıtan´ ak, akkor lenne-e a Holdon ap´ aly ´es dag´ aly? (3 pont) Megold´as. Az ´ arap´aly a k¨ ozeli ´egitestek egym´ asra gyakorolt t¨ omegvonz´ asa ´ altal l´etrehozott alakv´ altoz´ asok ¨ osszess´ege. A F¨ old¨ on az a´rap´ aly jelens´egeket nagyobbr´eszt a Hold, kisebbr´eszt a Nap gravit´aci´ os hat´ asai okozz´ak. A Hold a´ltal l´etrehozott f¨ oldi a´rap´ alyt k´et er˝ o ered˝ oje hozza l´etre: a Hold gravit´aci´ os vonz´asa (amely a F¨ old felsz´ın´en helyr˝ ol helyre v´ altozik), valamint a F¨ oldnek a F¨old–Hold rendszer t¨ omegk¨ oz´eppontja k¨ or¨ uli kering´es´eb˝ ol sz´armaz´ o centrifug´alis er˝o. E k´et er˝ o vektori´alis ered˝ojek´ent a F¨ oldnek a Hold fel˝oli ´es azzal ellent´etes oldal´ an t¨obbleter˝ o jelenik meg, itt megemelkedik az o´ce´ anok vize (dag´aly). (Az ´arap´alyt okoz´ o er˝ ok nemcsak a v´ızre, hanem a F¨ old szil´ard k´erg´ere ´es a l´egk¨ orre is hatnak, de ennek hat´ asa nem okoz megfigyelhet˝o v´altoz´ ast.) A Nap hasonl´ o m´odon hoz l´etre a´rap´alyt a F¨ old¨ on, ´es az a´rap´ alyjelens´egek ¨ osszegz˝ odnek. A Hold eset´eben ugyanezek az er˝ ok fenn´ allnak, teh´ at a Holdon is lehetne ´arap´aly, ezt azonban nem a F¨ old, hanem csak a Nap okozn´ a. Mivel a Hold 27,3 napos kering´esi ideje ´es tengely k¨ or¨ uli forg´as´ anak ideje megegyezik (ezt ´epp az u ´n. ´ arap´ alys´ url´od´ as jelens´ege alak´ıtotta ki), ez´ert a Holdnak mindig azonos oldala fordul a F¨ old fel´e. Emiatt a F¨ old okozta dag´ aly a Holdnak mindig ugyanazon a hely´en lenne a legnagyobb, teh´ at a Holdnak egy adott hely´en nem v´ altakozna a F¨ old keltette dag´aly ´es az ap´ aly. alyt a Holdon, hiszen a helyA Nap azonban okozna a f¨ oldihez hasonl´ o ´arap´ zete a Hold egy adott pontj´ ab´ ol n´ezve 29,5 napos peri´ odusid˝ ovel v´ altozik. Emiatt a Holdon kb. 15 naponta lenne dag´ aly, illetve ap´ aly. Hruby Laura (Budapest, Veres P´ aln´e Gimn., 10. ´evf.) dolgozata alapj´an 33 dolgozat ´erkezett. Helyes 9 megold´ as. Hi´ anyos (1–2 pont) 8, hib´ as 14, nem versenyszer˝ u 2 dolgozat.

G. 780. Zavarja-e a halakat, ha a partt´ ol 2 m´eter t´ avols´ agban besz´elgetnek a horg´ aszok? (A hang terjed´esi sebess´ege leveg˝ oben 340 m/s, v´ızben pedig 1500 m/s.)

P1 = F1 v,

F1 = Fk¨ozeg. ,

(4 pont)

P2 = F2 v,

F2 = Fk¨ozeg. + Fgyors´ıt ,

P3 = F3 v,

F3 = Fk¨ozeg. + Flejt˝o ,

P4 = F4 v,

F4 = Fk¨ozeg. + Flejt˝o + Fgyors´ıt .

Megold´as. A v´ızben gyorsabban terjednek a hanghull´ amok, mint leveg˝ oben, teh´at a hangterjed´es szempontj´ ab´ ol a v´ız ritk´abb, a leveg˝ o s˝ ur˝ ubb k¨ ozegnek sz´ am´ıt. (Az akusztikai ´ertelemben vett s˝ ur˝ us´eg term´eszetesen l´enyegesen k¨ ul¨ onb¨ ozik a t¨omegs˝ ur˝ us´egt˝ ol.)

A fenti egyenletekb˝ ol P4 = P2 − P1 + P3 k¨ ovetkezik, vagyis ezekkel ar´ anyosan az emelked˝on gyors´ıt´ o aut´ o fogyaszt´ as´ ara 10 − 5 + 12 = 17 liter/100 km ad´odik. ´ am (Sopron, Berzsenyi D. Ev. (L´ıceum) Gim´azium ´es Koll., 9. ´evf.) S´ os Ad´

A leveg˝ ob˝ ol a v´ız fel´e terjed˝ o hang eset´eben hangtanilag s˝ ur˝ ubb k¨ ozegb˝ ol hangtanilag ritk´ abb k¨ ozegbe l´ep¨ unk a´t, ´ıgy fell´ephet a teljes visszaver˝ od´es jelens´ege. Az α bees´esi sz¨ oggel ´erkez˝o hull´am olyan β t¨ or´esi sz¨ oggel halad tov´ abb, amelyre sin β =

cv´ız cleveg˝o

sin α.

28 dolgozat ´erkezett. Helyes 18 megold´ as. Hib´ as 9, nem versenyszer˝ u 1 dolgozat.

K¨ oz´ episkolai Matematikai ´ es Fizikai Lapok, 2022/7

433

434

K¨ oz´ episkolai Matematikai ´ es Fizikai Lapok, 2022/7

Mivel sin β  1, megt¨ort (a v´ızben is terjed˝o) hanghull´am csak akkor alakulhat ki, ha sin α 

cleveg˝o = 0,227, cv´ız

´ert´ekkel v´ altozott meg (ennyivel cs¨ okkent). A t¨ olt¨ ott goly´ o elektrosztatikus potenci´alis energi´ aja (a Coulomb-energia)

vagyis α  13,1◦ .

ΔECoulomb = k

qQ s qQ −k = kqQ d−s d d(d − s)

´ert´ekkel n˝ ott. A tiszta g¨ ord¨ ul´es miatt s´ url´od´ asi energiavesztes´eg nincs, ´ıgy az ¨ osszenergia nem v´ altozik meg:

al nagyobb bees´esi sz¨ ogEzek szerint az α0 = 13,1◦ -n´ ben ´erkez˝o hangsug´ar” m´ ar nem hatol be a v´ızbe, ” hanem teljes eg´esz´eben visszaver˝ odik. A v´ızpartt´ ol 2 m´eter t´ avols´ agra l´ev˝ o ember hangja akkor ´erne a v´ızhez α0 -n´ al kisebb sz¨ ogben, ha a horg´asz sz´ aja a talajt´ ol sz´am´ıtva legal´ abb 2m h= = 8,6 m tg α0 magasan lenne. Ilyen magas ember nincs, teh´ at a halakat nem zavarja a horg´aszok besz´elget´ese. Bir´ o Kata (Miskolc, F¨oldes F. Gimn., 9. ´evf.) Megjegyz´es. A teljes visszaver˝ od´es jelens´ege nem azt jelenti, hogy a hull´ am semennyire nem jut be a m´ asik k¨ ozegbe, hanem csak azt, hogy a hull´ am er˝ oss´ege (intenzit´ asa) nagyon er˝ osen (exponenci´ alis u okken, ´ıgy egy bizonyos behatol´ asi m´elys´eget” el´er¨temben) lecs¨ ” ve a hanger˝ oss´eg elhanyagolhat´ oan kicsiv´e v´ alik. A behatol´ asi m´elys´eg nagys´ agrendileg a hanghull´ am hull´ amhossz´ aval egyezik meg, teh´ at sok m´eter is lehet, vagyis elk´epzelhet˝ o, hogy a halakat m´eg a partt´ ol viszonylag messze besz´elget˝ o horg´ aszok hangja is elriaszthatja. (G. P.) 21 dolgozat ´erkezett. Helyes 9 megold´ as. Kicsit hi´ anyos (3 pont) 4, hi´ anyos (1 pont) 1, hib´ as 7 dolgozat.

ΔEhelyzeti + ΔECoulomb = kqQ

s − mgs sin α = 0. d(d − s)

Ennek az egyenletnek s = 0 megold´ asa:

    (9 · 109 Nm2 /C2 ) · 10−5 C · 5,55 · 10−6 C kqQ = (2 m) − = s=d− mgd sin α (0,1 kg) · (9,81 N/kg) · (2 m) · 0,5 = 1,49 m.

A t¨olt¨ott goly´ o teh´ at kb. 1,5 m utat tesz meg a v´aly´ uban a meg´all´ as´ aig. Waldhauser Mikl´ os (Szeged, Radn´oti M. K´ıs´erleti Gimn., 11. ´evf.) 56 dolgozat ´erkezett. Helyes 22 megold´ as. Hi´ anyos (1–2 pont) 15, hib´ as 12, nem versenyszer˝ u 7 dolgozat.

P. 5391. Egy m´ely k´ utba k¨ ovet ejt¨ unk. A csobban´ as hangj´ at 4,25 s-mal az elejt´es ut´ an halljuk meg. Milyen m´elynek tal´ aljuk a kutat, ha g = 10 m/s2 -tel ´es vhang = = 320 m/s-mal sz´ amolunk? Mekkor´ anak ad´ odik a k´ ut m´elys´ege, ha g = 9,81 m/s2 amolunk? (A k¨ ozegellen´ all´ as hat´ as´ at hanyagoljuk el.) tel ´es vhang = 340 m/s-mal sz´ (4 pont)

K¨ ozli: Honyek Gyula, Veresegyh´ az

as pillanat´at´ ol a hang ´erz´ekeMegold´as. Legyen a k˝o es´esi ideje t1 , a csobban´ o k˝ o ´es a hang l´es´eig eltelt id˝ o t2 . Tudjuk, hogy t1 + t2 = T = 4,25 s. A szabadon es˝ ugyanannyi utat tesz meg: g 2 t = vhang (T − t1 ), 2 1

Fizika feladatok megold´asa azaz P. 5386. Egy α = 30◦ -os lejt´es˝ u, d = 2 m´eter hossz´ u, szigetel˝ o anyagb´ ol k´esz¨ ult v´ aly´ u alj´ ahoz Q = 5,55 μC t¨ olt´es˝ u kis goly´ ot r¨ ogz´ıt¨ unk. A v´ aly´ u tetej´er˝ ol m = 100 g t¨ omeg˝ u, q = 10 μC t¨ olt´es˝ u kis goly´ ot enged¨ unk el. Milyen messzire jut el ez a goly´ o, ha tiszt´ an g¨ ord¨ ul? (A mozg´ asa sor´ an a goly´ o t¨ olt´ese nem v´ altozik meg.) (3 pont)

K¨ ozli: Kobzos Ferenc, Duna´ ujv´aros

a) Sz´ amoljunk el˝ osz¨ or g = 10 sm2 ´es vhang = 320 m adatokkal. A t1 -re vonatkoz´ o s m´asodfok´ u egyenlet (m´ert´ekegys´egek n´elk¨ ul): 5 t21 + 320 t1 − 1360 = 0, ut m´elys´eg´ere a amelynek pozit´ıv megold´ asa: t1 = 4 (s), ´es innen a k´

Megold´as. Jel¨ olj¨ uk a t¨ olt¨ ott goly´ o ´altal megtett utat s-sel. A goly´ o helyzeti aig energi´aja az indul´ as´at´ ol a meg´all´as´ ΔEhelyzeti = −mgs sin α K¨ oz´ episkolai Matematikai ´ es Fizikai Lapok, 2022/7

g 2 t + vhang t1 − vhang T = 0. 2 1

h=

g 2 t = 80 m 2 1

eredm´enyt kapjuk. 435

436

K¨ oz´ episkolai Matematikai ´ es Fizikai Lapok, 2022/7

b) Haszn´ aljuk most a pontosabb g = 9,81 sm2 ´es a vhang = 340 m adatokat. s A fenti m´ asodfok´ u egyenlet (m´ert´ekegys´egek n´elk¨ ul) most ´ıgy n´ez ki: 4,905 t21 + 340 t1 − 1445 = 0, aminek a pozit´ıv gy¨ oke: t1 ≈ 4,02 (s). A k´ ut m´elys´ege ilyen adatok mellett kb. 79,2 m-nek ad´ odik.

De azt is tudjuk, hogy a k¨ orlemez szimmetrikus az x ´es az y tengelyek felcser´el´ese, ´es ´ıgy k¨ or k¨ or   1 orlemez 1 2 mi x i = mi yi2 = Θk¨ = ma2 . O 2 4 i i Ugyanezt megtehetj¨ uk y ir´anyban, b/a ar´ any´ u ny´ ujt´ assal b sugar´ u k¨ orr´e alak´ıthatjuk az ellipszis¨ unket:

Elekes Dorottya (Budapest-Fasori Evang´elikus Gimn., 9. ´evf.)

ellipszis 

68 dolgozat ´erkezett. Helyes 47 megold´ as. Kicsit hi´ anyos (3 pont) 4, hi´ anyos (1–2 pont) 10, hib´ as 3, nem versenyszer˝ u 4 dolgozat.

P. 5394. Egy m t¨ omeg˝ u, homog´en t¨ omegeloszl´ as´ u, ellipszis alak´ u lemez f´eltengelyeinek hossza a ´es b. Mekkora a test tehetetlens´egi nyomat´eka a 2a hossz´ us´ ag´ u nagytengely v´egpontj´ an ´ atmen˝ o, a lemez s´ıkj´ ara mer˝ oleges tengelyre vonatkoztatva? (A feladat elemi u ´ton is megoldhat´ o.) (5 pont)

K¨ozli: Gelencs´er Jen˝ o, Kaposv´ ar

Megold´as. Sz´ am´ıtsuk ki el˝ osz¨ or az ellipszisat az O t¨ omeglemez ΘO tehetetlens´egi nyomat´ek´ k¨ oz´eppontj´an (geometriai k¨ oz´eppontj´an) ´ atmen˝ o, a lemez s´ıkj´ara mer˝ oleges tengelyre vonatkoztatva. Osszuk fel – gondolatban – a lemezt omeg˝ u darabk´ akra, amelyek t´ avols´ akicsiny mi t¨ ga a forg´astengelyt˝ ol ri . Ekkor a tehetetlens´egi nyomat´ek defin´ıci´ oja szerint ΘO =

ellipszis 

i

ellipszis 

mi x2i +

i

ellipszis 

mi x2i =

i

K¨ oz´ episkolai Matematikai ´ es Fizikai Lapok, 2022/7

Ezeket ¨osszeadva kapjuk, hogy ΘO =

1 m(a2 + b2 ). 4

A feladat nem ezt, hanem az A ponton a´tmen˝ o tengelyre vonatkoztatott tehetetlens´egi nyomat´ekot k´erdezte, amit a Steiner-t´etel seg´ıts´eg´evel hat´ arozhatunk meg: 1 1 m(a2 + b2 ) + ma2 = m(5a2 + b2 ). 4 4

(5 pont)

K¨ ozli: Gn¨ adig P´eter, V´acduka

Megold´as. Amikor a fonalak fels˝o v´eg´et elt´ avol´ıtjuk egym´ast´ ol, a szimmetrikus elrendez´es miatt a fonalakat fesz´ıt˝ o er˝ ok egyforma nagys´ ag´ uak lesznek: F1 = F2 = F,

mi yi2 .

i

k¨ or 

i

1 2 mb . 4

P. 5401. Egy kicsiny (pontszer˝ unek tekinthet˝ o), de neh´ez testet k´et egyforma hossz´ u, k¨ ozel azonos teherb´ır´ as´ u fon´ alon tartunk. A fonalak fels˝ o v´eg´et egy v´ızszintes egyenes ment´en lassan elt´ avol´ıtjuk egym´ ast´ ol. Amikor a fonalak 2α sz¨ oget z´ arnak be egym´ assal, az egyik fon´ al elszakad, ´es a test a m´ asik fon´ al r¨ ogz´ıtettnek tekinthet˝ o v´ege k¨ or¨ ul ingak´ent lengeni kezd. Mekkora lehetett α, ha a m´ asik fon´ al a leng´esek sor´ an nem szakad el?

Vegy¨ uk ´eszre, hogy a jobb oldal els˝o tagja nem f¨ ugg az yi koordin´ at´ akt´ ol, vagyis hogyha az ellipszist y ir´ anyban a/b ar´ anyban megny´ ujtjuk, teh´ at a sugar´ u k¨ orr´e alak´ıtjuk, az a tag nem fog megv´altozni, vagyis ellipszis 

mi yi2 =

10 dolgozat ´erkezett. Helyes 8 megold´ as. Hi´ anyos (2 pont) 1, hib´ as 1 dolgozat.

Helyezz¨ uk el a lemezt egy olyan s´ıkbeli koordin´ata-rendszerben, amelynek orig´ oja a t¨ omegk¨ oz´eppont, tengelyei pedig p´ arhuzamosak az ellipszis tengelyeivel. all Mivel a Pitagorasz-t´etel szerint ri2 = x2i + yi2 , fenn´ mi ri2 =

k¨ or 

´ Isk. ´es Gimn., 11. ´evf.) G´ abriel Tam´ as (Budapesti Fazekas M. Gyak. Alt.

mi ri2 .

ellipszis 

i

ΘA = ΘO + ma2 =

i

ΘO =

mi yi2 =

ahol F a fonalak sz¨ og´enek n¨ ovekedt´evel egyre nagyobb lesz. Ha a kicsiny test t¨ omege m, akkor a f¨ ugg˝oleges er˝ ok egyens´ ulya miatt az elszakad´ ast megel˝oz˝ o pillanatban (1. ´ abra) mg = 2F cos α, vagyis a fonalak szak´ıt´ oszil´ ards´ aga” ” (1)

mi x2i .

F =

mg . 2 cos α

i

437

438

K¨ oz´ episkolai Matematikai ´ es Fizikai Lapok, 2022/7

vagyis −4 cos2 α + 6 cos α − 1 < 0. Ez x = cos α-ra n´ezve m´ asodfok´ u egyenl˝ otlens´eg: f (x) ≡ −4x2 + 6x − 1 < 0. 3. ´ abra 1. a ´bra

√ 3+ 5 ≈ 1,3 x1 = 4

2. a ´bra

(A bek¨ uld¨ ott dolgozat nem tartalmazott a ´br´ akat, azokkal a jobb ´erthet˝ os´eg kedv´e´ert eg´esz´ıtett¨ uk ki a megold´ ast. – A szerk.)

Az egyik fon´al elszakad´asa ut´ an a m´ asik test  hossz´ us´ag´ u matematikai ingak´ent lengeni kezd. Amikor a fon´ al ϕ sz¨ oget z´ ar be a f¨ ugg˝olegessel (2. ´ abra), a test v sebess´ege az energiamegmarad´ as t´etele seg´ıts´eg´evel sz´ amolhat´ o: mgh = mg(cos ϕ − cos α) = ahonnan v=

1 mv 2 , 2

acp =

 2g(cos ϕ − cos α).

´es

an ´ertelmetlen), illetve Az egyenl˝otlens´eg megold´ asa: cos α > x1 > 1 (ez nyilv´ cos α < x2 ≈ 0,19,

vagyis

α > arccos x2 ≈ 79◦ .

Ha teh´at a sz´eth´ uzott fonalak sz¨ oge az egyik fon´al elszakad´as´ anak pillanat´aban abbi leng´esek sor´ an a m´ asik fon´ al biztosan nem fog legal´ abb 158◦ , akkor a tov´ elszakadni.

39 dolgozat ´erkezett. Helyes 22 megold´ as. Kicsit hi´ anyos (4 pont) 7, hi´ anyos (1–3 pont) 4, hib´ as 3, nem versenyszer˝ u 3 dolgozat.

v2 = 2g(cos ϕ − cos α), 

a neh´ezs´egi er˝ o fon´ alir´any´ u komponense pedig mg cos ϕ. Ha a fonalat valamekkora K er˝ o fesz´ıti, akkor a fon´ alir´any´ u mozg´as dinamikai egyenlete: K − mg cos ϕ = macp ,

P. 5407. A CERN egyik line´ aris gyors´ıt´ oj´ aban kezdetben ´ all´ onak tekinthet˝ o protonokat gyors´ıtanak L = 30,0 m hossz´ uu ´ton U = 500 MV fesz¨ ults´eggel. Feltehetj¨ uk, hogy a gyors´ıt´ oban az elektromos t´er homog´en. Mennyi id˝ o alatt teszik meg a protonok az L t´ avols´ agot? (5 pont)

Sv´ ajci versenyfeladat

Megold´as. Newton 2. t¨ orv´enye szerint (ami ebben az alakj´ aban a relativisztikus fizik´aban is ´erv´enyes):

vagyis (2) felhaszn´ al´as´aval K(ϕ) = mg(3 cos ϕ − 2 cos α).

F =

L´atszik, hogy a fon´alban ϕ = 0-n´ al ´ebred a legnagyobb er˝ o, vagyis amikor a kis test ´eppen a p´ alya legals´ o pontj´an halad ´at: Kmax = mg(3 − 2 cos α). A fon´ al biztosan nem szakad el, ha Kmax kisebb, mint az (1) a´ltal megadott szak´ıt´ oszil´ ards´ag: mg , mg(3 − 2 cos α) < 2 cos α

K¨ oz´ episkolai Matematikai ´ es Fizikai Lapok, 2022/7

√ 3− 5 x2 = ≈ 0,19. 4

Beke B´ alint (Budapest, ELTE Ap´ aczai Csere J. Gyak. Gimn. ´es Koll., Gimn., 11. ´evf.)

Ebben a helyzetben a test centripet´ alis gyorsul´asa: (2)

Mivel a f˝o egy¨ utthat´ o negat´ıv, a f¨ uggv´eny k´epe egy alul nyitott parabola (3. ´ abra), melynek z´erushelyei:

439

Δp , Δt

ahol p a relativisztikus impulzus, F = e U olt´es˝ u protont gyors´ıt´ o er˝ o. L pedig az e t¨ Mivel homog´en elektromos er˝ot´erben F ´ alland´ o, tov´ abb´ a a protonok kezd˝ osebess´ege null´anak tekinthet˝o, a gyors´ıt´as ideje: T =

pL , eU

ahol p a felgyors´ıtott proton impulzusa. Feladatunk teh´ at az, hogy ezt az impulzust meghat´arozzuk; bel˝ ole a gyors´ıt´ as ideje k¨ onnyen kisz´ am´ıthat´ o. 440

K¨ oz´ episkolai Matematikai ´ es Fizikai Lapok, 2022/7

A proton (nyugalmi) energi´aja kezdetben E0 = m0 c2 , a gyors´ıt´ as v´eg´en pedig E = E0 + eU = mc2 ,

Fizik´ab´ ol kit˝ uzo¨tt feladatok

ahol m (a szok´ asos sz´ohaszn´ alattal) a proton megn¨ovekedett t¨omege”, c pedig ” a v´akuumbeli f´enysebess´eg. A megadott ´es t´ abl´ azatb´ ol kikereshet˝o adatok szerint m = m0 +

eU = 2,56 · 10−27 kg = 1,53 m0 . c2

M. 416. Akasszunk egy kilincs v´eg´ere egy kis m´eret˝ u t´ argyat, ´es fokozatosan n¨ovelj¨ uk a terhel´est tov´abbi testek r´ ahelyez´es´evel. M´erj¨ uk meg a kilincs egyens´ ulyi sz¨oghelyzet´et a r´ aakasztott t¨ omeg f¨ uggv´eny´eben! A lehets´eges legnagyobb sz¨ og el´er´ese ut´ an fokozatosan cs¨ okkents¨ uk a terhel´est, ´es m´erj¨ uk meg ebben az esetben is a sz¨oghelyzetet a t¨ omeg f¨ uggv´eny´eben. Adatainkat ugyanazon a grafikonon ´abr´ azoljuk!

M´ asr´eszt igaz, hogy m= 

m0 1 − v 2 /c2

,

(6 pont)

ahonnan a felgyors´ıtott proton v´egsebess´ege: v=c



G. 789. Becs¨ ulj¨ uk meg, hogy mennyivel cs¨ okken test¨ unk t¨ omege, mik¨ ozben nyolc o´r´ an kereszt¨ ul, egyfolyt´ aban, nyugodtan alszunk! Haszn´ aljuk a k¨ ovetkez˝ o adatokat:

m m2 1 − 02 = 2,274 · 108 . m s

– Egy l´elegzetv´etel sor´ an 0,5 l leveg˝o cser´el˝odik ki. – Percenk´ent 15-¨ ot l´elegz¨ unk.

Ennek megfelel˝ oen a keresett id˝ otartam: T =

mvL (2,56 · 10−27 kg) · (2,274 · 108 m/s) · (30 m) = = 2,18 · 10−7 s. eU (1,6 · 10−19 C) · (500 · 106 V)

– A kil´elegzett leveg˝o sz´en-dioxid-tartalma 5 V/V%. – A kil´elegzett leveg˝o 6 V/V% v´ızg˝ozt tartalmaz.

A relativisztikus impulzust (a sebess´eg kisz´am´ıt´ asa n´elk¨ ul) az energia-impulzus rel´aci´ ob´ ol is kisz´am´ıthatjuk: 2

2

2

E 2 = (m0 c2 + eU ) = (m0 c2 ) + (pc) , ahonnan p= a keresett id˝ o pedig L T = c



2m0 eU +

1+



eU c

2

(3 pont) G. 790. Egy aut´ o´ atlagos fogyaszt´ asa 6 liter/100 km. Teljesen u os, ¨res, kanyarg´ k´ets´ avos aut´op´ aly´ an 300 km-t tesz meg a g´epkocsi. Az u ´tszakaszon a kanyarok o¨sszhossza 50 km, a kanyarok sugara a´tlagosan 1 km, a s´avok sz´eless´ege pedig 4 m. Becs¨ ulj¨ uk meg, hogy mennyivel cs¨ okken a j´ arm˝ u fogyaszt´ asa, ha egy felel˝otlen sof˝or minden kanyart a bels˝ o ´ıven tesz meg! (3 pont)

,

2m0 c2 = 218 ns. eU

K¨ urti Gergely (Kiskunf´elegyh´ azi Szent Benedek PG K´et Tan´ıt´ asi Nyelv˝ u Technikum ´es Koll., 12. ´evf.) ´es T´egl´ as Panna (R´evkom´arom, Selye J´ anos Gimn., 12. ´evf.) dolgozata alapj´an 25 dolgozat ´erkezett. Helyes Bencz Benedek, G´ abriel Tam´ as, Hauber Henrik, K¨ urti Gergely, T´egl´ as Pannha ´es Toronyi Andr´ as megold´ asa. Kicsit hi´ anyos (4 pont) 2, hi´ anyos (1–3 pont) 11, hib´ as 4, nem versenyszer˝ u 2 dolgozat.

K¨ oz´ episkolai Matematikai ´ es Fizikai Lapok, 2022/7

K¨ ozli: Sz´ asz Kriszti´ an, Budapest

441

G. 791. Zsongl˝ ork¨ od˝ o elm´eleti fizikus a k¨ ovetkez˝ o mutatv´anyt tal´ alja ki. Egym´as tetej´ere helyez n sz´ am´ u, t¨ ok´eletesen rugalmas labd´ at, k¨ oz¨ ott¨ uk igen kicsiny r´esekkel. A labdatornyot kem´eny fel¨ uletre ejti, ahov´a a labd´ ak v sebess´eggel ´erkeznek meg. A sorozatos pillanatszer˝ uu oz´esek ut´ an a legfels˝o labda kiv´etel´evel ¨tk¨ minden lejjebb l´ev˝ o labda meg´all, a legfels˝ o viszont nv sebess´eggel pattan fel. Bizony´ıtsuk be (p´eld´ aul a teljes indukci´ o m´ odszer´evel), hogy ez a mutatv´ any akkor teljes¨ ulhet, ha a labd´ ak t¨ omege kiel´eg´ıti a k¨ ovetkez˝ o formul´ at: mk =

2m0 , k(1 + k)

ahol m0 a legals´ o labda t¨ omege, valamint k = 1, 2, 3, . . . , (n − 1), n.

(4 pont) 442

K¨ oz´ episkolai Matematikai ´ es Fizikai Lapok, 2022/7

P. 5429. Egy elektromos aut´ o ´ all´ o helyzetb˝ ol indulva 10 s alatt egyenletes gyorsul´assal 108 km/h sebess´eget ´er el. Kerekeinek sugara 0,4 m, a ker´ekt´ arcs´ an tal´ alhat´ o egy 0,2 m sugar´ u d´ısz´ıt˝ ogy˝ ur˝ u. Az indul´ ast´ ol sz´am´ıtva mennyi id˝ o m´ ulva lesz ennek a v´ekony gy˝ ur˝ unek olyan pontja, amely nem gyorsul? Mekkora ebben a pillanatban az aut´o sebess´ege?

G. 792. Az ´ abr´ an l´ athat´ o ´aramk¨ orben egyforma izz´ ol´amp´ ak vannak. A kapcsol´o z´ ar´ as´ at k¨ ovet˝oen az A ´es a B l´ ampa f´enyesebben vagy halv´ anyabban fog vil´ag´ıtani? (Tekints¨ unk el az izz´ol´amp´ ak ellen´ all´ as´ anak h˝ om´ers´ekletf¨ ugg´es´et˝ ol.)

(5 pont)

K¨ ozli: Gn¨ adig P´eter, V´acduka

(3 pont) P. 5427. A fenti ´ abr´ an l´ athat´ o´ aramk¨ orben tal´ alhat´ o 230 V n´evleges fesz¨ ults´eg˝ u volfr´ amsz´ alas izz´ol´amp´ ak egyform´ak, melyeknek a´ram–fesz¨ ults´eg karakterisztik´aj´at az al´abbi grafikon mutatja. Az ´aramk¨ orben tal´ alhat´ o fesz¨ ults´egforr´ as 230 V-os.

P. 5430. A kl´ımav´ altoz´ as egyik okak´ent szokt´ ak eml´ıteni az intenz´ıv h´ uster” mel´est”. Egy szarvasmarha naponta 160-320 liter met´ang´azt bocs´ at ki. A glob´alis ´allatteny´eszt´esben egymilli´ ard marh´at tartanak nyilv´an. Becs¨ ulj¨ uk meg, hogy milyen vastag r´eteget alkotna az a´llatok egy ´evi met´ang´az-kibocs´ at´ asa a F¨ old felsz´ın´en! (A F¨oldet tekints¨ uk 6370 km sugar´ u g¨ ombnek.) K¨ ozli: Simon P´eter, P´ecs

(3 pont)

P. 5431. Egy 10 cm sugar´ u, g¨ omb alak´ u, homog´en, t¨ om¨ or testnek egy bizonyos t tengelyre vonatkoztatott tehetetlens´egi nyomat´eka 10%-kal nagyobb, mint a g¨omb lehet˝o legkisebb tehetetlens´egi nyomat´eka. Milyen messze van a t egyenes a g¨omb k¨oz´eppontj´at´ ol? (L´ asd m´eg a Merev testek mozg´ asegyenletei” c. r¨ ovid ” cikket a K¨oMaL honlapj´an∗ .) (3 pont)

K¨ ozli: Tornyos Tivadar E¨ ors, Budapest

P. 5432. Egy f¨ ugg˝oleges helyzetben r¨ ogz´ıtett, v´ekony szigetel˝op´alc´ ara h´ arom egyforma t¨omeg˝ u ´es egyenl˝ o t¨ olt´es˝ u szigetel˝ ogy¨ ongy¨ ot f˝ uzt¨ unk fel. Az als´o gy¨ ongy r¨ogz´ıtett, a f¨ ol¨ otte l´ev˝ o m´ asik kett˝ o szabadon elcs´ uszhat a p´alc´ an. Egyens´ ulyi helyzetben h´ anyszor messzebb van a legfels˝o gy¨ ongy a k¨ oz´eps˝ ot˝ ol, mint a k¨ oz´eps˝ o a legals´ ot´ ol? (5 pont) 100 W-os villanyk¨ orte ´ aram–fesz¨ ults´eg karakterisztik´ aja

a) Mekkora egy ilyen izz´ ol´ampa elektromos ellen´ all´asa a n´evleges fesz¨ ults´egen? b) Mekkora ellen´ all´ as´ u az A ´es a B l´ampa izz´ osz´ ala a kapcsol´ o nyitott a´ll´as´ aban? c) Mekkora ellen´ all´ as´ uak az izz´ osz´ alak a kapcsol´o z´ar´ asa ut´ an? d) Mekkora teljes´ıtm´enyt adnak le az izz´ ol´amp´ ak a fenti esetekben? (5 pont)

K¨ ozli: Honyek Gyula, Veresegyh´az

P. 5428. K´epzelj¨ uk el, hogy nap-´ej egyenl˝ os´eg idej´en egy egyenl´ıt˝oi orsz´ag tengerpartj´ an hason feksz¨ unk a homokban, ´es figyelj¨ uk a naplement´et. A tenger t¨ uk¨orsima, az ´eg tiszta k´ek, ´es abban a pillanatban, amikor a Nap utols´ o sugara elt˝ unik a horizonton, hirtelen fel´ allunk, ´es ´ıgy m´eg u ´jra l´ athatjuk a Nap fels˝o karim´ aj´at. Becs¨ ulj¨ uk meg, hogy fel´all´ asunk ut´ an mennyi id˝ ovel t˝ unik el u ´jra a napkorong! (4 pont) K¨ oz´ episkolai Matematikai ´ es Fizikai Lapok, 2022/7

K¨ ozli: Holics L´ aszl´ o, Budapest

P. 5433. Egy v´ızzel t¨ olt¨ ott, t´eglatest alak´ u, elhanyagolhat´ o falvastags´ ag´ u akv´ arium h´ arom f¨ ugg˝oleges oldala a v´ızb˝ ol r´ ajuk es˝ o f´enyt visszaveri. Az akv´arium sz´eless´ege d = 50 cm, hossza L = 120 cm. Az akv´arium r¨ ovidebb oldal´ ahoz v´ızszintes s´ıkban valamekkora bees´esi sz¨ ogben l´ezersug´ar ´erkezik. Az ´ abra fel¨ uln´ezetet mutat. (A v´ız t¨ or´esmutat´ oja: n = 4/3.) A kil´ep˝ o f´enysug´ ar – t¨ obbsz¨ ori t¨ ukr¨ oz˝ od´es ut´ an – ´eppen a bees˝ o f´enysug´arral p´ arhuzamosan haladva hagyja el az akv´ ariumot. Legfeljebb h´ any t¨ ukr¨ oz˝ od´es t¨ort´enhetett? (5 pont)

Amerikai feladat nyom´an



443

444

an Zsigri Ferenc (Budapest) feladata nyom´

https://www.komal.hu/cikkek/cikklista.h.shtml

K¨ oz´ episkolai Matematikai ´ es Fizikai Lapok, 2022/7

P. 5434. Az ´ abr´ an szerepl˝ o fesz¨ ults´egforr´ as elektromotoros ereje bekapcsol´ as ut´an id˝oben line´ arisan n¨ovekszik fel a kezdeti 0 voltos ´ert´ekr˝ ol; U (t) = U0 tt . 0

A K kapcsol´ o seg´ıts´eg´evel b´ armelyik pillanatban r´ akapcsolhat´ o a fesz¨ ults´egforr´ as az a´ramk¨ orre. A fesz¨ ults´egforr´ as bekapcsol´ asa ut´ an mennyi id˝ ovel kell z´ arni a kapcsol´ot, hogy ezut´ an az ellen´all´ ason ´atfoly´ o ´aram er˝ oss´ege id˝ oben line´arison n˝oj¨ on? Milyen u ¨temben n˝o ekkor az a´ramer˝ oss´eg? (5 pont) K¨ ozli: Sz´echenyi G´ abor, Budapest P. 5435. Egy cs˝ o bels˝ o sugara R, tengelye α sz¨oget z´ ar be a v´ızszintessel. A cs¨ ovet a´lland´ o ω sz¨ogsebess´eggel forgatjuk a tengelye k¨or¨ ul. A cs˝ obe egy pontszer˝ unek tekinthet˝ o, kicsiny testet helyez¨ unk. A cs˝ o fala ´es a kis test k¨ oz¨ otti cs´ usz´asi s´ url´od´ asi egy¨ utthat´ o μ (μ > tg α). Azt tapasztaljuk, hogy kell˝oen hossz´ u id˝ o eltelt´evel a kis test egyenes vonal´ u egyenletes mozg´ ast v´egez. Mekkora a mozg´ as sebess´ege? (6 pont)

K¨ozli: Balogh P´eter, G¨od¨oll˝ o Beku ¨ld´esi hat´arid˝o: 2022. november 15. Elektronikus munkafu ¨zet: https://www.komal.hu/munkafuzet

MATHEMATICAL AND PHYSICAL JOURNAL FOR SECONDARY SCHOOLS (Volume 72. No. 7. October 2022)

Problems in Mathematics New exercises for practice – competition K (see page 415): K. 734. Alex and his friends bought 6 bags of sunflower seeds and 4 bags of pumpkin seeds for 1900 HUF. Next week they bought 4 bags of sunflower seeds and 2 bags of pumpkin seeds for 1100 HUF. How much does a single bag of each type cost (assuming that the prices did not change during the week)? K. 735. Logic blocks were developed by Zolt´ an Dienes. Peter takes all the red and green disks and squares out of a set of blocks, altogether 16 pieces. No two pieces are identical, and they can be classified into two groups having the same number of elements according to each of the following properties: – either small or large, – either red or green, – either a disk or a square, – either hollow or not. Can Peter place the 16 blocks along the perimeter of a circle such that any two neighbors have exactly one of the above properties in common? K. 736. A company has 120 employees: plumbers, tilers, bricklayers and painters. All plumbers and bricklayers have a driving license, the others do not. The bricklayers and painters work in Pipacs street, the others in Kankalin street. The number of employees without a driving license is 64, and 84 employees work in Kankalin street. There are twice as many plumbers as painters. How many employees of each kind are there at the company? K/C. 737. Given two threads of known length, we K¨ oz´ episkolai Matematikai ´ es Fizikai Lapok, 2022/7

445

can measure and mark off the sum or difference of their lengths, and also the half length of a thread by folding it into two. We are given two threads of length 2240 cm and 1760 cm. Describe a procedure to mark off a length of 10 cm by using a single measurement (that is, measuring the sum or difference of lengths is allowed only once, but halving a length by folding can be performed many times). K/C. 738. In a certain calendar, the days of a month are arranged in 7 columns. Read from left to right and then from top to bottom, each column contains the same day of the week. For a certain integer n, we select an n × n square array of days and find that their sum is 198. What is the smallest number in this square? New exercises for practice – competition C (see page 416): Exercises up to grade 10: K/C. 737. See the text at Exercises K. K/C. 738. See the text at Exercises K. Exercises for everyone: C. 1733. At most how many different positive prime divisors can a 3-digit number have, if its digits are consecutive positive integers in a certain order? (Based on the idea of Erzs´ebet Berk´ o, Szolnok) C. 1734. A circle k with diameter AB has center O. We draw the circle k1 with diameter OB, and the line parallel with AB that touches the circle k1 at point C. This line intersects the circle k at points D1 and D2 . Determine the alint B´ır´ o, Eger) C. 1735. Find the angles ∠COD1 and ∠COD2 exactly. (Proposed by B´ √ √ 1 1 5 real solutions of the system x + y = 6, x + y = 16 . (The Mathematical Association of America) Exercises upwards of grade 11: C. 1736. Let P be an interior point of side CD of a parallelogram ABCD, and let Q be an interior point of side AB (being parallel with CD). The line segments P A and QD intersect each other at M , while the line segments P B and QC intersect each other at N . Find a condition to have M N  AB. (Based on a U.S. mathematics competition problem) C. 1737. Dick got two dice for his 32th birthday. He labelled the faces of one die with the numbers 1, 2, . . . , 6, and the faces of the other one with 0, 1, 2, 7, 8, 9. By using these dice, he can form all integers from 10 up to his age, 32, but the next number, 33, cannot be formed. Octavia uses two regular octahedra instead. Similarly, she wrote a digit on each face of both octahedra, so she can also form all integers from 10 up to her current age (in years), but not the next one. How old is Octavia now? (Proposed by Katalin Abig´el Kozma, Gy˝ or) New exercises – competition B (see page 418): B. 5262. Louisa wrote down a natural number, not containing 0 but containing at least two different digits. Then she also listed all the numbers which can be formed by permuting the digits of the original number. What is the maximum of the greatest common divisor of all the numbers (including the original one)? (3 points) (Proposed by Katalin Abig´el Kozma, Gy˝ or) B. 5263. Prove that the sum of the squares of the medians of a triangle is not less than the square of the semiperimeter of the triangle. (3 points) (Proposed by L´ aszl´ o N´emeth, Fony´ od) B. 5264. First Player and Second Player play the following game. First Player starts and prescribes arbitrarily many (even infinitely many) terms of a binary sequence (i.e., any term is 0 or 1) in a way that infinitely many terms can still be determined. Then Second Player sets the value of the first digit which has not been prescribed yet. They then repeat this procedure forever by taking turns. First Player wins if the binary sequence is periodic from a certain term, otherwise, Second Player wins. Is there a winning strategy, and if yes, who has it? (4 points) (Proposed by P´eter P´ al Pach, Budapest) B. 5265. Enlarge the incircle of a rightangled triangle by a scale factor of 2, where the center of enlargement is the vertex at the right angle. Show that this circle touches the circumcircle of the triangle. (4 points) (Proposed by Viktor V´ıgh, Szeged) B. 5266. Some football players are on holiday together. Altogether they are from k clubs and from n nations where k < n. Show that there are at least n − k + 1 players having more club fellows than compatriots. (5 points) B. 5267. We are given two line segments of length p and q, and a triangle ABC determined by the

446

K¨ oz´ episkolai Matematikai ´ es Fizikai Lapok, 2022/7

lines a, b and c (where the usual convention is used: points B and C lie on line a, and so on). Construct the point P on the circumcircle of the triangle for which the point Pa divides the line segment Pb Pc in a ratio p : q, where Px is the orthogonal projection of the point P onto the line x. (5 points) B. 5268. Let I denote the incenter of the triangle ABC. Let P denote an arbitrary interior point of the triangle on the circle ABI. The reflection of the line AP about the line AI intersects the circle ABI at a point Q different from the point A. Prove that CP = CQ. (6 points) (Proposed by Szilveszter Kocsis, Budapest) B. 5269. Let p  19 be an odd integer, and color the numbers 0, 1, . . . , p − 1 with two colors. For 1  i  p, let xi denote a random element of the set {0, 1, . . . , p − 1} (the choices are independent, and have uniform distribution). Prove that the probability of the event that x1 , . . . , xp have the same color and p divides x1 + · · · + xp is at least 3/(2p p). (6 points) (Proposed by P´eter P´ al Pach, Budapest) New problems – competition A (see page 419): A. 833. Some lattice points in the Cartesian coordinate system are colored red, the rest of the lattice points are colored blue. Such a coloring is called finitely universal, if for any finite, non-empty A ⊂ Z there exists k ∈ Z such that the point (x, k) is colored red if and only if x ∈ A. a) Does there exist a finitely universal coloring such that each row has finitely many lattice points colored red, each row is colored differently, and the set of lattice points colored red is connected? b) Does there exist a finitely universal coloring such that each row has a finite number of lattice points colored red, and both the set of lattice points colored red and the set of lattice points colored blue are connected? A set H of lattice points is called connected if, for any x, y ∈ H, there exists a path along the grid lines that passes only through lattice points in H and connects x to y. (Submitted by Anett Kocsis, Budapest) A. 834. Let A1 A2 . . . A8 be a convex cyclic octagon, and for i = 1, 2 . . . , 8 let Bi = Ai Ai+3 ∩ Ai+1 Ai+4 (indices are meant modulo 8). Prove that points B1 , . . . , B8 lie on the same conic section. A. 835. Let f (n) (x) denote the nth iterate of function f , i.e f (1) (x) = f (x), f (n+1) (x) = f (f (n) (x)). Let p(n) be a given polynomial with integer coefficients, which maps the positive integers into the positive integers. Is it possible that the functional equation f (n) (n) = p(n) has exactly one solution f that maps the positive integers into the positive integers? (Submitted by D´ avid Matolcsi andKrist´ of Szab´ o, Budapest)

Problems in Physics (see page 442) M. 416. Hang a small object on the end of a door handle and gradually increase the load, with placing additional bodies on the load. Measure the angular position of the handle in equilibrium as a function of the mass hung on it. After reaching the maximum possible angle, gradually reduce the load and measure the angular position of the handle as a function of mass. Plot your data on the same graph. G. 789. Estimate how much your body mass decreases while you sleep for eight hours peacefully! Use the following data: • During one breath 0.5 l of air is exchanged. • We breathe 15 times in a minute. • The carbon dioxide content of the exhaled air is 5 V/V%. • The exhaled air contains 6 V/V% water vapour. G. 790. The average consumption of a car is 6 litres/100 km. On a completely empty, winding two-lane road the car can travel 300 km. The total length of the curves on this road segment is 50 km, the radius of the curves is on average 1 km and the width of the lanes is 4 m. Estimate the reduction in fuel consumption if a careless driver makes every turn on the inside curve. G. 791. A juggling theoretical physicist invents the following stunt. He puts n perfectly elastic balls on top of each other with very small gaps between them. He drops the ball tower onto a hard surface, where the balls arrive at a speed of v. After a series of momentary collisions, all

K¨ oz´ episkolai Matematikai ´ es Fizikai Lapok, 2022/7

447

the balls except the top ball stop, and the top ball bounces up at a speed of nv. Prove (for example using the method of mathematical induction) that this stunt can be performed 2m0 , where m0 is the if the mass of the balls satisfies the following formula: mk = k(1+k) mass of the lowermost ball and k = 1, 2, 3, . . . , (n − 1), n. G. 792. In the circuit shown in the figure below there are identical incandescent lamps. After the switch is closed, will lamp A or B be brighter or dimmer? (Do not consider the temperature dependence of the resistances of the filament lamps.) P. 5427. The voltage rating of each of the identical tungsten filament incandescent lamp of the circuit shown in the figure above is 230 V. Their current-voltage characteristics are shown in the graph below. The voltage supply in the circuit is 230 V. a) What is the resistance of an incandescent lamp at its rated voltage? b) What is the resistance of the filaments of lamps A and B in the open position of the switch? c) What is the resistance of the filaments after the switch is closed? d) How much power is dissipated by each incandescent lamp in the above cases? P. 5428. Imagine that on a day of an equinox you are lying in the sand on the beach of an equatorial country and observe the sunset. The sea is very smooth, the sky is clear blue, and at the moment when the last ray of the Sun disappears over the horizon, you suddenly stand up, so you can see the Sun’s upper rim again. Estimate how long it takes for the Sun after you stand up to disappear again. P. 5429. An electric car accelerates uniformly from rest and reaches a speed of 108 km/h in 10 s. The radius of its wheels is 0.4 m, on the wheel there is a decorating ring of radius 0.2 m. How much time elapses from the start of the car until this narrow decorating ring will have a point which does not accelerate? What is the speed of the car at this moment? P. 5430. “Intensive meat production” is often cited as one of the causes of climate change. A single cow emits 160-320 litres of methane per day. There are one billion cattle in global livestock production. Estimate the thickness of the methane layer that would be formed on the surface of the Earth. (Consider the Earth as a sphere of radius 6370 km.) P. 5431. The rotational inertia of a spherical, uniform-density solid body of radius 10 cm with respect to a certain axis t is 10% greater than the minimum possible rotational of inertia of the sphere. How far is the axis t from the centre of the sphere? P. 5432. Three isolating beads having the same mass and given the same charge are stringed to a thin insulating stick fixed in a vertical position. The bottom bead is fixed and the above two beads are free to slide on the stick. At equilibrium, how many times further is the top bead from the middle bead than the middle bead from the bottom bead? P. 5433. The three vertical sides of a cuboid-shaped aquarium filled with water, with negligible wall thickness, reflect the light from the water. The aquarium has a width of d = 50 cm and a length of L = 120 cm. A horizontal laser beam is incident on the shorter side of the aquarium at a certain angle of incidence. The figure shows the top view. (The refraction index of water is n = 4/3.) The light beam – after being reflected several times – emerges from the aquarium parallel to the original incident light beam. At most how many reflections could occur? P. 5434. The electromotive force of the voltage supply shown in the figure increases linearly in t time after switching on, from an initial value of 0 volts; U (t) = U0 t . The switch K can 0 be closed at any moment to connect the voltage source to the circuit. After the voltage source is turned on, how much time should elapse till the switch is closed so that the current flowing through the resistor also increases linearly in time? At what rate does the current increase then? P. 5435. A tube has an internal radius of R, and its axis makes an angle of α with the horizontal. The tube is rotated at a constant angular speed of ω about its axis. A small point-like body is inserted into the tube. The coefficient of kinetic friction between the wall of the tube and the small body is μ (μ > tg α). We find that after a sufficiently long time the small body undergoes uniform straight line motion. What is the speed of the motion?

72. ´evfolyam 7. sz´am

K¨oMaL

Budapest, 2022. okt´ober

´ ´ FIZIKAI LAPOK ¨ EPISKOLAI KOZ MATEMATIKAI ES ˝ ´ITVE INFORMATIKA ROVATTAL BOV

A 63. Nemzetk¨ozi Matematikai Di´akolimpia feladatainak megold´asa II.

´ ALAP´ITOTTA: ARANY DANIEL 1894-ben 72. ´evfolyam 8. sz´am

Budapest, 2022. november

´ Megjelenik ´evente 9 sz´amban, janu´art´ol m´ajusig ´es szeptembert˝ol decemberig havonta 64 oldalon. ARA: 1100 Ft

´ TARTALOMJEGYZEK A 63. Nemzetk¨ozi Matematikai Di´akolimpia feladatainak megold´asa II. . . . . . . . . . . . . . . . . . . . . . . .

450

T´othm´er´esz Lilla: N´egysz´ın-sejt´es III: A sz´ınez´esi polinom, avagy mi´ert olyan neh´ez a n´egysz´ınt´etel . . . . . . . . . . . . . . . . . . . . . . . . . . . . . . . . . . . . . . . . . .

456

Helyesb´ıt´es ´es k¨ozlem´eny . . . . . . . . . . . . . . . . . . . . . . . .

461

Az a´ltal´anos iskolai tan´arok verseny´enek feladatai

461

J´ocsik Csilla: Gyakorl´o feladatsor emelt szint˝u matematika ´eretts´egire . . . . . . . . . . . . . . . . . . . . . . . .

465

N´emeth L´aszl´o: Megold´asv´azlatok a 2022/7. sz´am emelt szint˝u matematika gyakorl´o feladatsor´ahoz . . . . . . . . . . . . . . . . . . . . . . . . . . . . . . . . . . . . . . .

467

Matematika feladatok megold´asa (5109., 5226., 5244.) . . . . . . . . . . . . . . . . . . . . . . . . . . . . . . . . . . . . . . . .

475

A K pontversenyben kit˝uz¨ott gyakorlatok (739– 743.) . . . . . . . . . . . . . . . . . . . . . . . . . . . . . . . . . . . . . . . . .

480

A C pontversenyben kit˝uz¨ott gyakorlatok (742– 743., 1738–1742.) . . . . . . . . . . . . . . . . . . . . . . . . . . . . .

480

A B pontversenyben kit˝uz¨ott feladatok (5270– 5277.) . . . . . . . . . . . . . . . . . . . . . . . . . . . . . . . . . . . . . . . .

482

Az A pontversenyben kit˝uz¨ott nehezebb feladatok (836–838.) . . . . . . . . . . . . . . . . . . . . . . . . . . . . . . . . . . . .

483

Informatik´ab´ol kit˝uz¨ott feladatok (574–576., 66., 165.) . . . . . . . . . . . . . . . . . . . . . . . . . . . . . . . . . . . . . . . . .

484

Woynarovich Ferenc: A r¨ontgensz´or´as, m´as n´even Bragg-reflexi´o . . . . . . . . . . . . . . . . . . . . . . . . . . . . . . . .

489

Fizika gyakorlatok megold´asa (781., 782., 784.) . . .

491

Fizika feladatok megold´asa (5400., 5404., 5405., 5409., 5410., 5412., 5413., 5414., 5415.) . . . . . . . .

495

Fizik´ab´ol kit˝uz¨ott feladatok (417., 793–796., 5436–5444.) . . . . . . . . . . . . . . . . . . . . . . . . . . . . . . . . . .

506

Problems in Mathematics . . . . . . . . . . . . . . . . . . . . . . . .

509

Problems in Physics . . . . . . . . . . . . . . . . . . . . . . . . . . . . .

511

K¨ oz´ episkolai Matematikai ´ es Fizikai Lapok, 2022/8

´ EVA ´ F˝oszerkeszt˝o: RATKO ´ ¨ Fizikus szerkeszt˝o: GNADIG PETER ´ ILDIKO ´ M˝uszaki szerkeszt˝o: MIKLOS Bor´ıt´o: BURGHARDT ZSUZSA ´ Kiadja: MATFUND ALAP´ITVANY ´ RITA Alap´ıtv´anyi k´epvisel˝o: KOS Felel˝os kiad´o: KATONA GYULA Nyomda: OOK-PRESS Kft. ´ Felel˝os vezet˝o: SZATHMARY ATTILA INDEX: 25 450 ISSN 1215-9247 A matematika bizotts´ag vezet˝oje: ´ HERMANN PETER ´ BALINT, ´ ´ Tagjai: B´IRO GYENES ZOLTAN, ´ ´ KISS HUJTER BALINT, IMOLAY ANDRAS, ´ ´ GEZA, ´ ´ GEZA, KOS KOZMA KATALIN ABIGEL, ´ ´ ´ PACH ¨ ORDI ¨ MATOLCSI DAVID, OK PETERN E, ´ ´ SZMERKA GERGELY, V´IGH PETER PAL, VIKTOR A fizika bizotts´ag tiszteletbeli eln¨oke: ´ ´ HOLICS LASZL O ´ Tagjai: BARANYAI KLARA, HONYEK GYULA, ´ ´ KRISZTIAN, ´ OLOSZ BALAZS, SZASZ ´ ´ ´ E, ´ VLADAR ´ SZECHENYI GABOR, VIGH MAT ´ KAROLY, WOYNAROVICH FERENC Az informatika bizotts´ag vezet˝oje: ´ ´ SCHMIEDER LASZL O ´ E, ´ FARKAS CSABA, FODOR Tagjai: BUSA MAT ´ ´ ZSOLT, LOCZI LAJOS, SIEGLER GABOR, ´ ´ ´ SZENTE PETER, TOTH TAMAS ´ ANDREA, TASNADI ´ ANIKO ´ Ford´ıt´ok: GROF ´ ´ ¨ Szerkeszt˝os´egi titk´ar: TRASY GYORGYN E A szerkeszt˝os´eg c´ıme: 1117 Budapest, P´azm´any P´eter s´et´any 1/C III. emelet 3.405. Telefon: 372-2850 A lap megrendelhet˝o az Interneten: www.komal.hu/megrendelolap/reszletek.h.shtml. El˝ofizet´esi d´ıj egy ´evre: 9200 Ft K´eziratokat nem ˝orz¨unk meg ´es nem k¨uld¨unk vissza. Minden jog a K¨oMaL tulajdonosai´e. E-mail: [email protected] Internet: http://www.komal.hu This journal can be ordered from the Editorial office: P´azm´any P´eter s´et´any 1/C III. emelet 3.405. 1117–Budapest, Hungary telephone: +36 (1) 372-2850 or on the Postal address H–1518 Budapest 112, P.O.B. 32, Hungary, or on the Internet: www.komal.hu/megrendelolap/reszletek.e.shtml. A Lapban megjelen˝o hirdet´esek tartalm´a´ert felel˝oss´eget nem v´allalunk.

449

A hagyom´ anyoknak megfelel˝oen ebben az ´evben is k¨ oz¨ olj¨ uk a ny´ ari matematikai di´ akolimpia feladatainak a megold´ asait; l´enyeg´eben u ´gy, ahogyan a legillet´ekesebbek, a magyar csapat tagjai le´ırt´ak. K¨ ozrem˝ uk¨ od´es¨ uket k¨ osz¨ onj¨ uk ´es ez´ uton is gratul´ alunk eredm´enyeikhez. A szerkeszt˝ os´eg M´asodik nap∗ 4. Legyen ABCDE olyan konvex o og, hogy BC = DE. Tegy¨ uk fel, hogy ¨tsz¨ az ABCDE o tsz¨ o g belsej´ e ben l´ e v˝ o T pontra T B = T D, T C = T E ´ e s ABT  = ¨ = T EA. Messe az AB egyenes a CD ´es CT egyeneseket a P , illetve Q pontban. Tegy¨ uk fel, hogy a P , B, A, Q pontok az egyenes¨ uk¨ on ebben a sorrendben helyezkednek el. Messe az AE egyenes a CD ´es DT egyeneseket az R, illetve S pontban. Tegy¨ uk fel, hogy az R, E, A, S pontok az egyenes¨ uk¨ on ebben a sorrendben helyezor¨ on vannak. kednek el. Bizony´ıtand´ o, hogy a P , S, Q, R pontok egy k¨ Moln´ar-Szab´ o Vilmos megold´asa. A szakaszhosszok egyenl˝ os´egeib˝ ol k¨ ovetkezik, hogy T BC ∼ T DE. = Legyen az AE ´es T Q egyenesek metsz´espontja X, az AB ´es DT egyenesek´e pedig Y . Egy kis sz¨ ogsz´ amol´assal megmutatjuk, hogy EXT  ∼ BY T . Az egybev´ ag´ o h´ aromsz¨ ogekb˝ ol ET D = CT B, tov´ abb´ a DT X = CT Y  (cs´ ucssz¨ogek), teh´ at ET X = DT X − ET D = CT Y  − CT B = BT Y . Tudjuk m´eg, hogy ABT  = T EA, teh´at val´oban egym´ashoz hasonl´ oak az EXT ´es BY T h´aromsz¨ ogek. Emiatt az is igaz, hogy EXT  = BY T , amib˝ol pedig k¨ ovetkezik, hogy QXS = EXT  = BY T  = QY S. Mivel QAX = SAY  is teljes¨ ul, ´ıgy az XQA ´es Y SA h´ aromsz¨ ogek hasonl´ oak, ´es ez´ert CQP  = XQY  = XSY  = = RSD. Az EXT ´es BY T h´ aromsz¨ ogek hasonl´ os´ ag´ ab´ ol k¨ ovetkezik tov´ abb´ a, hogy TX TY TX TY = T B . Mivel T E = T C ´es T B = T D, ebb˝ ol T C = T D , ami azt jelenti, hogy TE XY CD trap´ez. A hasonl´ o h´ aromsz¨ ogek egym´ asnak megfelel˝o XT E ´es BT Y  sz¨ ogeinek egyenl˝ os´eg´eb˝ ol BT Q = ST E. Mivel a felt´etel szerint ABT  = AET , a T QB ´es T SE h´ aromsz¨ ogek is hasonl´ oak, hiszen megfelel˝ o sz¨ ogeik egyenl˝oek. A hasonl´ os´ag miatt QT : ST = T B : T E, ∗

450

´ıgy QT · T C = QT · T E = ST · T B = ST · T D,

Az els˝ o nap feladatainak megold´ as´ at az okt´ oberi sz´ amban k¨ oz¨ olt¨ uk.

K¨ oz´ episkolai Matematikai ´ es Fizikai Lapok, 2022/8

Vizsg´aljuk ´altal´ aban a b < 4 eseteket. ast, ´es a tov´ abbiakban Ha b = 1, akkor ap = 1 + p. Az a = 1 eset nem ad megold´ p a  2p > 1 + p. Ha b = 2, akkor ap = 2 + p. Az a = 1 eset nem ad megold´ ast, ´es a tov´ abbiakban ap  2p > 2 + p (p > 2). Ha b = 3, akkor ap = 6 + p. Az a = 1 eset nem ad megold´ ast, ´es a tov´ abbiakban ast: a  2p > 6 + p minden p > 3 eset´en, m´ıg a p = 3, b = 3 megint nem ad megold´ obsz´ am. a3 = 3! + 3 = 9 nem k¨ p

Teh´at a tov´ abbiakban azt is feltehetj¨ uk, hogy b  4. A marad´ek eseteket k´et r´eszben vizsg´aljuk: b < p vagy b  p. Ha b < p, akkor b!-t osztja minden b-n´el nem nagyobb pr´ım, vagyis a (b! + p) kifejez´est egyik sem oszthatja, azaz b! + p = ap minden pr´ımoszt´ oja nagyobb, mint b. ul bp > b! + p, ´es Teh´at a > b. Ekkor viszont ap > bp . Megmutatjuk, hogy ezen k´ıv¨ innen k¨ovetkezik, hogy nem lehet megold´ asa az egyenletnek, ha b < p. Mivel b < p, ez´ert bevezethetj¨ uk a c  1 ´es p = b + c jel¨ ol´est. Ennek alapj´ an amtani ´es m´ertani k¨ ozepek azt k´ene bebizony´ıtanunk, hogy bb+c > b! + b + c. A sz´ k¨oz¨otti egyenl˝ otlens´eget alkalmazva (egyenl˝ os´eg nem ´ all f¨ onn, hiszen b  4):

b! = b · (b − 1)! = b

b−1  i=1

i 2 p´aratlan pr´ım. K¨ oz´ episkolai Matematikai ´ es Fizikai Lapok, 2022/8

451

2b−1

Å b

·b +b+c=

1 2b−1

b(b − 1) 2(b − 1)

+

1 bb−1

ãb−1 =

1 2b−1

· bb .

ã · bb + c.

Mivel b  4, ez´ert ez tov´ abb becs¨ ulhet˝ o: Å ã 1 1 + b! + b + c < · bb + c < bb + c. 2b−1 bb−1 Teh´at m´ar csak azt kellene bel´atnunk, hogy bb + c < bb+c , vagyis

5. Hat´ arozzuk meg mindazon, pozit´ıv eg´eszekb˝ ol ´ all´ o (a, b, p) sz´ amh´ armasokat, amelyekre p pr´ım ´es ap = b! + p. Seres-Szab´ o M´arton megold´asa. Ha p = 2, akkor a bizony´ıtand´ o ´all´ıt´as a k¨ovetkez˝ o alakot ¨ olti: a2 = b! + 2. Itt most ha b  4, akkor 4 | b!, ´ıgy b! + 2 ≡ 2 (mod 4). Viszont 2 soha nem lehet egy n´egyzetsz´am 4-es marad´eka, vagyis ekkor nincs megold´ as. Ha b  3, akkor: b = 1 eset´en 1! + 2 = 3, ahol a 3 nem n´egyzetsz´am. b = 2 eset´en 2! + 2 = 4 = 22 , vagyis a (2, 2, 2) megold´as. b = 3 eset´en pedig 3! + 2 = 8, ami szint´en nem n´egyzetsz´am.

1

Å =b·

c < bb · (bc − 1),

ami pedig teljes¨ ul, ha b  4. abHa b  p, akkor p | b!, ´ıgy p | b! + p, de p2  b! + p, vagyis b  2b − 1. Tov´ ´ ez´ert b´a, mivel b  p, ez´ert minden p-n´el kisebb pr´ım osztja a b! kifejez´est. Es a b! + p kifejez´est egyik p-n´el kisebb pr´ım sem oszthatja, teh´ at minden pr´ımoszt´ oja legal´ abb p. Tudjuk, hogy p | b! + p. Ha l´etezik ezen k´ıv¨ ul m´eg olyan q > p pr´ım, hogy p q | b! + p, akkor pq | b! + p = ap , vagyis (pq) | ap = b! + p. Ez viszont azt jelenti, hogy 2p−1  p p2p  (pq)  b! + p  (2p − 1)! + p = p + i, i=1

452

K¨ oz´ episkolai Matematikai ´ es Fizikai Lapok, 2022/8

ami a sz´ amtani ´es m´ertani k¨ ozepek k¨ oz¨ otti egyenl˝ otlens´eg miatt fel¨ ulr˝ ol tov´abb becs¨ ulhet˝ o (p > 2, nem ´all f¨ onn egyenl˝os´eg a sz´ amtani-m´ertani becsl´esben):

p+

2p−1  i=1

i 2, ez´ert p < p2p−2 ami ellentmond´ as. Teh´ at nem lehet a b! + p kifejez´esnek p-n´el sem nagyobb, sem kisebb pr´ımoszt´ oja. Vagyis a nem m´ as mint p-nek egy pozit´ıv kitev˝os hatv´anya. ar t´ ul nagy, vagyis az egyetlen lehet˝ os´eg az a = p Az im´ent l´ attuk, hogy a = p2 m´ maradt. Ekkor az egyenlet a k¨ ovetkez˝ o alakot ¨ olti: pp = b! + p, azaz   b! = pp − p = p pp−1 − 1 .

N´ezz¨ uk meg, hogy a k´et oldal 2-nek mekkora hatv´ any´ aval oszthat´o; egyr´eszt b > p miatt p , ν2 (b!) > 2 m´ asr´eszt az LTE-lemm´at haszn´alva:   ν2 (pp − p) = ν2 pp−1 − 1p−1 = ν2 (p − 1) + ν2 (p + 1) + ν2 (p − 1) − 1. Ha 4 | p − 1, akkor ν2 (p + 1) = 1, ´ıgy ν2 (pp − p) = 2ν2 (p − 1)  2 log2 p.

Ha 4 | p + 1, akkor ν2 (p − 1) = 1, ´ıgy ν2 (pp − p) = ν2 (p + 1) + 1  2 log2 p, ha p  5. Viszont, ha p  19, akkor p p − 1 = > 2 log2 p, 2 2 Å ã √ 19 19 − 1 19 2 log2 19 = 2 log2 16 · < 8 + 2 log2 2 = 9 = . = 8 + 2 log2 16 16 2 Maradt azoknak az eseteknek a vizsg´alata, amikor p < 19: p = 3: 33 − 3 = 4! megold´ as. as. p = 5: 55 − 5 = 3120 nem megold´ p = 7: 77 − 7 = 7 · (76 − 1) = 7 · (73 − 1) · (73 + 1) = 7 · 342 · 344 = 7 · 342 · 8 · 43, ami nem lehet megold´ as, hiszen b < 14, ´ıgy 43  b!. p = 11:   11 − 1 , ν2 1111 − 11 = ν2 (11 + 1) + 1 = 3 < 2 vagyis ez sem megold´ as. K¨ oz´ episkolai Matematikai ´ es Fizikai Lapok, 2022/8

453

p = 13:

  13 − 1 ν2 1313 − 13 = 2ν2 (13 − 1) = 4 < , 2 vagyis ez sem megold´ as. p = 17: õ û õ û   17 17 ν2 1717 − 17 = 2ν2 (17 − 1) = 8 < + < ν2 (17!), 2 4 vagyis ez sem megold´ as.

Teh´at k´et lehets´eges megold´ as van: (a, b, p) = (2, 2, 2) vagy (3, 4, 3). 6. Legyen n pozit´ıv eg´esz. Skandin´ av n´egyzet egy n × n m´eret˝ u t´ abla, amely 1-t˝ ol n2 -ig az ¨ osszes eg´esz sz´ amot tartalmazza u ´gy, hogy minden mez˝ oben pontosan egy sz´ am ´ all. K´et k¨ ul¨ onb¨ oz˝ o mez˝ ot szomsz´edosnak tekint¨ unk, ha van k¨ oz¨ os oldaluk. Ha egy mez˝ onek minden szomsz´edj´ aban nagyobb sz´ am a ´ll, mint o ˝benne, akkor v¨ olgynek nevezz¨ uk. Kaptat´ o egy sorozat, amely egy vagy t¨ obb mez˝ ob˝ ol ´ all u ´gy, hogy (i) a sorozat els˝ o mez˝ oje egy v¨ olgy, (ii) a sorozat minden tov´ abbi mez˝ oje szomsz´edos az o ˝t k¨ ozvetlen¨ ul megel˝ oz˝ o mez˝ ovel, ´es (iii) a sorozat mez˝ oiben ´ all´ o sz´ amok n¨ ovekv˝ o sorrendben vannak. Adott n eset´en hat´ arozzuk meg egy skandin´ av n´egyzetben l´ev˝ o kaptat´ ok sz´ am´ anak legkisebb lehets´eges ´ert´ek´et. N´ador Benedek megold´asa. V´alasz: Legal´abb 2n(n − 1) + 1 kaptat´o van.

Bizony´ıt´as: Az a mez˝o, ami az 1-est tartalmazza mindig v¨ olgy, ´ıgy legal´ abb egy v¨olgy van. A mez˝ okbe ´ırt sz´ amok szerinti indukci´ oval ad´odik, hogy minden mez˝o vagy v¨olgy, vagy l´etezik olyan – legal´ abb k´et mez˝ ob˝ol a´ll´ o – kaptat´ o, amelynek ˝o az utols´o eleme. Nevezz¨ uk mez˝ ohat´ ar nak k´et szomsz´edos mez˝o k¨ oz¨ os hat´ ar´ at. Minden mez˝ ohat´ arra igaz, hogy van legal´ abb egy olyan kaptat´ o, amiben o˝ az utols´o mez˝ ohat´ ar: az a´ltala hat´ arolt k´et mez˝ o k¨ oz¨ ul a kisebb sz´ amot tartalmaz´ o szomsz´edja egy kaptat´ o utols´ o mezeje, ezt a kaptat´ ot pedig kieg´esz´ıthetj¨ uk a nagyobbik sz´ amot tartalmaz´ o mez˝ovel; ebben a kaptat´ oban az utols´o mez˝ohat´ ar ovetkezik, hogy a legal´abb k´et mez˝ ob˝ ol ´ all´ o ´eppen az a´ltalunk v´ alasztott. Ebb˝ ol k¨ kaptat´ ok sz´ama legal´abb annyi, mint a mez˝ ohat´ arok sz´ama. Mivel 2n sorban soronk´ent n − 1 mez˝ ohat´ ar van ´es az 1 v¨ olgy, ´ıgy a kaptat´ ok sz´am´anak minimuma legal´ abb 2n(n − 1) + 1. Mutatunk egy konstrukci´ot olyan Skandin´ av n´egyzetre, amiben pontosan 2n(n − 1) + 1 kaptat´o van. Nevezz¨ uk hegytet˝ onek azokat a mez˝oket, amelyekben nagyobb sz´ am a´ll, mint a szomsz´edaikban. Ha teljes¨ ul, hogy pontosan egy v¨ olgy van, ´es a hegytet˝ ok kiv´etel´evel minden mez˝ o csak egy kaptat´onak a v´ege, akkor pontosan 2n(n − 1) + 1 kaptat´o lesz, mivel az utols´ o mez˝ ohat´ ar, amin a kaptat´ o ´atmegy meghat´arozza a kaptat´ ot (a kaptat´ o kor´ abbi mez˝ oi egy´ertelm˝ uek), az 1-es 454

K¨ oz´ episkolai Matematikai ´ es Fizikai Lapok, 2022/8

mez˝ on v´egz˝ od˝o v¨ olgynek pedig nincs mez˝ ohat´ ara. Nevezz¨ uk ezt a tulajdons´ agot minimum felt´etelnek. A minimum felt´etel teljes¨ ul´es´et u ´gy ´erj¨ uk el, hogy a skandin´av n´egyzetb˝ ol k´esz´ıt¨ unk egy fa gr´ afot, amelynek cs´ ucsai a mez˝ ok k¨oz¨ ul ker¨ ulnek ki, az ´elek pedig a k¨ ozt¨ uk l´ev˝ o mez˝ohat´ arok. A f´ at u ´gy helyezz¨ uk el, hogy minden olyan mez˝ ovel legyen szomsz´edos cs´ ucsa, ami nem cs´ ucsa a gr´ afnak; ezen k´ıv¨ ul a kimarad´ o mez˝ ok k¨ oz¨ ott ne legyenek olyanok, amik egym´assal szomsz´edosak. Legyen a fa cs´ ucsainak sz´ama k. Ekkor a fa mez˝oibe az 1, . . . , k sz´ amokat ´ırjuk be u ´gy, hogy az 1-esb˝ol indulva a fa´eleken kereszt¨ ul minden facs´ ucsba eljuthassunk fa´eleken (vagyis az 1-esb˝ ol kiindulva n¨ ovekednek a fa mez˝oinek az ´ert´ekei). Ezt egy, a fa ar´ assal ´erhetj¨ uk el. Meggondolhat´ o, hogy a f´as 1-es cs´ ucs´ ab´ ol ind´ıtott sz´eless´egi bej´ elrendez´esre teljes¨ ul a minimum felt´etel, mert csak az 1 a v¨olgy ´es k´et f´ an k´ıv¨ uli mez˝ o nem lehet szomsz´edos. A fa l´etez´es´et n > 4-re konstrukt´ıvan bizony´ıtjuk az n-nek a 3-as marad´eka szerint. Tekints¨ uk az al´ abbi 2 × n-es ´es 3 × n-es elemeket:

Most m´ar minden adott, hogy minden n > 4-re konstrukci´ ot adjunk. Ezt az n-nek a 3-as marad´eka szerinti esetvizsg´ alattal tessz¨ uk meg. A 3 | n esetet a C1 ´es C2 elemek felv´ altva egym´ as al´a helyez´es´evel ´erj¨ uk el.

Az n ≡ 2 mod 3 esetben legfel¨ ulre rakunk egy A1-es elemet, al´ a egy C1-est ut´ ana felv´ altva C2-eseket ´es C1-eseket.

Az n ≡ 1 mod 3 esetet hasonl´ oan ´erj¨ uk el, mint az n ≡ 2 mod 3-at, csak az utols´o 3 × n-es helyett az oda passzol´ o 2 × n-eset rakjuk le (van ilyen, mert a A2 -t be tudjuk rakni). C1 al´a A1 -t, C2 al´ Ezzel minden n > 4-re adtunk konstrukci´ ot. V´eg¨ ul mutatunk konstrukci´ ot n = 2, 3, 4-re (n = 1 eset´en egyf´ele kit¨ olt´es van, ami j´o):

N´egysz´ın-sejt´es III: A sz´ınez´esi polinom, avagy mi´ert olyan neh´ez a n´egysz´ın-t´etel∗

Nevezz¨ uk el az elemeket az ´ abra szerint A1, A2, C1, C2-nek. Legyen A1 az A1 at elem saj´ at v´ızszintes tengely´ere t¨ ukr¨ oz¨ ott k´epe, hasonl´ oan A2 az A2-nek saj´ v´ızszintes tengely´ere t¨ ukr¨ oz¨ ott k´epe. Ekkor k¨ onny˝ u meggondolni, hogy minden elemben a narancss´ arga r´esz f´ at alkot. Azt is meggondolhatjuk, hogy a C1 ´es C2 elemeket tetsz˝ oleges sorrendben egym´ as al´a rakva a k´et elem a´ltal alkotott t´abl´ azatban is f´ at alkotnak a narancs sz´ın˝ u mez˝ok. Hasonl´ oan A1-et C2 f¨ol´e, A1 -t C1 al´a helyezve az elemek a´ltal alkotott t´ abl´ azatban a narancs mez˝ ok f´at alkotnak. (Az elemeket u ´gy rakjuk egym´as al´a, hogy a bal sz´el¨ uk egy vonalban legyen.) Ugyanez elmondhat´ o A2, A2 ´es C2-re is. K¨ oz´ episkolai Matematikai ´ es Fizikai Lapok, 2022/8

455

Az el˝oz˝ o r´eszekben megismerkedt¨ unk a n´egysz´ın-sejt´essel, mely szerint b´armely t´erk´epet ki lehet sz´ınezni 4 sz´ınnel u ´gy, hogy az egym´assal hat´ aros r´egi´ ok k¨ ul¨ onb¨ oz˝ o sz´ınt kapjanak. Majd ezt a k´erd´est a´tfogalmaztuk a s´ıkgr´ afok nyelv´ere, ahol a sejt´es azt mondta, hogy egy s´ıkbarajzolt gr´ af pontjai kisz´ınezhet˝ ok 4 sz´ınnel u ´gy, hogy ´ellel ¨osszek¨ot¨ott pontok ne kapjanak azonos sz´ınt. (Az ilyen sz´ınez´eseket nevezt¨ uk j´ o sz´ınez´eseknek.) az el˝ oz˝ o r´eszben l´ attuk azt is, hogy a 6 sz´ınnel val´ o j´o sz´ınezhet˝ os´eg bizony´ıt´asa eg´eszen egyszer˝ u volt, ´es az 5 sz´ınnel val´ o sz´ınezhet˝ os´eget is viszonylag k¨onnyen be lehetett bizony´ıtani. Mindezek ellen´ere a 4 sz´ınnel val´o j´o sz´ınezhet˝ os´eg ast bizony´ıt´as´ara 120 ´evre volt sz¨ uks´eg, ´es m´eg ekkor is csak egy olyan bizony´ıt´ tudtak adni, amihez komoly sz´am´ıt´ og´epes sz´ am´ıt´ asokra volt sz¨ uks´eg. Vajon mi´ert nehezedik meg ez a k´erd´es ennyire, ha 5 sz´ınr˝ ol 4-re t´er¨ unk a´t? Ebben a r´eszben bemutatjuk a sz´ınez´esi polinomot. Ezt a polinomot George David Birkhoff amerikai matematikus kezdte vizsg´ alni az 1910-es ´evekben, abban a rem´enyben, hogy a seg´ıts´eg´evel bel´athatja a n´egysz´ın-sejt´est. B´ ar a n´egysz´ın´ ´ Nemzeti Az ´ır´ as az Innov´ aci´ os ´es Technol´ ogiai Miniszt´erium UNKP-20-5 k´ odsz´ am´ u Uj Kiv´ al´ os´ ag Programj´ anak a Nemzeti Kutat´ asi, Fejleszt´esi ´es Innov´ aci´ os Alapb´ ol finansz´ırozott szakmai t´ amogat´ as´ aval k´esz¨ ult. ∗

456

K¨ oz´ episkolai Matematikai ´ es Fizikai Lapok, 2022/8

sejt´est v´eg¨ ul m´ as gondolatmenet seg´ıts´eg´evel l´att´ ak be, a sz´ınez´esi polinom nagyon ´erdekes objektumnak bizonyult, ´es valamennyire azt is meg fogja nek¨ unk mutatni, hogy mi´ert is olyan neh´ez a n´egysz´ın-sejt´es. T´erj¨ unk teh´ at vissza a n´egysz´ın-sejt´eshez. Azt szeretn´enk bel´ atni, hogy ha G egy hurok´elmentes s´ıkgr´ af, akkor G-nek l´etezik 4 sz´ınnel val´o j´o sz´ınez´ese. Van a matematik´ aban egy furcsa jelens´eg: ha elakadunk egy k´erd´essel, paradox m´odon n´eha ´erdemes egy nehezebb” k´erd´essel pr´ ob´ alkozni. M´egpedig egy olyan nehez´ıt´es” sel, amiben t¨ obb a strukt´ ura. Ez az extra strukt´ ura n´eha seg´ıt abban, hogy jobban meg´erts¨ uk a dolgokat. Legy¨ unk teh´ at ambici´ozusak, ´es k´erdezz¨ unk egy nehezebb k´erd´est. H´anyolef´elek´eppen lehet a G gr´ afot 4 sz´ınnel j´ol sz´ınezni? S˝ ot, k´erdezz¨ uk meg tetsz˝ ges pozit´ıv eg´esz k-ra, hogy h´any j´ o sz´ınez´ese van G-nek k sz´ınnel. Ha ezt meg tudn´ ank v´ alaszolni, akkor persze azt is meg tudn´ ank mondani, hogy van-e 4 sz´ınnel val´ o j´o sz´ınez´es, teh´ at ez egy nehezebb k´erd´es. De most m´ar van lehet˝ os´eg¨ unk szab´ alyoss´ agokat keresni a j´o sz´ınez´esek sz´ am´aban. afnak h´any j´o Legyen k egy pozit´ıv eg´esz, ´es jel¨ olj¨ uk pG (k)-val, hogy a G gr´ sz´ınez´ese van k sz´ınnel. L´assunk egy konkr´et p´eld´ at, ahol ezt ki is tudjuk sz´ amolni. ucsot sz´ınezhetj¨ uk Vegy¨ uk 6. ´ abra bal oldal´ an l´athat´ o h´ aromsz¨ oggr´ afot. A v1 cs´ uk valamelyik sz´ınnel, akkor b´ armelyik sz´ınnel, ez k lehet˝ os´eg. Ha a v1 -et kisz´ınezt¨ ot´es, hogy nem lehet ugyanilyen sz´ın˝ u. Teh´ at v1 tetsz˝ oleges a v2 -re az egyetlen megk¨ ul sz´ıne eset´en k − 1 lehet˝ os´eg van v2 sz´ın´ere. Ez id´aig k · (k − 1) lehet˝os´eg. V´eg¨ b´ arhogy is sz´ınezt¨ uk ki v1 -et ´es v2 -t, v3 sz´ın´ere az az egyetlen megk¨ot´es, hogy ett˝ol a k´et sz´ınt˝ ol k¨ ul¨ onb¨ oz˝ o legyen. Teh´ at a h´ aromsz¨ oggr´ afnak

a gr´ af egyik ´ele, melynek v´egpontjai u ´es v. A pG (k) kifejez´esr˝ ol szeretn´enk megmutatni, hogy k-ban egy polinom. Vegy¨ uk a G − e gr´ afot, azaz azt a gr´ afot, ahol az e ´elet kit¨or¨ olj¨ uk (l´ asd a 6. ´abr´ at). Ez egy t ´el˝ u gr´ af, teh´ at r´ola m´ar tudjuk, ossze G ´es G − e j´o sz´ınez´eseinek sz´am´at. hogy pG−e (k) egy polinom. Hasonl´ıtsuk ¨ A G gr´ af tetsz˝ oleges k sz´ınnel val´o j´ o sz´ınez´ese j´ o sz´ınez´ese G − e-nek is, hiszen G − e j´o sz´ınez´eseire kevesebb a megk¨ ot´es. Viszont (G − e)-nek lehet n´eh´ any olyan j´o sz´ınez´ese, amely G-nek nem j´ o sz´ınez´ese: ahol az u ´es v pont azonos sz´ınt kap. Vegy¨ uk most azt a gr´ afot, ahol az u ´es v pontot ¨ osszeragasztjuk egy pontt´ a, az e ´elt pedig elhagyjuk. Nevezz¨ uk ezt a gr´ afot (G/e)-nek (l´asd a 6. ´abr´ at). G/e tetu j´ o sz´ınez´est (G − e)-re, ahol az e sz˝ oleges k sz´ın˝ u j´ o sz´ınez´ese ad egy olyan k sz´ın˝ ul minden ´el k´et v´egpontja azonos sz´ın˝ u. Teh´ at pG (k) = pG−e (k) − pG/e (k) teljes¨ ol is pozit´ıv eg´esz k-ra. Vegy¨ uk ´eszre, hogy (G/e)-nek is t ´ele van, teh´ at pG/e (k)-r´ ol is tudjuk, hogy polinom. K´et polinom k¨ ul¨ onbs´ege szint´en polinom, teh´ at pG (k)-r´ bel´attuk, hogy polinom.  Ezzel bel´ attuk, hogy pG (k) val´ oban egy polinom. Ezt a polinomot nevezz¨ uk a G sz´ınez´esi polinomj´anak. Mi´ert hasznos ez nek¨ unk? Eddig a pG (k)-t csak pozit´ıv eg´esz sz´ amokra defini´ altuk. Most viszont, hogy tudjuk, hogy pG (k) egy polinom uk tetsz˝ oleges val´ os sz´amokra is. Mond(pl pΔ (k) = k 3 − 3k 2 + 2k), ezt ´ertelmezhetj¨ o hatjuk pl, hogy a h´ aromsz¨ oggr´ afnak pΔ (3,5) = 3,53 − 3 · 3,52 + 2 · 3,5 = 13,125 j´ sz´ınez´ese van 3,5 sz´ınnel. Mit jelent ez? Val´ oj´aban nem vil´ agos. Az sem vil´ agos, hogy mit jelentene egy 3,5 sz´ınnel val´o j´o sz´ınez´es. Viszont olyan szempontb´ ol m´egis hasznos ez a kiterjeszt´es, hogy most m´ ar haszn´alhatjuk azokat az eszk¨ oz¨ oket, amelyeket anal´ızisb˝ ol tanultunk a f¨ uggv´enyek elemz´es´ere. William Tutte angol ma√ tematikusnak p´eld´ aul siker¨ ult bebizony´ıtania, hogy egy G s´ıkgr´ afra pG ( 5+2 5 ) > 0 mindig teljes¨ ul (b´ armennyire nem is vil´ agos, hogy mit is jelent pontosan ez a sz´ am). √

pΔ (k) = k · (k − 1) · (k − 2) = k 3 − 3k 2 + 2k j´o sz´ınez´ese van k sz´ınnel.

Mivel 5+2 5 ≈ 3,618, lehetett abban b´ızni, hogy tal´an be lehet l´atni, hogy a pG (x) √

´ert´ek az 5+2 5 ut´ an nem cs¨ okken t´ ul gyorsan, ´es akkor k¨ ovetkezne, hogy m´eg pG (4) is pozit´ıv. Sajnos az der¨ ult ki, hogy ez a m´odszer nem m˝ uk¨ odhet. Gordon Royle ugyanis megmutatta, hogy ak´ armilyen pici √ ε > 0 sz´ amra van olyan G s´ıkgr´ af ´es

6. ´ abra. Bal oldalon: a h´ aromsz¨ oggr´ af. Jobb oldalon: P´elda a 3. t´etel bizony´ıt´ as´ aban szerepl˝ o G − e ´es G/e gr´ afokra

´ Eszrevehetj¨ uk, hogy a h´ aromsz¨ og eset´en pΔ (k) a k v´ altoz´ ora n´ezve egy polinom. Vajon igaz-e ez a´ltal´ aban? Megmutatjuk, hogy igen. oleges G gr´ af eset´en k-ban polinom. 3. t´etel. pG (k) tetsz˝ Bizony´ıt´as. Ezt a gr´ af ´elsz´am´ara vonatkoz´ o indukci´oval fogjuk bel´atni. Ha a gr´ afnak nincsen ´ele, ´es n cs´ ucsa van, akkor b´ armelyik cs´ ucsot b´ armilyen sz´ın˝ ure sz´ınezhetj¨ uk, teh´ at k n j´o sz´ınez´es van. Ez k-ban egy polinom, teh´at az alapeset k´eszen van. Most tegy¨ uk fel, hogy m´ ar tudjuk, hogy pG (k) egy polinom, ha G-nek legfeljebb t ´ele van (´es ak´ armennyi cs´ ucsa). Legyen G egy gr´ af t + 1 ´ellel, ´es legyen e K¨ oz´ episkolai Matematikai ´ es Fizikai Lapok, 2022/8

457

oz¨ ott a sz´ınez´esi polinom 4 − ε < x < 4, hogy pG (x) = 0. Azaz az 5+2 5 ´es a 4 k¨ ´ert´eke m´eg le tud menni 0-ra, s˝ ot, ez a nullhely a 4-hez ak´ armilyen k¨ ozel tud lenni. (Kicsit pongyol´an fogalmazva, van olyan s´ıkgr´ af, amelynek 3,99 sz´ınnel m´ar nincs j´o sz´ınez´ese, b´ armit is jelentsen ez.) Ez azt is mutatja, hogy a n´egysz´ın-sejt´es bizonyos ´ertelemben ´eppenhogy teljes¨ ul. Ez az ´eppenhogy teljes¨ ul´es” m´ar sejteti, hogy ” nem fogunk tudni olyan nagyvonal´ u bizony´ıt´ ast tal´ alni a n´egysz´ın-sejt´esre, mint amilyet a hatsz´ın-sejt´esre lehetett adni. Ha a n´egysz´ın-sejt´esre nem is adott nek¨ unk bizony´ıt´ ast a sz´ınez´esi polinom, az´ert m´egiscsak ´alljunk meg, ´es n´ezegess¨ uk meg egy kicsit jobban. √

L´attuk, hogy b´ ar nem ´ertj¨ uk, hogy mit jelent a pG ( 5+2 5 ), az´ert be lehetett l´atni r´ ola, hogy egy s´ıkgr´ afra mindig pozit´ıv. Most n´ezz¨ unk meg egy m´asik meglep˝o ´all´ıt´ast. Ehhez el˝ osz¨ or tiszt´azzunk n´eh´ any fogalmat. Egy G gr´ af ir´any´ıt´ as´ anak nevezz¨ uk azt, ha minden ´el´enek adunk valamilyen ir´any´ıt´ast. P´eld´ aul 7. ´ abra mutatja a h´ aromsz¨ oggr´ af ir´any´ıt´ asait. Egy ´elt k´etf´ele458

K¨ oz´ episkolai Matematikai ´ es Fizikai Lapok, 2022/8

k´eppen lehet ir´ any´ıtani, teh´ at egy m ´el˝ u gr´ afnak 2m k¨ ul¨onb¨oz˝ o ir´ any´ıt´asa van. Egy ir´ any´ıt´ ast k¨ ormentesnek nevez¨ unk, ha nincs olyan cs´ ucs, ahonnan elindulva vissza tudunk jutni ugyanoda u ´gy, hogy mindig ir´ anyhelyesen megy¨ unk a´t az ´eleken.

amely minden x = k pozit´ıv eg´esz sz´ amra 0. Tudjuk, hogy egy nemnulla polinomnak legfeljebb annyi nullhelye van, mint a foksz´ ama. Teh´ at pG (x) − pG−e (x) + pG/e (x) az azonosan 0 polinom. Azaz most m´ar azt is tudjuk, hogy pG (x) = pG−e (x) − pG/e (x)

7. a ´bra. A h´ aromsz¨ oggr´ af 8 k¨ ul¨ onb¨ oz˝ o ir´ any´ıt´ asa. Ezek k¨ oz¨ ul a harmadik ´es a hatodik nem k¨ ormentes, a t¨ obbi 6 ir´ any´ıt´ as k¨ ormentes

A k¨ ovetkez˝ o meglep˝ o ´all´ıt´ ast Richard Stanley amerikai matematikus fedezte fel. 2. ´all´ıt´as. Tetsz˝ oleges G gr´ afra n

pG (−1) = (−1) · (G k¨ ormentes ir´any´ıt´asainak sz´ ama), ahol n a G gr´ af cs´ ucsainak sz´ ama. 1. p´elda. A h´ aromsz¨ oggr´ af eset´en 3

2

pΔ (−1) = (−1) − 3 · (−1) + 2 · (−1) = −1 − 3 − 2 = −6, 3

ami val´oban (−1) · 6. Itt az a meglep˝ o dolog t¨ ort´ent, hogy pG (−1)-et u ´gy defini´ altuk, hogy a sz´ınez´esi polinomba behelyettes´ıtett¨ unk (−1)-et. Pozit´ıv eg´esz k eset´en tudjuk, hogy a sz´ınez´esi polinom a k sz´ınnel val´o j´o sz´ınez´eseket sz´ amolja meg. Viszont negat´ıv sz´amokra ugyan´ ugy nem vil´agos, hogy a pG (x) jelent-e valamit, ahogy t¨ortekre sem az. Az ´all´ıt´ as viszont azt mutatja, hogy var´ azs¨ ut´esre a pG (−1) m´egis valami ´ertelmes dolgot sz´amol meg. L´assuk is be ezt az ´all´ıt´ast. ´ amra vonatkoz´ Bizony´ıt´as. Elsz´ o indukci´ot fogunk haszn´alni. Ha a gr´afnak 0 n at pG (−1) = (−1) . ´ele van, akkor pG (k) = k n , teh´ M´ asr´eszt ilyenkor azt mondjuk, hogy 20 = 1 ir´ any´ıt´as van (az u ¨res ir´any´ıt´as), n ´es ez persze k¨ ormentes. Teh´at val´oban pG (−1) = (−1) · (k¨ormentes ir´any´ıt´asok sz´ ama). Ha valakit ez esetleg nem gy˝ oz¨ ott meg, n´ezz¨ uk meg az 1 ´el˝ u esetet is. Ilyenkor asodj´ ara megsz´ınezett cs´ ucs´ at nem sz´ınezhetj¨ uk pG (k) = k n−1 (k − 1), hiszen az ´el m´ ugyanolyan sz´ınre, mint az els˝ot, a t¨ obbi cs´ ucs sz´ın´ere viszont nincs megk¨ot´es. Azaz n−1 n · (−2) = (−1) · 2. M´ asr´eszt ilyenkor nyilv´ an 2 ir´ any´ıt´as van, ´es pG (−1) = (−1) mindkett˝ o k¨ ormentes. Azaz val´ oban teljes¨ ul a t´etel a´ll´ıt´asa 1 ´el˝ u gr´afokra is. Tegy¨ uk fel most, hogy teljes¨ ul az ´ all´ıt´ as tetsz˝ oleges n cs´ ucssz´am eset´en, ha az ´elsz´am kisebb, mint t, ´es vegy¨ unk egy t ´el˝ u G gr´ afot. M´ar kor´ abban l´attuk, hogy ul minden pozit´ıv eg´esz k-ra. tetsz˝ oleges e ´elre pG (k) = pG−e (k) − pG/e (k) teljes¨ L´assuk be, hogy ez minden val´ os x-re is igaz. A bal oldalon ´es a jobb oldalon is egy polinom van. Teh´at a k´et oldal k¨ ul¨ onbs´ege, pG (x) − pG−e (x) + pG/e (x) egy polinom, K¨ oz´ episkolai Matematikai ´ es Fizikai Lapok, 2022/8

459

minden val´ os sz´amra, speci´alisan x = −1-re is. (G − e)-nek ´es (G/e)-nek is kevesebb, mint t ´ele van, teh´ at az indukci´os feltev´es n miatt tudjuk, hogy pG−e (−1) = (−1) · (G − e k¨ ormentes ir´any´ıt´ asainak sz´ ama) ´es n−1 · (G/e k¨ ormentes ir´any´ıt´ asainak sz´ ama). Itt haszn´ altuk, hogy pG/e (−1) = (−1) n G − e cs´ ucssz´ama n, G/e cs´ ucssz´ama pedig n − 1. Azaz pG (−1) = (−1) · (G − e n ormentes ir´any´ıt´ asainak sz´ ama). k¨ormentes ir´any´ıt´ asainak sz´ ama) + (−1) · (G/e k¨ Most m´ar el´eg megmutatni, hogy G k¨ ormentes ir´any´ıt´ asainak sz´ ama = G − e k¨ormentes ir´any´ıt´ asainak sz´ ama + G/e k¨ ormentes ir´any´ıt´ asainak sz´ ama. N´ezz¨ uk meg el˝ osz¨ or, hogy hogyan viszonyulnak G − e k¨ ormentes ir´any´ıt´ asai a G k¨ormentes ir´any´ıt´ asaihoz. Vegy¨ uk G − e egy k¨ ormentes ir´any´ıt´ as´ at. Az biztos, hogy nem lehet u-b´ ol v-be ´es v-b˝ ol u-ba is ir´ any´ıtott u ´ton eljutni, hiszen akkor u-b´ ol u-ba vissza lehetne jutni ir´ anyhelyesen mozogva, ilyet pedig egy k¨ ormentes at 3 eset lehet: Vagy csak u-b´ ol v-be lehet ir´any´ıtott ir´any´ıt´asban nem lehet. Teh´ u ´ton eljutni (´es v-b˝ ol u-ba nem), vagy csak v-b˝ ol u-ba lehet ir´ any´ıtott u ´ton eljutni (´es u-b´ ol v-be nem), vagy pedig sem u-b´ ol v-be, sem pedig v-b˝ ol u-ba nem lehet ir´any´ıtott u ´ton eljutni. Most rajzoljuk vissza az e ´elet. Ha a G − e k¨ ormentes ir´any´ıt´ as´ ab´ol meg szeretn´enk kapni G egy k¨ ormentes ir´any´ıt´ as´ at, akkor az els˝ o esetben csak u-b´ ol v-be ir´ any´ıthatjuk az e ´elet, a m´ asodik esetben csak v-b˝ ol u-ba, a harmadik esetben viszont ak´armelyik ir´anyban megir´any´ıthatjuk, mindk´et esetben k¨ ormentes marad az ir´any´ıt´ as. uk ´eszre, hogy ha (G − e)-ben ¨ osszeragasztjuk az u ´es a v cs´ ucsoViszont vegy¨ kat, akkor a G − e egy k¨ ormentes ir´any´ıt´ asa pontosan akkor marad k¨ ormentes, ha sem u-b´ ol v-be, sem v-b˝ ol u-ba nem volt a G − e-ben ir´any´ıtott u ´t. Teh´ at G k¨ ormentes ir´ any´ıt´asainak sz´ ama = G − e k¨ ormentes ir´any´ıt´ asainak sz´ ama + G/e k¨ ormentes ir´any´ıt´asainak sz´ ama. Ezzel az indukci´os l´ep´est befejezt¨ uk.  Hivatkoz´asok [1] Hubai Tam´as, The chromatic polynomial, Diplomamunka, https://web.cs.elte.hu/blobs/diplomamunkak/mat/2009/hubai_tamas.pdf [2] Stanley, R. P., Acyclic orientations of graphs, Discrete Math. 5(2): 171–178, (1973), doi:10.1016/0012-365X(73)90108-8. [3] Gordon Royle, Planar triangulations with real chromatic roots arbitrarily close to 4, Annals of Combinatorics, 12(2): 195–210, July 2008, arXiv:math/0511304. T´ othm´er´esz Lilla ELTE

460

K¨ oz´ episkolai Matematikai ´ es Fizikai Lapok, 2022/8

5. Az ´ abra egy nyolcsz¨ oget mutat, amely 10 egys´egn´egyzetb˝ ol ´ all. A P Q szakasz felezi XQ any? a nyolcsz¨ og ter¨ ulet´et. Mekkora az QY ar´

Helyesb´ıt´es ´es ko ¨zlem´eny

2

(A) 5 ;

A szeptemberi sz´ amban k¨ oz¨ olt v´egeredm´enyben a csapatversenyekn´el t¨obb helyen t´evesen jelent meg a versenyz˝o ´evfolyama. A helyes v´egeredm´eny a honlapon l´ athat´ o, illetve let¨ olthet˝ o pdf-ben is szint´en a honlapr´ ol, a szeptemberi sz´ am tartalomjegyz´ek´en´el∗ kell keresni. Egy csapatn´ al pedig nem ´ırtuk oda, hogy dics´eretet kapott: A G-jel˝ u fizika gyakorlatok csapatverseny´eben dics´eretben r´eszesu ¨lt: 3. Vonal vonal vonal : J´ avor Bence 9. o. t. (Budapest, V´arosmajori Gimn.), (hi´ anyz´ o GDPR nyilatkozat) 9. o. t. (Budapest, V´arosmajori Gimn.) 65 pont. A hib´ ak´ert eln´ez´est k´er¨ unk! K¨ ozlem´eny A 2021–2022-es tan´ev pontversenyeinek ¨ osszes´ıtett eredm´eny´et nem ´all m´odunkban megjelentetni. Szerk.

sapka is van? (A) 3; (B) 5; (C) 8; (D) 15; (E) 20. ¨ k¨ 9. Ot ul¨ onb¨ oz˝ o pozit´ıv eg´esz sz´ am ´ atlaga 15, medi´ anja 18. Legfeljebb mekkora lehet az ¨ ot sz´ am k¨ oz¨ ul a legnagyobb? (A) 19; (B) 24; (C) 32; (D) 35; (E) 40. 10. Az ´ abra alapj´ an h´ any fok az α1 , α2 , ogek ¨ osszege? (A) 360◦ ; α3 , α4 , α5 , α6 , α7 sz¨ (B) 540◦ ; (C) 630◦ ; (D) 720◦ ; (E) 1080◦ .

1. H´ any k¨ ul¨ onb¨ oz˝ o h´etjegy˝ u palindrom sz´ am k´epezhet˝ o a 2, 2, 3, 3, 5, 5, 5 sz´ amjegyekb˝ ol? (A palindrom sz´ am olyan sz´ am, amelynek sz´ amjegyeit ford´ıtott sorrendben fel´ırva az eredeti sz´ amot kapjuk vissza.) (A) 6; (B) 8; (C) 12; (D) 36; (E) 64. aci´ ol´ anc fejezi ki helyesen x, 2. Legyen x = 220 · 35 , y = 25 · 510 , z = 710 . Melyik rel´ y, z nagys´ ag szerinti sorrendj´et? (A) x > y > z; (B) x > z > y; (C) y > z > x; (D) y > x > z; (E) z > x > y.

11. Bea egy lapra le´ırja az eg´esz sz´ amokat 1-t˝ ol 30-ig, majd ¨ osszeadja azokat. Bal´ azs egy m´ asik lapra le´ırja Bea sz´ amait azzal a m´ odos´ıt´ assal, hogy minden 2-es sz´ amjegy helyett 1-est ´ır, majd ˝ o is ¨ osszegzi a sz´ amait. Mennyivel t¨ obb Bea o azs´en´ al? (A) 13; (B) 26; ¨sszege Bal´ (C) 102; (D) 103; (E) 110. (A)

3. Z´ ar´ ojelekkel kieg´esz´ıtve a (A) 2;

(B) 3;

(C) 4;

(D) 5;

4. Egy szob´ aban az emberek k´etharmada a sz´ekek h´ aromnegyed´en u obbi ember ¨l, a t¨ ´ll. Ha 6 u a aban, akkor h´ any ember tart´ ozkodik a helyis´egben? (A) 12; ¨res sz´ek van a szob´ (B) 18; (C) 24; (D) 27; (E) 36. †

12. Az al´ abbi sz´ amok k¨ oz¨ ul melyik n´egyzetsz´ am? 17! · 18! 18! · 19! 19! · 20! 20! · 21! 16! · 17! ; (B) ; (C) ; (D) ; (E) . 2 2 2 2 2 13. H´ arom k¨ ul¨ onb¨ oz˝ o egyjegy˝ u pozit´ıv eg´esz sz´ amot ´ırunk az als´ o sorban lev˝ o n´egyzetekbe. A szomsz´edos n´egyzetekbe ker¨ ul˝ o sz´ amokat ¨ osszeadjuk, majd a kapott eredm´enyt a felett¨ uk lev˝ o cell´ akba ´ırjuk. A m´ asodik sorban ugyanezt az elj´ ar´ ast folytatjuk, ´ıgy kapunk egy sz´ amot a fels˝ o n´egyzetben. Mekkora lehet a fels˝ o n´egyzetbe ker¨ ul˝ o legnagyobb ´es legkisebb sz´ am k¨ ul¨ onbs´ege? (A) 18; (B) 22; (C) 24; (D) 26; (E) 28. 14. Az A, B, C, D, E, F , G, H, I olyan sz´ amok, amelyek teljes´ıtik az A + B + C = 1, B + C + D = 2, C + D + E = 3,

https://www.komal.hu/lap/2022-09/tart.h.shtml A k¨ oz´episkolai tan´ arverseny feladatai az okt´ oberi sz´ amban megjelentek.

K¨ oz´ episkolai Matematikai ´ es Fizikai Lapok, 2022/8

3

(E) 4 .

2 ut ´es 8. Egy t´eli napon egy f˝ utetlen v´ ar´ oteremben az emberek 5 r´esze visel keszty˝ 3 r´ e sz¨ u k¨ o n van sapka. Legal´ a bb h´ a ny olyan ember lehet a teremben, akiken keszty˝ u ´es 4

Az ´altal´anos iskolai tan´arok† verseny´enek feladatai



2

(D) 3 ;

7. 12 ember u oralak´ u asztaln´ al, lovagok ´es l´ ok¨ ot˝ ok. A lovagok mindig igazat ¨l egy k¨ mondanak, a l´ ok¨ ot˝ ok mindig hazudnak. Az emberek egyszer csak besz´elgetni kezdenek. Az els˝ o szem´ely azt mondja: Enn´el az asztaln´ al nincsenek lovagok.” Erre a m´ asodik ” ember azt v´ alaszolja: Legfeljebb egy lovag u al.” A harmadik ember azt mondja: ¨l az asztaln´ ” Legfeljebb k´et lovag u al.” A felsz´ olal´ asok a tov´ abbiakban a lovagok sz´ am´ anak ¨l az asztaln´ ” fels˝ o hat´ ar´ at mindig eggyel-eggyel n¨ ovelik, m´ıg v´eg¨ ul a 12. ember azt mondja, hogy Legfeljebb 11 lovag u al.” H´ any lovag van a 12 szem´ely k¨ oz¨ ott? (A) 4; (B) 5; ¨l az asztaln´ ” (C) 6; (D) 7; (E) 8.

Eger, 2022. j´ ulius 5–8.

2·3+4·5

3

(C) 5 ;

6. Petra, R´eka, Viki ´es Dia j´ o bar´ atn˝ ok. Egy k¨ oz¨ os kir´ andul´ asukra Dia elfelejtett 1 1 1 p´enzt hozni mag´ aval, ez´ert Petra a p´enz´enek 5 -´et, R´eka az 4 -´et, Viki pedig az 3 -´ at k¨ olcs¨ onadta neki. Dia ´ıgy mindh´ arom bar´ atn˝ oj´et˝ ol ugyanannyi p´enzt kapott. A csoport 1 1 1 2 1 p´enz´enek h´ anyad r´esz´e ker¨ ult Di´ ahoz? (A) 10 ; (B) 4 ; (C) 3 ; (D) 5 ; (E) 2 .

61. R´atz L´aszl´ o V´andorgy˝ ul´es

kifejez´est, h´ anyf´ele k¨ ul¨ onb¨ oz˝ o ´ert´eket kaphatunk? (E) 6.

1

(B) 2 ;

D + E + F = 4,

461

462

K¨ oz´ episkolai Matematikai ´ es Fizikai Lapok, 2022/8

E + F + G = 5,

– minden sorban a sz´ amok szorzata 1, – minden oszlopban a sz´ amok szorzata 1, – b´ armely 2 × 2-es n´egyzetben a sz´ amok szorzata 2. Melyik sz´ am ´ all a k¨ oz´eps˝ o mez˝ oben? (A) 2; (B) 4; (C) 8;

F + G + H = 6, G+H +I =7 egyenl˝ os´egeket. Mennyi A + E + I ´ert´eke?

(A) 3;

(B) 3,5;

(C) 4;

(D) 4,5;

(E) 5.

15. Az ´ abra szerinti ABCD n´egysz¨ ogben CD = DA, ABC = CDA = DEB = 90◦ ´es DE = 5. Mekkora az ABCD n´egysz¨ og ter¨ ulete? (A) 20; (B) 24; (C) 25; (D) 28; (E) 30.

16. Egy vir´ agcsokor feh´er ´es piros r´ ozs´ at, tov´ abb´ a feh´er ´es piros szegf˝ ut tartalmaz. A feh´er vir´ agok egyharmada r´ ozsa, a piros vir´ agok h´ aromnegyede szegf˝ u, a vir´ agok hattizede feh´er. A vir´ agok h´ any %-a szegf˝ u? (A) 15; (B) 30; (C) 40; (D) 60; (E) 70. 17. Jel¨ olje n´egy egy s´ıkban lev˝ o, egym´ ast´ ol p´ aronk´ent k¨ ul¨ onb¨ oz˝ o egyenes eset´en n azon pontok sz´ am´ at, amelyek illeszkednek k´et vagy t¨ obb egyenesre. Mennyi n ¨ osszes lehets´eges ´ert´ek´enek ¨ osszege? (A) 14; (B) 16; (C) 18; (D) 19; (E) 21.

20. A P (6; 8) ponton a ´thalad´ o e ´es f egyenesek olyan Q ´es R pontban metszik az y tengelyt, amelyekre OP = OQ = OR, ahol O a koordin´ atarendszer kezd˝ opontja. Mekkora a P QR h´ aromsz¨ og ter¨ ulete? (A) 45; (B) 48; (C) 54; (D) 60; (E) 72.

26. Egy 3 × 3-as n´egyzetr´ acs mez˝ oit kit¨ oltj¨ uk a  ´es  jelekkel. Az al´ abbi ´ abra egy ilyen kit¨ olt´est mutat be, melyen h´ arom  egy vonalban helyezkedik el. H´ any olyan kit¨ olt´ese van a n´egyzetr´ acsnak, amely eset´en h´ arom  ´es h´ arom  is egy egyenes ment´en helyezkedik el? (A) 39; (B) 42; (C) 78; (D) 84; (E) 96. 27. Az ABC h´ aromsz¨ ogben AB = 20, BC = 25 ´es CA = 17. Adott a h´ aromsz¨ og s´ıkj´ aban egy P pont. Mekkora a 2P A + 3P B + 5P C hossz´ us´ ag minim´ alis ´ert´eke? (A) 115; (B) 109; (C) 100; (D) 96; (E) 91. 28. Az al´ abbi ´ abr´ an egy ABCDEF GH soksz¨ og l´ athat´ o, mely t´eglalapokb´ ol ´es der´eksz¨ og˝ u h´ aromsz¨ ogekb˝ ol ´ all. A soksz¨ oget kiv´ agva ´es a szaggatott vonalak ment´en ¨ osszehajtogatva egy h´ aromsz¨ og alap´ u has´ abot kapunk. Ha AH = EF = 8 ´es GH = 14 egys´eg, akkor h´ any t´erfogategys´eg a has´ ab t´erfogata? (A) 112; (B) 128; (C) 192; (D) 240; (E) 288. 29. Egy t´eglalap alak´ u padl´ o 17 m hossz´ u, 10 m sz´eles ´es 170 db 1 m × 1 m-es csemp´evel van burkolva. Egy bog´ ar egyenes vonalban ´ ats´et´ al az egyik sarokb´ ol a szemk¨ ozti sarokba. Az els˝ o ´es az utols´ o lapot is figyelembe v´eve h´ any lapon halad kereszt¨ ul a bog´ ar? (A) 17; (B) 25; (C) 26; (D) 27; (E) 28. 30. Egy sz¨ ocske v´eletlenszer˝ uen ugr´ al n´egy lev´elen, ´es minden ugr´ as´ aval egyenl˝ o val´ osz´ın˝ us´eggel jut el a m´ asik h´ arom lev´el valamelyik´ere. Mi a val´ osz´ın˝ us´ege annak, hogy a sz¨ ocske 4 ugr´ as ut´ an visszajut arra

21. R´ oza egy k¨ ort 12 k¨ orcikkre oszt fel. Ezen k¨ orcikkekhez tartoz´ o k¨ oz´epponti sz¨ ogek nagys´ aga fokban m´erve eg´esz sz´ am, ´es sz´ amtani sorozatot alkot. H´ any fok lehet a k¨ oz´epponti sz¨ ogek k¨ oz¨ ul a legkisebb sz¨ og minim´ alis ´ert´eke? (A) 5; (B) 6; (C) 8; (D) 12; (E) 19. 22. B´ armely K-bet˝ ub˝ ol indulva, ´es oldalban szomsz´edos n´egyzetek anyf´elek´eppen olvashat´ o fel´e balra, jobbra, felfel´e vagy lefel´e haladva h´ ¨ OK ¨ sz´ ki a KOR o, ha a kiolvas´ asok sor´ an minden bet˝ u k´etszer haszn´ alhat´ o? (A) 12; (B) 24; (C) 108; (D) 126; (E) 144.

(E) 16.

25. Az ABCD n´egyzet oldalainak hossza 8 egys´eg. M a BC oldal azon pontja, melyre CM = 2. Ha N a BD ´ atl´ onak egy v´ altoz´ o helyzet˝ u pontja, akkor√mekkora a CN + M N √ t´ avols´ ag legkisebb ´ert´eke? (A) 8; (B) 6 2, (C) 10; (D) 8 2; (E) 12.

18. Viki ´es D´ avid egy k¨ orvonalon mozgatnak egy-egy b´ abut. A k¨ orvonal 12 ponttal egyenl˝ o hossz´ us´ ag´ u ´ıvekre van felosztva, ´es a pontok az ´ oramutat´ o j´ ar´ asa szerint meg vannak sz´ amozva 1-t˝ ol 12-ig. Viki ´es D´ avid is a 12-es jelz´es˝ u pontb´ ol ind´ıtja a b´ abuj´ at. avid Viki egy fordul´ oban 5 pontnyit halad az o ´ramutat´ o j´ ar´ as´ aval megegyez˝ o ir´ anyban, D´ pedig 9 pontnyit az o ´ramutat´ o j´ ar´ as´ aval ellent´etesen. A j´ at´ek akkor ´er v´eget, ha egy fordul´ o v´eg´en a k´et b´ abu azonos helyre ker¨ ul. H´ any fordul´ o alatt ´er v´eget a j´ at´ek? (A) 6; (B) 8; (C) 12; (D) 14; (E) 24. 19. Egy t´eglalap oldalainak hossza centim´eterben m´erve pozit´ıv eg´esz sz´ amok. Ha ulete pedig k cm, akkor az al´ abbiak k¨ oz¨ ul mekkora nem lehet a t´eglalap ter¨ ulete t cm2 , ker¨ a t+k ¨ osszeg? (A) 100; (B) 102; (C) 104; (D) 106; (E) 108.

(D) 12;

a helyre, ahonnan elindult? 1

(D) 4 ;

2

(A) 9 ;

19

(B) 80 ;

20

(C) 81 ;

7

(E) 27 .

A feladatsort Fony´ on´e N´emeth Ildik´ o ´es Fony´ o Lajos ´ all´ıtott´ ak ¨ ossze, ´es Kiss G´eza lektor´ alta.

Az ´altal´anos iskolai tan´arok verseny´enek eredm´enye ´ ´es Vera. Bea minden 23. Di´ at rendszeresen megl´ atogatja h´ arom bar´ atn˝ oje Bea, Evi ´ minden negyedik napon, Vera pedig minden ¨ harmadik napon, Evi ot¨ odik napon megy el Di´ ahoz. Tegnap mindh´ arom bar´ atn˝ o megl´ atogatta. Az elk¨ ovetkez˝ o 365 napos id˝ oszakban h´ any olyan nap lesz, amikor a h´ arom l´ any k¨ oz¨ ul pontosan ketten keresik fel Di´ at? (A) 48; (B) 54; (C) 60; (D) 66; (E) 72.

1. 2. 3. 4. 5.

´ as Utcai Alt. ´ Isk.), Palk´o L´aszl´o (Budapest, Ald´ Egyed L´aszl´o (Bajai III. B´ela Gimn.), ´ Isk.), B. Varga J´ozsef (Temerin, Petar Koˇci´c Alt. ´ Isk.), R´ ozsahegyi Eszter (Budapest XVI. Ker¨ uleti M´ ora Ferenc Alt. ´ T´ oth Gabriella (Csantav´er, Hunyadi J´ anos Alt. Isk.).

24. Egy 3 × 3-as n´egyzetr´ acs mez˝ oit pozit´ıv sz´ amokkal t¨ oltj¨ uk ki az al´ abbi szab´ alyok szerint: K¨ oz´ episkolai Matematikai ´ es Fizikai Lapok, 2022/8

463

464

K¨ oz´ episkolai Matematikai ´ es Fizikai Lapok, 2022/8

Gyakorl´ o feladatsor emelt szint˝ u matematika ´eretts´egire I. r´esz 1. a) Oldjuk meg a 2 · sin2 x + 3 · cos2 x + 2 · sin x = 0 egyenletet a val´ os sz´amok halmaz´ an. (5 pont) 2 b) Melyek azok a val´ os sz´amok, amelyek eleget tesznek az x − 4x − 5  0 ´es a √ π 2 + < cos 2 3 2 x

egyenl˝ otlens´egnek egyar´ant?

(8 pont)

2. Egy adott hossz´ us´ag´ u szakaszt az aranymetsz´es szerint u ´gy osztunk k´et r´eszre, hogy az eredeti ´es a keletkezett hosszabb szakasz hossz´anak ar´anya megegyezik a keletkezett hosszabb ´es a keletkezett r¨ ovidebb szakasz hossz´ anak ar´any´ aval. Bence szob´ aja egyik fal´ anak hossza 6,5 m´eter, magass´ aga 2,8 m´eter. Ezt a falfel¨ uletet Bence u ´gy szeretn´e lefesteni, hogy f¨ ugg˝olegesen ´es v´ızszintesen is az aranymetsz´esnek megfelel˝ oen osztja fel 4 t´eglalap alak´ u r´eszre u ´gy, Bence fala hogy a bal fels˝o sarok fel´e legyenek a r¨ ovidebb szakaszok. a) Hat´ arozzuk meg az egyes t´eglalapok ter¨ ulet´et. A sz´ amol´as sor´ an az oldalak hossz´ at ´es a ter¨ uleteket is pontosan 3 tizedesjegyre kerek´ıtve adjuk meg. (8 pont) b) Benc´enek otthon 4-f´ele sz´ın˝ u falfest´eke van, ezekb˝ ol v´alogat a fal fest´ese sor´ an. H´ any k¨ ul¨ onb¨ oz˝ o sz´ınez´es lehets´eges, ha az oldallal egym´ ashoz illeszked˝ o t´eglalapoknak k¨ ul¨ onb¨oz˝ o sz´ın˝ ueknek kell lennie? (4 pont) 3. A l´egk¨ ori nyom´ as f¨ ugg a tengerszinten m´erhet˝ o nyom´ as ´ert´ek´et˝ ol (p0 ), a tengerszint feletti m´eterben m´ert magass´ agt´ ol (h) ´es a leveg˝ o Celsius-sk´ al´an m´ert h˝ om´ers´eklet´et˝ ol (T ). A hozz´ arendel´es szab´ alya: p = p0 · e

h −0,0342· T +273

.

a) Mekkora a nyom´ as Bol´ıvia f˝ ov´ aros´ aban, La Pazban 3 600 m´eter magass´ ag(2 pont) ban, ha a tengerszinten 101 500 Pa a nyom´as 20 ◦ C-on?

4. Az AB0 v´ercsoportrendszerben az emberek n´egy alapvet˝ o fenot´ıpusba sorolhat´ ok. A magyarorsz´agi popul´aci´ ot figyelembe v´eve az A v´ercsoport´ uak a n´epess´eg 44%-´ at teszik ki, a 0 v´ercsoport´ uak 40%-ot. A B v´ercsoport´ uak ar´ anya 11%, m´ıg az AB v´ercsoport´ uak mind¨ ossze 5%-ot adnak. Ett˝ ol a csoportos´ıt´ ast´ ol f¨ uggetlen¨ ul a v¨or¨osv´ertestek felsz´ın´en tal´ alhat´ o D antig´en megl´ete eset´en Rh+ v´ercsoportr´ol besz´el¨ unk, a D antig´en hi´ anya eset´en Rh− a v´ercsoport, ahov´a az emberek 15%-a tartozik. a) Igazoljuk R´eka a´ll´ıt´ as´ at, aki azt mondja, hogy a Magyarorsz´agon ´el˝o 9,7 milli´o lakosb´ol mind¨ ossze k¨ or¨ ulbel¨ ul 72 750 ember tartozik a legritk´abb AB Rh− v´ercsoportba. (2 pont) b) Cseng´er˝ ol tudjuk, hogy van D antig´en a v´er´eben. Mekkora val´ osz´ın˝ us´eggel B v´ercsoport´ u Csenge? V´ alaszunkat indokoljuk. (2 pont) c) K´esz´ıts¨ unk k¨ ordiagramot a sz¨ uks´eges k¨ oz´epponti sz¨ ogek meghat´ aroz´ asa ut´ an, amely mutatja a magyar embereket v´ercsoportjuk alapj´ an, figyelembe v´eve mind az AB0 rendszert, mind a D antig´en megl´et´et. (5 pont) d) Egy v´erad´ asr´ ol sz´ol´o telth´ azas el˝oad´ ason a 150 f˝ os teremben f´erfiak, n˝ ok ´es gyerekek u ol kimenne 2 f´erfi, akkor az ott marad´o f´erfiak ´es n˝ ok ¨lnek. Ha a teremb˝ ar´ anya 2 : 3 lenne. Ha a terembe bej¨ onne m´eg 2 gyerek, akkor a n˝ ok pontosan h´aromszor annyian lenn´enek, mint a gyerekek. H´any n˝o vett r´eszt ezen az el˝ oad´ ason? (6 pont) II. r´esz osszeadni a 7-tel osztva 5 marad´ekot ad´ o pozit´ıv eg´esz sz´ amokat 5. a) Elkezdt¨ uk ¨ a legkisebb ilyen tulajdons´ ag´ u sz´ amt´ ol kezdve. H´any tagot adtunk ¨ ossze, ´es mi az utols´o sz´am, ha a kapott ¨ osszeg 54 875? (4 pont) b) Egy m´ertani sorozat hatodik ´es nyolcadik tagja egyar´ ant 6. Sz´ am´ıtsuk ki a sorozat els˝o 35 tagj´ anak o (4 pont) ¨sszeg´et. c) Egy sz´ amtani sorozat h´arom egym´ast k¨ ovet˝o elem´enek ¨ osszege 72. Ha az els˝o sz´ amb´ol elvesz¨ unk 4-et, a k¨ oz´eps˝ ot v´ altozatlanul hagyjuk, az utols´ ohoz pedig hozz´aadunk 16-ot, akkor egy m´ertani sorozat h´ arom egym´ast k¨ ovet˝o tagj´ at kapjuk. Hat´ arozzuk meg a m´ertani sorozat h´ anyados´ at. (8 pont) 6. Tekints¨ uk az f (x) = 4x−14 f¨ uggv´enyt. x−2 a) Adjunk meg egy olyan eg´esz sz´ amot, amelyre az f (x) f¨ uggv´eny helyettes´ıt´esi ´ert´eke is eg´esz sz´ am. (2 pont) b) Bizony´ıtsuk be, hogy pontosan 8 darab r´acsponton halad a´t az f (x) f¨ uggv´eny k´epe a Descartes-f´ele der´eksz¨ og˝ u koordin´ atarendszerben. (8 pont)

√ c) Oldjuk meg a f (x) = x − 3 egyenletet a val´os sz´amok halmaz´an. (6 pont)

b) A K´ekestet˝on 1 014 m´eter magass´ agban h´ any %-os nyom´ asv´ altoz´ as ´eszlelol 22 ◦ C-ra emelkedik? (3 pont) het˝ o, ha a h˝om´ers´eklet 8 ◦ C-r´

7. Egy konyhai m˝ uanyag t¨ olcs´er als´ o r´esze henger alak´ u, bels˝ o´ atm´er˝ oje 18 millim´eter, magass´aga 5 centim´eter. Fels˝ o r´esze a hengerre pontosan illeszked˝o csonkak´ up, amelynek fels˝ o´ atm´er˝ oje 7 centim´eter, illetve magass´aga 4 centim´eter.

c) Milyen magass´ agban m´erhet˝ o fele akkora nyom´ as, mint a tengerszinten, (6 pont) amikor a leveg˝ o h˝ om´ers´eklete 24 ◦ C?

a) A t¨olcs´er alj´ at befogjuk, ´es teljes magass´ ag´ anak 90%-´ aig megt¨ oltj¨ uk v´ızzel. H´any deciliter v´ız lesz a t¨ olcs´erben? (6 pont)

K¨ oz´ episkolai Matematikai ´ es Fizikai Lapok, 2022/8

465

466

K¨ oz´ episkolai Matematikai ´ es Fizikai Lapok, 2022/8

b) Mekkora egy t¨ olcs´er t¨ omege, ha a falvastags´ aga mindenhol 1 millim´eter, g uanyag t´erfogat´ anak kisz´am´ıt´as´ahoz haszn´aljuk a m˝ uanyag s˝ ur˝ us´ege 0,92 cm3 ? A m˝ azt a k¨ ozel´ıt´est, amely szerint a t¨ olcs´er bels˝o felsz´ın´et szorozzuk a falvastags´ aggal. (4 pont) c) L´ezerf´ennyel fel¨ ulr˝ ol f¨ ugg˝olegesen belevil´ag´ıtunk a t¨olcs´erbe. Mekkora a val´ osz´ın˝ us´ege, hogy a l´ezerf´eny a t¨ olcs´er als´ o ny´ıl´as´ an j¨on ki? (3 pont)

A K ´es L halmazokat ´ertelmezz¨ uk a k¨ ovetkez˝ ok´eppen: K := x | x ∈ Df ´es f (x)  0 ; L := x | x ∈ Dg ´es g(x)  0 . b) Adjuk meg a K ∩ L, K \ L ´es (K ∪ L) \ K halmazokat. (12 pont) ´ azolva a f¨ Megold´as. a) Abr´ uggv´enyeket, leolvassuk a metsz´espontok abszcissz´ ait, amelyek az egyenlet megold´ asai: x1 = 0; x2 = 3.

d) 50 darab t¨ olcs´erb˝ol a´tlagosan 2 anyaghib´asat k´esz´ıt a gy´ art´ osor. Mekkora a val´osz´ın˝ us´ege, hogy 135 darab elk´esz´ıtett t¨ olcs´er k¨oz¨ott van anyaghib´ as? A v´ alaszt n´egy tizedesjegyre kerek´ıtve adjuk meg. (3 pont)

Ellen˝ orz´es: f (0) = g(0); f (3) = = g(3). Az algebrai megold´as is a fenti eredm´enyekhez vezet.

8. a) Sheldon Cooper kedvenc sz´ama a 73, mert ez a 21. pr´ım ´es 7 · 3 ´eppen 21. S˝ ot, a 73 kettes sz´ amrendszerbeli alakja palindromsz´am, vagyis visszafel´e olvasva az eredetivel azonos. Igazoljuk ez ut´ obbi kijelent´est. (2 pont) b) Egy adott alap´ u, ´es az enn´el 2-vel nagyobb alap´ u sz´amrendszerben tekints¨ uk u h´aromjegy˝ u sz´ amokat, ezek ¨ osszege 69610 . Adjuk meg az ¨osszeadand´ o a 345 alak´ sz´ amok ´ert´ek´et a 10-es sz´ amrendszerben fel´ırva. (8 pont) c) V´eletlenszer˝ uen kiv´alasztunk egy 10-es sz´ amrendszerbeli h´ aromjegy˝ u sz´ amot. Mekkora a val´ osz´ın˝ us´ege, hogy a sz´ am 9-es sz´ amrendszerbeli alakja is h´ aromjegy˝ u? (6 pont) ´ azoljuk koordin´ 9. a) Abr´ atarendszerben a k¨ ovetkez˝ o A ponthalmazt: (3 pont) A = (x; y) ∈ R2 | 4x + 3y  15 . ´ azoljuk koordin´ b) Abr´ atarendszerben a B ponthalmazt: B = (x; y) ∈ R2 | x2 + y 2 − 14x − 8y + 40  0 .

b) Szint´en az ´ abr´ ar´ ol ad´ odik a K ´es az L halmaz, K ∩ L = {0} ∪ ]2; 4], illetve K \ L = ]0; 2[. Mivel K ∪ L = R, ol K = ez´ert (K ∪ L) \ K = K, amelyb˝ = ]−∞; 0[ ∪ {2} ∪ ]4; +∞[. 2. Egy h´ aromsz¨ og cs´ ucsai: A(−2; −2), B(7; 1), C(5; 5). a) Mekkora a h´ aromsz¨ og ter¨ ulete? b) Sz´ am´ıtsuk ki a h´ aromsz¨ og s´ ulypontja ´es magass´ agpontja t´ avols´ ag´ anak pontos ´ert´ek´et. (12 pont) ´ azoljuk a pontoMegold´as. a) Abr´ kat, majd foglaljuk t´eglalapba a h´ aromsz¨ oget. A t´eglalap oldalai 7 ´es 9 egys´eg hossz´ uak, ter¨ ulete 63 ter¨ uletegys´eg. A h´aromsz¨ og k¨ or¨ ul lev´ agott der´eksz¨ og˝ u ; h´ aromsz¨ ogek ter¨ ulete: bal fels˝ o´e t1 = 49 2 27 jobb als´ o´e t2 = 2 ; jobb fels˝ o´e t3 = 4, ez´ert a h´ aromsz¨ og ter¨ ulete

(5 pont)

c) Igazoljuk, hogy az F (−3; −4) f´okuszpont´ u v : y = −6 vez´eregyenes˝ u para(4 pont) bola egyenlete y = 0,25x2 + 1,5x − 2,75. aba h´ uzott d) ´Irjuk fel a y = 0,25x2 + 1,5x − 2,75 parabola (−1; −4) pontj´ ´erint˝oj´enek egyenlet´et. (4 pont)

T = 63 − (t1 + t2 + t3 ) = 21.

J´ocsik Csilla Gy˝or

Megold´asv´azlatok a 2022/7. sz´am emelt szint˝ u matematika gyakorl´o feladatsor´ahoz

b) A s´ ulypont koordin´at´ ai: Å ã Å ã −2 + 7 + 5 −2 + 1 + 5 10 4 ; ; S = . 3 3 3 3

I. r´esz

−−→ Az mc egyenes´enek egyenlete: AB(9; 3) =⇒ n(3; 1), ´ıgy 3x + y = 20; az mb −→ egyenes egyenlete: AC(7; 7) =⇒ n (1; 1), teh´ at x + y = 8.

f¨ uggv´eny, amelynek ´ertelmez´esi tartom´ anya 1. Adott az f (x) = x−2 Df = R \ {2}, ´es a g(x) = 3|x − 1| − 3 f¨ uggv´eny, amelynek ´ertelmez´esi tartom´ anya Dg = R. a) Oldjuk meg az f (x) = g(x) egyenletet.

A k´et egyenletb˝ ol ´ all´ o egyenletrendszer megold´ asa a magass´ agpont koordin´at´ait adja meg, amely M (6; 2), teh´ at az SM t´ avols´ ag:  Å √ ã Å ã 4 2 2 17 10 2 . + 2− = 6− 3 3 3

(x2 −4x)(2−x)

K¨ oz´ episkolai Matematikai ´ es Fizikai Lapok, 2022/8

467

468

K¨ oz´ episkolai Matematikai ´ es Fizikai Lapok, 2022/8

3. a) Hat´ arozzuk meg az al´ abbi kijelent´esek logikai ´ert´ek´et, ´ all´ıt´ asainkat indokoljuk. A) Minden pozit´ıv eg´esz sz´ amra teljes¨ ul, hogy az ¨ osszes pozit´ıv oszt´ oj´ anak atlaga kisebb a sz´ ´ am fel´en´el. B) Van olyan n cs´ ucs´ u teljes gr´ af, amelynek h´ aromszor annyi ´ele van, mint az n cs´ ucs´ u fagr´ afnak. b) Fogalmazzuk meg a k¨ ovetkez˝ o ´ all´ıt´ as megford´ıt´ as´ at: Ha P ´es Q, akkor ” nem R.” c) Tegy¨ uk fel, hogy Ha P ´es Q, akkor nem R.” Igaz-e most a Ha R, akkor ” ” nem P ´es nem Q.” ´ all´ıt´ as? V´ alaszunkat indokoljuk. (13 pont) Megold´as. a) A) Hamis. Legyen mondjuk a sz´ am a 2, amelynek pozit´ıv oszt´ oi az 1 ´es a 2, ezek a´tlaga 32 , amely nagyobb, mint a sz´ am fele. (B´armely pr´ımsz´ am eset´en hasonl´ o a helyzet.) B) Igaz. A 6 cs´ ucs´ u teljes gr´ afnak 15 ´ele van, m´ıg a 6 cs´ ucs´ u fagr´afnak 5. A 15 ´eppen h´ aromszorosa az 5-nek. b) Az a´ll´ıt´ as megford´ıt´ asa: Ha nem R, akkor P ´es Q.” ” c) Hamis. Igaz akkor volna, ha Ha R, akkor nem (P ´es Q).” lenne, amely ” egyen´ert´ek˝ u a Ha R, akkor nem P vagy nem Q.” ´ all´ıt´ assal (De Morgan azonoss´ ag). ” 4. A 32 lapos magyar k´ artya egyik leg´erdekesebb j´ at´eka az ulti. (A magyar k´ arty´ aban a sz´ınek: makk, piros, t¨ ok, z¨ old; sz´ınenk´ent ´ asz, kir´ aly, fels˝ o, als´ o, 10-es, 9-es, 8-as ´es 7-es alkotja a 32 lapot.) Ha pl. B´el´ anak a j´ at´ek elej´en leosztott 10 lapj´ ab´ ol egy o tlapos piros ultija van, ez azt jelenti, hogy n´ a la van a piros hetes ´es m´eg ¨ n´egy piros, tov´ abb´ ao t m´ a sik, nem piros lap. ¨ a) Mennyi a val´ osz´ın˝ us´ege, hogy B´el´ anak ¨ otlapos piros ultit osztottak a j´ at´ek elej´en? K´epzelj¨ uk el, hogy 10 j´ ol megkevert magyar k´ artyacsomag van el˝ ott¨ unk ´es mindegyikr˝ ol levessz¨ uk a legfels˝ o lapot. b) Mennyi a val´ osz´ın˝ us´ege annak, hogy a 10 lapb´ ol pontosan 5 piros? c) Ha az el˝ obbi h´ uz´ askor ¨ ot piros lapot h´ uztunk, mennyi a val´ osz´ın˝ us´ege, hogy k¨ oz¨ ott¨ uk legal´ abb egy hetes van? Minden v´egeredm´enyt n´egy tizedesjegyre kerek´ıtve adjunk meg.

(14 pont)

7

b) Egy csomagban 8 piros lap van, ez´ert egyet h´ uzva 14 annak es´elye, hogy 3 us´eg: a kih´ uzott lap piros, ´es 4 , hogy nem piros, ´ıgy a keresett val´osz´ın˝ P (B) =

c) Egyszer˝ ubb kisz´ amolni az esem´eny komplementer´et. Annak a val´ osz´ın˝ us´ege, hogy az ¨ot piros egyike sem a hetes: Å ã5 7 = 0,5129, 8 ´ıgy P (C) = 1 − 0,5129 = 0,4871. II. r´esz 5. a) Vizsg´ aljuk meg monotonit´ as ´es korl´ atoss´ ag szempontj´ ab´ ol az an =

32

K¨ oz´ episkolai Matematikai ´ es Fizikai Lapok, 2022/8

469

n2 + 3n + 2 , n ∈ Z+ 2

sorozatot. a b) Hat´ arozzuk meg n+1 hat´ ar´ert´ek´et, ha n → +∞. an

2

ol a k¨ ovetkez˝ oket tudjuk: bn = n2 + x · n (x ∈ R, c) Mennyi az x, ha a bn sorozatr´ (16 pont) n ∈ Z+ ), valamint (bn+1 − bn )2 = bn + bn+1 ? Megold´as. a) A sorozat szigor´ uan monoton n¨ ovekv˝ o, azaz minden n-re uk az an+1 − an k¨ ul¨ onbs´eget: an+1 > an . N´ezz¨ 2

(n + 1) + 3(n + 1) + 2 n2 + 3n + 2 − = n + 2 > 0, 2 2 ezzel a monotonit´ asra vonatkoz´ o meg´ allap´ıt´ asunkat igazoltuk. A sorozat alulr´ ol korl´atos, minden tagja pozit´ıv, egy als´ o korl´ atnak j´ o pl. a 0 is. (Legnagyobb als´o korl´atja az a1 = 3.) Fel¨ ulr˝ ol nem korl´atos, azaz minden K > 0-hoz tal´ alhat´ o n0 (k¨ usz¨obindex), hogy minden n > n0 eset´en an > K teljes¨ ul. Egy becs¨ ult k¨ usz¨obindexet adunk meg: n < n2 , ha n  2, ez´ert n+3n < an ; 2n < an , 2n > K; 2 n> K → n0 = [ K . 2 2]

Megold´as. a) A piros hetes mell´e a marad´ek h´et pirosb´ ol kell m´eg n´egy, ezt 4 24 f´elek´eppen tudjuk kiv´alasztani, majd a 24 nem piros lapb´ol ¨ot¨ot 5 -f´elek´eppen v´ alaszthatunk hozz´ a, ´ıgy a kedvez˝ o esetek sz´ ama: Å ã Å ã 7 24 k= · = 1 487 640; 4 5 osz´ın˝ us´ege: az ¨ osszes eset sz´ ama pedig: n = 10 = 64 512 240, ez´ert az esem´eny val´ 1 487 640 = 0,0231. P (A) = 64 512 240

Å ã Å ã5 Å ã5 3 1 10 · = 0,0584. · 4 4 5

Megjegyz´es: Az ´eles” k¨ usz¨ obindex n0 = ”

b)

ñ√

ô 8K + 1 − 3 . 2

1 + n5 + n62 an+1 n2 + 5n + 6 = lim lim = lim 2 , n→∞ an n→∞ n + 3n + 2 n→∞ 1 + 3 + 2 n n2

ar´ert´ek´ere vonatkoz´ o tanult t´etel: lim n1 = 0. Ezt, ´es a konvergens sorozatok hat´ n→∞ ismereteket felhaszn´ alva: an+1 = 1. lim n→∞ an 470

K¨ oz´ episkolai Matematikai ´ es Fizikai Lapok, 2022/8

Megoldhatjuk a feladatot k¨ ozvetlen¨ ul a defin´ıci´ o alapj´an vagy a rend˝ orelv” seg´ıt” s´eg´evel is. ñ c)

ô2 2 2 n2 (n + 1) (n + 1) n2 + x(n + 1) − − xn = + xn + + x(n + 1); 2 2 2 2 Å ã 1 1 2 = n2 + 2xn + n + x + ; n+x+ 2 2

7. K´et k¨ oz´episkola sakkbajnoks´ agot rendezett u ´gy, hogy a versenyz˝ ok el˝ osz¨ or a saj´ at iskol´ ajukon bel¨ ul lebonyol´ıtott h´ aziversenyen vettek r´eszt, melynek sor´ an mindenki mindenkivel egy partit j´ atszott. Ezut´ an ker¨ ult sor az iskol´ ak egym´ as elleni k¨ uzdelm´ere, ahol minden versenyz˝ o a m´ asik iskola mindegyik versenyz˝ oj´evel egy m´erk˝ oz´est v´ıvott. Az egym´ as elleni m´erk˝ oz´esek sz´ ama ´eppen annyi volt, mint a k´et h´ aziversenyen ¨ osszesen. a) Iskol´ ank´ent h´ anyan vettek r´eszt a bajnoks´ agban, ha az egyikben k´etszer annyian indultak, mint a m´ asikban?

Megjegyz´es: mindh´ arom sorozat a h´ aromsz¨ ogsz´ amok sorozata, csak a kezd˝ oelemben k¨ ul¨ onb¨ oznek:

K´et riv´ alis asztalitenisz csapat u ´gy d¨ onti el, melyik¨ uk a jobb, hogy kiv´ alasztj´ ak saj´ at maguk k¨ oz¨ ul a legjobbat, majd a k´et legjobb megk¨ uzd egym´ assal a c´ım´ert. A csapaton bel¨ uli kiv´ alaszt´ as u ´n. egyenes kies´eses rendszerben t¨ ort´enik, amelyben oket. A p´ ar j´ atszik egy m´erk˝ oz´est, minden fordul´ o el˝ ott p´ arokba sorsolj´ ak a r´esztvev˝ a gy˝ oztes tov´ abbjut a k¨ ovetkez˝ o fordul´ oba, a vesztes kiesik. Akinek a sorsol´ askor nem marad p´ ar, m´erk˝ oz´es n´elk¨ ul jut a k¨ ovetkez˝ o fordul´ oba. A pingpongban nincs d¨ on¨ tetlen, az utols´ o m´erk˝ oz´es gy˝ oztese lesz a csapat legjobbja. Osszesen 24 m´erk˝ oz´est j´ atszottak, mire kider¨ ult, hogy melyik csapat lett a gy˝ oztes.

{an } = 3, 6, 10, 15, 21, . . . ; {bn }1 = 1, 3, 6, 10, 15, 21 . . . ; {bn }2 = 0, 1, 3, 6, 10, 15, 21 . . . .

b) H´ any j´ at´ekos nevezett a versenyre az egyik, illetve a m´ asik csapatb´ ol, ha az egyikben ¨ ottel kevesebben voltak, mint a m´ asikban? (16 pont)

1 1 n2 + x2 + + 2nx + x + n = n2 + 2nx + n + x + , 4 2 amelyb˝ ol rendez´es ut´ an: x2 = 14 → x1 = 12 ; x2 = − 12 . A kapott k´et sorozat: 2 2 bn = n 2+n , ha x = 12 , vagy bn = n 2−n , ha x = − 12 .

6. a) Milyen sz´ amjeggyel kezd˝ odik a 162022 a t´ızes sz´ amrendszerben fel´ırva ´es mi az utols´ o k´et sz´ amjegye? o 43-mal. (16 pont) b) Igazoljuk, hogy 22023 + 1 oszthat´ Megold´as. a) Meghat´ arozzuk a sz´ am norm´alalakj´ at: 162022 = A · 10n ;

Megold´as. a) Tegy¨ uk fel, hogy az egyik iskol´ ab´ ol n, a m´ asikb´ ol 2n tanul´ o

n

n(n − 1) 2n(2n − 1) + = 2n2 , 2 2

lg 162022 = lg(A · 10n );

2022 · lg 16 = lg A + n,

2434,730 605 = lg A + n, amelyb˝ ol n = 2434 ´es lg A = 0,730 605. Az el˝oz˝ oek alapj´ an A = 100,730 605 ≈ 5,38, u teh´at 162022 ≈ 5,38 · 102434 , azaz a sz´am els˝o jegye 5. A 16 pozit´ıv eg´esz kitev˝oj˝ hatv´ anyainak utols´o sz´ amjegye mindig 6, az utols´o el˝ otti pedig periodikusan ism´etl˝ odik a k¨ ovetkez˝ o szab´ aly szerint: 165k = . . . 76, 165k+1 = . . . 16, 165k+2 = . . . 56, 165k+3 = . . . 96 ´es 165k+4 = . . . 36, k ∈ Z+ . Mivel 2022 = 5 · 404 + 2, ez´ert a 162022 sz´ am 56-ra v´egz˝ odik. b) 2023 = 7 · 172 = 7 · 289, teh´at 22023 + 1 = (27 )

289

+ 1289 = 128289 + 1289 =

= (128 + 1)(128288 − 128287 . . . ± . . . − 128 + 1) = 129 · (eg´esz sz´ am) =

= 3 · 43 · (eg´esz sz´ am), ezzel az ´all´ıt´ ast igazoltuk.

  Megjegyz´es: itt az an + bn = (a + b) an−1 − an−2 b + an−3 b2 . . . ± . . . − abn−2 + bn−1 azonoss´ agot haszn´ altuk, ahol n pozit´ıv p´ aratlan sz´ am. K¨ oz´ episkolai Matematikai ´ es Fizikai Lapok, 2022/8

471

n(n−1)

, m´ıg a m´asodikban nevezett a versenyre. Az els˝ o iskola h´aziverseny´en 2 = 2 2n 2n(2n−1) = m´erk˝ oz´es volt, az egym´ as elleni partik sz´ ama pedig n · 2n, ´ıgy 2 2 fel´ırhatjuk az n ∈ Z+

egyenletet. Rendezve n2 − 3n = 0, ahonnan n = 3. Az egyik iskol´ ab´ ol h´ arman, a m´asikb´ ol hatan vettek r´eszt a bajnoks´ agban.

3

6

Ellen˝ orz´es: 2 = 3 ´es 2 = 15, ´ıgy 3 + 15 = 3 · 6. b) El˝osz¨or bel´atjuk, hogy az egyenes kies´eses bajnoks´ agban n indul´ o eset´en n − 1 m´erk˝oz´est j´ atszanak, mire megkapj´ ak a gy˝ oztest. L´etes´ıts¨ unk k¨ olcs¨ on¨ osen egy´ertelm˝ u megfeleltet´est egy adott meccs vesztese ´es a m´erk˝ oz´es k¨ oz¨ ott. Minden m´erk˝oz´esnek egy ´es csak egy vesztese van, minden kies˝o j´ at´ekos pontosan egy m´erk˝oz´esen vesztett, teh´ at, ha minden veszteshez hozz´arendelj¨ uk azt a m´erk˝ oz´est, amelyiken kikapott, l´etrehoztuk a k¨ olcs¨ on¨ osen egy´ertelm˝ u megfeleltet´est. Mivel a gy˝ oztesen k´ıv¨ ul mindenki vesztes, ez´ert n − 1 vesztes, azaz n − 1 m´erk˝ oz´es volt a bajnoks´ agban. Ezek ut´ an jel¨ olj¨ uk az egyik csapat l´etsz´ am´at (x − 5)-tel, a m´asik´et x-szel, ekkor az egyik csapat h´aziverseny´en (x − 5) − 1, a m´ asik´en x − 1 m´erk˝ oz´est j´atszottak, ha ezek ¨ osszeg´ehez hozz´ aadunk 1-et (a d¨ ont˝ ot), akkor megkapjuk a bajnoks´agban ¨osszesen lej´ atszott m´erk˝ oz´esek sz´ am´at. (x − 6) + (x − 1) + 1 = 24; ebb˝ol x = 15, teh´at az egyik csapat 10, a m´asik 15 j´at´ekossal vett r´eszt a bajnoks´ agban. Ellen˝ orz´es: 9 + 14 + 1 = 24. 472

K¨ oz´ episkolai Matematikai ´ es Fizikai Lapok, 2022/8

8. Egy sz´ amtani sorozat els˝ o¨ ot tagj´ anak ¨ osszege 30; a sorozat els˝ o, m´ asodik ´es negyedik tagja egy m´ertani sorozat h´ arom szomsz´edos eleme. a) Mennyi a sz´ amtani sorozat els˝ o tagja ´es k¨ ul¨ onbs´ege?

Egy KLM N t´eglalap LN ´ atl´ oja a t´eglalapot k´et der´eksz¨ og˝ u h´ aromsz¨ ogre bontja. A KLN h´ aromsz¨ og K-b´ ol indul´ o magass´ aga, bels˝ o sz¨ ogfelez˝ oje ´es s´ ulyvonala legyen rendre m, f , s.

Egy m´ertani sorozat tagjaira fenn´ all, hogy a1 + a3 + a5 = 182 ´es a2 + a4 = −60. b) Hat´ arozzuk meg a sorozat els˝ o tagj´ at ´es h´ anyados´ at. (16 pont)

c) Mekkora sz¨ oget z´ ar be egym´ assal az LN ´ atl´ o ´es a KL oldal, ha 3f 2 = 2ms? (16 pont) Megold´as. a) Az ABC der´eksz¨ og˝ u h´ aromsz¨ ogben a CAB = 15◦ , ez´ert

Megold´as. a) A sorozat harmadik tagja legyen x, ekkor a tagok rendre x − 2d; x − d; x; x + d; x + 2d, ¨ osszeg¨ uk 5x = 30, teh´at x = 6. A 6 − 2d; 6 − d; 6 + d 2 egy m´ertani sorozat egym´ ast k¨ ovet˝o tagjai, ez´ert (6 − d) = (6 − 2d)(6 + d), ebb˝ ol o esetben rendez´es ut´an 3d(d − 2) = 0 k¨ ovetkezik, ahonnan d1 = 0; d2 = 2. Az els˝ a1 = 6, a m´asodikban a1 = 2. Ellen˝ orz´es: a sz´amtani sorozat els˝ o¨ ot tagja: 6; 6; 6; 6; 6, illetve 2; 4; 6; 8; 10. 2 4 b) a1 + a1 q + a1 q = 182 ´es a1 q + a1 q 3 = −60. Ekkor 182 a1 (1 + q 2 + q 4 ) =− , a1 (q + q 3 ) 60

b a sin 15◦ = 12 ´es cos 15◦ = 12 , amelyekb˝ ol a = 11,59 cm; b = 3,11 cm.

b) Az ´atl´ ok felezik egym´ast, teh´ at a f´el´ atl´ o 6 cm. Az a´tl´ ok metsz´espontja, a B pont ´es az m szakasz AC-re es˝ o v´egpontja egy olyan der´eksz¨ og˝ u h´aromsz¨ oget alkot, amelynek az m-mel szemk¨ ozti hegyessz¨ oge 30◦ , ´ıgy sin 30◦ = m → m = 3 cm. 6 A B pont 3 cm-re van az AC ´ atl´ ot´ ol. , m = s · sin 2α. Az OF K c) Az OT K der´eksz¨ og˝ u h´ aromsz¨ ogben sin 2α = m s h´aromsz¨ogben ´ırjuk fel a szinuszt´etelt: √ sin 2α 2 sin 2α f = =⇒ f = s · , ◦ s sin(135 − α) cos α + sin α

amelyb˝ ol rendez´es ut´ an az 30q 4 + 91q 3 + 30q 2 + 91q + 30 = 0 negyedfok´ u egyenletet kapjuk. Ezt visszavezethetj¨ uk m´ asodfok´ ura, ha mindk´et oldal´ at elosztjuk a nem nulla q 2 -nal: Å ã Å ã 1 1 2 30 q + 2 + 91 q + + 30 = 0. q q Legyen q + 1q = A, ekkor 1 1 A2 = q 2 + 2q + 2 , q q

azaz

q2 +

1 = A2 − 2. q2

majd helyettes´ıts¨ uk be ezeket a feladatban megadott k´epletbe: Ç √ å2 2 sin 2α 3· s = 2 · s · sin 2α · s, cos α + sin α

Az egyenlet: 30(A2 − 2) + 91A + 30 = 0,

30A2 + 91A − 30 = 0.

3 sin 2α

3 A1,2 = −91±109 , A1 = q + 1q = − 10 ´es A2 = q + 1q = 10 . Ez ut´ obbinak nincs val´ os 60 3 1 armazik. A sorozat els˝ o tagja a1 1 = 2 gy¨ oke, az els˝ob˝ ol pedig q1 = −3 ´es q2 = − 3 sz´ vagy a12 = 162. Ellen˝ orz´es. Az egyik sorozat 2; −6; 18; −54; 162, a m´ asik ugyanez, ford´ıtott sorrendben. Ekkor 2 + 18 + 162 = 182; −6 + (−54) = −60.

9. Az ABCD t´eglalap a ´tl´ oi 30◦ -os sz¨ oget z´ arnak be egym´ assal.

(sin α + cos α)

= 1,

3 sin 2α = sin2 α + 2 sin α cos α + cos2 α, 3 sin 2α = 1 + sin 2α. Rendezve: 2 sin 2α = 1, sin 2α = 12 → 2α = 30◦ , α1 = 15◦ , vagy 2α = 150◦ , atl´ o a KL oldallal 15◦ -os vagy 75◦ -os sz¨ oget z´ ar be. α2 = 75◦ . Az LN ´ N´emeth L´aszl´ o Fony´ od

a) Mekkora az AB ´es BC oldal, ha AC = 12 cm? b) Milyen messze van a B cs´ ucs az AC ´ atl´ ot´ ol? K¨ oz´ episkolai Matematikai ´ es Fizikai Lapok, 2022/8

2

473

474

K¨ oz´ episkolai Matematikai ´ es Fizikai Lapok, 2022/8

Megmutatjuk, hogy itt (1)

Matematika feladatok megold´asa

´ıgy

  2 (xk , x2k − 1) (αk + β k ) , α2k − αk β k + β 2k =   = α2k + 2αk β k + β 2k , α2k − αk β k + β 2k =     = 3αk β k , α2k − αk β k + β 2k = 3, α2k − αk β k + β 2k = (3, 2x2k − 1).

B. 5109. Legyen x1 = 2,

xn+1 = 4xn − xn−1

x2 = 7,

(n = 2, 3, . . .).

Van-e n´egyzetsz´ am ebben a sorozatban? (6 pont) Javasolta: George Stoica (Saint John, Canada) Megold´as. Pr´ ob´ aljunk meg k¨ ozvetlen k´epletet adni a sorozat tagjaira. Ehhez el˝osz¨ or keress¨ uk a rekurzi´ o megold´ as´ at xn = q n alakban. Ekkor a megadott k´eplet szerint q n = 4q n−1 − q n−2 , q n−2 (q 2 − 4q + 1) = 0. Mivel q = 0, ez´ert a q 2 − 4q + 1 = 0 egyenlet gy¨ okeire lesz sz¨ uks´eg¨ unk. (Azt szoktuk mondani, hogy √ a sorozat karakterisztikus polinomja∗ x2 − 4x + 1.) A polinom √ gy¨ √  nokei ot az xn = 2 + 3 ´es at az xn = 4xn−1 − xn−2 rekurzi´ 2 + 3 ´es 2 − 3 , teh´ √ n  az xn = 2 − 3 sorozatok is kiel´eg´ıtik, ´es ezek b´ armilyen line´ aris kombin´ aci´ oja: √ n √ n   xn = a 2 + 3 + b 2 − 3 , ahol a ´es b ´ert´eke tetsz˝oleges. Nek¨ unk azonban adott a sorozat els˝ o k´et eleme is, teh´ at nem minden a ´es b ´ert´ek lesz megfelel˝ o. Keress¨ uk meg a megfelel˝o a, b ´ert´ekeket. A rekurzi´ odik, hogy √  e alkalmazva √  ad´ √ x0 = 1, ´ıgy 1√= a + b. Mivel ot visszafel´ x1 = 2, 2 = a 2 + 3 + b 2 − 3 = 2(a + b) + 3(a − b) = 2 + 3(1 − 2b), ´ıgy √ 0 = 3(1 − 2b) ⇒ b = 12 ⇒ a = 12 . Teh´ at a sorozat n-edik eleme xn =

√ n √ n  1  2 + 3 + 2 − 3 ). ( 2

√ √ uk, A r¨ ovids´eg kedv´e´ert innent˝ ol legyen α = 2 + 3 , β = 2 − 3 . Megjegyezz¨ √ 2 2 hogy αβ = 2 − 3 = 1. A sorozat elemeire kapott k´eplet szerint: x3k =

α

3k

+β 2

k 3

3k

=

k 3

k

k



2k

k k

(α + β ) α − α β + β (α ) + (β ) = 2 2

2k



=

Ha van n´egyzetsz´am a sorozatban, akkor annak a sorsz´ ama biztosan oszthat´o 3-mal. Tekints¨ uk ugyanis a sorozatot mod 5: 1; 2; 2; 1; 2; 2; 1; 2; 2; . . . . A sorozat rekurzivit´asa miatt ez a ciklus ¨ or¨ okk´e folytat´ odik. Azonban egy n´egyzetsz´am 5-tel osztva csak 0, 1 vagy 4 marad´ekot adhat. Tegy¨ uk fel, hogy a sorozatban legel˝ osz¨ or szerepl˝o n´egyzetsz´am az x3m . Ekkor oje (1) miatt relat´ıv pr´ım, ´ıgy mux3m = xm · (2x2m − 1). De a szorzat k´et t´enyez˝ sz´ aj, hogy k¨ ul¨ on-k¨ ul¨ on is n´egyzetsz´amok legyenek. Ekkor xm is n´egyzetsz´am, ami ellentmond a fenti feltev´es¨ unknek. Lovas M´ arton (Budapest, B´ek´ asmegyeri Veres P´eter Gimn., 9. ´evf.) Megjegyz´es. Nem neh´ez bel´ atni a k¨ ovetkez˝ ot. Tegy¨ uk f¨ ol, hogy egy (xk )k=0,1,2,... sorozatot az xn+k = ak−1 xn+k−1 + ak−2 xn+k−2 + . . . + a0 xn line´ aris rekurzi´ oval (´es els˝ o k elem´enek megad´ as´ aval) defini´ alunk, ahol a0 , a1 , . . . , ak−1 adott konstansok. Ha az xk − ak−1 xk−1 − ak−2 xk−2 − . . . − a0 polinomnak (a rekurzi´ ohoz tartoz´ o karakterisztikus polinomnak) k k¨ ul¨ onb¨ oz˝ o gy¨ oke van: β1 , β2 , . . . , βk , akkor l´eteznek olyan c1 , c2 , . . . , ck konstansok, amelyekkel a sorozat t-edik elem´et (minden t-re) az explicit xt = c1 β1t + c1 β1t + c2 β2t + . . . + ck βkt alakban kaphatjuk meg. 28 dolgozat ´erkezett. 6 pontot kapott 6 versenyz˝ o: Baski Bence, F¨ uredi Erik ´ am, Lovas M´ Benj´ amin, Lengyel Ad´ arton, Seres-Szab´ o M´ arton, Tiderenczl D´ aniel. 4 pontos 1, 3 pontos 3, 2 pontos 14, 1 pontos 3, 0 pontos 1 dolgozat.

B. 5226. Egy h´ aromsz¨ og mindh´ arom oldal´ anak hossza legfeljebb 2 egys´eg. Minden cs´ ucsp´ art ¨ osszek¨ ot¨ unk egy-egy olyan k¨ or´ıvvel, amely egy-egy egys´egsugar´ u k¨ ornek a f´elk¨ orn´el nem hosszabb ´ıve. Igazoljuk, hogy

or´ıvek hossz´ at jel¨ oli. ahol a , b , c a k¨ (5 pont)

L´ asd a Megjegyz´est a megold´ as v´eg´en.

K¨ oz´ episkolai Matematikai ´ es Fizikai Lapok, 2022/8

Teh´at ez a legnagyobb k¨ oz¨ os oszt´ o 1 vagy 3 lehet. Megmutatjuk, hogy nem lehet 3. A sorozat elemeinek mod 3 marad´eka a k¨ ovetkez˝ o mint´ at mutatja (x0 -val kezdve): 1; 2; 1; 2; . . . . Ez a ciklus ¨ or¨ okk´e ism´etl˝odik, mert a sorozat rekurz´ıv. Vagyis at 2x2k − 1 ≡ 1 (mod 3). Ezzel bel´attuk x2k -nak a 3-as marad´eka mindig 1, teh´ (1)-et.

a + b > 2c /3,

αk + β k · (α2k − 1 + β 2k ) = xk · (2x2k − 1). = 2 ∗

(xk , 2x2k − 1) = 1 :

  (xk , x2k − 1) (2xk , 2x2k − 1) = αk + β k , α2k − αk β k + β 2k ,

475

476

K¨ oz´ episkolai Matematikai ´ es Fizikai Lapok, 2022/8

I. megold´as. Az egys´egsugar´ u k¨ orben nagyobb h´ urhoz nagyobb ´ıv tartozik, ez´ert ha a h´aromsz¨ ogben a vagy b nagyobb, mint c, akkor az egyenl˝ otlens´eg nyilv´an teljes¨ ul. Feltehet˝ o teh´at – az egyenl˝ otlens´eg a -re ´es b -re, ´es ´ıgy a-ra ´es b-re val´o szimmetri´aja miatt –, hogy a  b  c. Haszn´ aljuk az ´ abra jel¨ ol´eseit ´es legyen az A ´es B cs´ ucsokhoz tartoz´ o k¨ or´ıv k¨ oz´eppontja D, az A ´es C cs´ ucsokhoz tartoz´ o´e E, v´eg¨ ul a B ´es C cs´ ucsokhoz tartoz´ o´e F . Legyen a BF C = 2α, ahol 0 < 2α  2π. Ekkor a BF C egyenl˝ o sz´ ar´ u h´ aromsz¨ ogben (BF = F C = R = 1) az F -b˝ ol indul´o mao, der´eksz¨ og˝ u h´ aromgass´ agot beh´ uzva k´et egybev´ ag´ a/2 sz¨ oget kapunk, melyekben sin α = BF = a/2, amib˝ ol a = 2 sin α k¨ ovetkezik. Hasonl´oan b = 2 sin β ´es c = 2 sin γ. A h´ aromsz¨ og-egyenl˝ otlens´eg miatt tudjuk, hogy a + b > c. Ebbe behelyettes´ıtve a kapott ´ert´ekeket: 2 sin α + 2 sin β > 2 sin γ, teh´at sin α + sin β > sin γ. Mivel az egys´egsugar´ u k¨ orben nagyobb h´ urhoz nagyobb k¨oz´epponti sz¨og tartozik, ez´ert 2α  2β  2γ ´es ´ıgy α  β  γ.

  Most pedig l´ assuk be, hogy sin 3ϕ  2 sin ϕ, ha teljes¨ ulnek ϕ-re a felt´etelek, azaz 0 < ϕ  π/6. Az add´ıci´ os t´etelekb˝ ol sin (3ϕ) = 3 sin ϕ − 4 sin3 ϕ, amit az egyen3 l˝otlens´egbe be´ırva 3 sin ϕ − 4 sin ϕ  2 sin ϕ. Ezt ´atrendezve: sin ϕ  4 sin3 ϕ, amit ok¨ ot vonva mindk´et oldalb´ ol azt sin ϕ > 0-val egyszer˝ us´ıtve 1  4 sin2 ϕ. Most gy¨ kapjuk, hogy 1  2 sin ϕ, ami 0 < ϕ  π/6 eset´en igaz, hiszen π/6-n´ al egyenl˝os´eg ´all fenn, 0-t´ ol π/6-ig pedig a szinusz f¨ uggv´eny szigor´ uan monoton n˝ o. Ezzel bel´attuk, hogy sin (3ϕ)  2 sin ϕ. Teh´at sin (3ϕ) > 2 sin ϕ ´es 2 sin ϕ > sin γ is igaz, vagyis sin (3ϕ) > sin γ. ´Igy attuk mindig teljes¨ ul az α + β > 2/3 · γ egyenl˝ otlens´eg, azaz a + b > 2/3 · c . Bel´    az ´all´ıt´ast, a + b > 2c /3. (A feladat felt´etele, hogy a h´aromsz¨ og oldalai legfeljebb 2 egys´eg hossz´ uak, csak arra kellett, hogy tudjuk, hogy l´etezik minden oldalhoz egys´eg sugar´ u k¨ or´ıv.) ´ Isk. ´es Gimn., 11. ´evf.) M´ oricz Benj´ amin (Budapesti Fazekas M. Gyak. Alt. II. megold´as. El˝ osz¨ or is id´ezz¨ uk fel, hogy az arkuszszinusz f¨ uggv´eny∗ a [0, 1] intervallumon konvex, azaz b´armely x1 , x2 ∈ [0, 1] eset´en arcsin x1 + arcsin x2 x1 + x2  arcsin . 2 2

Az egys´egsugar´ u k¨ or ker¨ ulete 2π. A P ´es Q v´egpontok k¨oz´e h´ uzott O k¨oz´eppont´ u r¨ ovidebb k¨ or´ıv hossz´at a P OQ sz¨ og seg´ıts´eg´evel sz´ am´ıthatjuk ki: ´ıv hossza = ker¨ ulet ·

M´asodszor megmutatjuk, hogy arcsin x  arcsin(2x)/3 minden 0 < x  1/2 eset´en. A 0 < x  1/2 felt´etelb˝ ol azonnal k¨ ovetkezik, hogy x  4x3 . Ebb˝ ol alva a j´ol ismert sin 3ϕ = 3 sin ϕ − 4 sin3 ϕ azonoss´ agot 3x − 4x3  2x, ´es felhaszn´ kapjuk, hogy

P OQ , 2π

hiszen 2π a teljes sz¨ og. ´Igy az a ´ıv hossza 2π · BF2πC = 2α.

 3 sin (3 arcsin x) = 3 sin (arcsin x) − 4 sin (arcsin x) = 3x − 4x3 

Ugyan´ıgy a b ´ıv hossza AEC = 2β, valamint a c ´ıv hossza ADB = 2γ. ´Igy a bizony´ıtand´ o a´ll´ıt´ as a k¨ ovetkez˝ ovel ekvivalens: 



2α + 2β > 2/3 · 2γ.

Osztva 2-vel azt kapjuk, hogy α + β > 2/3 · γ. Mivel π/2  γ, ez´ert α + β > π/3 eset´en biztosan teljes¨ ul az egyenl˝ otlens´eg. ´Igy el´eg azt az esetet vizsg´ alni, amikor α + β  π/3. Jel¨ olj¨ uk innent˝ ol kezdve (α + β)-t 2ϕ-vel, 0 < 2ϕ  π/3. ´Igy azt kell bel´ atnunk, hogy 2ϕ > 2/3 · γ, azaz 3ϕ > γ. Mivel a [0, π/2] intervallumon a szinusz f¨ uggv´eny uan monoton n˝ o ´es 3ϕ ´es γ is ezen az intervallumon bel¨ ul van, ez´ert ez szigor´ az egyenl˝otlens´eg pontosan akkor teljes¨ ul, amikor a sin (3ϕ) > sin γ egyenl˝ otlens´eg.

 2x = sin (arcsin 2x).

Mivel az arkuszszinusz szigor´ uan monoton n¨ ov˝ o, ´ıgy 3 arcsin x  arcsin 2x k¨ ovetkezik, ahogy ´all´ıtottuk. Ezut´ an r´ at´er¨ unk a feladat megold´ as´ ara. Legyenek az oldalak rendre 2a, 2b ´es 2c, ´es tegy¨ uk fel, hogy a berajzolt k¨ or´ıvek hossza rendre 2α, 2β ´es 2γ, azaz az oldalak a k¨ or´ıvek k¨ oz´eppontjaib´ ol rendre 2α, 2β ´es 2γ sz¨ og alatt l´ atszanak (ha a k¨ oz´eppont illeszkedik valamely oldalra, akkor a megfelel˝ o l´ at´ osz¨ og π). Ekkor

Ha bel´ atjuk, hogy sin (3ϕ) > 2 sin ϕ ´es 2 sin ϕ > sin γ is fenn´ all, akkor k´esz vagyunk. L´assuk be el˝osz¨ or, hogy 2 sin ϕ > sin γ. Bel´ attuk m´ ar, hogy sin α + sin β > sin γ ´es mivel a [0, π/2] intervallumon a szinusz f¨ u ggv´ e ny konk´ av, ez´ert a Jensen  egyenl˝ otlens´eg miatt 2 sin (α + β)/2  sin α + sin β, hiszen α ´es β ebbe  az intervalluma tartozik. Emiatt azt is biztosan tudjuk, hogy 2 sin (α + β)/2 > sin γ, ahol α + β hely´ere (2ϕ)-t be´ırva 2 sin ϕ > sin γ. K¨ oz´ episkolai Matematikai ´ es Fizikai Lapok, 2022/8

477

sin α = a;

sin β = b;

sin γ = c.

´Igy a bizony´ıtand´ o 2α + 2β > 4γ/3 a´ll´ıt´ as az arcsin a + arcsin b > ∗

478

2 arcsin c 3

https://hu.wikipedia.org/wiki/Sz¨ ogf¨ uggv´ enyek#Inverz_f¨ uggv´ enyek

K¨ oz´ episkolai Matematikai ´ es Fizikai Lapok, 2022/8

ekvivalens alakban ´ırhat´o. Felhaszn´alva az a + b > c h´ aromsz¨og-egyenl˝ otlens´eget, valamint az arkuszszinusz f¨ uggv´eny m´ar eml´ıtett szigor´ u monotonit´ as´ at ´es konvexit´ as´at ad´odik, hogy Å ã c 2 a+b arcsin a + arcsin b  2 arcsin > 2 arcsin  arcsin c, 2 2 3

A K pontversenyben kit˝ uz¨ott gyakorlatok ABACUS-szal k¨oz¨os pontverseny 9. oszt´alyosoknak (739–743.)

ahol az utols´o becsl´esn´el a m´ asodik el˝ orebocs´ ajtott ´eszrev´etel¨ unket haszn´ altuk (valamint a 0 < c/2  1/2 nyilv´ anval´ oan teljes¨ ul˝ o o¨sszef¨ ugg´est). Ezzel az a´ll´ıt´ast bel´ attuk. 29 dolgozat ´erkezett. 5 pontot kapott 17 versenyz˝ o: B´enyei Borisz, Chrob´ ak Gerg˝ o, Diaconescu Tashi, Duchon M´ arton, Farkas Izabella, Fazok´ an Marcell, Kalocsai Zolt´ an, Lovas M´ arton, Mohay Lili Veronika, M´ oricz Benj´ amin, Nagy Levente, N´emeth M´ arton, ´ Szak´ acs Abel, Szanyi Attila, Tarj´ an Bern´ at, Wiener Anna, Z¨ ombik Barnab´ as. 4 pontos 1, 3 pontos 2, 2 pontos 3, 1 pontos 3, 0 pontos 3 dolgozat.

B. 5244. Hat´ arozzuk meg azokat az n > 4 eg´esz sz´ amokat, melyekre minden n-n´el kisebb k o sszetett sz´ a mra (k, n) > 1. ¨ (5 pont)

Javasolta: R´ oka S´ andor (Ny´ıregyh´ aza)

Megold´as. Ha egy ilyen n sz´ am nagyobb p2 -n´el, ahol p egy pozit´ıv pr´ımsz´ am, akkor oszthat´onak kell lennie p-vel, k¨ ul¨ onben (p2 ; n) = 1 lenne. Mivel n > 4, ez´ert biztosan p´ aros, k¨ ul¨ onben a 4-gyel relat´ıv pr´ım lenne. Ha 3-mal nem oszthat´o a keresett sz´ am, akkor 32 = 9-n´el kisebbnek kell lennie, valamint 4-n´el nagyobb ´es p´ aros, ez´ert ez csak a 8 lehet. Ha 3-mal is oszthat´o a sz´ am, de 5-tel nem, akkor 52 = 25-n´el kell kisebbnek lennie, vagyis, mivel 2 · 3 = 6-tal oszthat´ o, ez a sz´ am a 6; 12; 18 ´es a 24 is lehet. Ha a sz´ am 5-tel is oszthat´o, de 7-tel nem, akkor 72 = 49-n´el kisebb ´es 2 · 3 · 5 = = 30-cal oszthat´ o, vagyis csak a 30 lehet. Ha 7-tel is oszthat´o, de 11-gyel nem, akkor 30 · 7 = 210-zel oszthat´ o, ugyanakast, mivel ilyen pozit´ıv sz´ am nem kor 112 = 121-n´el kisebb, ´ıgy nem kapunk megold´ l´etezik. Tov´ abbmenve sem tal´ alunk megfelel˝ o sz´ amokat, mivel azt a sz´ amot, amellyel n-nek oszthat´ onak kell lennie, mindig egyre t¨ obbsz¨or¨os´ere, most p´eld´ aul 11-szeres´ere n¨ oveln´enk, m´ıg a k¨ ovetkez˝ o pr´ımsz´ am n´egyzete legfeljebb n´egyszerese az el˝ oz˝ o´enek. Ennek bel´at´ as´ahoz felhaszn´aljuk Csebisev t´etel´et, miszerint egy eg´esz sz´ am ´es k´etszerese k¨ oz¨ ott mindig van pr´ım, vagyis a k¨ ovetkez˝ o pr´ım az el˝oz˝ onek legfeljebb k´etszerese, ´ıgy a n´egyzete legfeljebb n´egyszerese az el˝ oz˝ o pr´ımsz´ am n´egyzet´enek, ez´ert nincs t¨ obb megold´ as. Megmutattuk, hogy csak a 6; 8; 12; 18; 24; 30 sz´amok felelnek meg a feladat felt´eteleinek. ´ ´ Szak´ acs Abel (Budapest, Jedlik Anyos Gimn., 8. ´evf.) dolgozata alapj´an

K. 739. F¨ ul¨ op a k¨ ovetkez˝ o megfigyel´eseket tette az ˝osz egy id˝oszak´ aban: 1. A megfigyelt id˝o alatt 11 napon esett az es˝o. 2. Es˝os d´elel˝ ott¨ ot mindig napos d´elut´ an k¨ ovetett. ¨ 3. Osszesen 9 d´elel˝ ott ´es 12 d´elut´ an volt napos id˝ o. H´any napon nem esett egy´altal´ an? K. 740. Egy 3 × 12-es t´eglalapot szeretn´enk lefedni 12 db 1 × 3-as t´eglalappal. H´anyf´elek´eppen tehetj¨ uk ezt meg? K. 741. Induljunk ki az 1, 2, 3, 4, 5, 6, 7, 8, 9 sz´ amokb´ ol. Egy l´ep´esben kiv´ alaszthatunk k´et sz´ amot, amelyeket 1-gyel megn¨ ovel¨ unk. El lehet-e n´eh´ any l´ep´esben ´erni, hogy mindegyik sz´am a 10-es legyen? K/C. 742. Dani most tanulja az angol a´b´ec´et, ´es el is mondta az els˝ o nyolc bet˝ uj´et (A, B, C, D, E, F, G, H), csak n´emileg rossz sorrendben. A nyolc bet˝ ub˝ ol csak ¨ot¨ot mondott j´ ol (annyiadik bet˝ uk´ent, ah´ anyadik az ABC-ben). H´any ilyen k¨ ul¨onb¨oz˝ o sorrendje van ennek a nyolc bet˝ unek? K/C. 743. Az ABCD t´eglalap BC oldal´ anak felez˝opontja E, CD oldal´ anak D-hez k¨ozelebbi harmadol´ opontja F . Az AE szakasz felez˝opontja G, az EF szakasz E-hez k¨ozelebbi harmadol´ opontja pedig H. H´ anyadr´esze az F GH h´ aromsz¨ og ter¨ ulete az ABCD t´eglalap ter¨ ulet´enek?

 Beku ¨ld´esi hat´arid˝o: 2022. december 10. Elektronikus munkafu ¨zet: https://www.komal.hu/munkafuzet



A C pontversenyben kit˝ uzo¨tt gyakorlatok (742–743., 1738–1742.)

Feladatok 10. ´evfolyamig

¨ Osszesen 59 dolgozat ´erkezett. 5 pontos 42, 4 pontos 5, 3 pontos 5, 2 pontos 4 dolgozat. 1 pontot 1, 0 pontot 1 versenyz˝ o kapott. Nem sz´ am´ıtjuk a versenybe a sz¨ ulet´esi d´ atum vagy a sz¨ ul˝ oi nyilatkozat hi´ anya miatt: 1 dolgozat.

K¨ oz´ episkolai Matematikai ´ es Fizikai Lapok, 2022/8

479

K/C. 742. A sz¨ oveg´et l´ asd a K feladatokn´ al. K/C. 743. A sz¨ oveg´et l´ asd a K feladatokn´ al. 480

K¨ oz´ episkolai Matematikai ´ es Fizikai Lapok, 2022/8

Feladatok mindenkinek C. 1738. Egy term´eszetes sz´ amot nevezz¨ unk kiegyens´ ulyozottnak, ha t´ızes sz´amrendszerben fel´ırva ´eppen annyi sz´ amjegye van, ah´ any k¨ ul¨onb¨oz˝ o pr´ımoszt´ oval rendelkezik. P´eld´ aul a 21 kiegyens´ ulyozott, de a 42 nem. Igaz-e, hogy v´egtelen sok kiegyens´ ulyozott sz´ am van? Javasolta: Kozma Katalin Abig´el (Gy˝or) C. 1739. A val´ os sz´amok halmaz´anak ovebb r´eszhalmaz´ an ´ertel√ lehet˝o legb˝ ´ e s h(x) = [x + 3]. mezz¨ uk a k¨ ovetkez˝ o f¨ uggv´enyeket: f (x) = x + 5 , g(x) = −2x+8 5 Hat´ arozzuk meg a h´ arom f¨ uggv´enygrafikon k¨ oz¨ os pontjainak koordin´at´ ait ([a] az a val´os sz´am eg´eszr´esz´et jelenti, vagyis azt a legnagyobb eg´esz sz´ amot, amely nem nagyobb a-n´ al). Javasolta: B´ır´ o B´ alint (Eger) C. 1740. Az ABCD paralelogramma CD oldal´ an felvessz¨ uk a P bels˝ o pontot, a CD-vel p´ arhuzamos AB oldalon a Q bels˝ o pontot. A P A ´es QD szakaszok metsz´espontja M , a P B ´es QC szakaszok metsz´espontja N . Tegy¨ uk fel, hogy M N ∦ AB, ´es M N a CD egyenes´et az X, AB egyenes´et az Y pontban metszi. Bizony´ıtsuk be, hogy DX = BY . (Amerikai versenyfeladat) Feladatok 11. ´evfolyamt´ol C. 1741. Az ABCD konvex n´egysz¨ og AC ´es BD ´atl´ oinak metsz´espontja M . Lehets´eges-e, hogy az ABM , BCM , CDM , DAM h´aromsz¨ogek ter¨ ulete ebben a sorrendben egy a) nemkonstans sz´ amtani sorozat, b) nemkonstans m´ertani sorozat k¨ ozvetlen egym´ as ut´ani n´egy tagja? Javasolta: B´ır´ o B´ alint (Eger) C. 1742. Tekints¨ uk a k¨ ovetkez˝ o (a val´ os sz´amok halmaz´anak lehet˝o legb˝ ovebb r´eszhalmaz´ an ´ertelmezett) f¨ uggv´enyeket: f0 (x) =

1 , 1−x

B. 5270. n2 darab egys´egnyi oldal´ u szab´alyos h´aromsz¨ ogb˝ ol egy n egys´eg oldal´ u h´aromsz¨oget a´ll´ıtottunk ¨ ossze, ´es a kis h´ aromsz¨ogeket felv´ altva s¨ ot´etre ´es vil´agosra sz´ınezt¨ uk. A h´ aromsz¨ ogekbe be´ırtuk sorban amokat az ´ abra szerint. az 1, 2, 3, . . . , n2 sz´ Mennyi a s¨ot´et h´ aromsz¨ ogekbe ´ırt sz´amok ¨osszege? (3 pont) Javasolta: N´emeth L´ aszl´ o (Fony´ od)

B. 5271. Legyen ABC olyan egyenl˝o sz´ ar´ u der´eksz¨ og˝ u h´ aromsz¨ og, amelyben a C cs´ ucsn´al van a der´eksz¨ og. Jel¨ olj¨ uk ki az AB oldal belsej´eben az A , a BC ´gy, hogy az A B  C  oldal belsej´eben a B  ´es a CA oldal belsej´eben a C  pontokat u h´aromsz¨og hasonl´o legyen az ABC h´ aromsz¨ ogh¨ oz. Mutassuk meg, hogy az AB oldal felez˝ opontja, az A B  szakasz felez˝opontja ´es a C pont egy egyenesre esik. (3 pont)

Javasolta: Hajdu Endre (Sopron) ´es Hujter Mih´ aly (Budapest)

B. 5272. Egy bolha a koordin´ atarendszer (a, b) pontj´ ab´ ol indul, ahol a, b pozit´ıv eg´eszek. Egy-egy ugr´ assal balra vagy lefele mozog egys´egnyit. Addig ugr´al, am´ıg el nem ´eri az x vagy az y tengelyt. A lehets´eges ugr´ assorozatok h´ anyadr´esze v´egz˝ odik az x tengelyen? (4 pont)

Melj´ an D´ avid (Kecskem´et) o ol ¨tlet´eb˝

B. 5273. Kijel¨ olj¨ uk az ABC egyenl˝ o oldal´ u h´ aromsz¨ og AB oldal´ an a D, a BC oldal´ an pedig az E pontot u ´gy, hogy BCD = 45◦ ´es CDE = 30◦ . Mutassuk meg, hogy BE = 2AD.

  valamint fn (x) = f0 fn−1 (x) ,

minden n pozit´ıv eg´eszre. Sz´am´ıtsuk ki f2022 (2022) ´ert´ek´et.

(4 pont)

(Kanadai feladat)



Javasolta: R´ oka S´ andor (Ny´ıregyh´ aza)

B. 5274. Az a < b pozit´ıv eg´eszek szorzata n´egyzetsz´am. Mutassuk meg, hogy van olyan x pozit´ıv eg´esz, amelyre a  x2  b. (5 pont)

Beku ¨ld´esi hat´arid˝o: 2022. december 10. Elektronikus munkafu ¨zet: https://www.komal.hu/munkafuzet

Javasolta: R´ oka S´ andor (Ny´ıregyh´ aza)

B. 5275. Van-e olyan irracion´alis a sz´ am, amelyre a



K¨ oz´ episkolai Matematikai ´ es Fizikai Lapok, 2022/8

A B pontversenyben kit˝ uz¨ott feladatok (5270–5277.)

(5 pont) 481

482



3

racion´ alis?

Javasolta: Hujter B´ alint (Budapest) K¨ oz´ episkolai Matematikai ´ es Fizikai Lapok, 2022/8

B. 5276. Bizony´ıtsuk be, hogy v´egtelen sok olyan pozit´ıv eg´esz k sz´ am l´etezik, amjegyeinek ¨ osszege amelyre 2k sz´ a) kisebb; b) nagyobb, osszege. mint 2k+1 sz´amjegyeinek ¨ (6 pont)

Javasolta: S´ andor Csaba (Budapest)

Legyen f : Z+ → R+ egy tetsz˝ oleges v´egtelenbe tart´ o f¨ uggv´eny. Bizony´ıtsuk X(n)

be, hogy l´eteznek olyan X ´es Y bajt´ arsias halmazok, hogy n tartanak, ´es tetsz˝oleges ε > 0-ra l´etezik olyan n ∈ Z+ , hogy min X(n), Y (n) < ε. f (n)

B. 5277. Az ABC h´ aromsz¨ ogbe ´ırt k¨ or k¨ oz´eppontja I. A BCA k¨or´ıv felez˝opontja F , az F I egyenes a k¨ or¨ ul´ırt k¨ ort m´ asodszor az M pontban metszi. Mutassuk meg, hogy a CM egyenes a´tmegy a be´ırt ´es a k¨ or¨ ul´ırt k¨or k¨ uls˝ o hasonl´ os´agi pontj´ an. (6 pont)

´es

Y (n) n

is a 0-hoz

 Beku ¨ld´esi hat´arid˝o: 2022. december 10. Elektronikus munkafu ¨zet: https://www.komal.hu/munkafuzet

Javasolta: K´ os G´eza (Budapest)



 Beku ¨ld´esi hat´arid˝o: 2022. december 10. Elektronikus munkafu ¨zet: https://www.komal.hu/munkafuzet

 Informatik´ab´ ol kit˝ uz¨ott feladatok Az A pontversenyben kit˝ u z¨ ott nehezebb feladatok (836–838.) A. 836. Minden i ∈ N eset´en legyen Ai , Bi ´es Ci h´arom v´eges ´es p´ aronk´ent diszjunkt r´eszhalmaza N-nek. Tegy¨ uk fel, hogy N minden, A, B ´es C halmazokb´ ol all´ ´ o part´ıci´ oj´ahoz l´etezik i ∈ N u ´gy, hogy Ai ⊂ A, Bi ⊂ B ´es Ci ⊂ C. Bizony´ıtsuk be, hogy ekkor l´etezik v´eges S ⊂ N is, melyre N minden A, B ´es C halmazokb´ ol all´ ´ o part´ıci´ oj´ahoz l´etezik i ∈ S u ´gy, hogy Ai ⊂ A, Bi ⊂ B ´es Ci ⊂ C. Javasolta: Imolay Andr´ as (Budapest)

A. 837. Az A1 A2 A3 A4 tetra´eder minden ´ele ´erint egy G g¨omb¨ot; az Ai cs´ ucsb´ ol a G-hez h´ uzott ´erint˝oszakasz hossza legyen ai . Mutassuk meg, hogy Å 4 i=1

1 ai

ã2

Å ã 4 1 >2 . a2 i=1 i Javasolta: V´ıgh Viktor (Szeged)

A. 838. Az X ⊂ Z+ ´es Y ⊂ Z+ halmazokat bajt´ arsiasnak nevezz¨ uk, ha minden pozit´ıv eg´esz n el˝ o´ all n = xy alakban, ahol x ∈ X ´es y ∈ Y . Jel¨olj¨ uk X(n)-nel ´es Y (n)-nel azt, hogy az X ´es Y halmazoknak h´ any eleme van az els˝o n pozit´ıv eg´esz k¨ oz¨ ott. K¨ oz´ episkolai Matematikai ´ es Fizikai Lapok, 2022/8

483

I. 574. Egy hossz´ u polcon dobozok helyezkednek el sorban egym´ as mellett, melyeket pozit´ıv eg´esz sz´ amok azonos´ıtanak. Egy robot k´epes arra, hogy a polcr´ol levegyen egy dobozt, k´epes arra, hogy mag´ an´ al tartson egy dobozt, ´es k´epes arra, hogy a polcon egy u res helyre elhelyezze a mag´an´al tartott dobozt. Ezenk´ıv¨ ul ¨ a robot a polc elej´et˝ ol a v´eg´eig tud mozogni el˝ ore ´es vissza, ak´ ar u ´gy is, hogy dobozt hoz mag´aval, valamint k´epes arra, hogy mozg´ as k¨ ozben egy polcon l´ev˝ o dobozt egy szomsz´edos u res helyre toljon a ´ t. A robot rendez˝ o algoritmusa a k¨ o vetkez˝ ok szerint ¨ vez´erli a robotot: 1. Elindul a polc elej´et˝ ol, ´es ha van olyan doboz, amely nagyobb azonos´ıt´ o sz´ ammal rendelkezik, mint az ut´ ana k¨ ovetkez˝ o doboz, akkor azt leveszi a polcr´ol. 2. Mozog tov´abb a polc v´ege fel´e, mik¨ ozben tartja a kivett dobozt, ´es minden olyan dobozt a polc eleje fel´e tol a szomsz´edos u ¨res helyre, amely kisebb azonos´ıt´ o sz´ am´ u, mint az a doboz, amit a kez´eben tart. 3. Ha tal´al olyan dobozt, amely nagyobb azonos´ıt´ o sz´ am´ u, mint a kez´eben tartott doboz, akkor azt m´ ar nem tolja a polc eleje fel´e, hanem az azt megel˝ oz˝ ou ¨res an visszat´er a polc elej´ere, ahol helyre leteszi a mag´ an´ al tartott dobozt. Ezut´ az 1. pont szerint kezdi ism´et a m˝ uk¨ od´es´et. 4. Ha a polc elej´et˝ ol indulva nem tal´al olyan dobozt, amely nagyobb azonos´ıt´ o sz´ ammal rendelkezne, mint a k¨ ovetkez˝ o doboz, akkor abbahagyja a rendez´est, mivel a dobozok az azonos´ıt´ o sz´ amok szerint n¨ ovekv˝ o sorrendben vannak. K´esz´ıts¨ unk programot, amely adott dobozok eset´en megadja, hogy a robotnak h´ anyszor kell levennie dobozt a polcr´ol, illetve h´ anyszor kell egy hellyel od´ebb tolnia dobozt! 484

K¨ oz´ episkolai Matematikai ´ es Fizikai Lapok, 2022/8

A program a standard bemenet els˝ o sor´ ab´ol olvassa be a dobozok N sz´ am´at (2  N  20), majd a m´ asodik sor´ ab´ ol a dobozok azonos´ıt´ o sz´ am´at, N darab k¨ ul¨ onb¨ oz˝ o pozit´ıv eg´eszet. A program a standard kimenet egyetlen sor´ aba ´ırja ki, hogy h´ anyszor kellett a robotnak a rendez´es sor´ an levennie egy dobozt, illetve h´ anyszor kellett egy szomsz´edos helyre od´ebb tolnia egy dobozt. P´eld´ ak: Bemenet 4 / 3 6 6 / 8 4 8 / 5 6

(a / jel sort¨ or´est helyettes´ıt) 7 4 5 9 3 1 1 9 7 3 10 2

Kimenet 2 2 6 11 12 13

4. A j´at´ek minden menet´eben a 3. pontban le´ırtak szerint j´ ar el a sz´ aml´ al´o, kiv´eve akkor, ha az o˝ sz´ ama m´ar egyjegy˝ u. Ebben az esetben a j´at´ek v´eget ´er, o˝ a nyertes. Szimul´aljuk a j´at´ekot t´ abl´ azatkezel˝ o seg´ıts´eg´evel az al´ abbi mint´ at felhaszn´alva. Minden sorban sz´am´ıtsuk ki az egyes j´at´ekosok sz´am´at, illetve jelezz¨ uk felt´eteles form´az´ assal, hogy ki a sz´ amol´o j´ at´ekos. A t´ abl´ azatban csak az utols´ o menetig jelenjenek meg adatok, de a munkaf¨ uzet legyen felk´esz´ıtve a lehet˝ o leghosszabb, azaz legt¨obb menetb˝ ol ´ all´ o j´at´ekra is. A megold´ ast a t´ abl´ azatkezel˝ o be´ep´ıtett f¨ uggv´enyeivel k´esz´ıts¨ unk el, az L oszlopt´ ol jobbra seg´edcell´akat haszn´alhatunk, de saj´ at f¨ uggv´enyt vagy makr´ ot ne alahoz hasonl´ oan alak´ıtsuk ki. kalmazzunk. A t´ abl´ azat form´az´ as´ at a mint´

Bek¨ uldend˝o egy t¨om¨ or´ıtett i574.zip ´ allom´anyban a program forr´ ask´odja, valamint a program r¨ ovid dokument´ aci´ oja, amely tartalmazza a megold´ as r¨ovid le´ır´ as´ at, ´es megadja, hogy a forr´ as´allom´any melyik fejleszt˝oi k¨ornyezetben ford´ıthat´ o.

Bek¨ uldend˝o egy t¨ om¨ or´ıtett i575.zip ´ allom´anyban a megold´ ast tartalmaz´ o munkaf¨ uzet ´es a megold´ as r¨ ovid le´ır´ as´ at bemutat´ o dokument´ aci´ o.

I. 575. Egy kisz´ amol´oban t´ız ember ´ all k¨ orben, ´es a k¨ovetkez˝ o szab´ alyok szerint j´atszanak: 1. El˝osz¨ or mindenki gondol egy n´egyjegy˝ u eg´esz sz´ amra. Ezut´ an a j´ at´ek menetekb˝ ol ´ all addig, am´ıg valakinek 0 nem lesz a sz´ama. Ekkor ˝o a kisz´ amol´o nyertese. 2. Az els˝o menet el˝ ott v´eletlenszer˝ uen kisorsolnak egy j´ at´ekost a 10-b˝ ol, ˝o lesz az els˝o menetben a sz´ amol´o”. ” 3. A sz´ amol´o” megn´ezi a sz´ am´anak utols´ o sz´ amjegy´et (legyen ez k), ´es sz´ amol ” at mag´at´ ol indulva k l´ep´est a k¨ orben el˝ ore, ´ıgy kisz´ amolja a k¨ovetkez˝ o saj´ k¨ orben sz´ amol´o” j´at´ekost. Ha k ´ert´eke 0, akkor ism´et o˝ lesz a sz´ amol´o. Ezut´an ” elhagyja a saj´at sz´am´anak utols´ o jegy´et, ´es az ´ıgy kapott sz´ am az o˝ sz´ama.

´ A digit´ I. 576 (E). alis kult´ ura emelt szint˝ u ´eretts´egi vizsga gyakorlati feladatsor´ aban az adatb´azis-kezel´esi feladatot az XAMPP ny´ılt forr´ ask´ od´ u webszerver ´es adatb´ azis-kezel˝o rendszerrel kell megoldani. A vizsg´az´ o SQL-parancsok form´ aj´aban kapja meg az adatb´ azist, a t´ abl´ akat l´etrehoz´ o ´es adatfelt¨ olt˝ o elj´ ar´asokat. Ebben a feladatban az ´eretts´egihez hasonl´ o feladatokat kell megoldani, illetve az adatb´ azis l´etrehoz´ as´at is nek¨ unk kell elv´egezni. A Nemzet M˝ uv´esze d´ıj a legmagasabb m˝ uv´eszi elismer´es, amelyet 2014 ´ota osztanak ki. A jelenlegi ´es a m´ ar elhunyt d´ıjazottakr´ ol a magyar nyelv˝ u Wikip´edia ask´ent haszn´ alhatunk: oldal´ an adatok ´allnak rendelkez´esre, amit forr´ esze. https://hu.wikipedia.org/wiki/A_Nemzet_M v´ Az itt tal´ alhat´ o adatok seg´ıts´eg´evel hozzuk l´etre azt a nemzetmuvesze.sql ´allom´anyt, amelyet v´egrehajtva l´etrej¨ on az adatb´azis, a sz¨ uks´eges t´ abl´ ak a megfelel˝ o sz´ am´ u, t´ıpus´ u, be´ all´ıt´ as´ u mez˝okkel ´es az adatok felt¨ olt´ese is megt¨ ort´enik a t´abl´ akba. ¨ Ugyelj¨ unk arra, hogy kisz´am´ıthat´ o, felesleges adatokat ne t´ aroljunk. A k¨ovetkez˝ o feladatok megold´ o SQL parancsokat r¨ ogz´ıts¨ uk a feladatok v´eg´en z´ar´ ojelben megadott nev˝ u ´es .sql kiterjeszt´es˝ u sz¨ oveges a´llom´anyokban. A lek´erdez´esekben pontosan a k´ıv´ ant mez˝ok szerepeljenek, felesleges mez˝ot ne jelen´ıts¨ unk meg. 1. Ments¨ uk le a megadott webc´ımr˝ ol a Nemzet M˝ uv´esze-d´ıj adatait. or¨ olj¨ uk ki a felesleges, illetve eg´esz´ıtTetsz˝ oleges alkalmaz´ assal rendezz¨ uk ´at, t¨ s¨ uk ki a sz¨ uks´eges adatokkal a t´ abl´ akat. Haszn´alhatunk p´eld´ aul sz¨ ovegszerkeszt˝ ot, t´abl´ azatkezel˝ ot vagy k´esz´ıthet¨ unk saj´ at programot is. Az ´atalak´ıt´ as egyes l´ep´eseit m´as-m´ as programmal is v´egezhetj¨ uk. A rendezett adatokat utols´ o l´ep´esk´ent TXT t´ıpus´ u, tabul´ atorokkal tagolt UTF-8 k´ odol´as´ u egyszer˝ u sz¨ oveges a´llom´anyokk´ent ments¨ uk, amelyek neve a t´ ablanevekkel egyezzen meg. Az ´allom´anyok els˝ o sora tartalmazza a mez˝ oneveket az azonos´ıt´ ashoz. 2. K´esz´ıts¨ unk u ´j adatb´ azist nemzetmuvesze n´even. K´esz´ıts¨ uk el az adatt´ abl´ akat az adatb´ azisban. A l´etrehoz´ as sor´ an ´ all´ıtsuk be a megfelel˝o t´ıpusokat ´es els˝ odleges kulcsokat! (2nemzet)

K¨ oz´ episkolai Matematikai ´ es Fizikai Lapok, 2022/8

485

486

K¨ oz´ episkolai Matematikai ´ es Fizikai Lapok, 2022/8

3. T¨ olts¨ uk be az adatt´ abl´ akba az adatokat a sz¨ oveges a´llom´anyokb´ ol! 4. Lek´erdez´es seg´ıts´eg´evel ´ırassuk ki, hogy Rubik Ern˝o milyen m˝ uv´eszeti a´gban ´es h´ any ´evesen nyerte el a c´ımet. (4rubik) 5. K´esz´ıts¨ unk lek´erdez´est, amely meghat´ arozza, hogy melyik ´evben adt´ ak ki utolj´ara a Nemzet M˝ uv´esze d´ıjat. (5utolso) 6. Lek´erdez´es seg´ıts´eg´evel adjuk meg, hogy ki a legfiatalabb d´ıjazott ´es mennyi id˝ os a lek´erdez´es futtat´ as´ anak pillanat´aban. (6fiatal) 7. Lek´erdez´es seg´ıts´eg´evel adjuk meg, hogy a jelenlegi d´ıjazottak k¨oz¨ ul h´ anyan tartoznak az egyes m˝ uv´eszeti a´gakhoz. A l´etsz´ am mellett a m˝ uv´eszeti a´gak nevei jelenjenek meg. (7stat) 8. Soroljuk fel lek´erdez´es seg´ıts´eg´evel Varga Imr´evel egy¨ utt azoknak a nev´et, akik ak meg a kit¨ untet´est. (8varga) vele azonos ´evben kapt´ 9. List´azzuk ki azon d´ıjazottak nev´et ´es m˝ uv´eszeti a´g´at, akiknek a m˝ uv´eszeti tev´ekenys´ege ebben a k¨ orben ritka, azaz kevesebb, mint 5 szem´elyn´el szerepel az adatb´ azisban. (9ritka) Bek¨ uldend˝o egy t¨ om¨ or´ıtett i576.zip ´ allom´anyban az adatb´azist l´etrehoz´ o sz¨ oveges a´llom´any ´es a feladatok megold´as´ at ad´o lek´erdez´esek. I/S. 66. B´ abel torny´at t¨ obb ´evsz´ azada folyamatosan ´ep´ıtik, ´es (a f¨oldszinten k´ıv¨ ul) m´ ar N emelettel rendelkezik. Hillalum (egy k˝ om˝ uves, akit most vettek fel, hogy seg´ıtsen az ´ep´ıtkez´esen) a f¨ oldszinten a´ll ´es felk´esz¨ ul az ak´ ar t¨obb h´etig tart´ o l´epcs˝ oz´esre, mire feljut a torony legfels˝ o emelet´ere. Mivel a torony minden emelet´en m´ as-m´ as turisztikai l´atv´ anyoss´ ag kapott helyet, Hillalum tudja, hogy egy nap csak D emeletet fog feljebb m´aszni. S˝ot, minden T -edik nap pihen˝ot tart, ´es egy´ altal´ an nem l´epcs˝ ozik aznap. Hillalum csak nappal m´aszik felfel´e, ´ejszaka azonban a k˝om˝ uvesek mindig hozz´ a´ep´ıtenek m´eg X darab emeletet a toronyhoz. Adjuk meg, hogy Hillalumnak h´any napba telik, mire feljut a torony legfels˝o emelet´ere. A bemenet egyetlen sor´ aban az N , D, T ´es X sz´ amok szerepelnek sz´ok¨ozzel elv´ alasztva. A kimenet egyetlen sor´ aban egy sz´ am szerepeljen, hogy h´any nap alatt jut fel Hillalum a torony tetej´ere (vagy -1, ha sosem ´er fel a legfels˝ o emeletre). P´eld´ ak: Bemenet (a / jel sort¨ or´est helyettes´ıt) 7 3 100 1 7 3 2 1 100 5 10 6

Kimenet 3 9 -1

Bek¨ uldend˝o egy is66.zip t¨ om¨ or´ıtett a´llom´anyban a megfelel˝oen dokument´ alt ´es kommentezett forr´ asprogram, amely tartalmazza a megold´ as l´ep´eseit, valamint megadja, hogy a program melyik fejleszt˝ oi k¨ ornyezetben futtathat´ o. A dokument´ aci´ o tartalmazza a megold´ as elm´eleti h´ atter´et, az esetleg felhaszn´alt forr´ asokat. Ne tartalmazzon k´ odr´eszleteket, azok magyar´azata k´ odkommentek form´aj´aban a forr´ asprogramban szerepeljen. S. 165. Egy gyors´ettereml´ anc k´et k¨ ul¨ onb¨ oz˝ o ´etteremben dolgoz´ o alkalmazottja r´aj¨ott, hogy ha a jelenlegi munkahely¨ uk helyett egym´ as munkahely´ere j´ arn´ anak dolgozni, akkor mindkett˝ oj¨ uknek kevesebbet kellene utazni. Szeretn´enek javaslatot tenni a felettes¨ uknek a munkahelyek u ´jraoszt´ as´ ara, de a probl´ema sajnos t´ ul bonyolultnak bizonyult, hogy pap´ıron kisz´amolj´ ak. Adott egy v´ aros u ´th´ al´ozata, mely cs´ ucsokb´ ol ´es az o˝ket ¨ osszek¨ ot˝ o s´ ulyozott ol a´ll. Van tov´ abb´ a valah´any ´etterm¨ unk ´es D alkalmazottunk, akikr˝ ol tudjuk, ´elekb˝ honnan ´es hova j´ arnak dolgozni. A feladatunk u ´gy u ´jraosztani a munkahelyeket, hogy az alkalmazottak munkahelyt˝ol vett t´avols´ ag´ anak ¨ osszege a lehet˝ o legkisebb legyen. (Tegy¨ uk fel, hogy a dolgoz´ oknak egy´eb preferenci´ aja nincs.) A bemenet els˝ o sor´ aban a cs´ ucsok N ´es az ´elek M sz´ ama tal´alhat´ o. A k¨ ovetkez˝ o M sor egy-egy utat ´ır le, a k´et v´egpontj´ anak sorsz´am´aval ´es az ´el s´ uly´ aval (az u ´t hossz´ aval). Ezut´an az alkalmazottak D sz´ ama, majd D sorban az alkalmazottak lakhely´enek ´es munkahely´enek cs´ ucssz´ama tal´alhat´ o. Mindent 0-t´ol indexel¨ unk ´es egy cs´ ucsban legfeljebb egy ´etterem van. A kimenet egyetlen sor´ aban az el´erhet˝ o legkisebb t´ avols´ ag¨ osszeg szerepeljen, ha az u ´jraoszt´ as ut´an minden ´etteremben ugyanannyian dolgoznak, mint el˝ otte. P´elda: Bemenet (a / jel sort¨ or´est helyettes´ıt) 6 5 / 0 1 1 / 1 2 2 / 1 5 1 / 3 4 2 / 4 5 1 2 / 0 5 / 3 2

487

6

Megjegyz´es: Mint ahogy a p´elda is mutatja, el˝ ofordulhat, hogy valaki ´ıgy t¨ obbet fog utazni. Korl´ atok: N  500, M  1000, D  100. Id˝olimit: 1 mp. ´ Ert´ekel´es: A pontok 50%-a kaphat´ o, ha a program helyes kimenetet ad a D  10 esetekre. Bek¨ uldend˝o egy s165.zip t¨ om¨ or´ıtett a´llom´anyban a megfelel˝oen dokument´alt ´es kommentezett forr´ asprogram, amely tartalmazza a megold´ as l´ep´eseit, valamint megadja, hogy a program melyik fejleszt˝ oi k¨ ornyezetben futtathat´ o. A dokument´ aci´ o tartalmazza a megold´ as elm´eleti h´ atter´et, az esetleg felhaszn´alt forr´ asokat. Ne tartalmazzon k´ odr´eszleteket, azok magyar´azata k´ odkommentek form´aj´aban a forr´ asprogramban szerepeljen. A feladatok megold´asai regisztr´aci´o ut´an a k¨ovetkez˝o c´ımen t¨olthet˝ok fel: https://www.komal.hu/munkafuzet Beku ¨ld´esi hat´arid˝o: 2022. december 15.

Korl´ atok: N, D, T, X  109 ; 0  X; 1  N, D; 2  T . Id˝ olimit: 0,4 mp. ´ Ert´ekel´es: a pontok 50%-a kaphat´ o, ha a program helyes kimenetet ad N, D, T, X  100 eset´en. K¨ oz´ episkolai Matematikai ´ es Fizikai Lapok, 2022/8

Kimenet

488

K¨ oz´ episkolai Matematikai ´ es Fizikai Lapok, 2022/8

A ro¨ntgensz´or´as, m´as n´even Bragg-reflexi´o

A krist´ alyos anyagok szerkezetvizsg´ alat´ anak legfontosabb m´odszere a r¨ontgensz´ or´ as. Ennek l´enyege a k¨ ovetkez˝ o. Ha a krist´ alyr´acsban periodikus rendben elhelyezked˝ o atomokat (ionokat, molekul´ akat) r¨ ontgensugarakkal, azaz elegend˝oen r¨ ovid hull´amhossz´ u elektrom´ agneses sugarakkal megvil´ ag´ıtjuk, azok maguk is hull´ amforr´ ass´ a v´ alnak: a bej¨ ov˝o sugarakhoz k´epest valamilyen a´lland´ o f´azissal eltolva, de azonos frekvenci´aval minden ir´anyba sug´aroznak. A sok k¨ ul¨onb¨oz˝ o centrumb´ ol j¨ ov˝o, u ´n. sz´ort sugarak ´altal´ aban kioltj´ak egym´ast, de bizonyos ir´ anyokban, hasonl´ oan az optikai r´acshoz, pozit´ıv interferencia l´ep fel, ´es j´ol detekt´ alhat´ o intenzit´ asmaximumokat ´eszlel¨ unk, ´es ezen maximumok poz´ıci´ oj´ab´ ol hat´ arozz´ ak meg a krist´ aly szerkezet´et.

Az 1. ´ abra a h´ aromdimenzi´ os jelens´egnek a r´ acss´ıkokra mer˝ oleges, a bees˝o (´es ov˝ o ´es kivisszavert”) sugarakkal p´arhuzamos vet¨ ulet´et mutatja. Itt F1 ´es F2 bej¨ ” men˝ o hull´ amfrontok (olyan s´ıkok, amelyeken bel¨ ul a f´azis azonos), a szaggatott vonalak a k´epzeletbeli visszaver˝od´est illusztr´ alj´ ak, az er˝ osebb folytonos vonalak pedig n´eh´ any, a Bragg-felt´etel teljes¨ ul´ese eset´en pozit´ıvan interfer´al´o sug´ armenetet jelen´ıtenek meg. B´ armely k´et r´ acsponthoz tartoz´ o, a bejel¨ oltekhez hasonl´ o sug´armenet u ´tk¨ ul¨ onbs´ege csak att´ ol f¨ ugg, hogy az egyes r´acspontok melyik r´ acss´ıkon helyezkednek el. A krist´alys´ıkok rendszere nemcsak a f˝o krist´alytani ir´ anyokhoz igazodva, hanem – ahogy azt a 2. ´ abra ´erz´ekelteti – sokf´elek´eppen kijel¨ olhet˝ o, ´es ezek o a krist´ alyszerkezetre. (A h´aromdimenzi´ os krist´alyban b´armely ¨osszess´ege jellemz˝ h´ arom, nem egy egyenesbe es˝ o r´ acspont kijel¨ ol egy s´ıkot, amelyre term´eszetesen nagyon ( v´egtelen”) sok m´asik r´ acspont is illeszkedik. A krist´aly periodikuss´ aga miatt ” asik r´ acspontot minden r´ acspontra fektethet˝o egy ezzel p´ arhuzamos, nagyon sok m´ is tartalmaz´ o s´ık, azaz minden r´ acspont illeszkedik egy, az eredetivel egybees˝ o, vagy azzal p´ arhuzamos r´ acss´ıkra. Ezek egy¨ utt alkotnak egy r´ acss´ıksereget.)

Azt, hogy milyen felt´etelek eset´en kapunk pozit´ıv interferenci´at valamilyen arozhatjuk meg. K´epzelj¨ unk el egy ir´ anyban, egy egyszer˝ u anal´ ogia seg´ıts´eg´evel hat´ olyan s´ıkt¨ uk¨ orsereget, amelyben a t¨ ukr¨ ok p´ arhuzamosak, a szomsz´edok t´avols´ aga d, a f´enynek csak egy r´esz´et verik vissza, ´es h´ atulr´ ol” ´atl´ atsz´ oak. Ezt a rendszert a s´ı” kokkal ϑ sz¨ oget bez´ ar´ o, λ hull´amhossz´ us´ag´ u f´ennyel megvil´ag´ıtva pozit´ıv interferenci´ at ´eszlel¨ unk a visszaver˝od´es ir´ any´ aban, ha a szomsz´edos s´ıkokr´ ol visszaver˝od˝o hull´amok u ´tk¨ ul¨ onbs´ege a hull´ amhossz eg´esz sz´ am´ u t¨obbsz¨or¨ose, azaz 2d sin ϑ = nλ,

ahol

2. ´ abra

n = 1, 2, 3, . . . .

ok, de a t´erben periodikus rendben elhelyezkeEgy krist´ alyr´ acsban nincsenek t¨ ukr¨ d˝ o sz´or´ ocentrumokhoz ilyen krist´ alys´ıkok (ahogy mondani szokt´ak: r´ acss´ıkseregek) rendelhet˝ ok, ´es nyilv´anval´ o, hogy ha ezeket t¨ uk¨ ornek k´epzelve egy adott ir´ anyban pozit´ıv interferencia l´epne fel, akkor pozit´ıvan interfer´al a megfelel˝ o s´ıkokban elhelyezked˝ o centrumokr´ ol az adott ir´ anyba sz´ort sug´ arz´ as is.

Ezek ut´ an a szerkezetmeghat´ aroz´ as menete l´enyeg´eben a k¨ ovetkez˝ o: az ismert hull´amhossz´ u monokromatikus sug´arz´ assal m˝ uk¨ od˝ o r¨ ontgendiffraktom´eterbe helyezett mint´ at szisztematikusan forgatj´ak, ´es megkeresik azokat az ir´anyokat, amelyekben f´enyes intenzit´ asmaximum ´eszlelhet˝ o. A sz´ or´ od´as n´elk¨ ul kimen˝ o primer sug´ ar ´es az intenzit´asmaximum ir´anya k¨ oz¨ otti sz¨ og (az elhajl´as sz¨ oge) ´eppen 2ϑ, a k´et sug´ar s´ıkj´ara a sz¨ ogfelez˝ oben a´ll´ıtott mer˝ oleges s´ık (sz¨ ogfelez˝ o s´ık) pedig p´ arhuzamos a megfelel˝ o r´acss´ıkokkal. Ezekb˝ ol ´es a minta pillanatnyi poz´ıci´ oj´ab´ ol a k¨ ul¨ onb¨ oz˝ o r´acss´ıkseregek t´ avols´ agadatai ´es egym´ ashoz viszony´ıtott helyzete meghat´ arozhat´ o, alyszerkezet rekonstru´ alhat´ o. ´ıgy a krist´ Az elj´ar´ as k´et Bragg, apa ´es fia, William Henry Bragg (1862–1942) ´es William Lawrence Bragg (1890–1971) nev´ehez f˝ uz˝ odik, akik ez´ert 1915-ben megkapt´ ak a fizikai Nobel-d´ıjat. A fenti felt´etelt Bragg-egyenletnek nevezik, ´es ´erdekess´egk´eppen megjegyezz¨ uk, a meg´ert´es´ehez haszn´ alt hasonlat annyira beiv´ odott a tudom´anyos k¨oztudatba, hogy a jelens´eget Bragg-reflexi´ onak nevezik, pedig helyesen diffrakci´onak kellene mondani. Gyakorl´ o feladatok 1. Egy r¨ ontgendiffraktom´eter sug´arforr´ asa λ = 154 pm hull´ amhossz´ us´ ag´ u, monokromatikus sug´arnyal´ abot a´ll´ıt el˝ o. Mekkora ennek a berendez´esnek a fel-

1. ´ abra K¨ oz´ episkolai Matematikai ´ es Fizikai Lapok, 2022/8

489

490

K¨ oz´ episkolai Matematikai ´ es Fizikai Lapok, 2022/8

bont´ asa, azaz legal´abb milyen t´ avol vannak egym´ast´ ol azok a r´acss´ıkok, amelyek seg´ıts´eg´evel m´eg ´eppen el˝o´ allhat diffrakci´o? 2. A fenti berendez´essel egy t´ercentr´ alt kocka (m´ as n´even t´ercentr´ alt k¨ob¨os) szerkezet˝ u, 613 pm r´ acs´ alland´ oj´ u krist´ alyt vizsg´ alunk. Adjuk meg a f˝o krist´ alytani tengelyekre (az elemi cell´ak ´eleire) mer˝ oleges r´ acss´ıkokon keletkez˝o elhajl´ asi maximumok ϑ poz´ıci´ oit! 3. Ugyanezen a mint´ an egy m´asik r´ acss´ıksereghez o¨t elhajl´asi maximum tarar be a s´ıkokkal. Azonos´ıtsuk tozik, melyek k¨ oz¨ ul a legmagasabb rend˝ u 62,65◦ -ot z´ a megfelel˝ o r´ acss´ıksereget! Megold´asok 1. A Bragg-egyenletnek nincs megold´ asa, ha d  λ/2, teh´at a berendez´essel csak azok a r´acss´ıkok l´ athat´ ok”, amelyek t´ avols´ aga legal´abb 77 pm. ” 2. Az elemi cell´ak ´eleire mer˝ oleges r´ acss´ıkokokat a cell´ak p´ arhuzamos als´ o, illetve fels˝ o lapjai, ´es a cell´ ak k¨ ozep´en a´tmen˝ o s´ıkok adj´ ak. Ezek t´ avols´ aga az a r´acsalland´ ´ o fele: d = a/2. Ilyen adatokkal a Bragg-egyenletnek n = 1, 2, ´es 3 mellett van megold´ asa. Ezek rendre 14,5◦ , 30,2◦ ´es 48,9◦ . 3. oggel n = 5 mellett d = 433,45 pm ad´ odik, azaz d = 0,7071 a  √ A megadott sz¨ ak lap´ atl´ oira mer˝ oleges r´ acss´ıkok t´avols´ aga.  a/ 2 . Ekkora az elemi cell´ Woynarovich Ferenc Budapest

G. 782. Egy ker´ekp´ ar egyenletesen, 3 m/s sebess´eggel halad v´ızszintes u ´ton. ´ Kerekeinek ´ atm´er˝ oje 70 cm. Abr´ azoljuk a ker´ek k¨ ul¨ onb¨ oz˝ o helyzeteiben az egyik ker¨ uleti pont sebess´egvektorait ´es gyorsul´ asvektorait egy-egy k¨ oz¨ os pontb´ ol indulva, azaz k´esz´ıts¨ uk el a sebess´eg- ´es gyorsul´ ashodogr´ afokat. (A hodogr´ afr´ ol r¨ ovid cikk olvashat´ o a K¨ oMaL honlapj´ an.∗ ) (4 pont)

Vermes Mikl´ os feladata nyom´ an

Megold´as. T´etelezz¨ uk fel, hogy a ker´ekp´ ar jobbra halad ´alland´ o, v0 = 3 m/s nagys´ ag´ u sebess´eggel, ´es a kerekei cs´ usz´ asmentesen g¨ ord¨ ulnek. Az egyik ker´ek valamelyik kiv´ alasztott ker¨ uleti pontj´anak sebess´egvektora a ker´ek tengely´enek v´ızszinab´ol ´es a tengely k¨ or¨ uli forg´as v k ker¨ uleti sebess´egvektor´ ab´ ol tes v t sebess´egvektor´ tev˝odik ¨ossze. Az 1. ´ abr´ an a tengely transzl´aci´ os mozg´as´ anak sebess´egvektor´ at barna, a ker¨ uleti sebess´egeket pedig k¨ ul¨ onb¨ oz˝ o id˝ opontokban (a kiv´alasztott pont k¨ ul¨onb¨oz˝ o helyzeteiben) k´ek nyilak jel¨ olik. A talajjal ´erintkez˝o A pontban a talajhoz viszony´ıtott (ered˝ o) sebess´eg nulla, emiatt |v t | = |v k | = v0 , teh´at az a´br´ an bejel¨ olt valamennyi k´ek sebess´egvektor ugyanakkora nagys´ ag´ u: |v A | = |v B | = |v C | = . . . = |v H | = |v k | = v0 . Az ´abr´ an a ker´ek kiv´ alasztott pontj´ anak a gyorsul´as´ at is jel¨ olt¨ uk (piros nyilakkal). Ezek ir´ anya mindenf´ele lehet, de a nagys´ aguk ugyanakkora: a0 =

Fizika gyakorlatok megold´asa

m v02 = 25,7 2 , R s

hiszen a ker´ek sugara R = 0,35 m. G. 781. Forraljunk vizet egy nagy l´ abosban a t˝ uzhelyen. Tegy¨ unk egy v´ekonyfal´ u poh´ arba csapvizet, majd mer´ıts¨ uk a forr´ asban l´ev˝ o v´ızbe u ´gy, hogy az sehol se ´erintkezzen a l´ abos fal´ aval. Felforr-e a poh´ arban a v´ız, ha elegend˝ oen hossz´ u ideig v´ arunk? (3 pont) Megold´as. Hanyagoljuk el a poh´ ar fala, illetve a leveg˝o okozta h˝ovesztes´eget. A l´abosban l´ev˝ o v´ız csak akkor tud energi´at (h˝ot) a´tadni a poh´ arban l´ev˝ o csapv´ıznek, hogyha k¨ ozt¨ uk h˝ om´ers´eklet-k¨ ul¨ onbs´eg van. Ez´ert a bels˝ o poh´ arban l´ev˝ o csapv´ız tetsz˝ olegesen meg tudja k¨ ozel´ıteni a 100 ◦ C-ot, azonban azt sosem ´eri el. R´ aad´ asul a poh´ arban l´ev˝ o v´ız elforral´as´ ahoz a l´abosban l´ev˝ o v´ıznek m´eg a forr´ ash˝ ot is biztos´ıtania kellene, de ezt h˝om´ers´eklet-k¨ ul¨ onbs´eg n´elk¨ ul (h˝ o´atad´ as hi´any´ aban) nem teheti meg. Teh´ at a poh´ arban l´ev˝ o v´ız biztosan nem forr fel. Klement Tam´ as (P´ecs, Le˝ owey Kl´ara Gimn., 9. ´evf.)

491

2. a ´bra

A sebess´eghodogr´ afot u ´gy kapjuk meg, hogy a sebess´egvektorok kezd˝opontj´at egy k¨oz¨os pontba toljuk el. A ker¨ uleti sebess´egek k´eken jelzett vektorait egy pontb´ol u k¨ or¨ on helyezkednek el (2. a ´bra). Ez felm´erve a vektorok v´egpontjai egy v0 sugar´ ∗

22 dolgozat ´erkezett. Helyes 19 megold´ as. Hib´ as 3 dolgozat.

K¨ oz´ episkolai Matematikai ´ es Fizikai Lapok, 2022/8

1. a ´bra

492

https://www.komal.hu/cikkek/cikklista.h.shtml

K¨ oz´ episkolai Matematikai ´ es Fizikai Lapok, 2022/8

a – szaggatott vonallal jelzett – k¨ or lenne a sebess´eghodogr´ af a ker´ekp´ arhoz r¨ogz´ıtett vonatkoztat´ asi rendszerben. A talajhoz viszony´ıtott sebess´egeket u ´gy kapjuk meg, hogy a k´ek vektorokhoz hozz´ aadjuk (a v´egpontjukb´ ol felm´erj¨ uk) a transzl´aci´ os mozg´ as v´ızszintes, barn´ aval jel¨ olt sebess´egvektor´ at. Az ered˝o sebess´egeket az a´br´ an lila nyilak jel¨ olik. Ezek u k¨ ort rajzolnak ki”, v´egpontjai egy ugyancsak v0 sugar´ ” amelynek k¨ oz´eppontja azonban a ker´ekp´ ar halad´ asi ir´ au k¨ orh¨ oz k´eny´ aban v0 -lal eltol´odott a szaggatott vonal´ pest (ha azonos pontb´ ol m´erj¨ uk fel azokat). A gyorsul´ ashodogr´ afot hasonl´ o m´ odon kapjuk: egy k¨ oz¨ os pontba toljuk el a gyorsul´asvektorok kezd˝ opontj´at. A gyorsul´asvektorok egyforma hossz´ us´ag´ uak, ´ıgy a v´egpontjaik egy 3. ´ abra u k¨ or¨ on helyezkednek el (3. a ´bra). a0 sugar´ A ker´ekp´ ar eg´esze (´es ´ıgy a kerekek tengelye) nem gyorsul, emiatt a gyorsul´ ashodogr´ af ugyan´ ugy n´ez ki mind a ker´ekp´ arhoz, mind pedig a talajhoz r¨ogz´ıtett vonatkoztat´ asi rendszerben. Hruby Laura (Budapest, Veres P´ aln´e Gimn., 10. ´evf.) dolgozat´anak felhaszn´al´as´ aval 16 dolgozat ´erkezett. Helyes 11 megold´ as. Kicsit hi´ anyos (3 pont) 2, hi´ anyos (2 pont) 3 dolgozat.

G. 784. Az al´abbi a´br´ an egyszer˝ u g´epek kavalk´ adj´ at l´ athatjuk. A s´ url´ od´ as, valamint a csig´ ak ´es emel˝ ok t¨ omege elhanyagolhat´ o. Melyik ir´ anyba indul el a legals´ o test?

emelkedik. M´asr´eszt a hengerker´ek forg´ asa miatt az egyoldal´ u emel˝o ´es vele egy¨ utt a mozg´ocsiga lefel´e, ´ıgy a k´etoldal´ u emel˝ o bal oldala felfel´e, a jobb oldali v´egpontja pedig lefel´e mozdulna el. Ez azonban nem lehets´eges, hiszen meg´ allap´ıtottuk, hogy az ´eken l´ev˝ o test felfel´e mozog. Ilyenfajta mozg´ as teh´at nem j¨ ohet l´etre. Lehets´eges-e, hogy az als´o test felfel´e mozduljon el? Ebben az esetben az a´br´ an l´athat´ o k´ek nyilak mutatj´ ak a mozg´asir´ anyokat. Ezek mindenhol ellent´etes ir´ any´ uak, mint a piros nyilak, teh´ at ez a mozg´as sem lehets´eges, mert az ´ek f¨ ol¨ otti test alj´anak lefel´e, a tetej´enek pedig felfel´e kellene mozognia. Teh´ at ilyen mozg´ as sem j¨ohet l´etre. Eddigi megfontol´asaink sor´ an felt´etelezt¨ uk, hogy a fonalak a mozg´as k¨ ozben feszesek maradnak. Elvben elk´epzelhet˝ o lenne, hogy a csig´akhoz ´es az emel˝okh¨ oz csatlakoz´ o fonalak meglazulnak, nem fejtenek ki er˝ ot, de az ´ek jobbra mozog, ´es athatjuk, hogy a legals´ o test emelkedik. Az energiaviszonyokat vizsg´alva k¨onnyen bel´ ez az eset sem val´osulhat meg. Ha ugyanis az ´eken l´ev˝ o test s utat tenne meg, akkor a helyzeti energi´ aja mgs-sel cs¨ okkenne, a m´asik test pedig 10 s-et emelkedne, teh´at a helyzeti energi´ aja 10 mgs-sel n¨ ovekedne. Az eg´esz rendszer helyzeti energi´aja megn˝ one, mik¨ ozben m´eg mozg´ asi energia is megjelenne, teh´ at nem teljes¨ ulhetne az energiamegmarad´ as t¨ orv´enye.

(4 pont)

Megjegyz´es. Ha az ´eken l´ev˝ o test t¨ omege – a feladatban szerepl˝ o adatokkal ellent´etben – t¨ obb mint 10-szer nagyobb lenne, mint az als´ o test t¨ omege, akkor az ´ek jobbra indulna el, mik¨ ozben a fonalak egy r´esze meglazulna.

Megold´as. T´etelezz¨ uk fel, hogy a legals´ o test lefel´e mozog. Rajzoljuk be az ´ abr´ aba piros nyilakkal, hogy ebben az esetben a t¨ obbi test milyen ir´ anyba fog mozogni! K¨ ovess¨ uk nyomon a mozg´ asir´ anyokat eg´eszen az ´eken l´ev˝ o testig u ´gy, hogy el˝osz¨or a hengerker´ekhez csatlakoz´o v´ızszintes, majd a f¨ ugg˝olegesen felfel´e halad´o fon´al ment´en indulunk el.

Minden esetet megvizsg´ altunk, ak´ ar azt t´etelezt¨ uk fel, hogy lefel´e, ak´ ar azt, hogy felfel´e mozog a legals´ o test, mindig ellentmond´ asba u ozt¨ unk. Ez azt jelenti, ¨tk¨ hogy semerre sem tud elmozdulni, az egyszer˝ u g´epek mindegyike nyugalomban marad. Feh´erv´ ari Don´ at (Miskolc, F¨ oldes F. Gimn., 10. ´evf.) ´es Klement Tam´ as (P´ecs, Le˝ ovey Kl´ara Gimn., 9. ´evf.)

A lefel´e s¨ ullyed˝o als´o test a hengerkereket az o´ramutat´o j´ ar´ as´ aval ellent´etes ir´ anyban forgatja, emiatt az ´ek balra fog mozogni, a felette l´ev˝ o test pedig megK¨ oz´ episkolai Matematikai ´ es Fizikai Lapok, 2022/8

493

10 dolgozat ´erkezett. Helyes 6 megold´ as, hib´ as 4 dolgozat.

494

K¨ oz´ episkolai Matematikai ´ es Fizikai Lapok, 2022/8

Megjegyz´es. Az egyenl´ıt˝ on, ahol s´ ulytalans´ ag van, feltehet˝ oen egy´ altal´ an nem n˝ onek f´ ak, vagy ha m´egis, akkor azok ir´ any´ at nem a neh´ezs´egi er˝ o, hanem valami m´ as (pl. a f´eny) hat´ arozhatn´ a meg.

Fizika feladatok megold´asa

30 dolgozat ´erkezett. Helyes 8 megold´ as. Hi´ anyos (1–2 pont) 9, hib´ as 10, nem versenyszer˝ u 3 dolgozat.

P. 5400. A kis herceg egyik g¨ omb alak´ u bolyg´ oja olyan gyorsan forog a tengelye k¨ or¨ ul, hogy az egyenl´ıt˝ oj´en nulla a neh´ezs´egi gyorsul´ as. Milyen ir´ anyban n˝ onek a f´ ak a bolyg´ on? (4 pont)

P. 5404. Egy ide´ alis Carnot-g´ep T1 ´es T2 (T2 < T1 ) h˝ om´ers´eklet˝ u h˝ otart´ alyok seg´ıts´eg´evel (izotermikus ´es adiabatikus ´ allapotv´ altoz´ asokon kereszt¨ ul) ciklusonk´ent W hasznos munk´ at tud v´egezni. Hogyan m´ odosul a h˝ oer˝ og´ep hat´ asfoka, ha a munkahenger dugatty´ uj´ anak kicsiny s´ url´ od´ asa miatt ciklusonk´ent 2q h˝ o fejl˝ odik (q  W ), ´es ez a h˝ o fele-fele ar´ anyban megosztva visszaker¨ ul a h˝ otart´ alyokba?

Megold´as. C´elszer˝ u a bolyg´ ohoz r¨ ogz´ıtett, forg´o koordin´ ata-rendszerb˝ ol szeml´elni a helyzetet. Ebben a rendszerben a gravit´aci´ os gyorsul´as (g grav ) mellett fell´ep ag´ u, a bolyg´ o forg´astengely´ere mer˝ oleges, att´ ol elfele” mutat´ o centriegy rω 2 nagys´ ” ol m´ert t´ avols´ ag, ω pedig a bolyg´o fug´ alis gyorsul´ as (g cf ), ahol r a forg´astengelyt˝ aga csak a bolyg´o k¨oz´eppontj´at´ ol sz¨ ogsebess´ege. A gravit´ aci´ os gyorsul´as g0 nagys´ m´ert t´ avols´ agt´ ol f¨ ugg, teh´at a bolyg´o felsz´ın´en mindenhol ugyanakkora.

(5 pont)

Az ered˝o neh´ezs´egi gyorsul´ as a gravit´ aci´ os ´es a centrifug´alis gyorsul´as vektori ¨ osszege (l´ asd az ´ abr´ at): g = g grav + g cf .

ahonnan

Megold´as. Az ide´ alis Carnot-g´ep hat´ asfoka (a szok´ asos jel¨ ol´esekkel): η=

Qfel =

Tudjuk, hogy az R sugar´ u bolyg´o egyenl´ıt˝ oje (E) ment´en ´es

|g| = 0,

teh´ at fenn´all, hogy g0 = Rω 2 . Tekints¨ unk most egy tetsz˝ oleges P pontot a bolyg´ o felsz´ın´en, ´es legyen az OP egyenesnek a forg´astengellyel bez´art sz¨ oge α. Az a´br´ an jel¨olt koordin´ ata-rendszerben a gyorsul´asok komponensei: ggrav,x = −g0 sin α, gcf,x = rω 2 = Rω 2 sin α,

W , η

illetve

Qle =

1−η W. η

Qfel = Qfel − q =

W − q, η

Qle = Qle + q =

1−η W + q, η

W  = Qfel − Qle = W − 2q. Ezek szerint a m´odosult hat´ asfok:

ggrav,y = −g0 cos α,

η =

gcf,y = 0,

´es ´ıgy valamint gy = −g0 cos α.

Ezek szerint a f´ak ezen a furcsa bolyg´on a forg´astengellyel p´ arhuzamosan n˝ onek, mivel a centrifug´ alis gyorsul´as ellens´ ulyozza a gravit´ aci´ os gyorsul´asnak a forg´ astengelyre mer˝ oleges komponens´et. Az ´eszaki feltek´en teh´ at felfel´e”, a d´elin le” ” fel´e” fognak n˝ oni a f´ ak. ´ ´ ad Gimn., 10. ´evf.) Sepr˝ odi Barnab´ as Bendeg´ uz (Budapest, Obudai Arp´

495

W − 2q W W − 2q = η. = Qfel (W/η) − q W − ηq

Sz´ am´ıtsuk ki, hogy mennyivel kisebb ez a hat´asfok a Carnot-hat´ asfokn´ al: Å ã q W − 2q (2 − η)ηq ≈ η(2 − η) . η − η = 1 − η= W − ηq W − ηq W

gx = ggrav,x + gcf,x = −g0 sin α + Rω 2 sin α = sin α(Rω 2 − g0 ) = 0,

K¨ oz´ episkolai Matematikai ´ es Fizikai Lapok, 2022/8

Qfel − Qle T1 − T2 W = = , Qfel Qfel T1

Ha a h˝oer˝ og´ep nem ide´alis, mert a dugatty´ u s´ url´od´ asa miatt ciklusonk´ent 2q h˝o fejl˝odik, amely fele-fele ar´ anyban visszaker¨ ul a h˝ otart´ alyokba, akkor a felvett ´es leadott h˝ o, valamint a munkav´egz´es a k¨ ovetkez˝ ok´epen m´ odosul:

Felt´etelezz¨ uk, hogy a f´ ak g-vel ellent´etes ir´ anyban n˝ onek.

|g cf | = Rω 2

K¨ ozli: Wiedemann L´ aszl´ o, Budapest

(Az utols´ o l´ep´esn´el kihaszn´ altuk, hogy q  W .)

A h˝ oer˝ og´ep hat´ asfoka teh´ at kicsiny s´ url´od´ as eset´en η  = η − η(2 − η)

496

q , W

K¨ oz´ episkolai Matematikai ´ es Fizikai Lapok, 2022/8

amelyet a h˝ otart´ alyok h˝om´ers´eklet´evel ´ıgy is megadhatunk: η =

P. 5409. Az ´ abr´ an egyszer˝ u g´epek kavalk´ adj´ at l´ athatjuk. A s´ url´ od´ as, valamint a csig´ ak ´es emel˝ ok t¨ omege elhanyagolhat´ o. Mekkora er˝ o ´ebred a fonalakban?

T1 − T2 T2 − T2 q . − 1 2 2 · T1 T1 W

Toronyi Andr´ as (Budapest, Ba´ar-Madas Gimn., 12. ´evf.) 17 dolgozat ´erkezett. 5 pontot kapott Toronyi Andr´ as megold´ asa. Hi´ anyos (1–3 pont) 9, hib´ as 5, nem versenyszer˝ u 2 dolgozat.

P. 5405. K´et k¨ ul¨ on´ all´ o ellen´ all´ ason ¨ osszesen I er˝ oss´eg˝ u´ aram folyik ´ at. Igazoljuk, hogy a k´et ellen´ all´ asra es˝ o¨ osszteljes´ıtm´eny akkor minim´ alis, ha a k´et ellen´ all´ asra es˝ o fesz¨ ults´eg megegyezik! (4 pont)

K¨ozli: Holics L´ aszl´ o, Budapest

Megold´as. Legyen az egyik ellen´all´ as R, a m´asik ellen´ all´as az els˝onek x-szerese, asikon ´atfoly´ o ´aram azaz xR. Az egyik ellen´ all´ ason ´atfoly´ o ´ aramer˝ oss´eg i1 , a m´ er˝ oss´ege i2 = I − i1 . Egy R nagys´ ag´ u ellen´ all´ asra es˝ o teljes´ıtm´eny (´ altal´ anoss´ agban):

(4 pont)

Holics L´ aszl´ o feladata nyom´ an

Megold´as. Haszn´ aljuk az ´ abr´ an l´ athat´ o jel¨ ol´eseket, ´es haszn´ aljuk ki, hogy

P = U · i = R i2 ,

mg = 47 N

´es

tg ϕ =

a 1 = . 5a 5

´ıgy a feladatban szerepl˝o k´et ellen´ all´ as ¨ osszteljes´ıtm´enye:   2 P¨osszes = P1 + P2 = R · i21 + xR · (I − i1 ) = R (1 + x)i21 − 2xI i1 + xI 2 .

b Ismert, hogy az f (x) = ax2 + bx + c alak´ u m´asodfok´ u kifejez´esnek x = − 2a n´ al van sz´els˝ o´ert´eke (a > 0 eset´en minimuma). Ennek megfelel˝ oen P¨osszes legkisebb ´ert´ek´ehez tartoz´ o a´ramer˝ oss´egek:

i1 =

x I, 1+x

i2 = I − i1 =

illetve

1 I. 1+x

´Igy az R1 ellen´ all´ asra es˝ o fesz¨ ults´eg: U 1 = R i1 =

x IR, 1+x

a m´asik ellen´ all´ as fesz¨ ults´ege U2 = (xR) · i2 =

x IR. 1+x

L´atjuk, hogy a legkisebb ¨ osszteljes´ıtm´eny eset´en U1 = U2 , teh´at a feladat sz¨ oveg´eben szerepl˝ o a´ll´ıt´ as val´oban igaz.

Mivel a rendszer eg´esze ´es annak minden egyes r´esze egyens´ ulyban van, a k¨ ovetkez˝ o egyenleteket ´ırhatjuk fel:

Hauber Henrik (Gy˝ or, R´evai Mikl´ os Gimn. ´es Koll., 11 ´evf.) K1 = mg = 47 N,

34 dolgozat ´erkezett. Helyes 17 megold´ as. Kicsit hi´ anyos (3 pont) 3, hi´ anyos (1–2 pont) 5, hib´ as 6, nem versenyszer˝ u 3 dolgozat.

K¨ oz´ episkolai Matematikai ´ es Fizikai Lapok, 2022/8

497

· K3 = 4 · K2 , 498

azaz

K3 = 4K2 ,

K¨ oz´ episkolai Matematikai ´ es Fizikai Lapok, 2022/8

K 4 = K5 = K6 =

1 K3 = 2K2 , 2

c) Hat´ arozzuk meg az optim´ alis menetidej˝ u mozg´ ashoz tartoz´ o sikl´ asi sz¨ oget! m m Adatok: v1 = 2 s , k = 10 s , L = 2 km.

K7 = K5 + K 6 = K3 .

(5 pont)

2 K6 = K2 , F1 = 3 3 2 F2 = F1 + mg = K2 + 47 N, 3 2 K8 = tg ϕ F2 = 9,4 N + K2 . 15

K¨ ozli: Simon P´eter, P´ecs

Megold´as. Az ´ abra alapj´an az emelked´es ´es a sikl´as teljes idej´ere ´erv´enyes: H L . + v1 v2 cos α

t=

Tudjuk tov´abb´ a, hogy a hengerker´ekre hat´ o ered˝ o forgat´ onyomat´ek nulla: Å ã 2 2r1 K1 − 3r1 K2 = r1 K8 = r1 9,4 N + K2 , 15 vagyis 2 K2 . 15 asa, amit a t¨obbi egyenletbe Ennek a line´ aris egyenletnek K2 = 27 N a megold´ vissza´ırva kapjuk: 2 · 47 N − 3K2 = 9,4 N +

K3 = K7 = 108 N,

K4 = K5 = K6 = 54 N,

L cos α = √ H 2 + L2

F8 = 13 N.

´ ´ ad Gimn., 10. ´evf.) Sepr˝ odi Barnab´ as Bendeg´ uz (Budapest, Obudai Arp´ 9 dolgozat ´erkezett. Helyes a Csillingek csapat´ anak, tov´ abb´ a Sepr˝ odi Barnab´ as Bendeg´ uz, Toronyi Andr´ as ´es Vig Zs´ ofia megold´ asa. Hi´ anyos (1–2 pont) 4, hib´ as 1 dolgozat.

P. 5410. A v´ andors´ olyom sz´ arnycsap´ asok n´elk¨ ul is k´epes megtenni nagyobb t´ avols´ agokat. Ilyenkor a mozg´ asa k´et r´eszb˝ ol ´ all. Az els˝ o r´eszben kiterjesztett sz´ arnyakkal k¨ or¨ ozve emelkedik egy f¨ olfel´e ´ araml´ o meleg leveg˝ ooszugg˝ oleges sebeslopban (termikben) v1 f¨ s´eggel. A m´ asodik r´eszben a termiket elhagyva a v´ızszintessel α sz¨ oget bez´ arva alland´ ´ o sebess´eggel siklik a k¨ ovetkez˝ o, L asi set´ avols´ agra l´ev˝ o termikig. A v2 sikl´ bess´eg j´ o k¨ ozel´ıt´essel egyenesen ar´ anyos a sikl´ as v´ızszintessel bez´ art α sz¨ og´enek szinusz´ aval: v2 = k sin α, ahol k egy ismert a ´lland´ o. a) Legal´ abb milyen magasra kell a mad´ arnak emelkednie a termikben, hogy egy emelked´esb˝ ol ´es egy sikl´ asb´ ol ´ all´ o mozg´ as a legr¨ ovidebb ideig tartson? b) Legal´ abb mennyi id˝ ore van sz¨ uks´ege a v´ andors´ olyomnak, hogy az egyik termik alj´ at´ ol eljuthasson a m´ asik termik alj´ aig? K¨ oz´ episkolai Matematikai ´ es Fizikai Lapok, 2022/8

Mivel

499

´es

v2 = k sin α = k √

H , H 2 + L2

az egyik termik alj´at´ ol a k¨ ovetkez˝ o termik alj´aig (1)

t=

H L2 + H 2 + v1 kH

id˝ o alatt jut el a v´ andors´ olyom. Ha valahonnan ismern´enk t ´ert´ek´et, akkor annak seg´ıts´eg´evel az (1) egyenletb˝ol ki tudn´ ank sz´ am´ıtani az emelked´es H magass´ ag´ at. Rendez´es ut´ an egy (H-ra n´ezve) m´asodfok´ u egyenlethez jutunk: (k + v1 ) H 2 − kv1 t H + v1 L2 = 0. Ennek csak akkor van (val´ os) megold´ asa, ha a diszkrimin´ ans nem negat´ıv: k 2 v12 t2 − 4(k + v1 )v1 L2  0, vagyis 2L t k

 

k + v1 = tmin . v1

b) Ezek szerint egy-egy rep¨ ul´esi ciklus legr¨ ovidebb ideje a fenti tmin , ami a megadott sz´ amadatok mellett kb. 980 s, azaz 16,3 perc. 500

K¨ oz´ episkolai Matematikai ´ es Fizikai Lapok, 2022/8

a) A legr¨ ovidebb id˝ oh¨ oz tartoz´o emelked´esi magass´ ag: H=

v1 ktmin =L 2(k + v1 )



A sz´en-dioxid nyom´ as´ at az a´br´ an C-vel jel¨ olt kritikus pont nyom´ asa f¨ ol´e emelve a sz´en-dioxid szuperkritikus ´allapotba ker¨ ul. Itt elt˝ unnek a f´azishat´ arok, vagyis a sz´en-dioxid nem g˝ozk´ent, de nem is folyad´ekk´ent fog viselkedni, teh´at h˝ ut´es hat´ as´ara nem fog cseppfoly´ osodni a fent le´ırt m´odon. Az a´bra alapj´ an a kritikus pont nyom´ asa kb. 75 bar. Teh´ at a sz´en-dioxid 5 bar ´es 75 bar nyom´ as k¨ oz¨ ott cseppfoly´ os´ıthat´ o a fent le´ırt m´odon. (A Wikipedia alapj´ an a pontosabb ´ert´ekek 5,1 bar ´es 73,8 bar).

v1 ≈ 816 m. k + v1

c) A fentebb kisz´ amolt optim´ alis menetid˝oh¨ oz” tartoz´ o sikl´asi sz¨og: ” … v1 ≈ 22◦ . α = arctg k + v1

Toronyi Andr´ as (Budapest, Ba´ar-Madas Ref. Gimn., 12. ´evf.)

T´egl´ as Panna (R´evkom´arom, Selye J. Gimn., 12. ´evf.)

9 dolgozat ´erkezett. Helyes Horv´ ath Zs´ oka, Szab´ o M´ arton ´es Toronyi Andr´ as megold´ asa. Kicsit hi´ anyos (2 pont) 4, hi´ anyos (1 pont) 1, nem versenyszer˝ u 1 dolgozat.

26 dolgozat ´erkezett. Helyes 17 megold´ as. Kicsit hi´ anyos (4 pont) 5, hi´ anyos (1–2 pont) 4 dolgozat.

P. 5412. Ha egy g´ azt (´ alland´ o nyom´ as mellett) leh˝ ut¨ unk, akkor elegend˝ oen alacsony h˝ om´ers´ekleten a g´ az ´ altal´ aban cseppfoly´ osodik (kondenz´ al´ odik, lecsap´ odik). Ez azonban csak bizonyos nyom´ astartom´ anyban t¨ ort´enik ´ıgy. Az ´abra a sz´en-dioxid f´ azisdiagramj´ at” mutatja. Legal´ abb, illetve legfeljebb mekkora nyom´ as mellett t¨ or” t´enik meg a cseppfoly´ osod´ as a fenti m´ odon? Mi t¨ ort´enik, ha a h˝ ut´est enn´el a tartom´ anyn´ al magasabb, illetve alacsonyabb nyom´ ason v´egezz¨ uk?

P. 5413. Egy 20 cm f´ okuszt´ avols´ ag´ u gy˝ ujt˝ olencs´et az ´ abra szerint egy dombor´ u g¨ ombt¨ uk¨ orre helyez¨ unk. Mekkora legyen a t¨ uk¨ or g¨ orb¨ uleti sugara, hogy a lencs´ere f¨ ugg˝ olegesen ´erkez˝ o, p´ arhuzamos f´enynyal´ ab a rendszerr˝ ol val´ o visszaver˝ od´es ut´ an is p´ arhuzamos maradjon? (4 pont)

P´eldat´ ari feladat nyom´ an

I. megold´as. Ha az optikai rendszerre p´arhuzamosan ´erkeznek a f´enysugarak, ´es ugyancsak p´ arhuzamosan haladva t´ avoznak onnan, akkor a rendszer dioptri´aja nulla. A f´enysugarak k´etszer haladnak a´t lencs´en, ´ıgy Drendszer = 2Dlencse + Dt¨uk¨or = 2 ·

1 2 − = 0, flencse Rt¨uk¨or

ahonnan a g¨ombt¨ uk¨ or g¨ orb¨ uleti sugara Rt¨uk¨or = flencse = 20 cm. Peth˝ o Dorottya (Kecskem´eti Katona J. Gimn., 11. ´evf.) II. megold´as. A rendszer forg´ asszimmetri´ aja miatt a visszaver˝od´es ut´an p´ arhuzamosan t´ avoz´o f´enynyal´ ab is f¨ ugg˝oleges. A t¨ uk¨ orr˝ ol visszavert f´enysugaraknak teh´at u ´gy kell a lencs´ehez ´erkezni¨ uk, mintha a lencse f´ okuszpontj´ ab´ol indultak volna, hiszen ekkor t´ avozik a lencs´eb˝ ol f¨ ugg˝oleges, p´ arhuzamos f´enynyal´ ab. (L´ asd m´eg A g˝ oz, g´ az ´es a kritikus h˝ om´ers´eklet” c. r¨ ovid cikket a K¨ oMaL ” honlapj´ an∗ .) (3 pont)

K¨ ozli: Honyek Gyula, Veresegyh´az

Megold´as. Az ´ abr´ an l´ athat´ o, hogy ha a H-val jel¨olt h´armaspont nyom´ asa alatti nyom´ ason l´egnem˝ u sz´en-dioxidot h˝ ut¨ unk, akkor a g´az nem fog cseppfoly´osodni, hanem bizonyos h˝om´ers´ekleten azonnal szil´ ard halmaz´ allapotba megy ´at, megszil´ ardul. A h´ armaspont nyom´ asa az ´ abra alapj´ an kb. 5 bar. ∗

https://www.komal.hu/cikkek/cikklista.h.shtml

K¨ oz´ episkolai Matematikai ´ es Fizikai Lapok, 2022/8

501

A lencs´ere ´erkez˝o p´ arhuzamos f´enynyal´ ab a lencs´en val´ o t¨ or´es ut´ an annak f´okuszpontja fel´e halad tov´ abb. Az el˝obbiekben t´ argyaltaknak megfelel˝ oen a dombor´ u t¨ uk¨orr˝ ol val´o visszaver˝ od´es ut´ an a f´enysugarak ugyanezen az u ´tvonalon, de ford´ıtott ir´anyban haladnak. Ez akkor k¨ ovetkezik be, ha minden egyes f´enysug´ar mer˝ olegeuk¨ orre. Ezek szerint a lencse sen, vagyis a g¨ omb k¨ oz´eppontja fel´e tartva esik a g¨ ombt¨ f´okuszpontj´ anak egybe kell esnie a g¨ omb k¨ oz´eppontj´aval, vagyis a g¨ ombt¨ uk¨ or g¨ orb¨ uleti sugara is 20 cm. 18 dolgozat ´erkezett. Helyes 13 megold´ as. Kicsit hi´ anyos (3 pont) 3, hi´ anyos (2 pont) 2 dolgozat.

502

K¨ oz´ episkolai Matematikai ´ es Fizikai Lapok, 2022/8

P. 5414. F´emdr´ otb´ ol egy R sugar´ u k¨ ort form´ aztunk, ´es ugyanebb˝ ol a dr´ otb´ ol az egyik a ´tm´er˝ ot is elk´esz´ıtett¨ uk. Mekkora legyen az AB = AC ´ıvek hossza, hogy az A ´es B pontok k¨ oz¨ ott m´erhet˝ o ered˝ o ellen´ all´ as megegyezzen a B ´es C pontok k¨ oz¨ ott m´erhet˝ o ered˝ o ellen´ all´ assal? (4 pont)

K¨ozli: G´ asp´ ar Merse El˝ od, Budapest

Megold´as. Legyen a huzal egys´egnyi hossz´ us´ ag´ u darabj´anak ellen´all´asa r. Hat´ arozzuk meg el˝ osz¨ or a B ´es a C pontok k¨ oz¨ otti ered˝ o ellen´ all´ast. Mivel az egyenes vezet´ek v´egpontjai (az elrendez´es szimmetri´aja miatt) ekvipotenci´alisak, k¨oz¨ott¨ uk nem folyik ´aram, az a´tm´er˝ o menti vezet´ek teh´at elt´ avol´ıthat´ o (1. ´ abra).

2. ´ abra

Az RAB = RBC felt´etel szerint fenn´ all, hogy 2Rr

α(π − α) (π − α)(2π + 2α + πα) = Rr. π π(π + 4)

Ennek az egyenletnek az egyik megold´ asa: α1 = π. Ha ez teljes¨ ul, akkor az A, B ´es amunkra ez a gy¨ ok ´erdekC pontok egybeesnek, ´es ´ıgy nyilv´ an RAB = RBC = 0. Sz´ telen, hiszen ebben az esetben nem is besz´elhet¨ unk AB ´es AC ´ıvekr˝ ol. Az egyenlet m´ asik gy¨oke: 2π ≈ 0,69 radi´an, α2 = 6+π ´es ´ıgy a keresett ´ıvhosszak: ˜ = BC ˜ = (π − α2 )R = π 4 + π R ≈ 2,45 R. AB 6+π Soml´ an Gell´ert (P´ecs, Le˝ owey Kl´ara Gimn., 12. ´evf.)

1. ´ abra

Å RBC =

1 1 + 2R(π − α)r 2Rαr

ã−1

23 dolgozat ´erkezett. Helyes 13 megold´ as. Kicsit hi´ anyos (3 pont) 6, hi´ anyos (1–2 pont) 4 dolgozat.

α(π − α) . = 2Rr π

P. 5415. Egy elhanyagolhat´ o ellen´ all´ as´ u, szigetel´es n´elk¨ uoget li huzalb´ ol, a v´ızszintes s´ıkban elhelyezked˝ o, α = 45◦ -os sz¨ bez´ ar´ o, V alakot hajl´ıtunk. Ezt az elrendez´est olyan m´ agneses mez˝ obe helyezz¨ uk, melynek B indukci´ ovektora mer˝ oleges osszef¨ uga v´ızszintes s´ıkra, ´es nagys´ aga a B(t) = B0 /t0 · t ¨ ´lland´ ok. g´es szerint v´ altozik az id˝ oben, ahol B0 ´es t0 ismert a A V alak´ u vezet˝ ore szigetel´es n´elk¨ uli, kezdetben r¨ ogz´ıtett f´emrudat helyez¨ unk az ´ abr´ anak megfelel˝ o m´ odon. A r´ ud egys´egnyi hossz´ us´ ag´ u darabj´ anak ellen´ all´ asa r.

Az A ´es B pontok k¨ oz¨ otti ered˝ o ellen´ all´ ast a 2. ´ abra alapj´an l´ep´esr˝ ol l´ep´esre sz´ amolhatjuk. A fels˝ o´ ag bal oldal´an l´athat´ o p´ arhuzamosan kapcsolt ellen´ all´asok ered˝oje: R1 =

2Rπ 2Rr · Rπr = r. 2Rr + Rπr 2+π

A fels˝o a´g k´et sorosan kapcsolt ellen´ all´ as´ anak ered˝oje: R2 = R1 + Rαr =

2π + πα + 2α Rr, π+2

a) Mennyi h˝ o fejl˝ odik a f´emr´ udban t0 id˝ o alatt? b) A bekapcsol´ ast´ ol (t = 0 id˝ opillanat) sz´ am´ıtott t0 id˝ opillanatban a m´ agneses indukci´ o v´ altoz´ asa megsz˝ unik. Ebben a pillanatban az eddig r¨ ogz´ıtett f´emrudat a v´ızuk. Mekkora szintes s´ıkban, a f´emr´ udra mer˝ olegesen v0 sebess´eggel mozgatni kezdj¨ legyen ez a sebess´eg, hogy a r´ udban foly´ o´ aram er˝ oss´ege ne v´ altozzon?

v´eg¨ ul pedig (algebrai a´talak´ıt´ asok ut´an) RAB =

(π − α)Rr · R2 (π − α)(2π + 2α + πα) Rr. = (π − α)Rr + R2 π(π + 4)

K¨ oz´ episkolai Matematikai ´ es Fizikai Lapok, 2022/8

503

504

K¨ oz´ episkolai Matematikai ´ es Fizikai Lapok, 2022/8

c) H´ anyszor t¨ obb h˝ o fejl˝ odik a f´emr´ udban a mozgat´ as sor´ an, mint a r¨ ogz´ıtett u ideig mozgatjuk? helyzetben, ha a f´emrudat 2t0 hossz´ (5 pont)

A r´ ud vezet´ekek k¨ oz¨ otti r´esz´enek hossza a mozg´ as v´eg´en

(3t0 ) = 2t0 v0 = 2L.

K¨ozli: Kotek L´ aszl´ o, P´ecs

Megold´as. Az ellen´ all´as n´elk¨ uli huzal ´es a f´emr´ ud z´ art a´ramk¨ort alkot, amelyben a v´ altoz´ o m´ agneses fluxus miatt fesz¨ ults´eg induk´al´odik, a´ram folyik ´es h˝ o fejl˝odik. a) A 0  t  t0 id˝ ointervallumban az induk´alt fesz¨ ults´eget a nyugalmi indukci´ o osszef¨ ugg´ese alapj´ an sz´am´ıthatjuk. Mivel a m´ agneses indukci´ o nagys´ aga Δt = t0 ¨ id˝ o alatt (egyenletesen v´ altozva) null´ ar´ ol B0 -ra n˝ o, a hurok ter¨ ulete pedig L2 /2, Faraday t¨ orv´enye szerint az induk´ alt fesz¨ ults´eg U=

ΔB L2 B 0 L2 ΔΦ = · = . Δt Δt 2 2t0

Ekkora fesz¨ ults´eg az R = Lr ellen´ all´ as´ u´ aramk¨ orben Ia =

c) Amikor a r´ ud m´eg nem mozog, a fesz¨ ults´eg is ´es az a´ramer˝ oss´eg is a´llanud mozg´ asa sor´ an d´o, teh´at a h˝ ofejl˝od´es teljes´ıtm´enye is id˝oben a´lland´ o P0 . A r´ az ´aramer˝ oss´eg id˝ oben a´lland´ o, de a r´ udnak az a´ramvezet´esben r´eszt vev˝ o hossza L-r˝ ol 2L-re n˝o, ´es emiatt az ellen´ all´ asa is a kezdeti ´ert´ek k´etszerese lesz. Ennek megfelel˝ oen a teljes´ıtm´eny is id˝ oben (egyenletesen) v´altozik, ´es a mozg´ as v´eg´en 2P0 lesz. ´ azoljuk a h˝ Abr´ ofejl˝od´es teljes´ıtm´eny´et az id˝o f¨ uggv´eny´eben. A grafikon alatti ter¨ uletek a fejl˝od¨ ott h˝ o nagys´ ag´ at adj´ak meg. Az ´ abr´ ar´ ol leolvashat´ o, hogy

B0 L U = R 2rt0

Qa = P0 t 0 ,

illetve

Qb =

P0 + 2P0 · 2t0 = 3P0 t0 , 2

vagyis Qb = 3 Qa .

er˝ oss´eg˝ u´ aramot hoz l´etre, ´es ´ıgy t0 id˝ o alatt

Soml´ an Gell´ert (P´ecs, Le˝ owey Kl´ara Gimn., 12. ´evf.) dolgozata felhaszn´ al´as´ aval

B 2 L3 Qa = Ia2 Rt0 = 0 4rt0 h˝ o fejl˝ odik. b) A t0  t  3t0 id˝ ointervallumban a m´agneses indukci´ o nagys´ aga ´alland´ o B0 , viszont a f´emr´ udnak a vezet´ekek k¨ oz´e es˝ o r´esz´enek hossza egyre nagyobb lesz:

15 dolgozat ´erkezett. Helyes 7 megold´ as. Kicsit hi´ anyos (4 pont) 2, hi´ anyos (2–3 pont) 6 dolgozat.

(t) = L + v0 (t − t0 ).

Fizik´ab´ ol kit˝ uz¨ott feladatok

Az ´ aramk¨ orben induk´ al´odott fesz¨ ults´eg a mozg´ asi indukci´ o t¨orv´enye szerint U (t) = B0 v0 (t), teh´ at id˝oben v´altozik, de az a´ramk¨ or ellen´ all´ asa is ugyanilyen u ¨temben n¨ovekszik: R(t) = r (t). A kialakul´ o´ aramer˝ oss´eg: Ib =

B0 v0 (t) B0 v 0 U (t) = = = ´alland´ o. R(t) r (t) r

Ez az ´aramer˝ oss´eg akkor egyezik meg a kor´ abban kisz´ am´ıtott Ia ´aramer˝ oss´eggel, ha B0 v 0 B0 L = , r 2rt0

M. 417. K´esz´ıts¨ unk 50 gemkapocsb´ol ´all´ o l´ ancot. Tartsuk a l´ ancot f¨ ugg˝olegesen az egyik v´eg´en´el fogva u ´gy, hogy a m´asik v´ege ´eppen az asztalhoz ´erjen. Sokszor egym´ as ut´an ejts¨ uk le a l´ ancot, ´es m´erj¨ uk meg minden alkalommal az ¨ osszegabalyodott l´ anckupac legnagyobb m´eret´et, illetve a l´ anc k´et v´ege k¨ oz¨ otti t´ avols´ agot. Sz´ amoljuk ki a m´ert ´ert´ekek a´tlag´ at ´es sz´ or´ as´ at! Hasonl´ıtsuk ¨ ossze ezeket a kifesz´ıtett l´anc teljes hossz´aval! (6 pont)

G. 793. Egy ember test´en 1000 hPa nyom´ ason 15 tonna s´ uly´ anak megfelel˝o nyom´oer˝ o oszlik el. a) Mekkora a testfel¨ ulete?

vagyis a r´ ud sebess´ege v0 =

b) Mekkora ez a nyom´ oer˝ o a Magas-T´ atra legmagasabb pontj´ an?

L . 2t0

K¨ oz´ episkolai Matematikai ´ es Fizikai Lapok, 2022/8

K¨ ozli: Schramek Anik´ o, F´ot

(3 pont) 505

506

K¨ oz´ episkolai Matematikai ´ es Fizikai Lapok, 2022/8

G. 794. U alak´ u cs˝ o keresztmetszete 1,5 cm2 . A cs˝ obe higanyt t¨olt¨ unk u ´gy, hogy az mindk´et sz´ arban el´eg magasan a´lljon. A cs˝ o egyik sz´ ar´ aba a higanyra 0,1 dl vizet ¨ ont¨ unk. Melyik sz´ arban ´es mennyivel fog magasabban a´llni a folyad´ek felsz´ıne? (4 pont) oget z´ ar be. A k´et t¨ uk¨or metsz´esvonaG. 795. K´et s´ıkt¨ uk¨ or egym´assal 60◦ -os sz¨ ◦ l´ at´ ol 30 cm-re 30 -os bees´esi sz¨ ogben f´enysug´ar ´erkezik az egyik t¨ uk¨orre. Legal´ abb mennyi id˝ o telik el, am´ıg a visszaver˝ od˝ o f´eny az egyik t¨ uk¨orr˝ ol a m´asikig ´er? (3 pont) G. 796. Egy ´ozongener´ ator ´or´ ank´ent 5 g o´zont ´all´ıt el˝o kis¨ ul´essel, ´es ventil´ atorral juttatja azt a fert˝otlen´ıtend˝ o fel¨ uletre. a) H´ any ´ ozonmolekula keletkezik 1 o´ra alatt? ulet fert˝otlen´ıt´es´ere 30 percet javasol. A leb) A haszn´ alati u ´tmutat´ o 28 m2 fel¨ veg˝ o tiszta ´es pormentes, ´ıgy a keletkez˝ o´ ozon csak a fel¨ uleten bomlik fel. Becs¨ ulj¨ uk meg, h´any o´zonmolekula jut egy olyan bakt´eriumra, amely 10 n´egyzetmikron fel¨ uletet foglal el! (4 pont)

b) Mekkora a olajbogy´ o gyorsul´ asa az A pontban? c) Mekkora ´es milyen ir´any´ u ered˝ o er˝ ot fejt ki a z´ uz´ oker´ek az olajbogy´ ora a legfels˝ o A pontban? (5 pont)

P. 5439. Egy g¨ omb alak´ u, kezdetben 20 ◦ C h˝ om´ers´eklet˝ u r´ezgoly´ ot v´ekony, h˝ oszigetel˝ o sz´ al seg´ıtunk. s´eg´evel nagy mennyis´eg˝ u, 80 ◦ C-os v´ızbe mer´ıt¨ o eltelt´evel melegszik fel 50 ◦ C-ra. A f´emgoly´ o t1 id˝ Ezut´ an a k´ıs´erletet megism´etelj¨ uk u ´gy, hogy a v´ız o´e pedig 80 ◦ C. A r´ezh˝ om´ers´eklete 20 ◦ C, a goly´ o alatt h˝ ul le 50 ◦ C-ra. Melyik id˝ o goly´ o most t2 id˝ ot r¨ ovidebb, t1 vagy t2 , ha a goly´ a) ´eppen csak belemer´ıtj¨ uk a v´ızbe, b) majdnem az ed´eny alj´ aig engedj¨ uk le?

K¨ozli: Gelencs´er Jen˝ o, Kaposv´ ar

P. 5436. K´et, egym´ ast mer˝ olegesen keresztez˝ o egyenes aut´ op´aly´ an egy-egy pontszer˝ unek tekinthet˝ o aut´ o a keresztez˝ od´esi pont fel´e tart a´lland´ o nagys´ ag´ u sebesu aut´ o´e vB = 40 km/h. Egy s´eggel. Az A jel˝ u aut´ o sebess´ege vA = 50 km/h, a B jel˝ adott id˝ opontban a k´et aut´ o a keresztez˝ od´esi pontt´ol m´ert t´ avols´ aga dA = 20 km, illetve dB = 36 km. a) Mekkora lesz k¨ ozt¨ uk a minim´ alis t´ avols´ ag? b) Mennyi id˝ o m´ ulva lesznek egym´ashoz legk¨ ozelebb? (4 pont)

K¨ozli:Holics L´ aszl´ o, Budapest

P. 5437. Egy kett˝ oscsillag egyik tagja h´aromszor nagyobb t¨omeg˝ u, mint a m´ asik csillag. A k´et ´egitest (amelyek m´erete sokkal kisebb, mint a t´avols´ aguk) k¨ozel´ıt˝ oleg k¨ or alak´ u p´ aly´ akon keringenek a k¨ oz¨ os t¨ omegk¨oz´eppontjuk k¨or¨ ul. Melyik csillagnak ´es h´ anyszor nagyobb a mozg´ asi energi´ aja a t¨omegk¨oz´epponti koordin´ atarendszerben? (3 pont)

Tank¨onyvi feladat

P. 5438. Egy spanyol gazdas´ agban a k´epen l´athat´ o olajbogy´ opr´essel t¨orik p´epul˝ o s´ıkja, amit az ´ abr´ an p´e a bogy´ okat. A 90 cm ´atm´er˝ oj˝ u z´ uz´ oker´ek tiszt´an g¨ord¨ szaggatott vonal jelez, a tengelyt˝ ol 75 cm t´ avols´ agban van. A csacsi farka a tengelyt˝ ol 180 cm t´ avols´ agban verdesi a rudat, mik¨ ozben az a´llat 2,4 m/s sebess´eggel k¨ orbe-k¨ orbe fut. A z´ uz´ oker´ekre egy 1 g t¨ omeg˝ u olajbogy´ o ragad. a) Mekkora az olajbogy´ o sebess´ege, amikor a fels˝ o A pontba ´er? K¨ oz´ episkolai Matematikai ´ es Fizikai Lapok, 2022/8

507

K¨ ozli: Baranyai Kl´ ara, Veresegyh´az

(4 pont)

P´eldat´ ari feladat nyom´an

P. 5440. Egy m´ehkapt´ art´ ol 2 km t´ avols´ agra van egy ak´ acos, ahonn´et egyegy m´eh fordul´onk´ent 30 mm3 t´erfogat´ u nekt´ art sz´ all´ıt be a kapt´ arba. A m´ez k´esz´ıt´esekor a m´ehek a nekt´ ar t¨ omeg´enek 55%-´ at kitev˝o v´ız egy r´esz´et a kapt´ arban elp´ arologtatj´ ak, a k´esz m´ezben a v´ız t¨ omege m´ ar csak 19%. A vir´agz´ as 12 napja alatt a m´ehcsal´ ad 25 kg m´ezet k´esz´ıt. A p´ arologtat´as energiaig´eny´et a hazahozott nekt´ ar egy r´esz´enek elfogyaszt´ as´ aval fedezik a m´ehek. a) H´any watt a m´ehcsal´ adnak csup´ an a p´ arologtat´asba fektetett a´tlagos teljes´ıtm´enye? b) H´any kilom´etert tesznek meg a csal´ ad gy˝ ujt˝ otagjai ¨ osszesen, am´ıg a sz¨ uks´eges nekt´ armennyis´eget a kapt´ arba hordj´ak? kg

ar 1 kg-ja 6000 kJ energi´at szolg´altat; 1 kg A nekt´ar s˝ ur˝ us´ege 1,2 dm3 ; a nekt´ v´ız elp´arologtat´as´ ahoz 2400 kJ energi´at haszn´alnak fel a m´ehek. ´ (4 pont) Eretts´ egi-felv´eteli feladat 508

K¨ oz´ episkolai Matematikai ´ es Fizikai Lapok, 2022/8

P. 5441. Egy f´emdr´ otb´ ol k¨ ort form´aztunk, ´es ugyanabb´ ol a dr´ otb´ ol az egyik h´ urt is szeretn´enk elk´esz´ıteni a k¨ or k´et pontja k¨ oz´e. Hol fusson a h´ ur, hogy a lehet˝o legnagyobb legyen az ered˝ o ellen´ all´ as a h´ ur k´et v´egpontja k¨oz¨ott, ´es mekkora lesz az ered˝o ellen´ all´ as ebben az esetben? Jel¨ olje R a sug´ arhossz´ us´ag´ u dr´ ot ellen´ all´as´ at. (4 pont)

K¨ ozli: G´ asp´ ar Merse El˝ od, Budapest

P. 5442. Egy eredetileg nyugv´ o atommag 20 kV potenci´alk¨ ul¨onbs´eg befut´ asa ut´ an a halad´ asi ir´ any´ ara mer˝ oleges, 1,0 T indukci´ oj´ u homog´en m´ agneses mez˝ obe ker¨ ul. A m´agneses mez˝ ot egy, a r´eszecske halad´ asi ir´ any´ ara mer˝ oleges s´ık v´alasztja ulva l´ep ki a m´ agneses el az er˝ot´ermentes tartom´ anyt´ol. A r´eszecske 3,3 · 10−8 s m´ mez˝ ob˝ ol. Melyik atommagr´ol van sz´ o? (4 pont)

K¨ ozli: Tornyos Tivadar E¨ ors, Budapest

P. 5443. A KCl lapcentr´alt kockarendszerben krist´ alyosodik, ´es a r´ acs´ alland´ oja 628 pm. Legfeljebb mekkora lehet a r¨ ontgenf´eny hull´amhossza, hogy l´etrej¨ohessen Bragg-reflexi´o az elemi cella test´atl´ oira mer˝ oleges r´ acss´ıkokon? (L´asd A r¨ ontgensz´ or´ as, m´ as n´even Bragg-reflexi´ o c. cikket lapunk 489. oldal´ an.) (4 pont)

K¨ ozli: Woynarovich Ferenc, Budapest

P. 5444. Egy v´ekony, hossz´ u, f¨ ugg˝oleges, szigetel˝ or´ udon s´ url´od´asmentesen mozoghat egy kicsiny t¨ olt¨ott goly´ ocska. Ha egy ezzel azonos t¨olt´es˝ u, ugyancsak kicsiny agban lesz egyens´ ulyban. testet helyez¨ unk a r´ ud t¨ ov´ebe, a mozg´ o goly´ o h0 magass´ Milyen messzire t´avol´ıthatjuk el a r´ udt´ ol v´ızszintes ir´anyba az als´ o testet u ´gy, hogy a r´ udon l´ev˝ o goly´ o m´eg egyens´ ulyban lehessen valahol? Milyen magasan van ez a hely? (6 pont)

Varga Istv´ an (1952–2007) feladata nyom´ an Beku ¨ld´esi hat´arid˝o: 2022. december 15. Elektronikus munkafu ¨zet: https://www.komal.hu/munkafuzet



MATHEMATICAL AND PHYSICAL JOURNAL FOR SECONDARY SCHOOLS (Volume 72. No. 8. November 2022)

Problems in Mathematics New exercises for practice – competition K (see page 480): K. 739. Phil made the following observations throughout a certain period in autumn: 1. During that period, there were 11 days when it rained. 2. A rainy morning was always followed by a sunny afternoon. 3. Altogether, there were 9 sunny mornings and 12 sunny afternoons. How many days were there when it did not rain at all? K. 740. In how many different ways is it possible to tile a 3 × 12 rectangle with twelve 1 × 3 rectangles? K. 741. Starting with the K¨ oz´ episkolai Matematikai ´ es Fizikai Lapok, 2022/8

509

numbers 1, 2, 3, 4, 5, 6, 7, 8, 9, in each step two numbers are chosen and increased by 1. Is it possible to achieve that each number is 10? K/C. 742. Danny is learning the alphabet. He has successfully named the first eight letters (A, B, C, D, E, F, G, H), but the order of the letters was not entirely correct. Only five of the eight letters were listed in the right position (i.e. in the position where it occurs in the alphabet). How many such orders of the eight letters are there? K/C. 743. The midpoint of side BC of a rectangle ABCD is E, and F is the point lying closer to D which divides side CD in a 2 : 1 ratio. The midpoint of line segment AE is G, and H is the point lying closer to E which divides line segment EF in a 2 : 1 ratio. What fraction is the area of triangle F GH of the area of rectangle ABCD? New exercises for practice – competition C (see page 480): Exercises up to grade 10: K/C. 742. See the text at Exercises K. K/C. 743. See the text at Exercises K. Exercises for everyone: C. 1738. A natural number is called balanced, if the number of digits in its representation in decimal notation equals the number of different prime factors it has. For example, 21 is balanced, but 42 is not. Is it true that there are infinitely many balanced numbers? (Proposed by K. A. Kozma, Gy˝ or) C. 1739. Define the following functions on √ −2x+8 the largest possible subset of the set of real numbers: f (x) = x + 5 , g(x) = 5 and h(x) = [x + 3] (here [a] denotes the integer part of the real number a, that is, the greatest integer which is not greater than a). Find the common points of the graphs of the three functions. (Proposed by B. B´ır´ o, Eger) C. 1740. P is an interior point of side CD of a parallelogram ABCD, and Q is an interior point of side AB parallel to CD. The intersection of line segments P A and QD is M , and the intersection of line segments P B and QC is N . Assume that M N ∦ AB, and M N intersects the line of CD at point X, and the line of AB at point Y . Prove that DX = BY . (American competition problem) Exercises upwards of grade 11: C. 1741. The diagonals AC and BD of a convex quadrilateral ABCD intersect at M . Is it possible that the areas of triangles ABM , BCM , CDM and DAM , in this order, are four consecutive terms of a) an arithmetic sequence; b) a geometric sequence? (Proposed by B. B´ır´ o, Eger) C. 1742. Consider the following 1 functions (defined on the largest possible subset of the set of real numbers): f0 (x) = 1−x ,   and fn (x) = f0 fn−1 (x) , for all positive integers n. Calculate the value of f2022 (2022). (Canadian problem) New exercises – competition B (see page 482): B. 5270. n2 regular triangles of unit side are used to make a large regular triangle of side n units. The small triangles are coloured alternately dark and light. The numbers 1, 2, 3, . . . , n2 are written in the triangles, as shown in the figure. What is the sum of the numbers in the dark triangles? (3 points) (Proposed by L. N´emeth, Fony´ od) B. 5271. ABC is an isosceles right angled triangle with the right angle lying at vertex C. A , B  and C  are interior points of sides AB, BC and CA, respectively, such that triangle A B  C  is similar to ABC. Show that the midpoint of the side AB, the midpoint of line segment A B  , and point C are collinear. (3 points) (Proposed by E. Hajdu, Sopron and M. Hujter, Budapest) B. 5272. A flea starts out from point (a, b) of the coordinate plane, where a, b are positive integers. With each jump, the flea will move one unit to the left or downwards. It keeps jumping until it reaches either the x axis or the y axis. What fraction of the possible sequences of jumps terminate on the x axis? (4 points) (Based on the idea of D. Melj´ an, Kecskem´et) B. 5273. D is a point on side AB of an equilateral triangle ABC, and E is a point on side BC such that oka, ∠BCD = 45◦ and ∠CDE = 30◦ . Show that BE = 2AD. (4 points) (Proposed by S. R´ Ny´ıregyh´ aza) B. 5274. The product of the positive integers a < b is a perfect square. Show oka, that there is a positive integer x such that a  x2  b. (5 points) (Proposed by S. R´

510

K¨ oz´ episkolai Matematikai ´ es Fizikai Lapok, 2022/8



Ny´ıregyh´ aza) B. 5275. Is there an irrational number a such that a 3 is rational? (5 points) (Proposed by B. Hujter, Budapest) B. 5276. Prove that there exist infinitely many positive integers k for which the sum of the digits of 2k is a) smaller; b) greater than the sum of andor, Budapest) B. 5277. The centre of the digits of 2k+1 . (6 points) (Proposed by Cs. S´ the inscribed circle of triangle ABC is I. The midpoint of the circular arc BCA is F , and line F I intersects the circumscribed circle again at point M . Show that line CM passes through the external centre of similitude of the inscribed circle and the circumscribed circle. (6 points) (Proposed by G. K´ os, Budapest) New problems – competition A (see page 483): A. 836. For every i ∈ N let Ai , Bi and Ci be three finite and pairwise disjoint subsets of N. Suppose that for every partition of N consisting of sets A, B and C there exists i ∈ N such that Ai ⊂ A, Bi ⊂ B and Ci ⊂ C. Prove that there also exists a finite S ⊂ N such that for every partition of N consisting of sets A, B and C there exists i ∈ S such that Ai ⊂ A, Bi ⊂ B and Ci ⊂ C. (Submitted by Andr´ as Imolay, Budapest) A. 837. Let all the edges of tetrahedron A1 A2 A3 A4 be tangent to sphere S. Let ai denote the length of the tangent from Ai to S. Prove that   4 4 1 2 1 > 2 . (Submitted by Viktor V´ıgh, Szeged) A. 838. Sets X ⊂ Z+ and 2 a a i=1

i

i=1

i

Y ⊂ Z+ are called comradely, if every positive integer n can be written as n = xy for some x ∈ X and y ∈ Y . Let X(n) and Y (n) denote the number of elements of X and Y , respectively, among the first n positive integers. Let f : Z+ → R+ be an arbitrary function X(n)

that goes to infinity. Prove that one can find comradely sets X and Y such that n min {X(n),Y (n)} Y (n) and n goes to 0, and for all ε > 0 exists n ∈ Z+ such that < ε. f (n)

Problems in Physics (see page 506) M. 417. Make a chain from 50 paper clips. Hold the chain vertically at one of its ends such that the other end just touches the table. Drop the chain several times in succession, and measure each time the maximum size of the tangled chain and the distance between the two ends of the chain. Calculate the mean and standard deviation of the measured values. Compare these with the total length of the chain when stretched. G. 793. At a pressure of 1000 hPa a force equivalent to the weight of a 15 ton object is distributed over the body of a human being. a) what is the surface area of the body of the person? b) what is this weight at the highest point of the High Tatra Mountains? G. 794. The cross-section of a U-shaped tube is 1.5 cm2 . The tube is filled with mercury such that there is high enough mercury in both arms of the tube. 0.1 dl of water is poured on the mercury in one arm of the tube. In which arm and by what distance will the surface of the liquid be higher? G. 795. The angle between two plane mirrors is 60◦ . At a distance of 30 cm from the intersection of the two mirrors, a ray of light is incident on one of the mirrors, its angle of incidence is 30◦ . What is the minimum time that it takes for the reflected light to travel from one mirror to the other? G. 796. An ozone generator produces 5 g of ozone per hour by corona discharge and delivers it by a fan to the surface, which is to be disinfected. a) How many ozone molecules are produced in one hour? b) The manual recommends 30 minutes to disinfect a surface of area 28 m2 . The air is clean and dustfree, so the generated ozone will only decompose on the surface. Estimate the number of ozone molecules per bacterium on a surface area of 10 square microns. P. 5436. Two cars, which can be considered to be point-like, are travelling at constant speed, each on a straight motorway towards the intersection of the motorways. (The angle

K¨ oz´ episkolai Matematikai ´ es Fizikai Lapok, 2022/8

511

between the two motorways is 90◦ .) The speed of car A is vA = 50 km/h, the speed of car B is vB = 40 km/h. At a given moment, the distance of the two cars from the intersection is dA = 20 km, and dB = 36 km. a) what will the minimum distance between the two cars be? b) How long will it be until they are closest? P. 5437. The mass of one of the starts of a binary star system is three times as much as that of the other. The two celestial bodies (whose size is much smaller than their distance) orbit in approximately circular orbits around their centre of mass. Which star has greater kinetic energy, with respect to the coordinate system fixed to the centre of mass of the system, and by what factor is it greater than that of the other star? P. 5438. On a Spanish farm, the olives are crushed to a pulp with the olive press shown in the figure. The plane of the crushing wheel, of diameter 90 cm, shown by the dashed line in the figure, is at a distance of 75 cm from the shaft. The wheel rolls without sliding. The tail of the donkey is 180 cm from the shaft, and the donkey undergoes circular motion at a speed of 2.4 m/s. An olive weighing 1 g got stuck to the crushing wheel. a) What is the speed of the olive when it is at the topmost point A of the wheel? b) what is the acceleration of the olive at this point? c) What is the magnitude and the direction of the force exerted by the wheel on the olive when it is at point A? P. 5439. A spherical copper ball at 20 ◦ C hanging on a thin insulator string is immersed in a large amount of water at 80 ◦ C. After a time of t1 the copper ball warms up to 50 ◦ C. Later the experiment is repeated in such a way that the initial temperature of water is 20 ◦ C, while the ball is at 80 ◦ C. In this way the copper ball cools down to 50 ◦ C during a time of t2 . What is shorter, t1 or t2 , if the ball a) is just immersed into the water, or b) is submerged almost to the bottom of the container? P. 5440. At a distance of 2 km from a beehive there is a black locust forest, from where a single bee can bring nectar of volume 30 mm3 to the hive in each turn. 55% of the mass of the collected nectar is water, and when making honey, the worker bees in the hive make some portion of the water evaporate, such that the finished honey contains only 19% water. During the 12 days of flowering, the colony of the hive produces 25 kg of honey. The bees cover the energy requirements of evaporation by eating some part of the nectar they brought to the hive. a) How many watts is the average power output of the colony in the hive invested in the evaporation of the water? b) Altogether how many kilometres are covered by the worker bees while the total amount of nectar is carried to the hive? The density of nectar kg

; 1 kg nectar gives 6000 kJ energy; in order to evaporate 1 kg water the bees is 1.2 dm3 need 2400 kJ energy. P. 5441. We have formed a circle from a piece of metal wire, and from the same wire we would like to to make one of the chords between the two points of the circle. Where should the chord be in order that the equivalent resistance between the two end points of the chord is maximum, and what is the value of this largest equivalent resistance? Let R be the resistance of a wire which has a length equal to the radius of the circle. P. 5442. A nucleus, which was initially at rest, was accelerated though a potential difference of 20 kV, and then it enters into a uniform magnetic field of induction 1.0 T. The magnetic induction is perpendicular to the velocity of the nucleus. The magnetic field is separated from the force-free region by a plane perpendicular to the particle’s direction of travel. The particle leaves the magnetic field after 3.3 · 10−8 s. Which nucleus is it? P. 5443. KCl crystallises in a face-centred cubic system and has a lattice constant of 628 pm. What is the maximum wavelength of X-ray that can be used to create a Bragg reflection on the lattice planes perpendicular to the diagonal of the unit cell in the crystal? P. 5444. A small charged ball can move frictionlessly along a long, thin, vertical, insulating rod. If an equally small body of the same charge is placed at the bottom of the rod, the moving ball will be in equilibrium at a height of h0 . How far away from the rod in the horizontal direction can we move the lower body so that the ball on the rod can still be in equilibrium somewhere. What is the height of this is this position?

72. ´evfolyam 8. sz´am

K¨oMaL

Budapest, 2022. november

´ ´ FIZIKAI LAPOK ¨ EPISKOLAI KOZ MATEMATIKAI ES ˝ ´ITVE INFORMATIKA ROVATTAL BOV ´ ALAP´ITOTTA: ARANY DANIEL 1894-ben 72. ´evfolyam 9. sz´am

Nyakl´ancok, lyukas n´egyzetek ´es oszthat´ os´ag Budapest, 2022. december

´ Megjelenik ´evente 9 sz´amban, janu´art´ol m´ajusig ´es szeptembert˝ol decemberig havonta 64 oldalon. ARA: 1100 Ft

´ TARTALOMJEGYZEK Freud R´obert, Horv´ath Eszter: Nyakl´ancok, lyukas n´egyzetek ´es oszthat´os´ag . . . . . . . . . . . . . . . . . . . . . .

514

Kozma Katalin Abig´el, Sz´amad´o L´aszl´o: Gyakorl´o feladatsor emelt szint˝u matematika ´eretts´egire

520

J´ocsik Csilla: Megold´asv´azlatok a 2022/8. sz´am emelt szint˝u matematika gyakorl´o feladatsor´ahoz . . . . . . . . . . . . . . . . . . . . . . . . . . . . . . . . . . . . . . .

522

Matematika C gyakorlatok megold´asa (1729., 1731.) . . . . . . . . . . . . . . . . . . . . . . . . . . . . . . . . . . . . . . . .

531

Matematika feladatok megold´asa (5241., 5246.) . . .

538

Matematikai k´epz´esek az ELTE TTK-n . . . . . . . . . .

540

Matematikatan´ar-k´epz´es az ELTE TTK-n . . . . . . . .

542

A K pontversenyben kit˝uz¨ott gyakorlatok (744– 748.) . . . . . . . . . . . . . . . . . . . . . . . . . . . . . . . . . . . . . . . . .

542

A C pontversenyben kit˝uz¨ott gyakorlatok (747– 748., 1743–1747.) . . . . . . . . . . . . . . . . . . . . . . . . . . . . .

544

A B pontversenyben kit˝uz¨ott feladatok (5278– 5285.) . . . . . . . . . . . . . . . . . . . . . . . . . . . . . . . . . . . . . . . .

545

Az A pontversenyben kit˝uz¨ott nehezebb feladatok (839–841.) . . . . . . . . . . . . . . . . . . . . . . . . . . . . . . . . . . . .

546

N´eh´anyan a 2021–2022-es tan´ev legszorgalmasabb megold´oi k¨oz¨ul. . . . . . . . . . . . . . . . . . . . . . . . . . . . . . . .

547

Informatik´ab´ol kit˝uz¨ott feladatok (577–579., 67., 166.) . . . . . . . . . . . . . . . . . . . . . . . . . . . . . . . . . . . . . . . . .

551

Fizika alapszak ´es fizikatan´ar-k´epz´es az ELTE TTK Fizikai Int´ezet´eben . . . . . . . . . . . . . . . . . . . . . .

557

Gn¨adig P´eter: T´erb˝ol s´ıkba visszal´ep´es fizikai probl´em´ak megold´as´an´al I. . . . . . . . . . . . . . . . . . . . .

559

Fizika gyakorlat megold´asa (783.) . . . . . . . . . . . . . . . .

565

Fizika feladatok megold´asa (5399., 5418., 5419.) . .

565

Fizik´ab´ol kit˝uz¨ott feladatok (418., 797–800., 5445–5453.) . . . . . . . . . . . . . . . . . . . . . . . . . . . . . . . . . .

570

Problems in Mathematics . . . . . . . . . . . . . . . . . . . . . . . .

573

Problems in Physics . . . . . . . . . . . . . . . . . . . . . . . . . . . . .

575

A K¨oz´episkolai Matematikai ´es Fizikai Lapok 72. ´evfolyam´anak tartalomjegyz´eke . . . . . . . . . . . .

XXI

K¨ oz´ episkolai Matematikai ´ es Fizikai Lapok, 2022/9

´ EVA ´ F˝oszerkeszt˝o: RATKO ´ ¨ Fizikus szerkeszt˝o: GNADIG PETER ´ ILDIKO ´ M˝uszaki szerkeszt˝o: MIKLOS Bor´ıt´o: BURGHARDT ZSUZSA ´ Kiadja: MATFUND ALAP´ITVANY ´ RITA Alap´ıtv´anyi k´epvisel˝o: KOS Felel˝os kiad´o: KATONA GYULA Nyomda: OOK-PRESS Kft. ´ Felel˝os vezet˝o: SZATHMARY ATTILA INDEX: 25 450 ISSN 1215-9247 A matematika bizotts´ag vezet˝oje: ´ HERMANN PETER ´ BALINT, ´ ´ Tagjai: B´IRO GYENES ZOLTAN, ´ ´ KISS HUJTER BALINT, IMOLAY ANDRAS, ´ ´ GEZA, ´ ´ GEZA, KOS KOZMA KATALIN ABIGEL, ´ ´ ´ PACH ¨ ORDI ¨ MATOLCSI DAVID, OK PETERN E, ´ ´ SZMERKA GERGELY, V´IGH PETER PAL, VIKTOR A fizika bizotts´ag tiszteletbeli eln¨oke: ´ ´ HOLICS LASZL O ´ Tagjai: BARANYAI KLARA, HONYEK GYULA, ´ ´ KRISZTIAN, ´ OLOSZ BALAZS, SZASZ ´ ´ ´ E, ´ VLADAR ´ SZECHENYI GABOR, VIGH MAT ´ KAROLY, WOYNAROVICH FERENC Az informatika bizotts´ag vezet˝oje: ´ ´ SCHMIEDER LASZL O ´ E, ´ FARKAS CSABA, FODOR Tagjai: BUSA MAT ´ ´ ZSOLT, LOCZI LAJOS, SIEGLER GABOR, ´ ´ ´ SZENTE PETER, TOTH TAMAS ´ ANDREA, TASNADI ´ ANIKO ´ Ford´ıt´ok: GROF ´ ´ ¨ Szerkeszt˝os´egi titk´ar: TRASY GYORGYN E A szerkeszt˝os´eg c´ıme: 1117 Budapest, P´azm´any P´eter s´et´any 1/C III. emelet 3.405. Telefon: 372-2850 A lap megrendelhet˝o az Interneten: www.komal.hu/megrendelolap/reszletek.h.shtml. El˝ofizet´esi d´ıj egy ´evre: 9200 Ft K´eziratokat nem ˝orz¨unk meg ´es nem k¨uld¨unk vissza. Minden jog a K¨oMaL tulajdonosai´e. E-mail: [email protected] Internet: http://www.komal.hu This journal can be ordered from the Editorial office: P´azm´any P´eter s´et´any 1/C III. emelet 3.405. 1117–Budapest, Hungary telephone: +36 (1) 372-2850 or on the Postal address H–1518 Budapest 112, P.O.B. 32, Hungary, or on the Internet: www.komal.hu/megrendelolap/reszletek.e.shtml. A Lapban megjelen˝o hirdet´esek tartalm´a´ert felel˝oss´eget nem v´allalunk.

513

Ebben az ´ır´ asban egy olyan m´ odszert mutatunk be, amely sz´amos lesz´aml´ al´asi feladatn´ al j´ol haszn´alhat´ o. I. Nyakl´ancok 1. feladat. Van 30 egyforma gy¨ ongy¨ unk, 2 piros, a t¨ obbi s´ arga. H´ anyf´ele nyakl´ anc k´esz´ıthet˝ o bel˝ ol¨ uk? Megold´as. A nyakl´ ancot b´ arhogyan forgatva ugyanaz marad. ´Igy csak az sz´ am´ıt, hogy az egyik piroshoz milyen k¨ ozel van a m´ asik. A kett˝ o k¨ oz¨ otti s´ arg´ak sz´ama lehet 0, 1, . . . , 14, teh´at 15-f´ele nyakl´ anc k´esz´ıthet˝ o. 2. feladat. Van 30 egyforma gy¨ ongy¨ unk, 6 piros, a t¨ obbi s´ arga. H´ anyf´ele nyakl´ anc k´esz´ıthet˝ o bel˝ ol¨ uk? Ismerked´es a neh´ezs´egekkel. Erre az esetre rem´enytelen az el˝oz˝ o m´odszert adapt´ alni (l´ atni fogjuk, hogy a lehets´eges nyakl´ ancok sz´ ama 10 133). Ha a szimmetri´akt´ ol eltekinten´enk, akkor 24 s´arga ´es 6 piros gy¨ ongy¨ ot kellene sorba raknunk, teh´ a t a 30 helyb˝ o l a 6 pirosnak a hely´ e t kellene kiv´ a lasztanunk, ami 30 lehet˝ os´eg. Azonban azok a piros hatosok, amelyek egym´ asba a szimmetri´ ak 6 alapj´an a´tvihet˝ ok, ugyanazt a nyakl´ ancot jelentik. Mik ezek a szimmetri´ ak? A nyakl´ancot kifesz´ıtve a gy¨ ongy¨ ok egy szab´ alyos 30-sz¨ og cs´ ucsainak tekinthet˝ ok, ´es ha egy nyakl´ancot a soksz¨ og k¨ oz´eppontja k¨ or¨ ul 12k fokkal elforgatunk (ahol 1  k  30), vagy a soksz¨og valamelyik szimmetriatengely´ere t¨ ukr¨ oz¨ unk, akkor ugyanaz a nyakl´ anc marad. Teh´ at egy piros hatosra ezen 60 darab egybev´ag´ os´ agot alkalmazva mindig ugyanazt a nyakl´ ancot kapjuk. Els˝ o k¨ ozel´ıt´esben azt gondolhatn´ ank, hogy akkor a piros hatosok 60-as´aval jelentik ugyanazt a nyakl´ ancot, teh´ at a piros hatosok sz´ am´at a transzfor-

30

1. a ´bra

514

m´ aci´ ok sz´am´aval osztva 6 /60 adja a nyakl´ ancok sz´ am´at. Ez viszont nem is eg´esz sz´ am. A probl´em´ at az okozza, hogy vannak olyan piros hatosok, amelyekre a 60 transzform´ aci´ ot alkalmazva nem kapunk 60 k¨ ul¨ onb¨ oz˝ o piros hatost. N´ezz¨ unk p´eld´ aul egy olyan esetet, amikor minden ¨ ot¨ odik gy¨ ongy piros (1. ´ abra). Azonnal l´ atszik, hogy erre a 60 transz-

K¨ oz´ episkolai Matematikai ´ es Fizikai Lapok, 2022/9

form´aci´ ot alkalmazva ¨osszesen 5 k¨ ul¨ onb¨ oz˝ o piros hatost kapunk. Vizsg´aljuk meg ezt alaposabban, hogy jobban l´ assuk az ¨ osszef¨ ugg´eseket. A piros gy¨ongy¨ok ekkor egy szab´ alyos hatsz¨ og cs´ ucsait alkotj´ ak. ´Igy a nyakl´ ancot 60j fokkal elforgatva (ahol 1  j  6), vagy a hatsz¨ og b´ armelyik szimmetriatengely´ere t¨ ukr¨ozve az adott piros hatos ¨ onmag´aba megy a´t. Ezt r¨ oviden u ´gy fogjuk h´ıvni, hogy ez a piros hatos ennek a 12 transzform´ aci´ onak fixalakzata. Jel¨ olje ezt a 12 transzform´aci´ ot τ1 , . . . , τ12 , ´es legyen  (mondjuk) a 12 fokkal t¨ ort´en˝ o elforgat´ as (2. ´ abra), ami teh´ at egy m´asik piros hatost eredm´enyez (de ugyanazt a nyakl´ ancot). Mely m´asik transzform´aci´ ok viszik ´at ugyanide az eredeti piros hatost? Az ¨ osszes ana a ρ) megfelel, hiszen ρτi (el˝obb a τi , ut´ akkor el˝ osz¨ or ¨ onmag´aba megy az eredeti piros ´ ez az ¨ hatos, majd elfordul 12 fokkal. Es osszes! Ugyanis ha π is ugyanide viszi az eredeti piros hatost, akkor ezut´an a −12 fokos forgat´ast, azaz a  inverz´et, −1 -et alkalmazva visszaker¨ ul¨ unk az eredeti piros hatosba. Ez azt jelenti, hogy −1 π-nek az eredeti piros hatos fixalakol zata, vagyis −1 π = τi , azaz π = τi . Ebb˝ 2. a ´bra k¨ ovetkezik, hogy minden poz´ıci´ oba pontosan ugyanannyi transzform´ aci´ o viszi az eredeti piros hatost. Ez´ert a l´etrej¨ov˝o poz´ıci´ ok sz´ am´at u ´gy kapjuk meg, hogy az ¨ osszes transzform´ aci´ o sz´ am´at elosztjuk azoknak a transzform´ aci´ oknak a sz´ am´aval, amelyeknek az adott piros hatos fixalakzata: 60/12 = 5. Ez az ´eszrev´etel a´ltal´ anosan is ´erv´enyes. Ez azt mutatja, hogy a fixalakzatok sz´ ama fontos szerepet j´atszik a lesz´aml´ al´asban. aly´ aja az ¨osszes olyan piros hatos, P´aly´ak. Legyen az x piros hatos P = Px p´ ahov´ a a transzform´ aci´ ok x-et a´tviszik. Egy szab´ alytalan piros hatos p´aly´ aja tipikusan 60 elem˝ u. A minden ¨ ot¨ odik gy¨ ongy piros p´aly´ aja 5 elem˝ u. Az egy p´ aly´ aba tartoz´ o piros hatosok jelentik ugyanazt a nyakl´ancot. ´Igy a feladat a p´aly´ ak sz´am´ anak a meghat´ aroz´ asa.

kapjuk, hogy a transzform´aci´ ok fixalakzatainak atlagos sz´ama ´eppen a piros hato´ 1/|Px |. Mivel egy P p´ aly´ aban lev˝o sokhoz tartoz´ o p´ alyam´eretek reciprok¨ osszege x

mind a |P | darab y piros hatosra Py = P , ez´ert egy adott P p´ alya hatosait tekintve a reciprok¨osszeg |P | · 1/|P | = 1. Vagyis a teljes reciprok¨ osszeg ´eppen a p´ aly´ ak keresett sz´ ama! Ezzel bel´attuk: Burnside-lemma. A p´ aly´ ak sz´ ama egyenl˝ o a transzform´ aci´ ok fixalakzatainak atlagos sz´ ´ am´ aval. A 2. feladat megold´asa el˝ ott bemeleg´ıt´esk´ent oldjuk meg az 1. feladatot, teh´at a 2 piros gy¨ongy eset´et a Burnside-lemm´aval. A szab´ alyos 30-sz¨ og szimmetri´ 30 ai k¨oz¨ul a helyben hagy´as minden piros p´art asok k¨ oz¨ ul egyed¨ ul ¨onmag´aba visz, teh´at annak 2 fixalakzata van. A forgat´ a 180 fokos forgat´asnak van fixalakzata, m´egpedig az a 15 piros p´ar, amikor a 2 piros gy¨ongy k¨oz¨ott mindk´et oldalon 14–14 s´ arga helyezkedik el. Ha a t¨ ukr¨ oz´es tengelye k´et ´atellenes oldalfelez˝o pontot ¨ osszek¨ ot˝ o egyenes, akkor az ennek egyik oldal´an lev˝o 15 gy¨ongyb˝ ol b´ armelyik lehet piros, a m´asik piros pedig ennek a tengelyre vonatkoz´o t¨ uk¨ork´epe, teh´ at itt is 15 fixalakzat van. Ha a t¨ ukr¨ oz´es tengelye k´et szemk¨ ozti cs´ ucsot ¨osszek¨ ot˝ o´ atl´ o, akkor vagy az ´atl´ o k´et v´egpontja piros, vagy pedig az egyik oldalon lev˝o 14 pont b´ armelyike ´es annak t¨ uk¨ ork´epe lesz piros, 30 teh´at itt is 15 a fixalakzatok sz´ama. Vagyis a fixalakzatok sz´ am´anak ´atlaga ( 2 + 31 · 15) 60 = = 15. N´ezz¨ ukakkor  ugyanezt a m´odszert a 6 piros gy¨ongy eset´ere. A helyben hagy´as30 nak mind a 6 piros hatos fixalakzata. A 60, illetve −60 fokos forgat´as fixalakzatai a szab´ alyos hatsz¨ ogek (1. a´bra), teh´ at 5–5 ilyen van. A ±120 fokos forgat´asok fixalakzatai k´et szab´ alyos h´ aromsz¨ og egyes´ıt´esei (3. ´ abra), azaz 10 (az a´br´ an x-szel jel¨olt) szomsz´edos pontb´ ol kiv´alasztunk 2-t, ´ e s ezek elforgatottjai adj´ a k a m´ asik 10 odik, hogy 4 pontot, teh´at mindk´et forgat´ asnak 2 fixalakzata van. Ugyan´ıgy ad´ a 180 fokos forgat´asnak

aci´ oknak a halmaz´ at, amelyeknek x fixalakJel¨ olje Fx azoknak a τ transzform´ zata, teh´ at amelyek x-et ¨ onmag´aba viszik. Ha x egy minden ¨ot¨odik gy¨ongy piros alyos hatsz¨ og 12 szimmetri´aj´anak megfelel˝o τi forgat´ıpus´ u hatos, akkor Fx a szab´ t´ asokat ´es t¨ ukr¨ oz´eseket jelenti. Jel¨ olje v´eg¨ ul az ¨ osszes transzform´ aci´ o halmaz´ at T , p´eld´ ankban ez a szab´ alyos 30-sz¨ og 60 szimmetri´aj´ab´ol a´ll. Az el˝oz˝ o r´esz gondolatmenet´eb˝ ol ´ altal´ anosan is azt kapjuk, hogy egy x piros alya b´ armely y piros hatos´ aba ugyanannyi transzform´aci´ o visz ´at, hatost a Px p´ mint amennyi x-et helyben hagyja. Ebb˝ ol az elemsz´amokra az k¨ovetkezik, hogy |T | = |Px | · |Fx |. A feladat megold´ as´ ahoz az ¨ osszes transzform´ aci´ ohoz tartoz´o ¨osszes fixalakzatot sz´ amoljuk o ssze, vagyis o sszegezz¨ u k minden transzform´ aci´ ora a fixalakzatok ¨ ¨  τ fixalakzatainak a sz´ama. Ha ezt most a piros hatosok szerint sz´ asz´ am´at: τ ∈T   ´ moljuk ¨ ossze, akkor ugyanez az ¨ osszeg |Fx | = |T |/|Px |. Atosztva |T |-vel azt x

K¨ oz´ episkolai Matematikai ´ es Fizikai Lapok, 2022/9

3. a ´bra

x

515

516

15 3

fixalakzata van (4. ´ abra).

4. a ´bra K¨ oz´ episkolai Matematikai ´ es Fizikai Lapok, 2022/9

Ugyanez ´ all az oldalfelez˝ o szimmetriatengelyekre t¨ ukr¨oz´esn´el: a tengely egyik oldal´ an lev˝o 15 pontb´ ol 3 szabadon v´ alaszthat´ o ´es ezek t¨ uk¨ork´epe a m´ asik 3 pont (5. ´ abra). Az a´tl´ o szimmetriatengelyekn´el a tengelyen lehet 0 vagy 2 pont ´es a tengely  14egyik15oldal´  an lev˝o 14 pontb´ol ´ıgy 3 vagy 2 v´alaszthat´o szabadon, ez 14 + = fixalakzat (6. ´ abra). 3 2 3

ennek a 90, 180 ´es 270 fokos elforgatottja, teh´at 12 fixalakzat van (7. ´ abra). Hasonl´o a helyzet a 180 fokos forgat´ asn´ al: a k¨ oz´eps˝ o n´egyzet k¨ otelez˝ oen benne van a fixalakzatban, a k¨ oz´eppontra szimmetrikus 24 n´egyzetp´ ar k¨ oz¨ ul pedig kett˝ ot 24 abra). kell teljesen kiv´alasztani, a fixalakzatok sz´ama ´ıgy 2 (8. ´

7. a ´bra

5. a ´bra

Egy t¨ ukr¨ oz´esn´el egy ¨ ot¨ os akkor fixalakzat, ha a tengelyen v´alasztunk 1, 3 vagy 5 n´egyzetet, ´es hozz´ a ennek megfelel˝  7oen 2, 1vagy  0 a tengelyre szimmetrikus 21 7 n´egyzetp´ art, ilyen ¨ ot¨ osb˝ ol teh´at 7 2 + 3 · 21 + 5 van (9. ´es 10. ´ abra). A p´ aly´ ak sz´ ama, azaz a fixalakzatok ´ atlagos sz´ama ´ıgy

6. a ´bra

49

A t¨ obbi transzform´ aci´ onak nincs fixalakzata. ´Igy a p´ aly´ ak sz´ama

30

10

15

+ 2 · 5 + 2 2 + 31 3 6 60

8. a ´bra

5

= 10 133.

24

21

7

7

+ 2 · 12 + 2 + 4 · (7 · 2 + 3 · 21 + 5 8

)

= 239 511.

II. Lyukas n´egyzetek Ebben a r´eszben nem lesz u ´jdons´ag, csak egy m´asik feladatra alkalmazzuk a tanult m´odszert. 3. feladat. Vagabundus Modernissimus, a kiv´ al´ o k´epz˝ om˝ uv´esz leg´ ujabb m˝ ualkot´ asa egy k´ezben vihet˝ o 70 × 70 centim´eteres v´ekony vaslemez, amelynek mindk´et egyform´ an piros oldala 49 egybev´ ag´ o kis n´egyzetre van osztva, ´es ezek k¨ oz¨ ul 5 kis n´egyzet k¨ oz´epen ´ at van lyukasztva. H´ anyf´ele lehet˝ os´ege volt a zseninek ily m´ odon kifejeznie korszakalkot´ o kreativit´ as´ at? Megold´as. Itt a lyukas n´egyzet¨ ot¨ os¨ ok sz´ama a k´erd´es, de a nagy n´egyzet szimmetri´aival egym´ asba vihet˝ o¨ ot¨ os¨ ok ugyanazt a m˝ uvet jelentik. A 8 transzform´ aci´ o teh´ at a 4 darab 90k fokos forgat´as (1  k  4) ´es a 4 szimmetriatengelyre vonatukr¨ oz´es. A p´aly´ ak sz´am´at keress¨ uk, ez a Burnside-lemma szerint a fixalakzat koz´ o t¨ ot¨ os¨ ok ´ atlagos sz´ama. ¨

49

A helyben hagy´ asnak mind a 5 ¨ ot¨ os fixalakzata. A ±90 fokos forgat´asn´ al a fixalakzatban a k¨ oz´eps˝ o kis n´egyzet mindenk´eppen benne van, egy m´ asik az egyik sarokn´ al lev˝o 3 × 4-es t´eglalapb´ ol szabadon v´alaszthat´ o, a marad´ek h´ arom pedig K¨ oz´ episkolai Matematikai ´ es Fizikai Lapok, 2022/9

517

9. a ´bra

10. a ´bra

III. Egy oszthat´ os´agi feladat 4. feladat. L´ assuk be, hogy b´ armely c ´es n pozit´ıv eg´eszre c(1,n) + c(2,n) + . . . + oszthat´ o n-nel. +c (n,n)

Megjegyz´es. Ha n = p = pr´ım, akkor az ¨ osszeg (p − 1)c + cp = cp − c + pc, teh´ at kapjuk, p o p-vel, ami a kis Fermat-t´etel. hogy c − c oszthat´

518

K¨ oz´ episkolai Matematikai ´ es Fizikai Lapok, 2022/9

Bizony´ıt´as. Tekints¨ uk a k¨ ovetkez˝ o probl´em´ at. Egy szab´alyos n-sz¨og cs´ ucsait c sz´ınnel tetsz˝olegesen kisz´ınezz¨ uk. H´ anyf´ele sz´ınez´es ad´ odik, ha az egym´asba forgat´ assal a´tvihet˝ o sz´ınez´esek k¨ oz¨ ott nem tesz¨ unk k¨ ul¨onbs´eget? Itt n transzform´ aci´ onk van, a 360k/n fokos forgat´asok a k¨ oz´eppont k¨ or¨ ul (ahol 1  k  n), ´es a p´ aly´ ak sz´ama a k´erd´es. A Burnsidelemma alkalmaz´ as´ ahoz megsz´amoljuk a 360k/n fokos forgat´as fixalakzatait. Legyen p´eld´ aul n = 10 ´es k = 4, azaz a transzform´ aci´ o a 144 fokos forgat´as ucsot a t˝ o(11. ´ abra). Ez minden cs´ le a forgat´ as ir´any´ aba es˝ o negyedik cs´ ucsba visz. Ha p´eld´ aul az 1-es cs´ ucs piros, akkor ennek alapj´ an az 5-¨ os, 9-es, 3-as ´es 7-es cs´ ucsnak is pirosnak kell lennie, de a t¨ obbi cs´ ucs sz´ın´et ez 11. ´ abra nem befoly´ asolja. Vagyis minden m´ asodik cs´ ucs lesz azonos sz´ın˝ u. Ez onnan ad´ odott, hogy az 1-b˝ ol kiindulva az 1, 1 + 4, 1 + 2 · 4, . . . cs´ ucsokba jutunk, pontosabban ezeknek a sz´ amoknak a 10-zel val´ o oszt´ asi marad´ekait kell venn¨ unk. Az 1-hez teh´ at hozz´aadjuk a 4 t¨ obbsz¨ or¨ oseit ´es ebb˝ ol levonjuk a 10 megfelel˝ o t¨obbsz¨or¨oseit: 1 + 4y − 10z. Mivel a 4y − 10z alak´ u eg´eszek ´eppen a 4 ´es 10 legnagyobb k¨oz¨os oszt´ oj´anak, a 2-nek a t¨ obbsz¨ or¨ osei, ez´ert az 1 + 2t sz´ amokhoz jutunk, ´ıgy j¨ottek ki az 1, 3, 5, 7, 9 cs´ ucsok. ´ Altal´ aban is ez a helyzet tetsz˝ oleges n ´es k eset´en. Egy i cs´ ucs ekkor a 360k/n ucsokba ker¨ ul, fokos forgat´as t¨ obbsz¨ ori alkalmaz´ as´ aval az i + k, i + 2k, i + 3k, . . . cs´ pontosabban ezeknek a sz´ amoknak az n szerinti marad´ek´ at kell venn¨ unk. Mivel az yk − zn sz´ amok (ahol y ´es z v´egigfutnak az eg´eszeken) megegyeznek a legnagyobb k¨ oz¨ os oszt´ o (k, n) t¨ obbsz¨ or¨ oseivel, ez´ert az i cs´ ucs k´epei ´eppen az i + t(k, n) cs´ ucsok lesznek (ahol t eg´esz sz´ am). Azok a sz´ınez´esek lesznek a fixalakzatok, ahol ezek a cs´ ucsok mind azonos sz´ın˝ uek. Ennek megfelel˝oen (k, n) cs´ ucsot tetsz˝ olegesen sz´ınezhet¨ unk, a t¨ obbi sz´ıne viszont m´ar egy´ertelm˝ u. Teh´ at a 360k/n fokos ama ´ıgy forgat´ asnak c(k,n) fixalakzata van. A keresett sz´ınez´esek sz´ n 

c(k,n)

k=1

. n Ez eg´esz sz´ am, ami bizony´ıtja az oszthat´os´ agot. Megjegyz´es. Kev´esb´e eleg´ ansan, egy n szerinti el´eg bonyolult teljes indukci´ oval is bizony´ıthatjuk az oszthat´ os´ agot. Itt az indukci´ os l´ep´esn´el minden k < n-re (´es minden c-re) feltessz¨ uk az a ´ll´ıt´ ast, ´es abb´ ol bizony´ıtunk n-re. Ehhez n-et n = pk r alakban ´ırjuk, ahol p pr´ım ´es (p, r) = 1, majd t´ amaszkodunk arra, hogy az ´ all´ıt´ as igaz n/p-re, r-re, s˝ ot tov´ abbi n-n´el kisebb sz´ amokra is, r´ aad´ asul c helyett esetenk´ent m´ as c eg´esszel.

Freud R´ obert, Horv´ath Eszter

K¨ oz´ episkolai Matematikai ´ es Fizikai Lapok, 2022/9

519

Gyakorl´o feladatsor emelt szint˝ u matematika ´eretts´egire I. r´esz 1. a) Az an sz´ amtani sorozat k¨ ul¨ onbs´ege 4, az els˝ o h´et tagj´ anak ¨ osszege 105. Adjuk meg a sorozat els˝ o tagj´ at. (3 pont) anyadosa 4, az els˝ o h´et tagj´ anak o sszege 16 383. b) A bn m´ertani sorozat h´ ¨ Adjuk meg a sorozat els˝ o tagj´ at. (3 pont) ofok´ u f¨ uggv´enye. A sorozat m´ asodik c) A cn sorozat minden tagja az n els˝ tagja 7, a hetedik tagja 27. Adjuk meg a sorozat els˝o tagj´ at. (5 pont) 2. a) Oldjuk meg az al´abbi egyenletet a val´ os sz´amok halmaz´an: 5−x+1 − 10 · 5−x−1 + 6 · 5−x−2 = 16,2.

(6 pont)

b) Igazoljuk, hogy  2 lg 32x · lg 72x  lg 3x + lg 7x

minden val´ os x eset´en fenn´ all.

(7 pont)

3. Egy u ag´ u, 80 centim´eter oldalhossz´ us´ ag´ u n´egy¨zemben 5 millim´eter vastags´ zet alak´ u ac´ellapokb´ ol a lehet˝ o legnagyobb, szab´ alyos tizenk´etsz¨ ogeket v´ agnak ki u ´gy, hogy a tizenk´etsz¨ ogek k´et-k´et oldala illeszkedjen a n´egyzetlapok oldal´ara. Tudg juk, hogy az ac´el s˝ ur˝ us´ege 7,8 cm3 . a) Mekkora lesz 75 darab legy´ artott tizenk´etsz¨ og t¨ omege? (7 pont) A szab´ alyos tizenk´etsz¨ og cs´ ucsait megsz´ amozzuk sorban 1-t˝ ol 12-ig. A sorsz´ amozott cs´ ucsok k¨ oz¨ ul b´ armelyik h´ arom egy-egy h´aromsz¨ oget alkot. b) Adjuk meg az ´ıgy kapott der´eksz¨ og˝ u ´es nem der´eksz¨ og˝ u h´ aromsz¨ ogek sz´ am´at. (6 pont) 4. Magyarorsz´agon 2022-ig a g´epkocsik (nem egyedi) rendsz´ ama h´ arom bet˝ ub˝ ol ´es h´ arom sz´amjegyb˝ ol a´llt. Az a´b´ec´e 26 bet˝ uje haszn´ alhat´ o ezekben a rendsz´ amokban. a) H´any aut´o kaphat ilyen m´ odon rendsz´amot? (4 pont) b) H´any olyan rendsz´am lehet, amelyikben k´etf´ele bet˝ u, ´es k´etf´ele sz´ amjegy szerepel? (5 pont) c) Az elk´epzelhet˝ o¨ osszes rendsz´ amb´ ol v´eletlenszer˝ uen v´ alasztunk egy darabot. Mekkora a val´ osz´ın˝ us´ege annak, hogy h´ arom k¨ ul¨ onb¨ oz˝ o bet˝ ub˝ ol ´es h´ arom k¨ ul¨ onb¨ oz˝ o sz´ amjegyb˝ ol ´all ez a rendsz´ am? (5 pont) 520

K¨ oz´ episkolai Matematikai ´ es Fizikai Lapok, 2022/9

II. r´esz

8. Egy v´ allalkoz´ as olyan n´egyzet alap´ u egyenes g´ ul´ akat rendel rekl´amaj´anoget z´ arnak be, d´ekt´argyk´ent, amelyeknek az oldallapjai az alaplappal 60◦ -os sz¨ ´es az alap´eleinek a hossza 12 centim´eter. A n´egy kiemelked˝ o term´ek¨ uk rekl´amj´ at szeretn´ek elhelyezni a g´ ula egy-egy oldallapj´an. a) Hat´arozzuk meg mekkora ter¨ ulet˝ u r´eszre kell megtervezni egy term´ek rekl´amj´ at. (3 pont)

5. Adott a k¨ ovetkez˝ o k´et f¨ uggv´eny: f : R → R;

´es g : R → R;

f (x) = −x − 1,

g(x) = −x2 − 10x − 19.

a) Oldjuk meg az f (x) = g(x) egyenletet a val´os sz´amok halmaz´an. (3 pont) b) ´Irjuk fel az y = f (x) ´es az y = g(x) egyenlet˝ u alakzatok k¨oz¨os pontjaiban az y = g(x) egyenlet˝ u g¨ orb´ehez h´ uzhat´ o ´erint˝ok egyenlet´et. (7 pont) c) Sz´am´ıtsuk ki az y = f (x) ´es az y = g(x) egyenlet˝ u alakzatok ´altal k¨ozbez´ art s´ıkidom ter¨ ulet´et. (6 pont) 6. Egy 600 m´eter oldalhossz´ us´ ag´ u, n´egyzet alak´ u parkot fut´ op´alya hat´ arol. A park egyik cs´ ucs´ at´ ol ind´ıtja Andr´as, az edz˝ o a k´et fut´ oatl´et´ aj´at, akik egyszerre indulnak, de m´ as ir´anyban. Andr´as helyben marad, Lali 9 km/h, M´at´e 8 km/h egyenletes sebess´eggel fut az edz´esen. 6 perc eltelt´evel a h´ arom szerepl˝o az ALM h´ aromsz¨ og cs´ ucsaiban van, ahol a h´aromsz¨ og cs´ ucsait a szerepl˝ ok nev´enek a kezd˝ obet˝ uj´evel jel¨ olt¨ uk. a) Milyen messze van ekkor Andr´ast´ ol Lali ´es M´at´e? (4 pont) b) Milyen messze van ekkor egym´ast´ ol a k´et fut´ o? (2 pont) atja az LM szakaszt. c) Igazoljuk, hogy ekkor Andr´ as pontosan 45◦ -os sz¨ogben l´ (5 pont) d) A parkban az L ´es M k¨ oz¨ ott van egy egyenes s´eta´ ut is. Milyen messze van Andr´as ett˝ol az u ´tt´ ol? (5 pont) 7. Bogl´arka ´erdekes sz´amh´ armasokat gy˝ ujt¨ ott, ´es a k¨ovetkez˝ oket a´llap´ıtotta meg: 32 + 42 = 52 , 52 + 122 = 132 , 2

2

b) Mekkora a g´ ula t´erfogata? c) Mekkora sz¨ oget z´ arnak be a g´ ula oldal´elei az alaplappal?

(3 pont) (4 pont)

d) A g´ ula alak´ u dobozok belsej´eben egy-egy (g¨ omb alak´ u) aj´ and´ek labd´ at is elhelyeznek, amelyek mind a n´egy oldallapot ´es az alaplapot is ´erintik. Mekkora a labda sugara? (6 pont) 9. a) Egy matematikatan´ar tan´ıt a 12. A ´es a 12. B oszt´alyban is. ´Iratott egy k¨oz¨os dolgozatot, amelyben 120 pont volt az el´erhet˝o legmagasabb pontsz´ am. Az A oszt´alyban 84 pont, a B oszt´ alyban 74 pont lett az a´tlag. Az A oszt´alyos fi´ uk ´atlagosan 81, a B oszt´ alyos fi´ uk pedig 71 pontot ´ertek el. Az A oszt´ alyban a l´ anyok ´atlagosan 90, m´ıg a B oszt´ alyban a l´ anyok a´tlagosan 76 pontos dolgozatot ´ırtak. Tudjuk tov´abb´ a, hogy az ¨ osszes fi´ u´ atlaga 79 pont lett. Mennyi a k´et oszt´ alyban az ¨osszes l´any ´ atlagpontsz´ ama? (8 pont) b) Az A oszt´ alyban tanul´o Andr´ asnak hat jegye van matematik´ab´ ol, ´es a hat jegynek a medi´anja 4. Mit mondhatunk a B oszt´ alyban tanul´o Benedek matematikajegyeinek medi´anj´ ar´ ol, ha hat jegye pontosan megegyezik Andr´as jegyeivel, de neki van m´eg ezen t´ ul egy hetedik jegye, amely 5-¨ os? (4 pont) c) H´arom bar´ atn˝ o, Anna, Bea ´es Cili matematik´ ab´ ol, fizik´ ab´ ol ´es k´emi´ ab´ ol el´ert f´el´evi eredm´enyeiket vizsg´alta. I. Kisz´amolt´ ak mindegyik¨ uknek az a´tlag´ at, majd ezeknek az a´tlagoknak vett´ek az a´tlag´at. II. Kisz´ amolt´ ak a h´ arom tant´argy ´atlag´ at, majd ezen a´tlagok ´atlag´ at vett´ek. Mutassuk meg, hogy a k´etf´ele m´ odon kapott a´tlag egyenl˝o egym´ assal. (4 pont)

2

7 + 24 = 25 , o Kozma Katalin Abig´el, Sz´amad´o L´aszl´ Budapest Gy˝or

92 + 402 = 412 . a) B´ela megl´ atta Bogl´ arka egyenleteit ´es elgondolkodott azon, hogy lehet-e egy ilyen tulajdons´ ag´ u sz´ amh´ armas legkisebb eleme a 2023. Seg´ıts¨ unk B´el´anak, 2 hat´ arozzuk meg k ∈ Z ´ert´ek´et, ha 20232 + k 2 = (k + 1) . (4 pont) b) B´ alint azt a´ll´ıtja, hogy a v´egtelens´egig folytathat´ o az egyenl˝ os´egek sorozata, azaz minden 2n + 1 (n ∈ N) alak´ u p´aratlan sz´ am n´egyzet´ehez hozz´ aadva a 2n(n + 1) n´egyzet´et, ´eppen a k¨ ovetkez˝ o n´egyzetsz´amot kapjuk. Igazoljuk B´ alint a´ll´ıt´ as´at. (6 pont) o 5-tel. c) Bizony´ıtsuk be, hogy 12023 + 22023 + 32023 + 42023 + 52023 oszthat´ (6 pont) K¨ oz´ episkolai Matematikai ´ es Fizikai Lapok, 2022/9

521

Megold´asv´azlatok a 2022/8. sz´am emelt szint˝ u matematika gyakorl´o feladatsor´ahoz I. r´esz 1. a) Oldjuk meg a 2 · sin2 x + 3 · cos2 x + 2 · sin x = 0 egyenletet a val´ os sz´ amok halmaz´ an. (5 pont) 522

K¨ oz´ episkolai Matematikai ´ es Fizikai Lapok, 2022/9

b) Melyek azok a val´ os sz´ amok, amelyek eleget tesznek az x2 − 4x − 5  0 ´es a

Megold´as. a) A fal 2,8 m´eteres magass´ ag´ anak aranymetsz´es szerinti feloszt´ asa: 2,8 − x 2,8 = . 2,8 − x x

√ π 2 + < cos 2 3 2 x

egyenl˝ otlens´egnek egyar´ ant?

Rendezve az x2 − 8,4x + 7,84 = 0 m´asodfok´ u egyenlethez jutunk. A feladatnak megfelel˝o megold´ as h´ arom tizedesjegyre kerek´ıtve 1,070. Ez´ert a keletkezett t´eglalapok f¨ ugg˝oleges oldalainak hossza 1,070 m ´es 1,730 m.

(8 pont)

Megold´as. a) A cos2 x = 1 − sin2 x helyettes´ıt´est haszn´ alva a k¨ovetkez˝ o m´ asodfok´ u egyenlethez jutunk: −1 · sin2 x + 2 · sin x + 3 = 0. Ennek egyik gy¨oke sin x = 3, aminek nincs val´ os megold´asa az f (x) = sin x f¨ uggv´eny ´ert´ekk´eszlete miatt. A m´ asodfok´ u egyenlet m´ asik gy¨ oke sin x = −1, amib˝ol x = − π2 + k · 2π, ahol k ∈ Z. b) A m´asodfok´ u kifejez´es z´erushelyei −1 ´es 5. A megfelel˝o f¨ uggv´eny felfele ny´ıl´o parabola, amelynek helyettes´ıt´esi ´ert´ekei a k´et z´erushely k¨oz¨ott negat´ıvak, ez´ert az egyenl˝otlens´eg megold´ asa x ∈ [−1; 5]. A trigonometrikus egyenl˝ otlens´eg megold´ asa:

A fal 6,5 m-es v´ızszintes hossz´ us´ ag´ at tekintve az aranymetsz´essel val´o feloszt´as: 6,5 − y 6,5 = . 6,5 − y y

u egyenletet kapjuk. A feladatnak Rendezve az y 2 − 19,5y + 42,25 = 0 m´asodfok´ megfelel˝o megold´ as h´ arom tizedesjegyre kerek´ıtve 2,483. Ez´ert a keletkezett t´eglalapok v´ızszintes oldalainak hossza 2,483 m ´es 4,017 m. A t´eglalapok ter¨ uleteinek nagys´ aga:

x π 7π π + k · 2π < + < + k · 2π. 4 2 3 4

T1 = 2,657 m2 ,

Ezt rendezve: − ó

π 17π + k · 4π < x < + k · 4π. 6 6 î

intervallumot adja. Mindk´et egyenl˝ otlens´egnek Ez k = 0 eset´en a − π6 ; 17π 6 ó ó π a − 6 ; 5 intervallum elemei tesznek eleget. 2. Egy adott hossz´ us´ ag´ u szakaszt az aranymetsz´es szerint u ´gy osztunk k´et r´eszre, hogy az eredeti ´es a keletkezett hosszabb szakasz hossz´ anak ar´ anya megegyezik a keletkezett hosszabb ´es a keletkezett r¨ ovidebb szakasz hossz´ anak ar´ any´ aval. Bence szob´ aja egyik fal´ anak hossza 6,5 m´eter, magass´ aga 2,8 m´eter. Ezt a falfel¨ uletet Bence u ´gy szeretn´e lefesteni, hogy f¨ ugg˝ olegesen ´es v´ızszintesen is az aranymetsz´esnek megfelel˝ oen osztja fel 4 t´eglalap alak´ u r´eszre u ´gy, Bence fala hogy a bal fels˝ o sarok fel´e legyenek a r¨ ovidebb szakaszok. a) Hat´ arozzuk meg az egyes t´eglalapok ter¨ ulet´et. A sz´ amol´ as sor´ an az oldalak hossz´ at ´es a ter¨ uleteket is pontosan 3 tizedesjegyre kerek´ıtve adjuk meg. (8 pont) b) Benc´enek otthon 4-f´ele sz´ın˝ u falfest´eke van, ezekb˝ ol v´ alogat a fal fest´ese sor´ an. H´ any k¨ ul¨ onb¨ oz˝ o sz´ınez´es lehets´eges, ha az oldallal egym´ ashoz illeszked˝ o t´eglalapoknak k¨ ul¨ onb¨ oz˝ o sz´ın˝ ueknek kell lennie? (4 pont) K¨ oz´ episkolai Matematikai ´ es Fizikai Lapok, 2022/9

523

T2 = 4,298 m2 ,

T3 = 6,949 m2

´es T4 = 4,296 m2 .

c) A fal k´etf´ele sz´ınnel 12 k¨ ul¨ onb¨ oz˝ o m´ odon sz´ınezhet˝ o, hiszen ki kell v´ a4laszta nunk a 4-f´ele sz´ınb˝ ol azt a kett˝ ot, amelyeket a fest´esn´el felhaszn´alunk. Ezt 2 = 6f´elek´eppen tehetj¨ uk meg. Majd a bal fels˝ o sarokban l´ev˝ o t´eglalap 4sz´ın´et kiv´alaszjuk (2 lehet˝ os´eg), m´ıg a t¨ obbi t´eglalap sz´ıne m´ar egy´ertelm˝ u lesz. 2 · 2 · 1 · 1 · 1 = 12.

4

H´aromf´ele sz´ınnel t¨ ort´en˝ o fest´es eset´en 3 · 3 · 2 · 2 · 1 = 48 az esetek sz´ama. N´egyf´ele sz´ınre 4! = 24 k¨ ul¨ onb¨ oz˝ o fest´est v´ alaszthat Bence, ez´ert ¨ osszesen 12 + 48 + 24 = 84 a lehet˝os´egek sz´ ama. 3. A l´egk¨ ori nyom´ as f¨ ugg a tengerszinten m´erhet˝ o nyom´ as ´ert´ek´et˝ ol (p0 ), a tengerszint feletti m´eterben m´ert magass´ agt´ ol (h) ´es a leveg˝ o Celsius-sk´ al´ an m´ert h˝ om´ers´eklet´et˝ ol (T ). A hozz´ arendel´es szab´ alya: p = p0 · e

h −0,0342· T +273

.

a) Mekkora a nyom´ as Bol´ıvia f˝ ov´ aros´ aban, La Pazban 3 600 m´eter magass´ ag(2 pont) ban, ha a tengerszinten 101 500 Pa a nyom´ as 20 ◦ C-on? b) A K´ekestet˝ on 1 014 m´eter magass´ agban h´ any %-os nyom´ asv´ altoz´ as ´eszlelheol 22 ◦ C-ra emelkedik? (3 pont) t˝ o, ha a h˝ om´ers´eklet 8 ◦ C-r´ c) Milyen magass´ agban m´erhet˝ o feleakkora nyom´ as, mint a tengerszinten, ami(6 pont) kor a leveg˝ o h˝ om´ers´eklete 24 ◦ C? Megold´as. a) A megfelel˝o ´ert´ekek behelyettes´ıt´ese ut´ an 3600 −0,0342· 20+273

p = 101 500 · e

= 66 676,6 ≈ 66 680 Pa

a l´egk¨ori nyom´ as La Pazban. 524

K¨ oz´ episkolai Matematikai ´ es Fizikai Lapok, 2022/9

b) A K´ekestet˝on 8◦ C-on a nyom´ as ´ert´eke p0 · e

22◦ C-on pedig p0 ·

1014 −0,0342· 22+273 e

1014 −0,0342· 8+273

= p0 · 0,8891. Mivel

0,8891 − 0,8839 = 0,0052 ´es

= p0 · 0,8839,

0,0052 = 0,0059, 0,8839

az´ert 0,59%-os n¨ oveked´es tapasztalhat´ o a l´egk¨ ori nyom´ asban. c) Megoldand´ oa 0,5 · p0 = p0 · e

h −0,0342· 24+273

egyenlet. Oszthatunk a p0 tengerszinten m´erhet˝ o nyom´assal, majd vegy¨ uk mindk´et oldal term´eszetes alap´ u logaritmus´ at (ln). ´Igy ln 0,5 = −0,0342 ·

h . 297

Rendez´es ut´ an h = 6 019,4 teh´ at k¨ or¨ ulbel¨ ul 6 020 m magass´ agban lesz a nyom´ as fele a tengerszinten m´erhet˝ onek. 4. Az AB0 v´ercsoportrendszerben az emberek n´egy alapvet˝ o fenot´ıpusba sorolhat´ ok. A magyarorsz´ agi popul´ aci´ ot figyelembe v´eve az A v´ercsoport´ uak a n´epess´eg 44%-´ at teszik ki, a 0 v´ercsoport´ uak 40%-ot. A B v´ercsoport´ uak ar´ anya 11%, m´ıg az AB v´ercsoport´ uak mind¨ ossze 5%-ot adnak. Ett˝ ol a csoportos´ıt´ ast´ ol f¨ uggetlen¨ ul ol bea v¨ or¨ osv´ertestek felsz´ın´en tal´ alhat´ o D antig´en megl´ete eset´en Rh+ v´ercsoportr´ a az emberek 15%-a sz´el¨ unk, a D antig´en hi´ anya eset´en Rh− a v´ercsoport, ahov´ tartozik. a) Igazoljuk R´eka ´ all´ıt´ as´ at, aki azt mondja, hogy a Magyarorsz´ agon ´el˝ o 9,7 milli´ o lakosb´ ol mind¨ ossze k¨ or¨ ulbel¨ ul 72 750 ember tartozik a legritk´ abb AB Rh− v´ercsoportba. (2 pont) b) Cseng´er˝ ol tudjuk, hogy van D antig´en a v´er´eben. Mekkora val´ osz´ın˝ us´eggel B v´ercsoport´ u Csenge? V´ alaszunkat indokoljuk. (2 pont) c) K´esz´ıts¨ unk k¨ ordiagramot a sz¨ uks´eges k¨ oz´epponti sz¨ ogek meghat´ aroz´ asa ut´ an, amely mutatja a magyar embereket v´ercsoportjuk alapj´ an, figyelembe v´eve mind az AB0 rendszert, mind a D antig´en megl´et´et. (5 pont) d) Egy v´erad´ asr´ ol sz´ ol´ o telth´ azas el˝ oad´ ason a 150 f˝ os teremben f´erfiak, n˝ ok ´es gyerekek u ol kimenne 2 f´erfi, akkor az ott marad´ o f´erfiak ´es n˝ ok ¨lnek. Ha a teremb˝ aar´ anya 2 : 3 lenne. Ha a terembe bej¨ onne m´eg 2 gyerek, akkor a n˝ ok pontosan h´ romszor annyian lenn´enek, mint a gyerekek. H´ any n˝ o vett r´eszt ezen az el˝ oad´ ason? (6 pont) Megold´as. a) Az AB0 v´ercsoportrendszer ´es a D antig´en megl´ete egym´ ast´ ol o emberek sz´ ama Magyarorsz´agon f¨ uggetlen, ´ıgy az AB Rh− v´ercsoportba tartoz´ 9 700 000 · 0,05 · 0,15 = 72 750.

b) Az AB0 v´ercsoportrendszer ´es a D antig´en megl´ete egym´ ast´ ol f¨ uggetlen, ez´ert Csenge 0,11 val´osz´ın˝ us´eggel B v´ercsoport´ u.

K¨ oz´ episkolai Matematikai ´ es Fizikai Lapok, 2022/9

525

c) Az AB0 rendszert, valamint a D antig´en megl´et´et figyelembe v´eve a keresett val´osz´ın˝ us´egek ´es k¨ oz´epponti sz¨ ogek, illetve a k¨ ordiagram:

A A 0 0 B B AB AB

Rh+ Rh− Rh+ Rh− Rh+ Rh− Rh+ Rh−

0,3740 0,0660 0,3400 0,0600 0,0935 0,0165 0,0425 0,0075

134,64◦ 23,76◦ 122,40◦ 21,60◦ 33,66◦ 5,94◦ 15,30◦ 2,70◦

d) Jel¨olje a teremben l´ev˝ o f´erfiak sz´ am´at f , n˝ ok sz´am´at n, a gyerekek sz´am´at pedig g. A feladat sz¨ ovege alapj´ an a k¨ ovetkez˝ o egyenletek ´ırhat´oak fel: f + n + g = 150,

2 f −2 = , n 3

3(g + 2) = n.

A m´asodik egyenletb˝ol kifejezz¨ uk a f´erfiak sz´ am´at: f = 23 n + 2. A harmadik egyenletb˝ ol kifejezz¨ uk a gyerekek sz´ am´at: g = 13 n − 2. Ezeket behelyettes´ıtj¨ uk az els˝ o egyenletbe, ´es rendezz¨ uk. Az eredm´eny n = 75, teh´at az el˝oad´ ason 75 n˝o vett r´eszt. Ellen˝orz´essel meggy˝ oz˝ od¨ unk a megold´ as helyess´eg´er˝ ol. A f´erfiak sz´ ama 52 volt a teremben. Ha ketten elmennek, val´oban 2 : 3 lesz a f´erfiak ´es n˝ ok ar´anya. Ha a teremben l´ev˝ o 23 gyerekhez m´eg ketten bej¨ onnek, akkor a n˝ ok t´enyleg h´ aromszor annyian lesznek. II. r´esz 5. a) Elkezdt¨ uk ¨ osszeadni a 7-tel osztva 5 marad´ekot ad´ o pozit´ıv eg´esz sz´ amokat a legkisebb ilyen tulajdons´ ag´ u sz´ amt´ ol kezdve. H´ any tagot adtunk ¨ ossze, ´es mi az utols´ o sz´ am, ha a kapott ¨ osszeg 54 875? (4 pont) b) Egy m´ertani sorozat hatodik ´es nyolcadik tagja egyar´ ant 6. Sz´ am´ıtsuk ki a sorozat els˝ o 35 tagj´ anak ¨ osszeg´et. (4 pont) c) Egy sz´ amtani sorozat h´ arom egym´ ast k¨ ovet˝ o elem´enek o sszege 72. Ha az els˝ o ¨ sz´ amb´ ol elvesz¨ unk 4-et, a k¨ oz´eps˝ ot v´ altozatlanul hagyjuk, az utols´ ohoz pedig hozz´ aadunk 16-ot, akkor egy m´ertani sorozat h´ arom egym´ ast k¨ ovet˝ o tagj´ at kapjuk. Hat´ arozzuk meg a m´ertani sorozat h´ anyados´ at. (8 pont) Megold´as. a) Az ¨ osszeg tagjai olyan sz´ amtani sorozatot alkotnak, amelynek aja d = 7. A sz´ amtani sorozat ¨ osszegk´eplet´ebe behelyetels˝ o tagja a1 = 5, differenci´ tes´ıtve az ismert ´ert´ekeket:  n · 2 · 5 + (n − 1) · 7 = 54 875. 2 526

K¨ oz´ episkolai Matematikai ´ es Fizikai Lapok, 2022/9

Rendezz¨ uk az egyenletet: 7n2 + 3n − 109 750 = 0. Ennek pozit´ıv eg´esz megold´ asa n = 125. Teh´ at 125 tagot adtunk ¨ ossze. Az utols´ o sz´am a sorozat 125. eleme: a125 = 5 + 124 · 7 = 873.

6 A f¨ uggv´eny helyettes´ıt´esi ´ert´ekei akkor lesznek eg´eszek, ha a x−2 t¨ ort ´ert´eke eg´esz. A nevez˝ o ´ert´eke −6, −3, −2, −1, 1, 2, 3 ´es 6 lehet. Teh´at a f¨ uggv´eny val´ oban 8 db r´acsponton halad a´t.

b) Az adott m´ertani sorozat eset´en a6 = a1 · q 5 = 6 ´es a8 = a1 · q 7 = 6. A k´et ol q = ±1. egyenlet h´ anyados´ at vessz¨ uk, ´ıgy q 2 = 1, amib˝

Ha q = 1, akkor a sorozat minden tagja ai = 6, az els˝ o 35 tag o¨sszege S35 = 35 · 6 = 210. Ha q = −1, akkor a sorozat els˝o tagja a1 = −6, az els˝o 35 tag ¨osszege 35

S35 = (−6) ·

(−1) − 1 = −6. (−1) − 1

c) Jel¨ olje a sz´ amtani sorozat k¨ oz´eps˝ o tagj´ at a, differenci´ aj´at d. Ekkor a sorozatok h´ arom egym´ ast k¨ ovet˝o tagja: sz´ amtani sorozat: m´ertani sorozat:

a − d, a − d − 4,

a, a,

a + d; a + d + 16.

A sz´ amtani sorozat tagjainak ¨ osszege 72. Teh´ at (a − d) + a + (a + d) = 72, amib˝ol a = 24. Ezt felhaszn´alva a m´ertani sorozat egym´ ast k¨ovet˝o tagjai: 20 − d,

24 ´es 40 + d.

Felhaszn´ aljuk, hogy a m´ertani sorozat egym´ ast k¨ ovet˝o tagjai eset´en a k¨oz´eps˝ o tag n´egyzete megegyezik a k´et sz´els˝ o tag szorzat´aval: (20 − d)(40 + d) = 242 . A z´ ar´ ojelek felbont´asa ut´ an d2 + 20d − 224 = 0. Az egyenlet k´et megold´ asa d = 8, illetve d = −28. Az els˝ o esetben a m´ertani sorozat tagjai 12, 24 ´es 48, h´ anyadosa q = 2. A m´asodik esetben a m´ertani sorozat tagjai 48, 24 ´es 12, h´ anyadosa pedig q = 12 . f¨ uggv´enyt. 6. Tekints¨ uk az f (x) = 4x−14 x−2 a) Adjunk meg egy olyan eg´esz sz´ amot, amelyre az f (x) f¨ uggv´eny helyettes´ıt´esi ´ert´eke is eg´esz sz´ am. (2 pont) b) Bizony´ıtsuk be, hogy pontosan 8 darab r´ acsponton halad a ´t az f (x) f¨ uggv´eny k´epe a Descartes-f´ele der´eksz¨ og˝ u koordin´ atarendszerben. (8 pont)

√ os sz´ amok halmaz´ an. (6 pont) c) Oldjuk meg a f (x) = x − 3 egyenletet a val´

x−2 x y = f (x)

−6 −4 5

−3 −1 6

−2 0 7

−1 1 10

1 3 −2

2 4 1

3 5 2

6 8 3

c) N´egyzetre emel´es ut´ an a 4x−14 = x − 3 egyenlethez jutunk. Szorozzunk be x−2 asodfok´ u a nevez˝ ovel, majd bontsunk z´ar´ ojelet. Rendez´es ut´ an x2 − 9x + 20 = 0. A m´ egyenlet k´et megold´ asa x1 = 4, illetve x2 = 5. Az egyenlet ellen˝orz´es´er˝ ol ne feledkezz¨ unk meg, hiszen nem vizsg´ altuk az ´ertelmez´esi tartom´anyt, ´es a n´egyzetre emel´es nem ekvivalens a´talak´ıt´ as. Mindk´et megold´ as kiel´eg´ıti az eredeti egyenletet. 7. Egy konyhai m˝ uanyag t¨ olcs´er als´ o r´esze henger alak´ u, bels˝ o´ atm´er˝ oje 18 millim´eter, magass´ aga 5 centim´eter. Fels˝ o r´esze a hengerre pontosan illeszked˝ o csonkak´ up, amelynek fels˝ o´ atm´er˝ oje 7 centim´eter, illetve magass´ aga 4 centim´eter. a) A t¨ olcs´er alj´ at befogjuk, ´es teljes magass´ ag´ anak 90%-´ aig megt¨ oltj¨ uk v´ızzel. H´ any deciliter v´ız lesz a t¨ olcs´erben? (6 pont) b) Mekkora egy t¨ olcs´er t¨ omege, ha a falvastags´ aga mindenhol 1 millim´eter, g uanyag s˝ ur˝ us´ege 0,92 cm3 ? A m˝ uanyag t´erfogat´ anak kisz´ am´ıt´ as´ ahoz haszn´ aljuk a m˝ azt a k¨ ozel´ıt´est, amely szerint a t¨ olcs´er bels˝ o felsz´ın´et szorozzuk a falvastags´ aggal. (4 pont) c) L´ezerf´ennyel fel¨ ulr˝ ol f¨ ugg˝ olegesen belevil´ ag´ıtunk a t¨ olcs´erbe. Mekkora a val´ osz´ın˝ us´ege, hogy a l´ezerf´eny a t¨ olcs´er als´ o ny´ıl´ as´ an j¨ on ki? (3 pont) d) 50 darab t¨ olcs´erb˝ ol ´ atlagosan 2 anyaghib´ asat k´esz´ıt a gy´ art´ osor. Mekkora a val´ osz´ın˝ us´ege, hogy 135 darab elk´esz´ıtett t¨ olcs´er k¨ oz¨ ott van anyaghib´ as? A v´ alaszt n´egy tizedesjegyre kerek´ıtve adjuk meg. (3 pont) uk be a k¨ ovetkez˝ o jel¨ ol´eseket: az als´o, henger alak´ u r´esz Megold´as. a) Vezess¨ sugara r = 0,9 cm, magass´ aga m = 5 cm. A fels˝ o, csonkak´ up alak´ u r´esz fed˝ ok¨ or´enek olcs´er magass´ aga Mteljes = sugara R = 3,5 cm, magass´ aga M1 = 4 cm. A teljes t¨ oltve 90%-´ aig, teh´at 8,1 cm-ig van. = M1 + m = 9 cm, v´ızzel t¨ Az als´ o henger tele van v´ızzel, ennek t´erfogata Vh = r2 π · m = 0,92 · π · 5 = up M2 = 8,1 − 5 = 3,1 cm-es magass´ agig van t¨ oltve, ennek = 12,72 cm3 . A csonkak´ keresztmetszete:

Megold´as. a) P´eld´ aul x = 1 eset´en f (1) = 10. (A b) feladatr´esz megold´ as´an´al l´ athat´ o t´abl´ azatban szerepel a 8 db lehets´eges ´ert´ek.) b) V´egezz¨ uk el az eg´eszr´esz lev´ alaszt´ ast: f (x) =

4(x − 2) − 6 6 4x − 14 = =4− . x−2 x−2 x−2

K¨ oz´ episkolai Matematikai ´ es Fizikai Lapok, 2022/9

527

528

K¨ oz´ episkolai Matematikai ´ es Fizikai Lapok, 2022/9

7−1,8

Megold´as. a) A 73 kettes sz´amrendszerbeli alakja 10010012 , mert

CG = 2 = 2,6 cm. A BCG ∼ BEH, mert a megfelel˝o sz¨ogek nagys´ aga ´ egyenl˝ o. Irjuk fel a h´ aromsz¨ ogek megfelel˝ o oldalainak ar´ any´ at: EH CG = , GB HB

ebb˝ ol x = 2,015 cm. Ezzel F E = 2 · 2,015 + 1,8 = 5,83 cm, a v´ız felsz´ın´enek sugara R2 = F2E = 2,915 cm. A csonkak´ up alak´ u r´eszben l´ev˝ o v´ız t´erfogata: Vcsk =

 3,1 · π   M2 · π  2 R2 + R2 r + r 2 = 2,9152 + 2,915 · 0,9 + 0,92 = 38,73 cm3 . 3 3

´Igy a belet¨ olt¨ ott v´ız teljes t´erfogata 12,72 + 38,73 = 51,45 cm3 , ami kerek´ıtve 0,51 dl. b) A henger bels˝ o felsz´ın´enek (pal´astj´ anak) nagys´ aga

32 0

16 0

8 1

4 0

2 0

1 1

Ez a sz´am visszafel´e olvasva val´ oban az eredetivel azonos. b) A k alap´ u sz´ amrendszerben a h´ aromjegy˝ u sz´ amok sz´amjegyeihez tartoz´ o u h´ aromjegy˝ u sz´ am 10-es sz´amrendhelyi´ert´ekek rendre k 2 , k ´es 1. ´Igy a 345 alak´ u sz´ amrendszerben a helyiszerbeli ´ert´eke 3 · k 2 + 4 · k + 5. A 2-vel nagyobb alap´ 2 u h´ aromjegy˝ u sz´ am 10-es ´ert´ekek hasonl´ oan (k + 2) ; (k + 2) ´es 1, ´ıgy a 345 alak´ 2 osszege ad 696-ot. ´Irjuk sz´ amrendszerbeli ´ert´eke 3 · (k + 2) + 4 · (k + 2) + 5. Ezek ¨ fel a megfelel˝ o egyenletet: 2

A csonkak´ up bels˝ o felsz´ın´ehez is csak a pal´ astj´anak ter¨ ulet´et kell kisz´ am´ıtani. Ehhez sz¨ uks´eg¨ unk van a csonkak´ up alkot´ oj´anak, a BC szakasz hossz´ anak kisz´am´ıt´as´ara. ´Irjunk fel Pitagorasz-t´etelt a BCG h´ aromsz¨ ogben: BC 2 = 2,62 + 42 . Ebb˝ ol a = BC = 4,77 cm. Acsk = (R + r)aπ = (3,5 + 0,9) · 4,77 · π = 65,94 cm2 . A teljes bels˝ o felsz´ın 94,21 cm2 . A m˝ uanyag t´erfogata a felsz´ın ´es az anyagvastaguanyag t¨omege a t´erfogat´ anak ´es s´ ag (1 mm = 0,1 cm) szorzata: 9,421 cm3 . A m˝ a s˝ ur˝ us´eg´enek ¨ osszeszorz´ as´aval sz´am´ıthat´ o ki, ez´ert egy t¨olcs´er 8,67 gramm. c) A keresett val´ osz´ın˝ us´eget a t¨ olcs´er fels˝ o k¨ or´enek ter¨ ulete (T ), ´es als´ o ny´ıl´as´ anak ter¨ ulete (t) meghat´ aroz´ asa ut´ an tudjuk megadni geometriai val´ osz´ın˝ us´egi modell alapj´an. 49 81 π, t = 0,92 π = π. T = 3,52 π = 4 100 ´Igy p = t = 81 annak val´ osz´ın˝ us´ege, hogy a t¨ olcs´er alj´ an j¨on ki a l´ezerf´eny.

A z´ar´ ojelek felbont´asa ´es a lehets´eges ¨ osszevon´ asok ut´an 6k 2 + 20k − 666 = 0. Az egyenlet eg´esz megold´ asa k = 9. A sz´ amrendszerek alapsz´ama teh´ at 9 ´es 11. amA 9-es sz´amrendszerbeli 345 sz´am 10-es sz´amrendszerbeli ´ert´eke 284, a 11-es sz´ rendszerbeli´e pedig 412. A k´et sz´ am o oban 696. ¨sszege val´ c) A legkisebb 10-es sz´amrendszerbeli h´ aromjegy˝ u sz´ am a 100, ennek 9-es aromjegy˝ u sz´ am. A legnagyobb 9-es alap´ u sz´ amrendszerbeli alakja 1219 . Ez h´ amrendszerbeli ´ert´esz´ amrendszerbeli h´ aromjegy˝ u sz´ am a 8889 , ennek 10-es sz´ ol 728-ig b´ armely sz´ am megfelel a felt´etelke 8 · 92 + 8 · 9 + 8 = 728. ´Igy 100-t´ nek, ez´ert a kedvez˝ o esetek sz´ ama 728 − 100 + 1 = 629. Az ¨ osszes eset sz´ ama 900, mert ennyi h´ aromjegy˝ u sz´ am van a 10-es sz´ amrendszerben. A keresett val´osz´ın˝ us´eg = 0,6989. p = 629 900 ´ azoljuk koordin´ 9. a) Abr´ atarendszerben a k¨ ovetkez˝ o A ponthalmazt: 2 A = (x; y) ∈ R | 4x + 3y  15 . ´ azoljuk koordin´ b) Abr´ atarendszerben a B ponthalmazt: B = (x; y) ∈ R2 | x2 + y 2 − 14x − 8y + 40  0 .

1225

2 50

d) Annak a val´ osz´ın˝ us´ege, hogy egy t¨ olcs´er anyaghib´ as p = = 0,04; annak, hogy j´o (1 − p) = 0,96. El˝osz¨ or sz´am´ıtsuk ki annak a val´ osz´ın˝ us´eg´et, hogy az o¨sszes elk´esz´ıtett t¨ olcs´er hib´ atlan. Ez 135 hib´ atlan term´eket jelent, ez´ert ennek val´ osz´ın˝ us´ege 0,96135 . A komplementer esem´eny jelenti azt, hogy van a t¨olcs´erek k¨oz¨ott anyaghib´ as. N´egy tizedesjegyre kerek´ıtve P (A) = 1 − 0,96135 = 0,9960. 8. a) Sheldon Cooper kedvenc sz´ ama a 73, mert ez a 21. pr´ım ´es 7 · 3 ´eppen 21. S˝ ot, a 73 kettes sz´ amrendszerbeli alakja palindromsz´ am, vagyis visszafel´e olvasva az eredetivel azonos. Igazoljuk ez ut´ obbi kijelent´est. (2 pont) b) Egy adott alap´ u, ´es az enn´el 2-vel nagyobb alap´ u sz´ amrendszerben tekints¨ uk u h´ aromjegy˝ u sz´ amokat, ezek ¨ osszege 69610 . Adjuk meg az ¨ osszeadand´ o a 345 alak´ sz´ amok ´ert´ek´et a 10-es sz´ amrendszerben fel´ırva. (8 pont) c) V´eletlenszer˝ uen kiv´ alasztunk egy 10-es sz´ amrendszerbeli h´ aromjegy˝ u sz´ amot. Mekkora a val´ osz´ın˝ us´ege, hogy a sz´ am 9-es sz´ amrendszerbeli alakja is h´ aromjegy˝ u? (6 pont) K¨ oz´ episkolai Matematikai ´ es Fizikai Lapok, 2022/9

64 1

3 · k 2 + 4 · k + 5 + 3 · (k + 2) + 4 · (k + 2) + 5 = 696.

Ah = 2rπm = 2 · 0,9 · π · 5 = 28,27 cm2 .

T

helyi´ert´ekek alaki ´ert´ekek

2,6 x = , 4 3,1

529

(3 pont)

(5 pont)

c) Igazoljuk, hogy az F (−3; −4) f´ okuszpont´ u v : y = −6 vez´eregyenes˝ u parabola egyenlete y = 0,25x2 + 1,5x − 2,75. (4 pont) 2 ´ aba h´ uzott ´erind) Irjuk fel a y = 0,25x + 1,5x − 2,75 parabola (−1; −4) pontj´ t˝ oj´enek egyenlet´et. (4 pont) Megold´as. a) Rendezz¨ uk y-ra az egyenl˝otlens´eget: y  − 43 x + 5. A kifejez´es

az y-tengelyt az 5-n´el metsz˝ o, − 43 meredeks´eg˝ u egyenest, ´es a felette l´ev˝ o s´ıkr´eszt adja meg, ez az A ponthalmaz. b) Teljes n´egyzett´e alak´ıt´ assal rendezz¨ uk az egyenl˝ otlens´eget: 2

2

(x − 7) − 49 + (y − 4) − 16 + 40  0, 2

2

(x − 7) + (y − 4)  25.

530

K¨ oz´ episkolai Matematikai ´ es Fizikai Lapok, 2022/9

I. megold´as. Legyen DE ´es AC metsz´espontja R, AF ´es BD metsz´espontja pedig S. Elegend˝ o bizony´ıtanunk, hogy R = P ´es S = Q. Az ABCD n´egyzet AC ´es BD ´ atl´ oi az M pontban mer˝ olegesen metszik egym´ ast.

a)

A Thal´esz-t´etel alapj´ an BEC = abb´ a E ´es F felezik = CF D = 90◦ , tov´ a megfelel˝ o k¨ or´ıveket, ez´ert BE = CE, valamint CF = DF , azaz BEC ´es CF D egyenl˝ o sz´ ar´ u der´eksz¨ og˝ u h´aromsz¨ ogek, amelyek BC = CD miatt egybev´ ag´ ok is. Tekints¨ uk az 1. ´ abr´ at.

b)

A B ponthalmaz egy (7; 4) k¨ oz´eppont´ u, r = 5 egys´eg sugar´ u z´art k¨orlap. c) A parabola param´etere a f´okuszpontj´ anak ´es vez´eregyenes´enek t´avols´ aga: p = 2, a parabola tengelypontj´ anak koordin´at´ ai T (−3; −5). A f´ okuszpont a vez´eregyenes felett helyezkedik el, ez´ert a keresett alakzat felfel´e ny´ıl´o parabola. Ezek felhaszn´ al´as´aval a parabola egyenlete: 2 1  y − (−5) = x − (−3) . 2·2 A nevezetes azonoss´ ag alkalmaz´asa ut´ an v´egezz¨ uk el a beszorz´ asokat ´es a rendez´est. Val´ oban az y = 0,25x2 + 1,5x − 2,75 egyenletet kapjuk.

d) Az alakzat ´erint˝oj´enek meredeks´eg´et az f (x) = 0,25x2 + 1,5x − 2,75 f¨ uggv´eny deriv´ al´asa ut´ an kapjuk, ha kisz´ am´ıtjuk annak x = −1 helyen felvett helyettes´ıt´esi ´ert´ek´et: f  (x) = 0,5x + 1,5;

A fentiek szerint BEC ´es CF D egybev´ ag´ o, egyenl˝o sz´ ar´ u der´eksz¨ og˝ u h´ aromsz¨ ogek, ez´ert EC = F C is igaz, tov´ abb´ a ECB + BCD + DCF  = 180◦ , vagyis az E, C, F pontok egy egyenesen vannak, ´es C az EF szakasz felez˝opontja. Az AC ´ atl´ o a n´egyzet BC ´es CD oldala1. a ´bra oget z´ ar be, ez az el˝ oz˝ oek ival is 45◦ -os sz¨ alapj´an azt is jelenti, hogy AC mer˝ oleges az EF szakaszra, ´es ´ıgy RC p´ arhuzamos DF -fel. Eszerint RC az EF D h´ aromsz¨ og k¨ oz´epvonala, ´es ez´ert R a DE szakasz felez˝opontja, teh´at val´oban teljes¨ ul, hogy R = P . Az el˝oz˝ oek alapj´ an k¨ onnyen l´ athat´ o, hogy ACF der´eksz¨ og˝ u h´ aromsz¨ og, amelynek AC befog´ oj´at az M S szakasz mer˝olegesen felezi, ´es mivel M S p´ arhuzamos CF -fel, ez´ert M S az ACF h´ aromsz¨ og k¨ oz´epvonalak´ent felezi az AF szakaszt. ´Igy azt is bel´ attuk, hogy S = Q, ´es ezzel a feladat a´ll´ıt´ as´ at igazoltuk.

f  (−1) = 1.

(A K¨ oMaL honlapon l´ athat´ o megold´ as)

Az ´erint˝o egyenlete: y − (−4) = = x − (−1), rendezve y = x − 3. J´ocsik Csilla Gy˝or

II. megold´as. Legyenek az AB, BC, CD, DA oldalak felez˝ opontjai rendre G, H, K, L, ´es legyen a n´egyzet oldalhossza a. Nyilv´ anval´o, hogy a H ´es K pontok a BC, illetve CD ´ atm´er˝ oj˝ u f´elk¨ or¨ ok k¨ oz´eppontjai, ez´ert (1)

a ; 2

KC = KD = KF =

a . 2

Thal´esz t´etele miatt BEC = CF D = 90◦ , ´ıgy (1) figyelembev´etel´evel BEC ´es CF D egybev´ ag´ o, egyenl˝o sz´ ar´ u der´eksz¨ og˝ u h´ aromsz¨ ogek, valamint HE ⊥ BC ´es F K ⊥ CD.

C gyakorlatok megold´asa

C. 1729. Az ABCD n´egyzet BC ´es CD oldal´ ara mint a ´tm´er˝ ore a k1 , illetve or¨ oket rajzoljuk a n´egyzeten k´ıv¨ ulre. A k´et f´elk¨ or´ıv felez˝ opontja E, illetve F . k2 f´elk¨ A DE ´es AF szakasz felez˝ opontja P , illetve Q. Mutassuk meg, hogy P a n´egyzet AC ´ atl´ oj´ ara, Q pedig a n´egyzet BD ´ atl´ oj´ ara illeszkedik. K¨ oz´ episkolai Matematikai ´ es Fizikai Lapok, 2022/9

HB = HC = HE =

531

Tekints¨ uk a 2. ´ abr´ at, amelyen el˝ osz¨ or az ADE h´ aromsz¨ og tulajdons´agait vizsg´aljuk. A n´egyzet LH k¨ oz´epvonala a´tmegy a n´egyzet O-val jel¨ olt k¨ oz´eppontj´an ´es mer˝ oleges az AD ´es a BC odalalakra. Mivel HE is mer˝ oleges BC-re, ´ıgy L, O, H ´es E egy egyenesre esnek, m´egpedig a n´egyzet LH k¨ oz´epvonal´ anak egyenes´ere. Ez azt 532

K¨ oz´ episkolai Matematikai ´ es Fizikai Lapok, 2022/9

jelenti, hogy EL a h´ aromsz¨ og s´ ulyvonala. Tudjuk m´eg, hogy LO = OH = HE = a2 , teh´at O a h´ aromsz¨ og s´ ulypontja.

Mivel HE is mer˝ oleges BC-re, ez´ert az O, H, E pontok egy egyenesen vannak, az OE szakasz teh´at p´arhuzamos CD-vel ´es OE = CD = a. Ez pontosan azt jelenti, hogy OECD paralelogramma, amelynek DE ´es CO ´atl´ oi a DE szakasz P felez˝opontj´aban metszik egym´ast, vagyis P rajta van CO szakaszon ´es ´ıgy az AC ´ atl´ on is. Hasonl´oan egyszer˝ u m´ odon l´athatjuk be, hogy OF DA is paralelogramma, amelynek AF ´es DO ´ atl´ oi az AF szakasz Q felez˝opontj´aban metszik egym´ast, ez´ert Q illeszkedik a DO szakaszra ´es ´ıgy a BD ´ atl´ ora is. Vint´e csapat: Haj´ os-Szab´ o M´ at´e ´es Krizs´ an Vince L´ aszl´ o (Budapest, Berzsenyi D´ aniel Gimn., 10. ´evf.) IV. megold´as. Mivel E, F a k1 , illetve k2 f´elk¨ or¨ ok felez˝opontjai, ez´ert a Thal´eszt´etelt is figyelembe v´eve a BEC ´es CF D egybev´ ag´ o, egyenl˝o sz´ ar´ u der´eksz¨ og˝ u h´aat az E, romsz¨ogek, ez´ert egyr´eszt ECF  = ECB + BCD + F CD = 180◦ , teh´ C, F pontok egy egyenesen vannak, m´asr´eszt

2. a ´bra

(1)

3. a ´bra

Most tekints¨ uk az OP szakaszt. Mivel O a s´ ulypont, ez´ert ez a szakasz a h´aromsz¨ og s´ ulyvonala, ´es ´ıgy P az ED oldal felez˝ opontja. M´asr´eszt AO a n´egyzet atl´ ´ oj´anak egyenese, teh´ at P illeszkedik az AC ´ atl´ ora.

EBC = F DC = 45◦ .

Tekints¨ uk az 5. a ´br´ at, ahol az A ponton kereszt¨ ul p´ arhuzamost h´ uztunk az EF egyenessel, ez a DF egyenest az R pontban metszi.

A 3. ´ abra seg´ıts´eg´evel igazolhatjuk, hogy az AF szakasz Q felez˝opontja illeszkedik a BD ´ atl´ ora. Az el˝oz˝ o esethez hasonl´ oan bel´ athatjuk, hogy az ABF h´aromsz¨ ognek GF az egyik s´ ulyvonala, O a s´ ulypontja ´es ´ıgy BQ szint´en s´ ulyvonal, ´es Q felezi az AF szakaszt. M´ asr´eszt Q illeszkedik BO egyenes´ere, vagyis a BD ´atl´ ora is. ´ Nagy Anna Eva (Szentendre, Ferences Gimn´ azium, 10. ´evf.) dolgozata alapj´an III. megold´as. Legyenek az AB, BC, CD, DA oldalak felez˝ opontjai rendre G, H, K, L, a n´egyzet a´tl´ oinak metsz´espontja O, ´es legyen a n´egyzet oldalhossza a. Nyilv´anval´ o, hogy a H ´es K pontok a BC, illetve CD ´ atm´er˝ oj˝ u f´elk¨ or¨ ok k¨ oz´eppontjai, ez´ert

5. ´ abra

Az (1) o¨sszef¨ ugg´esb˝ol CBA = 90◦ alapj´an az is k¨ ovetkezik, hogy az EB oget z´ ar be, de akkor AB CD ´es (1) szerint egyenes az AB egyenessel 45◦ -os sz¨ az EB ´es F D egyenesek is p´ arhuzamosak. Ugyanakkor a n´egyzet AC ´ atl´ oja 45◦ -os sz¨oget z´ ar be az AB egyenessel, ez pedig egyen´ert´ek˝ u azzal, hogy

a HB = HC = HE = ; 2 KC = KD = KF =

a . 2

(2)

Tudjuk, hogy OH ´es OK mer˝ oleges BC-re, illetve CD-re, valamint OH = OK =

a , 2

4. ´ abra

Az EC = F C ´es BO = DO, illetve (2) miatt AC az EB ´es F D egyenesek k¨ oz´epp´ arhuzamosa. Ez azt jelenti, hogy AC tartalmazza az EB ´es F D egyenesek E ´es D pontjait ¨osszek¨ ot˝ o szakasz felez˝ opontj´at, azaz a P pontot is.

ez´ert OE = OH + HE = a.

K¨ oz´ episkolai Matematikai ´ es Fizikai Lapok, 2022/9

AC EB F D.

533

534

K¨ oz´ episkolai Matematikai ´ es Fizikai Lapok, 2022/9

(1) ´es EF AR alapj´an k¨ onnyen bel´ athatjuk, hogy ARD a BEC ´es CF D h´ aromsz¨ ogekkel egybev´ ag´ o, egyenl˝o sz´ar´ u der´eksz¨og˝ u h´ aromsz¨og, amelyb˝ ol az is oget z´ ar be. Mivel a BD ´atl´ o is ad´odik, hogy az AR egyenes az AB-vel 45◦ -os sz¨ oget z´ ar be AB-vel, ez´ert 45◦ -os sz¨

hiszen H ´es K a BC = CD = a ´ atm´er˝ oj˝ u k1 , illetve k2 f´elk¨ or¨ ok k¨ oz´eppontjai. Mivel at´ ai HO = KO = a2 , ez´ert az E, F pontok koordin´ Å ã Å ã 3a a a 3a ; ; (1) E , F . 2 2 2 2

BD EF AR.

Szakasz felez˝opontj´anak koordin´at´ ai a szakaszv´egpontok koordin´ at´ ainak sz´ amtani k¨ozepei, ez´ert (1) ´es a D, A pontok koordin´ at´ ainak felhaszn´ al´as´ aval a DE ´es AF szakaszok P , illetve Q felez˝opontjainak koordin´at´ ai: ã Å ã Å a 3a 3a 3a ; ; , Q . (2) P 4 4 4 4

(3)

Az F D = RD ´es CO = AO, illetve (3) szerint BD az EF ´es AR egyenesek k¨oz´epp´arhuzamosa. Ez´ert BD tartalmazza az egyenesek A ´es F pontjait o¨sszek¨ot˝ o szakasz felez˝ opontj´ at, teh´at a Q pontot is. G´ al Andr´ as (Miskolc, F¨ oldes Ferenc Gimn´azium, 10. ´evf.) dolgozata alapj´an V. megold´as. Legyen a n´egyzet oldalainak hossza a. Helyezz¨ uk el a n´egyzetet a der´eksz¨ og˝ u koordin´ a ta-rendszerben u ´ gy, hogy az A pont az orig´o, a B pont  koordin´ at´ ai B a; 0 , ebb˝ ol k¨ ovetkez˝ oen a n´egyzet t¨obbi cs´ ucsa C(a; a) ´es D(0; a).

A n´egyzet AC ´ atl´ oj´anak egyenlete y = x, a BD ´ atl´ o egyenlete y = −x + a. (2) alapj´an egyszer˝ u sz´ am´ıt´ assal bel´ athatjuk, hogy a P pont koordin´ at´ ai kiel´eg´ıtik az y = x, Q koordin´ at´ ai pedig kiel´eg´ıtik az y = −x + a egyenletet, teh´ at P ´es Q val´oban illeszkednek a megfelel˝o ´ atl´ ok egyenes´ere. A pontok az a´tl´ okat alkot´ o szakaszok bels˝ o pontjai, hiszen az AC ´es BC ´ atl´ ok bels˝o pontjainak x; y koordin´ at´ aira egyar´ ant teljes¨ ul, hogy 0 < x < a, 0 < y < a, at igazoltuk. ´es (2) szerint ez a P , Q pontokra is fenn´ all. Ezzel a feladat a´ll´ıt´ as´ (T¨ obb versenyz˝ o dolgozata alapj´ an) ¨ Osszesen 211 dolgozat ´erkezett. 5 pontos 44, 4 pontos 37, 3 pontos 44, 2 pontos 28 dolgozat. 1 pontot 23, 0 pontot 22 versenyz˝ o kapott. Nem versenyszer˝ u: 13 dolgozat. Megjegyz´esek. a) A dolgozatokban sokf´ele kisebb-nagyobb hiba el˝ ofordult a legegyszer˝ ubb el´ır´ ast´ ol, sz´ amol´ asi hib´ at´ ol eg´eszen a bizony´ıtand´ o ´ all´ıt´ as felhaszn´ al´ as´ anak elvi hib´ aj´ aig. b) Hib´ anak sz´ am´ıtott az is, ha a versenyz˝ o helyesen igazolta, hogy a P pont illeszkedik AC-re, le´ırta, hogy a Q pontra a bizony´ıt´ as hasonl´ o, de azt t´enylegesen nem hajtotta v´egre. Az ilyen t´ıpus´ u megold´ asokban el˝ ofordult, hogy a versenyz˝ o utalt r´ a, hogy a feladatbeli DE ´es AF szakaszok, illetve a P ´es Q pont ´ atvihet˝ o egym´ asba forgat´ assal, de nem adta meg a forgat´ as le´ır´ as´ at. c) Sok megold´ as k´esz¨ ult koordin´ ata-geometriai u ´ton, m´egpedig u ´gy, hogy a n´egyzetet der´eksz¨ og˝ u koordin´ ata-rendszerben speci´ alis (p´eld´ aul egys´egnyi) oldalhossz´ us´ aggal ´es helyzetben vizsg´ alt´ ak, de nem utaltak r´ a, hogy ez elegend˝ o az eredeti feladat igazol´ as´ ahoz is.

6. ´ abra

d) T¨ obb olyan megold´ as is sz¨ uletett, ahol a felt¨ olt¨ ott dokumentum mind¨ ossze egy szerkesztett a ´br´ at tartalmazott sz¨ oveges indokl´ as n´elk¨ ul. Az ilyen dolgozatok 0 pontot kaptak.

Legyen a n´egyzet k¨ oz´eppontja O, ennek koordin´ at´ ai a a ; O . 2 2

or¨ ok felez˝opontjai, teh´at BE = CE, illetve Az E, F pontok a k1 , illetve k2 f´elk¨ CF = DF , ez´ert az E ´es F pontok rajta vannak a BC, illetve CD szakaszok felez˝omer˝ oleges´en ´es ´ıgy OE p´ arhuzamos az x-tengellyel, illetve OF p´arhuzamos az y-tengellyel. Ha a BC ´es CD szakaszok felez˝ opontjai H ´es K, akkor HE = KF = K¨ oz´ episkolai Matematikai ´ es Fizikai Lapok, 2022/9

a , 2 535

C. 1731. Az ABCD trap´ez p´ arhuzamos oldalai AB > CD, a trap´ez k¨ oz´epvonala az AC ´ atl´ ot az E, a BD ´ atl´ ot az F pontban metszi. A CD szakasz hossza az AB ´es EF szakaszok hossz´ anak a) sz´ amtani, b) m´ertani k¨ ozepe. AB ar´ any Hat´ arozzuk meg, hogy a k´et eset k¨ oz¨ ul melyikben lesz nagyobb az CD ´ert´eke. Javasolta: B´ır´ o B´ alint (Eger)

536

K¨ oz´ episkolai Matematikai ´ es Fizikai Lapok, 2022/9

Megold´as. Haszn´ aljuk az ´ abra jel¨ ol´eseit. A trap´ez k¨ oz´epvonala P Q, ´ıgy egy-

Megjegyz´es. Sokan pr´ ob´ alkoztak valamilyen k¨ ozepek k¨ oz¨ otti ¨ osszef¨ ugg´est haszn´ alni a megold´ asban, p´eld´ aul kihozt´ ak, hogy az els˝ o esetben

, m´asr´eszt P Q a b. Ekr´eszt P Q = a+b 2 kor P E ´es F Q az ACD, illetve a BCD h´ aromsz¨ og k¨ oz´epvonala, teh´ at

1 AB , = EF CD 1 + AB 2

b PE = FQ = . 2

AB

Ebb˝ ol m´ar fel´ırhatjuk EF hossz´ at: EF = x = P Q − P E − F Q =

b a−b a+b b − − = . 2 2 2 2

Fejezz¨ uk ki a k´et esetben ennek seg´ıts´eg´evel a k´erdezett ar´anyt. a) CD = AB+EF . Teh´ at 2 b=

AB

vagyis CD ´ert´eke ´eppen 1-nek, ´es EF -nek a harmonikus k¨ ozepe. A m´ asodik esetben

AB AB AB pedig CD = EF , ami pedig 1-nek ´es EF -nek a m´ertani k¨ ozepe. Mivel a m´ertani k¨ oz´ep legal´ abb akkora, mint a harmonikus k¨ oz´ep, ez´ert a b) esetben nagyobb a k´erd´eses ar´ any. Mi´ert nem j´ o gondolatmenet ez ´es a hozz´ a hasonl´ oak, ahol a k´et esetben egyegy k¨ oz´ep´ert´eket vesznek, majd a kett˝ ot ¨ osszehasonl´ıtj´ ak? Az´ert, mert val´ oj´ aban nem ugyanannak a k´et sz´ amnak veszik a k¨ ul¨ onb¨ oz˝ o k¨ oz´ep´ert´ekeit. Meg lehet gondolni, p´eld´ aul a fenti megold´ as alapj´ an, hogy a, b ´es x k¨ oz¨ ul b´ armelyik kett˝ o egy´ertelm˝ uen meghat´ arozza a harmadik ´ert´ek´et (vagy nem j¨ ohet l´etre a trap´ez). Vagyis pl. r¨ ogz´ıtett a ´ert´ek eset´en egy ozepe a-nak ´es x-nek. bizonyos x ´ert´ekre lesz b ´eppen a sz´ amtani, illetve m´ertani k¨

a+x a a−b 3a − b = + = , 2 2 4 4

58 dolgozat ´erkezett. 5 pontos 20, 4 pontos 5, 1 pontos 2, 0 pontos 28 dolgozat. Nem versenyszer˝ u: 3 dolgozat.

4b = 3a − b, 3a = 5b, AB a 5 = = . CD b 3 b) CD2 = AB · EF . Vagyis b2 = a ·

Matematika feladatok megold´asa

a2 − ab a−b = , 2 2

B. 5241. Az ABC h´ aromsz¨ ogben ABC > 90◦ , a k¨ or¨ ul´ırt k¨ or k¨ oz´eppontja O. A k¨ or¨ ul´ırt k¨ orh¨ oz C-ben h´ uzott ´erint˝ o az AB egyenest a P pontban, a P -b˝ ol BC-re all´ıtott mer˝ ´ oleges pedig az OC egyenest Q-ban metszi. Igazoljuk, hogy AB mer˝ oleges AQ-ra.

2b2 = a2 − ab, mindk´et oldalt osztva a pozit´ıv b2 ´ert´ek´evel: ab  a 2  a  a2 − 2= 2 − 2 = , b b b b  a 2  a  − 2 = 0. − b b

(4 pont)

Megold´as. Legyen a P -b˝ ol BC-re a´ll´ıtott mer˝ oleges talppontja T . Ekkor BT Q der´eksz¨ og, azaz T rajta van a BQ ´ atm´er˝ oj˝ u Thal´esz-k¨ or¨ on. A feladat ´all´ıt´ asa, mely szerint BAQ der´eksz¨ og, azzal ekvivalens a Thal´esz-t´etel alapj´ an, hogy A is rajta van a BQ ´ atm´er˝ oj˝ u k¨ or¨ on, vagyis az el˝ obbiek alapj´an a BT Q k¨ or¨ on. A feladat ´all´ıt´ asa teh´ at ekvivalens azzal, hogy A rajta van a BT Q k¨ or¨ on, azaz BT QA h´ urn´egysz¨ og, ezt fogjuk a tov´ abbiakban megmutatni.

Ez ab -re n´ezve egy m´ asodfok´ u egyenlet, ´ıgy √ a 1± 1+8 1±3 = = , b 2 2 ami vagy 2, vagy −1. Ut´obbi nyilv´ an nem lehets´eges, vagyis ebben az esetben AB a = = 2. CD b

AB Teh´ at a b) esetben nagyobb az CD ar´ any ´ert´eke. Hochenburger Zo´ ard (Budapest, V´arosmajori Gimn., 11. ´evf.)

K¨ oz´ episkolai Matematikai ´ es Fizikai Lapok, 2022/9

537

Javasolta: Nagy Zolt´ an L´ or´ ant (Budapest)

Megjegyezz¨ uk, hogy P C mer˝ oleges OC ≡ QC-re, hiszen egy adott pontba h´ u538

K¨ oz´ episkolai Matematikai ´ es Fizikai Lapok, 2022/9

zott ´erint˝o mer˝ oleges az adott pontba mutat´o sug´ arra, azaz a P QC h´ aromsz¨og der´eksz¨ og˝ u. Ebben a h´ aromsz¨ ogben BC ≡ CT ⊥P Q miatt CT az a´tfog´ ohoz tartoz´o alva, hogy magass´ ag, ´ıgy fel´ırva a befog´ot´etelt: P T · P Q = P C 2 . Most felhaszn´ a P pont ABC k¨ orre vonatkoz´ o hatv´ anya a´lland´ o (vagy a szel˝o- ´es ´erint˝oszakaszok obbi kett˝ ot ¨osszevetve t´etele alapj´ an): P C 2 = P B · P A. Az el˝ P T · P Q = P B · P A, ami a szel˝ ok t´etel´enek megford´ıt´asa miatt ´eppen azt jelenti, hogy T , Q, B, A konciklikus, AQ val´ oban mer˝ oleges AB-re. Diszkusszi´o. A bizony´ıt´ as sor´ an sehol sem haszn´ altunk sz¨ogsz´ am´ıt´asokat, csup´ an der´eksz¨ ogek szerepeltek a megold´ asban, ´es ilyenkor nyilv´an nem sz´am´ıt (ir´ any´ıtatlan sz¨ ogekkel sem), hogy az adott egyenes melyik f´elegyenes´en van a sz¨og harmadik cs´ ucsa, mindenk´eppen der´eksz¨ oget kapunk. A k¨orre fel´ırt hatv´ anyok is igazak el˝ojeles szakaszok n´elk¨ ul is, ugyanis P -b˝ ol h´ uzhat´ o ´erint˝o a k¨orh¨oz, ´es ´ıgy k¨ uls˝ o pont, azaz a hatv´ any sor´an fel´ırt t´ avols´ agok mindig azonos ir´ any´ uak, ´es ´ıgy nincs sz¨ uks´eg el˝ ojeles szakaszokra. Az egyetlen eset, amikor a bizony´ıt´asunk nem mondhat´ o el az euklideszi s´ıkon (b´ar a projekt´ıv s´ıkon, n´emi kieg´esz´ıt´essel elmondhat´ o lenne) az, amikor valamelyik metsz´espont nem j¨on l´etre, azaz a k´et megrajzolt egyenes p´ arhuzamos. Ez a k´et egyenes nem lehet OC ´es a P -b˝ ol BC-re a´ll´ıtott mer˝oleges, ugyanis ez esetben OC mer˝ oleges lenne BC-re, de ekkor BC ´erinten´e a k¨or¨ ul´ırt k¨ort, ami lehetetlen, hiszen akkor BC-nek csak egy k¨ oz¨ os pontja lehetne a k¨or¨ ul´ırt k¨orrel, azaz BC pontt´ a fajulna. Marad teh´at az az eset, hogy AB ´es a C-ben h´ uzott ur mindk´et ´erint˝o p´ arhuzamosak. Egy h´ urral p´ arhuzamosan k´et ´erint˝o h´ uzhat´ o, a h´ ´erint´esi ponttal egyenl˝o sz´ar´ u h´ aromsz¨ oget alkot. Teh´ at ekkor az ABC h´aromsz¨og egyenl˝ o sz´ ar´ u, melynek alapja AB, ´es ´ıgy ABC < 90◦ , ami ellentmond a feladat felt´eteleinek. ´Igy a bizony´ıt´ as minden tompasz¨ og˝ u h´aromsz¨ogre elmondhat´ o az euklideszi s´ıkon, a megold´ ast ezzel befejezt¨ uk.

1. eset: B s´ arga p´ ol´ot visel. Mivel ˝o is vegyes sz´ın˝ u szomsz´edokkal rendelkezik, ez´ert C sz´ıne k´ek, ´ıgy D sz´ıne k´ek (1. ´ abra), E sz´ıne s´ arga, F sz´ıne s´ arga, ´es ´ıgy tov´ abb: ss ut´ an mindig kk, azut´ an pedig ss k¨ ovetkezik. 1. ´ abra 2. a ´bra 2. eset: B k´ek p´ ol´ot visel. Ekkor C-nek is k´eket kell viselnie ahhoz, hogy B-re teljes¨ ulj¨ on a felt´etel (2. ´ abra). A mellette u o ember pedig ¨l˝ s´arga p´ol´ot kell, hogy viseljen. Itt is felv´altva k´et k´ek ´es k´ek s´ arga p´ ol´os embernek kell k¨ovetnie egym´ast ahhoz, hogy sorra mindegyik emberr˝ ol elmondhat´ o legyen az, hogy mellett¨ uk k¨ ul¨ onb¨ oz˝ o sz´ın˝ u p´ ol´os emberek u ¨lnek. A m´asik szomsz´edja (N ) az els˝ o esetben k´ek, a m´asodikban pedig s´ arga p´ ol´ot kell viseljen. Ezek alapj´an A, B, . . . , L, M, N sz´ıne rendre sskk | sskk | sskk | sk, vagy pedig s | kk | sskk | sskk | sss. Mindk´et esetben l´ athat´ o, hogy M szomsz´edjainak, L-nek ´es N -nek a p´ ol´osz´ıne megegyezik, ami ellentmond´ as. V´eg¨ ul egy konkr´et elhelyez´est mutatunk arra, hogy a k´erd´eses sz´ am lehet 12:

Varga Boldizs´ ar (Budapest, B´ek´ asmegyeri Veres P´eter Gimn., 9. ´evf.) megold´ asa ¨ Osszesen 56 dolgozat ´erkezett. 4 pontot kapott 51, 3 pontot 2 tanul´ o. 0 pontos 3 versenyz˝ o dolgozata.

Geretovszky M´ arton L´ aszl´ o (Szegedi Radn´ oti M. K´ıs. Gimn., 10. ´evf.) dolgozata alapj´an 81 dolgozat ´erkezett. 3 pontot kapott 67, 2 pontot 6 tanul´ o, 1 pontos 5, 0 pontos 2 versenyz˝ o dolgozata. Nem versenyszer˝ u 1 dolgozat.

B. 5246. 14 ember u or¨ ul, mindenki k´ek vagy s´ arga p´ ol´ oban. Legfel¨l egy asztal k¨ jebb h´ any emberre teljes¨ ulhet, hogy a k´et szomsz´edja k¨ ul¨ onb¨ oz˝ o sz´ın˝ u p´ ol´ oban van? (3 pont)

Matematikai k´epz´esek az ELTE TTK-n

Megold´as. A k¨ ul¨ onb¨ oz˝ o sz´ın˝ u szomsz´edokkal rendelkez˝o emberek sz´ ama legfeljebb 14. El˝osz¨ or azt mutatjuk meg, hogy sem 14, sem pedig 13 nem lehet. Tegy¨ uk fel, hogy a k´erd´eses sz´ am legal´abb 13. Ekkor legfeljebb egy olyan ember van, akire nem teljes¨ ul a felt´etel, teh´ at a 13 ember valakit˝ ol kezdve sorban egym´as mellett u uk a sor egyik sz´els˝ o tagj´at (A). Legyen a p´ol´oja s´arga sz´ın˝ u. Ekkor ¨l. Vegy¨ a mellette u o, a felt´eteleket teljes´ıt˝ o ember (B) s´ arga ´es k´ek sz´ın˝ u p´ol´ot is viselhet. ¨l˝ Eszerint k´et esetet vizsg´alunk. K¨ oz´ episkolai Matematikai ´ es Fizikai Lapok, 2022/9

539

Kedves leend˝ o Egyetemista! A K¨ oMaL olvas´ ojak´ent bizony´ ara sz´ıvesen foglalkozol matematik´ aval, ´es felmer¨ ulhetett m´ar Benned az a gondolat, hogy ´eletp´ aly´ adul ennek a sz´ep tudom´ anynak a m˝ uvel´es´et v´ alasztod, illetve szeretn´el megismerkedni alkalmaz´ asaival a m˝ uszaki, gazdas´ agi ´es p´enz¨ ugyi ´elet k¨ ul¨ onb¨ oz˝ o ter¨ uletein. 540

K¨ oz´ episkolai Matematikai ´ es Fizikai Lapok, 2022/9

Az alkalmazott matematika ma m´ ar az ´elet szinte minden ter¨ ulet´en n´elk¨ ul¨ozhetetlen, ´es az ilyen k´epzetts´eg˝ u munkaer˝ o ir´ ant egyre n¨ovekszik az ig´eny. A Fortune magazin cikke szerint a legjelent˝osebb v´altoz´ as az u ¨zleti ´eletben az ipari forradalom ota a matematikai algoritmusok t´erh´ ´ od´ıt´ asa (http://fortune.com/2015/01/22/ the-algorithmic-ceo/). Egy amerikai felm´er´es ´evr˝ ol ´evre a legjobb foglalkoz´ asok k¨ oz¨ ott tartja sz´ amon a matematikust ´es a szint´en matematikai el˝ok´epzetts´eget ig´enyl˝ o adattud´ ost, aktu´ ariust ´es statisztikust (https://www.careercast.com/ jobs-rated/best-jobs-2021). Mindez Magyarorsz´agra is igaz, az ELTE-n v´egzett matematikusokat nemcsak a kutat´oint´ezetek, egyetemek v´ arj´ ak, hanem sz´ amos c´eg is, igen j´ o fizet´essel. Esetleg m´eg nem d¨ ont¨ ott´el, de legink´ abb matematik´ ab´ol folytatn´al fels˝ofok´ u tanulm´ anyokat? Minderre kit˝ un˝ o lehet˝os´eg ny´ılik az orsz´ag egyik legnagyobb m´ ulotv¨ os Lor´ and Tudom´anyegyetem Term´eszettudom´anyi Kar´an, t´ u egyetem´en, az E¨ ahol vil´agh´ır˝ u professzorokt´ol ´es lelkes, k¨ ozvetlen fiatal oktat´ okt´ ol tanulhatsz. Pezsg˝o di´ak´elet v´ ar r´ ad az ELTE korszer˝ u sz´ am´ıt´ og´epparkkal felszerelt, a K¨oMaL szerkeszt˝ os´eg´enek is otthont ad´o modern l´ agym´anyosi ´ep¨ uletegy¨ uttes´eben. A bolognai k´epz´esi rendszerbe illeszkedik BSc k´epes´ıt´est ny´ ujt´ o h´arom´eves matematikai alapk´epz´es¨ unk. Itt az els˝ o ´evben hallgat´oi ´es oktat´ oi mentorok biztos´ıtj´ ak, hogy mindenki be tudjon illeszkedni ´es tal´ aljon el˝ oismereteinek, k´epess´egeinek ´es tanul´ asi sebess´eg´enek megfelel˝o neh´ezs´eg˝ u feladatokat. Az els˝o ´ev v´eg´en d¨onthetsz arr´ ol, hogy milyen t´em´akkal szeretn´el a tov´ abbiakban behat´obban foglalkozni. A k´ın´ alat sz´eles: aki szeretne, az elm´elyedhet az elm´eleti matematika k´erd´eseiben, hiszen szinte minden fontos ter¨ uletr˝ ol hirdet¨ unk kurzusokat. Ezek ´ep´ıtenek a magyar matematikai kutat´asok m´elt´ an vil´agh´ır˝ u hagyom´ anyaira, ugyanakkor szil´ ard alapokat ny´ ujtanak a modern matematika m˝ uvel´es´ehez, j´ ol felk´esz´ıtve hallgat´ oinkat a leend˝ o kutat´oi munk´ara. Akit viszont az alkalmaz´ asok ´erdekelnek, megteheti, hogy az alapok elsaj´at´ıt´asa ut´ an olyan modern t´em´ akkal is foglalkozzon, mint az adattudom´ any vagy a mesters´eges intelligencia matematikai k´erd´esei. Azoknak is aj´anljuk a matematika alapk´epz´esi szakot, akik ismereteiket k´es˝ obb ink´ abb a matematik´an k´ıv¨ ul szeretn´ek majd gy¨ um¨ olcs¨ oztetni. Itt szerzett tud´ asukat hasznos´ıthatj´ak p´eld´ aul gazdas´ agi ter¨ uleten, m´edi´aban, a matematika n´epszer˝ us´ıt´es´eben, a k¨ozm˝ uvel˝ od´esben – ´es a megszerzett matematikai gondolkod´ asm´ od mindv´egig seg´ıteni fogja ˝oket a munk´ajukban. ol tov´ abbi r´eszletek a http://www.math.elte. A k´epz´es egy´eb vonatkoz´ asair´ hu/ honlapon a K´epz´esek men¨ upont alatt tal´ alhat´ ok. Aj´ anljuk a k¨oz´episkol´asoknak sz´ ol´o oldalainkat is, ahol v´egzett di´ akjainkkal k´esz¨ ult interj´ uk is l´athat´ ok. A legkiemelked˝obb hallgat´ ok az egyetemi oktat´omunk´aba is bekapcsol´ odhatnak, ´es j´ o es´ellyel p´ aly´ azhatnak ¨ oszt¨ ond´ıjakra, k¨ ulf¨oldi r´eszk´epz´esre (pl. az Eras´ mus+ program keret´eben). Az UNKP oszt¨ ond´ıjprogramja m´ar a leend˝o els˝o´eve¨ seknek is el´erhet˝ o! R´eszletes t´ aj´ekoztat´ o: http://csikvarip.web.elte.hu/diak_ kutatas.html. Az alapk´epz´est tov´abbi k´et´eves szakasz k¨ ovet(het)i (mesterk´epz´es vagy r¨oviden MSc), egyetem¨ unk¨ on a Matematikai Int´ezet gondoz´ as´aban matematikus, alugyi matematika mesterszakok kalmazott matematikus, valamint biztos´ıt´ asi ´es p´enz¨ indulnak. BSc-t v´egzett hallgat´ oink term´eszetesen m´as (bel- ´es k¨ ulf¨oldi) oktat´ asi int´ezm´eny programjain is folytathatj´ ak tanulm´anyaikat. A mesterszakot v´egzettek K¨ oz´ episkolai Matematikai ´ es Fizikai Lapok, 2022/9

541

k¨oz¨ ul a legkiv´ al´obbak sz´ am´ara biztos´ıtjuk a doktori fokozat megszerz´es´enek lehet˝os´eg´et (PhD-k´epz´es). Egyetem¨ unk¨ on gondosan a´polt hagyom´any, hogy a r´ atermett, tehets´eges di´ akok neves professzorok vezet´es´evel bekapcsol´ odnak a tudom´ anyos kutat´ asba. A legkiv´al´obb hallgat´ ok matematikai versenyeken is sikerrel szerepelnek, p´eld´ aul az Egyetemi Hallgat´ ok Nemzetk¨ ozi Matematikaverseny´en az elm´ ult t´ız ´evben k´etszer is az ELTE csapata v´egzett az ´elen t¨ obb, mint 70 egyetem csapat´ anak verseny´eben – olyan nagyh´ır˝ u egyetemeket is megel˝ozve, mint a Yale, a Princeton vagy a Moszkvai ´ Allami Egyetem.

Matematikatan´ar-k´epz´es az ELTE TTK-n Az ELTE Term´eszettudom´ anyi Kar´ an sok ´evtizedes m´ ultra tekint vissza a matematika szakos tan´ ark´epz´es. Az ´altal´ anos ´es k¨ oz´episkol´ak r´esz´er˝ ol mindig jelent˝os ig´eny mutatkozott a n´ alunk v´egzett matematikatan´ arok ir´ant, akik k¨ oz¨ ul sokan k¨ ulf¨old¨on is sikeres oktat´oi p´ aly´ at futottak be. A matematika szakos tan´ ari p´ aly´ at els˝ osorban azoknak a k¨ oz´episkol´ as di´akoknak aj´ anljuk, akik sz´ am´ara ¨ or¨ omet jelent ´erdekes matematikai feladatokon gondolkodni, ´es j´o ´erz´est okoz a megold´ asokra m´ asokat is r´ avezetni, m´asokkal is megosztani azt az ´elvezetet, amit a matematika megismer´ese jelent. A tan´ark´epz´es osztatlan form´aban zajlik. A tan´ark´epz´esre val´o jelentkez´es sor´ an a leend˝o hallgat´ oknak egy szakp´ art kell megjel¨ olni. Az ELTE-n a matematika szak mell´e term´eszettudom´ anyos szakokon ´es az informatik´an k´ıv¨ ul v´ alasztani lehet aul a magyar, a t¨ ort´enelem vagy a nyelvszakok) k¨ oz¨ ul is. a b¨olcs´esz szakok (p´eld´ A szakt´ argyi tan´ıt´ asi gyakorlatok teljes´ıt´es´ere az ELTE hallgat´ oinak a legjobb budapesti iskol´ akban, kiv´ al´o vezet˝ otan´ arok ir´any´ıt´ asa mellett ny´ılik lehet˝os´eg¨ uk. B´atran a´ll´ıthatjuk teh´ at, hogy a K¨ oMaL minden olvas´oj´anak testhez´all´ o k´epz´est tudunk ny´ ujtani az ELTE Matematikai Int´ezet´eben. Az ELTE TTK novemberi ny´ılt napja itt visszan´ezhet˝ o: https://ttk.elte.hu/nyiltnap2023, janu´arban pedig online ny´ılt napon v´arjuk az ´erdekl˝ od˝ oket.

A K pontversenyben kit˝ uz¨ott gyakorlatok ABACUS-szal k¨oz¨os pontverseny 9. oszt´alyosoknak (744–748.) K. 744. Ha k´et szendvicset ´es egy u ot veszek, akkor az 1000 Ft-omb´ol ugyan¨d´ıt˝ annyi marad meg, mint amennyi hi´ anyzik az 1000 Ft-omhoz, ha h´arom szendvicset ´es egy u ot veszek. K´et szendvics ´es k´et u o´ ara 1100 Ft. ¨d´ıt˝ ¨d´ıt˝ Mennyibe ker¨ ul egy szendvics ´es mennyibe ker¨ ul egy u o? ¨d´ıt˝ K. 745. Egy t´ arsasj´ at´ekban a j´at´ekosok pontokat gy˝ ujtenek. A j´ at´ekosok sorban egym´as ut´an k¨ ovetkeznek. Amikor egy j´at´ekosra sor ker¨ ul, akkor ˝o (szerencs´et˝ ol f¨ ugg˝oen) ak´ armennyi, de nemnegat´ıv eg´esz sz´ am´ u pontot gy˝ ujthet (´ıgy ak´ ar 542

K¨ oz´ episkolai Matematikai ´ es Fizikai Lapok, 2022/9

0 pontot is kaphat). A j´ at´ekos a´ltal a j´ at´ek sor´ an szerzett pontok ¨osszead´ odnak. Ha az o at´ekos a´ltal szerzett pontok ¨ osszege el´eri az 1000-et, akkor a j´ at´ek ¨sszes j´ azonnal v´eget ´er (´ıgy a befejez˝ o j´ at´ekos az utols´ o k¨ or´eben csak annyi pontot tud szerezni, amennyivel az ¨ osszeg 1000 lesz). A j´ at´ekot az nyeri, akinek a legt¨obb pontja van, holtverseny eset´en az nyer, aki el˝ osz¨ or ´erte el az adott pontsz´ amot; a m´ asodik legt¨ obb pontot gy˝ ujt˝ o j´at´ekos ´er el m´ asodik helyez´est stb.

A C pontversenyben kit˝ uz¨ott gyakorlatok (747–748., 1743–1747.)

A j´at´ek ´ all´ asa jelenleg: Kati 314 pont, Sanyi 207 pont, Jancsi 58 pont, Gizi 31 pont, J´ ozsi 0 pont. a) Ha most Kati k¨ ovetkezik, minim´ alisan h´ any pontot kell szereznie ebben a k¨ orben, hogy biztosan legal´ abb m´asodik helyez´est ´erjen el? b) Ha most Sanyi k¨ ovetkezik, minim´ alisan h´ any pontot kell gy˝ ujtenie ebben a k¨ orben, hogy biztosan legal´ abb m´asodik helyez´est ´erjen el? c) Ha most J´ ozsi k¨ ovetkezik, minim´ alisan h´ any pontot kell gy˝ ujtenie ebben orben, hogy biztosan legal´ abb m´asodik helyez´est ´erjen el? a k¨ K. 746. Egy kis orsz´ agban bevezett´ek, hogy a f¨oldg´azfogyaszt´ as´ert az al´ abbiak szerint kell fizetni: Az u ´j elsz´ amol´as bevezet´es´et k¨ ovet˝o els˝o ´evben az els˝ o 1700 m3 g´ az a´ra 100 pet´ak/m3 , a tov´ abbi fogyaszt´ as ´ ara 750 pet´ak/m3 . A kedvezm´enyesen v´ as´arolhat´ o mennyis´eget az orsz´agos ´eves a´tlagfogyaszt´as alapj´ an ´allap´ıtj´ ak meg minden ´evben, az el˝ oz˝ o ´evi fogyaszt´ asi adatok alapj´ an. H´etk¨oznapi J´ anos ebben az orsz´agban ´el, ´es egy ´evig haszn´alja a g´ azt ezekkel a felt´etelekkel. Tekintettel azonban a t´ ul magas fizetend˝ o ¨ osszegre elhat´ arozza, hogy takar´ekoskodni fog, ´es kevesebb g´ azt haszn´ al el. Siker¨ ul is az ´eves g´ azfogyaszt´ as´at 10%-kal cs¨okkentenie a k¨ ovetkez˝ o ´evre, azonban, mivel mindenki hasonl´ oan gondolkodott, az ´eves g´ azfogyaszt´ as orsz´agos a´tlaga 15%-kal cs¨ okkent. H´etk¨ oznapi J´ anos azt vette ´eszre, hogy a m´asodik ´evben hi´aba fogyasztott kevesebb g´ azt, m´egis t¨obbet kellett fizetnie (az eg´esz ´evet tekintve), mint az els˝ o ´evben. Mennyi lehetett J´ anosunk els˝ o ´eves g´ azfogyaszt´ asa? K/C. 747. Egy negyvensz¨ oget valamelyik a´tl´ oja k´et olyan soksz¨ogre bontja, osszesen 298-cal kevesebb a´tl´ oja van, mint a negyvensz¨ognek. H´ any melyeknek ¨ oldal´ uak ezek a soksz¨ogek?

Feladatok 10. ´evfolyamig K/C. 747. A sz¨ oveg´et l´ asd a K feladatokn´ al. K/C. 748. A sz¨ oveg´et l´ asd a K feladatokn´ al. Feladatok mindenkinek C. 1743. Mutassuk meg, hogy h´et term´eszetes sz´ am k¨ oz¨ ott, amelyek egy 30 k¨ ul¨onbs´eg˝ u sz´amtani sorozat egym´ as ut´ani tagjai, pontosan egy 7-tel oszthat´o sz´ am van. Javasolta: B´ır´ o B´ alint (Eger) C. 1744. Az ABC h´aromsz¨ ogben CAB = 45◦ ´es ABC = 60◦ . Az AB szakasz egy pontja D. A CAD h´aromsz¨ og k¨ or¨ ul´ırt k¨ ore a´thalad az ABC h´ aromsz¨ og AD ar´ any pontos ´ert´ek´et u ´gy, hogy a megmagass´ agpontj´an. Hat´ arozzuk meg az BD old´ as sor´ an nem haszn´alunk sz¨ ogf¨ uggv´enyeket. Javasolta: B´ır´ o B´ alint (Eger) y−5

amok. C. 1745. Oldjuk meg az x2 + 8x − y = y+6 egyenletet, ha x, y eg´esz sz´ Javasolta: B´ır´ o B´ alint (Eger) Feladatok 11. ´evfolyamt´ol C. 1746. Az ABCD n´egyzet AB oldal´ at az A ponton t´ ul meghosszabb´ıtjuk az AE = 2 szakasszal, a B ponton t´ ul pedig a BF = 3 szakasszal. Az ED ´es F C oget z´ arnak be. Hat´ arozzuk meg a n´egyzet oldal´ anak lehets´eges egyenesek 45◦ -os sz¨ ´ert´ekeit. Javasolta: N´emeth L´ aszl´ o (Fony´ od)

K/C. 748. Egy k¨ orvonal ment´en le´ırjuk az eg´esz sz´ amokat 1-t˝ ol 100-ig. El˝ osz¨or minden p´ aros sz´amot ¨osszek¨ ot¨ unk a n´ ala kisebb p´ aratlan sz´ amokkal, majd minden p´ aratlan sz´ amot ¨ osszek¨ ot¨ unk minden n´ ala kisebb p´ aros sz´ammal. H´ any vonalat h´ uztunk be?

C. 1747. Legyen az n  3 pozit´ıv eg´esz sz´ am, a 10n − 4! sz´amban a sz´amjegyek 10n+1 −7 sz´ amban a sz´amjegyek ¨ osszege? ¨osszege k. Mennyi ekkor a 3





Beku ¨ld´esi hat´arid˝o: 2023. janu´ar 10. Elektronikus munkafu ¨zet: https://www.komal.hu/munkafuzet

Beku ¨ld´esi hat´arid˝o: 2023. janu´ar 10. Elektronikus munkafu ¨zet: https://www.komal.hu/munkafuzet





K¨ oz´ episkolai Matematikai ´ es Fizikai Lapok, 2022/9

543

Javasolta: Szalai M´ at´e (Szeged)

544

K¨ oz´ episkolai Matematikai ´ es Fizikai Lapok, 2022/9

B. 5284. Legyen n > 2. Alad´ar kiv´alasztotta a 2n cs´ ucs´ u teljes gr´ af egy ´el´et. Paula egy forint´ert r´ ak´erdezhet, hogy egy ´altala megadott teljes p´ aros´ıt´asban benne van-e a kiv´ alasztott ´el. Legal´abb h´any forint lapul Paula zseb´eben, ha u ¨gyes k´erd´esekkel biztosan ki tudja tal´alni, hogy melyik ´el lett kiv´ alasztva? (6 pont) Javasolta: Pach P´eter P´ al (Budapest)

A B pontversenyben kit˝ u z¨ ott feladatok (5278–5285.)

B. 5278. Nevesincs iskol´ aban a v´egz˝ os re´alosok n´egy csoportot alkotnak, vannak matekosok, fizik´ asok, k´emi´ asok ´es bioszosok. Egy napon a menz´ an mindannyian egy nagy, kerek asztaln´al u utt, mindenkivel szemben u ¨lnek egy¨ ¨l valaki ´es mindenkinek van bal oldali ´es jobb oldali szomsz´edja. B´ arkit v´alasztunk ki, k´et szomsz´edj´aval ´es a vele szemben u ovel csupa k¨ ul¨ onb¨ oz˝ o csoport tagjai. H´ anyan lehetnek a v´egz˝ os ¨l˝ re´ alosok, ha 20-n´ al kevesebben vannak? (3 pont)

B. 5285. A hegyessz¨ og˝ u ABC h´ aromsz¨ ogben AB = AC. A h´ aromsz¨ og k¨ or´e ´ırt k¨or¨on u ´gy mozognak az A , B  ´es C  pontok, hogy az A B  C  h´ aromsz¨ og mindig egybev´ ag´o ´es azonos ir´any´ıt´ as´ u az ABC h´ aromsz¨ oggel. Legyen a BB  ´es CC  egyenesek metsz´espontja P . Mutassuk meg, hogy az A P egyenesek egy r¨ ogz´ıtett ponton mennek ´at. (6 pont)

Beku ¨ld´esi hat´arid˝o: 2023. janu´ar 10. Elektronikus munkafu ¨zet: https://www.komal.hu/munkafuzet

Javasolta: Kozma Katalin Abig´el (Gy˝or)

B. 5279. Egy der´eksz¨ og˝ u sz¨ ogtartom´anyba k´et k¨ort ´ırtunk. Az egyik k¨or az egyik sz¨ ogsz´ arat az A pontban, a m´ asik k¨ or a m´ asik sz¨ogsz´ arat a B pontban ´erinti. A k´et k¨ or egym´ast is ´erinti a C pontban. Hat´ arozzuk meg az ACB nagys´ ag´ at. (3 pont) Javasolta: B´ır´ o B´ alint (Eger) B. 5280. Legyenek a > 2, b ´es c val´ os sz´amok. Tekints¨ uk a k¨ovetkez˝ o h´arom all´ıt´ ´ ast. os megold´asa. (1) Az ax2 + bx + c = 0 egyenletnek nincs val´ (2) Az (a − 1)x2 + (b − 1)x + (c − 1) = 0 egyenletnek 1 val´os megold´asa van. (3) Az (a − 2)x2 + (b − 2)x + (c − 2) = 0 egyenletnek 2 val´os megold´asa van. a) Ha tudjuk, hogy az (1)-es ´es a (2)-es a´ll´ıt´ as igaz, akkor k¨ovetkeztethet¨ unk-e arra, hogy a (3)-as ´all´ıt´ as is igaz? b) Ha tudjuk, hogy a (2)-es ´es a (3)-as a´ll´ıt´ as igaz, akkor k¨ovetkeztethet¨ unk-e arra, hogy az (1)-es ´all´ıt´ as is igaz? (4 pont) Javasolta: Hujter B´ alint (Budapest) B. 5281. Bizony´ıtsuk be, hogy minden d > 1 pozit´ıv eg´esz sz´ amhoz tal´alhat´ o egy olyan pozit´ıv eg´esz sz´ am, amelynek oszt´ oi k¨ oz¨ott pontosan ugyanannyi d-vel oszthat´ o van, mint d-vel nem oszthat´o. (5 pont)

Javasolta: Hujter B´ alint (Budapest)

B. 5282. Az ABC hegyessz¨ og˝ u h´ aromsz¨ ogben az A-b´ ol indul´o magass´ ag talppontja T , a T pont mer˝ oleges vet¨ ulete az AB oldalon D, az AC oldalon pedig E. Legyen F a BC oldal ´es az ABE k¨ or m´asodik, B-t˝ol k¨ ul¨onb¨oz˝ o metsz´espontja, ´es hasonl´ oan, legyen G a BC oldal ´es az ACD k¨ or m´asodik, C-t˝ ol k¨ ul¨onb¨oz˝ o metsz´espontja. Mutassuk meg, hogy T F = T G. (5 pont) Javasolta: K´ os G´eza (Budapest)

Az A pontversenyben kit˝ uz¨ott nehezebb feladatok (839–841.) A. 839. Adott egy v´eges, egyszer˝ u, ir´ any´ıtatlan gr´ af. Anna minden ´elre pozit´ıv val´os sz´amokat ´ır u ´gy, hogy b´ armely cs´ ucsra a cs´ ucsba befut´ o ´elekre ´ırt sz´ amok azs szeretn´e u ´gy megsz´ amozni a cs´ ucsokat nemnegat´ıv ¨osszege kisebb egyn´el. Bal´ val´os sz´amokkal, hogy ha tetsz˝oleges v cs´ ucsra a v0 sz´ amot ´ırta, ´es a cs´ ucsb´ ol amokat ´ırta, tov´ abb´ a ezen ´elek kiindul´ o ´elekre Anna rendre az e1 , e2 , . . . , ek sz´ k  m´asik v´egein rendre a v1 , v2 , . . . , vk sz´ amok szerepelnek, akkor v0 = ei vi + 2022 i=1

teljes¨ ulj¨on. Mutassuk meg, hogy Bal´azs mindig meg tudja ´ıgy sz´ amozni a cs´ ucsokat f¨ uggetlen¨ ul a gr´ aft´ ol ´es az Anna ´altal megadott sz´ amoz´ ast´ ol. Javasolta: Varga Boldizs´ ar (Ver˝ oce)

A. 840. Az ABC h´ aromsz¨ og be´ırt k¨ ore az oldalakat az X, Y ´es Z pontban ´erinti. Az XY Z h´aromsz¨ ogben az X ´es az Y cs´ ucsb´ ol indul´o magass´ agok talppontjai X  ´es Y  . Az X  Y  egyenes az ABC h´ aromsz¨ og k¨ or¨ ul´ırt k¨ or´et a P ´es a Q pontban metszi. Bizony´ıtand´ o, hogy X, Y , P ´es Q egy k¨ orre esnek. Javasolta: Simon L´ aszl´ o (Budapest) A. 841. Oldjuk meg a 2a + pb = np−1 egyenletet a nemnegat´ıv eg´esz sz´ amok halmaz´ an, ahol p pr´ımsz´ am. Javasolta: Weisz M´ at´e (Cambridge)

B. 5283. Az N konvex n´egysz¨ og tartalmaz egy r sugar´ u k¨orlapot. Mutassuk meg, hogy N ker¨ ulete legal´ abb 8r. (4 pont) Javasolta: V´ıgh Viktor (S´ andorfalva) K¨ oz´ episkolai Matematikai ´ es Fizikai Lapok, 2022/9

545

Javasolta: K´ os G´eza (Budapest)

Beku ¨ld´esi hat´arid˝o: 2023. janu´ar 10. Elektronikus munkafu ¨zet: https://www.komal.hu/munkafuzet

546

K¨ oz´ episkolai Matematikai ´ es Fizikai Lapok, 2022/9

5. Hat´ arozzuk meg a Valasz munkalap B7:G7 tartom´ any´ aban azt, hogy a teljes id˝ oszakra az eg´esz orsz´agot tekintve hogyan ´es mennyivel v´ altozott az u ´thossz, mind k¨oz-, mind vas´ uton. A B7 ´es E7 cell´ akban e h´ arom sz¨ oveg valamelyike szerepelhet: n˝ ott”, nem v´altozott” vagy cs¨ okkent”. ” ” ” 6. A B10 ´es E10 cell´ akba azon megy´ek neve ker¨ ulj¨ on, amelyekn´el 2012-ben a legnagyobb volt a n´egyetkilom´eterenk´enti u ´thossz, mind k¨ oz-, mind vas´ uton.

Informatik´ab´ ol kit˝ u zo ¨tt feladatok

I. 577. Egyenletesnek nevezz¨ uk azt az N jegy˝ u sz´ amot, amelyben az egyes sz´ amjegyek ´ert´eke az el˝ oz˝ o helyi´ert´eken a´ll´o sz´ amjegyt˝ ol legfeljebb 1-gyel t´er el. ´Irjunk programot, amely egy beolvasott N -jegy˝ u nem egyenletes sz´amhoz meghat´ arozza a n´ ala kisebb legnagyobb ´es a n´ ala nagyobb legkisebb egyenletes sz´ amot. A program az N -jegy˝ u (2  N  1 000 000) eg´esz sz´ amot a standard bemenetr˝ ol olvassa be. A standard kimenet els˝ o sor´ aban a beolvasott sz´ amn´ al kisebb legnagyobb egyenletes sz´ amot, a m´ asodik sor´ aban pedig a n´ala nagyobb legkisebb egyenletes sz´ amot ´ırjuk ki. P´elda bemenet 8778349

Kimenet 8777899 8778765

7. A B12 cell´ aba sz´ amoljuk ki haz´ ank ter¨ ulet´et az adatok alapj´ an. 8. AZ A16, A17, . . . cell´ akba ker¨ ulj¨ on azon megy´ek neve, amelyekben az id˝ oszak v´eg´en, 2021-ben nem volt nagyobb a k¨ oz´ uti ´es vas´ uti u ´tvonalhossz ¨ osszege, mint 2007-ben. any cell´ai k¨ oz¨ ul csak 9. Felt´eteles form´ az´ assal oldjuk meg, hogy az A16:A34 tartom´ azok kapjanak a minta szerinti keretet ´es t´ onust, amelyek nem u ¨resek. 10. Egy u ´j munkalapra k´esz´ıts¨ unk a minta szerint diagramot B´ek´es, Csongr´ ad– Csan´ ad ´es Fej´er megye k¨ oz´ uti hossz´ anak alakul´as´ ar´ ol. Az adatok forr´ asa: a KSH https://www.ksh.hu/stadat_files/sza/hu/sza0041.html; https://www.ksh.hu/stadat_files/sza/hu/sza0039.html ´es https://www.ksh.hu/stadat_files/fol/hu/fol0006.html oldalai. Mint´ ak:

´ ekel´es: a megold´ Ert´ as l´enyeg´et le´ır´ o dokument´ aci´ o 1 pontot ´er. 5 pont kaphat´ o arra a programra, amely a 2  N  1 000 bemenetekre helyes kimenetet ad. Tov´ abbi 4 pont kaphat´ o a 2  N  1 000 000 bemenetekre 1 m´ asodperc fut´ asid˝o alatt helyes kimenetet ad´o programokra. Bek¨ uldend˝o egy t¨om¨ or´ıtett i577.zip ´ allom´anyban a program forr´ ask´odja, valamint a program r¨ ovid dokument´ aci´ oja, amely tartalmazza a megold´ as r¨ovid le´ır´as´ at, ´es megadja, hogy a forr´ as´allom´any melyik fejleszt˝oi k¨ornyezetben ford´ıthat´ o. ´ A gazdas´ I. 578 (E). ag helyzet´er˝ ol rengeteget el´ arul a k¨ozleked´esi infrastrukt´ ura. Ebben a feladatban haz´ank 2007 ´es 2021 k¨ oz¨ otti megy´enk´enti adatait fogjuk g´ orcs˝ o al´ a venni. 1. Hozzunk l´etre a t´ abl´ azatkezel˝ oben egy u ´j munkaf¨ uzetet. Ennek k´et munkalapja ol kezdve t¨olts¨ uk be a munkalegyen Adatok ´es Valasz. Ezekre az A1-es cell´at´ lapok nev´evel megegyez˝o, UTF-8 k´odol´as´ u, tabul´ atorral tagolt txt-f´ ajlokat. 2. A k´et munkalapon v´egezz¨ uk el az al´ abbi mint´ akon l´athat´ o form´az´ asokat, ´es oldjuk meg a le´ır´as szerinti feladatokat. 3. Az Adatok munkalapon rendezz¨ uk a´t a megy´ek adatait a megye neve szerint n´evsorba. 4. Az ´evenk´enti legnagyobb fejleszt´es adatait hat´ arozzuk meg a Valasz munkalap B3:G3 tartom´ any´ aban, teh´ at azt, hogy az el˝oz˝ o ´evihez k´epest mikor ´es hol volt a legnagyobb n¨ oveked´es, ´es mekkora volt a hosszn¨oveked´es ´ert´eke, mind k¨oz-, mind vas´ uton. K¨ oz´ episkolai Matematikai ´ es Fizikai Lapok, 2022/9

551

Adatok munkalap

552

K¨ oz´ episkolai Matematikai ´ es Fizikai Lapok, 2022/9

diagram

Adatok munkalap

Seg´edsz´ am´ıt´ asokat az Adatok munkalap 22. sor´ at´ ol lefel´e vagy az AG oszlopt´ ol jobbra tal´ alhat´ o cell´ akban v´egezhet¨ unk. A megold´ asban saj´ at f¨ uggv´eny vagy makr´o nem haszn´ alhat´ o. Bek¨ uldend˝o egy t¨ om¨ or´ıtett i578.zip ´ allom´anyban a megold´ ast tartalmaz´ o munkaf¨ uzet ´es a megold´ as r¨ ovid le´ır´ as´ at bemutat´ o dokument´ aci´ o. I. 579. A Tekn˝ ocgrafika eredetileg a Logo programoz´asi nyelvhez k´esz¨ ult, de ma m´ar t¨obb oktat´ asi c´el´ u fejleszt˝ o k¨ ornyezetnek ´es nyelvnek r´esze, p´eld´ aul el´erhet˝o Scratchben vagy Pythonban. Egy magyar fejleszt´es keret´eben a Tekn˝ocgrafik´ at alkalmazhatjuk a JavaScript nyelv seg´ıts´eg´evel. Az Agent JS programoz´ asi fel¨ ulet, valamint a dokument´ aci´ o ´es n´eh´ any p´elda tal´alhat´ oa https://vimtaai.github.io/agent/ c´ımen. K¨ ul¨on¨osen izgalmas ter¨ ulete a Tekn˝ocgrafik´ anak a rekurz´ıv ´abr´ ak ´es frakt´ alok rajzol´ asa. K´esz´ıts¨ uk el a Koch-g¨ orbe, a s´ ark´ anyg¨ orbe, valamint a Sierpi´ nskih´aromsz¨og ´abr´ aj´at az Agent JS seg´ıts´eg´evel u ´gy, hogy a rekurzi´o szintje a felhasz-

Valasz munkalap

K¨ oz´ episkolai Matematikai ´ es Fizikai Lapok, 2022/9

553

554

K¨ oz´ episkolai Matematikai ´ es Fizikai Lapok, 2022/9

n´ al´o ´ altal be´ all´ıthat´ o legyen. A h´ arom frakt´ alr´ ol a K¨oMaL 2021. m´ ajusi sz´ am´aban is olvashattunk, innen sz´ armazik a s´ ark´ anyg¨ orbe k´esz´ıt´es´et szeml´eltet˝ o´ abra. Bek¨ uldend˝o egy t¨ om¨ or´ıtett i579.zip ´ allom´anyban a h´ arom ´abr´ at l´etrehoz´ o oldal forr´ ask´ odja. I/S. 67. Adott egy N × M -es t´eglalap, melyet le szeretn´enk helyezni a s´ıkra u ´gy, hogy az egyik cs´ ucsa az orig´ oba essen. Adjuk meg, hogy h´ anyf´elek´eppen tehetj¨ uk ezt meg u ´gy, hogy a m´asik h´ arom cs´ ucsa is r´ acspontra, azaz eg´esz koordin´ at´ akkal rendelkez˝o pontra essen. A bemenet egyetlen sor´ aban az N ´es M sz´ amok, a t´eglalap oldalhosszai tal´alhat´ ok sz´ok¨ ozzel elv´ alasztva. A kimenet egyetlen sor´ aban egy sz´ am szerepeljen: h´anyf´elek´eppen lehet lehe´gy, hogy az egyik cs´ ucsa az orig´ oba, a t¨obbi cs´ ucsa lyezni a t´eglalapot a s´ıkra u r´ acspontra essen. P´elda: Bemenet 2 2 5 10

Kimenet 4 24

Korl´ atok: 1  N, M  106 . Id˝ olimit: 0,4 mp. ´ ekel´es: a pontok 50%-a kaphat´ Ert´ o, ha a program helyes kimenetet ad az 1  N  100 esetekre. Bek¨ uldend˝o egy is67.zip t¨ om¨ or´ıtett a´llom´anyban a megfelel˝oen dokument´ alt ´es kommentezett forr´ asprogram, amely tartalmazza a megold´ as l´ep´eseit, valamint megadja, hogy a program melyik fejleszt˝ oi k¨ ornyezetben futtathat´ o. A dokument´ aci´ o tartalmazza a megold´ as elm´eleti h´ atter´et, az esetleg felhaszn´alt forr´ asokat. Ne tartalmazzon k´ odr´eszleteket, azok magyar´azata k´ odkommentek form´aj´aban a forr´ asprogramban szerepeljen.

´Irjunk programot, amely kisz´ amolja, mennyi a v´ arakoz´asi id˝ ok maximuma, ha minden rendel´est k¨ ul¨ on-k¨ ul¨ on szolg´alunk ki, illetve mennyi a v´ arakoz´asi id˝ ok maximum´anak lehet˝o legkisebb ´ert´eke, ha pont j´ okor v´ altunk a s¨ ut´es ´es t¨ olt´es k¨oz¨ott. Ezen a helyen nem szeretik a pazarl´ast, ´ıgy csak akkor a´llnak neki egy palacsinta megs¨ ut´es´enek, ha m´ ar van r´ a leadott rendel´es. Teh´ at olyan nem fordulhat el˝o, hogy t¨obb megs¨ ut¨ ott palacsint´ ank van, mint a kiszolg´ alatlan rendel´esek ¨ osszege. A bemenet els˝ o sor´ aban egy palacsinta megs¨ ut´es´enek ideje S, egy palacsinta t¨olt´es´enek ideje T , ´es a kett˝ o k¨ oz¨ otti v´ alt´ as ideje V szerepel. A m´ asodik sorban am a rendel´esek R sz´ ama van. A k¨ ovetkez˝ o R sor mindegyik´eben k´et eg´esz sz´ szerepel: az adott rendel´es lead´ as´ anak id˝opontja ´es a k´ert palacsint´ak sz´ama. A kimenet els˝ o sor´aba ´ırjuk ki, mennyi a v´ arakoz´asi id˝ ok maximuma, ha a rendel´eseket egyes´evel szolg´aljuk ki, azaz mindig csak annyi palacsint´ at s¨ ut¨ unk, amennyi a k¨ovetkez˝ o rendel´eshez kell. A kimenet m´ asodik sor´ aba pedig azt, hogy mennyi a v´ arakoz´asi id˝ ok maximum´anak lehet˝o legkisebb ´ert´eke. Minta: Bemenet (a / jel sort¨ or´est helyettes´ıt) 2 1 1 / 2 / 0 5 / 2 1

Magyar´ azat (z´ ar´ ojelben a befejez´esi id˝o szerepel): az els˝o esetben megs¨ ut¨ unk 5-¨ot (10), majd megt¨ oltj¨ uk (16), majd s¨ ut¨ unk m´eg egyet (18), majd azt is megt¨oltj¨ uk (20). A v´ arakoz´asi id˝ o cs¨ okkenthet˝ o, ha el˝obb megs¨ ut¨ unk 5-¨ ot (10), majd mivel van r´ a rendel´es, m´eg egyet (12), majd megt¨ oltj¨ uk az els˝ o ¨ ot¨ ot (18), majd m´eg egyet (19). am´aKorl´ atok: 1  S, T, V  104 , R  105 . A rendel´es ideje ´es a rendel´esek sz´ olimit: 1 mp. nak ¨osszege is legfeljebb 105 . Id˝ ´ ekel´es: a pontok 20%-a kaphat´ Ert´ o, ha a program ´altal adott kimenet els˝ o sora helyes. Tov´ abbi 40% kaphat´ o, ha a program helyes kimenetet ad az R  10 ´es a palacsint´ ak sz´am´anak ¨ osszege legfeljebb 20 esetekben. Bek¨ uldend˝o egy s166.zip t¨ om¨ or´ıtett a´llom´anyban a megfelel˝oen dokument´alt ´es kommentezett forr´ asprogram, amely tartalmazza a megold´ as l´ep´eseit, valamint megadja, hogy a program melyik fejleszt˝ oi k¨ ornyezetben futtathat´ o. A dokument´ aci´ o tartalmazza a megold´ as elm´eleti h´ atter´et, az esetleg felhaszn´alt forr´ asokat. Ne tartalmazzon k´ odr´eszleteket, azok magyar´azata k´ odkommentek form´aj´aban a forr´ asprogramban szerepeljen.



S. 166. Marika n´eni palacsint´ az´ oj´aban egy ember s¨ uti ´es t¨olti a palacsint´ akat. Eddig, ha ´erkezett egy rendel´es, akkor megs¨ ut¨ otte a megfelel˝ o sz´ am´ u palacsint´ at, majd belet¨ olt¨ otte a k´ert t¨ oltel´eket, ´es odaadta a p´enzt´arosnak, aki a fizet´est elu lett hanyagolhat´ o id˝ on bel¨ ul elint´ezte. Az ut´ obbi id˝ oben azonban nagyon n´epszer˝ a hely, ez´ert n´eha nagyon sokat kell v´arni. Hamar kider¨ ult, hogy a s¨ ut´es ´es palacsintat¨ olt´es k¨ oz¨ otti v´ alt´ as sok id˝ot vesz el. Azt szeretn´enk kider´ıteni, hogy ez val´ oban probl´ema-e. K¨ oz´ episkolai Matematikai ´ es Fizikai Lapok, 2022/9

555

Kimenet (a / jel sort¨ or´est helyettes´ıt) 20 / 19

A feladatok megold´asai regisztr´aci´o ut´an a k¨ovetkez˝o c´ımen t¨olthet˝ok fel: https://www.komal.hu/munkafuzet Beku ¨ld´esi hat´arid˝o: 2023. janu´ar 15.



556

K¨ oz´ episkolai Matematikai ´ es Fizikai Lapok, 2022/9

Fizika alapszak ´es fizikatan´ar-k´epz´es az ELTE TTK Fizikai Int´ezet´eben A vil´agon az egyik legizgalmasabb ´es legszebb feladat a term´eszet kutat´ asa, m˝ uk¨ od´es´enek meg´ert´ese. A kutat´ as egy ´eletre sz´ ol´o ´elm´eny, egy ´eletre sz´ ol´o kih´ıv´ as ´es izgalom. Ugyanakkor a hallgat´oink olyan nyitotts´ agot, probl´emamegold´ o k´eszs´eget is elsaj´at´ıtanak, amely az ´elet b´ armely ter¨ ulet´en nagyon j´ol hasznos´ıthat´ o. Az itt v´egzettek k¨ oz¨ ott kiv´ al´o, a nemzetk¨ ozi ´elvonalban dolgoz´o fizikusokat tal´alunk, de olyan c´egvezet˝ ot is, aki egy patin´as Wall Street-i befektet´esi bank budapesti matematikai modellez˝ o csoportj´ at vezeti, vagy p´eld´ aul olyan, ma m´ ar az USA-ban ´el˝o v´allalkoz´ ot, aki az amerikai l´egier˝ onek sz´ all´ıt folyad´ekkrist´aly-kijelz˝os sisakokat. A fizika alapk´epz´es mellett int´ezet¨ unkben k´epezz¨ uk a fizikatan´ arok jelent˝os r´esz´et. Aki szereti a fizik´ at ´es m´ar most is szereti t´ arsait tan´ıtani, aj´ anljuk figyelm´ebe ar-k´epz´es¨ unket! a fizikatan´ Hogy mi´ert ´erdemes a fizika alapszakot v´alasztani? • Modern oktat´as

A k´epz´es¨ unk t¨ obbszint˝ u ´es sokoldal´ u. A sokoldal´ us´ag abban mutatkozik meg, hogy a harmadik f´el´evvel kezd˝ od˝ oen ´erdekl˝ od´esi ter¨ ulet szerint specializ´aci´ ot (fizikus, informatikus-fizikus, biofizikus, csillag´ asz, geofizikus, meteorol´ ogus) lehet v´ alasztani. A k´epz´es k¨ oz¨ os r´esz´eben a magas szint˝ u fizikai ismereteken t´ ul matematik´ at, elektronik´ at ´es informatik´ at is oktatunk. Mivel nincs fizikus k´ıs´erletek n´elk¨ ul, az alapvet˝ o fizikai m´er´esi k´eszs´egeket ´es mag´ at a k´ıs´erletez˝ o szeml´eletet a fizikai laborat´oriumi gyakorlatokon lehet elsaj´at´ıtani. A laborokon a di´akok p´eld´ aul Raspberry Pi vez´erl´est haszn´ alva v´egzik alapm´er´eseiket, k´es˝ obb pedig olyan ´erdekes fizikai jelens´egekkel ´es berendez´esekkel tal´ alkoznak, mint a pozitronemisszi´ os az´ o elektronmikroszk´ op vagy ´eppen a kvantumratomogr´ afia, a hologr´ afia, a p´aszt´ d´ır. A kurzusok egy r´esze k´et (norm´al ´es emelt) szinten v´egezhet˝ o, melyek k¨onnyen atj´ ´ arhat´ oak. A norm´al szint biztos´ıtja, hogy a nem elit iskol´ab´ol ´erkez˝o, de motiv´ alt hallgat´ ok sz´am´ara is elsaj´ at´ıthat´ o ´es ´elvezhet˝ o legyen a tananyag. Az emelt szint˝ u´ or´ akon gyorsabb halad´ ast ´es kieg´esz´ıt˝ o tartalmakat biztos´ıtunk. • Vil´agsz´ınvonal´ u kutat´asok Az ARWU ranglist´at a fels˝ooktat´ asi szakma ´evek ´ota a legmegb´ızhat´ obb ´ert´ekel´esek k¨ oz¨ ott tartja sz´ amon. A 2022-es szakter¨ uleti felm´er´es (Global Ranking of Academic Subjects – GRAS) a kor´ abbiakhoz hasonl´ oan 5 nagyobb tudom´anyter¨ uleten (term´eszettudom´ anyok, m´ern¨ oki tudom´anyok, ´elettudom´anyok, eg´eszs´egtudom´anyok ´es t´ arsadalomtudom´anyok), azon bel¨ ul 54 sz˝ ukebb szakter¨ uleten vizsag egyetemeit. Az 5000 vizsg´alt int´ezm´eny k¨oz¨ ul 96 orsz´ag 1800 egyeteg´alta a vil´ me ker¨ ult be az idei rangsorba. Egyetem¨ unk a fizika ter¨ ulet´en szerepelt a legjobban: a 105. helyre ker¨ ult el˝ore, a tavalyi 145. helyr˝ ol. ´Igy b´ atran a´ll´ıthatjuk, hogy

K¨ oz´ episkolai Matematikai ´ es Fizikai Lapok, 2022/9

557

az ELTE TTK Fizikai Int´ezet nemzetk¨ ozi viszonylatban is kiemelked˝o hely a fizika tanul´ as´ara. Az ¨ osszes magyar int´ezet k¨ oz¨ ul itt a legsz´elesebb a v´ alaszt´eka azoknak a ter¨ uleteknek, amelyeket oktatunk ´es kutatunk. A fizika legmodernebb, legizgalmasabb ter¨ uleteivel foglalkozunk: a gravit´ aci´ os hull´amok kutat´ as´at´ ol a r´eszecske- ´es biofizik´ an kereszt¨ ul az asztrofizik´aig, a nanotechnol´ ogi´aig ´es a kvantumsz´ am´ıt´ og´epekig mindent lefed¨ unk, ami ma ´erdekes a fizik´ aban. K¨ or¨ ulbel¨ ul sz´ az oktat´ onkkal ´es kutat´ onkkal, valamint di´ akjainkkal nagyon sok nemzetk¨ ozi egy¨ uttm˝ uk¨ od´esben vesz¨ unk r´eszt. A kutat´ as ir´ant is ´erdekl˝ od˝ o di´akok sz´am´ara bej´ aratott u ´t vezet a tudom´anyos di´ akk¨ ori projektek fel´e. A di´ akk¨ ori kutat´ omunk´ ak kiv´al´o alapot adoldi egyetemeken t¨ ort´en˝ o mesterk´epz´esben vagy doktori iskol´aban t¨ ort´en˝ o nak a k¨ ulf¨ tov´ abbtanul´asra. Az ELTE TTK-n foly´o fizikai t´em´ aj´ u kutat´ asok sok esetben vil´ agsz´ınvonal´ u kutat´ ohelyekkel t¨ ort´en˝ o egy¨ uttm˝ uk¨ od´esben val´ osulnak meg. Di´ akjaink eljuthatnak a sv´ ajci CERN r´eszecskefizikai kutat´ ocentrumba, vagy a LIGO amerikai gravit´ aci´ oshull´ am-detektor eredm´enyeit elemezhetik. • Kit˝ un˝o elhelyezked´esi lehet˝os´egek A fizika t´ argy tud´asa, a fels˝ o szint˝ u matematika ´es a programoz´ asi ismeretek, amit a fizika alapszakokon el lehet saj´at´ıtani, sz´ amos munkahelyen ad lehet˝ os´eget a karrier ´ep´ıt´es´ere. A fizika szakon v´egzetteket nemcsak a kutat´oint´ezetekben, egyetemeken v´ arj´ ak, hanem p´eld´ aul a p´enz¨ ugyekkel, informatik´ aval, t´avk¨ ozl´essel, m´ern¨oki vagy orvostudom´anyokkal foglalkoz´ o c´egek is sz´ıvesen alkalmazz´ ak ˝ oket. • Hallgat´oi ´elet Az ELTE TTK hallgat´ oi ´elete vid´ am ´es szerte´ agaz´ o. A Magyar Fizikus Hallgat´ ok Egyes¨ ulete sz´ amos programot szervez a hallgat´oinknak. K¨ ulf¨ oldi di´akkonferenci´akon vagy cseregyakorlatokon lehet r´eszt venni, szabadid˝ os programok ´es a fizika t´ argyakban felk´esz´ıt˝ o programok szerepelnek a palett´ an. A fizika szakokhoz j´ ol szervezett mentorprogram t´arsul. Minden ´evfolyamon t¨ obb k´epzett mentor seg´ıt a t´argyak felv´etele k¨ or¨ uli k´erd´esekben, az optim´ alis egyetemi strat´egi´ ak megtal´ al´as´aban, ´es ´ atadj´ ak a fels˝ obb ´eves di´ akok a´ltal ¨ osszegy˝ ujt¨ ott tapasztalatokat. A fizika szak nem ´er v´eget a BSc-fokozat megszerz´es´evel. Az ELTE TTK Fizikai Int´ezet´eben 5 k´et´eves mesterk´epz´esi (MSc) szakra lehet jelentkezni: fizikus, geofizikus, csillag´asz, meteorol´ ogus, anyagtudom´ any. A fizikus mesterin bel¨ ul a kutat´ ofizikus, biofizikus ´es tudom´ anyos adatanalitika ´es modellez´es (ez ut´ obbi t¨ obbek ori´asi adathalmazokon v´egzett kutat´ asokkal, k¨oz¨ott napjaink forr´ o” t´em´aj´aval, az ´ ” a big data”-val foglalkozik) v´alaszthat´ o. A k´epz´es harmadik szintje az int´ezetben ” a n´egy´eves doktori iskola (PhD-fokozat). Hogy mi´ert ´erdemes fizikatan´ar szakot v´alasztani? • 2022-t˝ ol u ´j rendszer˝ u, 5 ´eves osztatlan k´epz´es 2022 szeptember´eben indult az u ´j rendszer˝ u fizikatan´ ar-k´epz´es. Az u ´j rendszerben a k´epz´esi id˝o 5 ´ev. L´enyeg´eben minden f´el´evben lesz tan´ıt´asi gyakorlat, ´es az utols´o f´el´ev szinte teljesen k¨ oz´episkol´ aban zajlik. Az ELTE h´arom gyakorl´oiskol´aja is kiv´al´o terep a tan´ ars´ ag komplex elsaj´at´ıt´ as´ ara. A szakmai ´es m´ odszertani ´or´ akat a Fizikai Int´ezet kit˝ un˝ o kutat´ oi-oktat´oi ´es a legjobb k¨ oz´episkolai tan´ arok

558

K¨ oz´ episkolai Matematikai ´ es Fizikai Lapok, 2022/9

tartj´ ak. Az ELTE nagyon sok szakot ind´ıt a fizik´aval p´ arban, ´ıgy szinte b´ armilyen tant´ argyat lehet m´asiknak v´ alasztani. • A tan´ari p´alya sz´eps´egei

Napjainkban igen sok sz´o esik a tan´ari p´ alya neh´ezs´egeir˝ ol. Mi´ert lehet m´egis ´erdemes a fizikatan´ari hivat´ ast v´ alasztani? Mert nagyon sok sz´eps´ege is van! Kreat´ıv, v´ altozatos, fiatalok k¨ oz¨ ott v´egzett munka. Egy fizikatan´ arnak hatalmas lehet˝ os´egei vannak arra, hogy megmutassa a gyerekeknek a fizikai vil´ag m˝ uk¨od´es´enek sz´eps´egeit.

• Hallgat´oi ´elet ¨ ond´ıj Program keret´eben egyetemistak´ent f´el´evente A Klebelsberg K´epz´esi Oszt¨ ak´ ar 375 000 Ft-ot lehet kapni, mely t¨ obb k¨ ul¨ onf´ele ¨oszt¨ond´ıjjal is kieg´esz´ıthet˝ o. Fontos megeml´ıteni, hogy lehet˝os´eg van oktat´ assal kapcsolatos kutat´asokba val´ o becsatlakoz´ asra ´es doktori tanulm´anyok folytat´ as´ara a Fizika Tan´ıt´asa Doktori Program keret´eben. A k´epz´esek r´eszleteir˝ ol az int´ezet honlapj´an (https://physics.elte.hu) lehet tov´ abbi inform´ aci´ okat szerezni, vagy ´erdemes ell´atogatni az ELTE TTK youtube csatorn´ aj´ara (https://www.youtube.com/ELTETTKbudapest).

II. r´esz´eben megismerked¨ unk egy olyan m´odszerrel (az u ´n. konform lek´epez´esekkel), amely akkor is alkalmazhat´o, amikor a t¨ ukr¨ oz´esek m´ odszere nem m˝ uk¨ odik. N´eh´any fizikai fogalom, j´ol haszn´alhat´ o m´ odszerek ´es hasznos matematikai osszefu ¨ ¨gg´esek ´ Arams˝ ur˝ us´eg-vektor. T´erben kiterjedt elektromos ´aramot a fel¨ uletegys´egenk´ent a´tfoly´ o ´arammal, az ´arams˝ ur˝ us´eggel jellemezhetj¨ uk: j = I/A (A egy kicsiny, a t¨olt´esek a´raml´ as´ ara mer˝ oleges fel¨ uletdarab ter¨ ulete). Az ´arams˝ ur˝ us´eget vektornak tekintj¨ uk, ir´anya a (pozit´ıv) t¨ olt´eshordoz´ ok mozg´asi ir´ anya. (Az a´ramer˝ oss´eg uletvektor skal´ aris skal´ aris mennyis´eg, ami fel´ırhat´ o az a´rams˝ ur˝ us´eg-vektor ´es a fel¨ szorzatak´ent: I = j · A.)

Az Ohm-to ozegben az – a´ltal´ aban ¨rv´eny differenci´alis alakja. Egy homog´en k¨ helyr˝ ol helyre v´ altoz´ o – a´rams˝ ur˝ us´eg ar´ anyos az ottani elektromos t´erer˝ oss´eggel: j(r) = σE(r), ahol σ az anyag vezet˝ ok´epess´ege, a fajlagos ellen´ all´ as () reciproka.

T´erb˝ol s´ıkba visszal´ep´es fizikai probl´em´ak megold´as´an´al I. r´esz (tu or´aramok) ¨ k¨

S´ıkbeli ´araml´asok. Ha egy v´ekony f´emlemezbe valahol ´aramot vezet¨ unk, akkor az ´arams˝ ur˝ us´eg-vektor (az ´aram bevezet´esi hely´enek sz˝ uk k¨ ornyezet´et lesz´ am´ıtva) a lemez s´ıkj´aval p´ arhuzamos, ´es a lemez egy-egy pontj´an´ al a lemezre mer˝olegesen haladva a´lland´ o. Ha a lemeznek valahol (egyenes vagy g¨ orbe) hat´ arvonala van, azon a vonalon nem folyhat ´at ´ aram, teh´ at az ´ arams˝ ur˝ us´eg-vektor a hat´ arvonaln´ al any´ u. (Ezt a k¨ ovetelm´enyt hat´ arfelt´etelnek nevezik.) ´erint˝oir´ ´ Arameloszl´ asok szuperpon´alhat´ os´aga. Ha egy lemezben valamilyen Φ1 (r) arameloszl´as alakul ki, egy m´asik, Φ2 (r) poelektromos potenci´al hat´ as´ ara j 1 (r) ´ tenci´ al hat´ as´ara j 2 (r) az a´rameloszl´as, akkor

Megjegyz´esek ´es ´altal´anos´ıt´asok a P. 5399. feladat megold´as´ahoz1

Φ(r) = Φ1 (r) + Φ2 (r) potenci´ alt´erben az a´rameloszl´as

Bevezet´es A k¨ ozelm´ ultban egy kilencr´eszes matematika cikksorozat jelent meg a K¨oMaLban2 , amely olyan bizony´ıt´ asokat mutatott be, amikor a s´ıkbeli geometriai alakzatokat t´erbe kil´epve”, h´ arom- vagy ak´ar m´eg magasabb dimenzi´ os objektumok ” vet¨ uletek´ent vagy metszetek´ent a´ll´ıtjuk el˝ o. Az itt k¨ ovetkez˝ okben a ford´ıtott utat j´ arjuk be: megmutatjuk, hogy bizonyos t´erbeli fizikai (´aramvezet´esi) probl´em´ ak k¨ onnyebben kezelhet˝ ok, ha valamilyen m´ odon siker¨ ul ´ atfogalmazni azokat s´ıkbeli feladatt´ a. Olyan feladatokkal fogunk foglalkozni, amelyekben v´ekony f´emlemezben foly´ o´ aramok szerepelnek, de a f´emlemez nem s´ıkban, hanem t´erben helyezkedik el. Csak olyan eseteket vizsg´alunk, amelyekben a t´erbeli lemez s´ıkba kiter´ıthet˝ o”. (Ilyen alakzat p´eld´ aul egy k´ up vagy egy ” henger pal´ astja.) A cikk I. r´esz´eben olyan s´ıkbeli elektromos ´arameloszl´asokat vizsg´alunk, amelyek – n´eh´ any t¨ uk¨ or” elhelyez´es´evel – k¨ onnyen megoldhat´ o feladatt´ a v´alnak. A cikk ” 1

j(r) = j 1 (r) + j 2 (r) alak´ u lesz. A szuperpon´ alhat´ os´ ag az´ert val´ osulhat meg, mert az Ohm-t¨ orv´eny line´aris, az a´rams˝ ur˝ us´eg egyenesen ar´ anyos az elektromos t´erer˝ oss´eggel. Tu any hat´arfelt´eteleit bizonyos ¨kr¨oz´esek m´odszere. Egy v´eges s´ıkbeli tartom´ esetekben u ´gy biztos´ıthatjuk, hogy egy (vagy t¨ obb), a tartom´ anyon k´ıv¨ ul foly´ o, elk´epzelt (fikt´ıv) ´arameloszl´as ´es a val´ odi lemezben foly´ o´ arameloszl´as szuperpoz´ıci´oj´at k´epezz¨ uk. Ha p´eld´ aul egy v´egtelen” f´els´ık alak´ u lemezbe valahol I er˝ oss´eg˝ u ” ´aramot vezet¨ unk, akkor a kialakul´ o´ arameloszl´as olyan, mintha az a´ram bevezet´esi pontj´ anak a f´els´ık hat´ arol´ o egyenes´ere vett t¨ uk¨ ork´ep´en´el is I ´aramot vezetn´enk be a teljes (v´egtelen) s´ıklemezbe. V´egtelen s´ıklemezbe vezetett, adott er˝ oss´eg˝ u ´aram sz´etoszl´asa a lemezen. Ha egy nagyon nagy m´eret˝ u ( v´egtelen”), δ vastags´ ag´ u lemez O pontj´ an´al I er˝ oss´eg˝ u ” ´aramot vezet¨ unk be, akkor a kialakul´ o´ arams˝ ur˝ us´eg az O pontt´ ol r t´ avols´ agban

A feladatot ´es annak megold´ as´ at l´ asd Lapunk 565. oldal´ an. K´ os G´eza: T´erbe kil´ep˝ o bizony´ıt´ asok I–VII. ´es egy r´ aad´ as, K¨ oMaL 2019. ´evi 10. sz´ am – 2020. ´evi 5. sz´ am

(1)

559

560

2

K¨ oz´ episkolai Matematikai ´ es Fizikai Lapok, 2022/9

  j(r) =

I , 2πrδ

K¨ oz´ episkolai Matematikai ´ es Fizikai Lapok, 2022/9

´es az a´rams˝ ur˝ us´eg ir´anya mindenhol az O pontb´ ol az adott pontba h´ uzott egyenessel p´ arhuzamos, teh´ at sug´arir´any´ u”. (Ez az elrendez´es szimmetri´aj´ab´ol k¨ovetkezik, ´es ” abb´ ol, hogy A = 2πrδ nagys´ ag´ u hengerfel¨ uleten ¨ osszesen I er˝ oss´eg˝ u ´aram folyik ´at.) uggv´eny Bizonyos fu ¨ggv´enyek kicsiny v´altoz´asa. Ha pl. az y = f (x) ≡ K xλ f¨ argumentum´at x-r˝ol x + Δx-re n¨ ovelj¨ uk (Δx x), akkor a f¨ uggv´eny ´ert´ek´enek megv´ altoz´ asa ñÅ ô ã Δx λ λ λ λ Δy = K(x + Δx) − Kx = Kx − 1 ≈ Kλxλ−1 · Δx. 1+ x (Az utols´ o l´ep´esben felhaszn´ altuk a Newton-f´ele (1 + ε)λ ≈ 1 + λε o ugg´est, ami ¨sszef¨ ε  1 eset´en tetsz˝ oleges λ kitev˝ ore ´erv´enyes. Zsebsz´ amol´ og´epen kipr´ ob´ alhatjuk, hogy pl. 1,0021,5 = 1,003 001 5 ≈ 1,003 = 1 + 0,002 · 1,5.)

1. a ´bra

A kapott k¨ ozel´ıt˝o o ugg´est ´ıgy is fel´ırhatjuk: ¨sszef¨ (2)

Δx Δy ≈λ , y x

ha

y = K xλ .

Hasonl´o megfontol´ assal kaphatjuk meg, hogy az exponenci´ alis f¨ uggv´eny n¨oveked´ese: (3)

Δy ≈ λ · Δx, y

ha

2. a ´bra

felt´etel mellett, hogy az AC egyenesen kereszt¨ ul ne folyjon a´ram. Ezt a felt´etelt u ´gy teljes´ıthetj¨ uk, hogy az elrendez´est t¨ ukr¨ ozz¨ uk az AC egyenesre (´ıgy kapjuk a s´arga f´els´ıkot), majd az ered˝ o´ arameloszl´ast sz´ am´ıtjuk ki a C pontban (3. ´ abra bal oldali r´esze).

y = K eλx .

Ezt az ¨ osszef¨ ugg´est is ´erdemes kipr´ ob´ alni” egy zsebsz´ amol´ og´epen, valamekkora konkr´et ” K, λ, x ´es Δx eset´en. Megjegyz´es. A (3) k´eplet a radioakt´ıv boml´ asok exponenci´ alis boml´ ast¨ orv´eny´eb˝ ol is ismer˝ os lehet a fizika feladatok megold´ oinak a k¨ ovetkez˝ o jel¨ ol´esekkel: ΔN (t) ≈ −λ · N (t), Δt

ha

N (t) = N (0) e−λt .

A k´ upba vezetett ´aram eloszl´asa A P. 5399. feladat megold´ as´ anak v´egeredm´eny´et n´ezve felt˝ unhet, hogy a C pontban az a´rams˝ ur˝ us´eg ugyanakkora, mintha egy v´egtelen” s´ıklemezbe an” nak A pontj´ an´al I er˝ oss´eg˝ u ´ aramot vezetn´enk be, az A pontt´ ol R t´avols´ agra l´ev˝ o B pontb´ ol pedig elvezetn´enk azt, ´es a C pont a B pont A-ra vett t¨ uk¨ork´epe (1. ´ abra). Val´oban, ebben az esetben (a megold´ as jel¨ ol´eseit k¨ovetve): j(C) = jA (C) − jB (C) =

I 1 I I 1 − = . 2πδ R 2πδ 2R 4πδR

Vajon hogyan m´odosul az eredm´eny, ha az eredeti feladat AB t´avols´ ag´at 3R-r˝ol 2R-re v´ altoztatjuk (2. ´ abra)? A k´ uppal´astot most is felv´ aghatjuk az AC egyenes ment´en, majd kiter´ıtve egy f´els´ıkot kapunk. Most is az A pontban bevezetett ´es a B pontn´ al elvezetett, I er˝ oss´eg˝ u´ aram hat´as´ at vizsg´aljuk a C pont(ok)ban olyan K¨ oz´ episkolai Matematikai ´ es Fizikai Lapok, 2022/9

561

3. ´ abra

Az A pontba ¨ osszesen 2I er˝ oss´eg˝ u ´ aram (a val´ odi ´es a t¨ ukr¨ oz¨ ott a´ramok avol l´ev˝ o C pontban az a´rams˝ ur˝ us´eg: ¨osszege) folyik be, ´ıgy a 2R t´ I 2I 1 = . jA (C) = 2πδ 2R 2πδR √ A B ´es a B  pont egyar´ ant 2 · 2R t´ avol van C-t˝ ol, teh´ at a C pontban a megfelel˝ o ´arams˝ ur˝ us´egek nagys´ aga (l´asd a 3. ´ abra jobb oldali r´esz´et): jB (C) = jB  (C) = 562

1 I √ , 2πδR 2 · 2R

K¨ oz´ episkolai Matematikai ´ es Fizikai Lapok, 2022/9

az ered˝oj¨ uk pedig az A pont fel´e mutat, ´es √     j B (C) + j B  (C) = 2 · 2 j B (C) = I . 2 4πδR

Az elrendez´es szimmetri´aja miatt a C pontbeli ered˝ o´ arams˝ ur˝ us´eg AC ir´ any´ u, elegend˝ o teh´at az egyes ´aramok j´ arul´ek´ anak AC ir´ any´ u komponens´et kisz´am´ıtanunk, majd ezeket ¨ osszegezn¨ unk kell. A v´egtelen s´ıkba vezetett a´ram (1) formul´aja szerint az A pontb´ ol indul´o ´ aram j´arul´eka:

A teljes ´ arams˝ ur˝ us´eg a C pontban:

jA (C) =

I I I − = jC = 2πδR 4πδR 4πδR

I nI 1 = . 2πδ nR 2πδR

Jel¨ olj¨ uk a Bk AC sz¨ oget ϕk -val (5. ´ abra). K¨ onnyen kisz´ am´ıthatjuk, hogy

nagys´ ag´ u, ir´ anya pedig A-t´ ol C fel´e mutat. Ez megegyezik a kor´ abban vizsg´ alt k´et eset v´egeredm´eny´evel.

ϕ1 =

A feladat ´altal´anos´ıt´asa tu oz´essel megoldhat´ o esetekre ¨kr¨ Kiv´ ancsiak lehet¨ unk arra, hogy milyen eredm´enyt kapunk a leg´ altal´ anosabb, t¨ ukr¨ oz´esekkel megoldhat´ o esetben. Legyen az AB t´avols´ ag nR, ahol n tetsz˝ oleges pozit´ıv eg´esz sz´ am.

1 3 2k − 1 α, ϕ2 = α, . . . , ϕk = α, 2 2 2

de – mint l´atni fogjuk – ezen sz¨ ogek konkr´et ´ert´ek´ere nem is lesz sz¨ uks´eg¨ unk.

Sejt´es¨ unk: Tetsz˝ oleges eg´esz n-re ugyanazt az eredm´enyt fogjuk kapni, mint az n = 1, 2, 3 esetekben.

Tekints¨ uk a Bk pontot, ´es az onnan elvezetett a´ram j´arul´ek´ at a C pontbeli a´rams˝ ur˝ us´eghez. Mivel (az a´bra jel¨ ol´eseivel)

5. ´ abra

Bk C = rk = 2nR sin

ϕk , 2

jk =

I 1 , 2πδ rk

az´ert a j k vektornak az A → C ir´ any´ u¨ osszetev˝ oje: (vet¨ ulet)

jk

= −jk cos γk = −

I 1 . 4πδ nR ϕ

Az utols´ o l´ep´esben kihaszn´ altuk, hogy γk = 90◦ − 2k , ´es ´ıgy cos γk = sin(ϕk /2). Azt a meglep˝ o eredm´enyt kaptuk, hogy az elvezetett ´aramok mindegyik´enek j´arul´eka a C-beli a´rams˝ ur˝ us´eghez ugyanakkora, teh´at az ¨ osszeg¨ uk: n 

k=1

4. ´ abra

A f´emlemezt most is felv´ aghatjuk az AC egyenes ment´en (4. ´ abra), majd s´ıkba kiter´ıtve egy α = 2π/n ny´ıl´assz¨ og˝ u sz¨ ogtartom´anyt kapunk. (A jobb oldali a´br´an a kiter´ıtett k´ uppal´astnak csak azt a r´esz´et (a s´ arga sz´ınnel jel¨olt k¨orcikket) t¨ untett¨ uk fel, amelynek pontjai a k´ up cs´ ucs´ at´ ol legfeljebb nR t´avols´ agra vannak. A f´emlemez term´eszetesen a k¨ orcikknek a k¨ or´ıven k´ıv¨ uli r´esz´en is folytat´ odik, hiszen a k´ up nagy m´eret˝ u” volt, ezt azonban az ´abr´ an nem ´abr´ azoltuk.) A sz¨ogtar” tom´anyt az egyenes oldalai ment´en t¨ obbsz¨ or t¨ ukr¨ ozhetj¨ uk, ´ıgy v´eg¨ ul a teljes s´ıkot ott ter¨ ulet a´rameloszl´asa: az A pontba ¨osszesen megkapjuk. Az eredeti ´es a t¨ ukr¨ oz¨ nI ´ aramot vezet¨ unk be, a B1 , B2 , . . . , Bn pontok mindegyik´en´el pedig I er˝ oss´eg˝ u aramot vezet¨ ´ unk el. Ezzel el´erj¨ uk, hogy a s´ arga sz¨ogtartom´any egyenes oldalain kereszt¨ ul nem folyik a´t a´ram, azok a´ramvonalak. K´erd´es most az, hogy mekkora az a´rams˝ ur˝ us´eg a C pontban? K¨ oz´ episkolai Matematikai ´ es Fizikai Lapok, 2022/9

563

(vet¨ ulet)

jk

(vet¨ ulet)

= n · j1

=−

I 4πδR.

Ezt az a´rams˝ ur˝ us´eget hozz´ aadva az A-pontban bevezetett a´ram j´arul´ek´ ahoz, v´egeredm´eny¨ unk: I I I − = , j(C) = 2πδR 4πδR 4πδR abban (az n = 1, 2, 3 esetekn´el) kapott eredm´enyekkel. ¨osszhangban a kor´ Ha a geometriai viszonyok olyanok, hogy a felv´ agott ´es kiter´ıtett k´ uppal´asttal ´es annak t¨ ukr¨ oz¨ ottjeivel nem tudjuk h´ezag- ´es ´ atfed´esmentesen lefedni a teljes s´ıkot, akkor a t¨ ukr¨ oz´esek m´ odszere nyilv´ an cs˝od¨ ot mond. A cikk II. r´esz´eben megmutatjuk, hogy egy alapvet˝ oen m´ asfajta elj´ar´ as, a konform lek´epez´esek m´ odszere m´eg ebben az esetben is eredm´enyre vezet. Gn¨adig P´eter 564

K¨ oz´ episkolai Matematikai ´ es Fizikai Lapok, 2022/9

Fizika gyakorlat megold´asa

G. 783. Egy homog´en, n t¨ or´esmutat´ oj´ u, R sugar´ u u omb k¨ oz´eppontj´ aban ¨vegg¨ pontszer˝ u f´enyforr´ as helyezkedik el. A g¨ omb¨ ot k´ıv¨ ulr˝ ol n´ezz¨ uk. Hol l´ atjuk a f´enyforr´ as k´ep´et?

amikor csak a B pontb´ ol vezet¨ unk ki ´ aramot (de az nem az A pontn´ al, hanem a k´ up t´avoli r´eszein´el folyik be a f´embe). A k´et eredm´enyt ¨ osszeadva megkapjuk a C pontbeli ´arams˝ ur˝ us´eget a feladatunkban szerepl˝ o elrendez´esre. (A t´avoli r´eszeken ki- ´es bevezetett a´ramok kioltj´ ak” egym´ast.) ” (i) Ha csak az A pontn´ al csatlakozik vezet´ek a k´ uphoz, akkor az I er˝ oss´eg˝ u ´aram szimmetrikusan, a k´ up alkot´oi ment´en haladva oszlik sz´et a f´emlemezben. A C pontra illeszked˝ o, 2Rπδ ter¨ ulet˝ u k¨ orgy˝ ur˝ un ¨ osszesen I er˝ oss´eg˝ u´ aram folyik ´at, teh´at az ´arams˝ ur˝ us´eg-vektor nagys´ aga a C pontn´ al |j A | = jA =

(3 pont)

I . 2Rπδ

Megold´as. Az u omb k¨ oz´eppontj´ab´ ol kiindul´o f´enysugarak sug´arir´anyban ¨vegg¨ haladnak, teh´ at mer˝ olegesen ´erkeznek az u omb fel¨ ulet´ere. Ez nulla fokos bees´esi ¨vegg¨ sz¨ ognek felel meg, teh´ at a f´enysugarak t¨ or´es n´elk¨ ul haladnak a´t a fel¨ uleten. A g¨ ombb˝ol kil´ep˝ o f´enysugarak tov´abbra is sug´arir´anyban haladnak, teh´ at omb k¨ oz´eppontj´aban fut o¨ssze, itt keletkezik a f´enyfora meghosszabb´ıt´asuk a g¨ r´ as l´atsz´ olagos (virtu´ alis) k´epe. ´ Nagy Csenge (Sz´ekelyudvarhely, Tam´ asi Aron Gimn., 9. ´evf.) 17 dolgozat ´erkezett. Helyes 13 megold´ as. Hi´ anyos (1 pont) 1, hib´ as 3 dolgozat.

1. a ´bra

(ii) Vizsg´ aljuk most azt az esetet, amikor csak a B pontn´ al elvezetett ´arammal sz´ amolunk. V´agjuk sz´et – gondolatban – a k´ upot az AC egyenes ment´en. (Itt az´ert v´aghatjuk sz´et a lemezt, mert az AC menti v´ekony s´ avon kereszt¨ ul – a szimmetria miatt – sehol nem folyik ´at a´ram.)

Fizika feladatok megold´asa

P. 5399. Egy v´ekony, δ vastags´ ag´ u f´emlemezb˝ ol nagy, k´ up alak´ u fel¨ uletet hegesztett¨ unk ¨ ossze. A k´ up A-val jel¨ olt cs´ ucs´ aba I er˝ oss´eg˝ u ´ aramot vezet¨ unk, majd az egyik alkot´ on l´ev˝ o B pontb´ ol elvezetj¨ uk azt. Hat´ arozzuk meg a B-vel ´ atellenes C pontban az ´ arams˝ ur˝ us´eg-vektor ir´ any´ at ´es nagys´ ag´ at! Ismert, hogy az AB t´ avols´ ag ´ert´eke 3R, m´ıg a B ´es C pontok t´ avols´ aga 2R. (6 pont)

(Vigy´ azat: Az ´ arams˝ ur˝ us´eg eloszl´ asa most nem forg´ asszimmetrikus, az a ´ramvonalak nem p´ arhuzamosak a k´ up alkot´ oival, az elrendez´es csak az ABC s´ıkra val´ o t¨ ukr¨ oz´esre szimmetrikus.)

A felv´agott k´ upot kiter´ıthetj¨ uk s´ıkba, ´ıgy valamekkora α ny´ıl´assz¨ og˝ u sz¨ ogtar” tom´anyt” (a s´ıknak egy olyan r´esz´et, amelyet egy pontban tal´ alkoz´ o k´et f´elegyenes hat´ arol) kapunk (2. ´ abra). Az α sz¨ oget a B ´es C ponton a´thalad´ o k¨ or´ıv hossz´anak ´es a k¨or ker¨ ulet´enek egyenl˝ os´ege hat´ arozza meg (2. ´ abra). Mivel AB = 3R, 3R · α = 2Rπ

K¨ ozli: Vigh M´ at´e, Biatorb´ any

Megold´as. Amikor az elektromos ´aram egy v´ekony f´emlemezben folyik, akkor az ´ arams˝ ur˝ us´eg-vektor fel¨ uletre mer˝ oleges komponens´et elhanyagoljuk. ´ fogjuk megoldani a felVegy¨ uk ´eszre, hogy az ´aramok szuperpon´alhat´ oak. Ugy adatot, hogy el˝osz¨ or vessz¨ uk azt az esetet, amikor csak az A pontban vezet¨ unk a´ramot a f´embe, de a B pontbeli vezet´eket lekapcsoljuk. (Az a´ram ebben az esetben a k´ up nagyon t´ avoli r´eszein´el folyik ki a f´emb˝ ol.) Kisz´ amoljuk ebben az elrendez´esben a C pontbeli a´rams˝ ur˝ us´eget, majd megism´etelj¨ uk a sz´ amol´ast arra az esetre is, K¨ oz´ episkolai Matematikai ´ es Fizikai Lapok, 2022/9

2. a ´bra

565

=⇒

α=

2 π = 120◦ . 3

ogtartom´anyban keress¨ uk az a´rameloszl´ast azzal a haEbben a 120◦ -os sz¨ t´arfelt´etellel, hogy az AC1 ´es az AC2 f´elegyenesek a´ramvonalak legyenek, vagyis az ´arams˝ ur˝ us´eg-vektornak ne legyen a f´elegyenesekre mer˝ oleges komponense. Tudjuk, hogy egy v´egtelen” s´ıklap egyetlen B  pontj´ aba bevezetett a´ram ” ol r t´ avols´ agban eset´en az a´rams˝ ur˝ us´eg sug´arir´any´ u, ´es a nagys´ aga a B  pontt´ j(r) = 566

I . 2rπδ

K¨ oz´ episkolai Matematikai ´ es Fizikai Lapok, 2022/9

Ez azonban nem j´o megold´ asa a feladatunknak, hiszen nem teljes´ıti a hat´ arfelt´etelt. Alkalmazhatjuk viszont (az elektrosztatika t¨ uk¨ort¨olt´es-m´ odszer´enek mint´ aj´aos ra) a t¨ uk¨ or´ aramok” m´ odszer´et. Eg´esz´ıts¨ uk ki a kig¨orb´ıtett f´emlemez 120◦ -os val´ ” sz¨ ogtartom´any´ at m´eg k´et ugyanilyen, de fikt´ıv (f´emet nem tartalmaz´ o) sz¨ogtarab´ol tom´ annyal (3. ´ abra), ´es hat´ arozzuk meg a v´egtelen s´ık B  , B1 ´es B2 pontj´ elvezetett, egyenk´ent I er˝ oss´eg˝ u´ aram ered˝o ´ arams˝ ur˝ us´eg-eloszl´ as´at.

vektorokat. Mivel ezek a vektorok ellent´etes ir´ any´ uak, az ered˝o ´arams˝ ur˝ us´eg nagys´aga I I I − = , |j| = 2Rπδ 4Rπδ 4Rπδ ir´ anya pedig A-b´ ol C fel´e, teh´ at az alkot´ o ment´en lefel´e” mutat. ” ´ Isk. ´es Gimn., 11. ´evf.) ´es G´ abriel Tam´ as (Budapesti Fazekas M. Gyak. Alt. Toronyi Andr´ as (Budapest, Ba´ar-Madas Ref. Gimn., 12. ´evf.) dolgozata alapj´an 6 dolgozat ´erkezett. Helyes G´ abriel Tam´ as ´es Toronyi Andr´ as megold´ asa. Kicsit hi´ anyos (5 pont) 1, hi´ anyos (2–3 pont) 3 dolgozat.

P. 5418. K´et k¨ ul¨ onb¨ oz˝ o sz¨ ogben, de megegyez˝ o kezd˝ osebess´eggel elr´ ugott labda azonos t´ avols´ agban ´ert f¨ oldet. A magasabb p´ aly´ an halad´ o labda k´etszer annyi ideig rep¨ ult, mint a m´ asik. Hogyan ar´ anylik egym´ ashoz a k´et p´ alya cs´ ucsmagass´ aga? Milyen sz¨ ogek alatt r´ ugt´ ak el a labd´ at? (4 pont)

P´eldat´ ari feladat nyom´ an Megold´as. Legyen az alacsonyabb p´ aly´ an halad´ o labda elr´ ug´as´ anak sz¨ oge α, a m´asik labd´ a´e β, kezd˝osebess´eg¨ uk nagys´ aga pedig v0 . Ferde haj´ıt´ asn´ al a f¨ oldet ´er´es t´ avols´ aga

3. ´ abra

A B  pontbeli a´ram ´altal a C1 pontban l´etrehozott a´rams˝ ur˝ us´eg C1 -b˝ ol B  fel´e ir´anyul´o, I j = 2πδ · 3R nagys´ ag´ u vektor. Ugyanekkora nagys´ ag´ u a B2 pontbeli a´ram j 2 j´arul´eka: j2 =

d= vagyis (1)

2β = 180◦ − 2α,

(2)

vagyis

β = 90◦ − α.

A leveg˝ oben t¨ olt¨ ott id˝ otartamok: t=

I 12 Rπδ

2v0 sin α g

´es

2t =

2v0 sin β, g

ahonnan

nagys´ ag´ u j´arul´ek´ at, azt kapjuk, hogy

(3)

I I I + = . |j + j 1 + j 2 | = 6Rπδ 12 Rπδ 4Rπδ 

M´ ar csak a szuperpoz´ıci´ o maradt h´ atra, adjuk h´ at ¨ossze a f´emk´ up A pontj´aba bevezetett, majd a B pontb´ ol elvezetett ´aramnak megfelel˝ o ´arams˝ ur˝ us´egK¨ oz´ episkolai Matematikai ´ es Fizikai Lapok, 2022/9

sin 2α = sin 2β.

ol k¨ ovetkezik, hogy Mivel α < β < 90◦ , (1)-b˝

I . 6Rπδ

I A j  ´es j 2 vektorok 120◦ -os sz¨ oget z´ arnak be egym´assal, az ered˝oj¨ uk is 6Rπδ nagyol A fel´e mutat´ o vektor. Ha ehhez hozz´ aadjuk a C1 -t˝ol 6R t´avols´ agra s´ag´ u, C1 -b˝ ol elvezetett ´aram l´ev˝ o B1 pontb´

j1 =

v02 v2 sin 2α = 0 sin 2β, g g

567

2 sin α = sin β.

Mivel (2) szerint sin β = cos α, (3)-b´ ol k¨ ovetkezik, hogy 2 sin α = cos α, teh´at tg α = 12 , vagyis 1 α = arctg ≈ 27◦ ´es β ≈ 63◦ . 2 568

K¨ oz´ episkolai Matematikai ´ es Fizikai Lapok, 2022/9

A p´ aly´ ak cs´ ucsmagass´ aga (a ferde haj´ıt´as ¨ osszef¨ ugg´esei szerint) hα = teh´ at

v02 sin2 α, g

illetve

hβ =

m´ odon k¨ ozel´ıtj¨ uk. Ha a hatv´ anysornak csak az els˝ o k´et tagj´ at vessz¨ uk figyelembe, a megtagn´ al ´ allunk meg, u ´gy x2 -re m´ asodfok´ u old´ as x = 1,00 lesz. Ha a sorfejt´esben az x5 -es

√ egyenletet kapunk, amelynek megold´ asb´ ol x = 10 − 80 = 1,027 ad´ odik. Ez 3 tizedesjegyre megegyezik a pontos” eredm´ennyel. ”

v02 sin2 β, g

A k¨orp´ alya sugara: 12 m = 5,84 m, α a k´erdezett centripet´ alis gyorsul´as pedig

hα sin2 α 1 sin2 α = = = tg2 α = . 2 2 ◦ hβ 4 sin β sin (90 − α)

r=

Elekes Dorottya (Budapest-Fasori Evang´elikus Gimn., 10. ´evf.)

2

a=

100 dolgozat ´erkezett. Helyes 66 megold´ as. Kicsit hi´ anyos (3 pont) 5, hi´ anyos (1–2 pont) 23, hib´ as 2, nem versenyszer˝ u 4 dolgozat.

P. 5419. V´ızszintes s´ıkon egyenletesen, 6 m/s nagys´ ag´ u sebess´eggel mozg´ o pontszer˝ u test gyorsul´ as´ anak nagys´ aga ´ alland´ o. A test p´ aly´ aj´ anak A ´es B pontja k¨ oz´e es˝ ou ´tja 1,2-szerese az elmozdul´ asvektor nagys´ ag´ anak. Ezt az utat a test 2 m´ asodperc alatt teszi meg. Mekkora a gyorsul´ asa? (4 pont)

(T¨ obb dolgozat alapj´ an) 80 dolgozat ´erkezett. Helyes 46 megold´ as. Kicsit hi´ anyos (3 pont) 5, hi´ anyos (1–2 pont) 14, hib´ as 11, nem versenyszer˝ u 4 dolgozat.

K¨ozli: Holics L´ aszl´ o, Budapest

Megold´as. Egyenletes sebess´eggel, a´lland´ o nagys´ ag´ u gyorsul´ assal mozg´o test vagy egyenesen, vagy k¨ orp´ aly´ an halad. Jelen esetben (mivel az elmozdul´ as nem egyezik meg a megtett u ´ttal) a mozg´as biztosan egyenletes k¨ormozg´ as. ˜ = 6 m · 2 s = 12 m, ami az AB szakasz Az A ´es B pont k¨oz¨ otti k¨ or´ıv hossza AB s hossz´ anak 1,2-szerese, a test elmozdul´asa teh´at 10 m. Jel¨ olj¨ uk a k¨ orp´ alya sugar´at r-rel, a k¨ or´ıv (radi´anban m´ert) k¨ oz´epponti sz¨ og´et pedig α-val. Az ´ abr´ ar´ ol leolvashatjuk, hogy az elmozdul´as nagys´ aga α AB = 2r sin , 2

Fizik´ab´ ol kit˝ uz¨ott feladatok

M. 418. M´erj¨ uk meg egy labda (pl. focilabda, pingponglabda vagy teniszlabda) tehetetlens´egi nyomat´ek´ at lejt˝on t¨ ort´en˝ o legur´ıt´ assal! Adjuk meg az eredm´enyt omege). Lehet-e k¨ ovetkeztetni a m´er´es mR2 egys´egben is (R a labda sugara, m a t¨ eredm´eny´eb˝ ol a labda falvastags´ ag´ ara? (6 pont)

(3 pont) G. 798. A sz´ azm´eteres s´ıkfut´ as versenyz˝ oi t´erdel˝ orajtb´ ol indulnak. Az ´ abra azt mutatja, hogy mekkora v´ızszintes er˝o hat a rajtg´epbe ´ep´ıtett els˝ o ´es h´ ats´ o ´erz´ekel˝ ore egy 70 kg t¨ omeg˝ u atl´eta indul´ asakor. Becs¨ ulj¨ uk meg, hogy mekkora sebess´eggel hagyja el a sportol´ o a rajtg´epet!

ami az x = α2 jel¨ ol´essel ´ıgy ´ırhat´ o: sin x =

K¨ ozli: N´emeth L´ aszl´ o, Fony´ od

G. 797. Felf´ ujt l´egg¨ omb¨ ot nyitott manom´eterre h´ uzunk. A manom´eter k´et sz´ ar´aban a petr´oleum szintj´enek k¨ ul¨ onbs´ege 72 cm. H´any mm lenne a szintk¨ ul¨ onbs´eg, ha a manom´eterben higany lenne? Mekkora a l´egg¨ ombben uralkod´ o t´ ulnyom´as?

˜ = αr. Ezek szerint fenna k¨ or´ıv hossza pedig AB all, hogy ´ α α 6 sin = , 5 2 2

(1)

v2 (6 m/s) m = = 6,2 2 . r 5,85 m s

5 x. 6

Ezt a trigonometrikus egyenletet megoldhatjuk numerikusan, internetes megold´ oprogrammal: x = 1,0267, teh´at α = 2,053 (rad) ≈ 118◦ .

(3 pont)

Megjegyz´es. Az (1) egyenlet k¨ ozel´ıt˝ o megold´ as´ at u ´gy is megkaphatjuk, hogy a szinuszf¨ uggv´enyt x5 x3 + − ... sin x ≈ x − 3! 5! K¨ oz´ episkolai Matematikai ´ es Fizikai Lapok, 2022/9

569

570

K¨ oz´ episkolai Matematikai ´ es Fizikai Lapok, 2022/9

G. 799. Legal´abb mekkora sebess´eggel ´es legfeljebb mekkora sz¨og alatt kell ind´ıtani egy testet, hogy ´atrep¨ ulj¨ on egy 100 m´eter hossz´ u, 5 m´eter magas, egyenes alag´ uton? A l´egellen´ all´ as elhanyagolhat´o. (3 pont)

K¨ozli: Ringler Andr´ as, Szeged

as´ u, vaG. 800. Egy gy˝ ujt˝ olencse egy bizonyos helyen l´ev˝ o t´ argyr´ ol N1 nagy´ıt´ l´odi k´epet hoz l´etre. Ha a t´argyat az optikai tengely ment´en d t´avols´ aggal messzebb vissz¨ uk a lencs´et˝ ol, a nagy´ıt´ as N2 lesz. Mekkora a lencse f´okuszt´ avols´ aga? (4 pont)

P. 5448. Edu´ ard szerint oldalsz´elben (a halad´ asi ir´ anyra pontosan mer˝oleges sz´elben) k¨or¨ ulbel¨ ul ugyanolyan neh´ez biciklizni, mint sz´elcsendben. Az oldalsz´el ugyanis csak oldalra akarja nyomni a ker´ekp´ art, ami nem lass´ıt, legfeljebb egy kicsit bed˝ olve kell egyenesen is haladni, az ´erz´ekelt szembesz´el pedig mindk´et esetben egyforma. Vajon igaza van-e Edu´ardnak? (4 pont)

u r´ezrudat j´ o h˝ oszigetel˝ o P. 5449. Egy 20 cm hossz´ u, 3 cm2 keresztmetszet˝ k¨openy vesz k¨ or¨ ul. A rudat f¨ ugg˝olegesen tartjuk, ´es az egyik v´eg´et olvad´ o jeget om´ers´ekleten tartjuk. tartalmaz´ o poh´ arba l´ ogatjuk; ´ıgy azt folyamatosan 0 ◦ C h˝ H´any fokra melegszik fel a r´ ud m´ asik v´ege, ha azt egy kicsiny, 100 W teljes´ıtm´eny˝ u f˝ ut˝ osz´ alas tekerccsel meleg´ıtj¨ uk? (A sz¨ uks´eges anyagi a´lland´ okat t´ abl´ azatokban, vagy az interneten megtal´alhatjuk.) (3 pont)

P. 5445. Egy M t¨ omeg˝ u robban´ o l¨ oved´eket α sz¨og alatt v0 kezd˝osebess´eggel l˝ ottek fel. P´aly´ aj´anak tetej´en a l¨ oved´ek m1 ´es m2 t¨omeg˝ u r´eszre robbant, ´es omeg˝ u r´esz ebben a pillanatban szabades´esbe kezdett. az m2 t¨ a) Milyen messze lesz egym´ ast´ ol a l¨ oved´ek k´et darabja, amikor mindkett˝ o talajt ´ert? b) Mekkora ´es milyen ir´any´ u sebess´eggel csap´ odnak a talajba? (Adatok: M = 0,6 kg, m1 = 0,2 kg, α = 60◦ , v0 = 100 m/s. A l´egellen´ all´ast hanyagoljuk el.) (4 pont)

K¨ozli: Holics L´ aszl´ o, Budapest

P. 5446. K´et di´ ak egy mutatv´ anyra k´esz¨ ul. A sportp´ aly´ an egym´ast´ ol bizonyos d t´ avols´ agra l´ev˝ o focilabd´ akat egyszerre megr´ ugj´ ak u ´gy, hogy a labd´ak a leveg˝ oben asik v2 = 10 m/s sebess´eggel l¨ovi el tal´ alkozzanak. Az egyik di´ ak v1 = 20 m/s, a m´ a labd´ at, de a kezd˝ osebess´eg ir´any´ at szabadon megv´alaszthatj´ak. Legfeljebb mekavols´ agra lehet egym´ ast´ ol a k´et labda ahhoz, hogy a mutatv´ any kora kezdeti dmax t´ siker¨ ulj¨ on? (A l´egellen´ all´as hat´ as´ at hanyagoljuk el.) (5 pont)

K¨ozli: Vigh M´ at´e, Biatorb´ agy

P. 5447. H´ arom egyforma, 5 cm sugar´ u j´eghengert k´esz´ıt¨ unk, ´es azokat az ´ abr´ an l´ athat´ o helyzetb˝ ol kezd˝ osebess´eg n´elk¨ ul elengedj¨ uk. A s´ url´od´ as mindenhol elhanyagolhat´ o. Mekkora gyorsul´ assal indulnak el a j´eghengerek? (5 pont) K¨ oz´ episkolai Matematikai ´ es Fizikai Lapok, 2022/9

K¨ ozli: Bodor Andr´ as, Budapest

K¨ ozli: Gn¨ adig P´eter, V´ acduka

P. 5450. Az f = 5 cm f´ okuszt´ avols´ ag´ u gy˝ ujt˝ olencse optikai tengely´en, a lencs´et˝ol jobbra 30 cm-re ´es balra 18 cm-re tal´alhat´ o, pontszer˝ unek tekinthet˝ o szentj´ anosbogarak elkezdenek egym´ as fel´e mozogni 2 cm/s sebess´eggel. Mennyi id˝ o m´ ulva ker¨ ul fed´esbe egym´ assal a k´et bog´ ar k´epe? (4 pont)

K¨ ozli: Sz´echenyi G´ abor, Budapest

P. 5451. Egy gar´ azs v´ alt´ o´ aram´ u ell´at´ as´ at h´ aromf´ azis´ u bet´ apl´ al´assal oldj´ ak abelt haszn´alnak, mind az ¨ ot ´er azonos anyag´ u ´es keresztmeg. Ehhez ¨oteres k´ metszet˝ u. Az egyik vezet´ek (z¨ old-s´ arga) az ´erint´esv´edelmi f¨ old, amin hibamentes haszn´ alat eset´en nem folyik a´ram. A (k´ek) nullvezet˝ o potenci´alja mindig nulla, gyakorlatilag mindig f¨ oldpotenci´ alon van. A h´ arom f´azisvezet˝ oben (barna, fekete ´es sz¨ urke) olyan m´odon v´ altozik a szinuszos potenci´ al, hogy az effekt´ıv ´ert´ek 230 V, azisk¨ ul¨ onbs´eg. Az egyes ´es b´ armely k´et f´azisvezet˝o potenci´ alja k¨ oz¨ ott 120◦ -os a f´ f´azisvezet˝ok ´es a nullvezet˝ o k¨ oz´e ohmos fogyaszt´ okat kapcsolunk. a) Mekkora effekt´ıv fesz¨ ults´eget m´erhet¨ unk k´et k¨ ul¨ onb¨ oz˝ o f´azisvezet˝ o k¨ oz¨ ott? b) Mekkora a nullvezet˝ o effekt´ıv ´ arama, ha k´et f´azisvezet˝ oben 10–10 A a´ram folyik, de a harmadik f´azisvezet˝ on nem folyik ´aram? c) Mekkora a nullvezet˝ o effekt´ıv ´arama, ha mind a h´ arom f´azisvezet˝ okben 10 A ´aram folyik? d) Mekkora a nullvezet˝ o legkisebb ´es legnagyobb effekt´ıv ´ arama, ha egyik f´azisvezet˝o effekt´ıv ´arama se haladja meg a 10 A-es ´ert´eket? (4 pont)

K¨ ozli: Honyek Gyula, Veresegyh´ az

P. 5452. Egy egyenes p´aly´ an halad´ o fotonrak´eta t¨ omege indul´ askor m0 . Adjuk meg a rak´eta sebess´eg´et a nyugalmi t¨ omeg pillanatnyi ´ert´ek´enek a f¨ uggv´eny´eben! (L´asd m´eg a P. 5426. feladatot a K¨ oMaL 2022. szeptemberi sz´ am´aban.)

K¨ozli: Cserti J´ ozsef, Budapest 571

(4 pont) 572

K¨ ozli: Woynarovich Ferenc, Budapest K¨ oz´ episkolai Matematikai ´ es Fizikai Lapok, 2022/9

P. 5453. Az r = 0,2 m sugar´ u, t¨ oltetlen f´emg¨omb k¨oz´eppontj´at´ ol d = 0,5 m t´ avols´ agra egy Q nagys´ ag´ u pontt¨ olt´es helyezkedik el. Hat´ arozzuk meg (ak´ ar numerikus sz´am´ıt´ assal), hogy a pontt¨ olt´est˝ ol n´ezve mekkora sz¨og alatt l´ atszanak a f´emg¨ omb azon pontjai, ahol a fel¨ uleti t¨ olt´ess˝ ur˝ us´eg z´erus! (6 pont)

K¨ozli: Sz´ asz Kriszti´ an, Budapest Beku ¨ld´esi hat´arid˝o: 2023. janu´ar 15. Elektronikus munkafu ¨zet: https://www.komal.hu/munkafuzet



MATHEMATICAL AND PHYSICAL JOURNAL FOR SECONDARY SCHOOLS (Volume 72. No. 9. December 2022)

Problems in Mathematics New exercises for practice – competition K (see page 542): K. 744. I have 1000 forints (HUF, Hungarian currency) in my pocket. If I buy two sandwiches and one soft drink then I will have as many forints remaining as the amount I should add to my 1000 in order to be able to buy three sandwiches and one soft drink. Two sandwiches and two soft drinks cost 1100 forints. What is the price of one sandwich, and what is the price of one soft drink? K. 745. In a game, players are collecting points. The players take turns in playing and scoring points. When a certain player is playing, he or she may get any nonnegative integer of points (including 0). The points scored by a player in successive turns add up. The game terminates when the total of the points scored by the players reaches 1000 (that is, the last player may only score as many points as needed to make the total equal to 1000). The player with the largest number of points will win the game. In the case of equality, the player reaching the same score earlier will win. The player with the second largest number of points will finish in the second place, and so on. At the moment, the scores of the players are as follows: Kate has 314 points, Sam has 207 points, John has 58 points, Gillian has 31 and Joe has 0. a) If it is Kate’s turn now, what is the minimum number of points she needs to gain in this turn in order to be certain that she will finish in the first or second place? b) If it is Sam’s turn now, what is the minimum number of points he needs to gain in this turn in order to be certain that he will finish in the first or second place? c) If it is Joe’s turn now, what is the minimum number of points he needs to gain in this turn in order to be certain that he will finish in the first or second place? K. 746. In a certain small country, citizens are charged for gas consumption as follows: in the first year of the new regulations, the first 1700 m3 of gas costs 100 pennies/m3 , and any further consumption costs 750 pennies/m3 . From the following year onwards, the quantity sold for the reduced price is determined each year from the nationwide average consumption of the previous year. John Average lives in this country, and he used gas with these conditions for a year. Then he realized that he was paying too much, and decided to economise with the gas used in his household. He succeeded in reducing his consumption by 10% in the following year. However, since everyone was trying to save money, the mean consumption decreased by 15%. Although John Average used less gas K¨ oz´ episkolai Matematikai ´ es Fizikai Lapok, 2022/9

573

than previously, he still had to pay more (for the whole year) than in the previous year. What may have been John’s consumption during the first year? K/C. 747. A forty-sided polygon is divided into two polygons by one of its diagonals. The two polygons altogether have 298 fewer diagonals than the forty-gon. How many sides do these polygons have? K/C. 748. The integers 1 to 100 are written along the circumference of a circle. First every even number is connected to each odd number that is smaller than the even number. Then every odd number is connected to each even number that is smaller than the odd number. How many connecting lines are drawn? New exercises for practice – competition C (see page 544): Exercises up to grade 10: K/C. 747. See the text at Exercises K. K/C. 748. See the text at Exercises K. Exercises for everyone: C. 1743. Seven natural numbers form an arithmetic sequence with a common difference of 30. Show that exactly one of the numbers is divisible by 7. (Proposed by B. B´ır´ o, Eger) C. 1744. In a triangle ABC, ∠CAB = 45◦ and ∠ABC = 60◦ . D is a point of the line segment AB. The circumscribed circle of triangle CAD passes through the orthocentre of triangle ABC. Without the use of trigonometry, find the exact value of the y−5 AD o, Eger) C. 1745. Solve the equation x2 + 8x − y = y+6 , ratio BD . (Proposed by B. B´ır´ where x, y are integers. (Proposed by B. B´ır´ o, Eger) Exercises upwards of grade 11: C. 1746. Side AB of a square ABCD is extended beyond point A by the line segment AE = 2, and beyond point B by the line segment BF = 3. Lines ED and F C enclose an angle of 45◦ . Determine the possible values of the length of the sides of the square. (Proposed by L. N´emeth, Fony´ od) C. 1747. Let n  3 be a positive integer, and let k be the sum of the digits in the number 10n − 4!. What is the sum of the digits in the number (Proposed by M. Szalai, Szeged)

10n+1 −7 ? 3

New exercises – competition B (see page 545): B. 5278. In Noname School of Nowhere, the graduating class consists of four groups, each specializing in a different subject: math, physics, chem and bio. On day, they are all sitting around a large round table in the school cafeteria: everyone has someone sitting directly opposite them, and everyone has neighbours sitting right next to them on either side. However a student is selected, it is observed that this student, the two neighbours, and the one sitting straight across the table all belong to different groups. How many students may there be in the graduating class if there are less than 20 of them? (3 points) (Proposed by K. A. Kozma, Gy˝ or) B. 5279. Two circles are drawn inside a right angle. One circle touches one of the arms at point A, and the other circle touches the other arm at point B. The circles also touch each other at point C. What is the measure of ∠ACB? (3 points) (Proposed by B. B´ır´ o, Eger) B. 5280. Let a > 2, b and c be real numbers. Consider the three statements below. (1) The equation ax2 + bx + c = 0 has no real solution. (2) The equation (a − 1)x2 + (b − 1)x + (c − 1) = 0 has 1 real solution. (3) The equation (a − 2)x2 + (b − 2)x + (c − 2) = 0 has 2 real solutions. a) Given that statements (1) and (2) are true, can we conclude that statement (3) is also true? b) Given that statements (2) and (3) are true, can we conclude that statement (1) is also true? (4 points) (Proposed by B. Hujter, Budapest) B. 5281. Let d > 1 denote a positive integer. Prove that there exists a positive integer that has the same number of factors divisible by d as factors not divisible by d. (5 points) (Proposed by B. Hujter, Budapest) B. 5282. In an acute-angled triangle ABC, the foot of the altitude drawn from vertex A is T . The projection of T on side AB is D, and its projection on side AC is E. Let F be the intersection of side BC and the circle ABE that is different from B. Analogously, let G be the intersection of side BC and circle ACD, different from C. Show that T F = T G. (5 points) (Proposed by G. K´ os, Budapest) B. 5283. A convex quadrilateral N contains a circular disc of radius r. Show that the perimeter of N is at

574

K¨ oz´ episkolai Matematikai ´ es Fizikai Lapok, 2022/9

least 8r. (4 points) (Proposed by V. V´ıgh, S´ andorfalva) B. 5284. Let n > 2. Alan has selected an edge of the complete graph of 2n vertices. Paula wants to find out which. If she pays 1 forint (Hungarian currency, HUF) she may name any pairing of all vertices and ask whether the selected edge is contained in it. What is the minimum number of forints that Paula needs to have in her pockets in order to be certain that she can find out the selected edge by asking the appropriate questions? (6 points) (Proposed by P. P. Pach, Budapest) B. 5285. In an acute-angled triangle ABC, AB = AC. Points A , B  and C  are moving along the circumscribed circle of the triangle, so that triangle A B  C  always remain congruent to triangle ABC, and have the same orientation. Let P be the intersection of lines BB  and CC  . Show that the lines A P all pass through a certain point. (6 points) (Proposed by G. K´ os, Budapest) New problems – competition A (see page 546): A. 839. We are given a finite, simple, non-directed graph. Ann writes positive real numbers on each edge of the graph such that for all vertices the following is true: the sum of the numbers written on the edges incident to a given vertex is less than one. Bob wants to write non-negative real numbers on the vertices in the following way: if the number written at vertex v is v0 , and Ann’s numbers on the edges incident to v are e1 , e2 , . . . , ek , and the numbers on the other endpoints of these k  edges are v1 , v2 , . . . , vk , then v0 = ei vi + 2022. Prove that Bob can always number the i=1

vertices in this way regardless of the graph and the numbers chosen by Ann. (Proposed by Boldizs´ ar Varga, Ver˝ oce) A. 840. The incircle of triangle ABC touches the sides in X, Y and Z. In triangle XY Z the feet of the altitude from X and Y are X  and Y  , respectively. Let line X  Y  intersect the circumcircle of triangle ABC at P and Q. Prove that points X, Y , P and Q are concyclic. (Proposed by L´ aszl´ o Simon, Budapest) A. 841. Find all non-negative integer solutions of the equation 2a + pb = np−1 , where p is a prime number. (Proposed by M´ at´e Weisz, Cambridge)

Problems in Physics (see page 570) M. 418. Measure the rotational inertia of a ball (e.g. football, table tennis ball or tennis ball) by rolling it down a slope. Give the result also in units of mR2 (where R is the radius of the ball, m is its mass). Can the result be used to infer the thickness of the wall of the ball? G. 797. An inflated balloon is attached to the open end of a liquid-column manometer. The difference in the level of petroleum in the two arms of the manometer is 72 cm. How many mm would the difference in level be if the manometer contained mercury? What is the gauge pressure (excess pressure) in the balloon? G. 798. In a 100 m flat race, the competitors will start from a kneeling start. The figure shows the horizontal force applied to the front and rear sensors in the starting machine when an athlete weighing 70 kg starts. Estimate the speed at which the athlete leaves the starting machine. G. 799. What is the minimum speed and maximum angle at which a body must be launched in order that it flies through a 100 metre long and 5 metre high straight tunnel? The air drag is negligible. G. 800. An object is located at a certain distance from

K¨ oz´ episkolai Matematikai ´ es Fizikai Lapok, 2022/9

575

a converging lens, which forms the real, inverted image of the object of magnification N1 . When the object is placed further from the lens, with a distance of d along the principal axis of the lens, the magnification becomes N2 . What is the focal length of the lens? P. 5445. An explosive projectile of mass M was fired at an angle of α and at an initial speed of v0 . At the top of its trajectory, the projectile exploded into two parts of masses m1 and m2 , and the part with mass m2 started to fall freely at this moment. a) How far apart will the two parts hit the ground? b) What are the velocities of the two parts at their impact with the ground? (Determine both the speed and the direction of he velocity.) (Data: M = 0.6 kg, m1 = 0.2 kg, α = 60◦ , v0 = 100 m/s. Air drag is negligible.) P. 5446. Two students prepare for a stunt. At the same moment they both kick a football, that are at a certain distance d apart on a sports field, so that the balls meet in the air. One student kicks the ball at v1 = 20 m/s and the other at v2 = 10 m/s, but they can both decide on the direction of the initial velocity. What is the maximum initial distance dmax between the two balls for the stunt to succeed? (Air drag is negligible.) P. 5447. Three identical cylinders of radius 5 cm are made of ice and they are released without initial speed from the position shown in the figure. Friction is negligible everywhere. What is the acceleration at which the ice cylinders start moving? P. 5448. According to Edward, riding the bicycle in crosswind (wind which blows perpendicularly to the direction of the motion) is about as hard as to ride the bicycle when there is no wind. The crosswind pushes the bike only sideways, which doesn’t slow you down. You just have to lean a little to go straight, and the headwind you’re feeling is the same in both cases. Is Edward right? P. 5449. A 20 cm long copper rod with a cross section of 3 cm2 is surrounded by a good insulating sheath. The rod is held vertically and suspended at one end such that the other is in a glass containing melting ice; thus it is kept at a constant temperature of 0 ◦ C. To what temperature does the other end of the rod warms up when heated with a small 100 W filament coil? (The required constants can be found in tables or on the internet.) P. 5450. On the principal axis of a converging lens of focal length f = 5 cm, there are (point-like) fireflies, which begin to move towards each other at a speed of 2 cm/s. Initially one is 30 cm to the right and the other is 18 cm to the left of the lens. How much time elapses until their images overlap? P. 5451. The AC power supply for a garage is provided by a three-phase supply. A five-strand cable is used, with all five strands made of the same material and cross-section. One of the wires (green-yellow) is the protective conductor (ground), on which no current flows in the event of fault-free operation. The neutral conductor (blue) always has a potential of zero, and is practically always at ground potential. In the three phase conductors (brown, black, and grey) the potential is a sinusoidal function of time and varies such that the root-mean-square (rms) value is 230 V. The phase difference between the potentials of any two phase conductors is 120◦ . Resistors are connected between each phase conductor and the neutral conductor. a) What is the rms value of the voltage between two different phase cables? b) What is the rms value of the current in the neutral wire if the current in two phase conductors is 10 A, in each, but there is no current in the third phase conductor? c) What is the rms current in the neutral wire if the current in each phase wire is 10 A? d) What is the minimum and the maximum rms current in the neutral conductor if none of the rms currents in the phase conductors exceed 10 A? P. 5452. The initial mass of a photon rocket when it starts moving along a straight path is m0 . Determine the speed of the rocket as a function of the instantaneous rest mass of the rocket. P. 5453. There is a point-like charge Q at a distance of d = 0.5 m from the centre of an uncharged metal sphere of radius r = 0.2 m. Determine the angle (with respect to the point charge) that is subtended by the points of the metal sphere with surface charge density equal to zero. (Numerical methods are also acceptable.)

72. ´evfolyam 9. sz´am

K¨oMaL

Budapest, 2022. december